0% found this document useful (0 votes)
254 views481 pages

Great Book One

Great book

Uploaded by

xonavew122
Copyright
© © All Rights Reserved
We take content rights seriously. If you suspect this is your content, claim it here.
Available Formats
Download as PDF, TXT or read online on Scribd
0% found this document useful (0 votes)
254 views481 pages

Great Book One

Great book

Uploaded by

xonavew122
Copyright
© © All Rights Reserved
We take content rights seriously. If you suspect this is your content, claim it here.
Available Formats
Download as PDF, TXT or read online on Scribd
You are on page 1/ 481

MRCP 1 Question Concepts

(Demystifying Exam Questions)

Mohammed Yasein Elamin


MBBS, DTM&H, MCTM, MRCPI 1&2

PassM & OnExam


OCT 2023
PassM 1, OnExam 1 & OnExam 2 2023
TABLE OF CONTENTS

1. Cardiology .................................................................................................................................................. 2
2. Clinical haematology/oncology ................................................................................................................ 48
3. oncology ................................................................................................................................................... 76
4. Clinical pharmacology and toxicology ..................................................................................................... 88
5. Clinical sciences ..................................................................................................................................... 126
6. Dermatology ........................................................................................................................................... 157
7. Endocrinology ........................................................................................................................................ 171
8. Gastroenterology .................................................................................................................................... 217
9. Geriatric medicine .................................................................................................................................. 253
10. Infectious diseases and STIs ............................................................................................................... 257
11. Nephrology ......................................................................................................................................... 298
12. Neurology ........................................................................................................................................... 322
13. Palliative medicine and end of life care.............................................................................................. 371
14. Ophthalmology ................................................................................................................................... 378
15. Psychiatry ........................................................................................................................................... 384
16. Respiratory medicine .......................................................................................................................... 400
17. Rheumatology ..................................................................................................................................... 432
18. Emergency .......................................................................................................................................... 457
19. Radiology............................................................................................................................................ 468
20. Photography ........................................................................................................................................ 471
21. Critical ................................................................................................................................................ 478

1
M Y Elamin
MBBS, DTM&H, MCTM, MRCPI 1& 2
CARDIOLOGY
1. 2-level PE Wells score > 4 points - immediate CTPA or interim anticoagulation whilst awaiting CTPA
2. 2-level PE Wells score ≤ 4 points - D-dimer with the result within 4 hours or interim anticoagulation whilst
awaiting D-dimer
3. 3 'stacked' defibrillation shocks - should be considered for VF/pulseless VT for a witnessed, monitored cardiac
arrest in an HDU setting (adult patient)
4. 50-year-old man with chest pain, STEMI changes on ECG, pulse 90/min, BP 140/80mmHg, sats 96% - aspirin,
nitrates, morphine, prasugrel, arrange immediate percutaenous coronary intervention
5. 60-year-old man with chest pain, NSTEMI changes on ECG, pulse 66/min, BP 130/82mmHg, sats 98% - aspirin,
nitrates, morphine, clopidogrel, calculate GRACE score
6. 70-year-old man with chest pain, STEMI changes on ECG, pulse 102/min, BP 160/90mmHg, sats 93% - aspirin,
nitrates, morphine, oxygen, prasugrel, arrange immediate percutaenous coronary intervention
7. A 10-year-old presents with fever. They complain of fleeting large joint pain and 'jerking' movements of the hand
and face. On examination a murmur is noted and subcutaneous nodules on the wrists - Streptococcus pyogenes
8. A 25-year-old man is investigated for recurrent syncope and dyspnoea. On examination he has an ejection systolic
murmur - hypertrophic obstructive cardiomyopathy
9. A 2-year-old who has had a fever for the past 7 days is noted to have conjunctivitis, erythema and oedema of the
hands and feet, cracked lips and a strawberry tongue - Kawasaki disease
10. A 30-year-old develops dyspnoea, chest pain and fever one week after an upper respiratory tract infection. On
examination he has a pericardial friction rub, gallop rhythm, and cardiomegaly - coxsackie B virus
11. A 30-year-old man presents with recurrent palpitations and syncope. A resting ECG shows T wave inversion in
V1-3 and epsilon waves. He has a family history of sudden death - arrhythmogenic right ventricular
cardiomyopathy
12. A 30-year-old woman from Singapore presents with headache, malaise and myalgia. On examination the blood
pressure is 160/100 mmHg, the right radial pulse is difficult to palpate and a carotid bruit is noted - Takayasu's
arteritis
13. A 40-year-old female presents with dyspnoea and fatigue. On examination a mid-diastolic murmur is heard. An
echocardiogram shows a pedunculated mass in the left atrium - atrial myxoma
14. A 50-year-old cycling enthusiast is found to have a PR interval of 0.24 seconds on a routine ECG. The ECG is
otherwise normal - first degree heart block
15. A 50-year-old man with Marfan syndrome presents with palpitations and dyspnoea. On examination he has a
collapsing pulse, the blood pressure is 160/60 mmHg and a high-pitched diastolic murmur is heard - aortic
regurgitation
16. A 50-year-old man with no history of arrhythmia develops atrial fibrillation whilst being treated for pneumonia in
hospital - first time atrial fibrillation
2
M Y Elamin
MBBS, DTM&H, MCTM, MRCPI 1& 2
17. A 50-year-old woman with a history of rheumatic fever presents with dyspnoea. On examination she is found to be
in atrial fibrillation, with a loud S1, split S2 and a diastolic murmur - mitral stenosis
18. A 60-year-old man has recurrent episodes of atrial fibrillation which don't self terminate. He uses flecanide to
cardiovert himself into sinus rhythm - persistent atrial fibrillation
19. A 60-year-old man is found to have recurrent episodes of atrial fibrillation than terminate sponatenously after 1-
2 days - paroxysmal atrial fibrillation
20. A 60-year-old man with a history of heavy smoking presents with intermittent claudication and ulceration of his toes
- Buerger's disease
21. A 60-year-old man with a history of lung cancer presents with dyspnoea. On examination he is tachycardic,
hypotensive, has a raised JVP with an absent Y descent and has pulsus paradoxus - cardiac tamponade
22. A 60-year-old man with a history of tuberculosis presents with dyspnoea and fatigue. On examination the JVP is
elevated, there is a loud S3 and Kussmaul's sign is positive. Hepatomegaly is also noted - constrictive pericarditis
23. A 6-week-old baby is investigated for cyanosis and feeding difficulties. An echocardiogram shows a right-to-left
shunt through a VSD, right ventricular outflow tract obstruction with associated right ventricular hypertrophy
and an overriding aorta - tetralogy of Fallot
24. A 70-year-old man is treated with a beta-blocker to control his atrial fibrillation. A previous attempt to cardiovert
him failed - permanent atrial fibrillation
25. A 70-year-old man presents with chest pain and dyspnoea. On examination he has an ejection systolic murmur
which radiates to his carotids - aortic stenosis
26. A 70-year-old woman is found to have a pan-systolic murmur after presenting with dyspnoea. A soft S1 and split
S2 is also noted - mitral regurgitation
27. A large A wave in the JVP may be caused by pulmonary hypertension
28. A large A wave in the JVP may be caused by tricuspid stenosis
29. A long QT interval - hypokalaemia
30. A long QT interval - myocardial ischaemia
31. A long QT interval - subarachnoid haemorrhage
32. A man develops a cardiac arrest shortly after being admitted with a myocardial infarction. The ECG monitor
shows rapid, irregular waveforms - ventricular fibrillation
33. A man presents with central, pleuritic chest pain and fever 4 weeks following a myocardial infarction. The ESR
is elevated - Dressler's syndrome
34. A man presents with pleuritic central chest pain and dyspnoea following a viral illness. His pain is worse when
lying down - acute pericarditis
35. A non-pulsatile raised JVP may be caused by SVC obstruction
36. A paradoxical rise in the JVP during inspiration may be caused by constrictive pericarditis
37. A patient develops a bradycardia of 36/min following a myocardial infarction. The ECG shows no association
between the P waves and QRS complexes - complete heart block
3
M Y Elamin
MBBS, DTM&H, MCTM, MRCPI 1& 2
38. A patient develops acute heart failure 10 days following a myocardial infarction. On examination he has a raised
JVP, pulsus paradoxus and diminished heart sounds - left ventricular free wall rupture
39. A patient develops acute heart failure 5 days after a myocardial infarction. A new pan-systolic murmur is noted
on examination - post-myocardial infarction ventricular septal defect
40. A patient is noted to have a new early-to-mid systolic murmur 10 days after being admitted for a myocardial
infarction - ischaemia of the papillary muscle
41. A patient is noted to have persistent ST elevation 4 weeks after sustaining a myocardial infarction. Examination
reveals bibasal crackles and the presence of a third and fourth heart sound - left ventricular aneurysm
42. A patient with an uncorrected ventricular septal defect presents with cyanosis and signs of right ventricular
failure. Further investigations demonstrate a right to left shunt and pulmonary hypertension - Eisenmenger's
syndrome
43. A prolonged PR interval - aortic root abscess
44. A prolonged PR interval - digoxin toxicity
45. A prolonged PR interval - hypokalaemia
46. A prolonged PR interval - Lyme disease
47. A prolonged PR interval - myocardial ischaemia
48. A prolonged PR interval - myotonic dystrophy
49. A prolonged PR interval - rheumatic fever
50. A prolonged PR interval - sarcoidosis
51. A short PR interval - Wolff-Parkinson-White syndrome (left-sided accessory pathway)
52. A short QT interval - digoxin toxicity
53. A short QT interval - hypercalcaemia
54. A single defibrillation shock - should be given for VF/pulseless VT every 2 minutes (adult patient)
55. ABCD2 - risk stratification of patients who've had a suspected TIA
56. ACE inhibitors may cause angioedema
57. Acute coronary syndrome (not treated with angioplasy) : DVLA group 1 rules - cannot drive for 4 weeks
58. Acute coronary syndrome (treated with angioplasy) : DVLA group 1 rules - cannot drive for 1 week
59. Acute coronary syndrome, medically managed (no drug allergies) - antiplatelets: aspirin (lifelong) & ticagrelor
(12 months), no other antiplatelets
60. HTN management: patients >= 55-years-old or of black African or African–Caribbean origin: Add a calcium
channel blocker
61. HTN management: patients < 55-years-old or a background of type 2 diabetes mellitus: Add an ACE inhibitor or an
angiotensin receptor blocker
62. HTN management: if already taking an ACE-i or ARB: Add a calcium channel blocker or a thiazide diuretic
63. HTN management: if already taking a Calcium channel blocker: add an ACE-i or ARB or a thiazide-like Diuretic

4
M Y Elamin
MBBS, DTM&H, MCTM, MRCPI 1& 2
64. HTN management: step 4 as resistant hypertension and suggest either adding a 4th drug: if potassium > 4.5
mmol/l; Add an alpha- or beta-blocker
65. HTN management: step 4 as resistant hypertension and suggest either adding a 4th drug: if potassium < 4.5 mmol/l;
Add spironolactone
66. HTN management: If blood pressure remains uncontrolled with the optimal or maximum tolerated doses of four
drugs, seek expert advice if it has not yet been obtained.
67. Adenosine - agonist of the A1 receptor in the AV node
68. Adenosine - contraindicated by: asthma
69. Adenosine , uses include: supraventricular tachycardia
70. Adenosine may cause bronchospasm
71. Adenosine may cause chest pain
72. Adenosine may cause flushing
73. Adenosine may cause heart block
74. Alteplase - activates plasminogen to form plasmin
75. Amiodarone - blocks potassium channels
76. Amiodarone may cause bradycardia
77. Amiodarone may cause corneal deposits
78. Amiodarone may cause hepatotoxicity
79. Amiodarone may cause hyperthyroidism
80. Amiodarone may cause hypothyroidism
81. Amiodarone may cause lengthens QT interval
82. Amiodarone may cause peripheral neuropathy
83. Amiodarone may cause photosensitivity
84. Amiodarone may cause pneumonitis
85. Amiodarone may cause pulmonary fibrosis
86. Amiodarone may cause slate-grey appearance
87. Amiodarone may cause thrombophlebitis
88. An absent A wave in the JVP may be caused by atrial fibrillation
89. An absent Y descent in the JVP may be caused by cardiac tamponade
90. An ECG shows a constant PR interval but the P wave is often not followed by a QRS complex - second degree
heart block (Mobitz II)
91. An ECG shows progressive prolongation of the PR interval until a dropped beat occurs - second degree heart block
(Mobitz I)
92. Anaemia - high-output heart failure
93. Angiotensin II receptor blockers may cause hyperkalaemia
94. Aortic regurgitation - collapsing pulse
5
M Y Elamin
MBBS, DTM&H, MCTM, MRCPI 1& 2
95. Aortic regurgitation - early diastolic murmur, high-pitched and 'blowing' in character
96. Aortic regurgitation - wide pulse pressure
97. Aortic stenosis - ejection systolic murmur
98. Aortic stenosis - narrow pulse pressure
99. Aortic stenosis - S4
100. Aortic stenosis - slow rising pulse
101. Aortic stenosis - soft S2
102. Aortic valve stenosis is acyanotic
103. APGAR - health of a newborn immediately after birth
104. Around 50% of patients have a mutation of one of the several genes which encode components of desmosome -
arrhythmogenic right ventricular dysplasia
105. Arrhythmias - systolic dysfunction
106. Arteriovenous malformation - high-output heart failure
107. Asian patient, ST elevation V1-V3 followed by a negative T wave, partial RBBB, sudden death - Brugada
syndrome
108. Aspirin , uses include: Kawasaki disease
109. Atrial septal defect - ejection systolic murmur
110. Atrial septal defect is acyanotic
111. AUDIT - problematic alcohol use
112. Austin-Flint murmur (severe aortic regurgitation) - mid-late diastolic murmur, 'rumbling' in character
113. Bendroflumethiazide - inhibits sodium reabsorption by blocking the Na+-Cl− symporter at the beginning of the
distal convoluted tubule
114. Beta-blockers - contraindicated by: asthma
115. Beta-blockers - contraindicated by: concurrent verapamil therapy
116. Beta-blockers may cause bronchospasm
117. Beta-blockers may cause cold peripheries
118. Beta-blockers may cause insomnia
119. Beta-blockers may cause lethargy/drowsiness
120. Bisferiens pulse - hypertrophic obstructive cardiomyopathy
121. Bisferiens pulse - mixed aortic valve disease
122. Bishop score - assess the whether induction of labour will be required
123. Bivalirudin - reversible direct thrombin inhibitor
124. Blood pressure target (< 80 years, ABPM/HBPM) - 135/85 mmHg
125. Blood pressure target (< 80 years, clinic reading) - 140/90 mmHg
126. Blood pressure target (> 80 years, ABPM/HBPM) - 145/85 mmHg
127. Blood pressure target (> 80 years, clinic reading) - 150/90 mmHg
6
M Y Elamin
MBBS, DTM&H, MCTM, MRCPI 1& 2
128. Bosentan - endothelin receptor antagonists
129. B-type natriuretic peptide - increases renal water and sodium excretion
130. B-type natriuretic peptide - inhibits renin secretion
131. B-type natriuretic peptide - vasodilation of vascular smooth muscle
132. Buerger's disease - an association with heavy smoking
133. Buerger's disease - intermittent claudication
134. CAGE - problematic alcohol use
135. Cannon A waves in the JVP may be caused by complete heart block
136. Cannon A waves in the JVP may be caused by ventricular tachycardia
137. Cardiac tamponade - diastolic dysfunction
138. CHA2DS2-VASc - anticoagulation strategy for patients with AF
139. CHA2DS2-VASc of 0 - no anticoagulation
140. CHA2DS2-VASc of 1 (female) - no anticoagulation
141. CHA2DS2-VASc of 1 (male) - consider (rather than offer) anticoagulation with warfarin or new oral anticoagulants
(NOACs)
142. CHA2DS2-VASc of 2 or more - offer (rather than consider) anticoagulation with warfarin or new oral
anticoagulants (NOACs)
143. Characterised by transient, apical ballooning of the myocardium - Takotsubo cardiomyopathy
144. Child with fever, conjunctivitis, desquamating rash, cracked lips, strawberry tongue - kawasaki disease
145. Child-Pugh classification - severity of liver cirrhosis
146. Clopidogrel - antagonist of the P2Y12 adenosine diphosphate (ADP) receptor
147. Coarctation of the aorta is acyanotic
148. Collapsing pulse - aortic regurgitation
149. Collapsing pulse - hyperkinetic states
150. Collapsing pulse - patent ductus arteriosus
151. Collapsing pulse, wide pulse pressure, diastolic murmur - aortic regurgitation
152. Colorectal cancer - Streptococcus bovis
153. Constrictive pericarditis - diastolic dysfunction
154. Continuous 'machinery' murmur - patent ductus arteriosus
155. Coronary artery bypass graft (CABG): DVLA group 1 rules - cannot drive for 4 weeks
156. Criteria for considering immediate treatment based on a clinic reading - 180/110 mmHg
157. CTPA negative - consider the possibility of DVT and arrange proximal leg vein ultrasound if suspected
158. CTPA positive - diagnosis PE and continue anticoagulation
159. CURB-65 - prognosis of a patient with pneumonia
160. DAS28 - disease activity in rheumatoid arthritis
161. D-dimer negative - stop anticoagulation and consider an alternative diagnosis
7
M Y Elamin
MBBS, DTM&H, MCTM, MRCPI 1& 2
162. D-dimer positive - immediate CTPA or interim anticoagulation whilst awaiting CTPA
163. Definition of stage 1 hypertension (ABPM/HBPM) - 135/85 mmHg
164. Definition of stage 1 hypertension (Clinic reading) - 140/90 mmHg
165. Definition of stage 2 hypertension (ABPM/HBPM) - 150/95 mmHg
166. Definition of stage 2 hypertension (Clinic reading) - 160/100 mmHg
167. Delta waves - Wolff-Parkinson-White syndrome (left-sided accessory pathway)
168. Dilated cardiomyopathy - 'balloon' appearance of the heart on the chest x-ray
169. Dilated cardiomyopathy - causes include alcohol abuse
170. Dilated cardiomyopathy - causes include Chagas disease
171. Dilated cardiomyopathy - causes include cocaine
172. Dilated cardiomyopathy - causes include Coxsackie B
173. Dilated cardiomyopathy - causes include doxorubicin
174. Dilated cardiomyopathy - causes include Duchenne muscular dystrophy
175. Dilated cardiomyopathy - causes include pregnancy
176. Dilated cardiomyopathy - causes include thiamine deficiency
177. Dilated cardiomyopathy - causes predominately systolic dysfunction
178. Dilated cardiomyopathy - is classically associated with an S3
179. Dilated cardiomyopathy - is the most common form of cardiomyopathy
180. Dilated cardiomyopathy - systolic dysfunction
181. Dipyridamole - non-specific phosphodiesterase inhibitor and decreases cellular uptake of adenosine
182. Dressler's syndrome - persistent fever following a myocardial infarction
183. ECG abnormalities typically include T wave inversion in V1-3 - arrhythmogenic right ventricular dysplasia
184. Echo findings include MR, systolic anterior motion (SAM) of the anterior mitral valve and asymmetric septal
hypertrophy - hypertrophic obstructive cardiomyopathy
185. Elective angioplasty : DVLA group 1 rules - cannot drive for 1 week
186. Electrical alternans - cardiac tamponade
187. Endocarditis caused by fully-sensitive streptococci (e.g. viridans) - benzylpenicillin
188. Epsilon wave - arrhythmogenic right ventricular dysplasia
189. Epworth Scale - suspected obstructive sleep apnoea
190. Eruptive xanthomas - familial hypertriglyceridaemia
191. False lumen on CT angiography - aortic dissection
192. Familial hypercholesterolaemia - tendon xanthoma
193. FAST - problematic alcohol use
194. Fixed split S2 - atrial septal defect
195. Fourth heart sound - aortic stenosis
196. Fourth heart sound - HOCM
8
M Y Elamin
MBBS, DTM&H, MCTM, MRCPI 1& 2
197. Fourth heart sound - systemic hypertension
198. FRAX - osteoporosis risk
199. Furosemide - inhibits the Na-K-Cl cotransporter in the thick ascending limb of the loop of Henle
200. GAD-7 - severity anxiety of symptoms
201. Giant V waves in the JVP may be caused by tricuspid regurgitation
202. Gleason score - prognosis in prostate cancer
203. Graham-Steel murmur (pulmonary regurgitation) - early diastolic murmur, high-pitched and 'blowing' in
character
204. Hydralazine - contraindicated by: recent myocardial infarction
205. Hydralazine - contraindicated by: systemic lupus erythematous
206. Hydralazine - increases cGMP leading to smooth muscle relaxation
207. Hydralazine may cause drug-induced lupus
208. Hydralazine may cause tachycardia
209. Hypertension : DVLA group 1 rules - no need to notify
210. Hypertrophic obstructive cardiomyopathy - causes predominately diastolic dysfunction
211. Hypertrophic obstructive cardiomyopathy - characterized by myofibrillar hypertrophy with chaotic and
disorganized fashion myocytes ('disarray') and fibrosis on biopsy
212. Hypertrophic obstructive cardiomyopathy - classically presents with exertional syncope
213. Hypertrophic obstructive cardiomyopathy - diastolic dysfunction
214. Hypertrophic obstructive cardiomyopathy - ejection systolic murmur
215. Hypertrophic obstructive cardiomyopathy - is associated with Friedreich's ataxia
216. Hypertrophic obstructive cardiomyopathy - is associated with sudden death in young athletes due to ventricular
arrhythmia
217. Hypertrophic obstructive cardiomyopathy - is associated with Wolff-Parkinson White
218. Hypertrophic obstructive cardiomyopathy - is characterized by decreased compliance, secondary to ventricular
hypertrophy
219. Hypertrophic obstructive cardiomyopathy - is classically associated with an S4
220. Hypotension, raised JVP, muffled heart sounds - cardiac tamponade
221. Increased P wave amplitude - cor pulmonale
222. Inherited long QT syndrome, normal hearing - Romano-Ward syndrome
223. Inherited long QT syndrome, sensorineural deafness - Jervell and Lange-Nielsen syndrome
224. Initial blind therapy of infective endocarditis (native valve) - amoxicillin, consider adding low-dose gentamicin
225. Initial blind therapy of infective endocarditis (native valve, penicillin allergic) - vancomycin + low-dose
gentamicin
226. Initial blind therapy of infective endocarditis (prosthetic valve) - vancomycin + rifampicin + low-dose gentamicin

9
M Y Elamin
MBBS, DTM&H, MCTM, MRCPI 1& 2
227. INR > 8.0 (minor bleeding) - stop warfarin, give intravenous vitamin K 1-3mg, repeat dose of vitamin K if INR
high after 24 hours, restart when INR < 5.0
228. INR > 8.0 (no bleeding) - stop warfarin, give oral vitamin K 1-5mg, repeat dose of vitamin K if INR high after 24
hours, restart when INR < 5.0
229. INR 5.0-8.0 (minor bleeding) - stop warfarin, give intravenous vitamin K 1-3mg, restart when INR < 5.0
230. INR 5.0-8.0 (no bleeding) - withhold 1 or 2 doses of warfarin, reduce subsequent maintenance dose
231. Intravenous immunoglobulin , uses include: Kawasaki disease
232. Inverted T waves - arrhythmogenic right ventricular cardiomyopathy
233. Inverted T waves - Brugada syndrome
234. Inverted T waves - digoxin toxicity
235. Inverted T waves - myocardial ischaemia
236. Inverted T waves - pulmonary embolism
237. Inverted T waves - subarachnoid haemorrhage
238. IPSS - severity of prostate symptoms
239. Ischaemic changes in leads I, aVL +/- V5-6 - left circumflex
240. Ischaemic changes in leads II, III, aVF - right coronary
241. Ischaemic changes in leads V1-6, I, aVL - left anterior descending
242. Ischaemic changes in leads V1-V4 - left anterior descending
243. Ischaemic heart disease - systolic dysfunction
244. Ischaemic stroke (cannot take clopidogrel) - antiplatelets: aspirin (lifelong) & dipyridamole (lifelong), no other
antiplatelets
245. Ischaemic stroke (no drug allergies) - antiplatelets: clopidogrel (lifelong), no other antiplatelets
246. Isosorbide mononitrate - increases cGMP leading to smooth muscle relaxation
247. Isosorbide mononitrate may cause flushing
248. Isosorbide mononitrate may cause headache
249. Isosorbide mononitrate may cause hypotension
250. Isosorbide mononitrate may cause tachycardia
251. IV adrenaline 1mg - during a VF/VT cardiac arrest, should be given once chest compressions have restarted after
the third shock (adult patient)
252. IV amiodarone 300mg - during a VF/VT cardiac arrest, should be given once chest compressions have restarted
after the third shock (adult patient)
253. IV amiodarone 150mg - during a VF/VT cardiac arrest, should be given once chest compressions have restarted
after the fifth shock (adult patient)
254. IV adrenaline 1mg - should be repeated every 3-5 minutes whilst ALS continues (adult patient)
255. IV adrenaline 1mg - should be given as soon as possible for non-shockable rhythms (adult patient)
256. IV thrombolytic - should be considered if a pulmonary embolus is suspected (adult patient)
10
M Y Elamin
MBBS, DTM&H, MCTM, MRCPI 1& 2
257. Ivabradine - acts on the I f ion current (inhibiting the ionic current I f) in the sinoatrial node

258. Ivabradine may cause blurred vision


259. Ivabradine may cause bradycardia
260. Ivabradine may cause headache
261. Ivabradine may cause heart block
262. IVDUs who present acutely - Staphylococcus aureus
263. J waves - hypothermia
264. Jerky pulse - hypertrophic obstructive cardiomyopathy
265. Kawasaki disease - bright red, cracked lips and strawberry tongue
266. Kawasaki disease - coronary artery aneurysm
267. Kawasaki disease - red palms of the hands and the soles of the feet which later peel
268. Kawasaki disease complication - coronary artery aneurysm
269. Leading cause of sudden cardiac death in young athletes - hypertrophic obstructive cardiomyopathy
270. Left axis deviation - atrial septal defect (ostium primum)
271. Left axis deviation - left anterior hemiblock
272. Left axis deviation - Wolff-Parkinson-White syndrome (right-sided accessory pathway)
273. Left bundle branch block - aortic stenosis
274. Left bundle branch block - hypertension
275. Left bundle branch block - myocardial ischaemia
276. Loop diuretics may cause gout
277. Loop diuretics may cause hypocalcaemia
278. Loop diuretics may cause hypokalaemia
279. Loop diuretics may cause hypomagnasaemia
280. Loop diuretics may cause hyponatraemia
281. Loop diuretics may cause impaired glucose tolerance
282. Loop diuretics may cause ototoxicity
283. Loud S1 - atrial premature beats
284. Loud S1 - hyperdynamic states
285. Loud S1 - left-to-right shunts
286. Loud S1 - mitral stenosis
287. Loud S1 - short PR interval
288. Loud S2 - hyperdynamic states
289. Loud S2 - pulmonary hypertension
290. Loud S2 - systemic hypertension
291. Low (< 15%) pre-test probability of PE - use PERC criteria

11
M Y Elamin
MBBS, DTM&H, MCTM, MRCPI 1& 2
292. Magnesium sulphate , uses include: torsades de pointes
293. Major bleeding - stop warfarin, give intravenous vitamin K 5mg, prothrombin complex concentrate
294. Medium-high (> 15%) pre-test probability of PE - calculate 2-level PE Wells score
295. Mitral regurgitation - holosystolic murmur, high-pitched and 'blowing' in character
296. Mitral regurgitation - soft S1
297. Mitral regurgitation - split S2
298. Mitral stenosis - loud S1
299. Mitral stenosis - low volume pulse
300. Mitral stenosis - malar flush
301. Mitral stenosis - mid-late diastolic murmur, 'rumbling' in character
302. Mitral stenosis - opening snap
303. Mitral valve prolapse - late systolic murmur
304. MMSE - cognitive impairment
305. MUST - malnutrition status
306. Myocarditis - systolic dysfunction
307. Narrow pulse pressure, slow rising pulse - aortic stenosis
308. Native valve endocarditis caused by staphylococci - flucloxacillin
309. Newly diagnosed patient of Afro-Caribbean origin with hypertension - add a calcium channel blocker
310. Newly diagnosed patient with hypertension (< 55 years) - add an ACE inhibitor or an angiotensin receptor blocker
311. Newly diagnosed patient with hypertension (> 55 years) - add a calcium channel blocke
312. Newly diagnosed patient with hypertension who has a background of type 2 diabetes mellitus - add an ACE
inhibitor or an angiotensin receptor blocker
313. Nicorandil - potassium-channel activator
314. Nicorandil may cause anal ulceration
315. Nicorandil may cause flushing
316. Nicorandil may cause headache
317. Nicotinic acid may cause flushing
318. NSTEMI identified - give aspirin 300mg + fondaparinux
319. NSTEMI identified, aspirin given - assess 6-month mortality using a tool such as GRACE
320. NSTEMI identified, aspirin given, 6-month mortality intermediate/high(>3%) - PCI within 72 hours
321. NSTEMI identified, aspirin given, 6-month mortality low (<3%) - conservative management - give ticagrelor
322. NSTEMI identified, aspirin given, intermediate/high risk, having PCI within 72 hours - give prasugrel or
ticagrelor
323. NSTEMI identified, aspirin given, intermediate/high risk, ticagrelor given, having PCI - radial access, give
unfractionated heparin, use drug-eluting stent
324. NYHA - heart failure severity
12
M Y Elamin
MBBS, DTM&H, MCTM, MRCPI 1& 2
325. Pacemaker insertion : DVLA group 1 rules - cannot drive for 1 week
326. Paget's disease - high-output heart failure
327. Palpitations, short PR interval, wide QRS complexes with a slurred upstroke - Wolff-Parkinson White syndrome
328. Patent ductus arteriosus - collapsing pulse
329. Patent ductus arteriosus - continuous 'machinery' murmur
330. Patent ductus arteriosus - left subclavicular thrill
331. Patent ductus arteriosus - wide pulse pressure
332. Patent ductus arteriosus , treatment of choice: indomethacin
333. IE Patients with no past medical history - Staphylococcus aureus
334. IE Patients with very poor dental hygiene - Streptococcus viridans
335. Peaked T waves - hyperkalaemia
336. Peaked T waves - myocardial ischaemia
337. Percutaneous coronary intervention (no drug allergies) - antiplatelets: aspirin (lifelong) & prasugral or ticagrelor
(12 months), no other antiplatelets
338. Peripheral arterial disease (cannot take clopidogrel) - antiplatelets: aspirin (lifelong), no other antiplatelets
339. Peripheral arterial disease (no drug allergies) - antiplatelets: clopidogrel (lifelong), no other antiplatelets
340. Persistent ST elevation following recent MI, no chest pain - left ventricular aneurysm
341. PHQ-9 - severity of depression symptoms
342. Pleuritic chest pain, MI 4 weeks ago - Dressler syndrome
343. Poorly controlled hypertension, already taking a calcium channel blocker - add an ACE inhibitor or an
angiotensin receptor blocker or a thiazide diuretic
344. Poorly controlled hypertension, already taking an ACE inhibitor - add a calcium channel blocker or a thiazide
diuretic
345. Poorly controlled hypertension, already taking an ACE inhibitor and a thiazide diuretic - add a calcium channel
blocker
346. Poorly controlled hypertension, already taking an ACE inhibitor, calcium channel blocker and a standard-dose
thiazide diuretic. K+ > 4.5mmol/l - add an alpha- or beta-blocker
347. Poorly controlled hypertension, already taking an ACE inhibitor, calcium channel blocker and a thiazide diuretic.
K+ < 4.5mmol/l - add spironolactone
348. PR depression - pericarditis
349. PR interval is constant but the P wave is often not followed by a QRS complex - second degree heart block (Mobitz
type II)
350. Prasugrel - antagonist of the P2Y12 adenosine diphosphate (ADP) receptor
351. Pregnancy-induced hypertension , treatment of choice: labetalol
352. Progressive prolongation of the PR interval until a dropped beat occurs - second degree heart block (Mobitz type
I)
13
M Y Elamin
MBBS, DTM&H, MCTM, MRCPI 1& 2
353. Prosthetic valve endocarditis caused by staphylococci - flucloxacillin + rifampicin + low-dose gentamicin
354. Prosthetic valves after two months - Staphylococcus aureus
355. Pulmonary embolism (history of CKD stage 4) - ventilation-perfusion scan
356. Pulmonary embolism (no past medical history) - CTPA
357. Pulmonary stenosis - ejection systolic murmur
358. Pulsus alternans - severe LVF
359. Pulsus parodoxus - cardiac tamponade
360. Pulsus parodoxus - severe asthma
361. Quiet S1 - mitral regurgitation
362. Ranson criteria - acute pancreatitis severity
363. Recent viral illness, pleuritic chest relieved by sitting forwards, PR depression, ST elevation - acute pericarditis
364. Restrictive cardiomyopathy - diastolic dysfunction
365. Reversed split S2 - aortic stenosis
366. Reversed split S2 - LBBB
367. Reversed split S2 - patent ductus arteriosus
368. Reversed split S2 - right ventricular pacing
369. Reversed split S2 - WPW type B
370. Right axis deviation - atrial septal defect (ostium secundum)
371. Right axis deviation - cor pulmonale
372. Right axis deviation - left posterior hemiblock
373. Right axis deviation - myocardial ischaemia
374. Right axis deviation - pulmonary embolism
375. Right axis deviation - right ventricular hypertrophy
376. Right axis deviation - Wolff-Parkinson-White syndrome (left-sided accessory pathway)
377. Right bundle branch block - atrial septal defect (ostium secundum)
378. Right bundle branch block - cor pulmonale
379. Right bundle branch block - myocardial ischaemia
380. Right bundle branch block - pulmonary embolism
381. Right bundle branch block - right ventricular hypertrophy
382. SCOFF - eating disorders
383. Severe tearing chest pain, unequal BP in arms - aortic dissection
384. Slow-rising/plateau pulse - aortic stenosis
385. Soft S2 - aortic stenosis
386. ST depression - digoxin toxicity
387. ST depression - hypokalaemia
388. ST depression - myocardial ischaemia
14
M Y Elamin
MBBS, DTM&H, MCTM, MRCPI 1& 2
389. ST elevation - Brugada syndrome
390. ST elevation - left ventricular anuerysm
391. ST elevation - myocardial ischaemia
392. ST elevation - pericarditis
393. ST elevation - subarachnoid haemorrhage
394. Statins - inhibit the action of HMG-CoA reductase
395. STEMI identified - give aspirin 300mg
396. STEMI identified, aspirin given - assess whether PCI possible within 120 minutes
397. STEMI identified, aspirin given, having PCI - give prasugrel
398. STEMI identified, aspirin given, having PCI, prasugrel given - radial access, give unfractionated heparin, use
drug-eluting stent
399. STEMI identified, aspirin given, having fibrinolysis - give an antithrombin
400. STEMI identified, aspirin given, has had fibrinolysis + antithrombin given - give ticagrelor
401. Streptococcus bovis endocarditis - colorectal cancer
402. Successful catheter ablation : DVLA group 1 rules - cannot drive for 2 days
403. Sudden heart failure, raised JVP, pulsus parodoxus, recent MI - left ventricular free wall rupture
404. Takayasu's arteritis - absent radial pulse
405. Takayasu's arteritis - granulomatous thickening of the aortic arch
406. Takayasu's arteritis - intermittent claudication
407. Takotsubo cardiomyopathy - is typically triggered by stress
408. Takotsubo cardiomyopathy - the majority of patients improve with supportive treatment
409. Tendon xanthoma - familial hypercholesterolaemia
410. Tetralogy of Fallot - ejection systolic murmur
411. Tetralogy of Fallot is cyanotic
412. IE in the first 2 months following prosthetic valve surgery - Staphylococcus epidermidis
413. Thiamine deficiency (wet Beri-Beri) - high-output heart failure
414. Thiazides may cause gout
415. Thiazides may cause hypercalcaemia
416. Thiazides may cause hypokalaemia
417. Thiazides may cause hyponatraemia
418. Thiazides may cause impaired glucose tolerance
419. Thiazides may cause impotence
420. Thiazides may cause pancreatitis
421. Third heart sound - constrictive pericarditis
422. Third heart sound - left ventricular failure
423. Third heart sound - mitral regurgitation
15
M Y Elamin
MBBS, DTM&H, MCTM, MRCPI 1& 2
424. Thyrotoxicosis - high-output heart failure
425. TIA (no drug allergies) - antiplatelets: clopidogrel (lifelong), no other antiplatelets
426. TIA (cannot take clopidogrel) - antiplatelets: aspirin (lifelong) & dipyridamole (lifelong), no other antiplatelets
427. Ticagrelor - antagonist of the P2Y12 adenosine diphosphate (ADP) receptor
428. Transposition of the great arteries is cyanotic
429. Tricuspid atresia is cyanotic
430. Tricuspid regurgitation - holosystolic murmur, high-pitched and 'blowing' in character
431. Typically triggered by stress or emotion - Takotsubo cardiomyopathy
432. U waves - hypokalaemia
433. Usually due to a mutation in the gene encoding beta-myosin heavy chain protein - hypertrophic obstructive
cardiomyopathy
434. Variable intensity of S1 - complete heart block
435. Ventricular myocardium is replaced by fatty and fibrofatty tissue - arrhythmogenic right ventricular dysplasia
436. Ventricular septal defect - holosystolic murmur, 'harsh' in character
437. Ventricular septal defect is acyanotic
438. Warfarin - antagonises the effects of vitamin K
439. Warfarin - inhibits carboxylation of clotting factor II, VII, IX and X (1972)
440. Warfarin may cause skin necrosis
441. Waterlow score - pressure sore risk
442. Wells score - risk of a patient having a deep vein thrombosis
443. Widely split S2 - deep inspiration
444. Widely split S2 - mitral regurgitation
445. Widely split S2 - pulmonary stenosis
446. Widely split S2 - RBBB
447. Widened mediastinum on chest x-ray - aortic dissection
448. Guidance on use of ICD has changed, and where there is QRS prolongation in class III heart failure, an ICD is
now potentially indicated.
449. Experimental evidence supports a positive effect of metformin on cardiac function, both in models of ischaemia
and in models of heart failure
450. Ivabradine is recommended for patients who are tachycardic with heart failure, (who have a pulse of 75 or more).
In this situation valsartan does not bring a significant enough improvement in symptoms of heart failure, versus the
risk of worsening hyperkalaemia.
451. 'Provoked' pulmonary embolisms are typically treated for 3 months
452. 'Unprovoked' pulmonary embolisms are typically treated for 6 months
453. Streptococcus bovis endocarditis is associated with colorectal cancer
454. Streptococcus gallolyticus is the subtype of Streptococcus bovis most linked with colorectal cancer
16
M Y Elamin
MBBS, DTM&H, MCTM, MRCPI 1& 2
455. A beta-blocker or a calcium channel blocker is used first-line to prevent angina attacks
456. A potassium above 6mmol/L should prompt cessation of ACE inhibitors in a patient with CKD (once other agents
that promote hyperkalemia have been stopped)
457. A prolonged PR interval - aortic root abscess
458. A single episode of paroxysmal atrial fibrillation, even if provoked, should still prompt consideration of
anticoagulation
459. A stable patient presenting in AF with an obvious precipitating cause may revert to sinus rhythm without specific
antiarrhythmic treatment
460. ACE inhibitors can cause first dose hypotension
461. ACE inhibitors have reduced efficacy in black patients and are therefore not used first-line
462. ACE inhibitors prevent the breakdown of inflammatory peptides such as bradykinin and cough is a frequent side
effect.
463. ACE-inhibitors should be avoided in patients with HOCM
464. Acute vasodilator testing should be used in patients with pulmonary artery hypertension to determine which
patient show a significant fall in pulmonary arterial pressure following vasodilators and help guide treatment
465. Adenosine
a. dipyridamole enhances effect
b. aminophylline reduces effect
466. ALS - give adrenaline in non-shockable rhythm as soon as possible
467. Ambrisentan - endothelin-1 receptor antagonist
468. Aminophylline reduces the effect of adenosine
469. Amiodarone - MOA: blocks potassium channels
470. Amiodarone has a long half-life - it is highly lipophilic and widely absorbed by tissue, which reduces its
bioavailability in serum. Therefore, a prolonged loading regime is required to achieve stable therapeutic levels
471. An undersized blood pressure cuff may lead to an overestimation of blood pressure
472. Angiotensin II receptor blockers block the effects of angiotensin 2 at the AT1 receptor
473. Angiotensin-receptor blockers should be used where ACE inhibitors are not tolerated
474. Antibiotic prohylaxis to prevent infective endocarditis is not routinely recommended in the UK for dental and
other procedures
475. Anticoagulation should be considered for the following:
a. Men: CHA2DS2-VASC >= 1
b. Women CHA2DS2-VASC >= 2
476. Aortic dissection
a. type A - ascending aorta - control BP (IV labetalol) + surgery
b. type B - descending aorta - control BP(IV labetalol)
477. Aortic regurgitation - early diastolic murmur, high-pitched and 'blowing' in character
17
M Y Elamin
MBBS, DTM&H, MCTM, MRCPI 1& 2
478. Aortic regurgitation typically causes an early diastolic murmur
479. Aortic stenosis - most common cause:
a. younger patients < 65 years: bicuspid aortic valve
b. older patients > 65 years: calcification
480. Aortic stenosis - S4 is a marker of severity
481. Aortic stenosis management: AVR if symptomatic, otherwise cut-off is gradient of 40 mmHg
482. Arrhythmogenic right ventricular cardiomyopathy - T wave inversion in V1-3
483. Arrhythmogenic right ventricular cardiomyopathy is characterised by right ventricular myocardium replaced by
fatty and fibrofatty tissue
484. Aschoff bodies are granulomatous nodules found in rheumatic heart fever
485. Asymmetric dosing regimes should be used for standard-release ISMN to prevent nitrate tolerance
486. Asymmetric septal hypertrophy and systolic anterior movement (SAM) of the anterior leaflet of mitral valve on
echocardiogram or cMR support HOCM
487. Atrial fibrillation - cardioversion: amiodarone + flecainide
488. Atrial fibrillation: rate control - beta blockers preferable to digoxin
489. Atrial myxoma - commonest site = left atrium
490. Atrial septal defect - ejection systolic murmur louder on inspiration
491. Atrioventricular dissociation suggests VT rather than SVT with aberrant conduction
492. AV block can occur following an inferior MI
493. B-type natriuretic peptide is mainly secreted by the ventricular myocardium
494. Beck’s triad of falling BP, rising JVP and muffled heart sound is characteristic of cardiac tamponade
495. Bisferiens pulse - mixed aortic valve disease
496. Bleeding on dabigatran? Can use idarucizumab to reverse
497. Blood pressure target (< 80 years, clinic reading) - 140/90 mmHg
498. BNP - actions:
a. vasodilator: can decrease cardiac afterload
b. diuretic and natriuretic
c. suppresses both sympathetic tone and the renin-angiotensin-aldosterone system
499. Bosentan - endothelin-1 receptor antagonist
500. Cardiac catheterisation results - jump in oxygen saturation from right atrium to right ventricle → ?VSD
501. Cardiac catheterisation results - An atrial septal defect would be characterised by a higher level of oxygenated blood
in the right atrium (approximately 85%).
502. Cardiac catheterisation results - Eisenmenger syndrome refers to any untreated congenital cardiac defect with
intracardiac communication that leads to pulmonary hypertension, reversal of flow, and cyanosis one would
expect deoxygenated blood to travel from the right to the left side resulting in a low oxygen saturation of blood in
the left atrium (approximately 85%).
18
M Y Elamin
MBBS, DTM&H, MCTM, MRCPI 1& 2
503. Cardiac catheterisation results - ductus arteriosus, so one would expect oxygenated blood to travel from the aorta
into the pulmonary artery via the ductus arteriosus, resulting in a higher-than-expected oxygen saturation of blood
in the pulmonary artery (approximately 85%).
504. Cardiac catheterisation results - Eisenmenger's defect is in the ventricle, so one would expect deoxygenated blood
to travel from the right ventricle to the left ventricle resulting in a low oxygen saturation of blood in the left ventricle
(approximately 85%).

Diagnosis & notes RA RV PA LA LV Aorta


Normal 70% 70% 70% 100% 100% 100%
Atrial septal defect (ASD) 85% 85% 85% 100% 100% 100%
Ventricular septal defect (VSD) 70% 85% 85% 100% 100% 100%
Patent ductus arteriosus (PDA) 70% 70% 85% 100% 100% 100%
VSD with Eisenmenger's 70% 70% 70% 100% 85% 85%
PDA with Eisenmenger's 70% 70% 70% 100% 100% 85%
ASD with Eisenmenger's 70% 70% 70% 85% 85% 85%
505. Clopidogrel inhibits ADP binding to platelet receptors
506. Complete heart block causes a variable intensity of S1
507. Complete heart block following a MI? - right coronary artery lesion
508. Complete heart block following an inferior MI is NOT an indication for pacing, unlike with an anterior MI
509. Congenital heart disease
a. cyanotic: TGA most common at birth, Fallot's most common overall
b. acyanotic: VSD most common cause
510. Contrast-enhanced CT coronary angiogram is the first line investigation for stable chest pain of suspected
coronary artery disease aetiology
511. Creatine kinase (CK-MB) remains elevated for 3 to 4 days following infarction. Troponin remains elevated for
10 days. This makes CK-MB useful for detecting re-infarction in the window of 4 to 10 days after the initial insult
512. Dabigatran is a direct thrombin inhibitor
513. Dentistry in warfarinised patients - check INR 72 hours before procedure, proceed if INR < 4.0
514. Dipyridamole is a non-specific phosphodiesterase inhibitor and decreases cellular uptake of adenosine
515. DOACs should be offered first-line for reducing stroke risk in AF
516. DVLA advice following angioplasty - cannot drive for 1 week
517. DVLA advice post MI - cannot drive for 4 weeks
518. Dyspnoea in ticagrelor-treated patients is due to the impaired clearance of adenosine
519. Ebstein's anomaly → tricuspid regurgitation → pansystolic murmur, worse on inspiration
520. Eclampsia - give magnesium sulphate first-line
521. Eisenmenger's syndrome - the reversal of a left-to-right shunt
19
M Y Elamin
MBBS, DTM&H, MCTM, MRCPI 1& 2
522. Electrical cardioversion is synchronised to the R wave
523. Endothelin receptor antagonists decrease pulmonary vascular resistance in patients with primary pulmonary
hypertension
524. Erythromycin can cause a prolonged QT interval
525. First line management of acute pericarditis involves combination of NSAID and colchicine
526. For patients of black African or African–Caribbean origin taking a calcium channel blocker for hypertension, if
they require a second agent consider an angiotensin receptor blocker in preference to an ACE inhibitor
527. Functional tricuspid regurgitation often occurs secondary to pulmonary hypertension
528. Furosemide - inhibits the Na-K-Cl cotransporter in the thick ascending limb of the loop of Henle
529. Gallop rhythm (S3) is an early sign of LVF
530. Global ST and PR segment changes: think pericarditis
531. HOCM - poor prognostic factor on echo = septal wall thickness of > 3cm
532. HOCM is usually due to a mutation in the gene encoding β-myosin heavy chain protein or myosin binding protein
C
533. Hydralazine - increases cGMP leading to smooth muscle relaxation
534. Hypertension - NICE now recommend ambulatory blood pressure monitoring to aid diagnosis
535. Hypertension in diabetics - ACE inhibitors/A2RBs are first-line regardless of age
536. Hypocalcemia is associated with QT interval prolongation;
537. Hypercalcemia is associated with QT interval shortening
538. Hypokalaemia - U waves on ECG
539. Hypokalaemia can lead to long QT syndrome
540. ICD means loss of HGV licence, regardless of the circumstances
541. Palpitations should first be investigated with a Holter monitor after initial bloods/ECG
542. In mitral stenosis, an opening snap indicates the leaflets still have some mobility
543. If angina is not controlled with a beta-blocker, a longer-acting dihydropyridine calcium channel blocker should
be added
544. If high-risk of failure of cardioversion (previous failure), offer electrical cardioversion after at least 4 weeks
treatment with amiodarone
545. In the context of a tachyarrhythmia, a systolic BP < 90 mmHg → DC cardioversion
546. Indomethacin or ibuprofen is used in patent ductus arteriosus to promote duct closure
547. Infective endocarditis - indications for surgery:
a. severe valvular incompetence
b. aortic abscess (often indicated by a lengthening PR interval)
c. infections resistant to antibiotics/fungal infections
d. cardiac failure refractory to standard medical treatment
e. recurrent emboli after antibiotic therapy
20
M Y Elamin
MBBS, DTM&H, MCTM, MRCPI 1& 2
548. Infective endocarditis - streptococcal infection carries a good prognosis
549. Infective endocarditis - strongest risk factor is previous episode of infective endocarditis
550. Infective endocarditis causing congestive cardiac failure is an indication for emergency valve replacement surgery
551. Inferior MI - right coronary artery lesion
552. Inherited long QT syndrome, sensorineural deafness - Jervell and Lange-Nielsen syndrome
553. INR > 8.0 (no bleeding) - stop warfarin, give oral vitamin K 1-5mg, repeat dose of vitamin K if INR high after 24
hours, restart when INR < 5.0
554. Irregular cannon 'a' waves points towards complete heart block
555. Ischaemic changes in leads I, aVL +/- V5-6 - left circumflex
556. Ischaemic changes in leads V1-V4 - left anterior descending
557. IV magnesium sulfate is used to treat torsades de pointes
558. Ivabradine use may be associated with visual disturbances including phosphenes and green luminescence
559. J-waves are associated with hypothermia
560. JVP: C wave - closure of the tricuspid valve
561. JVP: giant v waves in tricuspid regurgitation
562. JVP: x descent = fall in atrial pressure during ventricular systole
563. JVP: y descent = opening of tricuspid valve
564. Labetalol is first-line for pregnancy-induced hypertension
565. Left axis deviation - Wolff-Parkinson-White syndrome (right-sided accessory pathway)
566. Left parasternal heave is a feature of tricuspid regurgitation
567. Long QT syndrome - usually due to loss-of-function/blockage of K+ channels
568. Magnesium sulphate - monitor reflexes + respiratory rate
569. Major bleeding - stop warfarin, give intravenous vitamin K 5mg, prothrombin complex concentrate
570. Massive PE + hypotension - thrombolyse
571. Mechanical valves - target INR:
a. aortic: 3.0
b. mitral: 3.5
572. Methadone is a common cause of QT prolongation
573. Migraine, stroke → ?patent foramen ovale
574. Most common cause of endocarditis:
a. Staphylococcus aureus
b. Staphylococcus epidermidis if < 2 months post valve surgery
575. Mutations in the SCN5A gene (which encode the myocardial sodium ion channel protein are the most common gene
abnormality seen in Brugada syndrome
576. Myoglobin rises first following a myocardial infarction
577. Naftidrofuryl is a 5-HT2 receptor antagonist which can be used for peripheral vascular disease
21
M Y Elamin
MBBS, DTM&H, MCTM, MRCPI 1& 2
578. New LBBB is always pathological and never normal
579. New onset AF is considered for electrical cardioversion if it presents within 48 hours of presentation
580. Newly diagnosed patient with hypertension (> 55 years) - add a calcium channel blocker
581. Nicorandil is a potassium channel activator
582. Nicorandil may cause ulceration anywhere in the gastrointestinal tract
583. Nitrates should be avoided in the likely diagnosis of right ventricular myocardial infarct due to causing reduced
preload
584. Obesity can caused decreased BNP levels, leading to a false negative for heart failure screening
585. Offer a mineralcorticoid receptor antagonist, in addition to an ACE inhibitor (or ARB) and beta-blocker, to people
who have heart failure with reduced ejection fraction if they continue to have symptoms of heart failure
586. Palpitations should first be investigated with a Holter monitor after initial bloods/ECG
587. Paradoxical embolus - PFO most common cause - do TOE
588. Patent ductus arteriosus - large volume, bounding, collapsing pulse
589. Patent ductus arteriosus: indomethacin is given to the neonate in the postnatal period, not to the mother in the
antenatal period
590. Patients on warfarin undergoing emergency surgery - give four-factor prothrombin complex concentrate
591. Patients with stable CVD who have AF are generally managed on an anticoagulant and the antiplatelets stopped
592. Patients with a suspected pulmonary embolism should be initially managed with a direct oral anticoagulant
(DOAC)
593. Patients with heart failure with reduced LVEF should be given a beta blocker and an ACE inhibitor as first-line
treatment
594. Patients with recurrent venous thromboembolic disease may be considered for an inferior vena cava filter
595. Patients with SVT who are haemodynamically stable and who do not respond to vagal manoeuvres, the next step
is treating with adenosine
596. Patients with very poor dental hygiene may develop endocarditis secondary to Viridans streptococci e.g.
Streptococcus sanguinis
597. Patients with VT should not be prescribed verapamil
598. PCI - patients with drug-eluting stents require a longer duration of clopidogrel therapy
599. PCI: stent thrombosis - withdrawal of antiplatelets biggest risk factor
600. People with cardiac syndrome X have normal coronary angiograms despite ECG changes on exercise stress
testing
601. Percutaneous mitral commissurotomy is the intervention of choice for severe mitral stenosis
602. Persistent ST elevation following recent MI, no chest pain - left ventricular aneurysm
603. Poorly controlled hypertension, already taking a calcium channel blocker - add an ACE inhibitor or an angiotensin
receptor blocker or a thiazide-like diuretic

22
M Y Elamin
MBBS, DTM&H, MCTM, MRCPI 1& 2
604. Poorly controlled hypertension, already taking an ACE inhibitor - add a calcium channel blocker or a thiazide-
like diuretic
605. Poorly controlled hypertension, already taking an ACE inhibitor and a thiazide diuretic - add a calcium channel
blocker
606. Poorly controlled hypertension, already taking an ACE inhibitor, calcium channel blocker and a standard-dose
thiazide diuretic. K+ > 4.5mmol/l - add an alpha- or beta-blocker
607. Poorly controlled hypertension, already taking an ACE inhibitor, calcium channel blocker and a thiazide diuretic.
K+ < 4.5mmol/l - add spironolactone
608. Pregnancy is a contraindication to statin therapy
609. Primary percutaneous coronary intervention is the gold-standard treatment for ST-elevation myocardial
infarction
610. Prinzmetal angina - treatment = dihydropyridine calcium channel blocker
611. Prominent V waves on JVP → tricuspid regurgitation
612. Prostacyclins is used in the treatment of primary pulmonary hypertension
613. Prosthetic heart valves - antithrombotic therapy:
a. bioprosthetic: aspirin
b. mechanical: warfarin + aspirin
614. Prosthetic heart valves - mechanical valves last longer and tend to be given to younger patients
615. Prosthetic valve endocarditis caused by staphylococci → Flucloxacillin + rifampicin + low-dose gentamicin
616. Pulmonary arterial hypertension most commonly presents with exertional dyspnoea. Patients may also experience
exertional chest pain, syncope and peripheral oedema
617. Pulmonary arterial hypertension patients with negative response to vasodilator testing should be treated with
prostacyclin analogues, endothelin receptor antagonists or phosphodiesterase inhibitors. Often combination
therapy is required
618. Pulmonary arterial hypertension patients with positive response to vasodilator testing should be treated with
calcium channel blockers
619. Pulmonary embolism - CTPA is first-line investigation
620. Pulmonary embolism - normal CXR
621. Pulmonary hypertension is a cause of a loud S2 (due to a loud P2)
622. Pulmonary stenosis is louder on inspiration
623. Pulsus alternans - seen in left ventricular failure
624. QT interval: Time between the start of the Q wave and the end of the T wave
625. Rate-limiting CCBs should be avoided in patients with AF with heart failure with reduced EF (HFrEF) due to
their negative inotropic effects
626. Renal dysfunction (eGFR < 60) can cause a raised serum natriuretic peptides

23
M Y Elamin
MBBS, DTM&H, MCTM, MRCPI 1& 2
627. Restrictive cardiomyopathy: amyloid (most common), haemochromatosis, Loffler's syndrome, sarcoidosis,
scleroderma
628. Right axis deviation - left posterior hemiblock
629. Risk of falls or old age alone is not sufficient reasoning to withhold anticoagulation
630. Rupture of the papillary muscle due to a myocardial infarction → acute mitral regurgitation → widespread
systolic murmur, hypotension, pulmonary oedema
631. S4 coincides with the P wave on ECG
632. Sacubitril-valsartan is considered in heart failure patients with a LVEF < 35% who are still symptomatic on ACE-
inhibitors & beta-blockers
633. Second heart sound (S2)
a. loud: hypertension
b. soft: AS
c. fixed split: ASD
d. reversed split: LBBB
634. Severe pre-eclampsia - restrict fluids
635. Sotalol is known to cause long QT syndrome
636. Staphylococci is the leading organism contributing to mortality in infective endocarditis
637. Statins + erythromycin/clarithromycin - an important and common interaction
638. Statins inhibit HMG-CoA reductase, the rate-limiting enzyme in hepatic cholesterol synthesis
639. STEMI management: fibrinolysis should be offered within 12 hours of onset of symptoms if primary PCI cannot
be delivered within 120 minutes
640. Symptomatic bradycardia is treated with atropine
641. Tachycardia with a rate of 150/min ?atrial flutter
642. Takayasu's arteritis is an obliterative arteritis affecting the aorta
643. Takotsubo cardiomyopathy is a differential for ST-elevation in someone with no obstructive coronary artery
disease
644. The first-line management of SVT is vagal manoeuvres: e.g. Valsalva manoeuvre or carotid sinus massage
645. The main ECG abnormality seen with hypercalcaemia is shortening of the QT interval
646. The most specific ECG finding in acute pericarditis is PR depression
647. The recommended dose of adrenaline to give during advanced ALS is 1mg
648. The two level Well's score can be used in patients presenting with signs and symptoms suggestive of PE to guide
the next investigation
649. Thiazide diuretics can cause hyponatraemia, metabolic alkalosis, hypokalaemia and hypocalciuria
650. Thiazides can worsen glucose tolerance
651. Thiazides/thiazide-like drugs (e.g. indapamide) - inhibits sodium reabsorption by blocking the Na+-Cl−
symporter at the beginning of the distal convoluted tubule
24
M Y Elamin
MBBS, DTM&H, MCTM, MRCPI 1& 2
652. Third heart sound - constrictive pericarditis
653. Ticagrelor has a similar mechanism of action to clopidogrel - inhibits ADP binding to platelet receptors
654. Transposition of great vessels is due to the failure of the aorticopulmonary septum to spiral
655. Tricuspid regurgitation becomes louder during inspiration, unlike mitral regurgitation
656. Tricuspid valve endocarditis can cause tricuspid regurgitation, which may manifest with a new pan-systolic
murmur, large V waves and features of pulmonary emboli
657. Ventricular tachycardia - verapamil is contraindicated
658. Verapamil and beta-blockers should never be taken concurrently - possibility of heart block and fatal arrest
659. VF/pulseless VT should be treated with 1 shock as soon as identified
660. Warfarin - clotting factors affected mnemonic - 1972 (10, 9, 7, 2)
661. When treating angina, if there is a poor response to the first-line drug (e.g. a beta-blocker), the dose should be
titrated up before adding another drug
662. Witnessed cardiac arrest while on a monitor - up to three successive shocks before CPR
663. Women with pulmonary hypertension should avoid becoming pregnant due to very high mortality levels
664. Aortic stenosis occurs frequently in the population suffering proximal femoral fractures.
665. In women with oopthrectomy unopposed oestrogen HRT is appropriate.
666. Septic arthritis needs to be ruled out in cases of an acute, hot, swollen, painful joint.
667. Positive ANA is present in nearly all cases of SLE
668. Infiltrative lymphocytic syndrome (DILS) can present in many ways and occurs in patients who are HIV positive.
669. SLE is a cause of splenomegaly and should be considered when there is arthralgia and inflammatory change
670. Biopsy in IBM shows intranuclear or cytoplasmic tubofilaments on electron microscopy.
671. Behçet's syndrome is a systemic vasculitis with an unknown aetiology, which affects small and large vessels (venous
and arterial).
672. Cardiac resynchronisation therapy with defibrillator CRT-D is recommended for patients with class III heart
failure who also have LBBB or have a very prolonged QRS, (>150ms) without LBBB. CRT-P is recommended for
patients with class IV heart failure who have a prolonged QRS.
673. implantable cardioverter defibrillator (ICD). Where there is no LBBB and QRS is between 120-149ms, ICD is the
recommended option according to NICE guidelines. This is because of the risk of VT on account of the low ejection
fraction (<35%) and symptomatic heart failure.
674. Cardiac resynchronisation therapy with defibrillator (CRT-D) is also an option for patients with class III heart
failure who have LBBB or a markedly prolonged QRS without LBBB.
675. 10 ml of 1 in 10,000 is the recommended dose and concentration and is considered the optimum volume of adrenaline
during cardiac arrest, and is recommended by the UK Resuscitation Council.
676. 300 mg of amiodarone made up to 20 ml with 5% dextrose given as an intravenous bolus is the drug of choice for
treating refractory ventricular fibrillation or pulseless ventricular tachycardia

25
M Y Elamin
MBBS, DTM&H, MCTM, MRCPI 1& 2
677. A history of collapses in this young woman with echocardiographic features of hypertrophy are highly suggestive of
hypertrophic obstructive cardiomyopathy.
678. A history of congenital heart disease presents with a pansystolic murmur, large V waves in the JVP, and pulsatile
hepatomegaly are suggestive of Ebstein's anomaly.
679. A loud first heart sound is due to abrupt closure of the mitral valve against a high left atrial pressure.
680. A pansystolic or holosystolic murmur is seen in septal defects and, more commonly, mitral regurgitation.
681. A patent foramen ovale can result in a transient ischaemic attack or stroke when thrombus travels from a DVT - a
'paradoxical embolus'.
682. A patient showing signs of haemodynamic decompromise (that is, systolic blood pressure less than 90) should be
immediately DC cardioverted under sedation/anaesthesia.
683. A precordial thump can be successful if given within seconds of the onset of a shockable rhythm.
684. A precordial thump can be successful if given within seconds of the onset of a shockable rhythm. Delivery should not
delay calling for help, or accessing a defibrillator, but would be indicated here whilst awaiting the defibrillator.
685. A rise in cardiac troponin compared with previous one accompanied with a good history for cardiac chest pain would
support the presence of myocardial ischaemia
686. A standard beta-blocker is recommended for patients as rate control in AF.
687. A temporary wire is an appropriate measure until inferior MI has fully resolved.
688. A volume overload, as with AR, would result in dilatation of the left ventricle.
689. A wide fixed splitting of the second heart sound during auscultation occurs in uncomplicated ASD.
690. Abdominal aortic plaque rupture is the commonest cause of acute emobolus causing distal ischaemia.
691. Abnormal blood pressure response to exercise is a marker for increased risk of sudden cardiac death in hypertrophic
cardiomyopathy.
692. About 6 out of 10 people with Marfan's syndrome have dislocated lenses in one or both eyes.
693. Absolute contraindications to thrombolysis include previous haemorrhagic stroke.
694. According to NICE CG127, for people identified as having a white-coat effect, consider ABPM or HBPM as an adjunct
to clinic blood pressure measurements to monitor the response to treatment
695. ACE inhibitors and beta blockers are first line treatments in heart failure.
696. Acute aortic dissection presents with collapse, preceded by severe chest pain radiating to the back.
697. Adenosine has a particularly short half life, acts on specific adenosine cell surface receptors (A1 and A2) and is
inactivated by adenosine deaminase. It results in coronary vasodilatation and depression of sinus node automaticity
and AVN conduction.
698. Administration of bisoprolol to patients with chronic heart failure is associated with increases in left ventricular
function and reductions in heart rate; increases in heart rate variability are also seen.
699. After myocardial infarction (MI), CK-MB levels become elevated within three to eight hours, peak within nine to 30
hours, and return to normal after 48 to 72 hours.

26
M Y Elamin
MBBS, DTM&H, MCTM, MRCPI 1& 2
700. After the third shock the next step is amiodarone 300 mg intravenously . If amiodarone is not available lidocaine is a
suitable alternative.
701. Alcohol withdrawal is associated with anxiety, tachyarrhythmias and seizures and should be treated with
benzodiazepines.
702. All patients diagnosed with Streptococcus bovis infective endocarditis should undergo evaluation including
investigation of the gastrointestinal tract.
703. Alternative treatment strategies are no longer recommended by NICE for treatment of hypertension
704. Amiodarone can cause hypothyroidism by inhibiting the peripheral conversion of T4 to T3.
705. Amiodarone has been shown to be superior in maintaining sinus rhythm following DC cardioversion of AF.
706. Amiodarone is a class 3 anti-arrhythmic.
707. Amiodarone is a class III antiarrhythmic and as such is used in many supra- and ventricular arrhythmias.
708. Amiodarone may cause both hypo- and hyperthyroidism. In hyperthyroidism, it interferes in the peripheral conversion
of T4 to T3 and increases reverse T3, TSH. After a few months, all return to normal other than T4, FT4 and reverse T3
which remain mildly elevated or high normal.
709. Amyloidosis typically causes an infiltrative restrictive cardiomyopathy in elderly patients.
710. An anteroseptal myocardial infarction (MI) is due to an infarct in the territory of the left anterior descending artery.
711. An ascending aortic dissection needs immediate surgery. Whilst en route to surgery, beta blockade to control
hypertension is appropriate.
712. An atrial or a ventricular septal defect can provide the persistent shunt suggested by pulmonary hypertension.
713. An ejection click or ejection sound, best heard at the apex, implies that the site of the stenosis is mostly valvular and
of congenital origin, that is, bicuspid valvular disease.
714. Angiotensin II, when infused intravenously, produces an immediate rise in blood pressure being a potent
vasoconstrictor.
715. Angiotensin-converting enzyme (ACE) inhibitors and angiotensin 2 receptor blockers (A2RBs) commonly cause
rashes some of which appear to be photosensitive.
716. Ankle-brachial pressure index is a simple way of assessing arterial disease in a primary care setting as a first line.
717. Annual risk is calculated by the population size divided by the number of patients who developed the condition in
question.
718. Appropriate blood pressure treatment in the elderly yields not only a decrease in cardiovascular events, but also
appreciably improved all cause mortality
719. Aspirin is not advised for primary prevention of ischaemic heart disease in a low-risk population
720. AST levels become elevated at 6-10 hours, peak at 24-36 hours and remain high for 3-5 days, but it is not widely tested
due to being non-specific.
721. At high respiratory rates, with severe air flow limitation (for example, acute asthma) there is an increased and sudden
negative intrathoracic pressure on inspiration and this will enhance the normal fall in blood pressure.
722. Atrial myxomas in the heart make up 50% of primary cardiac tumours.
27
M Y Elamin
MBBS, DTM&H, MCTM, MRCPI 1& 2
723. Atrioventricular septal defect is the commonest congenital heart defect in Down's
724. Based upon isolated systolic hypertension studies such as SHEP and Syst-Eur, guidelines suggest treatment with either
calcium antagonists or diuretics.
725. Beta-blockers improve mortality and quality of life in chronic heart failure (COPERNICUS, MERIT, CIBIS trials).
726. Beta-blockers, excluding sotalol, are the mainstay of treatment in long QT syndrome.
727. Bicarbonate therapy can increase extracellular pH only if the carbon dioxide (CO2) produced can be removed by
adequate ventilation.
728. BNP is a peptide synthesised in the cardiac ventricles which can be used in the diagnosis of congestive cardiac failure.
729. B-type natriuretic peptide BNP leads to reduced blood pressure, and reduced pre-load due to relaxing effects on
vascular smooth muscle.
730. Bumetanide is a loop diuretic and may cause hypokalaemia as a side effect. The potassium loss caused by bumetanide
increases the toxicity of digoxin.
731. By five years following cardiac transplantation, nearly all patients have some degree of small coronary vascular
narrowing.
732. Calcification is the commonest reason for a prosthetic aortic valve to require replacement
733. Calcitonin-gene related peptide causes vasodilatation
734. Cardiac resynchronisation therapy is useful in optimally treated NYHA class III/IV heart failure, SR, ejection fraction
< 35% and BBB.
735. Cardiac tamponade is characterised by Beck's triad of hypotension, raised JVP, and muffled heart sounds.
736. Celecoxib (rofecoxib has been withdrawn) acts by inhibiting prostaglandin synthesis via inhibition of cyclo-oxygenase-
2 (COX-2). It causes fluid retention and can worsen an already pre-existing heart failure.
737. Chest pain in association with nausea is a good clinical indicator of a diagnosis of acute coronary syndrome.
738. Cholesterol emboli can occur following thrombolysis, and result in a petechial rash, marked eosinophilia and raised
IgE.
739. Chronic hypertension is defined as blood pressure exceeding 140/90 mm Hg before pregnancy or before 20 weeks'
gestation.
740. Chronic hypertension refers to high BP predating the pregnancy or recognized at <20 weeks’ gestation. In practice,
this is often diagnosed for the first time at the first or early second trimesters booking visit.
741. Chronic obstructive airways disease and subsequent cor pulmonale can lead to right heart failure.
742. Churg-Strauss syndrome typically presents with asthma, mild renal impairment, microscopic haematuria, and raised
eosinophilic count.
743. Clinical trials have demonstrated that warfarin reduces the risk of stroke in patients with AF. This benefit outweighs
the risk of bleeding.
744. Clopidogrel is now recommended for prevention of occlusive vascular events in those with established peripheral
vascular disease.
745. Clopidogrel prevents platelet aggregation through antagonism of the adenosine diphosphate (ADP) receptor.
28
M Y Elamin
MBBS, DTM&H, MCTM, MRCPI 1& 2
746. CO = HR × SV
747. Coarctation of the aorta may present in adulthood in patients who have poor access to health service, with headaches,
nosebleeds and signs of arterial insufficiency distal to the coarctation.
748. Complete heart block (whether symptomatic or not) is a indication for a permanent pacemaker.
749. Concentric left ventricular hypertrophy is the response to out flow obstruction.
750. Conn’s syndrome. Secondary causes of hypertension.
751. Conn's syndrome (hyperaldosteronism) is characterised by hypertension, hypokalaemia, high aldosterone, low renin.
752. Consider renal artery stenosis if someone with a history of hypertension presents with flash pulmonary oedema.
753. Considering the potential complications of major abdominal surgery.
754. Constrictive pericarditis can be a mid to long term complication of cardiac surgery, due to a number of mechanisms,
and often has a non-specific presentation.
755. Constrictive pericarditis can be caused by prior tuberculous infection.
756. Constrictive pericarditis is the commonest cardiac complication of rheumatoid arthritis, and presents with dyspnoea,
atrial fibrillation, lower limb oedema, ascites, raised JVP, and biatrial enlargement with normal systolic ventricular
function.
757. Contraindications to thrombolysis include intracranial neoplasm, pregnancy, aoritc dissection and active internal
bleeding.
758. Coordinated electrical conduction pathway of the heart allows the passage of blood through the heart and the rest of
the body.
759. Current AHA/EHA criteria for the treatment of SVTs in pregnancy do suggest using metoprolol (level of evidence 1B)
rather than verapamil (C), although they recommend avoiding the former in the first trimester.
760. Current guidelines suggest that a resting ECG and TTE (transthoracic ECHO) are the most effective screening strategies
for relatives of patients with HOCM.
761. Cyclo-oxygenase (COX) inhibits platelet aggregation through inhibition on both COX I and II.
762. Decisions regarding anticoagulation depend on patient’s risk of embolic stroke, rather than duration of AF.
763. Different cardiac enzymes have different half-lives and can help identify roughly when an MI happened (hours-days-
weeks)
764. Digoxin follows first order kinetics and has a half life of 1.6 days in a patient with normal renal function.
765. Diphenhydramine can cause prolongation of the QT interval.
766. Diphtheria, coxsackie virus, Chagas disease, and toxoplasmosis are all associated with myocarditis.
767. Drugs causing gynaecomastia include spironolactone, digoxin, and cimetidine.
768. During pregnancy there is usually an increase in heart rate of 10-20 bpm.
769. Early revascularisation for ST elevation MI improves prognosis and outcome.
770. Ebstein’s anomaly is commonly associated with a right bundle branch block pattern on ECG.
771. Ebstein’s anomaly, hypoplastic (atrialised) RV, apical displacement of the septal and posterior tricuspid valve leaflets,
ASD.
29
M Y Elamin
MBBS, DTM&H, MCTM, MRCPI 1& 2
772. Ebstein's anomaly is associated with maternal lithium use if exposed in the first trimester.
773. ECHO and thrombophillia screen shoudl be performed on all young patients with stroke.
774. Eisenmengers syndrome is the process in which a long-standing left-to-right cardiac shunt causes pulmonary
hypertension and eventual reversal of the shunt into a cyanotic right-to-left shunt.
775. Ejection fraction can be calculated using the formula ](end-diastolic volume - end systolic volume) / end-diastolic
volume] x 100
776. Emergency treatment of AF is DC cardioversion independent of cause
777. Endocardial cushion defects, more commonly known as atrioventricular (AV) canal or septal defects, include a range
of defects characterised by involvement of the atrial septum, the ventricular septum, and one or both of the AV valves.
778. Eruptive xanthoma occur in a number of types of hypertriglyceridaemia and also in uncontrolled diabetes mellitus.
779. Erythromycin has been associated with prolonged QT interval and torsades de pointes, and is used in diabetic
gastropathy.
780. Even a moderate rise in ESR can be found in temporal arteritis
781. Evidence suggests angioplasty is superior to thrombolysis in the general population.
782. Excessive alcohol is a recognised cause for atrial fibrillation and is a likely cause if the rhythm has reverted to sinus
after 12 hours.
783. Exposure to lithium in utero is associated with Ebstein's anomaly.
784. Ezetimibe localises at the brush border of the small intestine, where it inhibits the absorption of cholesterol from the
diet.
785. Ezetimibee is the most appropriate additional therapy in cases of statin-induced myalgia, where the low density
lipoprotein (LDL) cholesterol needs to be targeted.
786. Fatigue is a frequent side effect which typically is felt two hours and beyond after taking the drug.
787. Features of mitral stenosis include the loud first heart sound, opening snap and if in sinus rhythm, a pre-systolic
accentuation.
788. Fibrates increases HDL-C by 10-15% and reduces triglycerides by 15-20%.
789. Flecainide is the drug most likely to restore sinus rhythm in atrial fibrillation.
790. Flucloxacillin and gentamicin is the regime of choice for methicillin-sensitive Staphylococcus aureus, with linezolid
an appropriate alternative in MRSA.
791. Following a myocardial infarction (MI), a patient must wait four weeks before he is permitted to drive.
792. GPBB is an isoenzyme of glycogen phosphorylase, which rises within 3 hours of a MI and is therefore a marker for
early myocardial injury.
793. Guidelines suggest the use of nitrates in the first instance, coupled with calcium antagonists, to treat cocaine induced
acute myocardial infarction (MI).
794. HER2 activation is potentially protective against the damage that this stress signaling induces, and HER2 inhibition
removes this layer of protection, leading to dilated cardiomyopathy.

30
M Y Elamin
MBBS, DTM&H, MCTM, MRCPI 1& 2
795. Hereditary haemochromatosis (HHC) is suspected with a serum ferritin greater than 1000 and confirmed by genetic
testing.
796. High doses of parenteral and nebulised salbutamol are associated with hypokalaemia. This effect may have been
potentiated by concomitant treatment with prednisolone
797. Hypercalcaemia, hypermagnesaemia, digoxin, or thyrotoxicosis cause QT shortening.
798. Hypercholesterolaemia with hypertriglyceridaemia is associated with subclinic hypothyroidism.
799. Hypertrophic cardiomyopathy is associated with non-specific ECG changes, and often presents as arrhythmias in
previously well young individuals.
800. Hypertrophic obstructive cardiomyopathy (HOCM) gets louder with a Valsalva.
801. Hypertrophic obstructive cardiomyopathy (HOCM) is typically associated with a jerky pulse although it may present
with entirely normal clinical findings.
802. Hypocalcaemia causes prolonged QT interval due to an increase in ST segment duration.
803. Hypomagnesaemia is a cause of long QT syndrome.
804. Hypomagnesaemia may be due to diuretic use.
805. Hyponatraemia is a useful negative prognostic indicator in heart failure.
806. If hyperkalaemia has resulted in ECG changes, immediate treatment should be with IV calcium gluconate to stabilise
the myocardium and reduce progression to cardiac arrest.
807. If VT is proven patients should be on amiodarone and the indication for an automated implantable
cardioverter/defibrillator strongly considered if the overall LV function is reduced.
808. In acutely haemodynamically compromised trauma patients, a blood transfusion is the most appropriate fluid.
809. In iatrogenic long QT, it is safer to stop the offending drug rather than add further drugs
810. In mitral stenosis the opening snap is characteristically lost with heavy valvular calcification.
811. In patients admitted for chest pain, full bloods should be done on admission to hospital, including a baseline troponin
(I or T). This troponin is not used immediately for interpretation of the cause of chest pain, but can be used to show a
rise in patients who have an elevated baseline troponin (for example those with renal impairment).
812. In patients with systolic murmur, the severity of valvular calcification is prognostically important in an asymptomatic
patient, but the most important predictor is LV function.
813. Indications for carotid artery surgery versus anti-platelet therapy alone following TIA/amarousis fugax
814. Indications for carotid endarterectomy
815. Indications for surgery in aortic stenosis include a gradient of 50 mmHg or more, or associated symptoms such as
syncope, breathlessness, and episodes of pulmonary oedema.
816. Infarction of the LAD would cause necrosis of the left ventricle.
817. Infective endocarditis (IE) is a rare condition, it arises following bacteraemia in any patient, but those with a
predisposing cardiac lesion are at an increased risk.
818. Inferior myocardial infarction is usually due to right coronary artery occlusion
819. Intraosseous access is still perfectly viable in the adult patient
31
M Y Elamin
MBBS, DTM&H, MCTM, MRCPI 1& 2
820. Ischaemia, typically in hypotensive hospitalised patients, is the most frequent antecedent to acute tubular necrosis.
Blood pressure should be maintained in cardiogenic shock with fluids and/or inotropic agents.
821. It is important to avoid beta blockers in treating cocaine induced chest pains or acute myocardial infarctions, as this
may result in unopposed 1 adrenergic action with worsening coronary spasm.
822. IV beta-blockade is the most approprtiate initial therapy for patients with suspected aortic dissection.
823. JVP in constrictive pericarditis
824. Know the criteria for rheumatic fever
825. Lesions limited to the descending aorta (type B) generally have better survival compared with those involving the
ascending aorta.
826. Liddle's syndrome is typically associated with hypokalaemic hypertension and low renin and aldosterone
concentrations - the so-called pseudo-hyperaldosteronism.
827. Lipodystrophy, lipoatrophy, and alterations in serum lipid values have been observed in patients with human
immunodeficiency virus (HIV) disease who are taking highly active antiretroviral therapy.
828. Low aortic compliance results in an increased pulse pressure.
829. Low body weight is associated with increased bleeding risk.
830. Malignant hypertension is associated with papilloedema, convulsions, and pulmonary oedema and IV nitroprusside is
the treatment of choice.
831. Management of chronic hypertension in a patient of African heritage.
832. Managing chronic hypertension in pregnancy.
833. Marfan's syndrome is a connective tissue disorder that is associated with floppy mitral valve and also with cystic medial
necrosis that predisposes to aortic dissection.
834. Marfan's syndrome is associated with dialtation of the aortic sinuses, causing a dilated aortic root. This can lead to
aortic root rupture or dissection.
835. Massive PEs can lead to right ventricular strain, which can lead to a troponin rise.
836. Mediastinal irradiation is a common cause of constrictive pericarditis.
837. Medical management is preferred for Type B aortic dissections, and treatment is usually with IV labetalol to achieve a
systolic BP 100-120 mmHg, with pulse less than 80bpm.
838. Methylprednisolone and cyclophosphamide is the treatment of choice for induction of remission in Granulomatosis
with polyangiitis
839. Mitral valve calcification is an incidental finding at echocardiography (ECHO) which leaves a calcified bicuspid aortic
valve leading to aortic stenosis as the likely explanation.
840. Mitral valve stenosis leads to left atrial enlargement.
841. Multiple trials have demonstrated reductions in overall mortality, not just cardiovascular mortality, following treatment
with statins.
842. Myocarditis can have features similar to cardiomyopathy and one of the most likely organisms is Coxsackie B virus.
843. Myoglobin rises within 2 hours of cardiac myocyte damage, and is the most sensitive early marker of MI.
32
M Y Elamin
MBBS, DTM&H, MCTM, MRCPI 1& 2
844. Myxomatous degeneration of the mitral valve is by far the most common cause of MR in the United Kingdom.
845. Negative Predictive Value = (True negative) / (true and false negatives)
846. NICE advise against using beta-blockers as routine 'first line' therapy for uncomplicated hypertension.
847. NICE guidance on Prophylaxis against infective endocarditis (CG64) suggests that no prophylaxis is required for
patients with biscupid aortic valve.
848. NICE guidance on Prophylaxis against infective endocarditis (CG64) suggests that prophylaxis is not required during
dental procedures.
849. NICE guidelines advocate use of ACEi or ARB in diabetics with hypertension.
850. NICE recommends 20-30 minutes of physical activity per day post MI.
851. Nicotinic acid can be used by specialists in combination with a statin, where a statin alone has failed to adequate control
dyslipidaemia.
852. Non-atherosclerotic angina would be associated with conditions such as aortic regurgitation.
853. Non-atherosclerotic angina would be associated with conditions such as aortic regurgitation.
854. Non-bacterial thrombotic endocarditis (marantic endocarditis) is due to platelet-fibrin thrombi that are prone to
embolising.
855. Non-classical risk factors for hypertension.
856. Occlusion of the left anterior descending coronary artery (LAD) represents an anterior MI seen in V1-5.
857. Omacor increases peroxisomomal beta-oxidation of fatty acids in the liver.
858. One should consider a diagnosis of rheumatic fever in a patient who has mitral stenosis.
859. Optimal management of NYHA class III-IV CCF is with an ACE-inhibitor, beta-blocker and spironolactone.
860. Ototoxicity is associated with vancomycin, and is more likely in patients with high plasma concentrations, renal
impairment or pre-existing hearing loss.
861. Palmar crease xanthoma are associated with remnant hyperlipidaemia (type III hyperlipidaemia).
862. Partial AV canal defects or ostium primum ASDs are seen in Down's syndrome.
863. Patients who receive clarithromycin (or other macrolides) whilst on warfarin should have their INR closely monitored,
as the dose may need to be reduced to avoid an unsafe INR.
864. Patients who undergo long-term haemodialysis suffer from increasing arterial calcification, which is associated with
both increased risk of myocardial infarction and stroke, but the greatest absolute increase is in MI rates.
865. Patients who undergoe successful cardioversion for idiopathic AF need to remain on warfarin as there is risk of further
thromboembolism
866. Patients with HCM are at increased risk of sudden cardiac death (SCD) due to ventricular fibrillation/tachycardia
(VF/VT). Implantable cardio-defibrillators (ICD) are superior to amiodarone or beta-blockers for preventing this.
867. Patients with hypertrophic cardiomyopathy (HCM) are at increased risk of sudden cardiac death due to ventricular
fibrillation/tachycardia (VF/VT).
868. Peripartum cardiomyopathy presents within a few weeks either side of delivery with symptoms of biventricular heart
failure, and is managed with vasodilators, diuretics and beta blockade as required.
33
M Y Elamin
MBBS, DTM&H, MCTM, MRCPI 1& 2
869. PFO can result in paradoxical emboli, which is a cause of cerebral infarcts in the young.
870. Phenoxybenzamine should be intiated first to treat phaeochromocytoma.
871. PiCCO gives indications of cardiac output, extravascular lung water, intravascular filling and only requires a central
line and a PiCCO femoral arterial line and as such is relatively simple to use.
872. Pioglitazone can cause noticeable fluid retention and is contraindicated in patients with cardiac failure
873. Pioglitazone can result in fluid retention of unknown aetiology which may cause a mild dilutional anaemia
(haemoglobin typically falls by 10 to 20 g/L) and ankle oedema. It is contraindicated in congestive heart failure
874. Post MI ventricular tachycardia (VT) is most commonly due to scar tissue.
875. Prediction of sudden cardiac death risk in hypertrophic obstructive cardiomyopathy (HOCM) is notoriously difficult,
but septal wall thickness greater than 3 cm is associated with a significantly increased risk.
876. Presence of prominent calcification in the pelvic arteries in older persons is a benign finding that does not require
treatment
877. Presence of spherocytes or fragmented red blood cells on the film is suggestive of haemolytic anaemia.
878. Previous history of hemorrhagic stroke is a contraindication for thrombosis
879. Prophylaxis not now required for simple instrumentation in common cardiac abnormalities
880. Prosthetic valve endocarcitis occuring within the first year after surgery affects 0.7-3% of cases and is often due to
Staphylococci.
881. Prosthetic valve endocarditis arising within two months of valve surgery is generally the result of intraoperative
contamination of the prosthesis or a bacteraemia postoperative complication.
882. Pulsus alternans is a physical finding characterised by a regular alternation of the force of the arterial pulse. It almost
invariably indicates the presence of severe left ventricular systolic dysfunction.
883. Radiation of pain to the jaw is relatively specific for pain of myocardial ischaemia.
884. Rate control is best achieved with direct current cardioversion (DCCV).
885. Remnant hyperlipidaemia is associated with genotype apo E-2/E-2 and occurs with a frequency of 1:100.
886. Renal clearance is the main factor that determines the choice of loading dose of digoxin.
887. Repeat renal function is advised following any increase in ACE inhibition.
888. Right bundle branch block in acute anterior myocardial infarction suggests obstruction prior to the first septal branch
of the left anterior descending coronary artery
889. Right coronary artery (RCA) occlusion may cause posterior infarction.
890. Right heart catheterisation and oximetry would be most useful in establishing a diagnosis of acute left ventricular
failure (LVF).
891. Second degree heart block with RBBB implies a significantly increased risk of complete heart block.
892. Selenium deficiency is one of the reversible causes of dilated cardiomyopathy.
893. Sodium bicarbonate is recommended as first line therapy for VT/QRS widening in tricyclic poisoning.
894. Somatisation disorder is characterised by multiple recurring pains and gastrointestinal, sexual, and pseudo-neurologic
symptoms that occur over a period of years.
34
M Y Elamin
MBBS, DTM&H, MCTM, MRCPI 1& 2
895. Sotalol is a cause of drug-induced long QT.
896. Spironolactone (aldosterone receptor antagonist) is the optimal add on medication for advanced heart failure symptoms
to prevent cardiac muscle remoddeling.
897. ST elevation of 2 mm in V4-V6 would be an indication fro thrombolysis within 12 hours, where PCI is not available
or indicated
898. ST elevation without reciprocal changes in context of cardiac syncope suggests possible LV aneurysm from previous
MI.
899. Staphylococci are skin organisms most commonly introduced during pacemaker insertion.
900. Starling's law states that when the myocardium stretches due to blood pooling, the force of contraction increases, to
preserve the stroke volume and thus the cardiac output.
901. statin myositis management
902. Statin-associated myopathy may be exacerbated by the co-prescription of other drugs such as calcium channel blockers,
macrolide antibiotics, fibrates, amiodarone and grapefruit juice.
903. Statins act by competitively inhibiting HMG-CoA reductase, the first committed enzyme of the HMG-CoA reductase
pathway.
904. Streptococcus bovis is a normal commensal of the gastrointestinal (GI) tract. However, S. bovis bacteraemia and
endocarditis have a strong association with GI malignancy.
905. Structural heart disease reduces the chance of successful cardioversion from atrial fibrillation to sinus rhythm
906. Studies such as CIBIS, MERIT HF, and COPERNICUS clearly demonstrate the advantage of beta blockers even with
severe heart failure.
907. Studies such as CIBIS-II and MERIT-HF reveal that beta blockers significantly reduce morbidity and mortality in heart
failure.
908. Subacute bacterial endocarditis (Streptococcus viridans) has a better prognosis in cases of infective endocarditis.
909. Subcutaneous insulin offers tighter glycaemic control.
910. Tachyarrhythmias should be managed according to the ALS algorithm. Valsalva manouvres are first line, except in an
unstable patient, where DC cardio version is indicated.
911. The "c" wave of the jugular venous waveform is associated with the closure of the tricuspid valve.
912. The aortic valve is tricuspid
913. The atrial fibrillation suggested by an irregularly irregular pulse, is an indicator of diastolic dysfunction and poor
ventricular filling.
914. The Austin Flint murmur is a low frequency mid/late diastolic murmur which may show pre-systolic accentuation and
is virtually indistinguishable from that of mitral stenosis.
915. The best drugs for Autosomal dominant APKD-1 are ACE inhibitors (e.g. captopril, enalapril, or lisinopril) or
angiotensin II receptor antagonist blockers (e.g. telmisartan, losartan, irbesartan, or candesartan).
916. The best management strategy for patients with obesity-related hypertension is lifestyle advice.
917. The cardiomyopathy of alcoholism is a dilated or congestive form.
35
M Y Elamin
MBBS, DTM&H, MCTM, MRCPI 1& 2
918. The commonest cause of a 'paradoxical embolus' is a patent foramen ovale. Although atrial septal defects are also a
cause, they are more rare
919. The concept of coronary dominance refers to which coronary artery supplies the posterior descending coronary artery.
In the case of approximately 85% of patients this is the right coronary artery with about 15% of patients having a
dominant left circumflex.
920. The DIGAMI study has demonstrated that there is a survival advantage in initially treating such patients with elevated
glucose concentrations with sliding scale insulin for 24 hours post-infarct, even if they are not known to be diabetic
921. The double apical impulse, mid systolic murmur and ECG appearance is typical of that seen with hypertrophic
obstructive cardiomyopathy (HOCM).
922. The ECG suggests cardiotoxicity related to hyperkalaemia and the history of palpitations is suggestive of arrhythmias.
Therefore cardio protection with calcium chloride or gluconate should be first priority and lowering potassium levels
immediately thereafter
923. The effect of fibrates on the metabolism of triglyceride-rich lipoproteins is due to a PPAR-alpha-dependent stimulation
of lipoprotein lipase and of apolipoprotein (apo)A-V and to an inhibition of apoC-III expression.
924. The ejection fraction is the amount of blood pumped (stroke volume = end diastolic volume - end systolic volume)
divided by the amount of blood the ventricle contains (end diastolic volume).
925. The electrocardiogram (ECG) appearances of a delta wave occur in approximately 1.5 per 1000 of the population, but
many individuals never experience paroxysmal tachycardias.
926. The evolution of Q waves is the most suggestive of an infarct. A Q wave is any negative deflection that precedes an R
wave on the ECG. Small Q waves are normal in most leads, and they can be prominent in leads III and aVR as a normal
variant, but should not be seen in leads V1-V3.
927. The immediate management of hyperkalaemia in patients with serum potassium >6.5 mmol/L and ECG changes or
serum potassium >7.5 mmol/L is always intravenous calcium.
928. The JBS 2005 guidelines list asthma or heart block as 'compelling contraindications' to ß blockers as life-threatening
complications may occur.
929. The KCNE1 and KCNQ1 genes provide instructions for making proteins that work together to form a channel across
cell membranes.
930. The latest guidelines now suggest 30:2 as the correct ratio of chest compressions to non-intubated ventilations
931. The majority of patients with a bicuspid aortic valve will ultimately require valve replacement.
932. The majority of patients with a right ventricular infarction improve after a fluid challenge but this should not be
continued if blood pressure fails to improve, to reduce the risk of overload.
933. The most appropriate agent in CCF with evidence to support its use for reducing morbidity and mortality in failure
would be the addition of a beta blocker
934. The most appropriate initial therapy for a patient who has a high risk of thrombo-embolic stroke is anticoagulation with
warfarin maintaining an international normalised ratio (INR) between 2-3.

36
M Y Elamin
MBBS, DTM&H, MCTM, MRCPI 1& 2
935. The most appropriate strategy for secondary prevention of a heart attack would involve further blood pressure reduction
with an angiotensin converting enzyme inhibitor (ACEi), which would not only reduce cardiovascular (CV) risk as
suggested by the HOPE study, but would also reduce microvascular risk as revealed by UKPDS.
936. The most likely explanation in a patient with a prior inferior myocardial infarct is mitral valve prolapse due to papillary
muscle rupture.
937. The optimal antihypertensive therapy for black Afro-Caribbean patients (if a CCB is not suitable, for example because
of oedema) will be a thiazide-like diuretic according to current NICE guidance, 2011.
938. The patient's symptomatology is the most important determinant in terms of the decision to operate on stenotic aortic
valves.
939. The perimembranous septum is the most common site for VSDs
940. The presence of a small pericardial effusion on echo is quite common if otherwise well, no further action is required.
941. The pulmonary circulation is a low pressure, high flow, high compliance system.
942. The right coronary artery gives off branches to the sinus and AV node, therefore disease within this vessel can result
in bradyarrhythmias.
943. The subscapular artery arises from the axillary and is its largest branch, eventually anastomosing with the lateral
thoracic and intercostal arteries.
944. The sympathetic and parasympathetic nervous systems play different roles which allow effective regulation and
compensation of the cardiovascular system.
945. The symptoms of hyperthyroidism are usually a result of increased beta-adrenergic tone, and propranolol is therefore
an effective treatment.
946. The thicker the muscle the more abnormal the cardiac architecture and the higher the risk of arrhythmia and sudden
death.
947. There is a reduced tachycardic response during exercise associated with age. The heart has to compensate by increasing
stroke volume and failure to do so will reduce aerobic capacity.
948. There is an increased risk of strokes in patients with atrial fibrillation and hence with the given symptoms formal
anticoagulation with warfarin should be considered.
949. There is no evidence that increasing a dose of digoxin above 62.5 µg in a patient in sinus rhythm would have any added
benefit in CCF.
950. There is no evidence to support a beneficial effect of nifedipine post- MI in terms of long term morbidity or mortality.
951. Thioridazine, an antipsychotic, and many other drugs can prolong the QT interval and increase the risk of torsade de
pointes VT.
952. Three drugs to remember that are associated with gingival hyperplasia are Phenytoin Ciclosporin Nifedipine.
953. Thrombolytic therapy is indicated for pulmonary embolism when circulation is severely compromised, and is as
effective delivered peripherally.
954. Tilt table test is a useful test to support the diagnosis of vasovagal syncope.
955. Total anomalous pulmonary venous connection (TAPVC) is associated with cyanosis in the newborn.
37
M Y Elamin
MBBS, DTM&H, MCTM, MRCPI 1& 2
956. Transvenous pacing takes longer to instigate than transcutaneous pacing.
957. Treatment with a dihydropyridine short acting calcium antagonist nifedipine in moderate to high doses is associated
with increased cardiovascular mortality in patients with acute myorcardial infarction.
958. Troponin is a component of thin filaments (along with actin and tropomyosin), and is the protein to which calcium
binds to accomplish this regulation.
959. Turner syndrome is associated with essential hypertension (10%).
960. Turners syndrome is a cause of coarctation of the aorta.
961. Type 2 diabetes results in small, dense LDL which is more atherogenic, being able to be oxidised more readily and
penetrate endothelium and macrophages.
962. Type III hyperlipidaemia is characterised by elevated serum cholesterol and triglyceride levels and early onset
atherosclerotic disease.
963. Typically, a small VSD generates a pansystolic murmur at the LSE accompanied by a thrill. The murmur may be heard
at the apex but is usually loudest at the LSE.
964. U waves are characteristic features of hypokalaemia.
965. Under the New York Heart Association classifications: "Class II. Patients with cardiac disease resulting in slight
limitation of physical activity. They are comfortable at rest. Ordinary physical activity results in fatigue, palpitation,
dyspnea or anginal pain."
966. Up to half of patients presenting with Streptococcus bovis endocarditis have colorectal tumours.
967. Uraemia leads to exudation of fibrin onto the epicardial and pericardial surfaces.
968. Vasovagal syncope occurs when standing, with very minimal abnormal movements and a rapid recovery.
969. Viagra is contraindicated in patients taking nitrates
970. VT associated with exercise is normally non-sustained and has no significant impact on mortality.
971. Warfarin anticoagulation titrated to an INR of 2.0-3.0 is recommended for the average patient with a CHA2DS2-VASc
score of 2 unless contraindicated
972. When potassium falls below 3 mmol/l, the ECG often demonstrates ST depression.
973. When the risk level for sudden cardiac death (SCD) is judged by contemporary criteria to be unacceptably high and
deserving of intervention, the ICD is the most effective and reliable treatment option available
974. When there are no abnormal physical signs in a paitent with repeated episodes of breathlessness and palpitations lasting
a short period of time and resolving gradually, the likley diagnosis is panic attacks
975. When using medicines to treat hypertension in pregnancy, aim for a target blood pressure of 135/85 mmHg.
976. Wolff-Parkinson-White (WPW) syndrome can be associated with negative delta waves in II, III and aVF.
977. Wolff-Parkinson-White syndrome is suggested by a delta wave on ECG.
978. 80% of digital ischaemias have an emboli originating from the heart and require urgent echocardiogram.
979. A catheter data showing step up between RA and RV is indicative of a VSD
980. A 'ferning' or 'arborescent' rash is pathognomonic of a lightning strike, also known as Lichtenberg figures.
981. A negative D-dimer rules out a PE but a positive D-Dimer is not diagnostic of one.
38
M Y Elamin
MBBS, DTM&H, MCTM, MRCPI 1& 2
982. A patent ductus arteriosus will lead to differences in oxygen saturation in the ascending and descending aorta.
983. A precordial thump can be successful if given within seconds of the onset of a shockable rhythm.
984. A profoundly hypothermic patient who fails to respond to three shocks should not receive any further shocks or drugs
until they are rewarmed.
985. A VSD gives a higher oxygen saturation in the RV compared with the RA.
986. ACE- inhibitors have benefits in reducing mortality and morbidity in heart failure by preventing myocardial
remoddeling.
987. ACE- inhibitors improve both symptoms and prognosis in heart failure by diureses and prevention of cardiac
remodeling.
988. ACE inhibitors prevent breakdown of bradykinin which causes angioedema.
989. Adenosine is contraindicated in asthma and verapamil should be used in SVT for these patients.
990. Adenosine is contraindicated in patients with asthma and other drugs such as amiodarone should be tried.
991. Adenosine is safe in pregnancy, but other agents are potentially contraindicated.
992. Administering folic acid without vitamin B12 in folate deficiency would be hazardous and could precipitate subacute
combined degeneration of the spinal cord.
993. AF, new murmur, syncope and a raised ESR should raise the possibility of a cardiac tumour.
994. All people who have had an MI should normally be discharged from hospital with the following drug treatments: ACE
inhibitor, beta-blocker, statin and Antiplatelet therapy
995. Although less commonly placed in the 21st century, pulmonary artery catheters may be beneficial in cases of right
ventricular dysfunction to assess filling.
996. Amiodarone can prolong QT interval
997. Amiodarone-induced thyrotoxicosis may be due to thyroiditis and can be treated with steroids or withdrawal of
amiodarone
998. An angiotension-converting enzyme inhibitor and calcium channel blocker such as amlodipine are first line agents in
patients with hypertension without a history of ischaemic heart disease.
999. An ASD is likely to present with right bundle branch block and left axis deviation on ECG
1000. An ECHO in Marfan's is a good prognostic indicator of risk of aortic dissection in relation to the aortic root.
1001. An overestimation of the severity of aortic stenosis can occur due to large volumes of blood passing over the valve at
high velocities, which occurs in aortic regurgitation.
1002. Antibiotic prophylaxis against infective endocarditis is not recommended for people undergoing dental procedures or
for people undergoing non-dental procedures at upper and lower gastrointestinal tract, genitourinary tract or upper and
lower respiratory tract.
1003. Aortic aneurysmal rupture in elderly is most likely due to atherosclerosis.
1004. Aortic regurgitation presents with dyspnoea and an EDM
1005. Aortic stenosis is a more likely cause of a higher aortic valve gradient in the elderly than HOCM.

39
M Y Elamin
MBBS, DTM&H, MCTM, MRCPI 1& 2
1006. Aortic valve endocarditis can be associated with aortic root abscess which can be identified by prolongation of the PR
interval on a 12 lead ECG.
1007. ASD will lead cause a higher level of oxygenated blood in the right atrium and ventricle
1008. Atrial septal defect with a deep vein thrombosis (DVT) can lead to a paradoxical embolus.
1009. Atropine is the first step in the management of bradycardia secondary to beta blocker or calcium channel blocker
overdose
1010. AV node ablation is reserved for those patients where pharmacological rate control is unsuccessful or not tolerated.
The procedure is invasive and requires permanent pacemaker implantation.
1011. Beta blockade is the first choice intervention for rate control in AF according to current NICE guidelines.
1012. Beta blocker and ACE inhibitors are used for treatment of heart failure
1013. Beta blockers should be avoided in cocaine associated myocardial ischaemia or infarction as they can potentiate
coronary vasoconstriction.
1014. Bicarbonate is indicated in tricyclic overdose
1015. BP control shows greater reduction in CV risk than tight glyceamic control in UKPDS
1016. Bradycardia induce syncope requires inpatient telemtry.
1017. Calcium antagonists and nitrates do not prevent cardiovascular events occurring.
1018. Carcinoid heart disease has a poor prognosis
1019. Cardiac biomarkers and infarction, diagnosis of infarction.
1020. Cardiac biomarkers, diagnosis of myocardial infarction.
1021. Cardiac physiology and hormones, pathophysiology of ischaemic heart disease.
1022. Cardiac resynchronisation therapy is recommended for use in patients with NYHA class 3 symptoms and above.
1023. Cardiac tamponade should be considered following blunt force trauma to the chest and requires urgent ultrasound and
paracentesis.
1024. Cardiology investigations for valvular disease, ECG, echo values and limitations
1025. Cardiovascular risk assessment, statin therapy, primary prevention of IHD.
1026. CARDS showed a significant reduction in all-cause mortality, stroke and coronary events in T2 diabetics on
atrovastatin
1027. CHADS2-VASc scoring
1028. Clinical cardiovascular examination.
1029. Common intra-operative management goals in mitral regurgitation include avoiding raised SVR, avoiding bradycardia,
and avoiding factors exacerbating pulmonary hypertension.
1030. Complete destruction of the aortic valve due to endocarditis is usually caused by Staph Aureus.
1031. Complete heart block is a medical emergency that requires immediate admission. Remember to look for p waves that
have no relationship to QRS.
1032. condition. Vasodilators, nitrates and cardioselective beta blockers have all shown evidence of benefitIn peripartum
cardiomyopathy.
40
M Y Elamin
MBBS, DTM&H, MCTM, MRCPI 1& 2
1033. Congenital heart disease, dissection, cardiac manifestations of medical disorders.
1034. "Conservative management of Aortic dissection is with IV metoprolol to lower blood pressure.
1035. "
1036. Consider CRT in patients with reduced EF and need a PPM with a high percentage of RV pacing.
1037. Continuous positive airway pressure (CPAP) is commonly used in heart failure When there is pulmonary oedema.
1038. Current UK resuscitation guidelines emphasise the importance of minimising breaks in chest compressions in a cardiac
arrest situation. Current guidelines recommend single shock strategy
1039. Daytime average ABPM or average HBPM blood pressure during the usual waking hours of people aged over 80 is
lower than 145/85 mmHg.
1040. Dialysis should be considered in patients with acute kidney injury, particularly when fluid overloaded and
hyperkalaemic
1041. Dilated cardiac myopathy secondary to alcohol is potentially reversible.
1042. Diuretic therapy in heart failure helps in symptom management, but does not improve prognosis
1043. Dual antiplatelet therapy should be continued following the insertion of a drug eluting cardiac stent insertion for a year
unless the surgery is urgent.
1044. ECG changes in pacemaker syndrome show AV dyssynchronisation with small P waves.
1045. Echocardiography can be used in the periarrest diagnose (or exclude) pericardial tamponade.
1046. Endotracheal tube is no longer recommended as a route of administration for drugs
1047. Eruptive xanthomata occur in hyperlipidaemic states associated with hypertriglyceridaemia.
1048. Exacerbation of angina attack during an exercise test should lead to the test stopping and an angiogram being ordered.
1049. Fabry's disease is a rare X linked condition which results in accumulation of sphingolipids in organs of the body.
1050. Fallot's tetralogy consists of ventricular septal defect ,pulmonary stenosis, right ventricular hypertrophy and an over-
riding aorta.
1051. Fallot's tetralogy consists of VSD, PS, RVH and an over-riding aorta.
1052. Features of myotonic dystrophy include frontal baldness in men, atrophy of temporalis, masseters, and facial muscle,
and bilateral ptosis.
1053. Fenofibrate is used to treat isolated hypertriglyceridaemia, though it should not be prescribed routinely for primary or
secondary prevention of hypercholesterolaemia.
1054. First degree heart block (PR interval >5 small squares) can be seen in normal fit adults and often requires no treatment
1055. First line anti-hypertensives in under 55 year-olds who are Caucasian are ACE-i.
1056. Flecainide is now considered an acceptable anti-arrhythmic in patients with WPW who do not have underlying
structural heart disease.
1057. Flecainide is the treatment of choice for paroxysmal AF with aberrant conduction due to accessory pathway (Wolff-
Parkinson- White syndrome)
1058. Fleccanide is contraindicated in fast AF when there is structural heart disease.
1059. Fleccanide is the best oral agent for chemical cardioversion but is contraindicated in ischaemic heart disease.
41
M Y Elamin
MBBS, DTM&H, MCTM, MRCPI 1& 2
1060. For diagnosing myocardial infarction in anaesthetised patients, serial troponin assay is a critical biochemical test.
1061. Gestational hypertension with moderate hypertension is managed first line with oral labetalol.
1062. Giving oral folic acid without vitamin B12 would be hazardous and could precipitate subacute combined degeneration
of the spinal cord in pernicious anaemia.
1063. Guidance from the European Society of Cardiology states that, after a silent cerebral embolism or TIA, surgery is
recommended without delay if an indication remains.
1064. Guidance on use of ICD has changed, and where there is QRS prolongation in class III heart failure, an ICD is now
potentially indicated.
1065. Hamman's sign is a clicking or crunching sound in time with the heart beat audible on the left chest, in the context of
pneumothorax or pneumomediastinum.
1066. High output cardiac failure can be a complication of Pagets disease
1067. Hyperkalaemia treatment is initiated when over 6.5 or 6 with ECG changes.
1068. IABPC is crucial in achieving haemodynamic stabilisation in ventricular septal rupture.
1069. If blood pressure remains uncontrolled with optimal or maximum tolerated doses of four drugs, seek expert advice if
it has not yet been obtained.
1070. If ECG leads are place incorrectly it can lead to abnormal traces on the ECG print out.
1071. If INR is >8.0 and there is no bleeding, stop warfarin, give 0.5-1mg IV vitamin K and restart warfarin once INR is <
5.0 in accordance with BJH guidelines.
1072. Immediately after defibrillation, chest compressions should be restarted and pulse and rhythm reassessed after two
minutes
1073. Immediately assess eligibility (irrespective of age, ethnicity or sex) for coronary reperfusion therapy (either primary
Percutaneous coronary intervention or fibrinolysis) in people with acute STEMI.
1074. Immediately assess eligibility (irrespective of age, ethnicity or sex) for coronary reperfusion therapy in people with
acute ST elevation MI. Preferred strategy is primary percutaneous coronary intervention.
1075. In cardiac catheterisation the oxygen saturation in the RA and SVC should be the same.
1076. In complete heart block P waves show no relation to the QRS complexes.
1077. In questions with heart pressure tables is important not to miss another non-cardiac diagnosis.
1078. In stent re-stenosis the likely underlying pathophysiological cause is neointimal proliferation.
1079. Independent atrial activity is seen with VT but not with an SVT
1080. Indications for surgery in aortic stenosis include a gradient of 50 mmHg or more or severe symptoms.
1081. Indications for surgery in native infective endocarditis include infection caused by fungi or multiresistant organisms.
1082. Individualising treatment of an elderly patient in an attempt to minimise unwanted side-effects that may result in a
reduction in quality of life
1083. Insulin is the safest agent to address hyperglycaemia in renal insufficiency.
1084. Insulinoma can present with collapse/sweating relieved by carbohydrate

42
M Y Elamin
MBBS, DTM&H, MCTM, MRCPI 1& 2
1085. Intra-aortic balloon pump counter-pulsation may be an effective bridging therapy in cardiogenic shock, particularly in
the setting of acute myocardial infarction.
1086. Ischaemic cardiomyopathy should be treated with an implantable ICD and anti-arrythmatic medication long term.
1087. It is recommended to maintain glucose <10 mmol/L post successful resuscitation.
1088. IVDU will have had years of damage to there arterial system following intravenous drug abuse could result in
atherosclerosis and high susceptibility to thrombus formation.
1089. Kawasaki disease is a childhood febrile illness, resulting in arterial aneurysms, which may present with cardiac
symptoms later in life.
1090. Knowledge of broad complex tachycardia algorithm (Advanced Life Support)
1091. Knowledge of coding systems used for pacemaker functions and that asystole with no ventricular escape rhythm
requires dual chamber pacing
1092. Labetalol is first line for moderate hypertension in pregnancy
1093. Left ventricular failure is common post MI and should be treated with diuretics and ACE-inhibitors.
1094. Left ventricular pressures are high with a steep drop-off between the LV and aortic systolic pressures are suggestive of
hypertrophic cardiomyopathy.
1095. Lipodystrophy, lipoatrophy and alterations in serum lipid values have been observed in patients with human
immunodeficiency virus (HIV) disease taking highly active antiretroviral therapy. Elevated serum lipid levels have
been associated with premature coronary artery disease.
1096. Lipoprotein lipase deficiency will produce hyperchylomicronaemia/hypertriglycerideamia.
1097. Low TSH with normal thyroid hormone levels is subclinical hyperthyroidism and is unlikely to cause LVF
1098. Magnesium sulphate IV is recommend for the treatment of refractory VF, if there is anything to suggest the patient
may be hypomagnasaemic
1099. Manage significant haemodynamic compromise in presumed PE with fluid resuscitation prior to further investigation
1100. Management and investigation of heart failure.
1101. Management of tachyarrhythmias in patients with pacemakers
1102. Mangement of arrhythmias
1103. Many drugs have been associated with gynecomastia, including phytoestrogens, oestrogens and drugs with oestrogen-
like properties, inhibitors of testosterone synthesis or action, and other agents with unknown mechanisms.
1104. MArfan's disease is a risk factor for spontaneous aortic dissection.
1105. Marfan's syndrome is a connective tissue disease inherited in an autosomal dominant manner.
1106. Metformin is contraindicated immediately following MI due to tissue hypoxia which is a risk factor for the development
of lactic acidosis.
1107. Mitral regurgitation can occur post-MI and new murmurs post-infarct should be investigated by ECHO.
1108. Mobitz type II heart block post MI is an indication for transvenous cardiac pacing.
1109. Modern NICE guidance recommends much earlier referral for ablation than was previously planned.
1110. Multiple myeloma may present with roleaux formation on blood film and raised total protein (globulin component).
43
M Y Elamin
MBBS, DTM&H, MCTM, MRCPI 1& 2
1111. Myocardial rupture is a complication following pacemaker insertion characterised by chest pain, dyspnoea and
hypotension.
1112. NICE guidelines for ICD/CRT pacemaker implantation
1113. NICE guidelines suggest ambulatory blood pressure monitoring (ABPM) or home blood pressure monitoring (HBPM)
before commencing therapy unless patient has severe hypertension.
1114. Ninety five per cent of patients presenting with hypertension have essential hypertension.
1115. Note RV systolic pressures exceeding LV systolic pressures is very suggestive of Eisenmenger's syndrome in context
of a VSD.
1116. Obstructive cardiomyopathy, clinical examination, pulses.
1117. Oesophageal rupture presents with sever pain radiating to the back following eating with equal blood pressures and
normal pulses.
1118. Old age, presence of prosthetic valve endocarditis, insulin dependent diabetes mellitus and severe co-morbidities are
all poor prognostic factors in infective endocarditis.
1119. Oral prednisolone should be considered in the management of acute gout where NSAIDs are contra-indicated
1120. Pacemaker complications can be a common problem in the period following insertion and can be divided into early
complications (<6 weeks) or late (>6 weeks).
1121. Pacemaker syndrome occurs in people fitted with a pacemaker who have inadequate AV synchronisation leading to
disalignment between atrial and ventricular beats.
1122. Pacemakers are classified by the nature of their pacing mode and the chambers paced using a code of up to five letters.
1123. Pacemakers are classified by the nature of their pacing mode using a code of up to five letters.
1124. Patent ductus arteriosus will show an unexpected step-up in oxygen saturation between the RV and PA.
1125. pathogens in endocarditis
1126. Patients should be risk stratified in cases of chest pain with a normal ECG to asses the need for imaging and treatment
following this.
1127. Patients under the age of 40 with less than 20% cardiovascular risk at 10 years should be initially offered lifestyle
interventions to deal with hypertension.
1128. Patients undergoing endarterectomy should have their ischaemic heart disease addressed before hand.
1129. Patients who are asymptomatic should be considered for CRT because it can reverse LV impairment.
1130. Patients with a CHADS2VASC score of 0 who receive no antithrombotic therapy are at low risk of stroke (rate of 0.5
to 1.7 percent per year), and a benefit from aspirin has not been conclusively shown in these patients.
1131. Patients with a CHADSVASC score of 2 or more should be anticoagulated with warfarin or a DOAC.
1132. Patients with heart failure or iron deficiency have been shown to show benefit from IV iron replacement.
1133. Patients with large myocardial infarctions can develop profuse acute heart failure and the mainstay treatment is
revascularisation by PCI.
1134. Patients with short runs of premature ventricular ectopic with no syncope only require reassurance.

44
M Y Elamin
MBBS, DTM&H, MCTM, MRCPI 1& 2
1135. Patients with suspected cardiac aneurysms who are haemodynamically stable require CCU admission and observation
with ECHO.
1136. Patients with taccyarrythmias that are haemodynamically compromised or in heart failure require DC cardioversion.
1137. Persistent ST elevation after acute MI may be the result of a ventricular aneurysm
1138. Persistent ST elevation after an MI suggests the possibilty of aneurysm formation.
1139. Persistent symptomatic AF resistant to beta blockers in a young patient with normal ECHO requires ablation.
1140. Posteromedial papillary muscle is twice as likely to rupture as the anerolateral papillary muscle
1141. Post-MI angiography is indicated with ECG positive but symptom negative exercise tests.
1142. Post-MI repurfusion arythmies can occur and observation under close ECG monitoring is the best immediate
management if the patient is haemodynamically stable.
1143. Presentation and management of emergencies – cardiac tamponade.
1144. Prolonged QT interval can lead to torsade de pointes.
1145. Pulmonary regurgitation is a systolic and diastolic murmur heard loudest at the upper left sternal edge common
following tetralogy of Fallot repair.
1146. QT prolongation can lead to torsade de pointes and this should be treated in the first instance (if no adverse signs) with
IV magnesium
1147. RCA myocardial infarction will produce an ECG with ST change in the inferior leads.
1148. Recognition of common conduction problems on ECG
1149. Recurrent episodes of haemodynamically compromising VT is high risk for sudden death.
1150. Reduction of alcohol intake should be employed before initiation of anti-hypertensives in patients with a significant
alcohol intake with hypertension.
1151. Refer for urgent specialist care on the same day if patient has accelerated hypertension or suspected
phaeochromocytoma.
1152. Renal crisis associated with systemic sclerosis should be treated with ACE inhibition, orally.
1153. Resynchronisation may improve pump performance and reverse the deleterious process of ventricular remodelling in
severe heart failure.
1154. Right bundle branch block (RBBB) is not an exclusion from a CRT device.
1155. S. bovis is an important cause of infective endocarditis. Patients with S. bovis bacteraemia should be offered GI
investigations for possibility of malignancy.
1156. Severe aortic stenosis with symptoms of cardiac compromise are indication for valve replacement.
1157. Simple dental procedures do not increase the risk of developing infective endocarditis.
1158. Skin appearance and systemic manifestations of pseudoxanthoma elasticum
1159. Smoking cessation will have greatest impact on CV risk in diabetic obese population
1160. Spironolactone reduces mortality in patients with NYHA class 3/4 heart failure
1161. Streptococcus bovis has a strong association with underlying colonic carcinoma and patients should be promptly
investigated by colonoscopy.
45
M Y Elamin
MBBS, DTM&H, MCTM, MRCPI 1& 2
1162. Subconjunctival haemorrhage is an alarming adverse effect of aspirin therapy (and other antiplatelets).
1163. Symptomatic left ventricular failure leads to the worst prognosis in heart failure.
1164. Tachycardia is an expected physiological response in pregnancy.
1165. Tamponade after pacemaker insertion should be considered in the hypotensive patient.
1166. Tendinous xanthomata is pathognomonic for type II(a) hyperlipidaemia (familial hypercholesterolaemia)
1167. The added sound is the pericardial 'knock' rather than a third/fourth heart sound in constrictive pericarditis.
1168. The Adult Advanced Life Support guidelines 2010 emphasise the importance of minimal interruption to chest
compressions.
1169. The commonest cause of MS is rheumatic valve disease or senility.
1170. The European Society of Cardiology recommends urgent extraction of the implanted device followed by prolonged
antibiotic therapy in patients with cardiac device related infective endocarditis.
1171. The first step in post-cardiac arrest care is to give aspirin and clopidogrel. This can usually be achieved quickly and
easily whilst other investigations and treatments are organised.
1172. The goal of digoxin therapy in patients with congestive heart failure is to improve quality of life by reducing symptoms
and preventing hospitalisations regardless if in AF or not.
1173. The left anterior descending coronary artery supplies the anterior wall and is a common vessel for left internal
mammary artery (LIMA) or saphenous vein grafts during CABG.
1174. The management of tachyarrhythmias requires the candidate to demonstrate knowledge of unstable vs. stable features
and then broad vs. narrow complex arrhythmias to instigate treatment correctly.
1175. The most appropriate management of STEMI is percutaneous coronary intervention (PCI.
1176. The presence of a VSD would be confirmed by detecting a step-up in the oxygen saturation between the RA and PA
1177. The presence of protein and blood with right flank pain in a patient with fever and new murmur and atrial fibrillation
suggest right renal infarction from infective endocarditis or an intracardiac thrombus.
1178. The primary treatment of coarctation is usually surgical or percutaneous balloon dilatation in re-stenoses.
1179. Theophylline is indicated for the management of bradycardia in patients who have had cardiac transplantation
1180. There are new cardiopulmonary resuscitation guidelines and these emphasise the importance of chest compressions for
as long as possible during the arrest situation
1181. There is no evidence that insulin reduces CV death in primary prevention
1182. Thrombolysis can be given during an arrest situation if PE is suspected, but CPR must be continued for 90 minutes
1183. Treatment of heart failure, aldosterone antagonists.
1184. Trifasicular block presents when their is abnormality in the AV node and Purkinje fibres.
1185. Type 3 hyperlipidaemia is associated with tendon xanthoma, pancreatitis and premature cardiovascular disease.
1186. UKPDS showed reduction in CV risk with BP control versus glycaemia control.
1187. Understanding of haemodynamic consequences of different valvular lesions
1188. Unopposed beta-antagonism should be avoided in malignant hypertension until the cause is identified
1189. Venous thromboembolism is a relatively common complication after major pelvic/orthopaedic surgery
46
M Y Elamin
MBBS, DTM&H, MCTM, MRCPI 1& 2
1190. Ventricular perforation is a complication post-pacemaker and require immediate medical and surgical intervention.
1191. Ventricular pre-excitation commonly masquerades as other conditions, such as bundle branch block or ischaemia.
1192. When dual therapy is maintained for less than 12 months, early cessation of clopidogrel is associated with an increased
risk of further ischaemic events.
1193. When to consider surgery in aortic stenosis (a common valve lesion in older adults)
1194. Where atrial fibrillation is of an acute onset, cardioversion is the preferred method (this may be chemical or electrical
in nature). For stable patients, chemical cardioversion manoeuvres should be tried first.
1195. Xanthelasma and corneal arcus are seen in common 'polygenic' hypercholesterolaemia, familial combined
hyperlipidaemia and familial hypercholesterolaemia.
1196.

47
M Y Elamin
MBBS, DTM&H, MCTM, MRCPI 1& 2
CLINICAL HAEMATOLOGY/ONCOLOGY
1. 2-level DVT Wells score ≤ 1 point - D-dimer with result within 4 hours or interim anticoagulation whilst
awaiting D-dimer
2. 2-level DVT Wells score ≤ 1 point, D-dimer negative - stop anticoagulation and consider an alternative
diagnosis
3. 2-level DVT Wells score ≤ 1 point, D-dimer positive - proximal leg ultrasound within 4 hours or (D-dimer if
not already done + anticoagulation + scan within 24 hours)
4. 2-level DVT Wells score ≥ 2 points - proximal leg ultrasound within 4 hours or (D-dimer if not already done +
anticoagulation + scan within 24 hours)
5. A 10-year-old child with a history of neonatal jaundice develops pallor and jaundice after an upper respiratory
tract infection associated with erythematous cheeks. Splenomegaly is noted on examination - hereditary
spherocytosis
6. A 15-year-old Greek boy develops pallor and jaundice after having a lower respiratory tract infection. He has
a history of neonatal jaundice. The blood film shows Heinz bodies - G6PD deficiency
7. A 40-year-old woman presents with fever, fluctuating neurological signs, thrombocytopenia and renal failure -
thrombotic thrombocytopenic purpura
8. A 60-year-old woman presents with cold peripheries. Investigations show a normocytic anaemia and a positive
direct antiglobulin test - autoimmune haemolytic anaemia
9. A 65-year-old man is noted to have a IgG paraprotein band. He is currently asymptomatic and other bloods
including calcium and renal function are normal - monoclonal gammopathy of undetermined significance
10. A 65-year-old man presents with back pain and lethargy. Bloods show a raised calcium, renal dysfunction and
a raised ESR - multiple myeloma
11. A 70-year-old is investigated after developing a DVT. A monoclonal IgM paraprotein band is found. He has
recently lost weight and feels unwell. His calcium and renal function are however normal - Waldenstrom's
macroglobulinaemia
12. A minor allergic reaction - pruritus, urticaria
13. A minor allergic reaction temporarily stop the transfusion, antihistamine, monitor
14. A minor allergic reaction thought to be caused by foreign plasma proteins
15. A non-haemolytic febrile reaction - fever, chills
16. A non-haemolytic febrile reaction slow or stop the transfusion, paracetamol, monitor
17. A non-haemolytic febrile reaction thought to be caused by antibodies reacting with white cell fragments in the
blood product and cytokines that have leaked from the blood cell during storage
18. Abdominal pain, peripheral neuropathy, blue line on gum - lead poisoning
19. Acanthocytes - abetalipoproteinemia
20. Acanthocytes - hyposplenism
21. Acute intermittent porphyria - defect in porphobilinogen deAminase
48
M Y Elamin
MBBS, DTM&H, MCTM, MRCPI 1& 2
22. Acute promyelocytic leukaemia (M3) - Auer rods
23. Acute promyelocytic leukaemia (M3) - disseminated intravascular coagulation
24. Acute promyelocytic leukaemia (M3) - PML/RAR fusion gene
25. Acute promyelocytic leukaemia (M3) - t(15:17)
26. Aflatoxin may cause hepatocellular carcinoma
27. An acute haemolytic reaction - fever, abdominal pain, hypotension, no dyspnoea
28. An acute haemolytic reaction ABO-incompatible blood e.g. secondary to human error
29. An acute haemolytic reaction stop the transfusion, check the identity of patient/name on blood product, send
blood for direct Coombs test, supportive care including fluid resuscitation
30. Anaphylaxis - hypotension, dyspnoea, wheezing, angioedema
31. Anaphylaxis - serum tryptase
32. Anaphylaxis can be caused by patients with IgA deficiency who have anti-IgA antibodies
33. Anaphylaxis stop the transfusion, IM adrenaline, supportive care including oxygen
34. Anastrozole - is a type of aromatase inhibitors
35. Anastrozole - is associated with an increased risk of osteoporosis
36. Anastrozole - is used in post-menopausal women if ER positive
37. Aniline dyes may cause transitional cell carcinoma
38. Aplastic anaemia, cafe au lait spots, short stature, acute myeloid leukaemia - Fanconi anaemia
39. Asbestos may cause mesothelioma and bronchial carcinoma
40. Auer rods - acute myeloid leukaemia
41. Basophilic stippling - lead poisoning
42. Basophilic stippling - thalassaemia
43. B-cell disorder - Bruton's congenital agammaglobulinaemia
44. B-cell disorder - common variable immunodeficiency
45. B-cell disorder - IgA deficiency
46. Bleeding problems, normal platelets, raised bleeding time, slightly raised APTT - von Willebrand's disease
47. Bleomycin - degrades preformed DNA
48. Bleomycin may cause pulmonary fibrosis
49. Blood transfusion --> anaphylaxis - IgA deficiency
50. Breast cancer - CA 15-3
51. Breast cancer - risk factors include: combined oral contraceptive pill use
52. Breast cancer - risk factors include: early menarche
53. Breast cancer - risk factors include: hormone replacement therapy (combined oestrogen + progesterone)
54. Breast cancer - risk factors include: late menopause
55. Breast cancer - risk factors include: mutations of the BRCA1 gene
56. Breast cancer - risk factors include: mutations of the BRCA2 gene
49
M Y Elamin
MBBS, DTM&H, MCTM, MRCPI 1& 2
57. Breast cancer - risk factors include: nulliparity
58. Breast cancer - risk factors include: obesity
59. Breast cancer - risk factors include: p53 gene mutations
60. Burkitt's lymphoma - c-MYC
61. Burkitt's lymphoma - Epstein-Barr virus
62. Burkitt's lymphoma - t(8:14)
63. Burkitt's lymphoma - tumour lysis syndrome
64. Burkitt's lymphoma (treated with chemotherapy) complication - tumour lysis syndrome
65. Burr cells (echinocytes) - pyruvate kinase deficiency
66. Burr cells (echinocytes) - uraemia
67. Chronic lymphocytic leukaemia - deletion of the long arm of chromosome 13
68. Chronic lymphocytic leukaemia (requiring treatment) , treatment of choice: fludarabine, cyclophosphamide and
rituximab (FCR)
69. Chronic lymphocytic leukaemia complication - anaemia
70. Chronic lymphocytic leukaemia complication - hypogammaglobulinaemia
71. Chronic lymphocytic leukaemia complication - transformation to high-grade lymphoma
72. Chronic lymphocytic leukaemia complication - warm autoimmune haemolytic anaemia
73. Chronic myeloid leukaemia - BCR-ABL fusion protein
74. Chronic myeloid leukaemia - decreased leukocyte alkaline phosphatase
75. Chronic myeloid leukaemia - t(9:22)
76. Chronic myeloid leukaemia , treatment of choice: imatinib
77. Ciprofloxacin may cause haemolysis in patients with G6PD deficiency
78. Cisplatin - causes cross-linking in DNA
79. Cisplatin may cause hypomagnesaemia
80. Cisplatin may cause ototoxicity
81. Cisplatin may cause peripheral neuropathy
82. Cold/purple peripheries, anaemia, direct Coombs test +ve - cold agglutinin disease
83. Colorectal cancer - carcinoembryonic antigen
84. Combined B- and T-cell disorder - ataxic telangiectasia
85. Combined B- and T-cell disorder - severe combined immunodeficiency
86. Combined B- and T-cell disorder - Wiskott-Aldrich syndrome
87. Congenital sideroblastic anaemia - delta-aminolevulinate synthase-2 deficiency
88. Cyclophosphamide - causes cross-linking in DNA
89. Cyclophosphamide may cause haemorrhagic cystitis
90. Cyclophosphamide may cause myelosuppression/agranulocytosis
91. Dark-coloured urine, haemolytic anaemia, thrombosis - paroxysmal nocturnal haemoglobinuria
50
M Y Elamin
MBBS, DTM&H, MCTM, MRCPI 1& 2
92. Docetaxel - prevents microtubule depolymerisation & disassembly, decreasing free tubulin
93. Doxorubicin - stabilizes DNA-topoisomerase II complex, inhibits DNA & RNA synthesis
94. Doxorubicin may cause cardiomyopathy
95. Essential thrombocytosis , treatment of choice: hydroxyurea
96. Extravascular haemolysis - haemolytic disease of newborn
97. Extravascular haemolysis - hereditary spherocytosis
98. Extravascular haemolysis - sickle cell anaemia
99. Extravascular haemolysis - thalassaemia
100. Extravascular haemolysis - warm autoimmune haemolytic anaemia
101. Factor V Leiden - activated protein C resistance
102. First proximal leg ultrasound is negative and D-dimer negative - stop anticoagulation and consider an
alternative diagnosis
103. First proximal leg ultrasound is negative and D-dimer positive - stop anticoagulation and repeat scan in 1
week
104. First proximal leg ultrasound is positive - diagnosis DVT and continue anticoagulation
105. Fluorouracil (5-FU) - pyrimidine analogue which induces cell cycle arrest and apoptosis by blocking
thymidylate synthase
106. Fluorouracil (5-FU) may cause dermatitis
107. Fluorouracil (5-FU) may cause mucositis
108. Fluorouracil (5-FU) may cause myelosuppression/agranulocytosis
109. Giant B cells with bilobed nuclei that have prominent eosinophilic inclusions - Hodgkin's lymphoma
110. ALL Good prognostic factor - common ALL
111. ALL Good prognostic factor - del(9p)
112. ALL Good prognostic factor - french-American-British (FAB) L1 type
113. ALL Good prognostic factor - low initial WBC
114. ALL Good prognostic factor - pre-B phenotype
115. Haemophilia A - deficiency of factor VIII
116. Haemophilia B - deficiency of factor IX
117. Heinz bodies - G6PD
118. Heinz bodies - thalassaemia
119. Hepatocellular carcinoma - alpha-feto protein
120. Hereditary angioedema - C1-INH deficiency
121. Hodgkin's lymphoma (lymphocyte depleted) - has the worst prognosis and is least common
122. Hodgkin's lymphoma (lymphocyte predominant) - has the best prognosis
123. Hodgkin's lymphoma (nodular sclerosing) - has a good prognosis and is most common
124. Hodgkin's lymphoma stage I - single lymph node
51
M Y Elamin
MBBS, DTM&H, MCTM, MRCPI 1& 2
125. Hodgkin's lymphoma stage II - 2 or more lymph nodes/regions on same side of the diaphragm
126. Hodgkin's lymphoma stage III - nodes on both sides of the diaphragm
127. Hodgkin's lymphoma stage IV - spread beyond lymph nodes
128. Howell-Jolly bodies - hyposplenism
129. HTLV-1 may cause adult T-cell leukaemia/lymphoma
130. Hydroxyurea - inhibits ribonucleotide reductase, decreasing DNA synthesis
131. Hydroxyurea may cause myelosuppression/agranulocytosis
132. Hypercalcaemia secondary to myeloma - increased osteoclastic bone resorption secondary to IL-1 and tumour
necrosis factor
133. Hypercalcaemia secondary to squamous cell lung cancer - secretion of parathyroid hormone-related protein
by the tumour
134. Hypersegmented neutrophils - megaloblastic anaemia
135. Imatinib - inhibitor of the tyrosine kinase associated with the BCR-ABL defect
136. Intravascular haemolysis - cold autoimmune haemolytic anaemia
137. Intravascular haemolysis - disseminated intravascular coagulation
138. Intravascular haemolysis - G6PD deficiency
139. Intravascular haemolysis - haemolytic uraemic syndrome
140. Intravascular haemolysis - mismatched blood transfusion
141. Intravascular haemolysis - paroxysmal nocturnal haemoglobinuria
142. Intravascular haemolysis - thrombotic thrombocytopenic purpura
143. Irinotecan - inhibits topoisomerase I which prevents relaxation of supercoiled DNA
144. Irinotecan may cause myelosuppression/agranulocytosis
145. ITP - antibodies directed against the glycoprotein IIb/IIIa or Ib-V-IX complex
146. Lead poisoning - abdominal pain
147. Lead poisoning - blue lines on gum margin
148. Lead poisoning - defective ferrochelatase and ALA dehydratase function
149. Lead poisoning - peripheral neuropathy
150. Lymphocyte sheets interspersed with macrophages containing dead apoptotic tumour cells - Burkitt's
lymphoma
151. Macrocytic (megaloblastic) - folate deficiency
152. Macrocytic (megaloblastic) - vitamin B12 deficiency
153. Macrocytic (normoblastic) - alcohol
154. Macrocytic (normoblastic) - hypothyroidism
155. Macrocytic (normoblastic) - liver disease
156. Macrocytic (normoblastic) - myelodysplasia
157. Macrocytic (normoblastic) - reticulocytosis
52
M Y Elamin
MBBS, DTM&H, MCTM, MRCPI 1& 2
158. Mantle cell lymphoma - over-expression of cyclin D1
159. Mantle cell lymphoma - t(11:14)
160. Massive splenomegaly may be caused by chronic myeloid leukaemia
161. Massive splenomegaly may be caused by Gaucher's syndrome
162. Massive splenomegaly may be caused by malaria
163. Massive splenomegaly may be caused by myelofibrosis
164. Massive splenomegaly may be caused by visceral leishmaniasis
165. Mesna - binds to and inactivates acrolein
166. Methaemoglobinaemia - IV methylene blue
167. Methotrexate - inhibits dihydrofolate reductase and thymidylate synthesis
168. Methotrexate may cause liver fibrosis
169. Methotrexate may cause mucositis
170. Methotrexate may cause myelosuppression/agranulocytosis
171. Methotrexate may cause pulmonary fibrosis
172. Microcytic - alpha-thalassaemia
173. Microcytic - beta-thalassaemia
174. Microcytic - congenital sideroblastic anaemia
175. Microcytic - iron-deficiency anaemia
176. Microcytic - lead poisoning
177. Monoclonal IgG or IgA paraprotein band, asymptomatic - monoclonal gammopathy of undetermined
significance
178. Monoclonal IgG or IgA paraprotein band, osteolytic lesions, increased infections - multiple myeloma
179. Monoclonal IgM paraprotein band, hyperviscosity syndrome - Waldenstrom's macroglobulinaemia
180. Neutrophil disorder - Chediak-Higashi syndrome
181. Neutrophil disorder - chronic granulomatous disease
182. Neutrophil disorder - leukocyte adhesion deficiency
183. Nitrosamines may cause gastric carcinoma
184. Nitrosamines may cause oesophageal carcinoma
185. Normocytic - anaemia of chronic disease
186. Normocytic - aplastic anaemia
187. Normocytic - chronic kidney disease
188. Normocytic - haemolytic anaemia
189. Ovarian cancer - CA 125
190. Pancreatic cancer - CA 19-9
191. Pappenheimer bodies - hyposplenism
192. Paroxysmal nocturnal haemoglobinuria - flow cytometry of blood to detect low levels of CD59 and CD55
53
M Y Elamin
MBBS, DTM&H, MCTM, MRCPI 1& 2
193. Paroxysmal nocturnal haemoglobinuria - increased sensitivity of cell membranes to complement secondary to
a lack of glycoprotein glycosyl-phosphatidylinositol (GPI)
194. Pencil poikilocytes - iron-deficiency anaemia
195. Polycythaemia rubra vera - mutation in JAK2
196. Polycythaemia rubra vera , treatment of choice: venesection
197. ALL Poor prognostic factor - age < 2 years or > 10 years
198. ALL Poor prognostic factor - cNS involvement
199. ALL Poor prognostic factor - fAB L3 type
200. ALL Poor prognostic factor - high initial WBC (e.g. > 100 * 109/l)
201. ALL Poor prognostic factor - male sex
202. ALL Poor prognostic factor - non-Caucasian
203. ALL Poor prognostic factor - philadelphia translocation, t(9;22)
204. ALL Poor prognostic factor - t or B cell surface markers
205. Prevention of haemorrhagic cystitis for patients taking cyclophosphamide , treatment of choice: mesna
206. Primaquine may cause haemolysis in patients with G6PD deficiency
207. Purpuric rash, hepatitis C positive - mixed (type II) cryoglobulinaemia
208. Rasburicase - metabolizes uric acid to allantoin
209. Rasburicase - recombinant version of urate oxidase
210. Recurrent bacterial infections, eczema, thrombocytopaenia - Wiskott-Aldrich syndrome
211. Schistocytes ('helmet cells') - intravascular haemolysis
212. Schistocytes ('helmet cells') - mechanical heart valve
213. Second proximal leg ultrasound is negative - stop anticoagulation and consider an alternative diagnosis
214. Second proximal leg ultrasound is positive - diagnosis DVT and continue anticoagulation
215. Siderotic granules - hyposplenism
216. Smudge/smear cells - chronic lymphocytic leukaemia
217. Sulfonylureas may cause haemolysis in patients with G6PD deficiency
218. Sulphasalazine may cause haemolysis in patients with G6PD deficiency
219. Sulphonamides may cause haemolysis in patients with G6PD deficiency
220. Tamoxifen - is a type of Selective oEstrogen Receptor Modulators (SERM)
221. Tamoxifen - is associated with an increased risk of endometrial cancer
222. Tamoxifen - is associated with an increased risk of venous thromboembolism
223. Tamoxifen - is used in pre- and peri-menopausal women if ER positive
224. Target cells - hyposplenism
225. Target cells - iron-deficiency anaemia
226. Target cells - liver disease
227. Target cells - sickle-cell anaemia
54
M Y Elamin
MBBS, DTM&H, MCTM, MRCPI 1& 2
228. Target cells - thalassaemia
229. T-cell disorder - DiGeorge syndrome
230. Tear drop shaped red blood cells - myelofibrosis
231. Tear-drop poikilocytes - myelofibrosis
232. Teratoma - alpha-feto protein
233. Thrombotic thrombocytopenic purpura - deficiency of ADAMTS13 (a metalloprotease enzyme)
234. Tranexamic acid - reversibly binds to lysine receptor sites on plasminogen or plasmin, preventing plasmin
from binding to and degrading fibrin
235. Transfusion-associated circulatory overload (TACO) - pulmonary oedema, hypertension
236. Transfusion-associated circulatory overload (TACO) excessive rate of transfusion, pre-existing heart failure
237. Transfusion-associated circulatory overload (TACO) slow or stop transfusion, supportive care including
oxygen, IV furosemide
238. Transfusion-related acute lung injury (TRALI) - pulmonary oedema, hypotension, fever
239. Transfusion-related acute lung injury (TRALI) non-cardiogenic pulmonary oedema thought to be secondary to
increased vascular permeability caused by host neutrophils that become activated by substances in donated
blood
240. Transfusion-related acute lung injury (TRALI) stop the transfusion, supportive care including oxygen
241. Vincristine - inhibits formation of microtubules
242. Vincristine may cause peripheral neuropath
243. 'B' symptoms in Hodgkin's lymphoma are associated with a poor prognosis
a. weight loss > 10% in last 6 months
b. fever > 38ºC
c. night sweats
244. 'CRAB' features of multiple myeloma = hyperCalcaemia, Renal failure, Anaemia (and thrombocytopenia)
and Bone fractures/lytic lesions
245. AIP - porphobilinogen deAminase; PCT - uroporphyrinogen deCarboxylase
246. Helicobacter pylori infection can lead to gastric lymphoma (MALT)
247. A low fibrinogen level is the major criteria determining the use of cryoprecipitate in bleeding
248. A risk-stratified approach should be taken when discharging patients who have presented with anaphylaxis
249. Abdominal pain, constipation, neuropsychiatric features, basophilic stippling → lead poisoning
250. Acquired inhibition of the protein ADAMTS13 which cleaves vWF multimers is the most common cause of TTP
251. Activated protein C resistance (Factor V Leiden) is the most common inherited thrombophilia
252. Acute intermittent porphyria typically presents with abdominal, neurological and psychiatric symptoms
253. Acute myeloid leukaemia - good prognosis: t(15;17)
254. Acute myeloid leukaemia - poor prognosis: deletion of chromosome 5 or 7
255. Acute promyelocytic leukaemia - t(15;17)
55
M Y Elamin
MBBS, DTM&H, MCTM, MRCPI 1& 2
256. Antiphospholipid syndrome: arterial/venous thrombosis, miscarriage, livedo reticularis
257. Ondansetron is a 5-HT 3 serotonin antagonist
258. An MRI whole spine should be performed in a patient suspected of spinal metastases
259. Anaphylaxis - serum tryptase levels rise following an acute episode
260. Anastrozole and letrozole are aromatase inhibitors that reduces peripheral oestrogen synthesis
261. Anterior mediastinal mass + symptoms of myasthenia = thymoma
262. Anthracyclines (e.g. doxorubicin) may cause cardiomyopathy
263. Antiphospholipid syndrome in pregnancy: aspirin + LMWH
264. Aplastic crises in sickle cell disease are associated with a sudden drop in haemoglobin
265. Aprepitant is an anti-emetic which blocks the neurokinin 1 (NK1) receptor
266. Aromatase inhibitors (e.g. anastrozole) may cause osteoporosis
267. Arterial/venous thrombosis, miscarriage, livedo reticularis → anticardiolipin antibody +ve → to make a diagnosis
of antiphospholipid syndrome
268. Benign ethnic neutropaenia is common in people of black African and Afro-Caribbean ethnicity
269. Bombesin is a tumour marker in small cell lung carcinomas
270. Burkitt's lymphoma - c-myc gene translocation
271. Burkitt's lymphoma - t(8:14)
272. Burkitt's lymphoma is a common cause of tumour lysis syndrome
273. CA 15-3 is a tumour marker in breast cancers
274. Cancer patients with VTE - 6 months of a DOAC
275. Carcinoembryonic Antigen (CEA) is a tumour marker in colorectal cancer and has a role in monitoring disease
activity
276. Chronic myeloid leukaemia - imatinib = tyrosine kinase inhibitor
277. Cisplatin - causes cross-linking in DNA
278. Cisplatin is associated with hypomagnesaemia
279. Cisplatin may cause ototoxicity
280. Cisplatin may cause peripheral neuropathy
281. CKD is the most common cause of antithrombin III deficiency
282. CLL - immunophenotyping is investigation of choice
283. CLL - treatment: Fludarabine, Cyclophosphamide and Rituximab (FCR)
284. CLL is caused by a monoclonal proliferation of B-cell lymphocytes
285. CML - Philadelphia chromosome - t(9:22)
286. Coagulase-negative, Gram-positive bacteria such as Staphylococcus epidermidis are the most common cause
of neutropenic sepsis
287. Combined B- and T-cell disorders: SCID WAS ataxic (SCID, Wiskott-Aldrich syndrome, ataxic telangiectasia)

56
M Y Elamin
MBBS, DTM&H, MCTM, MRCPI 1& 2
288. Common variable immunodeficiency - low antibody levels, specifically in immunoglobulin types IgG, IgM
and IgA. → recurrent chest and other infections
289. Cyclophosphamide - causes cross-linking in DNA
290. Cyclophosphamide - haemorrhagic cystitis - prevent with mesna
291. Cyclophosphamide may cause haemorrhagic cystitis
292. Dapsone + dyspnoea → methaemoglobinaemia
293. del 17p is associated with a poor prognosis in CLL
294. Desmopressiin - induces release of von Willebrand's factor from endothelial cells
295. DIC is associated with schistocytes due to microangiopathic haemolytic anaemia
296. Differentiating chronic myeloid leukaemia from leukaemoid reactions: leukocyte alkaline phosphatase score
is low in CML, high in leukaemoid reaction
297. Disproportionate microcytic anaemia - think beta-thalassaemia trait
298. During blood transfusions, minor allergic reactions may be managed by temporarily stopping the transfusion
and giving an antihistamine
299. EBV infection is implicated in the pathogenesis of Burkitt's lymphoma
300. Exposure to aflatoxin is a risk factor for hepatocellular carcinoma
301. Exposure to asbestos is a risk factor for bronchial carcinoma as well as mesothelioma
302. Extravascular haemolysis - hereditary spherocytosis
303. Factor V Leiden mutation results in activated protein C resistance
304. Factor V Leiden is the commonest inherited thrombophilia
305. Filgrastim is a granulocyte-colony stimulating factor used to treat neutropenia
306. First-line treatment for ITP is oral prednisolone
307. Flow cytometry for CD59 and CD55 is the gold standard test for paroxysmal nocturnal haemoglobinuria
308. Follicular lymphoma is characterised by a t(14:18) translocation
309. G6PD deficiency: sulph- drugs: sulphonamides, sulphasalazine and sulfonylureas can trigger haemolysis
310. Generally supportive treatment is given in the management of a patient in haemolytic crisis secondary to
hereditary spherocytosis. I.e. folic acid, transfusion if symptomatic from anaemia. Steroids are not indicated
311. Haptoglobin binds to free haemoglobin
312. Hb SC is a milder form of sickle disease
313. HbA2 is raised in patients with beta thalassaemia major
314. Hereditary angioedema (HAE) is pathophysiologically separate from anaphylaxis and is treated differently.
Therapeutic options are: intravenous infusion of human C1-esterase inhibitor or subcutaneous injection of the
bradykinin receptor inhibitor icatibant
315. Hereditary angioedema - C1-INH deficiency
316. Hereditary angioedema - C4 is the best screening test inbetween attacks
317. Hereditary angioedema is autosomal dominant
57
M Y Elamin
MBBS, DTM&H, MCTM, MRCPI 1& 2
318. Hereditary angioedema is caused by deficiency of C1 esterase inhibitor
319. Hodgkin's lymphoma - best prognosis = lymphocyte predominant
320. Hodgkin's lymphoma - most common type = nodular sclerosing
321. Howell-Jolly bodies and siderocytes are typical blood film findings of hyposplenism
322. Howell-Jolly bodies are present in hereditary spherocytosis post-splenectomy
323. HUS or TTP? Neuro signs point towards TTP
324. Hydroxyurea increases the HbF levels and is used in the prophylactic management of sickle cell anemia to
prevent painful episodes
325. Hypercalcaemia, renal failure, high total protein = myeloma
326. If neoplastic spinal cord compression is suspected, high-dose oral dexamethasone should be given whilst
awaiting investigations
327. IgM paraproteinaemia - ?Waldenstrom's macroglobulinaemia
328. IM adrenaline should be injected in the anterolateral aspect of the middle third of the thigh
329. In acute intermittent porphyria, the urine classically turns deep red on standing
330. In acute intermittent porphyria, urinary porphobilinogen is typically raised
331. In anaphylaxis, biphasic reactions can occur in up to 20% of patients
332. In chronic myeloid leukaemia there is an increase in granulocytes at different stages of maturation +/-
thrombocytosis
333. In patients with factor V Leiden, activated factor V is inactivated 10 times more slowly by activated protein C
than normal
334. In sickle-cell, acute painful vaso-occlusive crises should be diagnosed clinically
335. Intravascular haemolysis - paroxysmal nocturnal haemoglobinuria
336. Irinotecan inhibits topoisomerase I which prevents relaxation of supercoiled DNA
337. Irradiated blood products are used as they are depleted in T-lymphocytes
338. Irradiated blood products are used to avoid transfusion-associated graft versus host disease
339. Itching and tingling of the lips, tongue and mouth are the most common symptoms of oral allergy syndrome
340. ITP - give oral prednisolone
341. ITP should be considered in the presence of symptoms that suggest isolated thrombocytopenia e.g. epistaxis,
menorrhagia
342. Leucocyte alkaline phosphatase is low in CML but raised in myelofibrosis
343. Leukemoid reaction has a high leucocyte alkaline phosphatase score
344. Li-Fraumeni syndrome is caused by germline mutations to p53 tumour suppressor gene
345. Low haptoglobin levels are found in haemolytic anaemias
346. Lung adenocarcinoma
a. most common type in non-smokers
b. peripheral lesion
58
M Y Elamin
MBBS, DTM&H, MCTM, MRCPI 1& 2
347. Malaria prophylaxis (e.g. primaquine) can trigger haemolytic anaemia in those with G6PD deficiency
348. Methaemoglobinaemia = oxidation of Fe2+ in haemoglobin to Fe3+
349. Methotrexate - inhibits dihydrofolate reductase and thymidylate synthesis
350. Myelofibrosis - most common presenting symptom - lethargy
351. Myelofibrosis is associated with ‘tear drop’ poikilocytes on blood film
352. Normal pO2 but decreased oxygen saturation is characteristic of methaemoglobinaemia
353. Oral allergy syndrome is strongly linked with pollen allergies and presents with seasonal variation
354. Ovarian cancer - CA 125
355. Pancreatic cancer - CA 19-9
356. Breast cancers - CA 15-3
357. Patients with suspected neoplastic spinal cord compression should have an urgent MRI of the whole spine
358. Patients with Waldenstrom's macroglobulinaemia often present with issues secondary to hyperviscosity
359. Philadelphia translocation, t(9;22) - good prognosis in CML, poor prognosis in AML + ALL
360. Piperacillin with tazobactam (Tazocin) is the empirical antibiotic of choice for neutropenic sepsis
361. Platelet transfusions have the highest risk of bacterial contamination compared to other types of blood products
362. Polycythaemia rubra vera - around 5-15% progress to myelofibrosis or AML
363. Polycythaemia rubra vera - JAK2 mutation
364. Polycythaemia rubra vera is associated with a low ESR
365. Prostate cancer is the most common primary tumour that metastasises to the bone
366. Prothrombin complex concentrate is used for the emergency reversal of anticoagulation in patients with severe
bleeding or a head injury
367. Rasburicase - a recombinant version of urate oxidase, an enzyme that metabolizes uric acid to allantoin
368. Rasburicase and allopurinol should not be given together in the management of tumour lysis syndrome as this
reduces the effect of rasburicase
369. Raynaud's - Type I cryoglobulinaemia
370. Sickle cell patients should be started on long term hydroxycarbamide to reduce the incidence of complications
and acute crises
371. SLE is a risk factor for warm autoimmune haemolytic anaemia
372. Spread into the liver, bone marrow, lungs or other organs would be classified as stage IV on the Ann Arbor
staging system for Hodgkin's lymphoma
373. Stage III of the Ann-Arbor clinical staging of lymphomas involve lymph nodes on both sides of the diaphragm
374. SVC obstruction - dyspnoea is the most common symptom
375. SVC obstruction - oncological emergency
376. SVC obstruction can cause visual disturbances such as blurred vision
377. Target cells and Howell-Jolly bodies may be seen in coeliac disease → hyposplenism
378. Taxanes such as docetaxel - prevents microtubule depolymerisation & disassembly, decreasing free tubulin
59
M Y Elamin
MBBS, DTM&H, MCTM, MRCPI 1& 2
379. The EMA binding test should be used to diagnose hereditary spherocytosis - the osmotic fragility test is no longer
widely used
380. The sulfamethoxazole in co-trimoxazole causes haemolysis in G6PD, not the trimethoprim
381. The t(14;18) translocation causes increased BCL-2 transcription and causes follicular lymphoma
382. The universal donor of fresh frozen plasma is AB RhD negative blood
383. TRALI is differentiated from TACO by the presence of hypotension in TRALI vs hypertension in TACO
384. Trimethoprim may cause pantcytopaenia
385. TTP - plasma exchange is first-line
386. TTP is caused by the failure to cleave vWF normally
387. TTP presents with a pentad of fever, neuro signs, thrombocytopenia, haemolytic anaemia and renal failure
388. Venous thromoboembolism - length of warfarin treatment
a. provoked (e.g. recent surgery): 3 months
b. unprovoked: 6 months
389. Vincristine - peripheral neuropathy
390. Vitamin B12 is actively absorbed in the terminal ileum
391. Waldenstrom's macroglobulinaemia - Organomegaly with no bone lesions
392. Multiple myeloma - Bone lesions with no organomegaly
393. Warm autoimmune haemolytic anaemia involves IgG-mediated haemolysis
394. Wiskott-Aldrich syndrome
a. recurrent bacterial infections (e.g. Chest)
b. eczema
c. thrombocytopaenia
395. Woman with bone metastases- most likely to originate in the breast
396. A combination of thrombophilia, recurrent miscarriage, thrombocytopenia and leucopenia is sugestive of a
diagnosis of antiphospholipid syndrome.
397. A diagnosis of myelodysplastic syndrome is supported by the finding of ring sideroblasts in the marrow following
Perls' stain.
398. A 'group and save' is adequate for elective surgeries
399. A low partial pressure of oxygen (PO2) causes HbS to polymerise and precipitate resulting in sickling of the
erythrocyte. HbSS patients sickle at PO2 of 5-6 kPa and HbAS patients sickle at PO2 of 2.5-4 kPa.
400. A megaloblastic bone marrow occurs in vitamin B12 or folate deficiency and with some cytotoxic drugs.
401. A negative Coombs test would help differentiate auto-immune haemolysis.
402. A normal factor VIII would suggest haemophilia B where there is a lack of factor IX.
403. A positive direct antiglobulin test would be most likely to indicate a delayed transfusion reaction.

60
M Y Elamin
MBBS, DTM&H, MCTM, MRCPI 1& 2
404. A raised globulin and total protein level together with normocytic anaemia and probable vertebral collapse would
be highly suggestive of multiple myeloma.
405. A urine concentrating defect is quite common in sickle cell anaemia, has its onset in early childhood, and may be
reversible with prevention of sickle crises.
406. Absorption of oral iron is improved by ascorbic acid.
407. Acute chest syndrome, a complication of sickle cell disease, is defined as a 'new infiltrate consistent with
consolidation at least segmental in size, and one of: chest pain, a temperature > 38.5°C, tachypnoeic, wheezing or
cough'.
408. Acute human immunodeficiency virus (HIV) seroconversion illness should be suspected where there has been a
risk of exposure.
409. Acute intermittent porphyria (AIP) should be considered in cases where there are intermittent symptoms
characterised by repeated attacks of abdominal pain where no obvious cause is found.
410. Acute tumour lysis syndrome treatment revolves around institution of aggressive hydration, aiming for 3 L/m2
and control of electrolyte disturbances (typically, hypocalcaemia, hyperphosphataemia, hyperkalaemia and
uraemia) via the administrtion of Rasburicase.
411. Addison's disease is associated with primary hypothyroidism in the complex of autoimmune polyendocrine
syndrome.
412. AL amyloidosis is associated with deposition of immunoglobulin light chains and is caused by multiple myeloma.
413. Alpha interferon at 2 million U/m2 subcutaneously three times a week for 12-18 months can be used to salvage
relapsed or refractory hairy cell leukaemia.
414. Although bone x rays can show bone infection and avascular necrosis they are not done routinely as they will not
aid management in the majority of patients presenting with a sickle cell crisis.
415. Although female carriers of the haemophilia gene do not normally suffer from increased bleeding risk, APTT may
be prolonged.
416. An exchange transfusion requires blood which is plasma reduced whole blood, irradiated and less than 5-days-old.
417. Anaemia associated with diabetes is often due to reduced erythropoietin release due to chronic renal failure.
418. Anaemia is common in chronic kidney disease patients, and serum ferritin and transferrin saturation should be
checked to ensure the patient is iron replete prior to the initiation of recombinant erythropoiesis-stimulating agents.
419. Antigen presented on APCs is recognised by CD4 positive cells
420. Antiphospholipid syndrome predisposes to arterial and venous thrombosis, and can result in hypoadrenalism due
to adrenal infarcts.
421. Anti-TTG antibodies are most likely to be present in patients with coeliac disease.
422. Aplastic crisis in sickle cell anaemia (SSA) is caused by infection with the parvovirus B19.
423. Appropriate infusion times for various blood products.
424. Approximately 0.1% of body iron circulates in the plasma.
425. Atopy or allergic reactions show mildly elevated IgE concentration and normal C4 and C3 concentrations.
61
M Y Elamin
MBBS, DTM&H, MCTM, MRCPI 1& 2
426. Autoimmune haemolytic anaemia can follow antibiotic prescription.
427. Avascular necrosis of the femoral head can occur as a consequence of acute lymphoblastic leukaemia, or the
treatment for the disease.
428. Beta-thalassaemia trait is associated with minor suppression of beta-chain manufacture and a very low MCV
microcytic hypochromic anaemia.
429. Blasts are not present in chronic myeloid leukaemia, unless in accelerated or blast phase and basophilia is usually
present.
430. Bleomycin related pulmonary fibrosis is a major toxicity of the widely used ABVD regimen for treatment of
lymphoma.
431. Blood product irradiation is a necessary intervention in patients who have previously had Hodgkin lymphoma or
who have had autologous bone marrow transplantation.
432. Bronchial Ca can give rise to an autoimmune haemolytic process but the antibody is usually warm.
433. Burkitt's lymphoma is a monoclonal proliferation of B lymphocytes,which results (approximately 90% of the
cases) from chromosome translocations that involve the Myc gene.
434. Burkitt's lymphoma is associated with a t(8;14)(q24;q32) translocation, which is observed in approximately 80%
of patients with the disease.
435. Chemotheraphy should be started in CLL when there are B Symptoms present or progressive lymphoctosis
436. Chemotherapy can result in peripheral neuropathy, which classically resolves on cessation of the drug.
437. Chronic lymphocytic leukaemia (CLL) is commonly complicated by panhypogammaglobulinaemia. Although
intravenous immunoglobulin prevents recurrent infections it does not prolong survival.
438. Chronic myeloid leukaemia is easily diagnosed due to its consistent clinical and blood film examination.
Confirmation via bcr-abl assay permits early and effective control using tyrosine kinase inhibitors.
439. Chronic myeloid leukaemia is usually associated with tender splenomegaly. The Philadelphia chromosome is
present in 95% of cases.
440. CLL is a low-grade condition, and does not require immediate treatment.
441. Consider meningioma in a patient presenting with slowly progressive weakness associated with a sensory level.
442. Cytogenetic evaluation of malignant haematological cells may have important implications for the prognosis and
treatment options in acute myelogenous leukaemia (AML).
443. DDAVP is required as bleeding prophylaxis perioperatively in patients with von Willebrand disease.
444. DDAVP may be given to increase the amount of the von Willebrand factor long enough for surgery or dental
procedures to be performed.
445. Deranged coagulation associated with nephrotic syndrome is a consequence of AT III deficiency, increased
fibrinogen and increased factor VIIIc.
446. Diagnosis of multiple myeloma revolves around quantification of plasma cell mass in bone marrow, elucidation
of paraprotein, and imaging to assess for evidence of myeloma related organ or tissue impairment (myeloma
ROTI).

62
M Y Elamin
MBBS, DTM&H, MCTM, MRCPI 1& 2
447. Diagnostic criteria for CLL (lymphocytosis, lymphocyte morphology and immunophenotyping)
448. DIC is associated with an elevated D-dimer as well as decreased platelets and fibrinogen with normal clotting
factors.
449. Disorders that are potentially reversible in haemochromatosis include the dermal pigmentation and
cardiomyopathy.
450. Eighty-five percent of untreated subjects with granulomatosis with polyangiitis will have circulating anti-
neutrophil cytoplasmic antibody (cANCA) and those with limited disease are less likely to have positive serology.
451. Erythema nodosum has a typical appearance with nodular lesions on the extensor aspect of the shins, and the
cause is usually elicited by a careful clinical history.
452. Factor V leiden mutation (also known as activated protein C resistance) is the commonest inherited coagulopathy,
and results in a 30% lifetime risk of VTE for homozygotes and 5-10% for heterozygotes.
453. Familial hypocalciuric hypercalcaemia is an autosomal dominant condition
454. Flow cytometry showing a specific pattern of monoclonal B cell proliferation (CD19/5 coexpressing, CD23
positive, light chain restricted B cell population) is diagnostic of CLL.
455. Folic acid deficency is associated with alcoholism and presents with macrocytic anaemia.
456. Following delivery, the degree of FMH should be calculated on a blood sample from a D negative mother to
adjust the dose of anti-D in the D negative mother delivering a D positive child.
457. Fresh frozen plasma (FFP) contains more dilute clotting factors and therefore produces inferior correction and
should not be used in the management of life-threatening bleeding (unless prothrombin complex concentrate is not
available).
458. Fresh frozen plasma can be stored for up to 36 months below -25°C.
459. G6PD deficiency can lead to acute intravascular haemolysis after exposure to certain drugs, infection or food.
460. G-6-PD deficiency is more common in patients of Afro-Caribbean origin. Sulphonylureas are a class of drugs
associated with increased risk of red cell oxidation and the absence of G-6-PD leads to haemolytic anaemia and
increased levels of unconjugated bilirubin.
461. G6PD is X-linked, therefore females are carriers, and are not usually affected unless there is inactivation of their
X chromosome. Males are affected.
462. G6PDH (X-linked recessive) predisposes affected individuals to a haemolytic anaemia reaction with drugs
including quinine.
463. Gum hypertrophy may be seen in conditions such as acute myeloid leukaemias and with drugs such as phenytoin.
464. Haemolytic uraemic syndrome (HUS) presents with acute renal failure, microangiopathic haemolytic anaemia,
and thrombocytopenia with normal clotting.
465. Haemolytic uraemic syndrome bears resemblance to TTP except for prominent renal impairment and a lack of
neurological signs. It should be diagnosed quickly and managed with dialysis and plasma exchange.
466. Haemolytic uraemic syndrome bears resemblance to TTP except for prominent renal impairment, and should be
diagnosed quickly and managed with dialysis and plasma exchange.

63
M Y Elamin
MBBS, DTM&H, MCTM, MRCPI 1& 2
467. Haemophilia is an X-linked disease.
468. HbS is a result of a point mutation resulting in the subsitiution of glutamate for valine.
469. HbS is caused by a single base mutation on the beta-chain
470. Heinz bodies are oxidised denatured haemoglobin are present in glucose 6 phosphate dehydrogenase deficiency.
471. Heparin can induce thrombocytopenia which can require substitution with an alternative anticoagulant.
472. Heparin is only indicated in pregnancy if homozygous for factor V Leiden, or heterozygous with a previous
thrombotic event.
473. Hereditary haemorrhagic telangiectasia (HHT) is a multisystem vascular dysplasia characterised by the presence
of multiple arteriovenous malformations (AVMs) that lack intervening capillaries and result in direct connections
between arteries and veins.
474. Hereditary haemorrhagic telangiectasia (Osler-Weber-Rendu syndrome) is characterised by bleeding from
telangiectasia on mucous membranes such as the nose, mouth and gastrointestinal tract.
475. Hereditary spherocytosis presents with spherocytes on FBC film, raised mean corpuscular haemoglobin
concentration (MCHC), and an increase in reticulocytes, raised unconjugated bilirubin and reduced haptoglobulin.
476. Hypertension is a frequent problem associated with erythropoietin and may induce seizures.
477. Hypocalcaemia causes prolonged QT interval due to an increase in ST segment duration.
478. Hypoglycaemia is an important side effect of quinine therapy and should be monitored in those having
intravenous quinine.
479. Identifying blood products and potential infectious risk
480. Idiopathic thrombocytopenic purpura (ITP) is characterised by isolated thrombocytopenia with normal bone
marrow and in the absence of other causes of thrombocytopenia, presenting with bruising involving mucosa and
skin only, and treated initially with corticosteroids.
481. If both parents carry the sickle cell gene there is a 25% chance that their child will inherit sickle cell disease since
the gene is inherited in an autosomal recessive fashion.
482. If transferrin saturation >45% then genotyping should be considered to diagnose haemachromatosis.
483. Imatinib is an inhibitor of tyrosine kinase and is used in the treatment of conditions such as chronic myelocytic
leukaemia (CML) and gastrointestinal stromal tumours (GIST).
484. Immediate life-threatening reactions with intravascular haemolysis are caused by complement activating IgG or
IgM antibodies.
485. Immune thrombocytopenia is caused by antibody-mediated destruction of platelets and results in isolated
thrombocytopenia.
486. In 95% of cases of acute promyelocytic leukaemia, retinoic acid receptor-alpha (RARA) gene on chromosome 17
is involved in a reciprocal translocation with the promyelocytic leukaemia gene (PML) on chromosome 15.
487. In a patient with severe tonsillitis/pharyngitis and atypical lymphocytes, EBV infection should be considered the
most likely underlying cause.
488. In acute lymphoblastic leukaemia the presence of Philadelphia chromosome is associated with a poor prognosis.
64
M Y Elamin
MBBS, DTM&H, MCTM, MRCPI 1& 2
489. In acutely haemodynamically compromised trauma patients, a blood transfusion is the most appropriate fluid.
490. In APML, one of the retinoic acid receptor genes (RARA) is fused to PML in the great majority of patients as a
result of the chromosomal translocation t(15; 17).
491. In cases of major bleeding, omit warfarinand consult your local haematology guidelines on the use of Vitamin K
and Berriplex.
492. In cases suggestive of acute myeloid leukaemia, cytogenetic karyotype is of most prognostic value.
493. In contrast to CML, AML produces a marked reduction in platelets, red cells and mature neutrophils and
hepatosplenomegaly is seen in less than 10% patients. The main differential diagnoses of CML are Leukaemoid
reaction, Juvenile myelomonocytic leukemia, Chronic myelomonocytic leukemia, "Atypical CML", Chronic
eosinophilic leukemia, Chronic neutrophilic leukemia, other myeloproliferative neoplasms and other Philadelphia
chromosome-positive malignancies.
494. In donors who report conditions after donation, the blood products must be recalled till further testing and
clarification of the donor illness.
495. In DVt secondary to SLE lifelong anticoagulation with warfarin is required maintaining an international
normalised ratio (INR) above 2.5.
496. In essential thrombocytosis low-risk patients have a risk of thrombosis similar to that of the age and sex-matched
population and a very low risk of life-threatening bleeding, supporting close observation as the most sensible
approach.
497. In G-6-PD deficient patients, oxidative stress exposes interior sulfhydryl groups that are oxidised and cannot be
reduced, leading to irreversible denaturation of the haemoglobin with Heinz body formation.
498. In haemophilia B it is factor IX which is reduced.
499. In haemophilia B there is lack of factor IX. Factor VIII levels are normal.
500. In high prevalence trusts, all women undergo the initial laboratory screening to identify if the mother carries the
sickle cell gene, regardless of family origin.
501. In many cases of idiopathic thrombocytopenic purpura, the antibodies are against platelet membrane
glycoproteins IIb-IIIa or Ib-IX, and are of the IgG type.
502. In porphyria, aspirin is least likely to precipitate an acute attack.
503. In type I vWD the prothrombin time (PT) and platelet aggregation will be normal. Bleeding time, partial
thromboplastin time (APTT) and factor VIII-coagulant (FVIIIc) are likely to be abnormal.
504. Individuals with blood group O are more susceptible than other individuals to severe cholera, although the
mechanism underlying this association is unknown.
505. initial managment of ITP is with steroids.
506. IV glucose should be used, in saline solution, to reduce the production of porphyrin precursors during an acute
attack of acute intermittent porphyria.
507. JAK2 V617F is detectable in 95% of patients with primary polycythaemia. It is a cytoplasmic tyrosine kinase
which is responsible for signal transduction of haemopoietic growth factors, including erythropoietin.
508. Learn the antigen/antibody patterns for common blood groups
65
M Y Elamin
MBBS, DTM&H, MCTM, MRCPI 1& 2
509. Macrocytic anaemia should promt checking serum B12 levels (and folate).
510. Management of acute hyperleukocytosis should include reduction of the white cell mass by leukapheresis until the
appropriate diagnosis is reached.
511. Management of major bleed in patients on warfarin (pharmacological reversal of effects)
512. Medication can cause hyperkalaemia via various mechanisms.
513. Metformin can lead to reduced B12 absorption.
514. Methaemoglobinaemia is the accumulation of reversibly oxidised methaemoglobin causing reduced oxygen
affinity of the Hb molecule with consequent cyanosis. It can occur due to an inherited condition or as a
consequence of drugs such as nitrates.
515. Methotrexate, pyrimethamine, and trimethoprim inhibit dihydrofolate reductase and can affect folic acid
metabolism.
516. Microcytic anaemia should prompt investigation for occult GI malignancies
517. Microcytic anaemia would immediately raise the suspicion of iron deficiency perhaps from gastrointestinal or
menstrual blood loss. However, when the MCV is disproportionately low and combined with a raised HbA2 it
makes the diagnosis of beta-thalassaemia trait the most likely.
518. Minimum data set for transfusion include Full name, Date of Birth, Patient identity number and, in some areas,
Address.
519. Mislabelling of samples, requests or wrongly identifying recipients are the commonest transfusion errors.
520. MRI is indicated urgently when there is suspicion of spinal cord compression.
521. Multiple myeloma is characterised by plasma cell infiltration of the bone marrow, elevated calcium levels, lytic
bone lesions and normochromic normocytic anaemia; management is with chemotherapy and bone marrow
transplantation.
522. Myelofibrosis is evidenced by bone marrow fibrous infiltration, splenomegaly, tear drop poikilocytosis and JAK-
2 positivity in 50% of cases.
523. NASH is associated with increased prevalence of insulin resistance/type 2 diabetes.
524. Observation is the most appropriate action for patients with prognostic markers of essential thrombocythaemia
(ET) who are regarded as low risk.
525. Organ failure is a common finding, being as common as bleeding in DIC, and is likely to be due to fibrin
deposition within the organ.
526. Overview of transfusion reactions and ability to distinguish haemolytic vs non-haemolytic reactions
527. Pancytopenia is commonly seen in patients undergoing azathioprine therapy.
528. Parvovirus B19 has been shown to play a role in aplastic crises, both in those just suffering a de novo parvovirus
B19 infection, together with causing crises in those with underlying haemoglobinopathies.
529. Patients with hyposplenism should receive immunisation with pneumococcal, Haemophilus B and meningococcal
vaccines and prophylactic antibiotics with penicillin V or a macrolide if penicillin allergic.

66
M Y Elamin
MBBS, DTM&H, MCTM, MRCPI 1& 2
530. Patients with myelodysplastic syndrome (MDS) are more likely to have serious infections or life-threatening
bleeds than blastic transformation.
531. Patients with sickle cell disease have functional hyposplenism and should have regular vaccinations.
532. Pernicious anaemia is an autoimmune disease causing B12 malabsorption leading to a macrocytic anaemia.
533. Pernicious anaemia is usually a megaloblastic anaemia but may also be associated with a pancytopenia.
534. Phenytoin can lead to folate deficiency and present as a macocytic anaemia.
535. Plasma protein electrophoresis would be the investigation of choice in patients with suspected myeloma.
536. PML is associated with the 15:17 translocation, resulting in significant response to treatment with all-trans
retinoic acid.
537. Polycythaemia rubra vera is a form of primary polycythaemia and is commonly associated with JAK 2 mutations.
538. Post-translational modification by the GPI glycolipid anchor is essential for the surface expression of many
membrane proteins.
539. Products implicated in cases of transfusion associated (TA)-GVHD include packed red blood cells.
540. Prognosis in Hodgkin's disease depends on staging and presence of B symptoms; night sweats are a B symptom.
541. Prognosis in Hodgkin's lymphoma is influenced by age, stage, presence of B symtpoms and elevated ESR.
542. Prothrombin concentrates are products of choice for warfarin reversal in the setting of active bleeding and a
markedly raised INR.
543. Pure red cell aplasia is associated with thymoma.
544. Recognise common indications for gamma irradiated blood products
545. Recognise the risks for common viral infections transmitted by blood donations
546. Recognition and management of platelet refractoriness
547. Red cell mass studies can distinguish between depleted plasma volume and raised red cell mass, and are important
in the investigation of polycythaemia.
548. Salmonella osteomyelitis is seen in patients with sickle cell anaemia.
549. Schistocytes (fragmented red blood cells) and thrombocytopenia on blood film with neurological symptoms
(confusion) suggests thrombotic thrombocytopenic purpura.
550. Scurvy can present with a lowered haemoglobin concentration.
551. Several mechanisms resulting in heparin resistance have been identified, including antithrombin deficiency,
increased heparin clearance, elevated heparin-binding proteins, and elevated factor VIII and fibrinogen levels.
552. Significant improvements in survival may be expected through the addition of thalidomide to standard
chemotherapeutic regimes.
553. Stimulation of the production of fetal haemoglobin is its use in sickle cell anaemia; its usual mode of action is on
its reduction of production of deoxyribonucleotides via inhibition of the enzyme ribonucleotide reductase.

67
M Y Elamin
MBBS, DTM&H, MCTM, MRCPI 1& 2
554. Symptoms consistent with malabsorption, coupled with low albumin and iron deficiency anaemia fit best with a
diagnosis of coeliac disease.
555. T cell function
556. Teardrop poikilocytes are characteristic of myelofibrosis, which presents with chronic fatigue, anaemia and
abdominal fullness related to hepatosplenomegaly.
557. The 8:14 translocation is rare, but is associated with a very poor prognosis and CNS involvement at diagnosis in
patients with ALL.
558. The best product to correct fibrinogen is cryoprecipitate.
559. The best way to diagnose a lymphoproliferative disorder, such as chronic lymphocytic leukaemia, is
immunophenotyping of the blood: non-invasive and will give a diagnosis.
560. The best way to measure iron overload is via transferrin saturation which is calculated from the serum iron and
the serum total iron binding capacity.
561. The commonest cause of reduced APTT is in-vitro clotting cascade activation, but tests should be repeated to
exclude pathological causes of hypercoagulability.
562. The correct management of a sickle cell crisis would entail oxygenation, hydration with careful management of
fluid balance, analgesia, predominantly based on opioids, and blood product support,
563. The Duffy antigen receptor facilitates the entry of P. vivax into the red blood cells and Duffy negative individuals
are therefore resistant to this strain.
564. The first step in analysis of a spherocytic hemolytic anaemia is to determine whether the process is hemolytic or
not. The best way to do it is a direct antiglobulin test.
565. The incidence of patients with haemolytic disorders such as sickle cell disease and hereditary spherocytosis
forming pigment gallstones is approximately 50% due to an increase in bilirubin excretion.
566. The karyotype of bone marrow is the most important prognostic factorin patients with acute myeloid leukaemia,
as this result stratifies patients into lower risk, standard risk, and poor risk, which has prognostic significance.
567. The management of acute ITP with a life threatening bleed should be oriented towards normalising the platelet
count using multiple treatment modalities and curtailing any further bleeding and the ensuing mortality.
568. The most common cause of thrombocytosis is a reactive thrombocytosis.
569. The most likely cause for the persisting pyrexia plus hepatitis in this immunocompromised patient treated with
appropriate antibiotics would be a Cytomegalovirus infection.
570. The most likely explanation for the failure of an iron deficiency anaemia to respond to iron therapy in a
menstruant female is poor compliance.
571. The oral contraceptive and antipsychotics are possible causes of thromboembolism.
572. The Philadelphia chromosome (translocation 9;22) is present in approximately 90% of subjects with chronic
myelocytic leukaemia (CML).
573. The process of pooled plasma derivatives involves several chemical steps including ethanol extraction,
chromatography and viral inactivation steps which results in a freeze dried product.

68
M Y Elamin
MBBS, DTM&H, MCTM, MRCPI 1& 2
574. The product of the bcr/abl gene that is seen in 97% cases of chronic myeloid leukaemia is a constitutively active
tyrosine kinase; this is responsible for the leukaemic process.
575. The reduced red cell life cycle in HbAS reduces parasitaemia, reducing the risk of severe disease and neurological
complications.
576. The risk of developing overwhelming pneumococcal sepsis in NHL is lifelong
577. The shorter chain low molecular weight (LMW) fractions of heparin inhibit activated factor X.
578. There is no specific treatment for Zieve's syndrome but supportive therapy is indicated
579. There may be some response to pyridoxine therapy in patients with acquired sideroblastic anaemia.
580. Total serum IgE is frequently increased in those with atopy but serum IgE does not rise acutely during an
asthmatic attack.
581. Tranexamic acid competitively inhibits activation of plasminogen, thereby reducing the conversion of
plasminogen to plasmin, which reduces the risk of perioperative bleeding in patients with factor IX deficiency.
582. Transfusion associated lung injury (TRALI) occurs in patients who have received a multi-unit blood transfusion
previously and are then re-transfused some time later.
583. Treatment of beta thalassaemia major revolves around lifesaving red cell concentrate support, with the inherent
development of transfusional iron overload, which can be managed with iron chelation.
584. Treatment of G-6-PD deficiency revolves around avoidance of all known precipitating factors.
585. Trimethoprim can cause depression of haematopoiesis and lead to pancytopaenia on bone marrow aspiration.
586. TTP is characterised by thrombocytopenia, MAHA, and fluctuating neurological signs, rarely also with renal
impairment and fever, and is treated with urgent plasma exchange.
587. Type I von Willebrand disease is the most common form, and is inherited in an autosomal dominant manner with
variable penetrance.
588. Tyrosine kinase inhibitors are the gold standard therapy for CML.
589. Vaccinations should be given atleast two weeks before elective splenectomy.
590. Vitamin B12 deficiency following gastric bypass is a common complication which must be actively tackled with
supplementation, to avoid the repercussions from haematological, neurological and cognitive complications.
591. Von Willebrand disease (VWD) is inherited in an autosomal dominant fashion, and VWD type 1 is associated
with only minor increases in bleeding tendency.
592. von Willebrand is an under-recognised clotting disorder due to the variable inheritance pattern; a careful bleeding
and family history are imperative. It is managed with repletion of the missing clotting factor.
593. Von Willebrand's disease is a common inherited bleeding disorder.
594. Warfarin inhibits vitamin K epoxide reductase.
595. Warfarin is generally avoided in pregnancy.

69
M Y Elamin
MBBS, DTM&H, MCTM, MRCPI 1& 2
596. When a patient hyperventilates, there is a rise in intracellular pH (fall in partial pressure of carbon dioxide, which
can readily diffuse across cell membranes). The rise in pH then stimulates phosphofructokinase activity, which in
turn activates glycolysis.
597. When bleeding is the major problem in a pregnancy, the aim is to maintain the prothrombin and activated
thromboplastin time at a ratio of 1.5 times of the control and the fibrinogen level above 1 g/L.
598. Whilst clopidogrel does not have a clinically significant effect on the pharmacokinetics of warfarin it does
increase the bleeding risk when both are used concurrently.
599. A platelet count less than 50 in major procedures or CNS procedure such as lumbar puncture will require a
transfusion.
600. ABCDE assessment if first line in all patients with reduced conscious level.
601. Acquired haemophilia is a rare disorder, with acquired allo-antibodies to factor VIII (rarely factor IX); it has
known associations with underlying malignancy and autoimmune conditions.
602. Acute graft versus host disease (GVHD) occurs in the first 100 days post transplant with chronic GVHD
occurring 100-300 days post transplant and should be treated with high dose steroids.
603. Acute graft versus host disease is a multi system disease requiring high dose steroids urgently.
604. Addison's disease is associated with primary hypothyroidism in the complex of autoimmune polyendocrine
syndrome
605. Administering folic acid without vitamin B12 in folate deficiency would be hazardous and could precipitate
subacute combined degeneration of the spinal cord.
606. African trypanosomiasis has two recognised forms both of which are treated with Suramin
607. Agranulocytosis (neutrophil count <0.5 ×109/L) associated with carbimazole is rare but life threatening
608. All red cell components in the United Kingdom have been leucodepleted since 1999.
609. Anaemia associated with diabetes is often due to reduced erythropoietin release due to chronic renal failure.
610. Angioid Streaks are breaks in Bruch's membrane. They may be associated with pseudoxanthoma elasticum,
Ehlers-Danlos syndrome, sickle cell disease, acromegaly (and other pituitary disorders), and Paget's disease.
611. angioid streaks are caused by breaks in Bruch's membrane
612. anti fungal agents should be considered in neutopaenic sepsis where fevers persist and there is minimal clinical
improvement.
613. Antiphospholipid syndrome is characterised by recurrent venous and arterial thrombosis, recurrent abortions and
various neuropsychiatric illnesses.
614. Any patient with malignancy and a hot swollen leg is likely going to need investigation for DVT following Well's
scoring
615. APML M3 is associated with a t(15;17) translocation and treatment is with all-trans retinoic acid.
616. Blood film showing basophilic (blue) stippling of the red cells is typical of lead poisoning
617. Blood film that shows ring forms within erythrocytes; some erythrocytes contain two to three parasites per cell -
are typical of falciparum; other forms of malaria seldom have more than one parasite per red cell.
70
M Y Elamin
MBBS, DTM&H, MCTM, MRCPI 1& 2
618. Bone pain secondary to metastatic spread should be treated with bisphosphonates.
619. Chronic lymphocytic leukaemia (CLL) is a monoclonal disorder characterised by a progressive accumulation of
functionally incompetent lymphocytes.
620. Chronic lymphocytic leukaemia (CLL) presents with raised wcc and lymphocytosis, polychromasia on bld film
indicates reticulocytosis.
621. CML should be investigate for by bone marrow biopsy and analysis of cytogenics.
622. Common presentations and complications of chronic lymphocytic leukemia
623. Common variable immunodeficiency (CVID) is the most prevalent of the primary immunodeficiency diseases and
patients are deficient in both IgG and IgA.
624. Consider haemochromatosis if features of pigmentation, arthralgia, gynaecomastia and reduced libido
625. Diagnosis of myeloproliferative disorders
626. Differential diagnosis for anaemia and hepatosplenomegaly, and blood film findings in myelofibrosis
627. Differential diagnosis of bone marrow failure syndromes
628. E. coli 0157 can result in HUS and TTP, characterised by uraemia, anaemia, thrombocytopenia and MAHA.
629. Early initiation of chemotherapy has failed to show benefit in CLL, therefore no treatment required until
symptoms develop or evidence of rapid disease progression.
630. Emergency anticoagulation reversal in patients with major bleeding should be with 25-50 u/kg four-factor
prothrombin complex concentrate and 5 mg intravenous vitamin K.
631. Febrile neutropenia in haematological malignancies is a medical emergency.
632. Febrile transfusion reactions are common and are due to anti HLA antibodies in recipient serum or granulocyte
specific antibodies.
633. Fibrinogen consumption in DIC should be replaced with cryoprecipitate.
634. Fludarabine chemo increases ones risk of Pneumocystis jirovecii pneumonia and so prophylactic co-trimoxazole
should be given.
635. G6PD deficiency is an inherited deficiency which predisposes patients to acute haemolytic episodes, in the setting
of oxidant injury from medications, acute illnesses, and certain foods.
636. G6PD deficiency is the commonest inherited genetic abnormality worldwide and treatment is based on avoidance
of precipitants.
637. G6PD deficiency is the most commonly inherited genetic disorder worldwide.
638. Giving oral folic acid without vitamin B12 would be hazardous and could precipitate subacute combined
degeneration of the spinal cord in pernicious anaemia.
639. haemolysis with negative Coombes test should prompt one to think of hereditary spherocytosis.
640. Haemoptysis in patients on immunosuppression patients should be investigated with CT thorax and covered for
fungal infection.

71
M Y Elamin
MBBS, DTM&H, MCTM, MRCPI 1& 2
641. HE linical manifestations range from asymptomatic carriage to severe haemolytic anaemia and is diagnosed on
blood film showing cigar-shaped elliptocytes.
642. Hereditary haemorrhagic telangectasia is usually an autosomal dominant condition that may present with anaemia
due to bleeding from occult GI telangectasia
643. HIT is characterised by a more than 50% fall in platelet count or platelet nadir 20-100 ×109/L that usually occurs
between days five to 10 of starting heparin.
644. Howell-Jolly bodies are red cells seen on the peripheral blood film in which the nuclear remnant is still seen and
caused by splenic atrophy or splenectomy.
645. hyper viscosity syndrome in myeloma requires urgent plasmapheresis.
646. Hyposplenism is a cause of Howell-Jolly bodies on a blood film.
647. If INR is >8.0 and there is no bleeding, stop warfarin, give 0.5-1mg IV vitamin K and restart warfarin once INR is
< 5.0 in accordance with BJH guidelines.
648. Imatanib is first line treatment for CML.
649. In cases of Mild vWB prophylactic tranexamic acid should be used for minor procedures.
650. In cases of suspected TTP urgent blood film is required to look for red cell fragments.
651. In DIC there is the microangiopathic haemolytic anaemia with thrombocytopenia and coagulopathy due to
fibrinogen consumption.
652. In immune thrombocytopaenia the most important investigation is a blood film.
653. In ITP the Fibrinogen will be normal compared to a low fibrinogen in DIC
654. In leukoerythroblastic reactions the peripheral blood contains immature white cells and nucleated red cells
irrespective of the total white cell count and circulating blasts may be seen.
655. Initial diagnostic tests in suspected CML should be molecular analysis of peripheral blood for BCR-ABL
translocation
656. Interaction between co-trimoxazole and warfarin results in enhanced anticoagulant effect and requires reduction
in warfarin dose.
657. Intravenous iron replacement is now very widely used in patients not tolerating or failing on oral therapy.
658. It is imperative to recognize the syndrome of thrombotic thrombocytopenic purpura, and institute plasma
exchange with fresh frozen plasma as soon as possible to avoid the disease’s high mortality rate.
659. It is important to understand the issues surrounding the management of peri-operative assessment of
anticoagulation.
660. ITP in CLL is indication for chemotherapy initiation.
661. IV iron tends to work much more effectively in patients with CKD.
662. IVDU will have had years of damage to there arterial system following intravenous drug abuse could result in
atherosclerosis and high susceptibility to thrombus formation.
663. K-DOQI guidelines suggest maintaining Hb >110 g/L in CRF
72
M Y Elamin
MBBS, DTM&H, MCTM, MRCPI 1& 2
664. Knowledge of some of the common chemotherapy related complications
665. Knowledge of treatment options for low grade lymphoma
666. Low haematocrit and all cell lines affected in the Full blood Count are a sign of anaplastic anaemia.
667. lupus anticoagulant does not correct when mixed with normal plasma, suspected DVT, which with recurrent
miscarriages, strokes and thrombocytopenia, are features of the antiphospholipid syndrome
668. Management of polycythaemia
669. Management of thrombocytosis
670. Manifestations and management of hereditary haemorrhagic telangectasia
671. Massive and painless lymphadenopathy should prompt investigation for Lymphoma.
672. Massive transfusion guidelines suggest replacement of red blood cells once haematocrit falls below 0.3 and fresh
frozen plasma given when PT and/or APTT is more than 1.5 times normal ranges.
673. Multiple myeloma may present with roleaux formation on blood film and raised total protein (globulin
component).
674. Mycoplamsa Pneumonia can present with haemolytic anaemia
675. Mycosis fungoides (a cutaneous T cell lymphoma) presents as a pruritic eczematous rash and develops
telangiectasias and areas of 'cigarette paper' atrophy.
676. Oral iron chelation with deferasirox is the first line gold standard treatment for iron chelation in iron overload
related to repeated transfusions in patients with chronic anaemias.
677. paroxysmal nocturnal haemoglobinuria predisposes patients to DVT's
678. Patients with CLL who have immunparesis and recurrent infections may require IV Immunoglobulins regularly.
679. Patients with multiple lines of high dose chemotheraphy are high risk for secondary malignancy.
680. Patients with neutropaenic sepsis with no obvious source should be initiated on broad spectrum antibiotics
immediately.
681. Patients with sickle cell disease have a predisposition to develop osteomyelitis due to Salmonella species.
682. PBC is associated with non-specific symptoms such as fatigue and pruritis and is commonly associated with
autoimmune disorders
683. PCV is characterised by high HB and HCV and commonly is associated with JAK-2 mutation.
684. Peripheral blood flow cytometry (is an often overlooked diagnostic test for chronic lymphocytic leukaemia, but is
now the gold standard for diagnosis, so candidates should be aware of this development)
685. Peripheral or autonomic neuropathy is a feature of treatment with all vinca alkaloids and is a limiting side effect
of vincristine.
686. Phenothiazines can cause drug induced neutropaenia and as a result can lead to neutropaenic related infections.
687. Plasmapheresis is a useful therapeutic modality with straightforward indications where it is imperative to institute
it as soon as possible once the pathology is identified and diagnosed.

73
M Y Elamin
MBBS, DTM&H, MCTM, MRCPI 1& 2
688. Premedication should be avoided unless there is a history of hypersensitivity. A test dose prior to commencing IV
iron is always indicated, and low molecular weight iron dextran is the preferred preparation.
689. Presence of a Lupus anticoagulant demonstrates a significantly prolonged APTT which does not correct by a
significant amount when patient's plasma is mixed with normal plasma.
690. Presentation of myelodysplastic syndrome (pancytopenia, blood film findings, and bone marrow findings)
691. Primary haemochromatosis is diagnosed with fasting serum ferritin and transferrin saturation prior to moving to
more expensive and specific genetic studies and imaging/biopsy.
692. Pruritis is a common side effect of PCV and should be treated with an anti-histamine.
693. Radiotherapy is very useful in the management of back pain secondary to myeolma
694. Rapid rise in temperature following platelet transfusion is most likely due to Leucocyte cytokines present in the
unit transfused.
695. Recognition of acute haematological problems such as hyperviscosity
696. Recognition of common complications of chemotherapy
697. Recognition of some of the common complications of chemotherapy
698. Red cell membrane disorders
699. Risk factors for ATLS include high grade tumours and large tumour burden. Treatment is with raspuricase.
700. Secondary cold agglutinin disease typically presents with anaemia and haemoglobinuria due to intravascular
haemolysis two to three weeks following infection
701. Sepsis may be complicated by disseminated intravascular coagulation
702. Sickle-cell anaemia may present with occlusive sickling crises which may start as vague pain in back, abdomen or
long bones.
703. Subconjunctival haemorrhage is an alarming adverse effect of aspirin therapy (and other antiplatelets).
704. Target cells and Howell Jolly bodies are commonly seen in hyposplenism on a blood film.
705. Testing for heritable thrombophilia is not recommended in a patient with central venous catheter related
thrombosis, nor in central retinal vein thrombosis.
706. The absence of any other bulky or extra-gastric disease allows triple therapy for H.pylori eradication to be an
appropriate first line option
707. The aim in CKD is to maintain the haemoglobin concentration above 110 g/L to prevent ventricular hypertrophy,
which increases the cardiovascular morbidity and mortality.
708. The characteristic cytogenetic abnormality in Burkitt's lymphoma is a reciprocal translocation of t(8;14).
709. The cornerstone of the treatment of DIC is treatment of the underlying condition.
710. The most important treatment for neutropaenic sepsis is IV broad spectrum antibiotics.
711. The most likely cause of abnormal haematology following the loss of a large volume of blood is inadequate fluid
resuscitation. This can be compounded by hypothermia and acidosis.

74
M Y Elamin
MBBS, DTM&H, MCTM, MRCPI 1& 2
712. The plasma volume increases with increasing viscosity and may compromise cardiac function in patents with
myeloma receiving blood products.
713. The platelet count and the fibrinogen level are almost always decreased with a raised D-dimer in the presence of
DIC.
714. The present of smooth muscle antibodies are suggestive of autoimmune hepatitis.
715. The treatment of TTP is single volume daily plasma exchange with pathogen reduced plasma and
methylprednisolone 1 gram daily for three days.
716. This management of essential thrombocythaemia is based on cytoreduction with hydroxycarbamide and an
antiplatelet agent such as aspirin to decrease the thrombotic risk.
717. This question concerns the management of acute graft versus host disease post stem cell transplantation.
718. This question concerns the management of disseminated intravascular coagulation.
719. This question concerns the management of haemophilia.
720. This question concerns the management of pre-operative assessment of bleeding risk.
721. This question concerns the management of von Willebrand disease.
722. This question covers the investigation of thrombophilia.
723. This question covers the management of immune thrombocytopenic purpura.
724. This question covers the management of one of the serious side effects of IMIDs in the setting of myeloma.
725. This question covers the management of travel related venous thromboembolism.
726. This question covers the management of von Willebrand disease.
727. This question relates to the management of acute lymphoblastic leukaemia.
728. This question relates to the management of relapsed Hodgkin lymphoma.
729. This question relates to the management of thrombotic thrombocytopenic purpura.
730. Thrombophilia testing
731. Transfusion reactions. Investigation of suspected haemolysis post transfusion.
732. Treatment of TTP is with plasma exchange.
733. Trephine biopsy is the preferred initial investigation in myelofibrosis.
734. Use of IV iron should be considered early on in the event of adverse events to oral iron therapy rather than later.
735. von Willebrand disease can often present for the first time following surgery. In day-to-day practice, anaesthetists
will be expected to know about common bleeding disorders and be able to manage them.

75
M Y Elamin
MBBS, DTM&H, MCTM, MRCPI 1& 2
ONCOLOGY

1. A contrast CT thorax can help differentiate between benign and malignant disease and also guide further
investigations if an exudae effusion is suspected.
2. A laxative should always be started in conjunction with narcotics to avoid distressing constipation
3. A pattern of early breast cancers in a family may indicate a genetic cause such as the BRCA-1 mutation
4. A post-bronchodilator FEV1 of >1.5 litres is needed for a lobectomy, > 2 litres for a pneumonectomy.
5. A post-bronchodilator FEV1 of >1.5 litres is sufficient if a lobectomy is all that is required. If the tumour
necessitates a pneumonectomy, the post-bronchodilator FEV should be more than 2 litres.
6. A raised globulin and total protein level together with normocytic anaemia and probable vertebral collapse
would be highly suggestive of multiple myeloma.
7. A strong family history is a feature in 20-25% of all breast cancer patients.
8. A tumour size of greater than 3 cm (providing there is no lymphadenopathy or evidence of distant spread)
may still be amenable to surgery as it may fall within stages IB and IIB.
9. Acoustic neuromas are benign neoplasms that can be resected with a good prognosis.
10. Adjuvant chemotherapy reduces the risk of micrometastatic disease.
11. Adjuvant radiotherapy is given after wide local excision of a breast tumour to reduce the risk of local
recurrence.
12. Adult T cell leukaemia is associated with HTLV-1 infection.
13. AFP levels are part of post resection tesiticular teratoma surveillance.
14. Alpha interferon at 2 million U/m2 subcutaneously three times a week for 12-18 months can be used to
salvage relapsed or refractory hairy cell leukaemia.
15. An MRI spine is the investigation of choice for malignant spinal cord stenosis.
16. An understanding of the most common primary tumour sites of cerebral metastases is essential knowledge.
17. Anaplastic thyroid cancer often has local infiltration and invasion at diagnosis.
18. Anaplastic thyroid carcinoma has worse outcome, followed by medullary (particularly in MEN2B).
19. Aromatase inhibitors are the first line treatment option in early locally advanced breast cancer in post-
menopausal women.
20. Aromatase inhibitors cause oestrogen deprivation which can lead to osteoporosis.
21. Autoimmune thrombocytopaenia is uncommon in CLL.
22. Barrett's oesophagus is a premalignant state and surveillance endoscopies are recommended, with some
guidance suggesting two-yearly endoscopy.
76
M Y Elamin
MBBS, DTM&H, MCTM, MRCPI 1& 2
23. Bcl-2 is associated with follicular lymphoma.
24. Bisphosphonates rarely cause jaw osteonecrosis.
25. Bronchial Ca can give rise to an autoimmune haemolytic process but the antibody is usually warm.
26. Burkitt's lymphoma is a monoclonal proliferation of B lymphocytes,which results (approximately 90% of
the cases) from chromosome translocations that involve the Myc gene.
27. By crosslinking DNA in various ways, Cisplatin makes it impossible for rapidly dividing cells to duplicate
their DNA for mitosis.
28. CA 125 is a tumour marker for ovarian carcinoma. No tumour marker is specific and should be interpreted
in the clinical context.
29. CA125 is most commonly used to monitor ovarian cancer.
30. Carcinoid tumour of the lung originate from K cells
31. Carcinoid tumour usually occur in the major bronchi, 85% can be seen bronchoscopically.
32. Causes of cavitating lung lesions include malignancy (of which squamous cell carcinoma is the most
common), infection (S. aureus, TB, Klebsiella, PCP), infarction, granulomatosis with polyangiitis and
rheumatoid nodules.
33. CEA is the most appropriate tumour marker for colonic cancer
34. Cells are most resistant to radiation-induced apoptosis in G0, early G1 and the late S phase of the cell cycle.
Resistance in S phase is thought to be due to elevated levels of glutathione, as well as rapid DNA synthesis
and repair enzymes.
35. Cells are most sensitive in G2-M phase.
36. Cervical cancer is associated with strain types 16 and 18.
37. Cetuximab works by blocking the extracellular domain of EGFR preventing ligand binding and therefore
preventing downstream signal transduction. The patient’s tumour must express k-ras wild-type as k-ras
mutated is constitutively active regardless of whether a ligand is attached or not.
38. Chemotherapeutic agents like vincristine, cisplatin, taxanes and bortezomib may cause peripheral
neuropathy.
39. Chronic myeloid leukaemia is usually associated with tender splenomegaly. The Philadelphia chromosome
is present in 95% of cases.
40. Cisplatin covalently binds DNA, resulting in intra-strand and inter-strand crosslinks, thereby preventing
DNA, RNA and protein synthesis.
41. CML is associated with a reciprocal translocation of chromosomes 9 and 22.
42. Combination chemo-radiotherapy is the recommended first-line treatment for limited stage SCLC.

77
M Y Elamin
MBBS, DTM&H, MCTM, MRCPI 1& 2
43. Combining opiates with partial opioid antagonists, such as buprenorphine, may lead to reduced overall
analgesic effect.
44. Constipation is a commonly known side effect of opiate medications.
45. Continuously infused opiates are the most effective and controllable parenteral analgesia in palliative
situations.
46. Conus medullaris syndrome is caused by compression of the T12-L2 cord and nerve roots, and therefore
results in a mix of upper and lower motor neuron signs.
47. Corticosteroids, typically 16mg dexamethasone STAT, should be given in all cases of suspected metastatic
spinal cord compression whilst the diagnosis is being confirmed and definitive treatment planned.
48. Criteria for referral for urgent endoscopy include dyspepsia in a patient aged 55 or above with onset of
dyspepsia within one year and persistent symptoms.
49. Cytogenetic evaluation of malignant haematological cells may have important implications for the
prognosis and treatment options in acute myelogenous leukaemia (AML).
50. Decreased GTPase activity results in a constitutively active Ras.
51. Eaton-Lambert syndrome is a paraneoplastic syndrome with seventy percent of cases occuring in
association with small cell carcinoma.
52. Eaton-Lambert syndrome is associated with the presence of antibodies against the voltage dependent
calcium channels an is a paraneoplastic phenomenon associated with bronchial carcinoma.
53. Eaton-Lambert syndrome is characterised by proximal muscle weakness, depressed or absent tendon
reflexes, and autonomic features.
54. Endocrine treatment with an aromatase inhibitor is essential to prevent this patient’s cancer recurrence. It
can be continued in patients without fragility fractures providing a bisphosphonate is prescribed.
55. Endometrial cancer is increasingly recognised as a cause of paraneoplastic cerebellar degeneration, which
is associated with anti-Yo antibodies.
56. Epirubicin is limited to a lifetime maximum dose of 900 mg/m2.
57. Epstein-Barr virus is commonly associated with undifferentiated nasopharyngeal carcinoma.
58. Erlotinib specifically targets the epidermal growth factor receptor (EGFR) tyrosine kinase (which is
required for the conformational change) and binds in a reversible fashion to the adenosine triphosphate
binding site.
59. Etoposide is an inhibitor of topoisomerase 2.
60. Exemestane is an oral steroidal aromatase inhibitor used in second line treatment of metastatic breast
cancer.
61. Familial adenomatous polyposis (FAP) is caused by the loss of the APC gene on the long arm of
chromosome 5.

78
M Y Elamin
MBBS, DTM&H, MCTM, MRCPI 1& 2
62. Fifty to 70% of all presentations of LEMS are due to an underlying small cell lung cancer with the vast
majority linked to smoking tobacco.
63. Findings in hypercalcaemia.
64. Following a confrimed total excision of adenocarcinoma, patients should continue to be reviewed with
colonoscopy annually for at least two years.
65. G alpha subunit mutation is present in 30% of GH secreting pituitary tumours.
66. Gardener's syndrome is associated with intestinal tumours, lipomas and papillary thyroid cancer.
67. H. pylori has been shown in population studies to decrease the risk of oesophageal cancer.
68. Hashimoto's disease carries a small increased risk of thyroid lymphoma.
69. Herceptin is known to be cardiotoxic in approximately 5-10% of cases with a reduction in ejection fraction.
70. High ß-HCG levels cause increased oestrogen production leading to gynaecomastia.
71. HNPCC is inherited in an autosomal dominant manner.
72. HNPCC is relatively commonly associated with ovarian carcinoma, although colorectal carcinoma is the
most common malignancy.
73. Hypercalcaemia - nausea, fatigue, confusion, polyuria and constipation are all presenting clinical features
of multiple myeloma
74. Hypercalcaemia causes acute renal failure, a shortened Q-T interval on ECG as well as polyuria and
polydipsia.
75. Hypercalcaemia in absence of bony metastases occurs in about 15% of squamous cell lung carcinoma from
parathyroid hormone related protein (PTHrP) production.
76. Imatinib is a Bcr-abl tyrosine kinase inhibitor which is first line in the treatment of CML.
77. Imatinib is a tyrosine kinase inhibitor which blocks the active site of the BCR-Abl protein, resulting in
reduced cell proliferation and motility, and increased apoptosis.
78. In 95% of cases of acute promyelocytic leukaemia, retinoic acid receptor-alpha (RARA) gene on
chromosome 17 is involved in a reciprocal translocation with the promyelocytic leukaemia gene (PML) on
chromosome 15.
79. In acute lymphoblastic leukaemia the presence of Philadelphia chromosome is associated with a poor
prognosis.
80. In cases of suspected lung carcinaoma following chest radiograph, contrast-enhanced CT chest (including
the liver and adrenals) should be carried out. If the CT demonstrates a peripheral lung lesion, CT or
ultrasound-guided transthoracic needle biopsy should be offered.
81. In familial polyposis coli, the increased cancer risk is due to inheritance of a mutated tumour suppressor
gene.

79
M Y Elamin
MBBS, DTM&H, MCTM, MRCPI 1& 2
82. In pancreatic cancer a lack of pancreatic enzymes results in increased fats in the small intestine, leading to
sequestration of calcium and malabsorption of calcium as a consequence.
83. In patients unable to produce sputum, undertaking a bronchoscopy with bronchial washings for microscopy
staining and culture is the investigation of choice.
84. In patients with gastric carcinoma, endoscopic ultrasonography is superior to conventional CT scanning
for local tumour staging.
85. In young patients with a BRCA mutation, mammographic screening has a low sensitivity for detecting
tumours
86. Increased alkaline phosphatase is an adverse prognostic feature in cases of small cell lung cancer.
87. Initial treatment for superior vena cava obstruction is with steroids, which will reduce the oedema and
improve venous return.
88. Insulinoma presents with sweating and weight gain commonly.
89. Irinotecan is a chemotherapy agent that is a topoisomerase 1 inhibitor. Chemically, it is a semisynthetic
analogue of the natural alkaloid Camptothecin.
90. Kaposi’s sarcoma is caused by infection with human Herpesvirus 8 (HHV8).
91. Life threatening haemoptysis is a medical emergency often associated with lung malignancy; embolisation
of this vessel(s) will immediately stem the bleeding whne performed by interventional radiology.
92. Live vaccines including the rubella vaccine, should not be given whilst having chemotherapy and for six
months afterwards.
93. Lofgren's syndrome is a subtype of acute sarcoidosis which involves the combination of erythema nodosum
and bilateral hilar lymphadenopathy (stage 1 radiograph).
94. Low-grade gastric MALT tumours associated with Helicobacter pylori infection respond in over 80% to
Helicobacter eradication as the primary mode of treatment.
95. Lynch syndrome predisposes patients to development of many cancers including endometrial cancer.
96. Mediastinal radiotherapy leads to symptomatic relief in 80% of patients with superior vena cava obstruction
(SVCO), although case studies have shown this does not always correlate to patency of the superior vena
cava.
97. Methotrexate is an inhibitor of dihydrofolate reductase.
98. Morphine undergoes extensive first pass metabolism in the liver. However, it is the plasma half-life which
defines the timing of the doses.
99. Nephrotic syndrome is the main presentation of membranous glomerulonephritis and associated with
malignancy.
100. Non small cell lung cancer can give rise to gynaecomastia, which may be painful and associated with
testicular atrophy

80
M Y Elamin
MBBS, DTM&H, MCTM, MRCPI 1& 2
101. Of all the endocrine agents used to treat primary breast cancer only tamoxifen significantly increases the
risk of DVT.
102. Ondansetron is the best antiemetic in this list for chemotherapy-induced nausea and vomiting.
103. Ondansteron is a selective 5-HT3 antagonist both centrally and peripherally and as such is a potent
antiemetic.
104. Osteogenic sarcoma complicates 5% of cases of Paget's disease.
105. Overall 5 year survival of lung carcinoma is approximately 14%
106. p53 is a tumour suppressor gene and inactivating mutations of this gene occur in a large proportion of
human cancers.
107. Pain is the most common presenting feature of small bowel lymphoma with the ileum the site most
commonly affected.
108. Pancoast's tumour causes pain in the C8 and T1 distribution and Horner's syndrome.
109. Papillary thyroid cancer is the commonest thyroid epithelial tumour, and may present with a discrete
thyroid swelling which is cold on isotope scanning.
110. Paraneoplastic syndrome can present with an asymmetrical arthralgia which more commonly affects the
lower limbs.
111. Paraneoplastic syndromes are a result of antibody generation from or against malignant cells attacking
normal tissue. Examples include antineuronal antibodies directed against the Purkinje cells of the
cerebellum leading to cerebellar syndrome.
112. Patients who receive external beam radiotherapy alone are not radioactive, but those who have
brachytherapy or are treated with unsealed sources (such as radioactive iodine for thyroid conditions) may
be.
113. Patients with potentially operable tumours who are either too unfit for surgery, or who refuse surgery may
be suitable for radical radiotherapy.
114. P-glycoprotein is involved in the efflux of drugs from cells.
115. Pigmented spots on the iris are a characteristic feature of neurofibromatosis type 1 (NF1).
116. Platinum based chemotherapies cause cell death by DNA cross linkage.
117. Polycythaemia rubra vera is a form of primary polycythaemia and is commonly associated with JAK 2
mutations.
118. Products implicated in cases of transfusion associated (TA)-GVHD include packed red blood cells.
119. Prognosis in Hodgkin's disease depends on staging and presence of B symptoms; night sweats are a B
symptom.
120. Prognosis in Hodgkin's lymphoma is influenced by age, stage, presence of B symtpoms and elevated ESR.

81
M Y Elamin
MBBS, DTM&H, MCTM, MRCPI 1& 2
121. Prostate cancer is graded by the Gleason grading system.
122. Pure red cell aplasia is associated with thymoma.
123. Raised ß-HCG levels are associated with germ cell tumours which confer a good prognosis.
124. Remember that ptosis and constriction of the pupil can arise as a consequence of a pulmonary lesion
125. Risk factors for bladder cancer include exposure to aniline dyes in the printing and textile industry.
126. Salivary gland pleomorphic adenomas are the most common salivary gland tumour and are benign.
127. Significant improvements in survival may be expected through the addition of thalidomide to standard
chemotherapeutic regimes.
128. Small cell lung cancer typically produces ectopic ACTH.
129. Smoking is a risk factor for bladder cancer
130. Sometimes patients require stenting of the SVCO for symptomatic relief prior to more definitive treatment.
131. Stage 4 breast cancer is defined as a any breast cancer of any size which has spread outside the body.
132. Surgical resection of a lung tumour may be expected to have increased success with limitation of disease
spread.
133. SVCO can result in hoarseness, due to recurrent laryngeal nerve palsy, Horner's syndrome, facial and
periorbital oedema and distended chest wall veins. Treatment is dependent on the underlying cause.
134. Tamoxifen is used for adjuvant hormone treatment in pre-menopausal women first line. Anastrozole is
used for adjuvant hormone treatment in post-menopausal women first line.
135. The absolute risk reduction is an important figure and should always be quoted instead of the the relative
risk reduction.
136. The bcr-abl fusion protein is a potent tyrosine kinase which stimulates signal transduction and hence
mitosis.
137. The best way to diagnose a lymphoproliferative disorder, such as chronic lymphocytic leukaemia, is
immunophenotyping of the blood: non-invasive and will give a diagnosis.
138. The BRCA proteins 1 and 2 are involved in the repair of DNA double strand breaks.
139. The Dukes' staging system has been proven to correlate well with a patient’s chance of survival. Dukes'
C colon cancer has a 30-40% 5 year survival rate.
140. The karyotype of bone marrow is the most important prognostic factorin patients with acute myeloid
leukaemia, as this result stratifies patients into lower risk, standard risk, and poor risk, which has prognostic
significance.
141. The key to management of bony metastases is a multimodal approach to managing pain, which involves
opiates, paracetamol and an appropriate non-steroidal anti-inflammatory agent.
142. The only marker given here which is used in monitoring metastatic breast cancer is CA15-3
82
M Y Elamin
MBBS, DTM&H, MCTM, MRCPI 1& 2
143. The Philadelphia chromosome (translocation 9;22) is present in approximately 90% of subjects with
chronic myelocytic leukaemia (CML).
144. The product of the bcr/abl gene that is seen in 97% cases of chronic myeloid leukaemia is a constitutively
active tyrosine kinase; this is responsible for the leukaemic process.
145. The risk of mesothelioma is not affected by smoking but smoking and asbestos exposure greatly increase
the risk of lung cancer.
146. There is a higher prevalence of HBV in the Far East which can lead to hepatoma development if left
untreated.
147. This patient has a germ cell tumour of her ovary, the monitoring marker is AFP.
148. This patient has a severe grade 3 or 4 mucositis and needs to be admitted.
149. This patient has classical Lhermitte’s sign.
150. This patient has malignant spinal cord compression. She needs surgical decompression and radiotherapy.
151. This patient has received a single dose of targeted radiotherapy to the tumour bed, she is not radioactive.
152. This patient is in bed less than 50% of the day and self-caring but unable to work. She is performance
status 2.
153. This patient likely has an underlying breast cancer which needs triple assessment with physical
examination, mammography and ultrasound guided fine needle aspiration.
154. This patient likely has Paget’s disease of the breast; a skin biopsy will confirm this.
155. Thyroid lymphoma is frequently associated with Hashimoto's thyroiditis.
156. Trastuzamab (Herceptin) may be used to treat HER-2 positive breast cancers
157. Type 2 diabetics have a 40-60% increased risk of colon cancer.
158. Typical side effects of cisplatin include marrow toxicity, ototoxicity and peripheral neuropathy
159. Vaccinations should be given atleast two weeks before elective splenectomy.
160. VIPomas are endocrine tumours that secrete excessive amounts of VIP 32 which cause a distinct syndrome
characterised by large-volume watery diarrhoea, hypokalaemia, and dehydration.
161. Whilst ensuring adequate hydration and alkalinisation of urine is of value, rasburicase, (recombinant urate
oxidase) is effective when used both pre and post the onset of chemotherapy in reducing urate
accumulation.
162. Zolendronic acid (Zometa) has been proven to be the most effective bisphosphonate for breast cancer
metastasis.
163. 50-70% of all presentations of Lambert-Eaton myasthenic syndrome are due to an underlying small cell
lung cancer with the vast majority linked to smoking tobacco

83
M Y Elamin
MBBS, DTM&H, MCTM, MRCPI 1& 2
164. Adjuvant chemotherapy with an oxaliplatin-based regimen has been shown to improve disease-free survival in
patients with stage II and stage III colon cancers.
165. An elevated serum AFP is indicative of a nonseminoma; seminomas are associated with a normal serum alpha-
fetoprotein level.
166. CA125 is most commonly used to monitor ovarian cancer but can also be raised in endometrial, lung, breast and
gastrointestinal cancers.
167. Cetuximab works by blocking the extracellular domain of EGFR preventing ligand binding and therefore
preventing downstream signal transduction. The patient’s tumour must express K-Ras wild type as K-Ras mutated
is constitutively active regardless of whether a ligand is attached or not.
168. Clinical signs of papilloedema and gait ataxia suggest cerebellar pathology which may represent metastatic spread
of malignancy.
169. Consider food impaction as a cause of acute dysphagia and vomiting with those with pre-existing stents and acute
symptoms.
170. CT thorax is the imaging modality of choice in suspected cases of SVCO.
171. CTPA is the first line investigation for cases of suspected PE and the best diagnostically.
172. Dermatomyositis is linked with underlying bronchogenic, pancreatic, gastrointestinal, and breast cancer.
173. Dermatomyositis is very rarely seen in primary care but can be associated with lung, ovary and GI malignancies.
174. Disease process and management – common malignancies.
175. Eighty per cent of patients will maintain mobility if ambulatory function is good at presentation of malignant
spinal cord compression.
176. Epstein-Barr virus is believed to be the primary etiologic agent in the pathogenesis of nasopharyngeal carcinoma.
177. Exogenous PTH secretion and clubbing is associate with squamous cell carcinoma of the lung.
178. Fludarabine chemo increases ones risk of Pneumocystis jirovecii pneumonia and so prophylactic co-trimoxazole
should be given.
179. Fulvestrant is a new novel therapy for endocrine treatment of metastatic breast cancer, it selectively down
regulates oestrogen receptors and has been shown to be equivalent to anastrazole in terms of efficacy.
180. Gastric CA may progress from gastric ulcer and may present with Acanthosis nigricans
181. High dose steroid treatment is the first step in suspected malignant spinal cord compression.
182. HNPCC increases the risk of ovarian cancer (along with many other cancers).
183. Hospice care is appropriate for patients who have metastatic squamous cell lung cancer to the brain.
184. Hypercalcaemia of malignancy is an important complication of metastatic disease and knowing the patient has
bone metastases should prompt the clinician to make a rapid diagnosis and initiate treatment.
185. Hyphaema is blood in the anterior chamber. It is commonly seen after trauma to the eye
186. In a patient over the age of 40 with early satiety a diagnosis of gastric adenocarcinoma must be excluded in the
presence of acanthosis nigricans.
84
M Y Elamin
MBBS, DTM&H, MCTM, MRCPI 1& 2
187. In HIV infection, KS lesions occur most commonly on the face; however lesions may be widely disseminated in
skin, bronchial tree and gastrointestinal tract.
188. In membranous GN forty per cent remit without treatment, 30% develop endstage renal failure (ESRF).
189. Initial empiric antibiotic therapy for patients with suspected neutropenic sepsis with indwelling central venous
access devices includes Tazocin, Gentamicin and Vancomycin.
190. IV bisphosphonates should be used in metastatic bone disease and can improve pain in patients.
191. Kaposi's sarcoma (KS) lesion is a red-purple mass arising from the conjunctiva. It is not limited to the sclera
(which could suggest a subconjunctival haemorrhage) and it protrudes, which also favours a malignant growth.
192. Larynx preservation therapy with external beam radiation is curative and preferred over laryngectomy in patients
with early-stage cancer of the larynx.
193. Long term cancer survivors do not need any more screening than age appropriate cancer screening.
194. Lumpectomy with sentinel lymph node biopsy followed by breast irradiation is the appropriate management of
women with small, focal breast cancer.
195. Malignant spinal cord compression is an emergency, managed in the initial stages with high dose steroids
196. Melanoma commonly spreads to the brain and lesions can cavitate causing neurological decline.
197. MEN2 - phaeochromocytoma, medullary thyroid cancer and parathyroid adenoma.
198. MRI spine is the investigation of choice for investigating for potential metastatic spread of cancer.
199. MRI whole spine is the imagin modality of choice in cases of suspected malignant spinal cord compression
200. Mycosis fungoides (a cutaneous T cell lymphoma) presents as a pruritic eczematous rash and develops
telangiectasias and areas of 'cigarette paper' atrophy.
201. Neuroendocrine tumours usually respond well to somatostatin analogues
202. NICE recommends that if patients are of a good performance status then they should be offered combination
carboplatin and paclitaxel chemotherapy as this offers approximately an 8% increased response from single agent
carboplatin chemotherapy alone.
203. NICE recommends that if simple initial investigations fail to indicate a site for further investigation of a
malignancy of unknown origin (MUO) then a CT chest, abdomen and pelvis should be attempted to elicit any
suspicious areas which may be consistent with a primary malignancy.
204. One of the significant complications of trastuzumab is its effect on the heart. Trastuzumab is associated with
cardiac dysfunction in 2-7% of cases. As a result, regular cardiac screening with echocardiography is commonly
undertaken during the trastuzumab treatment period.
205. Only alcohol has been linked with nasopharyngeal cancer and breast cancer; a high consumption may increase the
risk of developing both of these cancers.
206. Only CA15-3 is used routinely to monitor metastatic breast cancer
207. Osteonecrosis of the jaw is a complication of bisphosphonate therapy, particularly with intravenous agents such as
zoledronic acid.

85
M Y Elamin
MBBS, DTM&H, MCTM, MRCPI 1& 2
208. Ovarian cancer screening is not recommended in general population as no survival benefit from earlier diagnosis
and therapy has been shown.
209. Papillary thyroid cancer is treated with total thyroidectomy followed radioiodine-131 therapy.
210. Patients diagnosed with stage I, grade 1 ovarian cancer have 90% cure rate with surgery alone.
211. Patients with a new diagnosis of multiple endocrine neoplasia type 2A should undergo screening for the RET
mutation.
212. Patients with active cancer who are found to have venous thromboembolic disease are recommended to have
anticoagulation indefinitely or till the time they are cured from cancer.
213. Patients with history of cancer need anticoagulation indefinitely or till the underlying malignancy is cured when
they have any history of deep vein thrombosis or pulmonary embolism.
214. Patients with neutropaenic sepsis with no obvious source should be initiated on broad spectrum antibiotics
immediately.
215. Patients with ovarian cancer peritoneal metastases have improved survival with debulking surgery and
chemotherapy as opposed to chemotherapy alone.
216. Prophylactic cranial irradiation reduces central nervous system relapse and improves survival in patients with
limited stage small cell lung cancer.
217. Recognition of a critically unwell patient is an essential skill, however one must always consider what is best for
the patient in every situation. It is very easy to escalate patients to inappropriate levels of care when they have an
extremely low probability of recovery from this.
218. Renal biopsy can help to determine the underlying disease process in a patient presenting with nephrotic
syndrome
219. Small cell carcinoma may be associated with ectopic ACTH production, causing Cushing's syndrome
220. Somatostatin analogues are the treatment of choice for carcinoid symptoms
221. Sunitinib is a tyrosine kinase inhibitor that improves progression-free survival in patients with metastatic renal
cell carcinoma.
222. Surgery and adjuvant combined chemotherapy and radiation therapy is the standard treatment for patients with
stage II and III rectal cancer
223. Surgical castration or gonadotropin hormone-releasing hormone (GnRH) agonists are first line therapies for an
asymptomatic patient with metastatic prostate cancer.
224. Surgical resection of an isolated hepatic metastases or small number of hepatic metastases from a previous colon
cancer is potentially curative in approximately 25% of patients.
225. Symptoms of bladder cancer may mimic those of more benign disorders, such as urinary tract infection; diagnosis
is established by cystoscopy and biopsy.
226. Tamoxifen is well known to cause an increased risk of DVT and patients must be consented for this prior to
commencement of treatment.
227. Tamoxifen treatment is indicated for patients with ductal carcinoma in situ to reduce the risk for local recurrences
and second primary breast cancer.
86
M Y Elamin
MBBS, DTM&H, MCTM, MRCPI 1& 2
228. Testicular cancer can be divided into germ cell and non-germ cell tumours the former of which is 90% of all
testicular tumours.
229. Tetracyclines can cause idiopathic intracranial hypertension
230. The best initial treatment in SVCO is with high dose steroids.
231. The Dukes' staging system has been proven to correlate well with a patient’s chance of survival; a Dukes' C colon
cancer has a 40-50% chance of cure at five years.
232. The importance of early recognition of clostridium difficile colitis/diarrhoea and the need to start treatment
urgently
233. The most common cause of SVCO is primary lung cancer but biopsy should be performed to exclude other
causes.
234. The platelet count and the fibrinogen level are almost always decreased with a raised D-dimer in the presence of
DIC.
235. There is a soft tissue mass destroying a large portion of the scapula. In a patient of this age a metastasis is the
most likely cause.
236. This patient has a likely diagnosis of Lambert-Eaton myasthenic syndrome from an underlying small cell lung
cancer. 50-70% of all presentations of LEMS are due to an underlying small cell lung cancer with the vast majority
linked to smoking tobacco.
237. This patient has developed osteonecrosis of the jaw due to bisphosphonates. Zoledronic acid is the
bisphosphonate of choice in metastatic breast cancer causing malignant hypercalcaemia.
238. This patient has likely spinal cord compression from an underlying undiagnosed malignancy and needs
emergency investigation.
239. Transbronchial lung biopsy will provide positive histology in about 80% of patients and provided it can be done
safely is the gold standard for diagnosis of sarcoidosis.
240. Treatment with adjuvant chemotherapy plus trastuzumab reduces the risk of recurrence and mortality in patients
with early stage, HER2-positive breast cancer.
241. Typhlitis or neutropenic colitis is a rare but serious complication of profound neutropenia which requires
intravenous antibiotics.
242. Unintentional weight loss may be a sing of underlying malignancy.
243. Zoledronate is an initial option for management of hypercalcaemia of malignancy.

87
M Y Elamin
MBBS, DTM&H, MCTM, MRCPI 1& 2
CLINICAL PHARMACOLOGY AND TOXICOLOGY
1. Abciximab - glycoprotein IIb/IIIa receptor antagonist
2. ACE inhibitors monitor with U&E
3. Acute alcohol intake is a P450 enzyme inhibitor
4. Alemtuzumab - anti-CD52 monoclonal antibody
5. Allopurinol - xanthine oxidase inhibitor
6. Allopurinol is a P450 enzyme inhibitor
7. Amiloride - blocks the epithelial sodium channel (ENaC) in the distal convoluted tubule
8. Amiloride may cause hyperkalaemia
9. Aminoglycosides - inhibits protein synthesis by acting on the 30S subunit of ribosomes
10. Amiodarone is a P450 enzyme inhibitor
11. Amiodarone may cause precipitation of digoxin toxicity
12. Amiodarone monitor with TFT, LFT
13. Amlodipine may cause ankle swelling
14. Amlodipine may cause flushing
15. Amlodipine may cause headache
16. Aspirin - blocks the action of cyclooxygenase-1 and 2 and thromboxane A2 formation
17. Azathioprine monitor with FBC, LFT
18. Benzodiazepine (iatrogenic) - flumazenil
19. Beta-blocker - atropine, glucagon
20. Botox - blocks the release of acetylcholine
21. Carbamazepine is a P450 enzyme inductor
22. Carbon monoxide - 100% oxygen, hyperbaric oxygen
23. Carbon monoxide poisoning - headache
24. Carbon monoxide poisoning - hyperpyrexia
25. Carbon monoxide poisoning - pink skin and mucosae
26. Carbopenems - inhibits cell wall formation
27. Cephalosporins - inhibits cell wall formation
28. Cetuximab - epidermal growth factor receptor antagonist
29. Chloramphenicol - inhibits protein synthesis by acting on the 50S subunit of ribosomes
30. Chronic alcohol intake is a P450 enzyme inductor
31. Ciclosporin - decreases IL-2 release by inhibiting calcineurin
32. Ciclosporin may cause fluid retention
33. Ciclosporin may cause gingival hyperplasia
34. Ciclosporin may cause hepatotoxicity
35. Ciclosporin may cause hyperkalaemia
88
M Y Elamin
MBBS, DTM&H, MCTM, MRCPI 1& 2
36. Ciclosporin may cause hyperlipidaemia
37. Ciclosporin may cause hypertension
38. Ciclosporin may cause hypertrichosis
39. Ciclosporin may cause impaired glucose tolerance
40. Ciclosporin may cause increased susceptibility to severe infection
41. Ciclosporin may cause nephrotoxicity
42. Ciclosporin may cause precipitation of digoxin toxicity
43. Ciclosporin may cause tremor
44. Cimetidine is a P450 enzyme inhibitor
45. Ciprofloxacin is a P450 enzyme inhibitor
46. Ciprofloxacin may cause lengthens QT interval
47. Ciprofloxacin may cause reduced seizure threshold
48. Ciprofloxacin may cause tendon damage
49. Clarithromycin - contraindicated by: concurrent statin therapy
50. Clindamycin - inhibits protein synthesis by acting on the 50S subunit of ribosomes
51. Coarse tremor, hyperreflexia, confusion - lithium toxicity
52. Cocaine - blocks the uptake of dopamine, noradrenaline and serotonin
53. Combined oral contraceptive pill may cause hypertension
54. Combined oral contraceptive pill may cause increased risk of breast cancer
55. Combined oral contraceptive pill may cause increased risk of cervical cancer
56. Combined oral contraceptive pill may cause increased risk of ischaemic stroke
57. Combined oral contraceptive pill may cause increased risk of myocardial infarction
58. Combined oral contraceptive pill may cause increased risk of venous thromboembolism
59. Cyanide - hydroxocobalamin
60. Decongestants (e.g. phenylephrine/oxymetazoline) - alpha 1-adrenoceptor agonist
61. Digoxin may cause bradycardia
62. Digoxin may cause confusion
63. Digoxin may cause gynaecomastia
64. Diltiazem may cause ankle swelling
65. Diltiazem may cause bradycardia
66. Diltiazem may cause heart block
67. Diltiazem may cause heart failure
68. Diltiazem may cause hypotension
69. Diltiazem may cause precipitation of digoxin toxicity
70. Erythromycin is a P450 enzyme inhibitor
71. Ethambutol - inhibits the enzyme arabinosyl transferase which polymerizes arabinose into arabinan
89
M Y Elamin
MBBS, DTM&H, MCTM, MRCPI 1& 2
72. Ethambutol may cause optic neuritis
73. Ethylene glycol - ethanol, fomepizole, haemodialysis
74. Exhibits zero-order kinetics - ethanol
75. Exhibits zero-order kinetics - heparin
76. Exhibits zero-order kinetics - high-dose aspirin
77. Exhibits zero-order kinetics - phenytoin
78. Fenofibrate may cause cholesterol gallstones
79. Fenofibrate may cause myositis
80. Fibrates - agonist of PPAR-alpha therefore increases lipoprotein lipase expression
81. Flecainide - sodium channel blocker
82. Fluconazole is a P450 enzyme inhibitor
83. Fluoxetine is a P450 enzyme inhibitor
84. Fomepizole - competitive inhibitor of alcohol dehydrogenase
85. Glitazones monitor with LFT
86. Glycaemic control in diabetes may be worsened by atypical antipsychotics
87. Glycaemic control in diabetes may be worsened by beta-blockers
88. Glycaemic control in diabetes may be worsened by ciclosporin
89. Glycaemic control in diabetes may be worsened by corticosteroids
90. Glycaemic control in diabetes may be worsened by interferon-alpha
91. Glycaemic control in diabetes may be worsened by nicotinic acid
92. Glycaemic control in diabetes may be worsened by tacrolimus
93. Glycaemic control in diabetes may be worsened by thiazides
94. Glycopeptides - inhibits cell wall formation
95. Heparin - protamine sulphate
96. Heparin may cause bleeding
97. Heparin may cause hyperkalaemia
98. Heparin may cause osteoporosis
99. Heparin may cause thrombocytopaenia
100. Heparin-induced thrombocytopaenia - antibodies form against complexes of platelet factor 4
101. Iron - desferrioxamine
102. Is affected by acetylator status - dapsone
103. Is affected by acetylator status - hydralazine
104. Is affected by acetylator status - isoniazid
105. Is affected by acetylator status - procainamide
106. Is affected by acetylator status - sulfasalazine
107. Isoniazid - inhibits mycolic acid synthesis
90
M Y Elamin
MBBS, DTM&H, MCTM, MRCPI 1& 2
108. Isoniazid is a P450 enzyme inhibitor
109. Isoniazid may cause hepatotoxicity
110. Isoniazid may cause peripheral neuropathy
111. Ketoconazole is a P450 enzyme inhibitor
112. Lead - dimercaprol, calcium edetate
113. Linezolid - inhibits protein synthesis by acting on the 50S subunit of ribosomes
114. Lithium - supportive, haemodialysis if severe
115. Lithium monitor with drug level, TFT, U&E
116. Loop diuretics may cause precipitation of digoxin toxicity
117. Low molecular weight heparin - activates antithrombin III. Forms a complex that inhibits factor ONLY Xa
118. Low molecular weight heparin - if needed, monitor with anti-Factor Xa
119. Low molecular weight heparin - longer acting duration
120. Low molecular weight heparin - lower risk of heparin-induced thrombocytopaenia
121. Low molecular weight heparin - routine monitoring not required
122. Low molecular weight heparin - should be used with caution in renal failure
123. Low molecular weight heparin - subcutaneous
124. Low-molecular weight heparin - activates antithrombin III. Forms a complex that inhibits factor Xa
125. Macrolides - inhibits protein synthesis by acting on the 50S subunit of ribosomes
126. Metformin - contraindicated by: CKD (eGFR < 30ml/min/1.73m2)
127. Metformin - contraindicated by: recent myocardial infarction
128. Metformin - increases insulin sensitivity and decreases hepatic gluconeogenesis
129. Metformin may cause lactic acidosis
130. Methanol poisoning - ethanol, fomepizole, haemodialysis
131. Methotrexate monitor with FBC, LFT, U&E
132. Metronidazole - damages DNA
133. Mixed metabolic acidosis and respiratory alkalosis - aspirin poisoning
134. Nausea/vomiting, confusion, yellow-green vision - digoxin toxicity
135. Nicotinic acid - decreases hepatic VLDL secretion
136. Nitrofurantoin may cause pulmonary fibrosis
137. Octreotide - analogue of somatostatin that inhibits the release of growth hormone, glucagon and insulin
138. Omeprazole is a P450 enzyme inhibitor
139. Opiate - naloxone
140. Organophosphate insecticide - atropine
141. Organophosphate insecticide poisoning - bradycardia
142. Organophosphate insecticide poisoning - diarrhoea
143. Organophosphate insecticide poisoning - miosis
91
M Y Elamin
MBBS, DTM&H, MCTM, MRCPI 1& 2
144. Organophosphate insecticide poisoning - salivation
145. Paracetamol - N-acetylcysteine (NAC)
146. Paracetamol-induced hepatic damage - decreased glutathione results in toxins forming covalent bonds with cell
proteins
147. Penicillins - inhibits cell wall formation
148. Phenobarbitone is a P450 enzyme inductor
149. Phenytoin is a P450 enzyme inductor
150. Prolactinoma , treatment of choice: bromocriptine
151. Pyrazinamide may cause gout
152. Pyrazinamide may cause hepatotoxicity
153. Quinidine may cause precipitation of digoxin toxicity
154. Quinolones - inhibits DNA synthesis
155. Rifampicin - inhibits bacterial DNA dependent RNA polymerase preventing transcription of DNA into mRNA
156. Rifampicin - inhibits RNA synthesis
157. Rifampicin is a P450 enzyme inductor
158. Rifampicin may cause hepatotoxicity
159. Rifampicin may cause orange tears and urine
160. Ritonavir is a P450 enzyme inhibitor
161. Rituximab - anti-CD20 monoclonal antibody
162. Salicylate - supportive, haemodialysis if severe
163. Salicylate - supportive, IV bicarbonate
164. Sertraline is a P450 enzyme inhibitor
165. Sildenafil - contraindicated by: concurrent nitrate therapy
166. Sildenafil - contraindicated by: recent myocardial infarction
167. Sildenafil - phosphodiesterase type V inhibitor
168. Sildenafil may cause blue discolouration of vision
169. Sildenafil may cause flushing
170. Sildenafil may cause nasal congestion
171. Sildenafil may cause non-arteritic anterior ischaemic neuropathy
172. Smoking is a P450 enzyme inductor
173. Sodium valproate is a P450 enzyme inhibitor
174. Sodium valproate monitor with LFT
175. Sotalol - blocks potassium channels
176. Spironolactone may cause precipitation of digoxin toxicity
177. St John's Wort is a P450 enzyme inductor
178. Statins monitor with LFT
92
M Y Elamin
MBBS, DTM&H, MCTM, MRCPI 1& 2
179. Streptogrammins - inhibits protein synthesis by acting on the 50S subunit of ribosomes
180. Sulphonamides - inhibits folic acid formation
181. Tamoxifen - selective oestrogen receptor modulator
182. Tamoxifen may cause flushing
183. Tamoxifen may cause increased risk of endometrial cancer
184. Tamoxifen may cause increased risk of venous thromboembolism
185. Tetracyclines - inhibits protein synthesis by acting on the 30S subunit of ribosomes
186. Thiazides may cause precipitation of digoxin toxicity
187. Trastuzumab - HER2/neu receptor antagonist
188. Trastuzumab - is a type of monoclonal antibody that targets HER2
189. Trastuzumab - is used if women are HER2 positive
190. Trastuzumab - may cause cardiac toxicity
191. Tricyclic antidepressant - supportive, IV bicarbonate
192. Trimethoprim - inhibits folic acid formation
193. Undergoes first-pass metabolism - glyceryl trinitrate
194. Undergoes first-pass metabolism - hydrocortisone
195. Undergoes first-pass metabolism - isoprenaline
196. Undergoes first-pass metabolism - isosorbide dinitrate
197. Undergoes first-pass metabolism - lignocaine
198. Undergoes first-pass metabolism - propranolol
199. Undergoes first-pass metabolism - testosterone
200. Undergoes first-pass metabolism - verapamil
201. Unfractionated heparin - activates antithrombin III. Forms a complex that inhibits thrombin, factors Xa, Ixa,
Xia and XIIa
202. Unfractionated heparin - higher risk of heparin-induced thrombocytopaenia
203. Unfractionated heparin - intravenous
204. Unfractionated heparin - monitor using activated partial thromboplastin time (APTT)
205. Unfractionated heparin - routine monitoring required
206. Unfractionated heparin - shorter acting duration
207. Unfractionated heparin - useful in patients with renal failure
208. Urinary retention may be worsened by NSAIDs
209. Urinary retention may be worsened by opioids
210. Urinary retention may be worsened by tricyclic antidepressants
211. Verapamil may cause bradycardia
212. Verapamil may cause constipation
213. Verapamil may cause flushing
93
M Y Elamin
MBBS, DTM&H, MCTM, MRCPI 1& 2
214. Verapamil may cause heart block
215. Verapamil may cause heart failure
216. Verapamil may cause hypotension
217. Verapamil may cause precipitation of digoxin toxicity
218. Warfarin - vitamin K
219. Carcinoid heart disease has a poor prognosis
220. Neuroleptic malignant syndrome (NMS) is characterised by fever, muscular rigidity, altered mental status, and
autonomic dysfunction.
221. procyclidine overdose include:
a. Agitation
b. Confusion
c. Sleeplessness lasting up to 24 hours or more, and
d. Pupils are dilated and unreactive to light
222. In women with oopthrectomy unopposed oestrogen HRT is appropriate.
223. Several drugs are known to exacerbate or trigger the onset of psoriasis, including beta blockers, lithium,
NSAIDs and anti-malarials.
224. Treatment of drug-induced psoriasis comprises withdrawal of all beta-blocking medications, NSAIDs,
antimalarials and lithium, unless absolutely necessary. Bed rest, bland topical compresses, and low potency
topical steroids are useful. Frequent emollient use is advisable.
225. Chlorpromazine may be associated with the development of drug-induced lupus.
226. Acute dystonic-dyskinetic reactions can occur following metoclopromide and are treated with IV benzatropine.
227. Acute dystonic-dyskinetic reactions mostly occur in children and young adults and about 70% of cases are
female. It occurs more commonly when excess of the recommended dose of metoclopramide is administered.
Symptoms include: oculogyric crisis opisthotonus torticollis trismus, and tetanus-like reactions. A blue
discolouration of the tongue has also been described. The effects usually occur within 72 hours but have been
reported to occur within 30 minutes of starting treatment. Although generally self-limiting, the reaction can be
reversed by an anticholinergic such as benzatropine or procyclidine or an antihistamine such as
diphenhydramine.
228. Patients with neutropaenic sepsis with no obvious source should be initiated on broad spectrum antibiotics
immediately. coverage that includes cover for Pseudomonas aeruginosa. penicillin - Piperacillin/tazobactam
(Tazocin). If infection with Staph. aureus is suspected, the addition of vancomycin should be considered.
229. Patients with Henoch-Schönlein purpura HSP and hypertension plus proteinuria should be treated with an
ACE-inhibitor which may control the blood pressure and proteinuria. If there were worsening renal function or
proteinuria, she should have a renal biopsy, and if this showed changes of a crescentic glomerulonephritis (GN),
then an immunosuppression regime similar to that used in renal vasculitis should be started (probably with high
dose steroids in the first instance +/- cyclophosphamide).
94
M Y Elamin
MBBS, DTM&H, MCTM, MRCPI 1& 2
230. Omeprazole is principally dependent upon hepatic clearance and safe even with marked renal impairment.
231. Spironolactone should probably be avoided with this degree of renal impairment owing to the risk of
hyperkalaemia.
232. Allopurinol toxicity is increased in moderate to severe renal impairment.
233. ADPK disease progresses to renal replacement therapy after 40-60 years. patients progress to end stage renal
failure between the ages of 40 and 60 years
234. In cases of high clinical suspicion for DVT with negative USS doppler scan a repeat scan should be ordered in
one week. Most D-dimer assays have a high negative predictive value (NPV) (approximately 90%). (NPV is the
probability that the patient does not have the disease given a negative test result, that is, if the test is negative they
are unlikely to have the disease.) However the positive predictive value (PPV) is often low (approximately
15%). (PPV is the probability that the patient will have a condition given a positive test result, that is, in the case of
D-dimer a positive result does not correlate with a high probability of disease). The test is therefore useful to
exclude deep vein thrombosis (DVT) rather than to confirm DVT.
235. Metformin is known to cause vitamin B12 deficiency, so should always be suspected as a cause of raised mean
corpuscular volume or megaloblastic anaemia.
236. Angiotensin-converting enzyme (ACE) inhibitor related angioedema has an incidence of 0.1-0.2% with a higher
incidence in the Afro-Caribbean population (as high as 5%). There is emerging evidence to suggest that ARBs
can also cause angioedema but less commonly. As ARBs have no effect on bradykinin levels an alternative
mechanism may be responsible for ACE related angioedema.
237. G6PDH (X linked recessive) is seen in people of African, Mediterranean, Iraqi, Jewish, South East Asian and
Chinese origin, and predisposes to a haemolytic anaemia reaction with drugs or infection. Implicated drugs
include: Aspirin Sulphonamides Antimalarials, and Quinine/quinidine.
238. Carcinoid heart disease has a poor prognosis
239. Several drugs are known to exacerbate or trigger the onset of psoriasis, including beta blockers, lithium,
NSAIDs and anti-malarials.
240. Omeprazole is principally dependent upon hepatic clearance and safe even with marked renal impairment.
241. ADPK disease progresses to renal replacement therapy after 40-60 years
242. Patients with HSP and hypertension plus proteinuria should be treated with an ACE-inhibitor.
243. Acute dystonic-dyskinetic reactions can occur following metoclopromide and are treated with IV benzatropine.
244. In cases of high clinical suspicion for DVT with negative USS doppler scan a repeat scan should be ordered in
one week.
245. Abuse of androgenic steroids can cause paranoid dellusions and aggression.
246. ACEi is first line blood pressure treatment in diabetes (but needs to be gradually titrated to avoid side-effects)
247. Acute cocaine overdose is initially best treated with benzodiazepines.
248. Acute dystonic-dyskinetic reactions can occur following metoclopromide and are treated with IV benzatropine.

95
M Y Elamin
MBBS, DTM&H, MCTM, MRCPI 1& 2
249. Acute interstitial nephritis is inflammation of the renal tubulo-interstitium and commonly occurs after a
prolonged course of NSAIDs due to hypersensitivity.
250. Acute iron poisoning typically presents with gastrointestinal symptoms - vomiting, diarrhoea, abdominal pain
and gastrointestinal haemorrhage Followed by CNS symptoms.
251. ADPK disease progresses to renal replacement therapy after 40-60 years
252. Agranulocytosis occurs in less than 1% of patients taking thionamides
253. Amiodarone can prolong QT interval
254. Amiodarone inhibits peripheral conversion of T4 to T3.
255. Amitryptilline overdose may result in QT prolongation and torsades de pointes
256. Angio-oedema can be precipitated by ACE-inhibitors.
257. Anion gap = (Na + K) - (Cl + HCO3); normal range 7-17 mmol/L.
258. Antiretroviral therapies are frequent causes of anaemia in HIV-seropositive patients.
259. Beta-blockers should not be commenced in the setting of acute exacerbations of COPD or cardiac failure.
260. Bisphosphonates are the drug of choice in established osteoporosis
261. BP control shows greater reduction in CV risk than tight glyceamic control in UKPDS
262. Bromocriptine should be used in the treatment of NMS.
263. Calcium antagonists and nitrates do not prevent cardiovascular events occurring.
264. Carbamezapine is a recognised cause of TEN.
265. Carbimazole may cause neutropenia (defined as neutrophils <0.5 ×109/L)
266. Carcinoid heart disease has a poor prognosis
267. Cardiac biomarkers, diagnosis of myocardial infarction.
268. Chronic paroxysmal hemicrania almost invariably responds to indomethacin.
269. Consider alcohol withdrawal as the cause of seizures in a patient with heavy alcohol intake. Management is with
benzodiazepines, not anti-epileptics.
270. Consider opiate toxicity with patients who have a history of chronic pain, respiratory depression and pin point
pupils.
271. CURB 65 score of 2 require oral dual antibiotic therapy.
272. Cyclophosphamide therapy is the most common cause of haemorrhagic cystitis.
273. Desferrioxamine is used for iron overload.
274. Ecstasy overdose has no specific antidote and treatment is supportive.
275. Elimination of weak acids from the body can be promoted by alkalinising the urine.
276. First generation antihistamines are more sedating than the newer agents.
277. Flumenazil is used in cases of benzodiazepine toxicity.
278. Group B Streptococcus has a predilection for cellulitis in diabetes
279. Heparin causes a false rise in free T4 due to displacement on assay
280. Heparin induced thrombocytopenia can be either immune or non-immune mediated.
96
M Y Elamin
MBBS, DTM&H, MCTM, MRCPI 1& 2
281. Hyperkalaemia treatment is initiated when over 6.5 or 6 with ECG changes.
282. In minor respiratory depression following opiate administration, flumenazil is first line for reversal.
283. In women with oopthrectomy unopposed oestrogen HRT is appropriate.
284. Li toxicity: greater than 4meq/L, consider HD
285. Lithium toxicity can be drugs which affect creatinine clearance such as ramipril.
286. Lithium toxicity can cause diarrhoea, but may also be precipitated by dehydration.
287. Macrocytosis can be caused by zidovudine.
288. Manifestations and management of hereditary haemorrhagic telangectasia
289. Mechanism of action of cyanide and other poisons
290. Methanol poisoning causes a metabolic acidosis with a raised anion gap.
291. Methanol poisoning presents with a wide anion gap,the definitive management involves haemodiaylysis.
292. Normal anion gap 8-16 mEq/L - in INH overdose it is wide
293. Patients on anti-obesity drugs who gain weight should have the drug withdrawn.
294. PCT may be precipitated by alcohol, exposure to sunlight, iron and oestrogen
295. Peripheral or autonomic neuropathy is a feature of treatment with all vinca alkaloids and is a limiting side effect
of vincristine.
296. Radioiodine therapy can worsen thyroid eye disease
297. Raised insulin with low c-peptide indicates exogenous insulin
298. Revise NICE guidance on type 2 diabetes and the introduction of insulin
299. Ritonavir is an extremely potent cytochrome P450-3A4 inhibitor.
300. Selective serotonin reuptake inhibitor (SSRI) discontinuation syndrome occurs following abrupt withdrawal of
SSRIs and symptoms occur after days
301. Smoking cessation will have greatest impact on CV risk in diabetic obese population
302. Somatostatin analogues improve symptoms and prognosis in carcinoid syndrome
303. Spironolactone reduces mortality in patients with NYHA class 3/4 heart failure
304. Statins cause a hepatocellular pattern of injury with transaminases elevated in excess of alkaline phosphatase.
305. Testosterone replacement may improve bone mineral density
306. Tetrabenezine is used in the management of hemiballismus
307. Tetracyclines can cause idiopathic intracranial hypertension
308. The combination of history and clinical findings are suggestive of malignancy as the likely cause
309. Threadworm infection is common in children and institutions
310. To consider steroid sparing therapy where reductions in corticosteroid have resulted in repeated relapses of
PMR symptoms
311. Use of opioids as pain management
312. Weaverfish stings should be treated with immersing the area in warm water.
313. Zolendronate is an initial option for management of hypercalcaemia of malignancy.
97
M Y Elamin
MBBS, DTM&H, MCTM, MRCPI 1& 2
314. A small proportion (0.5 - 6.5%) of patients with an IgE medicated penicillin allergy will also be allergic
to cephalosporins
315. Abciximab is a glycoprotein IIb/IIIa receptor antagonist
316. Activated charcoal can be used within an hour of an aspirin overdose
317. Adrenaline induced ischaemia - phentolamine
318. Allopurinol inhibits xanthine oxidase
319. Although rare, lactic acidosis is an important side-effect of metformin
320. Amiloride selectively blocks the epithelial sodium transport channels
321. Aminoglycoside nephrotoxicity is secondary to tubular necrosis
322. Aminoglycosides inhibit protein synthesis by acting on the 30S ribosomal unit
323. Amiodarone can cause corneal opacities
324. Amiodarone is a cause of photosensitivity
325. Amitriptyline can cause urinary retention
326. Aspirin decreases the formation of thromboxane A2 resulting in decreased platelet aggregation
327. Aspirin is a common cause of urticaria
328. Aspirin is a non reversible COX 1 and 2 inhibitor
329. Avoidance of using hypotonic (0.45%) in paediatric patients - risk of hyponatraemic encephalopathy
330. Beta-blocker - atropine, glucagon in resistant cases
331. Beta-blocker overdose management: atropine + glucagon
332. Beta-blockers can cause sleep disturbance
333. Biliary stasis and subsequently gallstones is a common adverse effect of octerotide
334. Botulinum toxin is used therapeutically in achalasia
335. Calcium channel blockers - side-effects: headache, flushing, ankle oedema
336. Carbamazepine can cause agranulocytosis
337. Carbamazepine is a P450 enzyme inductor
338. Carbon monoxide poisoning - most common feature = headache
339. Cetuximab - monoclonal antibody against the epidermal growth factor receptor
340. Ciclosporin + tacrolimus: inhibit calcineurin thus decreasing IL-2
341. Ciclosporin may cause nephrotoxicity
342. Ciclosporin side-effects: everything is increased - fluid, BP, K+, hair, gums, glucose
343. Ciprofloxacin is a P450 enzyme inhibitor
344. Ciprofloxacin is contraindicated in G6PD deficiency
345. Ciprofloxacin lowers the seizure threshold
346. Ciprofloxacin may lead to tendinopathy
347. Cocaine can induce preterm labour
348. Combined oral contraceptive pill
98
M Y Elamin
MBBS, DTM&H, MCTM, MRCPI 1& 2
a. increased risk of breast and cervical cancer
b. protective against ovarian and endometrial cancer
349. Cyanide inhibits the enzyme cytochrome c oxidase, resulting in cessation of the mitochondrial electron
transfer chain
350. Digoxin - inhibits the Na+/K+ ATPase pump
351. Digoxin may cause yellow-green vision
352. Digoxin normally binds to the ATPase pump on the same site as potassium. Hypokalaemia → digoxin more
easily bind to the ATPase pump → increased inhibitory effects
353. Doxazosin - alpha-1 antagonist
354. DRESS syndrome presents with a triad of extensive skin rash, high fever, and organ involvement
355. Drug metabolism
a. phase I: oxidation, reduction, hydrolysis
b. phase II: conjugation
356. Drugs which exhibit zero-order kinetics include phenytoin, alcohol and salicylates
357. Early endoscopy and risk stratification is important in patients with symptomatic caustic ingestion
358. Ergot-derived dopamine receptor agonists may cause pulmonary fibrosis
359. Ethylene glycol toxicity management - fomepizole. Also ethanol / haemodialysis
360. Exhibits zero-order kinetics - phenytoin
361. Flecainide blocks the Nav1.5 sodium channels in the heart
362. Fomepizole - used in ethylene glycol and methanol poisoning - competitive inhibitor of alcohol
dehydrogenase
363. Gastrointestinal side-effects such as diarrhoea and bloating are a common side effect with metformin
364. Glitazones can cause fluid retention and decompensation of heart failure
365. Glycaemic control in diabetes may be worsened by interferon-alpha
366. Glycaemic control in diabetes may be worsened by nicotinic acid
367. Heparin can cause drug induced thrombocytopaenia
368. Heparin-induced thrombocytopaenia - antibodies form against complexes of platelet factor 4 (PF4) and
heparin
369. Humanising is a process that decreases the immunogenicity of non-human derived monoclonal antibodies
370. Hydroxychloroquine can cause retinopathy
371. If metformin is not tolerated due to GI side-effects, try a modified-release formulation before switching to a
second-line agent
372. In carbon monoxide poisoning the oxygen saturation of haemoglobin decreases leading to an early plateau in
the oxygen dissociation curve
373. Infliximab is an anti-TNF monoclonal antibody used in the treatment of Crohn's disease
374. Is affected by acetylator status - hydralazine
99
M Y Elamin
MBBS, DTM&H, MCTM, MRCPI 1& 2
375. Isoniazid inhibits the P450 system
376. Isoniazid causes peripheral neuropathy
377. Ketamine is an NMDA receptor antagonist
378. Levothyroxine acts via nuclear receptors
379. Lithium toxicity can be precipitated by ACE inhibitors
380. Lithium toxicity can be precipitated by thiazides
381. Lithium: fine tremor in chronic treatment, coarse tremor in acute toxicity
382. Liver transplantation criteria in paracetamol overdose: pH < 7.3 more than 24 hours after ingestion
383. Local anesthetic toxicity can be treated with IV 20% lipid emulsion
384. Low-molecular weight heparin activates antithrombin III. Forms a complex that inhibits factor Xa
385. Macrolides - inhibits protein synthesis by acting on the 50S subunit of ribosomes
386. Mercury poisoning can cause visual field defects, hearing loss and paraesthesia
387. Metformin acts by activation of the AMP-activated protein kinase (AMPK)
388. Metformin should be titrated slowly, leave at least 1 week before increasing dose
389. Michaelis-Menten kinetics describes enzymatic reactions where a maximum rate of reaction is reached when drug
concentration achieves 100% enzyme saturation. This results in non-linear elimination kinetics.
390. Motion sickness - hyoscine > cyclizine > promethazine
391. N-acetylcysteine is a precursor of glutathione
392. Nitrofurantonin may cause pulmonary fibrosis
393. NSAIDs are a cause of thrombocytopenia
394. Octreotide is a somatostatin analogue
395. Optic neuritis is common in patients taking ethambutol
396. Organophosphate insecticide - atropine
397. Organophosphate insecticide poisoning - bradycardia
398. Oxycodone is a safer opioid to use in patients with moderate to end-stage renal failure
399. Paracetamol overdose occurs when glutathione stores run-out leading to an increase in NAPQI (N-acetyl-p-
benzoquinone imine)
400. Paracetamol overdose: acute alcohol intake is not associated with an increased risk of developing
hepatotoxicity and may actually be protective
401. Patients who take a staggered paracetamol overdose should receive treatment with acetylcysteine
402. Patients with MI secondary to cocaine use should be given IV benzodiazepines as part of acute (ACS)
treatment
403. PDE 5 inhibitors (e.g. sildenafil) - contraindicated by nitrates and nicorandil
404. Penicillin is a common cause of urticaria
405. Phenylephrine is an alpha-1 agonist
406. Pilocarpine is a muscarinic agonist
100
M Y Elamin
MBBS, DTM&H, MCTM, MRCPI 1& 2
407. Quinine toxicity (cinchonism) presents with myriad ECG changes, hypotension, metabolic acidosis,
hypoglycaemia and classically tinnitus, flushing and visual disturbances. Flash pulmonary oedema may occur
408. Recommend Adult Life Support (ALS) adrenaline doses
a. anaphylaxis: 0.5mg - 0.5ml 1:1,000 IM
b. cardiac arrest: 1mg - 10ml 1:10,000 IV or 1ml of 1:1000 IV
409. Rifampicin - inhibits RNA synthesis
410. Rifampicin is a P450 enzyme inductor
411. Rituximab - monoclonal antibody against CD20
412. Sarin gas is a highly toxic synthetic organophosphorus compound which causes inhibition of the enzyme
acetylcholinesterase
413. Screen for HLA-B *5801 allele in a patient at high risk for allopurinol induced severe cutaneous adverse
reaction
414. Severe lithium toxicity is an indication for haemodialysis
415. Sildenafil - phosphodiesterase type V inhibitor
416. Smoking is a P450 enzyme inductor
417. SSRIs + MDMA = higher risk of serotonin syndrome
418. Sulphonylureas may cause syndrome of inappropriate ADH
419. Suxamethonium is a depolarising muscle relaxant
420. Tacrolimus is a cause of impaired glucose tolerance
421. Tachyphylaxis is defined as a rapidly diminishing response to successive doses of a drug, rendering it less
effective. The effect is common with drugs acting on the nervous system.
422. Tamoxifen may cause hot flushes
423. Teicoplanin is similar to vancomycin (e.g. a glycopeptide antibiotic), but has a significantly longer duration of
action, allowing once daily administration after the loading dose
424. The arterial pH is the single most important prognostic factor in paracetamol overdose
425. There is no evidence that antibiotics other than enzyme inducing antibiotics such as rifampicin reduce the
efficacy of the combined oral contraceptive pill
426. Thiazides may cause photosensitivity
427. Think ischaemic colitis in patients presenting with abdominal pain or rectal bleeding following cocaine
ingestion
428. Trastuzumab (Herceptin) - cardiac toxicity is common
429. Trastuzumab (Herceptin) - monoclonal antibody that acts on the HER2/neu receptor
430. Unfractionated heparin - activates antithrombin III. Forms a complex that inhibits thrombin, factors Xa,
Ixa, Xia and XIIa
431. Verapamil can cause constipation
432. Verapamil commonly causes constipation
101
M Y Elamin
MBBS, DTM&H, MCTM, MRCPI 1& 2
433. Visual changes secondary to drugs
a. blue vision: Viagra ('the blue pill')
b. yellow-green vision: digoxin
434. Widened QRS or arrhythmia in tricyclic overdose - give IV bicarbonate
435. Steven Johnson's syndrome (SJS) and toxic epidermal necrolysis (TEN) are both on a spectrum of skin disorders
associated with drug reactions. These, however, are limited to the skin and would not cause deranged LFTs. SJS
would typically present with mucosal involvement, whilst TEN causes desquamating skin lesions.
436. Abuse of androgenic steroids can cause paranoid dellusions and aggression.
437. ACEi is first line blood pressure treatment in diabetes (but needs to be gradually titrated to avoid side-
effects)
438. Acute cocaine overdose is initially best treated with benzodiazepines.
439. Acute dystonic-dyskinetic reactions can occur following metoclopromide and are treated with IV
benzatropine.
440. Acute interstitial nephritis is inflammation of the renal tubulo-interstitium and commonly occurs after a
prolonged course of NSAIDs due to hypersensitivity.
441. ADPK disease progresses to renal replacement therapy after 40-60 years
442. Agranulocytosis (neutrophil count <0.5 ×109/L) associated with carbimazole is rare but life threatening
443. Agranulocytosis occurs in less than 1% of patients taking thionamides
444. Amiodarone-induced thyrotoxicosis is due to thyroiditis and can be treated with steroids
445. Amiodarone-induced thyrotoxicosis may be due to thyroiditis and can be treated with steroids or
withdrawal of amiodarone
446. Angio-oedema can be precipitated by ACE-inhibitors.
447. Baclofen toxicity leads to respiratory depression and diaphragm contraction which requires intubation
and ventilation.
448. Bisphosphonates are first line use for prevention of osteoporosis in post-menopausal women on steroids.
449. Bisphosphonates are the drug of choice in established osteoporosis
450. Botulinum toxin type A (or trade name Botox®) acts by decreasing transmission of acetylcholine across
the neuromuscular junction and can induce temporary flaccid paralysis.
451. BP control shows greater reduction in CV risk than tight glyceamic control in UKPDS
452. C1INHRP is the best-studied first-line therapy for acute episodes of angioedema in patients with HAE.
453. Carbamezapine is a recognised cause of TEN.
454. Carcinoid heart disease has a poor prognosis

102
M Y Elamin
MBBS, DTM&H, MCTM, MRCPI 1& 2
455. Cardiac biomarkers, diagnosis of myocardial infarction.
456. Co-existing conditions must be taken into account when deciding which pharmacologic intervention is
best for a patient.
457. Consider opiate toxicity with patients who have a history of chronic pain, respiratory depression and pin
point pupils.
458. CURB 65 score of 2 require oral dual antibiotic therapy.
459. Elimination of weak acids from the body can be promoted by alkalinising the urine.
460. Females over the age of 75 who have a history of fragility fracture should be started on oral
bisphosphonate therapy
461. Fenofibrate is used to treat isolated hypertriglyceridaemia, though it should not be prescribed routinely
for primary or secondary prevention of hypercholesterolaemia.
462. Fentanyl should be used in pain management in renal failure for palliative care prescribing.
463. First generation antihistamines are more sedating than the newer agents.
464. Flumenazil is used in cases of benzodiazepine toxicity.
465. Gout typically presents with pain, joint swelling, erythema and tenderness. Needle-shaped crystals
which are negatively birefringent are demonstrated on joint aspiration.
466. Heparin causes a false rise in free T4 due to displacement on assay
467. Hyperkalaemia treatment is initiated when over 6.5 or 6 with ECG changes.
468. Idiopathic hypercalciuria is often familial, the most common cause being increased gastrointestinal
absorption of calcium which causes calcium oxalate stones.
469. If INR is >8.0 and there is no bleeding, stop warfarin, give 0.5-1mg IV vitamin K and restart warfarin
once INR is < 5.0 in accordance with BJH guidelines.
470. In Adder bites tissue swelling above the ankle is indication for treatment with anti-venom.
471. In women with oopthrectomy unopposed oestrogen HRT is appropriate.
472. Indication for digbind treatment in digoxin toxicity include severe hyperkalaemia resistant to treatment,
bradyarrythmia unresponsive to atropine with cardiac compromise and tachyarrythmia associated with
cardiac compromise.
473. Individualising treatment of an elderly patient in an attempt to minimise unwanted side-effects that may
result in a reduction in quality of life
474. Interstitial nephritis may be caused by numerous aetiologies such as: infection, autoimmunity or
glomerular disease as well as hypersensitivity to medicines.
475. Li toxicity: greater than 4meq/L, consider HD

103
M Y Elamin
MBBS, DTM&H, MCTM, MRCPI 1& 2
476. Lipodystrophy, lipoatrophy and alterations in serum lipid values have been observed in patients with
human immunodeficiency virus (HIV) disease taking highly active antiretroviral therapy. Elevated serum
lipid levels have been associated with premature coronary artery disease.
477. Lithium toxicity can cause diarrhoea, but may also be precipitated by dehydration.
478. Magnesium toxicity is more common when intravenous magnesium is given in absence of
hypomagnesaemia such as pre-eclampsia and is exacerbated in the presence of renal failure.
479. Many drugs have been associated with gynecomastia, including phytoestrogens, oestrogens and drugs
with oestrogen-like properties, inhibitors of testosterone synthesis or action, and other agents with unknown
mechanisms.
480. Mechanism of action of cyanide and other poisons
481. Mercaptopurine is also inactivated by the enzyme xanthine oxidase. This enzyme is involved in the
production of uric acid and is inhibited by allopurinol.
482. Metformin is thought to inhibit vitamin B12 absorption from the ileum by affecting the calcium-
dependent membrane action.
483. Morphine acts for four to five hours and for that reason is used in cases of acute and breakthrough pain
control.
484. Necrobiosis lipoidica is commonly associated with diabetes and is painless lesion due to collagen
degeneration.
485. Omeprazole is principally dependent upon hepatic clearance and safe even with marked renal
impairment.
486. Oral prednisolone should be considered in the management of acute gout where NSAIDs are contra-
indicated
487. Pancrelipase are a collection of pancreatic enzymes used in treatment of chronic pancreatitis
488. Paracetamol concentration is unhelpful in delayed presentation of paracetamol overdose and N-
acetylcysteine should be administered without delay.
489. Patients with alcoholic liver disease are often surprisingly sensitive to opiate analgesia which should
only be used with caution.
490. Patients with Guillain-Barré syndrome have very limited therapeutic options. A short course of
intravenous immunoglobulin is one of the few treatments that have proved effective.
491. Patients with HSP and hypertension plus proteinuria should be treated with an ACE-inhibitor.
492. Peripheral or autonomic neuropathy is a feature of treatment with all vinca alkaloids and is a limiting
side effect of vincristine.
493. Phenytoin toxicity commonly causes cerebellar symptoms such as nystagmus and liver dysfunction.

104
M Y Elamin
MBBS, DTM&H, MCTM, MRCPI 1& 2
494. PPI's and H2 antagonists have not been researched enough in pregnancy and children, therefore the
safest option is antacid therapy.
495. PUVA is an effective treatment for psoriasis and may be useful where compliance with topical therapy
may be an issue
496. Quadriparesis, spinothalamic sensory loss (with sparing of posterior columns) and sphincter dysfunction
is suggestive of an anterior spinal cord syndrome.
497. Raised insulin with low c-peptide indicates exogenous insulin
498. Revise NICE guidance on type 2 diabetes and the introduction of insulin
499. Ricin's toxicity lies in its ability to inhibit protein synthesis, via the endonuclease activity of its A-chain.
500. Rifampicin may be expected to reduce the efficacy of the oral contraceptive through liver enzyme
induction.
501. Scombroid poisoning is associated with consumption of dark meat fish such as tuna and leads to rapid
onset of symptoms such as headache, dizziness and abdominal pain.
502. Seizure secondary to alcohol withdrawal should be treated with IV Pabrinex and chlordiazepoxide post-
seizure.
503. Seizures, metabolic acidosis and arrhythmias represent severe toxicity. When the QRS is widened,
sodium bicarbonate should be given until this normalises or plasma pH reaches 7.50.
504. Selective serotonin reuptake inhibitor (SSRI) discontinuation syndrome occurs following abrupt
withdrawal of SSRIs and symptoms occur after days
505. Severe tricyclic overdose with adverse features can be treated with sodium bicarbonate.
506. Somatostatin analogues improve symptoms and prognosis in carcinoid syndrome
507. Spironolactone reduces mortality in patients with NYHA class 3/4 heart failure
508. Testosterone replacement may improve bone mineral density
509. " Recombinant human erythropoeitin is effective in anaemia of chronic renal failure, in patients
receiving platinum-based chemotherapy and the prevention of anaemia in premature babies with low birth
weight. "
510. (SJS) is an immune-complex-mediated hypersensitivity complex that is a severe expression of erythema
multiforme. It typically involves the skin and mucous membranes. The drugs most closely associated with
causing SJS include antibacterials, sulfonamides, and anticonvulsants (oxicam).
511. 0.5mL adrenaline 1:1000 solution should be administered intramuscularly in anaphylaxis. Another
0.5mL can be given after 5 minutes in the absence of clinical improvement.
512. 1 mg prednisolone is equivalent to 4 mg hydrocortisone.
513. 300 mg of amiodarone made up to 20 ml with 5% dextrose given as an intravenous bolus is the drug of
choice for treating refractory ventricular fibrillation or pulseless ventricular tachycardia
105
M Y Elamin
MBBS, DTM&H, MCTM, MRCPI 1& 2
514. A history of polyuria and polydipsia, with only a marginally reduced sodium make psychogenic
polydipsia a likely diagnosis.
515. A pH of less than 7.3 is a poor prognostic factor for patients who have taken an overdose of
paracetemol.
516. A Phase III study is where a drug's efficacy and safety should be tested against a placebo.
517. A reduced rate of paracetemol absorption is caused by gastrointestinal stasis and reduced rate of gastric
emptying.
518. A serious side effect of clozapine is agranulocytosis. Leukopaenia is an indication to permanently stop
clozapine treatment.
519. ACE inhibitors and angiotensin antagonists are also capable of precipitating lithium toxicity through
reduced lithium clearance.
520. ACE inhibitors and angiotensin antagonists are capable of precipitating lithium toxicity through reduced
lithium clearance.
521. ACE inhibitors and beta blockers are first line treatments in heart failure.
522. ACE inhibitors and dihydropyridine calcium antagonists are known to increase serum lithium levels.
523. ACE inhibitors are not safe in pregnancy.
524. ACE inhibitors have low efficacy in black patients in the clinical trials of ACE inhibitors.
525. ACE inhibitors in combination with potassium sparing diuretics may result in profound hyperkalaemia.
526. ACh can stimulate postganglionic receptors to produce effects such as bradycardia and miosis.
527. Aciclovir acts through inhibition of viral deoxyribonucleic acid (DNA) polymerase but it is a pro-drug
and first requires phosphorylation by thymidine kinase.
528. Aciclovir is a synthetic purine nucleotide analogue and as such is a specific inhibitor of herpes virus
DNA polymerase.
529. Adalimumab is a human monoclonal antibody that forms complexes with TNF-a, preventing the binding
of TNF-a to its receptors.
530. Addisonian crisis should be treated with intravenous hydrocortisone in the first instance.
531. Adjustment disorder occurs within three months of an identifiable stressor and lasts six months from the
withdrawal of the stressor
532. Administration of bisoprolol to patients with chronic heart failure is associated with increases in left
ventricular function and reductions in heart rate; increases in heart rate variability are also seen.
533. Adrenalin is used for its alpha-agonist effects. IM administration is preferred because of a superior
safety profile with respect to cardiac adverse events compared with the IV route.

106
M Y Elamin
MBBS, DTM&H, MCTM, MRCPI 1& 2
534. Adrenaline is indicated in allergic reactions if there is stridor, wheeze, respiratory distress or cardiogenic
shock.
535. Affinity and intrinsic activity are determinants of potency.
536. Alcohol abuse and diabetes are the commonest causes of peripheral neuropathy in the United Kingdom.
537. Alcohol withdrawal is associated with anxiety, tachyarrhythmias and seizures and should be treated with
benzodiazepines.
538. Alcoholic hallucinosis can appear 12-24 hours after stopping alcohol and includes visual, auditory and
tactile hallucinations
539. Alcoholic hallucinosis is known to appear some 12-24 hours after the last alcoholic drink has been taken
and hallucinations may be visual, auditory or tactile in nature.
540. Alendronate inhibits osteoclast activity and this prevents the breakdown of bone and thus bone loss.
541. Aliskiren binds to the active site of the renin molecule, blocking angiotensinogen cleavage, thus,
preventing the formation of angiotensin I.
542. Allopurinol inhibits xanthine oxidase, the enzyme involved in the conversion of purines into uric acid.
543. Allopurinol works by inhibiting purine breakdown and synthesis
544. Alpha interferon at 2 million U/m2 subcutaneously three times a week for 12-18 months can be used to
salvage relapsed or refractory hairy cell leukaemia.
545. Aminoglycosides, such as gentamicin, have limited tissue distribution and are renally cleared
546. Amiodarone can cause hypothyroidism by inhibiting the peripheral conversion of T4 to T3.
547. Amiodarone has been shown to be superior in maintaining sinus rhythm following DC cardioversion of
AF.
548. Amlodipine is the most effective in reducing systolic and diastolic blood pressures over a range of doses
widely used in clinical practice.
549. Amoxicillin is an important cause of cholestatic jaundice.
550. An allergy to co-trimoxazole would be a contraindication to the use of sulphasalazine.
551. An ecstasy toxic reaction includes agitation, tachycardia, hypertension, dilated pupils and sweating.
552. An overdose of amitriptyline and diazepam may lead to QRS widening, so an ECG would be an
appropriate initial investigation on presentation.
553. Analgesia should be given as per the WHO ladder - paracetamol, NSAIDs (if not contraindicated), weak
opiates, strong opiates.
554. Analgesic misuse headache is one of the commonest causes of chronic daily headache and is caused by
analgesics such as codeine phosphate and paracetemol. Treatment is based on gradually withdrawing
analgesics.
107
M Y Elamin
MBBS, DTM&H, MCTM, MRCPI 1& 2
555. Anatomy of muscle of facial expression. Pharmacology of Botox.
556. Angiotensin II, when infused intravenously, produces an immediate rise in blood pressure being a potent
vasoconstrictor.
557. Angiotensin-converting enzyme (ACE) inhibitors and angiotensin 2 receptor blockers (A2RBs)
commonly cause rashes some of which appear to be photosensitive.
558. Anticholinergic drugs such as benzhexol remain the treatment of choice in parkinsonian tremor.
559. Anticholinergic side effects, including acute urinary retention, are common with amitriptyline.
560. Anticholinergic treatment (for example, benzhexol) is the treatment of choice for tremor predominantly
Parkinson's disease.
561. Anticoagulation with subcutaneous heparin is recommended in most guidelines for treatment of
pulmonary embolism. IV heparin and warfarin are contraindicated in pregnancy.
562. Antipsychotic medications, in particular the typical antipsychotics and risperidone, are known to elevate
prolactin levels.
563. Approximately 50% of subjects with focal segmental glomerulosclerosis (FSGS) do not respond to
steroid therapy but angiotensin-converting enzyme (ACE) inhibitors are a recognised strategy to slow the
progression of renal disease.
564. Aprepitant is a neurokinin receptor blocker used in the prevention of chemotherapy induced nausea.
565. Around 80% of patients treated with radioactive iodine will result in hypothyroidism.
566. At least 38 drugs currently in use can cause DILE. However, most cases have been associated with
procainamide (15-20%), hydralazine (7-13%), or quinidine.
567. Azathioprine can be used in pregnancy without significant risk to the fetus.
568. Based upon isolated systolic hypertension studies such as SHEP and Syst-Eur, guidelines suggest
treatment with either calcium antagonists or diuretics.
569. Bergamottin is a constituent of grapefruit juice and is metabolised by the cytochrome p450 3A4
pathway.
570. Bicarbonate therapy can increase extracellular pH only if the carbon dioxide (CO2) produced can be
removed by adequate ventilation.
571. Bisphosphonates are generally considered first line treatment for the prevention of steroid induced
osteoporosis.
572. Bisphosphonates rarely cause jaw osteonecrosis.
573. Blue-grey discolouration of the legs is associated with minocycline use.
574. Bumetanide is a loop diuretic and may cause hypokalaemia as a side effect. The potassium loss caused
by bumetanide increases the toxicity of digoxin.

108
M Y Elamin
MBBS, DTM&H, MCTM, MRCPI 1& 2
575. By crosslinking DNA in various ways, Cisplatin makes it impossible for rapidly dividing cells to
duplicate their DNA for mitosis.
576. Calcitonin-gene related peptide causes vasodilatation
577. Capecitabine is a way to deliver 5 fluorouracil orally.
578. Carbamazepine has been shown to be as effective as oxazepam in treating alcohol withdrawal
579. Caution should be exercised when taking lithium and diuretics as the latter may reduce renal clearance
of lithium and increase serum lithium concentrations.
580. Celecoxib has a lower level of anti-platelet activity than naproxen.
581. Cholestyramine is an anion exchange resin and will interfere with the absorption of fat-soluble vitamins,
including vitamin D.
582. Cholinergic features with a relative bradycardia, small pupils and increased salivation are highly
suggestive of organophosphorus poisoning which is an anticholinesterase, thus prolonging the effects of
acetylcholine.
583. Ciclosporin can cause coarse tremor, in a dose-dependent fashion.
584. Cimetidine can cause phenytoin toxicity.
585. Clarithromycin is a potent 3A4 inhibitor.
586. Clarithromycin leads to a significant increase in the theophylline AUC, precipitating toxicity with atrial
fibrillation, nausea and vomiting.
587. Clarithromycin strongly inhibits CYP3A4 - the enzyme responsible for simvastatin metabolism which
can lead to rhabdomyolysis and renal failure.
588. Clexane does not need to be monitored in pregnancy, but the dose is altered (depending on the weight of
the patient).
589. Clinical studies have shown that metformin reduces insulin resistance and have demonstrated a fall in
serum androgens, luteinising hormone and weight in PCO
590. Clinicians are recommended to check FBC fortnightly until 6 weeks after the last dose increase.
Provided it is stable, it can be checked monthly thereafter until the dose and disease is stable for one year.
591. Clopidogrel inhibits the platelet ADP receptor, which leads to decreased propensity to platelet
aggregation.
592. Clopidogrel prevents platelet aggregation through antagonism of the adenosine diphosphate (ADP)
receptor.
593. Clostridium difficile, a Gram-positive anaerobic bacterium, is the cause of pseudomembranous colitis.
594. Co-amoxiclav (Augmentin) is notorious for causing drug-induced jaundice, often with a mixed
hepatitic/cholestatic picture. A four week delay in symptoms and signs is not unusual.

109
M Y Elamin
MBBS, DTM&H, MCTM, MRCPI 1& 2
595. Co-amoxiclav is recommended as first-line treatment for all cat or human bites and other complicated
animal bites.
596. Co-amoxiclav is recommended as first-line treatment for mild-to-moderate infections following a dog,
cat, or human bite.
597. Combining opiates with partial opioid antagonists, such as buprenorphine, may lead to reduced overall
analgesic effect.
598. Consider the severity of the allergic reaction when selecting antibiotics in penicillin allergic patients.
The chance of cross reactivity with beta-lactams is 10%.
599. Contraindications to thrombolysis include intracranial neoplasm, pregnancy, aoritc dissection and active
internal bleeding.
600. Corticosteroid-related psychosis is seen within a few days of starting high dose oral or intravenous
corticosteroids, although some patients have been diagnosed with the condition up to 12 weeks or more
after commencing therapy.
601. Cranberry juice has been recognised to be responsible for a deranged INR, it being postulated that it
inhibits cytochrome p450.
602. Current AHA/EHA criteria for the treatment of SVTs in pregnancy do suggest using metoprolol (level
of evidence 1B) rather than verapamil (C), although they recommend avoiding the former in the first
trimester.
603. Cutaneous anthrax is associated with a black eschar without pus, tends to be painless and to have
widespread oedema.
604. Cyclo-oxygenase (COX) inhibits platelet aggregation through inhibition on both COX I and II.
605. Cyclosporin is a well used drug in the treatment of atopic dermatitis (AD). It is usually used as a
systemic therapy when topical treatment with steroids, calcineurin inhibitors (tacrolimus, pimecrolimus) are
ineffective. It is not recommended for use in infants and young children but should be reserved in older
children and adeloscents with very severe disease with a significant negative impact on the quality of life.
606. Delirium should be treated with short-term haloperidol or olanzapine, if verbal or behavioural
techniques are not successful.
607. Desaturation with patchy infiltration on CXR suggests a diagnosis of amiodarone-induced lung disease.
608. Diazepam, neuromuscular blockade, and intubation may all be required during the acute phase of tetanus
infection.
609. Diclofenac decreases renal lithium clearance and increases lithium concentrations.
610. Different formulations of ciclosporin have different pharmacokinetic properties and it is essential that
ciclosporin is prescribed by brand and not generically.

110
M Y Elamin
MBBS, DTM&H, MCTM, MRCPI 1& 2
611. Dihydrocodeine is an opiate analgesic and when taken in overdose, acts as a respiratory depressant
leading to reduced respiratory rate. It can cause bradycardia and hypotension in large doses. Pupillary
constriction is a diagnostic feature in opiate overdose.
612. Diltiazem, as with nearly all the calcium channel blockers, causes gravitational/ankle oedema.
613. Diltiazem, like dihydropyridine calcium channel antagonists is associated with ankle swelling due to
peripheral venous dilatation.
614. Disulfiram irreversibly inhibits the oxidation of acetaldehyde, the subsequent increased levels of which
are thought to produce the typically unpleasant side effects
615. Dose-related adverse reactions are liekly to be due to the patient having received the drug previously.
616. Drugs associated with gingival hypertrophy include calcium channel blockers, phenytoin and
cyclosporin
617. Drugs that are frequently associated with urticaria include non-steroidal anti-inflammatory drugs
(NSAIDs), penicillin, angiotensin-converting enzyme (ACE) inhibitors, thiazides and codeine
618. Drugs that can cause gynaecomastia include cimetidine, spironolactone, digoxin, methyldopa,
gonadotrophins, and cyproterone acetate.
619. Drugs with anti-muscarinic effects may cause urinary retention.
620. During pregnancy, treatment options for chlamydia infection are erthromycin or amoxicillin or
azithromycin
621. Ecstasy (3,4-methylenedioxymethamphetamine, MDMA) can cause tachycardia and hypertension.
622. EN is reported in up to 10% of patients prescribed the combined oral contraceptive pill, but is much
more rarely described in patients prescribed the progesterone only pill.
623. Erlotinib specifically targets the epidermal growth factor receptor (EGFR) tyrosine kinase (which is
required for the conformational change) and binds in a reversible fashion to the adenosine triphosphate
binding site.
624. Erythema multiforme is associated with numerous drugs, allopurinol being the most common.
625. Essential tremor is the commonest cause of head tremor, and can be reduced with propranolol.
626. Exenatide (GLP1) can be used as third line agent if HbA1c >7.5% and BMI >35, and only continue
GLP-1 mimetic therapy if the person with type 2 diabetes has had a beneficial metabolic response (a
reduction of at least 11 mmol/mol [1.0%] in HbA1c and a weight loss of at least 3% of initial body weight
in 6 months).
627. Exposure to lithium in utero is associated with Ebstein's anomaly.
628. Ezetimibe localises at the brush border of the small intestine, where it inhibits the absorption of
cholesterol from the diet.

111
M Y Elamin
MBBS, DTM&H, MCTM, MRCPI 1& 2
629. Ezetimibe reduces the absorption of cholesterol through the gut, although the exact mechanism of action
is unclear. Unlike bile acid sequestrants, ezetimibe is systemically absorbed.
630. Fatigue is a frequent side effect which typically is felt two hours and beyond after taking the drug.
631. Features of hypercortisolism include femoral avascular necrosis, weight gain, hirsutism, hypertension,
and impaired glucose tolerance.
632. Features of the amphetamine MDMA abuse include hyponatraemia, tachycardia, hyperventilation,
hyperthermia, and pyrexia.
633. Fibrates increases HDL-C by 10-15% and reduces triglycerides by 15-20%.
634. Finasteride is a 5-alpha-reductase inhibitor preventing the conversion of testosterone to the active
dihydrotestosterone (DHT).
635. Fludarabine use is associated with defective T cell function and thus increased risk of herpes virus
reactivation.
636. Fluoxetine is a CY-P450 enzyme inhibitor. Both inhibitors and inducers of CYP450 enzymes may lead
to changes in the anti-coagulant effect of warfarin.
637. For asymptomatic patients with isolated slightly elevated urate is lifestyle advice with an appropriately
reduced purine diet.
638. Fresh frozen plasma (FFP) contains more dilute clotting factors and therefore produces inferior
correction and should not be used in the management of life-threatening bleeding (unless prothrombin
complex concentrate is not available).
639. GDM should be initially treated with lifestyle modifications unless macrosomia on foetal scan.
640. Gentamicin is toxic to cochlear hair cells, and it is known to disrupt mitochondrial protein synthesis,
which leads to increased oxygen free radical generation via inducible nitric oxide synthase activity.
641. GI side effects are common with bisphosphonates, with the most severe resulting in oesophagitis
642. Gliclazide action can be potentiated predominantly by displacement of the drug from plasma proteins to
give more free (unbound) drug and interference with the hepatic metabolism of the drug.
643. GLP-1 is a hormone released by the gut in response to food intake that acts on the pancreatic beta cells
to trigger insulin secretion.
644. Glucagon is the conventional antidote for beta-blocker overdose. It reverses hypoglycaemia, and
improves myocardial contractility and heart rate by stimulating production of cyclic AMP.
645. Grapefruit juice contains bergamottin which INHIBITS CYP3A4, thereby increasing the bioavailability
of felodipine and the risk of toxicity.
646. Grapefruit juice significantly increases serum concentrations of some statins
647. Grapefruit juice significantly increases serum concentrations of some statins due to inhibiton of the
CYP3A4 mediated first pass metabolism.

112
M Y Elamin
MBBS, DTM&H, MCTM, MRCPI 1& 2
648. Haemodialysis is the treatment of choice for patients with aspirin overdose when the plasma-salicylate
concentration is greater than 700 mg/litre (5.1 mmol/litre) or in the presence of severe metabolic acidosis as
recommended within the British National Formulary (BNF) poisons section.
649. Haloperidol is contraindicated in Lewy Body dementia due to risk of severe neuroleptic reactions
650. Hepatotoxicity is a feature of antituberculous treatment. The Joint Tuberculosis Committee of the British
Thoracic Society recommend that liver function should be checked before treatment for clinical cases.
651. High viral load is associated with poor prognosis in hepatitis B related hepatic decompensation, and
entecavir or tenofovir are therefore indicated in the acute treatment.
652. Hyoscine acts as a competitive antagonist at muscarinic acetylcholine receptors.
653. Hypercalcaemia, hypermagnesaemia, digoxin, or thyrotoxicosis cause QT shortening.
654. Hypomagnesaemia may be due to diuretic use.
655. Hypotension in paracetamol overdose is unusual but is feasible in a massive overdose and usually
associated with lactic acidosis. IV fluids are the most appopriate immediate step in patients presenting with
vomiting and hypotension following overdose.
656. Ideal body weight should be used, rather than total body weight, when calculating doses of digoxin.
657. If aspartate aminotransferase (AST) or alanine aminotransferase (ALT) is between two and three times
the upper limit of normal, and the leflunomide dose is more than 10 mg daily, the dose should be reduced to
10 mg and LFTs rechecked weekly until normalised.
658. If visual symptoms are present in oatients with giant cell arteritis (GCA), intravenous
methylprednisolone should be given wihhout delay to preserve vision.
659. Imatinib is a Bcr-Abl inhibitor originally developed for CML, but now has an extended role across a
number of haematological malignancies, including gastric MALtoma.
660. Imatinib is a tyrosine kinase inhibitor which blocks the active site of the BCR-Abl protein, resulting in
reduced cell proliferation and motility, and increased apoptosis.
661. Imatinib is an inhibitor of tyrosine kinase and is used in the treatment of conditions such as chronic
myelocytic leukaemia (CML) and gastrointestinal stromal tumours (GIST).
662. Immediate treatment of anaphylaxis includes cessation of the causative agent, then oxygen, fluids and
adrenaline 0.5 mg intramuscularly.
663. In a student population where compliance may well be a problem, giving a single dose of antibiotics for
the treatment of Chlamydia is the most sensible option.
664. In cases of major bleeding, omit warfarinand consult your local haematology guidelines on the use of
Vitamin K and Berriplex.
665. In cases of suspected overdoses of tricyclic antidepressants, an electrocardiogram (ECG) would
immediately indicate if there is a risk of significant tricyclic toxicity by showing a wide QRS complex or
abnormal axis deviation.
113
M Y Elamin
MBBS, DTM&H, MCTM, MRCPI 1& 2
666. In chronic obstructive pulmonary disease (COPD) patients with HF, cardioselective ß blockers appear
safer at lower doses than higher doses or non-selective ß blockers (refs in DTB article).
667. In G-6-PD deficient patients, oxidative stress exposes interior sulfhydryl groups that are oxidised and
cannot be reduced, leading to irreversible denaturation of the haemoglobin with Heinz body formation.
668. In hepatic impairment benzodiazepines with a shorter half life are preferred.
669. In hypersensitive patients aspirin can cause angioedema, bronchospasm, and urticaria (skin rashes).
670. In patients with renal failure, dose reduction of drugs that are usually highly protein bound is required to
reduce the risk of toxicity.
671. In people with delirium tremens, offer oral lorazepam as first-line treatment. If symptoms persist or oral
medication is declined, offer parenteral lorazepam or haloperidol.
672. In poor compliance/absorption one would expect elevated TSH with normal range T4.
673. In situations involving attempted suicide, you need to assess whether the patient has capacity.
674. In TCA overdose, 50 ml of 8.4% sodium bicarbonate should be given if the pH is less than 7.1, QRS
interval is more than 0.16 s, or there are cardiac arrhythmias or hypotension.
675. In therapeutic doses N-acetyl-p-benzoquinoneimine is detoxified by conjugation with glutathione in the
liver, but once the protective intracellular glutathione stores are depleted hepatic and renal damage may
ensue.
676. In this situation, where she is anticoagulated yet has sustained a further PE, there are two potential
choices, either to consider increasing her INR target to 3-4, or considering an IVC filter.
677. In those in whom initial atropine is unsuccessful, IV glucagon is a recommended treatment for beta-
blocker overdose,
678. In women with stable inflammatory bowel disease on azathioprine, there is no reason to discontinue
therapy.
679. Increased levels of TNF-alpha are known to result in protein catabolism, weight loss, and muscle
weakness, all features of disseminated carcinoma where TNF-alpha is elevated. As such an antagonist of
TNF-alpha would be expected to result in decreased protein catabolism.
680. Incretin effect - glucose given via the gut will elicit a greater insulin response than IV.
681. Infliximab is given with methotrexate and is associated with the development of tuberculosis. Other side
effects include injection site reactions, infusion reactions, neutropenia, demyelinating disease and heart
failure.
682. Infliximab is recommended by NICE for adults with rheumatoid arthritis who have active rheumatoid
arthritis, and who have undergone trials of two disease-modifying anti-rheumatic drugs including
methotrexate (unless contraindicated).
683. Inhaled solvents are rapidly absorbed through the lungs and then quickly distributed, meaning their
effects are short-lived.
114
M Y Elamin
MBBS, DTM&H, MCTM, MRCPI 1& 2
684. Inhibition of metabolism of methanol by alcohol dehydrogenase with either ethanol or fomepizole is the
treatment of choice for methanol poisoning.
685. Initial treatment of cocaine poisoning involves intravenous administration of diazepam to control
agitation, and cooling measures for hyperthermia.
686. Initial treatment of spontaneous bacterial peritonitis is with broad-spectrum antibiotics such as
cefotaxime. Quinolones may be offered as prophylaxis in patients with cirrhosis and ascites in patients with
an ascitic fluid protein of 15g/L or less until the ascites has resolved. (NICE)
687. Insulin acts via a similar mechanism to cell surface receptors.
688. Intention to treat compares the study groups in terms of the treatment to which they were randomly
allocated, irrespective of the treatment they actually received or other trial outcomes
689. Intravenous furosemide has much better bioavailability and thus therapeutic effect, especially where
there is gut oedema.
690. It is well recognised that carbamazepine is a P450 enzyme inducer but it is less well appreciated that it
causes auto-induction and so would require increase in dose to maintain the same therapeutic concentration.
691. IV fluids with bicarbonate is used to correct the metabolic acidosis
692. Ivabradine is a novel agent which blocks the If channel in the sinoatrial node, resulting in a reduced
heart rate and a subsequent anti-anginal effect.
693. IVC filters are indicated in the treatment of emboli where there are contraindications to systemic
anticoagulation.
694. Lactulose is used in patients with cirrhosis/hepatic encephalopathy to limit the proliferation of ammonia-
forming gut organisms and increase the clearance of protein load in the gut.
695. Lipodystrophy, lipoatrophy, and alterations in serum lipid values have been observed in patients with
human immunodeficiency virus (HIV) disease who are taking highly active antiretroviral therapy.
696. Lithium can cause diabetes insipidus.
697. Lithium can cause nephrogenic DI.
698. Lithium toxicity occurs at levels above 1.4 mmol/L and can result in anorexia, diarrhoea, vomiting,
ataxia, nystagmus, dysarthria, confusion and seizures.
699. Low dose corticosteroids have a role in the management of sepsis
700. Magnesium has been shown significantly to reduce maternal mortality in eclampsia and a favourable
outcome may also be expected in pre-eclampsia.
701. Magnesium sulphate (MgSO4) is the most effective agent for the treatment of eclampsia and
prophylaxis in severe pre-eclampsia.

115
M Y Elamin
MBBS, DTM&H, MCTM, MRCPI 1& 2
702. Malignant hyperpyrexia (MH) is characterised by increased temperature and muscle rigidity during
anaesthesia, which results from abnormal skeletal muscle contraction and increased metabolism. Muscle
biopsies may appear histologically normal.
703. Malignant hypertension is associated with papilloedema, convulsions, and pulmonary oedema and IV
nitroprusside is the treatment of choice.
704. Malnourishment is an increased risk factor for hepatoxicity from paracetemol oversdose.
705. Management of typhoid fever
706. Medication can cause hyperkalaemia via various mechanisms.
707. Metformin can commonly cause gastrointestinal disturbances.
708. Metformin can lead to reduced B12 absorption.
709. Metformin is an insulin sensitiser and although its actions are not fully understood its main role appears
to be through inhibition of hepatic gluconeogenesis.
710. Metformin is contraindicated immediately following MI due to tissue hypoxia which is a risk factor for
the development of lactic acidosis.
711. Metformin should be stopped if creatinine is above 150 µmol/L or eGFR less than 30 mL/1.73m2
712. Metformin should not be prescribed if creatinine >150 µmol/L or eGFR <30 mmol/min/L
713. Methotrexate is a chemotherapeutic agent; it acts through inhibition of dihydrofolate reductase thus
depleting folate concentrations.
714. Methotrexate is a folic acid antagonist which can result in multi-organ failure in overdose. Folinic acid
is the antidote and should be given intravenously as soon as possible, regardless of the liver function tests.
715. Methotrexate is an inhibitor of dihydrofolate reductase.
716. Methotrexate is teratogenic and, according to the British National Formulary (BNF), the manufacturers
advise effective contraception during, and for at least three months, after stopping methotrexate (both males
and females).
717. Methotrexate is teratogenic and, according to the British National Formulary (BNF), the manufacturers
advise effective contraception during, and for at least three months, after stopping methotrexate.
718. Methotrexate would be the only correct treatment for someone with erythrodermic psoriasis.
719. Methyldopa is effective and safe for both mother and baby in pregnancy
720. Methyldopa should be avoided in patients with deranged LFT.
721. Metoclopramide can cause hyperprolactinaemia.
722. Metoclopramide is a dopamine receptor antagonist that can induce parkinsonism.
723. Metoclopramide results in hyperprolactinaemia by blocking dopamine and therefore stimulating
prolactin release.
116
M Y Elamin
MBBS, DTM&H, MCTM, MRCPI 1& 2
724. Minocycline can account for polyarthritis and antinuclear antibody (ANA), due to its ability to cause
drug-induced lupus erythematosus.
725. Morphine undergoes extensive first pass metabolism in the liver. However, it is the plasma half-life
which defines the timing of the doses.
726. Moxonidine is a centrally acting anti-hypertensive which is a selective agonist at the imidazoline-1
receptor in the medulla oblongata.
727. N-acetylcysteine intravenously is the most appropriae treatment for paracetemol overdose. The need to
consider transplant is based on the Kings College Criteria.
728. N-Acetylcysteine is an antidote for paracetamol overdosage.
729. N-acetylcysteine should be given to all patients who present with staggered paracetamol overdoses.
730. Nephrotoxicity is the most frequent side effect of ciclosporin.
731. Neuroleptic malignant syndrome is a potentially fatal idiosyncratic reaction to neuroleptic drugs that
antagonise the central dopamine D2 receptors or result in dopamine depletion.
732. NICE guidance on Prophylaxis against infective endocarditis (CG64) suggests that no prophylaxis is
required for patients with biscupid aortic valve.
733. NICE guidance on Prophylaxis against infective endocarditis (CG64) suggests that prophylaxis is not
required during dental procedures.
734. NICE guidelines advocate use of ACEi or ARB in diabetics with hypertension.
735. NICE guidelines state that an oral triptan and an NSAID or paracetamol should be used for the acute
treatment of migraine.
736. Nicorandil is a potent potassium channel activator.
737. Nicotinic acid may increase blood glucose levels, although this effect is minimal for most diabetics.
738. Non-steroidal anti-inflammatory drugs (NSAIDs) may cause a deterioration in renal function and would
be associated with an increased risk of bleeding in the elderly.
739. Octreotide is a somatostatin analogue that directly inhibits growth hormone secretion.
740. Oculogyric crisis should be treated with procyclidine (usually IV or IM) or benztropine.
741. Olanzapine can cause significant weight gain, and the management of this should be carefully
considered prior to treatment initiation.
742. Omacor increases peroxisomomal beta-oxidation of fatty acids in the liver.
743. Ondansetron is a serotonin 5-HT3 receptor antagonist used mainly to treat nausea and vomiting
following chemotherapy.
744. Ondansteron is a selective 5-HT3 antagonist both centrally and peripherally and as such is a potent
antiemetic.
117
M Y Elamin
MBBS, DTM&H, MCTM, MRCPI 1& 2
745. Optic neuritis is described as being associted with diclofenac therapy but rarely.
746. Oral bioavailability of THC, whether given in the pure form or as THC in marijuana, is low and
extremely variable, ranging between 5% and 20%, with effects occurring 0.5-3 hours later.
747. Oseltamivir (Tamiflu) may be used in the prophylactic treatment of healthcare workers during flu
epidemics.
748. Ototoxicity is associated with vancomycin, and is more likely in patients with high plasma
concentrations, renal impairment or pre-existing hearing loss.
749. P. jiroveci can rarely present in extra pulmonary locations.
750. Pancytopenia is commonly seen in patients undergoing azathioprine therapy.
751. Paracetamol absorption is reduced during migraine attacks and reduced absorption is associated with
increased nausea.
752. Paracetamol and/or topical NSAIDs (for knee or hand OA) should be offered before considering oral
NSAIDs. Rubefacients, intra-articular hyaluronan, electro-acupuncture and chondroitin and glucosamine
products are not recommended.
753. Paracetamol is conjugated to glucuronic acid and sulphate under normal conditions.
754. Paracetamol is predominantly metabolised to glucuronide and sulphate conjugates which are excreted in
the urine.
755. Patients prescribed Antabuse (disulfiram) should be warned of serious reactions if using alcohol based
perfumes.
756. Patients receiving phenytoin may develop pseudolymphoma or, rarely, malignant lymphoma and
mycosis-fungoides-like lesions.
757. Patients who receive clarithromycin (or other macrolides) whilst on warfarin should have their INR
closely monitored, as the dose may need to be reduced to avoid an unsafe INR.
758. Patients who undergoe successful cardioversion for idiopathic AF need to remain on warfarin as there is
risk of further thromboembolism
759. Patients with a history of drug abuse are at high risk of developing seizures, which occur
characteristically within 48 hours of alcohol discontinuation, usually in the morning.
760. Patients with aspirin sensitivity can present with either mucosal reactions (the aspirin triad of nasal
polyposis, sinusitis, and asthma) or cutaneous reactions (urticaria or anaphylaxis).
761. Phenytoin can lead to folate deficiency and present as a macocytic anaemia.
762. Phenytoin is a potent enzyme inducer of the cytochrome P450 system; as such it reduces plasma levels
of agents which undergo hepatic metabolism, including sex steroids given in the oral contraceptive pill.
763. Phenytoin is well known to cause neurological side effects such as peripheral sensory neuropathy and
cerebellar ataxia.

118
M Y Elamin
MBBS, DTM&H, MCTM, MRCPI 1& 2
764. Photosensitivity is seen very commonly in those patients who are prescribed amiodarone therapy. It is
distinct from the slate grey skin discolouration which can occur with prolonged amiodarone use and can be
prevented by using a total sun block preparation.
765. Pioglitazone can result in fluid retention of unknown aetiology which may cause a mild dilutional
anaemia (haemoglobin typically falls by 10 to 20 g/L) and ankle oedema. It is contraindicated in congestive
heart failure
766. Pioglitazone is contra-indicated in heart failure.
767. Pioglitazone should not be used in patients with either a history or a high risk of developing of bladder
cancer.
768. Polycystic ovarian syndrome is recognised to be a condition associated with increased insulin resistance
and metformin is effective through improvements in insulin sensitivity resulting in ovulation and
improvements in hormonal perturbations.
769. Porphyric attacks may be precipitated by drugs including phenobarbitone, sulphonylureas, and
oestrogens.
770. Prochlorperazine is first line in management of psychotic symptoms during acute attacks of acute
intermittent porphyria.
771. Prophylaxis not now required for simple instrumentation in common cardiac abnormalities
772. Quinolones have been associated with tendon rupture
773. Quinolones should be used with caution in patients with a history of epilepsy or conditions that
predispose to seizures.
774. Randomised controlled trials have shown that human immunoglobulins and plasma exchange improve
outcome in Guillain-Barré Syndrome (GBS).
775. Reactive arthritis classically presents with the combination of urethritis, conjunctivitis and arthritis, with
NSAIDs the initial treatment of choice.
776. Recognised side effects of phenytoin include lymphadenopathy.
777. Refeeding syndrome is a potentially fatal consequence of a glucose load in patients who have previously
been malnourished. It results in fluid retention, hypokalaemia, hypomagnesaemia and hypophosphataemia.
778. Regimes comprising chlorambucil or cyclophosphamide, either alone or with steroids, are more
effective than symptomatic treatment for membranous nephropathy.
779. Renal clearance is the main factor that determines the choice of loading dose of digoxin.
780. Rifampicin enhances the metabolism of oral contraceptive pills, decreasing its effectiveness and
resulting in pill failure.
781. Rifampicin is known to induce the action of some enzymes, therefore increasing the metabolism of
warfarin and so reducing its anticoagulant effect.

119
M Y Elamin
MBBS, DTM&H, MCTM, MRCPI 1& 2
782. Risperidone is the only antipsychotic licensed for treating challenging behaviour in dementia.
Antipsychotics are associated with increased mortality in patients with dementia and should be avoided
unless absolutely necessary.
783. Ropinirole and rotigotine are some of the effective drugs in the treatments of restless legs, whilst
gabapentin may be effective as a second line alternative
784. Sarin is an organophosphorus and pralidoxime reactivates the acetyl cholinesterase enzyme. It should be
used in the first few hours.
785. Selegiline is a MAO-B inhibitor.
786. Several drugs, including chlormethiazole, can cause hypersalivation.
787. Several mechanisms resulting in heparin resistance have been identified, including antithrombin
deficiency, increased heparin clearance, elevated heparin-binding proteins, and elevated factor VIII and
fibrinogen levels.
788. Severe cases of lithium overdose require haemodialysis.
789. Side effects of drugs for TB
790. Side-effects of thiazides and related diuretics include mild gastro-intestinal disturbances, postural
hypotension, altered plasma-lipid concentrations, metabolic and electrolyte disturbances including
hypokalaemia, hyponatraemia, hypomagnesaemia, hypercalcaemia, hyperglycaemia, hypochloraemic
alkalosis, hyperuricaemia, and gout.
791. SIGN guidelines point out that in children without significant systemic features of infection such as
fever and generalised illness apart from pain, there is no significant benefit of antibiotics.
792. Sildenafil is a PDE-5 inhibitor, but at high dose it inhibits the activity of PDE-6, which is essential for
the functioning of retinal rods cells. Inhibition of the enzyme leads to patients reporting blue tinged vision,
particularly in low light conditions.
793. Sildenafil leads to significant hypotension with nitrates and drugs such as nicorandil which act as nitrate
donors. As such it is contraindicated for use with these agents.
794. Simple bisphosphonates inhibit bone resorption through their actions on osteoclasts.
795. Single fibre electromyography (EMG) is the most sensitive test for myasthenia gravis. It simultaneously
records the variability in potentials of two muscle fibres innervated by an individual axon: jitter.
796. Sitagliptin works by inhibiting the DPP-4 enzyme that destroys GLP and GIP hormones, allowing both
to function more effectively.
797. Six monthly denosumab is effective in managing patients with severe osteoporosis intolerant or with
contraindicaitons to Alendronate or Zolendronic acid.
798. Slow acetylators often experience toxicity from drugs such as isoniazid, sulfonamides, procainamide,
and hydralazine, whereas fast acetylators may not respond to isoniazid and hydralazine in the management
of tuberculosis and hypertension.

120
M Y Elamin
MBBS, DTM&H, MCTM, MRCPI 1& 2
799. Sodium valproate can occasionally have an idiosyncratic response leading to severe or even fatal hepatic
toxicity.
800. Sodium valproate is associated with an increased risk of neural tube defects when used in pregnancy
801. Sodium valproate is associated with weight gain, tremor, hair loss, POD and teratogenicity.
802. Sodium valproate, lamotrigine, and topiramate are the treatments of choice for absences, generalized
tonic clonic seizures and myoclonus in primary generalized epilepsy.
803. Sotalol is a cause of drug-induced long QT.
804. Spironolactone (aldosterone receptor antagonist) is the optimal add on medication for advanced heart
failure symptoms to prevent cardiac muscle remoddeling.
805. Spironolactone causes gynaecomastia can block androgen production by inhibiting enzymes in the
testosterone synthetic pathway, and can also block recceptor binding of testosterone and
dihydrotestosterone.
806. SSRIs are a cause of SIADH.
807. St John's wort is a liver enzyme inducer that may reduce the efficacy of warfarin and digoxin
808. St John's wort is a potent CYP-450 inducer, and use can lead to rapid decreases in sex steroids
administered as the combined pill.
809. St John's wort is an enzyme inducer and therefore can increase the metabolism of, for example, warfarin,
and make it less effective
810. Statin-associated myopathy may be exacerbated by the co-prescription of other drugs such as calcium
channel blockers, macrolide antibiotics, fibrates, amiodarone and grapefruit juice.
811. Statins act by competitively inhibiting HMG-CoA reductase, the first committed enzyme of the HMG-
CoA reductase pathway.
812. Steroids have been shown to improve prognosis in severe sepsis, and should be administered within 24
hours.
813. Strep. pyogenes and Staph. aureus are the common organisms found causing cellulitis.
814. Studies such as CIBIS, MERIT HF, and COPERNICUS clearly demonstrate the advantage of beta
blockers even with severe heart failure.
815. Studies such as CIBIS-II and MERIT-HF reveal that beta blockers significantly reduce morbidity and
mortality in heart failure.
816. Supportive measures are first line for beta-blocker overdose.
817. TB meningitis requires 2 months of four-drug therapy and 10 months on two-drug therapy, to a total of
12 months.
818. Tetracyclines should not be given to children under the age of 12 or to pregnant or breastfeeding
women.
121
M Y Elamin
MBBS, DTM&H, MCTM, MRCPI 1& 2
819. The approximate equivalent glucocorticoid action of prednisolone to hydrocortisone is 4:1.
820. The best management strategy for patients with obesity-related hypertension is lifestyle advice.
821. The Buteyko technique controls chronic hyperventilation, as such patients perceive less symptoms of
shortness of breath, and their use of short acting bronchodilators is reduced.
822. The combination of venous ulceration, iritis, mouth ulceration and arthritis is typical of Behçet's disease.
823. The combined oral contraceptive pill is a safe contraceptive choice, but embolic risks have to be
considered.
824. The Committee on Safety of Medicines (CSM) have reported that hyponatraemia is associated with all
types of antidepressants.
825. The commonest form of amiodarone induced hyperthyroidism is caused by thyroid cell destruction, and
is best treated with prednisolone.
826. The effect of fibrates on the metabolism of triglyceride-rich lipoproteins is due to a PPAR-alpha-
dependent stimulation of lipoprotein lipase and of apolipoprotein (apo)A-V and to an inhibition of apoC-III
expression.
827. The enzyme ACE is also responsible for the metabolism of bradykinin in mast cells.
828. The equivalent ratio of prednisolone to hydrocortisone is 1:4
829. The GLP-1 mimetics are associated with weight loss and are a good choice in patients who are
significantly overweight.
830. The HLA B 1502 subtype, most prevalent in Chinese and Thai individuals, is associated with the
development of Stevens Johnson syndrome on commencement of carbamazepine.
831. The major difference between capecitabine and 5-FU is that capecitabine is an oral prodrug of 5-FU.
832. The major toxic effects of quinine are on the nervous system, in particular the optic and auditory nerve.
833. The majority of research evidence is for cognitive behavioural therapy (CBT) and/or graded exercise
therapy and these should be offered to all people with mild or moderate CFS.
834. The mechanism by which it has its effect is not fully understood, but it is thought that low magnesium
levels in bronchial smooth muscle favour bronchoconstriction.
835. The mechanism of action of bisphosphonates involves the inhibition of osteoclasts
836. The most accepted indication for HRT is the relief of the symptoms of menopause.
837. The most appropriate agent in CCF with evidence to support its use for reducing morbidity and mortality
in failure would be the addition of a beta blocker
838. The most appropriate initial therapy for a patient who has a high risk of thrombo-embolic stroke is
anticoagulation with warfarin maintaining an international normalised ratio (INR) between 2-3.

122
M Y Elamin
MBBS, DTM&H, MCTM, MRCPI 1& 2
839. The most appropriate strategy for secondary prevention of a heart attack would involve further blood
pressure reduction with an angiotensin converting enzyme inhibitor (ACEi), which would not only reduce
cardiovascular (CV) risk as suggested by the HOPE study, but would also reduce microvascular risk as
revealed by UKPDS.
840. The most appropriate treatment for legionellosis is clarithromycin.
841. The Number Needed to Treat (NNT) is the number of patients you need to treat to prevent one
additional bad outcome
842. The optimal antihypertensive therapy for black Afro-Caribbean patients (if a CCB is not suitable, for
example because of oedema) will be a thiazide-like diuretic according to current NICE guidance, 2011.
843. The oral contraceptive and antipsychotics are possible causes of thromboembolism.
844. The phosphodiesterase 5 inhibitors should be avoided in patients taking nitrates or nicorandil. This is
due to vasodilatation potentially causing hypotension and precipitating a myocardial event.
845. The purpose of combining buprenorphine and naloxone for the treatment of opioid action is that it
prevents addicts from achieving a high if they try to create an IV preparation from the tablets for injection.
846. The right coronary artery gives off branches to the sinus and AV node, therefore disease within this
vessel can result in bradyarrhythmias.
847. The shorter chain low molecular weight (LMW) fractions of heparin inhibit activated factor X.
848. The symptoms of hyperthyroidism are usually a result of increased beta-adrenergic tone, and
propranolol is therefore an effective treatment.
849. The use of antidepressants has been linked with suicidal thoughts and behaviour. Where necessary
patients should be monitored for suicidal behaviour, self harm or hostility, particularly at the beginning of
treatment or when the dose is changed.
850. The whole class of quinolone antibiotics is associated with case reports of tendon rupture.
851. There is an increased risk of neural tube defects associated with valproate.
852. There is an increased risk of strokes in patients with atrial fibrillation and hence with the given
symptoms formal anticoagulation with warfarin should be considered.
853. There is evidence to support the use of oseltamivir as a prophylactic agent against influenza
854. There is no evidence that increasing a dose of digoxin above 62.5 µg in a patient in sinus rhythm would
have any added benefit in CCF.
855. Thiazide diuretics act on the cortical diluting segment of the nephron, predominantly in the early distal
tubule.
856. Thiazide diuretics act on the distal convoluted tubule of the nephron to prevent sodium and chloride
reabsorption. Of the above diuretics, metolazone has a thiazide-like action.
857. Thiazide diuretics are associated with hypercalcaemia.

123
M Y Elamin
MBBS, DTM&H, MCTM, MRCPI 1& 2
858. Thiazides block sodium reabsorption in the proximal segment of the distal convoluted tubule.
859. Three drugs to remember that are associated with gingival hyperplasia are Phenytoin Ciclosporin
Nifedipine.
860. Thrombolysis with alteplase can be given up to 4.5 from the known onset of symptoms as long as a
haemorrhagic stroke is excluded and there are no contraindications to thrombolysis.
861. Toxic epidermal necrolysis (TEN) can be caused by carbamazepine therapy. TEN is a severe
mucocutaneous exfoliative disease with an uncertain pathogenesis and a high mortality rate.
862. TPMT activity is a useful screening test with respect to reducing risks of toxicity from azathioprine, and
is recommended prior to treatment initiation.
863. Trastuzamab (Herceptin) may be used to treat HER-2 positive breast cancers
864. Treatment of community-acquired pneumonia is guided by the CURB-65 severity score.
865. Treatment of knee OA.
866. Treatment with a dihydropyridine short acting calcium antagonist nifedipine in moderate to high doses is
associated with increased cardiovascular mortality in patients with acute myorcardial infarction.
867. Trials of N-acetylcysteine suggest that the incidence of hepatotoxicity is 1% in those treated within eight
hours as opposed to 46% in those treated after 16 hours.
868. Trimethoprim can cause depression of haematopoiesis and lead to pancytopaenia on bone marrow
aspiration.
869. Typical side effects of cisplatin include marrow toxicity, ototoxicity and peripheral neuropathy
870. Unlike the tricyclic antidepressants, many of the SSRIs are safe in overdose and cause very few effects.
871. Up to 12% of patients taking sodium valproate report significant hair loss in clinical trials.
872. Upper urinary tract infection is one area where resistance to antimicrobials is increasing. For this reason
trimethoprim and amoxicillin are not recommended for treatment of upper urinary tract infections, and even
resistance to quinolones such as ciprofloxacin is now beginning to prove problematic.
873. Urinary retention is a known side effect of amitriptyline and similar tricyclic antidepressants.
874. Urticaria is a common condition and usually responds very well to systemic antihistamines which are
the correct first line treatment.
875. Vancomycin-resistant enterococci may be found in healthy community volunteers not recently
hospitalised .
876. Varenicline is a non-nicotine drug that is a partial agonist of the alpha4beta2 nicotinic receptor
877. Varenicline is recommended as a possible treatment to help smokers who have said they want to stop
smoking
878. Very high osmolality with normal anion gap and acid base balance reflects the presence of ethanol.
124
M Y Elamin
MBBS, DTM&H, MCTM, MRCPI 1& 2
879. Viagra is contraindicated in patients taking nitrates
880. Warfarin antagonises vitamin K.
881. Warfarin anticoagulation titrated to an INR of 2.0-3.0 is recommended for the average patient with a
CHA2DS2-VASc score of 2 unless contraindicated
882. Warfarin inhibits production of factors II, VII, IX and X, and it does this by restricting the activity and
availability of vitamin K.
883. Warfarin inhibits vitamin K epoxide reductase.
884. Warfarin interacts with cranberry juice. Simvastatin interacts with grapefruit juice.
885. Warfarin is generally avoided in pregnancy.
886. Weight loss is more likely to reduce insulin resistance than metformin
887. Where the FEV1/FVC is greater than 0.7, referral for specialist advice is recommended if significant
chest disease is suspected.
888. Whilst angio-oedema may be hereditary or idiopathic, it is also associated with use of both ACE
inhibitors and angiotensin receptor blockers (ARB)
889. Whilst clopidogrel does not have a clinically significant effect on the pharmacokinetics of warfarin it
does increase the bleeding risk when both are used concurrently.
890. Wilson's disease is associated with parkinsonian features due to copper deposits within the substantia
nigra.
891. Wolff-Parkinson-White syndrome is suggested by a delta wave on ECG.
892. Yellow fever vaccines should be avoided in patients with severe egg allergy.

125
M Y Elamin
MBBS, DTM&H, MCTM, MRCPI 1& 2
CLINICAL SCIENCES
1. A 25-year-old man is investigated for lens dislocation. He is noted to be tall with arachnodactylyl. His past
medical history includes deep vein thrombosis and learning difficulties - homocystinuria
2. A 6-month-old boy is investigated for developmental delay and infantile spasms. On examination he has fair
hair and blue eyes. There is some eczema on the torso and he is noted to have a slightly 'musty' odour -
phenylketonuria
3. A 6-year-old girl is investigated for precocious puberty. On examination she is noted to have cafe-au-lait spots
predominately on one side of her body and a skull deformity - McCune-Albright syndrome
4. A baby is born with microcephaly, small eyes, low-set ears, cleft lip and polydactyly - Patau syndrome
5. A baby is born with micrognathia, low-set ears, rocker bottom feet and overlapping of fingers - Edward's
syndrome
6. A baby is noted to have micrognathia and a cleft palate. He is placed prone due to upper airway obstruction.
There is no family history of similar problems - Pierre-Robin syndrome
7. A baby is reviewed shortly after birth as a fetal ultrasound showed a cystic hygroma. On examination the baby
has swollen hands and feet and echocardiography shows a bicuspid aortic valve - Turner's syndrome
8. A boy is noted to have reduced tone, hypogonadism and obesity - Prader-Willi syndrome
9. A boy presents with arm problems after falling from a tree, during which he caught a branch to save himself.
On examination he has a claw hand with a hyperextended wrist - lower trunk of the brachial plexus (C8, T1)
10. A boy with learning difficulties is noted to be extremely friendly and extroverted. He has short for his age and
has supravalvular aortic stenosis - William's syndrome
11. A 'down and out' eye - CN III
12. A newborn's arm is noted to be hanging loosely following a difficult forceps delivery. The arm is adducted and
internally rotated, elbow extended - upper trunk of the brachial plexus (C5,C6)
13. A patient cannot adduct his thigh following an anterior hip dislocation - obturator nerve
14. A patient complains of pins/needles and pain around the thumb and index finger which is worse at night -
median nerve
15. A patient develops a 'claw' hand following a medial epicondyle fracture - ulnar nerve
16. A patient develops foot drop following a fibular neck fracture - common peroneal nerve
17. A patient develops loss of knee extension and sensory loss to the anterior and medial aspect of the thigh
following a stab injury - femoral nerve
18. A patient develops tingling and numbness on the outer aspect of the thigh - lateral cutaneous nerve of the thigh
19. A patient has difficult abducting their arm following a humeral neck fracture - axillary nerve
20. A patient is found to have a positive Trendelenburg sign - superior gluteal nerve
21. A patient is noted to have impaired hip extension and lateral rotation. He has difficulty rising from a seat and
can't climb stairs - inferior gluteal nerve
22. A patient is noted to have wrist drop following a humeral midshaft fracture - radial nerve
126
M Y Elamin
MBBS, DTM&H, MCTM, MRCPI 1& 2
23. A patient loses the ability to plantarflex and invert his foot - tibial nerve
24. A woman develops a winged scapula following a mastectomy - long thoracic nerve
25. A young boy is noted to have a webbed neck, pulmonary stenosis, ptosis and short stature. The karyotype is
normal - Noonan syndrome
26. A young boy is noted to have learning difficulties, macrocephaly, large ears and macro-orchidism - fragile X
27. Abducens nerve passes through the superior orbital fissure
28. ABL - chronic myeloid leukaemia
29. ABL - oncogene
30. Absolute risk reduction - 1 / number needed to treat
31. Absolute risk reduction - the difference between the event rate in the intervention group and that in the control
group
32. Achondroplasia - mutation in the fibroblast growth factor receptor 3 (FGFR-3) gene
33. Achondroplasia is autosomal dominant
34. Acute anterior uveitis - HLA-B27
35. Acute intermittent porphyria is autosomal dominant
36. Acute tubular necrosis - a recent history of aminoglycoside use
37. Acute tubular necrosis - a recent history of rhabdomyolysis
38. Acute tubular necrosis - exposure to lead
39. Acute tubular necrosis - exposure to radiocontrast agents
40. Acute tubular necrosis - granular ('muddy brown') casts
41. Acute tubular necrosis - loss of nuclei and detachment of tubular cells from the basement membrane
42. Acute tubular necrosis - occlusion of the tubular lumen by casts and cell debris
43. Adult polycystic disease is autosomal dominant
44. Albinism is autosomal recessive
45. Aldosterone - zona glomerulosa of adrenal cortex
46. Alkaptonuria - deficiency of homogentisic dioxygenase
47. Alport's syndrome is x-linked dominant
48. Alternative hypothesis - a hypothesis that there is a difference in effectiveness between two treatments
49. Amiodarone in breastfeeding must be avoided
50. An infant is noted to have short limbs associated with a large head and frontal bossing - achondroplasia
51. Androgen insensitivity syndrome is x-linked recessive
52. Ankylosing spondylitis - HLA-B27
53. Antipsychotics in breastfeeding is considered safe to use
54. Antithrombin III deficiency is autosomal dominant
55. APC - colorectal cancer
56. APC - tumour suppressor gene
127
M Y Elamin
MBBS, DTM&H, MCTM, MRCPI 1& 2
57. Aspirin in breastfeeding must be avoided
58. Ataxia telangiectasia is autosomal recessive
59. Bartter's syndrome - metabolic alkalosis
60. BCL-2 - follicular lymphoma
61. BCL-2 - oncogene
62. Becker muscular dystrophy is x-linked recessive
63. Behcet's disease - HLA-B51
64. Benzodiazepines in breastfeeding must be avoided
65. Beta-blockers in breastfeeding is considered safe to use
66. bias: The situation in a trial where one outcome is systematically favoured
67. Big toe - L5
68. Binomial variable - data may take one of two values (for example gender)
69. BRCA1 - breast and ovarian cancer
70. BRCA1 - tumour suppressor gene
71. BRCA2 - breast and ovarian cancer
72. BRCA2 - tumour suppressor gene
73. Bronchoconstriction - LTC4, LTD4, LTE4
74. C1 inhibitor (C1-INH) protein deficiency - hereditary angioedema
75. C1q, C1rs, C2, C4 deficiency - immune complex diseases
76. C3 deficiency - recurrent bacterial infections
77. C5 deficiency - Leiner disease
78. C5-9 deficiency - Neisseria meningitidis infection
79. Cafe-au-lait spots, precocious puberty, short stature - McCune-Albright syndrome
80. Carbamazepine in breastfeeding is considered safe to use
81. Carbenoxolone - metabolic alkalosis
82. Carbimazole in breastfeeding must be avoided
83. Case-control study - compares a group with a disease to a group without, looking at past exposure to a possible
causal agent for the condition
84. Case-control study - observational and retrospective
85. Case-control study - the usual outcome measure is the odds ratio
86. CD8 - co-receptor for MHC class I
87. Cephalosporins in breastfeeding is considered safe to use
88. Cerebral calcification, chorioretinitis and hydrocephalus - toxoplasmosis
89. Chi-squared test - non-parametric test used to compare proportions or percentages
90. Ciprofloxacin in breastfeeding must be avoided
91. C-MYC - Burkitt's lymphoma
128
M Y Elamin
MBBS, DTM&H, MCTM, MRCPI 1& 2
92. C-MYC - oncogene
93. Coeliac disease - HLA-DQ2/DQ8
94. Cohort study - observational and prospective
95. Cohort study - the usual outcome measure is the relative risk
96. Cohort study - two groups are selected according to their exposure to a particular agent and followed up to see
how many develop a disease or other outcome
97. Colour blindness is x-linked recessive
98. Confidence interval - a range of values that has a specified probability of including the true value of the variable
99. Confounding - a variable which correlates with other variables within a study leading to spurious results
100. Congenital adrenal hyperplasia is autosomal recessive
101. Continuous variable - data can take any value with certain range (for example weight)
102. Correlation coeffecicient - a measure of strength of the linear relationship between two variables
103. Cortisol - zona fasciculata of adrenal cortex
104. Cross-sectional study - collects data to measure disease prevalence
105. Cushing's syndrome - metabolic alkalosis
106. Cystic fibrosis is autosomal recessive
107. Cystinuria is autosomal recessive
108. Decreased gastric acid secretion - PGE2
109. Decreased platelet aggregation - PGI2
110. Decreased uterine tone - PGI2
111. Dehydroepiandrosterone - zona reticularis of adrenal cortex
112. Dermatitis herpetiformis - HLA-DR3
113. Digoxin in breastfeeding is considered safe to use
114. Dilated cardiomyopathy, malnutrition/alcoholism - thiamine deficiency
115. Discrete variable - observed values are confined to a certain values, usually a finite number of whole numbers
(for example the number of asthma exacerbations in a year)
116. Diuretics - metabolic alkalosis
117. Down syndrome - acute lymphoblastic leukaemia
118. Down syndrome - Alzheimer's disease
119. Down syndrome - atlantoaxial instability
120. Down syndrome - Brushfield spots in iris
121. Down syndrome - duodenal atresia
122. Down syndrome - endocardial cushion defect
123. Down syndrome - epicanthic folds
124. Down syndrome - flat occiput
125. Down syndrome - Hirschsprung's disease
129
M Y Elamin
MBBS, DTM&H, MCTM, MRCPI 1& 2
126. Down syndrome - hypothyroidism
127. Down syndrome - hypotonia
128. Down syndrome - isolated patent ductus arteriosus
129. Down syndrome - low-set ears
130. Down syndrome - pronounced 'sandal gap' between big and first toe
131. Down syndrome - recurrent chest infections
132. Down syndrome - secundum atrial septal defect
133. Down syndrome - short stature
134. Down syndrome - single palmar crease
135. Down syndrome - subfertility/infertility
136. Down syndrome - tetralogy of Fallot
137. Down syndrome - upslanting palpebral fissures
138. Down syndrome - ventricular septal defect
139. Duchenne muscular dystrophy is x-linked recessive
140. Early repolarisation - efflux of potassium
141. Edward's syndrome (trisomy 18) - low-set ears
142. Edward's syndrome (trisomy 18) - micrognathia
143. Edward's syndrome (trisomy 18) - overlapping of fingers
144. Edward's syndrome (trisomy 18) - rocker bottom feet
145. Effectiveness - clinical trial phase III
146. Efferent limb of the pupillary light reflex - CN III
147. Efficacy - clinical trial phase IIb
148. Ehlers-Danlos syndrome is autosomal dominant
149. ELISA - an example includes the initial HIV test
150. ELISA - detect antigens and antibodies using a colour changing enzyme
151. Erb-B2 (HER2/neu) - breast and ovarian cancer
152. Erb-B2 (HER2/neu) - oncogene
153. Expectation bias (Pygmalion effect) - observers may subconsciously measure or report data in a way that
favours the expected study outcome.
154. Fabry's disease is x-linked recessive
155. Facial nerve passes through the internal acoustic meatus
156. Familial adenomatous polyposis is autosomal dominant
157. Familial Mediterranean Fever is autosomal recessive
158. Fanconi anaemia is autosomal recessive
159. Final repolarisation - efflux of potassium
160. Flaccid paralysis of upper + lower face - CN VII
130
M Y Elamin
MBBS, DTM&H, MCTM, MRCPI 1& 2
161. Follicular lymphoma - BCL-2
162. Follicular lymphoma - t(14:18)
163. Fragile X - CGG
164. Friedreich's ataxia - GAA
165. Friedreich's ataxia is autosomal recessive
166. G0 - quiescence
167. G1 - determines length of cell cycle
168. G1 - under influence of p53
169. G1 - cells increase in size
170. G2 - cells increase in size
171. G6PD deficiency is x-linked recessive
172. GABA-C - ligand-gated ion channel receptor
173. Gaussian distribution - mean = median = mode
174. Ghrelin - increases appetite
175. Ghrelin - P/D1 cells lining the fundus of the stomach and epsilon cells of the pancreas
176. Gilbert's syndrome is autosomal recessive
177. Glucocorticoids in breastfeeding is considered safe to use
178. Glycogen storage disease is autosomal recessive
179. Golgi apparatus - modifies, sorts, and packages these molecules that are destined for cell secretion
180. Goodpasture's - HLA-DR2
181. Growth retardation, purpuric skin lesions, sensorineural deafness and seizures - cytomegalovirus
182. Haemochromatosis - HLA-A3
183. Haemochromatosis is autosomal recessive
184. Haemophilia A,B is x-linked recessive
185. Hawthorne effect - describes a group changing it's behaviour due to the knowledge that it is being studied
186. Heparin in breastfeeding is considered safe to use
187. Hereditary haemorrhagic telangiectasia is autosomal dominant
188. Hereditary non-polyposis colorectal carcinoma is autosomal dominant
189. Hereditary spherocytosis is autosomal dominant
190. Homocystinuria - deficiency of cystathionine beta synthase
191. Homocystinuria - downwards lens dislocation
192. Homocystinuria - learning difficulties
193. Homocystinuria , treatment of choice: vitamin B6 (pyridoxine) supplements
194. Homocystinuria is autosomal recessive
195. Hunter's disease is x-linked recessive
196. Huntington's - CAG
131
M Y Elamin
MBBS, DTM&H, MCTM, MRCPI 1& 2
197. Huntington's disease is autosomal dominant
198. Hydralazine in breastfeeding is considered safe to use
199. Hyperlipidaemia type II is autosomal dominant
200. Hypersensitive carotid sinus reflex - CN IX
201. Hypoglossal nerve passes through the hypoglossal canal
202. Hypokalaemia - metabolic alkalosis
203. Hypokalaemic periodic paralysis is autosomal dominant
204. Incidence - the number of new cases per population in a given time period
205. Increased gastric mucus secretion - PGE2
206. Increased pain - PGE2
207. Increased platelet aggregation - TXA2
208. Increased temperature - PGE2
209. Increased uterine tone - PGE2
210. Interval variable - a measurement where the difference between two values is meaningful, such that equal
differences between values correspond to real differences between the quantities that the scale measures (for
example temperature)
211. Kendall rank - non-parametric test of correlation
212. Late-look bias - gathering information at an inappropriate time
213. Leads to activation of protein kinase C - Gq receptor
214. Lead-time bias - occurs when two tests for a disease are compared, the new test diagnoses the disease earlier,
but there is no effect on the outcome of the disease
215. Leptin - adipose tissue
216. Leptin - decreases appetite
217. Lesch-Nyhan syndrome is x-linked recessive
218. Levothyroxine in breastfeeding is considered safe to use
219. Likelihood ratio for a negative test result - how much the odds of the disease decrease when a test is negative
220. Likelihood ratio for a negative test result = (1 - sensitivity) / specificity
221. Likelihood ratio for a positive test result - how much the odds of the disease increase when a test is positive
222. Likelihood ratio for a positive test result = sensitivity / (1 - specificity)
223. Linear regression - used to predict how much one variable changes when a second variable is changed
224. Lipid storage disease: Tay-Sach's, Gaucher, Niemann-Pick is autosomal recessive
225. Liquorice - metabolic alkalosis
226. Lithium in breastfeeding must be avoided
227. Long-term side-effects - clinical trial phase IV
228. Loss of corneal reflex - CN V
229. Loss of corneal reflex - CN VII
132
M Y Elamin
MBBS, DTM&H, MCTM, MRCPI 1& 2
230. Lysosome - breakdown of large molecules such as proteins and polysaccharides
231. M - cell division
232. Malignant hyperthermia is autosomal dominant
233. Mann-Whitney test - non-parametric test of unpaired data
234. Marfan's syndromes is autosomal dominant
235. McCune-Albright syndrome - cafe-au-lait spots
236. McCune-Albright syndrome - polyostotic fibrous dysplasia
237. McCune-Albright syndrome - precocious puberty
238. Mean - the average of a series of observed values
239. Median - the middle value if series of observed values are placed in order
240. Methotrexate in breastfeeding must be avoided
241. Mitochondrion - aerobic respiration. Contains mitochondrial genome as circular DNA
242. Mode - the value that occurs most frequently within a dataset
243. Mucopolysaccharidoses: Hurler's is autosomal recessive
244. Multiple tumor suppressor 1 (MTS-1, p16) - melanoma
245. Multiple tumor suppressor 1 (MTS-1, p16) - tumour suppressor gene
246. Myotonic dystrophy - CTG
247. Myotonic dystrophy is autosomal dominant
248. Narcolepsy - HLA-DR2
249. Negative predictive value - the chance that the patient does not have the condition if the diagnostic test is
negative
250. Negative predictive value = TN / (TN + FN)
251. Negatively skewed distribution - mean < median < mode
252. Nephrogenic diabetes insipidus is x-linked recessive
253. Neurofibromatosis is autosomal dominant
254. Neutrophil chemotaxis - LTB4
255. NF1 - neurofibromatosis
256. NF1 - tumour suppressor gene
257. N-MYC - neuroblastoma
258. N-MYC - oncogene
259. Nominal variable - observed values can be put into set categories which have no particular order or hierarchy.
You can count but not order or measure this data type
260. Noonan syndrome is autosomal dominant
261. Northern blotting - detects RNA
262. Null hypothesis - a hypothesis that two treatments are equally effective
263. Number needed to treat - 1 / absolute risk reduction
133
M Y Elamin
MBBS, DTM&H, MCTM, MRCPI 1& 2
264. Number needed to treat - a measure that indicates how many patients would require an intervention to reduce
the expected number of outcomes by one
265. Ocular albinism is x-linked recessive
266. Odds - a ratio of the number of people who incur a particular outcome to the number of people who do not
incur the outcome
267. Odds ratio - the ratio of the odds of a particular outcome with experimental treatment and that of control
268. Optimal dosing - clinical trial phase IIa
269. Ordinal variable - observed values can be put into set categories which themselves can be ordered (for example
NYHA classification of heart failure symptoms)
270. Osteogenesis imperfecta is autosomal dominant
271. P value - is the probability of obtaining a result by chance at least as extreme as the one that was actually
observed, assuming that the null hypothesis is true
272. P53 - Li-Fraumeni syndrome
273. P53 - tumour suppressor gene
274. Paralysis of mastication muscles - CN V
275. Patau syndrome (trisomy 13) - cleft lip/palate
276. Patau syndrome (trisomy 13) - microcephalic, small eyes
277. Patau syndrome (trisomy 13) - polydactyly
278. Pearson's product-moment coefficient - parametric test of correlation
279. Penicillins in breastfeeding is considered safe to use
280. Peutz-Jeghers syndrome is autosomal dominant
281. Pharmacokinetics and pharmacodynamics - clinical trial phase I
282. Phase 0 - rapid sodium influx
283. Phase 1 - efflux of potassium
284. Phase 2 - slow influx of calcium
285. Phase 3 - efflux of potassium
286. Phase 4 - resting potential is restored by Na+/K+ ATPase
287. Phenylketonuria - musty odour to urine and sweat
288. Phenylketonuria - seizures
289. PKU is autosomal recessive
290. Plateau - slow influx of calcium
291. Positive predictive value - the chance that the patient has the condition if the diagnostic test is positive
292. Positive predictive value = TP / (TP + FP)
293. Positively skewed distribution - mean > median > mode
294. Power - 1 - the probability of a type II error
295. Power - the probability of (correctly) rejecting the null hypothesis when it is false
134
M Y Elamin
MBBS, DTM&H, MCTM, MRCPI 1& 2
296. Power - the probability of detecting a statistically significant difference
297. Prevalence - the total number of cases per population at a particular point in time
298. Primary biliary cirrhosis - HLA-DR3
299. Primary hyperaldosteronism - metabolic alkalosis
300. Probability - the fraction of times you'd expect to see an event in many trials
301. Prolactin - non-receptor tyrosine kinase
302. Proteasome - degradation of protein molecules that have been tagged with ubiquitin
303. Ptosis - CN III
304. Publication bias - failure to publish results from valid studies, often as they showed a negative or uninteresting
result
305. Randomised controlled trial - practical or ethical problems may limit use
306. Range - the difference between the largest and smallest observed value
307. Rapid depolarisation - rapid sodium influx
308. RAS - neurofibromatosis
309. RAS - oncogene
310. RAS - pancreatic cancer
311. Rb - retinoblastoma
312. Rb - tumour suppressor gene
313. Reactive arthritis - HLA-B27
314. Recall bias - difference in the accuracy of the recollections retrieved by study participants, possibly due to
whether they have disorder or not
315. Relative risk reduction - absolute risk change / control event rate
316. Reliability - an assessment of whether a test is consistent
317. RET - multiple endocrine neoplasia (types II and III)
318. RET - oncogene
319. Retinitis pigmentosa is x-linked recessive
320. Retinoblastoma is autosomal dominant
321. Rett syndrome is x-linked dominant
322. Rheumatoid arthritis - HLA-DR4
323. Ring + little finger - C8
324. Risk - the probability that an event will occur
325. Risk ratio - the risk of an event happening in one group divided by the risk in another group
326. Romano-Ward syndrome is autosomal dominant
327. S - centrosome duplication
328. S - synthesis of DNA, RNA and histones
329. Sampling bias - subjects are not representative of the population
135
M Y Elamin
MBBS, DTM&H, MCTM, MRCPI 1& 2
330. Selection bias - a general term describing the non-random assignment of patients to a study group
331. Sensitivity - proportion of patients with the condition who have a positive test result
332. Sensitivity = TP / (TP + FN )
333. Sickle cell anaemia is autosomal recessive
334. Significance - the probability of getting the results if the null hypothesis is true
335. Sjogren's syndrome - HLA-DR3
336. Spearman rank - non-parametric test of correlation
337. Specificity - proportion of patients without the condition who have a negative test result
338. Specificity = TN / (TN + FP)
339. Standard deviation - a measure of how much dispersion exists from the mean
340. Standard deviation - square root (variance)
341. Standard error of the mean - a measure of the spread expected for the mean of the observations
342. Standard error of the mean - standard deviation / square root (sample size)
343. Student's t-test - parametric test of paired or unpaired data
344. Sulphonamides in breastfeeding must be avoided
345. Sulphonylureas in breastfeeding must be avoided
346. Tall, long fingered, downward lens dislocation, learning difficulties, DVT - homocystinuria
347. Tetracyclines in breastfeeding must be avoided
348. Thalassaemias is autosomal recessive
349. Theophyllines in breastfeeding is considered safe to use
350. Thumb + index finger - C5, C6
351. Tricyclic antidepressants in breastfeeding is considered safe to use
352. Trimethoprim in breastfeeding is considered safe to use
353. Tuberous sclerosis is autosomal dominant
354. Turner's syndrome - bicuspid aortic valve
355. Turner's syndrome - coarctation of the aorta
356. Turner's syndrome - elevated gonadotrophin levels
357. Turner's syndrome - lymphoedema in neonates
358. Turner's syndrome - primary amenorrhoea
359. Turner's syndrome - shield chest with widely spaced nipples
360. Turner's syndrome - subfertility/infertility
361. Turner's syndrome - webbed neck
362. Type 1 diabetes mellitus - HLA-DR4
363. Type I error - the null hypothesis is rejected when it is true
364. Type II error - the null hypothesis is accepted when it is false
365. Type I hypersensitivity reaction - asthma
136
M Y Elamin
MBBS, DTM&H, MCTM, MRCPI 1& 2
366. Type I hypersensitivity reaction - hayfever
367. Type II hypersensitivity reaction - acute hemolytic transfusion reactions
368. Type II hypersensitivity reaction - bullous pemphigoid
369. Type II hypersensitivity reaction - ITP
370. Type II hypersensitivity reaction - pernicious anemia
371. Type II hypersensitivity reaction - rheumatic fever
372. Type III hypersensitivity reaction - post-streptococcal glomerulonephritis
373. Type III hypersensitivity reaction - serum sickness
374. Type IV hypersensitivity reaction - allergic contact dermatitis
375. Type IV hypersensitivity reaction - graft versus host disease
376. Type IV hypersensitivity reaction - multiple sclerosis
377. Type IV hypersensitivity reaction - scabies
378. Type IV hypersensitivity reaction - tuberculin skin reaction
379. Type IV hypersensitivity reaction - tuberculosis
380. Umbilicus - T10
381. Uvula deviated away from the side of lesion - CN X
382. Vagus nerve passes through the jugular foramen
383. Validity - an assessment of whether a test accurately measures what it is supposed to measure
384. Variance - square of standard deviation
385. Vasoconstriction - TXA2
386. Vasodilation - PGI2
387. Vertigo - CN VIII
388. Vestibulocochlear nerve passes through the internal acoustic meatus
389. Vitamin B1 deficiency may be caused by alcohol excess
390. Vitamin B1 deficiency may cause amnesia, confabulation
391. Vitamin B1 deficiency may cause beriberi
392. Vitamin B1 deficiency may cause dilated cardiomyopathy
393. Vitamin B1 deficiency may cause Korsakoff's syndrome
394. Vitamin B1 deficiency may cause nystagmus, ophthalmoplegia and ataxia
395. Vitamin B1 deficiency may cause peripheral neuropathy
396. Vitamin B1 deficiency may cause Wernicke's encephalopathy
397. Vitamin D resistant rickets is x-linked dominant
398. Vomiting / aspiration - metabolic alkalosis
399. Von Hippel-Lindau syndrome is autosomal dominant
400. Von Willebrand's disease is autosomal dominant
401. Warfarin in breastfeeding is considered safe to use
137
M Y Elamin
MBBS, DTM&H, MCTM, MRCPI 1& 2
402. Western blotting - an example includes the confirmatory HIV test
403. Western blotting - detects proteins
404. Western blotting - uses gel electrophoresis to separate native proteins by 3-D structure
405. Wilcoxon matched-pairs - non-parametric test that compares two sets of observations on a single sample
406. Wilson's disease is autosomal recessive
407. Wiskott-Aldrich syndrome is x-linked recessive
408. WT1 - tumour suppressor gene
409. WT1 - Wilm's tumour
410. Reticulocytes have a mesh-like (reticular) network of ribosomal RNA that is visible when stained. This is not
present in mature erythrocytes
411. A normal temporal artery biopsy in a patient with suspected giant cell arteritis does not exclude the disease
because of the potential for skip lesions
412. Absolute risk reduction = (Control event rate) - (Experimental event rate)
413. Acute tubular necrosis is associated with granular, muddy-brown urinary casts
414. Adrenal cortex mnemonic: GFR - ACD
415. Adrenaline acts via G protein-coupled receptors
416. Amiodarone in breastfeeding must be avoided
417. Anti-A, B blood antibodies - IgM
418. Anticipation in trinucleotide repeat disorders = earlier onset in successive generations
419. Antidiuretic hormone (ADH) - site of action = collecting ducts
420. Antidiuretic hormone promotes water reabsorption by the insertion of aquaporin-2 channels
421. Atrial natriuretic factor - guanylate cyclase receptor
422. Atrial natriuretic peptide - powerful vasodilator
423. Autosomal recessive conditions are 'metabolic' - exceptions: inherited ataxias
424. Autosomal dominant conditions are 'structural' - exceptions: Gilbert's, hyperlipidaemia type II
425. Autosomal recessive conditions are 'metabolic' - exceptions: inherited ataxias
426. Autosomal dominant conditions are 'structural' - exceptions: hyperlipidaemia type II, hypokalaemic periodic
paralysis
427. B cells mediate hyperacute organ rejection
428. Bartter's syndrome is a cause of metabolic alkalosis
429. Behcet's disease is associated with HLA-B51
430. Burkitt's lymphoma is commonly associated with c-MYC
431. C5-9 deficiency predisposes to Neisseria meningitidis infections
432. C8 is the ONLY cervical nerve root that comes out BELOW the vertebra
433. Cardiac abnormalities of DiGeorge syndrome include truncus arteriosus and tetralogy of Fallot

138
M Y Elamin
MBBS, DTM&H, MCTM, MRCPI 1& 2
434. Case-control study - compares a group with a disease to a group without, looking at past exposure to a possible
causal agent for the condition
435. Case-control study - the usual outcome measure is the odds ratio
436. CD15 is found on Reed-Sternberg cells
437. CD21 is the receptor for the Ebstein-Barr virus
438. CD8 - co-receptor for MHC class I
439. Chi-squared test - used to compare proportions or percentages e.g. compares the percentage of patients who
improved following two different interventions
440. Clinical audit seeks to improve patient care and outcomes through systematic review of care against explicit
criteria and the implementation of change
441. CN6 palsy manifesting as diplopia could be the first sign of brain metastasis
442. Coeliac disease is linked to HLA-DQ2
443. Cohort study - the usual outcome measure is the relative risk
444. Congenital rubella
a. sensorineural deafness
b. congenital cataracts
445. Congenital toxoplasmosis
a. cerebral calcification
b. chorioretinitis
446. Contrast MRI scan is the gold standard investigation for cerebral metastases - provided no contraindications
447. Correlation
a. parametric (normally distributed): Pearson's coefficient
b. non-parametric: Spearman's coefficient
448. Deficiency in C1q, C1rs, C2 and C4 predisposes to immune complex disease such as SLE
449. Deletion of chromosome 15
a. Prader-Willi - paternal
b. Angelman syndrome - maternal
450. Dermatitis herpetiformis is associated with HLA-DR3
451. Dexamethasone is used to treat cerebral oedema in patients with brain tumours
452. Diabetic nephropathy histological findings- Kimmelstiel-Wilson lesions, nodular glomerulosclerosis
453. DiGeorge syndrome - a T-cell disorder
454. Down's syndrome risk - 1/1,000 at 30 years then divide by 3 for every 5 years
455. Dry beriberi is caused by thiamine deficiency and causes peripheral neuropathy
456. During mitosis, sister chromatids move to opposite ends of the cell during anaphase
457. Dysmorphic red blood cells if found in urine sediment indicates a glomerular origin of hematuria

139
M Y Elamin
MBBS, DTM&H, MCTM, MRCPI 1& 2
458. Endothelin causes pulmonary vasoconstriction (hence why antagonists are used in primary pulmonary
hypertension)
459. Epidermis - 5 layers - bottom layer = stratum germinativum which gives rise to keratinocytes and contains
melanocytes
460. Fabry disease: X-linked recessive
461. Fluorescence in situ hybridization uses fluorescent DNA or RNA probe to bind to specific gene site of interest
for direct visualisation of chromosomal anomalies
462. Foam cells are fat-laden macrophages
463. For a man with mitochondrial disease, none of his children will inherit the condition
464. Friedreich's ataxia is autosomal recessive
465. Funnel plots - show publication bias in meta-analyses
466. G1 phase determines cell cycle length
467. Gaucher's disease is the most common lipid storage disorder and a cause of hepatosplenomegaly
468. Golgi adds mannose-6-phosphate to proteins for trafficking to lysosomes
469. Goodpasture's - HLA-DR2
470. Goodpasture's syndrome is caused by autoantibodies against collagen type IV
471. HIV uses CD4 to enter cells
472. HLA antigens are encoded for by genes on chromosome 6
473. Homocystinuria - give vitamin B6 (pyridoxine)
474. Homocystinuria is caused by a deficiency of cystathionine beta synthase
475. Horseshoe kidney is the most common renal abnormality in Turner's syndrome
476. Human papillomavirus 16/18 is a risk factor for oropharyngeal cancer
477. IgD is involved in the activation of B-cells
478. IgE is synthesised by plasma cells
479. IL-8 - main functions include: neutrophil chemotaxis
480. Immune cells bind to the crystallising region (Fc) of immunoglobulins
481. In genetics, expressivity describes the extent to which a genotype shows its phenotypic expression in an
individual
482. Insulin binds to a receptor tyrosine kinase in the cell membrane
483. Interferon-gamma is produced primarily by natural killer cells and T helper cells
484. Interferon-γ is responsible for activating macrophages
485. Isoniazid therapy can cause a vitamin B6 deficiency causing peripheral neuropathy
486. Kearns-Sayre syndrome
a. mitochondrial inheritance
b. onset < 20-years-old
c. external ophthalmoplegia
140
M Y Elamin
MBBS, DTM&H, MCTM, MRCPI 1& 2
d. retinitis pigmentosa
487. Lead-time bias - occurs when two tests for a disease are compared, the new test diagnoses the disease earlier,
but there is no effect on the outcome of the disease
488. Least abundant isotype in blood serum - IgE
489. Leber’s Hereditary Optic Neuropathy: mitochondrial inheritance pattern
490. Left ventricular ejection fraction = (stroke volume / end diastolic LV volume ) * 100%
491. Leptin is secreted by adipose tissue
492. Lesch-Nyhan syndrome causing hyperuricemia is genetically inherited in an X-linked recessive pattern
493. Li-Fraumeni syndrome (p53 mutation) is associated with early-onset breast cancer, sarcoma and leukaemia
494. Likelihood ratio for a negative test result - how much the odds of the disease decrease when a test is negative
495. Likelihood ratio for a positive test result = sensitivity / (1 - specificity)
496. Mann-Whitney U test - compares ordinal, interval, or ratio scales of unpaired data
497. Men with BRCA mutations have an increased risk of prostate cancer
498. Meningiomas are typically benign tumours that arise from the arachnoid cap cells of the meninges but are
typically located next to the dura
499. Microtubules help guide movement during intracellular transport and also help bind internal organelles
500. Mitochondrial diseases follow a maternal inheritance pattern
501. Molecular biology techniques
a. SNOW (South - NOrth - West)
b. DROP (DNA - RNA - Protein)
502. n-MYC is an oncogene for neuroblastoma
503. Narcolepsy - HLA-DR2
504. Negative predictive value = TN / (TN + FN)
505. Nicotinic acetylcholine - ligand-gated ion channel receptor
506. Nitric oxide , second messenger = cGMP
507. Nitric oxide - vasodilation + inhibits platelet aggregation
508. NNT = 1 / Absolute Risk Reduction
509. Non-parametric tests are more appropriate when the data sets are not normally distributed
510. Non-REM stage 1 (N1) sleep is the lightest sleep which is associated with hypnagogic jerks
511. Nondisjunction is the commonest cause of Down's syndrome
512. Norepinephrine - G protein-coupled receptor
513. Obesity hormones
a. Leptin Lowers appetite
b. Ghrelin Gains appetite
514. Odds - remember a ratio of the number of people who incur a particular outcome to the number of people
who do not incur the outcome
141
M Y Elamin
MBBS, DTM&H, MCTM, MRCPI 1& 2
515. Probability, NOT a ratio of the number of people who incur a particular outcome to the total number of people
516. P value - is the probability of obtaining a result by chance at least as extreme as the one that was actually
observed, assuming that the null hypothesis is true
517. P value - is the probability of obtaining a result by chance at least as extreme as the one that was actually
observed, assuming that the null hypothesis is true
518. p53 - plays a crucial role in the cell cycle, preventing entry into the S phase until DNA has been checked and
repaired
519. Pellagra is caused by vitamin B3 (niacin) deficiency
520. Peroxisomes are responsible for the catabolism of long chain fatty acids
521. Phosphate is reabsorbed in the proximal tubule
522. Polymerase chain reactions are used to detect mutated oncogenes
523. Positive predictive value - the chance that the patient has the condition if the diagnostic test is positive
524. Positive predictive value = TP / (TP + FP)
525. Power - the probability of (correctly) rejecting the null hypothesis when it is false
526. Power - the probability of detecting a statistically significant difference
527. Power = 1 - the probability of a type II error
528. Prolactin release is persistently inhibited by dopamine
529. Pulmonary arteries vasoconstrict in the presence of hypoxia
530. Pulmonary fibrosis results in decreased lung compliance
531. Pulmonary surfactant - main constituent is dipalmitoyl phosphatidylcholine (DPPC)
532. Rapid sodium influx causes rapid depolarisation
533. Reactive arthritis - HLA-B27
534. Recall bias is a particular problem in case-control studies
535. Recurrent urease-positive bacteria (eg. proteus mirabilis) infections predispose individuals to struvite renal
stones
536. Relative risk = EER / CER
537. Relative risk reduction = (EER - CER) / CER
538. REM sleep is the deepest stage of sleep which is associated with dreaming and loss of muscle tone
539. Renal stones are most commonly composed of calcium oxalate
540. Rheumatoid arthritis - HLA DR4
541. Riboflavin deficiency causes angular cheilitis
542. RNA splicing takes place within the nucleus
543. Schistocytes are seen on blood smears in DIC
544. Scurvy causes gum disease
545. Sensitivity - proportion of patients with the condition who have a positive test result
546. Sensitivity = TP / (TP + FN )
142
M Y Elamin
MBBS, DTM&H, MCTM, MRCPI 1& 2
547. Sepsis triggers the release of IL-1 causing vasodilation → hypotension
548. Short stature + primary amenorrhoea ?Turner's syndrome
549. Shoulder abduction - deltoid muscle - axillary nerve (C5,C6)
550. Sjogren's syndrome- HLA- DR3
551. Skeletal muscle contraction is dependent on acetylcholine which activates nicotinic acetylcholine receptors
552. Skewed distributions
a. alphabetical order: mean - median - mode
b. '>' for positive, '<' for negative
553. Smooth muscle proliferation and migration from the tunica media into the intima is the last step in the
formation of an atheroma
554. Specificity - proportion of patients without the condition who have a negative test result
555. Specificity = TN / (TN + FP)
556. Standard error of the mean = standard deviation / square root (number of patients)
557. Tall, long fingered, downward lens dislocation, learning difficulties, DVT - homocystinuria
558. Tay-Sachs disease typically presents with developmental delay and cherry red spot on the macula, without
hepatomegaly or splenomegaly
559. Tertiary hyperparathyroidism is an important differential in hypercalcaemia post renal replacement therapy
560. The C6 dermatome is located on the index finger and thumb
561. The Cushing reflex is a physiological nervous system response to increased intracranial pressure (ICP) that
results in hypertension and bradycardia
562. The fastest conduction velocities in the heart are in the Purkinje fibres
563. The HLA-DRB1 gene is the major genetic susceptibility locus for rheumatoid arthritis (DRB1*04:01 and
DRB1*04:04 hence the association with DR4)
564. The main source of IL-1 is macrophages
565. The majority of glucose reabsorption in the nephron occurs within the proximal convoluted tubule
566. The spinothalamic tract decussates at the same level the nerve root enters the spinal cord. The corticospinal
tract, dorsal column medial lemniscus, and spinocerebellar tracts decussate at the medulla
567. The thin ascending limb of the loop of Henle is impermeable to water
568. Trimethoprim in breastfeeding is considered safe to use
569. Troponin C binds to calcium ions
570. Troponin I binds to actin to hold the troponin-tropomyosin complex in place
571. Troponin T binds to tropomyosin, forming a troponin-tropomyosin complex
572. Turner's syndrome - most common cardiac defect is bicuspid aortic valve (more common than coarctation of
the aorta)
573. Turner's syndrome is associated with aortic coarctation
574. Type 2 pneumocytes secrete surfactant
143
M Y Elamin
MBBS, DTM&H, MCTM, MRCPI 1& 2
575. Type I error - the null hypothesis is rejected when it is true
576. Type I hypersensitivity reaction - anaphylaxis
577. Type II error - the null hypothesis is accepted when it is false
578. Type II hypersensitivity reaction - ITP
579. Type IV hypersensitivity reaction - allergic contact dermatitis
580. Type IV hypersensitivity reaction - scabies
581. Ubiquitin tagging destines proteins to proteasome for degradation
582. Ulnar nerve supplies sensory innervation to the palmar and dorsal aspects of 1 and 1/2 fingers medially
583. Variance = square of standard deviation
584. Vincristine acts during the metaphase
585. Vitamin C (ascorbic acid) supplementation can aid iron absorption from the gut by conversion of Fe3+ to
Fe2+
586. Vitamin D increases serum calcium primarily through raising its absorption via the small intestine
587. Vitamin K acts as a cofactor in the carboxylation of clotting factors (II, VII, IX, X)
588. Vomiting / aspiration - metabolic alkalosis
589. When analyzing data on a scatter plot, linear regression may be used to predict how much one variable
changes when a second variable is changed
590. Wilcoxon signed-rank test - compares two sets of observations on a single sample, e.g. a 'before' and 'after'
test on the same population following an intervention
591. X-linked conditions: Duchenne/Becker, haemophilia, G6PD
592. X-linked recessive conditions - no male-to-male transmission
593. X-linked recessive conditions - there is no male-to-male transmission. Affected males can only have unaffected
sons and carrier daughters.
594. A high serum urate concentration can lead to gout. As such, patients with high levels should be given lifestyle
advice about reducing intake of substances containing high purines (including some alcohol and red meat).
595. Achilles tendinitis is an idiosyncratic reaction associated with prolonged use of quinolones.
596. ADPKD is autosomal dominant and offspring of a parent with the disease have a 50% chance of having the
condition
597. Age-related muscle loss in the elderly is due to a decline in glomerular filtration rate and plasma flow rate.
598. Agreement, implementation, and monitoring of necessary changes are an essential part of a significant event audit.
599. Alendronate inhibits osteoclast activity and this prevents the breakdown of bone and thus bone loss.
600. Alport's syndrome is a disorder of type 4 collagen assembly and is inherited as an X-linked disorder in 85% of
cases.
601. An understanding of the anatomical layout of the thorax as seen on a CT is essential in the interpretation of these
images.

144
M Y Elamin
MBBS, DTM&H, MCTM, MRCPI 1& 2
602. Antiphospholipid antibody syndrome can be diagnosed if the patient has anticardiolipin antibodies, or lupus
anticoagulant on two occasions, over a period of 12 weeks, and has had either a thrombus, or a history of recurrent
<10 week pregnancy loss, or one pregnancy loss >10 weeks in gestation when other causes of pregnancy loss have
been excluded.
603. Any symptoms or features of hypogonadism or hypercalcaemia should be elicited. Hyperprolactinaemia
suppresses gonadotropin secretion so a testosterone concentration would be far more relevant.
604. Approximately 0.1% of body iron circulates in the plasma.
605. Being an infectious disease, mumps does not have a genetic aetiological component and is not inherited
606. BRCA1 is associated with increased risk of pancreatic cancer, and BRCA2 is associated with pancreatic cancer,
prostate cancer, and melanoma.
607. By definition, there are no other neurological deficits in transient global amnesia.
608. CD8 positive T lymphocytes bind MHC class I in conjunction with a specific cell surface antigen.
609. Certain drugs can cause hyperkalaemia, including cyclosporin.
610. Chi squared testing is useful when comparing nominal or categorical outputs from a study, and may also be of
value in some ordinal analyses.
611. Ciprofloxacin interferes with DNA synthesis by disrupting the function of DNA gyrase.
612. Class II MHCs are present on all antigen presenting cells, including B cells
613. Clinical features of vitamin C deficiency (scurvy)
614. Components C5 to C9 comprise the membrane attack complex, and it is deficiencies of these components which
predispose to bacterial infection.
615. Conjugated bilirubin passes into the enterohepatic circulation and the bilirubin which evades this system is
metabolised by bacteria, primarily in the large intestine, to urobilinogen, then stercobilinogen and eventually
oxidised to stercobilin.
616. Deficiencies of complement C5-9 predispose to disseminated Neisseria infections.
617. DHFR is an enzyme that converts dihydrofolic to tetrahydrofolic acid, an essential stage in bacterial purine and,
ultimately DNA synthesis.
618. Drugs like labetalol undergo extensive first-pass metabolism in the liver and can have increased effect as patients
get older due to reduced blood flow to the liver.
619. Fragile X syndrome is the most common form of inherited learning difficulties with characteristic post puberty
phenotype.
620. Gastrin release release is stimulated by gastric luminal peptides.
621. Gaucher's disease is the most frequent of the lysosomal storage diseases leading to an elevation in acid phosphatase
and reduction in glucocerebrosidase.
622. Genetic anticipation occurs when an inherited disorder presents at a younger age with successive generations.
623. Gentamicin is synergistic to the action of benzylpenicillin. Benzylpenicillin is bactericidal, inhibiting cell wall
synthesis, enabling gentamicin to enter the bacterial cell.
145
M Y Elamin
MBBS, DTM&H, MCTM, MRCPI 1& 2
624. GLP-1 reduces postprandial glucose and the analogue exenatide is used in combination theraphy in type 2 diabetes.
625. Glycopeptides inhibit cell wall synthesis through steric hindrance of peptidoglycans, components of the bacterial
cell wall.
626. Guidelines recommend transferrin saturation, ferritin, and 282Y mutation testing in first degree relatives of a case
of haemochromatosis once they reach 18 years or older.
627. HbS is caused by a single base mutation on the beta-chain
628. HHT carries an autosomal dominant inheritance pattern.
629. HLA DR-4 is associated with rheumatoid arthritis with extra-articular features.
630. Huntington's disease is inherited in an autosomal dominant fashion
631. Hypocalcaemia is seen as a result of massive blood transfusion because of the high citrate load, and can lead to
tetany.
632. If the MHC I is presenting material recognised as foreign, then it is detected and destroyed by CD8+ T cells.
633. In cases of suspected type V McArdle's syndrome, the most appropriate investigation is muscle biopsy.
634. 'In situ hybridisation' uses a labelled complementary DNA or RNA probe to localise a specific sequence within a
tissue.
635. Inadequate intake of calories in any form is the most common cause of malnutrition.
636. Increased levels of TNF-alpha are known to result in protein catabolism, weight loss, and muscle weakness, all
features of disseminated carcinoma where TNF-alpha is elevated. As such an antagonist of TNF-alpha would be
expected to result in decreased protein catabolism.
637. Interpretation of blood gases is a key element of training. A logical approach is required.
638. Isoniazid can cause dose related peripheral neuropathy, which results from the depletion of pyridoxine in the
presence of the drug.
639. It is believed that the cystic fibrosis gene offers a possible survival advantage against cholera and enteropathogenic
bacteria.
640. ITT is considered to be the analysis which is least subject to bias.
641. Lactose intolerance is best diagnosed with a methane breath test
642. LDL receptor mutations may range from complete absence of the receptor to receptors which are present but only
partially functioning.
643. LHON is a mitochondrial disorder associate with optic atrophy with signs of central visual loss. The visual loss
usually affects one eye but can affect both.
644. Linkage analysis is used to determine the rough location of the gene responsible for disease, relative to another
DNA sequence which has its position already known (a 'genetic marker').
645. Marfan's syndrome occurs due to a mutation in the fibrillin gene.
646. Mediators involved in the development of anaphylaxis include IL4 and lead to mast cell degranulation.
647. Membranous glomerulonephritis presents with proteinuria, and is caused by immune complex deposition in the
subepithelial aspect of the capillary loop.
146
M Y Elamin
MBBS, DTM&H, MCTM, MRCPI 1& 2
648. Methyldopa should be avoided in patients with deranged LFT.
649. Monogenic diabetes/MODY is associated with gene mutations, commonly glucokinase and HNF1a and HNF4a.
650. Mutations in PBPs (enzymes required for cell wall synthesis) result in penicillin resistance.
651. Mutations in rpoB gene cause alterations in the bacterial DNA dependent RNA transcriptase, which prevents the
binding of rifampicin.
652. Natural selection is defined as the process whereby organisms possessing certain characteristics that make them
better adjusted to the environment tend to survive and reproduce in greater numbers.
653. Number needed to treat= 1/ absoolute risk reduction.
654. Over 100 allelic variants of the SERPINA-1 gene are known, which are inherited in a autosomal codominant
manner with certain combinations resulting in alpha-1 antitrypsin deficiency.
655. Phenylketonuria (PKU) is the result of hyperphenylalaninaemia. Urinary phenylalanine metabolites are typically
high.
656. Physiologically, thyroid hormones enhance insulin sensitivity as well as increasing gluconeogenesis.
657. Pompe's disease results from a deficiency in alpha-glucosidase, and is also known as glycogen storage disease type
2.
658. Post-marketing surveillance (PMS)/observational studies (phase IV studies) are generally designed to assess the
potential side effects of new drugs, but under everyday conditions and with a minimum of intervention.
659. Presence of oral mucosal and/or lip AVM spots should prompt a search for HHT in a patient presenting with
bleeding disorder.
660. Prolonged exposure to gentamicin is likely to cause nephrotoxicity, particularly in elderly patients with renal
impairment.
661. PTH acts through cAMP as the second messenger
662. Role of lysosomes in cellular function
663. Sarcoidosis is a multisystem granulomatous disorder. It is relatively uncommon in the United Kingdom as a whole,
but is more prevalent and more severe in Afro-Caribbean populations.
664. Sensitivity of a screening test measures the proportion of positive cases which are identified as such, and is
calculated from true positives/(true positives + false negatives).
665. Sensitivity relates to the probability that the person with a disease will be correctly identified with the disease
666. Side-effects of thiazides and related diuretics include mild gastro-intestinal disturbances, postural hypotension,
altered plasma-lipid concentrations, metabolic and electrolyte disturbances including hypokalaemia,
hyponatraemia, hypomagnesaemia, hypercalcaemia, hyperglycaemia, hypochloraemic alkalosis, hyperuricaemia,
and gout.
667. Skier's thumb was formerly known as gamekeeper's thumb, and it relates to injury to the base of the thumb,
resulting in damage or rupture of the ulnar collateral ligament.
668. Some Escherichia coli isolates produce an ESBL that inactivates second and third generation cephalosporins.
669. Splicing occurs in the nucleus before transport to the cytoplasm.
147
M Y Elamin
MBBS, DTM&H, MCTM, MRCPI 1& 2
670. Surfactant is produced by type 2 pneumocytes and is responsible for the ability of the air-filled alveoli to expand
without collapse.
671. Systemic mastocytosis is a disease which usually affects the elderly and is associated with urticaria pigmentosa,
diarrhoea,hypotension,sclerotic bone changes, and mast cell infiltration of organs such as spleen, liver, and kidneys.
672. Th2 CD4 positive lymphocytes are involved in the pathogenesis of anaphylaxis, via the production of IL-4/IL-13
that act on B cells to increase IgE production and precipitate the development of acute hypersensitivity.
673. The acute rejection of a kidney after transplantatioon is recognised and due to anti-IgG antibodies to the human
leukocyte antigen (HLA) incompatible tissues with primary activation of T cells.
674. The cerebellum comprises the vermis medially and the cerebellar hemispheres laterally.
675. The classical clinical features of Horner's syndrome are ptosis, miosis, and anhidrosis.
676. The exertional thigh cramps, the presence of myoglobin, and change in colour of urine after exercise suggests
glycogen storage disease type V - McArdle's syndrome.
677. The human karyotype consists of 22 pairs of autosomes and 1 pair of sex chromosomes.
678. The most common measurement for nutritional status in routine clinical practice is the body mass index (BMI).
679. The negative predictive value is calculated by dividing the number of true negatives by the sum of the number of
true negatives and false negatives.
680. The number needed to treat to prevent one specified outcome event is 100 divided by the absolute risk reduction.
681. The presence of anti-Sm antibodies is the most SPECIFIC for SLE than the other options.
682. The radial nerve innervates the triceps muscle; it is primarily derived from the C7 nerve root.
683. The risk of developing prostate cancer is increased by increasing age, ethnicity (highest in black Africans) and a
family history of prostate cancer.
684. The root supply to the peroneal muscles (which control eversion of the foot and which also participate in the reflex
arc of the ankle jerk reflex) is S1 via the tibial and superficial peroneal nerves
685. There is a reduced tachycardic response during exercise associated with age. The heart has to compensate by
increasing stroke volume and failure to do so will reduce aerobic capacity.
686. Ureteric diversion results in metabolic acidosis because implantation of the ureters into the sigmoid colon or a
vesicocolic leads to absorption of Cl- in exchange for HCO3- across the bowel mucosa. Absorption of urinary NH4+
in the sigmoid colon may also contribute to the development of acidosis as metabolism of the ammonium in the
liver results in production of H+.
687. Vitamin D replacement (800 IU daily) with calcium, has been shown to reduce falls and fractures by 47% compared
with controls who received calcium only.
688. When using linear regression Y is a dependent variable and X is an independent variable. The values of Y depends
on the values of X.
689. Wilson’s is an autosomal recessive disorder with a carrier frequency of 1 in 90 in a Northern European Caucasian
population.

148
M Y Elamin
MBBS, DTM&H, MCTM, MRCPI 1& 2
690. X linked recessive diseases can occur with equal severity in males and females.A number of mutations associated
with early onset Alzheimer’s disease have now been identified and screening for them is available.
691. A number of mutations associated with early onset Alzheimer’s disease have now been identified, and screening
for them is available.
692. CF is caused by mutations in a single large gene on chromosome 7 that encodes the cystic fibrosis transmembrane
conductance regulator (CFTR) protein
693. Citalopram is the antidepressant of choice in Alzheimer's disease due to it's effect on mood and well-being.
694. Denosumab is a RANK ligand inhibitor, reducing osteoclast activity which is used when bisphosphonates aren’t
tolerated.
695. Experimental evidence supports a positive effect of metformin on cardiac function, both in models of ischaemia
and in models of heart failure
696. Flecainide is now considered an acceptable anti-arrhythmic in patients with WPW who do not have underlying
structural heart disease.
697. Guidance on use of ICD has changed, and where there is QRS prolongation in class III heart failure, an ICD is
now potentially indicated.
698. Hyperkalaemia treatment is initiated when over 6.5 or 6 with ECG changes.
699. In acute psychosis associated with dementia NICE recommends using olanzapine, lorazepam, or haloperidol to
control symptoms.
700. IV diuretic therapy is the initial intervention of choice for patients presenting with acute heart failure.
701. Modern guidance recommends moving to early ablation in patients who suffer from paroxysmal AF, because it is
associated with a much greater chance of success when performed earlier.
702. NICE now supports initiation of acetylcholinesterase inhibitors in mild Alzheimer’s disease.
703. On ultrasound, fibroids appear as well-defined, solid masses with a whorled appearance usually of similar
echogenicity to the myometrium, but sometimes may be hypoechoic. They cause the uterus to appear bulky or may
cause an alteration of the normal uterine contour
704. Osmolality changes with environmental stressors, path physiology of hyperthermia, and pyrexia
705. Patients with borderline personality disorder may present with self harm, sudden outburst and a history of substance
misuse.
706. Patients with close contact of high risk patients should be offered influenza prophylaxis with oseltamivir within
48 hours of contact.
707. Pellagra is characterised by weakness, dermatitis, diarrhoea and dementia due to niacin deficiency.
708. Psuedohypoparathyroidism is associated with slipped epiphyseal plate in childhood.
709. Sepsis may be complicated by disseminated intravascular coagulation
710. Sildenafil and other phosphodiesterase inhibitors may substantially reduce urine output in nephrogenic diabetes
insipidus.
711. The mechanism by which lithium leads to diabetes insipidus, reduced GSK3 beta signalling, is well established
149
M Y Elamin
MBBS, DTM&H, MCTM, MRCPI 1& 2
712. The most common clinical signs prior to a cardiac arrest are respiratory distress (RR >25/min) and altered mental
state.
713. Thrombocytopenia is an unusual complication of heparin therapy
714. Ticagrelor is associated with superior efficacy versus clopidogrel in patients who have suffered a STEMI.
715. Achilles tendinitis is an idiosyncratic reaction associated with prolonged use of quinolones.
716. Although other organisms can cause meningitis, N. meningitidis is the likeliest cause in infants and young children.
717. An infected intravascular catheter is the likeliest cause of candidaemia.
718. BAL is more sensitive than sputum collection at detecting TB, either by smear or culture.
719. Blood cultures are the most important investigation for enteric fevers caused by Salmonella typhi or S. paratyphi.
720. Ciprofloxacin interferes with DNA synthesis by disrupting the function of DNA gyrase.
721. Deficiencies of complement C5-9 predispose to disseminated Neisseria infections.
722. DHFR is an enzyme that converts dihydrofolic to tetrahydrofolic acid, an essential stage in bacterial purine and,
ultimately DNA synthesis.
723. Exposure to parvovirus B19 can be confirmed or excluded by testing specific IgM. Failure to detect these
antibodies excludes infection in the four weeks prior to testing.
724. Gentamicin is synergistic to the action of benzylpenicillin. Benzylpenicillin is bactericidal, inhibiting cell wall
synthesis, enabling gentamicin to enter the bacterial cell.
725. Glycopeptides inhibit cell wall synthesis through steric hindrance of peptidoglycans, components of the bacterial
cell wall.
726. Gram positive cocci in pairs is most likely to represent Streptococcus pneumonia when assessing meningitis
727. Herpes simplex is a sexually transmitted disease and the commonest cause of multiple painful genital lesions.
728. Infections with Group A Streptococci are potentially severe and this is an example of streptococcal toxic shock
syndrome.
729. Invasive pneumococcal disease (IPD) is 20-30 times more common in HIV infected patients compared to non-
HIV infected patients. Consideration should be given to offering HIV testing to all patients with IPD presenting to
hospital.
730. Isoniazid can cause dose related peripheral neuropathy, which results from the depletion of pyridoxine in the
presence of the drug.
731. Lysosomes carry hydrolases that degrade nucleotides, proteins, lipds, and phospholipids.
732. Mutations in PBPs (enzymes required for cell wall synthesis) result in penicillin resistance.
733. Mutations in rpoB gene cause alterations in the bacterial DNA dependent RNA transcriptase, which prevents the
binding of rifampicin.
734. Normal cerebrospinal fluid, a CSF lymphocytosis, an increased opening pressure, and a raised CSF protein are
typical of viral meningitis.
735. PCP is the likeliest diagnosis as this man probably has undiagnosed advanced HIV infection.
736. Peroxisomes are responsible for the metabolism of fatty acids within cells.
150
M Y Elamin
MBBS, DTM&H, MCTM, MRCPI 1& 2
737. Pleural tuberculosis (TB) typically presents with pleurisy, pleuritic chest pain, cough, fever and a small to moderate
unilateral pleural effusion. Investigation should include a chest radiograph and sputum cultures initially but the
investigation most likely to yield a positive diagnosis is pleural biopsy.
738. Post-splenectomy patients are at risk of infections due to capsulated organisms.
739. Prolonged exposure to gentamicin is likely to cause nephrotoxicity, particularly in elderly patients with renal
impairment.
740. Prosthetic valve endocarditis arising within two months of valve surgery is generally the result of intraoperative
contamination of the prosthesis or a bacteraemia postoperative complication.
741. Pulmonary cavitation is caused by septic embolisation from the tricuspid valve, not aortic or mitral valve.
742. S. aureus and S. epidermidis are the two most common pathogens identified in cases of CAPD peritonitis.
743. S. aureus is the commonest cause of vertebral spine osteomyelitis.
744. S. milleri is a common cause of brain abscesses and is often associated with sinus infection.
745. Scombrotoxin food poisoning is caused by the ingestion of foods that contain high levels of histamine; linked to
spoilage of foods , particularly tuna or mahi-mahi.
746. Shigella is spread by faeco-oral contact via infected food or water, often during travel, especially overcrowded
facilites - or in long-term care facilities such as daycare centres or nursing homes.
747. Some Escherichia coli isolates produce an ESBL that inactivates second and third generation cephalosporins.
748. Staphylococcus aureus is the most common infecting organism of diabetic foot ulcers.
749. Strep. pyogenes and Staph. aureus are the common organisms found causing cellulitis.
750. Streptococcus viridans is NOT associated with a worse prognosis of infective endocarditis.
751. Subacute bacterial endocarditis (Streptococcus viridans) has a better prognosis in cases of infective endocarditis.
752. The most likely organisms responsible for digital cellulitis are Streptococcus pyogenes or Staphylococcus aureus.
753. The varicella zoster IgG status of an exposed pregnant lady is required to direct appropriate treatment.
754. The vomiting-type outbreaks of Bacillus cereus food poisoning have generally been associated with rice products.
755. Urinary tract infection, catheter associated, is a cause of severe hospital-acquired infection, particularly when the
catheter is blocked or following manipulation.
756. Urological procedures, such as transurethral resection of the prostate (TURP), may cause a transient E. faecalis
bacteraemia.
757. With a recent history of travel to Nigeria, cerebral malaria caused by P. falciparum should be considered.
758. With regards to Syphillis staging the absence of symptoms, late latent infection is more likely than early latent
infection.
759. A presumed non immune pregant woman exposed to chickenpox should first have IgG antibodies to VZV checked
to clarify immune status
760. Acanthosis nigricans is found in obese patients, diabetes from insulin resistance, familial, some genetic syndromes,
from drug resistance and rarely in some malignancies (eg gastric adenocarcinomas).
761. ACE inhibitors are not safe in pregnancy.
151
M Y Elamin
MBBS, DTM&H, MCTM, MRCPI 1& 2
762. As a general rule, pregnant patient and fetus are at far more risk from uncontrolled seizures than from any potential
teratogenic effect of carbamazepine
763. Azoospermia may be due to obstruction or inadequate production.
764. CA 125 is a tumour marker for ovarian carcinoma. No tumour marker is specific and should be interpreted in the
clinical context.
765. Cardiac output and blood volume increase in pregnancy but hepatic blood flow does not.
766. Chronic hypertension is defined as blood pressure exceeding 140/90 mm Hg before pregnancy or before 20 weeks'
gestation.
767. Chronic hypertension refers to high BP predating the pregnancy or recognized at <20 weeks’ gestation. In practice,
this is often diagnosed for the first time at the first or early second trimesters booking visit.
768. Clinical studies have shown that metformin reduces insulin resistance and have demonstrated a fall in serum
androgens, luteinising hormone and weight in PCO
769. Combined oral contraceptive pill can help with hirsuitism in PCOS.
770. Cortical vein thrombosis typically presents with headache, seizures and focal neurological deficit and is more
common in the postpartum period.
771. Current AHA/EHA criteria for the treatment of SVTs in pregnancy do suggest using metoprolol (level of evidence
1B) rather than verapamil (C), although they recommend avoiding the former in the first trimester.
772. Current guidelines for the outpatient treatment of pelvic inflammatory disease recommend ceftriaxone 500 mg IM
STAT, followed by oral doxycycline 100 mg BD and metronidazole 400 mg BD for 14 days.
773. Forty percent of Down's syndrome babies have atrioventricular septal defects.
774. Hypoactive sexual desire disorder is well recognised in post-menopausal females as well as in patients following
ovarian failure.
775. Infective endocarditis (IE) is a rare condition, it arises following bacteraemia in any patient, but those with a
predisposing cardiac lesion are at an increased risk.
776. Leflunomide reduces sperm count.
777. Live vaccines should not be given in pregnancy.
778. Long term LMWH treatment has been associated with low platelet counts.
779. Magnesium has been shown significantly to reduce maternal mortality in eclampsia and a favourable outcome may
also be expected in pre-eclampsia.
780. Magnesium sulphate (MgSO4) is the most effective agent for the treatment of eclampsia and prophylaxis in severe
pre-eclampsia.
781. Management of typhoid fever
782. Methyldopa is effective and safe for both mother and baby in pregnancy
783. Methyldopa is the treatment of choice for hypertension in pregnancy, as it has the largest evidence base for use.
784. Methyldopa should be avoided in patients with deranged LFT.
785. Peripartum cardiomyopathy can occur in women during labour or in the days following delivery.
152
M Y Elamin
MBBS, DTM&H, MCTM, MRCPI 1& 2
786. Polycystic ovarian syndrome is recognised to be a condition associated with increased insulin resistance and
metformin is effective through improvements in insulin sensitivity resulting in ovulation and improvements in
hormonal perturbations.
787. Pregnancy is a pro-coagulant state and therefore there is increased risk of PE, which is treated with either LMWH
or warfarin, depending on the stage of pregnancy.
788. Puerperal psychosis is a mood disorder with features of loss of contact with reality, hallucinations, thought disorder
and abnormal behaviour. Prognosis is good.
789. RCOG guidance is clear that when primary herpes infection occurs within six weeks of expected delivery then
caesarean delivery is the recommended course of action.
790. Recurrent first trimester miscarriages warrant further investigation which would include karyotyping and
assessment for lupus anticoagulant.
791. Rifampicin enhances the metabolism of oral contraceptive pills, decreasing its effectiveness and resulting in pill
failure.
792. Serum alkaline phosphatase can rise by up to 4x normal due to increased placental function during a normal
pregnancy
793. Sodium valproate is associated with an increased risk of neural tube defects when used in pregnancy
794. Tetracycline is excreted into human milk in small amounts. Theoretical risks of dental staining and inhibition of
bone growth exist, although are unlikely.
795. The cessation of the normal menstrual cycle caused by rapid loss of weight was defined as secondary amenorrhea
due to weight loss.
796. The most likely organism as a cause of a vaginal discharge with few other symptoms, is Candida. Vulvovaginal
candidiasis (VVC) is a clinical diagnosis based on typical features supported by laboratory confirmation of Candida
sp. from a vaginal sample.
797. The RCOG advise that any pregnant female presenting with shortness of breath should be reviewed by a senior
member of the obstetric team.
798. Treatment of chlamydia in a pregnant woman with erythromycin, amoxicillin or azithromycin first line is advised
prior to getting test results if strongly suspected clinically
799. Turner syndrome is associated with essential hypertension (10%).
800. Vitamin D-resistant rickets is X-linked dominant inheritance.
801. Warfarin is generally avoided in pregnancy.
802. Acute pancreatitis can present with a severe systemic inflammatory response, and is most commonly due to
gallstones or alcohol.
803. An ascending aortic dissection needs immediate surgery. Whilst en route to surgery, beta blockade to control
hypertension is appropriate.
804. As it descends in the carotid sheath the internal jugular vein lies lateral first to the internal then the common carotid
artery within the carotid sheath.
153
M Y Elamin
MBBS, DTM&H, MCTM, MRCPI 1& 2
805. Brain stem function is checked through set criteria, and the findings must be agreed by at least two senior doctors.
One should be a consultant, and the other must have at least five years post registration who has experience in the
testing of brain stem death.
806. Early transfusion is warranted in significant post-operative haemorrhage, but should not delay return to theatre.
807. In cases of suspected Guillain-Barré syndrome (GBS), first line therapy is intravenous IgG.
808. Insertion of a central line in SIRS allows measurement of central venous pressure and jugular venous oxygen
saturation, and also the use of inotropes, to pursue goal-directed therapy.
809. Lumbar puncture (LP) is not usually required unless the history is suggestive and the CT is normal.
810. Nifedipine is effective in managing oesophageal spasm in GI malignancy
811. Optimal treatment for acute altitude sickness is to descend to lower altitude if possible.
812. Oral bioavailability of THC, whether given in the pure form or as THC in marijuana, is low and extremely variable,
ranging between 5% and 20%, with effects occurring 0.5-3 hours later.
813. Patients with acute coronary syndrome in the post-operative period can be treated as per standard as long as there
is no evidence of bleeding.
814. Post-operative confusion can be caused by a low perfusion state.
815. Prolonged neuromuscular junction (NMJ) blockade may be exacerbated by both corticosteroids and magnesium.
816. Resuscitation of the sick patient still follows the ABC algorithm: Airway, Breathing, and Circulation.
817. Right or left subclavian is regarded as the cleanest site for central venous access
818. The most appropriate antibiotic therapy in dog bites associated with cellulitis is co-amoxiclav.
819. The pulmonary circulation is a low pressure, high flow, high compliance system.
820. The subclavian vein is a continuation of the axillary vein, beginning at the lateral border of the first rib.
821. The use of extracorporeal membrane oxygenation (ECMO) in patients with ARDS who are hypoxic on maximal
ventilatory therapy has been shown to promote a significant increase in survival without significant disability.
822. Urgent CT will confirm diagnosis in 95% of patients with subarachnoid haemorrhage
823. A left lacunar infarct will cause right sided motor symptoms.
824. A TACI comprises higher cortical dysfunction, contralateral homonymous hemianopia and contralateral motor or
sensory deficits.
825. Achrondroplasia may be diagnosed radiographically at birth, or becomes obvious within the first year with
disparity between a large skull, normal trunk length, and short limbs.
826. Acute chest pain in third trimester of pregnancy may be caused by AMI, pneumothorax, aortic dissection or
pulmonary embolism.
827. Acute MI can present with abdominal rather than chest pain, especially in older women of non-white racial origin.
828. Anaphylactic shock may be fatal within a very short time and immediate epinephrine should be used.
829. Anaplastic carcinomas of the thyroid are those which are most likely to cause compression or infiltration of the
trachea.

154
M Y Elamin
MBBS, DTM&H, MCTM, MRCPI 1& 2
830. Avascular necrosis of the femoral head is a well known complication of high dose steroid therapy, for example, in
patients with renal transplants. This patient had steroids for temporal arteritis.
831. Brain stem function is checked through set criteria, and the findings must be agreed by at least two senior doctors.
One should be a consultant, and the other must have at least five years post registration who has experience in the
testing of brain stem death.
832. Cardiac and pulmonary causes of dyspnoea can present in very similar ways, but the presence of orthopnoea is
classically associated with cardiac failure rather than pulmonary disease.
833. Clarithromycin strongly inhibits CYP3A4 - the enzyme responsible for simvastatin metabolism which can lead to
rhabdomyolysis and renal failure.
834. Delirium is diagnosed using the DSM-IV classification or short confusion assessment method (CAM).
835. Dementia in early stages can present with attacks of confusion at night and at unfamiliar places. The patient is
quite alright during day time.
836. Donepezil is a cholinesterase inhibitor.
837. Doxazocin is a likely cause of postural hypotension.
838. Drugs like labetalol undergo extensive first-pass metabolism in the liver and can have increased effect as patients
get older due to reduced blood flow to the liver.
839. Hyperkalaemia secondary to acute kidney injury is rapidly life-threatening and warrants emergency treatment with
calcium gluconate (if there is evidence of myocardial instability) and insulin/dextrose infusion.
840. Hypoglycaemia is common and mimics signs and symptoms of acute stroke; hence determination of blood glucose
and treatment of hypoglycaemia is part of the initial assessment of a patient with symptoms or signs of acute stroke.
841. Intravenous furosemide has much better bioavailability and thus therapeutic effect, especially where there is gut
oedema.
842. Lewy bodies are characteristic in Parkinson's disease.
843. Live attenuated vaccines are contraindicated in pregnancy.
844. Macrocytic anaemia should promt checking serum B12 levels (and folate).
845. Metastatic cauda equina compression is characterised by escalating back pain, associated with urinary disturbance,
lower limb weakness, loss of anal tone and saddle anaesthesia.
846. Minimal right facial weakness, impaired elevation of the right shoulder, with relatively preserved right hand
strength, and global weakness in the right leg which appears to be maximal in the foot, is a classical presentation of
anterior cerebral artery stroke.
847. Most cases of Parkinson’s disease occur in people with no family history of the disorder. The inheritance pattern,
if any, is unknown. Genetic screening is not offered in the absence of family history.
848. Multiple falls, a broad-based gait, and a jerky tremor are red flags pointing to a Parkinson's plus syndrome i.e
Multiple Systems Atrophy,
849. Oestrogen increases urethral closing pressure by increasing para-urethral collagen concentration and organisation.
850. Primarily postural tremor suggests essential tremor, as essential tremor is primarily a postural tremor.
155
M Y Elamin
MBBS, DTM&H, MCTM, MRCPI 1& 2
851. Sudden, short-lived loss of consciousness associated with pallor and rapid recovery is suggestive of a cardiac cause
of syncope and is best investigated with ECG monitoring.
852. The Attending Registered Medical practitioner should fill out a notification certificate immediately on diagnosis
of a suspected notifiable disease and should not wait for laboratory confirmation of the suspected infection or
contamination before notification.
853. The most widely recognised limitation of the Barthel index is floor and ceiling effects.
854. The NICE guidelines on stroke suggest that we use terminology to describe the type of impairment in accordance
with the WHO International Classification of Functioning, Disability and Health.
855. the patient has poor glycaemic control, but otherwise has no features that preclude him from driving such as
retinopathy, neuropathy or hypoglycaemic episodes
856. The prognosis of heart failure has been improved significantly with the use of angiotensin converting enzyme
(ACE) inhibitors and angiotensin receptor blockers that inhibit renin-angiotensin-aldosterone system.
857. There is a reduced tachycardic response during exercise associated with age. The heart has to compensate by
increasing stroke volume and failure to do so will reduce aerobic capacity.
858. Tilt table test is a useful test to support the diagnosis of vasovagal syncope.
859. To calculate alcohol units, multiply the strength (in %ABV) by volume (in millilitres) and divide by 1000
860. Transient global amnesia lasts less than 24 hours and normal cognition and an awareness of personal identity is
retained
861. Vitamin D replacement (800 IU daily) with calcium, has been shown to reduce falls and fractures by 47% compared
with controls who received calcium only.
862. Whilst angio-oedema may be hereditary or idiopathic, it is also associated with use of both ACE inhibitors and
angiotensin receptor blockers (ARB)

156
M Y Elamin
MBBS, DTM&H, MCTM, MRCPI 1& 2
DERMATOLOGY
1. A 55-year-old man presents with pruritus and an exfoliative erythroderma of the palms and soles. Generalised
lymphadenopathy and hepatosplenomegaly are noted - Sezary syndrome
2. A man presents with a raised, red, spherical lesion on his neck. It initially was a red spot but rapidly grew over 4
weeks - pyogenic granuloma
3. A man presents with an itchy, purple, papular rash on the palms and flexor surfaces of arms. The lesions are
polygonal and covered in a 'white-lace' - lichen planus
4. A man with poorly controlled coeliac disease develops itchy vesicles on his elbows and buttocks - dermatitis
herpetiformis
5. A middle-aged woman develops painful, erythematous, nodular lesions over both shins - erythema nodosum
6. A teenager with tear drop papules on the trunk and limbs 3 weeks after a sore throat - guttate psoriasis
7. A young adult notices hypopigmented patches on her trunk after coming getting a suntan. The patches are mildly
pruritic with some scale - pityriasis versicolor
8. A young child with a history of atopic eczema develops a sudden eruption of painful, oedematous vesicles and
pustules. They have are systemically unwell and have a fever - eczema herpeticum
9. Acne vulgaris (severe) , treatment of choice: oral isotretinoin
10. An adult with erythematous target lesions on the hands which later spread to the torso following a herpes simplex
infection - erythema multiforme
11. An elderly Jewish woman develops painful, flaccid, easily ruptured vesicles and bullae on the skin. They are not
itchy and were preceded by mouth lesions - pemphigus vulgaris
12. An elderly woman develops a reticulated, hyperpigmented rash after using a hot water bottle excessively -
erythema ab igne
13. An elderly woman presents with an itchy lesion on her vulva. It started as a red patch but has now turned into a white
plaque - lichen sclerosus
14. An elderly women develops itchy, tense blisters around the flexures. There is no mucosal involvement - bullous
pemphigoid
15. An obese man with type 2 diabetes mellitus develops brown, velvety plaques on his neck and axillae - acanthosis
nigricans
16. Bullous pemphigoid - antibodies against hemidesmosomal proteins BP180 and BP230
17. Chlorpheniramine may cause lethargy/drowsiness
18. Dermatitis, diarrhoea, dementia - niacin deficiency
19. Diabetes, waxy yellow shin lesions - necrobiosis lipoidica diabeticorum
20. Epistaxis, spider naevi, arteriovenous malformations - hereditary haemorrhagic telangiectasia
21. Fungal nail infection , treatment of choice: oral terbinafine
22. Gastric cancer - acanthosis nigricans
23. Grave's disease, orange peel shin lesions - pretibial myxoedema
157
M Y Elamin
MBBS, DTM&H, MCTM, MRCPI 1& 2
24. Isotretinoin may cause depression
25. Isotretinoin may cause dry mouth
26. Isotretinoin may cause dry skin
27. Isotretinoin may cause epistaxis
28. Isotretinoin may cause hair thinning
29. Isotretinoin may cause hyperlipidaemia
30. Isotretinoin may cause intracranial hypertension
31. Isotretinoin may cause photosensitivity
32. Itchy, red skin lesions, lymphadenopathy, hepatosplenomegaly - mycosis fungoides
33. Large vessel vasculitis - Takayasu's arteritis
34. Large vessel vasculitis - temporal arteritis
35. Medium vessel vasculitis - Kawasaki disease
36. Medium vessel vasculitis - polyarteritis nodosa
37. Mild acne rosacea , treatment of choice: topical metronidazole
38. Moderate-severe acne rosacea , treatment of choice: oral oxytetracycline
39. Pemphigus vulgaris - antibodies directed against desmoglein 3
40. Porphyria cutanea tarda - defect in uroporphyrinogen decarboxylase
41. Psoriasis may be worsened by ACE inhibitors
42. Psoriasis may be worsened by alcohol
43. Psoriasis may be worsened by beta-blockers
44. Psoriasis may be worsened by chloroquine
45. Psoriasis may be worsened by lithium
46. Psoriasis may be worsened by NSAIDs
47. Purple, pruritic, polygonal plaques - lichen planus
48. Seborrhoeic dermatitis (face and body) , treatment of choice: topical ketoconazole
49. Small vessel, ANCA-associated vasculitis - eosinophilic granulomatosis with polyangiitis
50. Small vessel, ANCA-associated vasculitis - granulomatosis with polyangiitis
51. Small vessel, ANCA-associated vasculitis - microscopic polyangiitis
52. Small vessel, immune complex vasculitis - cryoglobulinaemic vasculitis
53. Small vessel, immune complex vasculitis - Goodpasture's syndrome (anti-glomerular basement membrane
disease)
54. Small vessel, immune complex vasculitis - Henoch-Schonlein purpura
55. Small vessel, immune complex vasculitis - hypocomplementemic urticarial vasculitis (anti-C1q vasculitis)
56. Sulphonamides may cause Steven-Johnson syndrome
57. Tender shin lesions - erythema nodosum
58. Ulcerative colitis, red papule --> necrotic ulcer - pyoderma gangrenosum
158
M Y Elamin
MBBS, DTM&H, MCTM, MRCPI 1& 2
59. White, pruritic, vulval plaques - lichen sclerosus
60. Young adult with an abdominal herald patch, followed by erythematous, oval, scaly patches in a 'fir-tree'
distribution - pityriasis rosea
61. Young child with multiple small, pearly, umbilicated lesions - molluscum contagiosum
62. A non-healing painless ulcer associated with a chronic scar is indicative of squamous cell carcinoma (SCC)
63. A skin biopsy is used to diagnose dermatitis herpetiformis
64. Acne vulgaris in pregnancy - use oral erythromycin if treatment needed
65. Acral lentiginous melanoma: Pigmentation of nail bed affecting proximal nail fold suggests melanoma
(Hutchinson's sign)
66. Acute onset of tear-drop scaly papules on trunk and limbs → ?guttate psoriasis
67. An itchy rash affecting the face and scalp distribution is commonly caused by seborrhoeic dermatitis
68. Antibodies against desmoglein 3 are seen in pemphigus vulgaris
69. An itchy rash affecting the face and scalp distribution is commonly caused by seborrhoeic dermatitis
70. A non-healing painless ulcer associated with a chronic scar is indicative of squamous cell carcinoma (SCC).
Surgical excision with 6mm margins is preferred if the lesion is >20mm in diameter.
71. Beta-blockers are known to exacerbate plaque psoriasis
72. Blisters/bullae
a. no mucosal involvement (in exams at least*): bullous pemphigoid
b. mucosal involvement: pemphigus vulgaris
73. Deficiency of niacin (B3) causes pellagra
74. Dermatitis herpetiformis - caused by IgA deposition in the dermis
75. Dermatitis in acral, peri-orificial and perianal distribution → ?zinc deficiency
76. Dermatophyte nail infections - use oral terbinafine
77. Diabetes, waxy yellow shin lesions - necrobiosis lipoidica diabeticorum
78. Dry skin is the most common side-effect of isotretinoin
79. Eczema herperticum rash can be described as monomorphic punched-out erosions (circular, depressed, ulcerated
lesions) usually 1–3 mm in diameter
80. Eczema herpeticum is a primary infection of the skin caused by herpes simplex virus (HSV) and uncommonly
coxsackievirus
81. Eczema herpeticum is a serious condition that requires IV antivirals
82. Erythema gyratum repens is a paraneoplastic eruption with a 'wood-grain' pattern and figurate erythema commonly
seen in patients with lung cancer
83. Erythema nodosum can be caused by streptococcal infection
84. Erythema nodosum may be caused by pregnancy
85. Extensive umbilicated lesions in HIV - molluscum contagiosum
86. Flexural psoriasis - topical steroid
159
M Y Elamin
MBBS, DTM&H, MCTM, MRCPI 1& 2
87. Grave's disease, orange peel shin lesions - pretibial myxoedema
88. Hepatitis C may lead to porphyria cutanea tarda
89. Hereditary haemorrhagic telangiectasia - autosomal dominant
90. Herpes hominis virus 7 (HHV-7) is thought to play a role in the aetiology of pityriasis rosea
91. HIV is associated with seborrhoeic dermatitis
92. Immune related adverse events in a patient on checkpoint inhibitors (e.g. nivolumab)? Need to treat with
corticosteroids
93. Impetigo - topical fusidic acid if hydrogen peroxide not suitable
94. Isotretinoin adverse effects
a. teratogenicity - females MUST be taking contraception
b. low mood
c. dry eyes and lips
d. raised triglycerides
e. hair thinning
f. nose bleeds
95. Keloid scars - more common in young, black, male adults
96. Keloid scars are most common on the sternum
97. Ketoconazole shampoo is used to treat pityriasis versicolor
98. Lentigo maligna melanoma: Suspicious freckle on face or scalp of chronically sun-exposed patients
99. Lichen
a. planus: purple, pruritic, papular, polygonal rash on flexor surfaces. Wickham's striae over surface.
Oral involvement common
b. sclerosus: itchy white spots typically seen on the vulva of elderly women
100. Lichen sclerosus: itchy white spots typically seen on the vulva of elderly women
101. Linear, well-demarcated skin lesions that appear suddenly, with 'la belle indifference' → ?dermatitis artefacta
102. Livedo reticularis can be caused by SLE
103. Management of venous ulceration - compression bandaging
104. Melanoma: the invasion depth of the tumour is the single most important prognostic factor
105. Mucosal involvement is prominent and severe in Stevens-Johnson syndrome
106. Niacin (B3) deficiency is characterised by dermatitis, diarrhoea and dementia, a condition known as pellagra
107. Nodular melanoma: Invade aggressively and metastasise early
108. Oral lichen planus typically presents with buccal white-lace pattern lesions and ulcers
109. Otitis externa and blepharitis are common complications of seborrhoeic dermatitis
110. Parkinson's disease is associated with seborrhoeic dermatitis
111. Pemphigus vulgaris often presents with mucosal blistering. The skin blisters are often painful but not usually
itchy
160
M Y Elamin
MBBS, DTM&H, MCTM, MRCPI 1& 2
112. Pityriasis versicolor is caused by Malassezia furfur
113. Polymorphic eruption of pregnancy is not associated with blistering
114. Pompholyx eczema is a subtype of eczema characterised by an intensely pruritic rash on the palms and soles
115. Pompholyx eczema may be precipitated by humidity (e.g. sweating) and high temperatures
116. Porphyria cutanea tarda - photosensitive rash with blistering and skin fragility on the face and dorsal aspect
of hands
117. Potent topical steroids are the first-line treatment for lichen planus
118. Psoriasis: lithium may trigger an exacerbation
119. Renal transplant patients - skin cancer (particularly squamous cell) is the most common malignancy secondary
to immunosuppression
120. Rosacea features:
a. Nose, cheeks and forehead
b. Flushing, erythema, telangiectasia → papules and pustules
121. Rosacea treatment:
a. Mild/moderate: topical metronidazole
b. Severe/resistant: oral tetracycline
122. Rosacea: a combination of topical ivermectin + oral doxycycline is first-line for patients with severe papules
and/or pustules
123. Rosacea: topical ivermectin is first-line for patients mild papules and/or pustules
124. Same-day referral to a dermatologist is recommended if eczema herpeticum is suspected
125. Scabies - permethrin treatment: all skin including scalp + leave for 12 hours + retreat in 7 days
126. SCCs arising in a chronic scar are typically more aggressive and carry an increased risk of metastasis
127. Seborrhoeic dermatitis - first-line treatment is topical ketoconazole
128. Tender shin lesions - erythema nodosum
129. The most common malignancy associated with acanthosis nigricans is gastrointestinal adenocarcinoma
130. The most common malignancy in the cell lower lip is a squamous carcinoma
131. Topical aluminium chloride preparations are first-line for hyperhidrosis
132. Topical eflornithine is the treatment of choice for facial hirsutism
133. Topical steroids
a. Moderate: Clobetasone butyrate 0.05%
b. Potent: Betamethasone valerate 0.1%
c. Very potent: Clobetasol propionate 0.05%
134. Urinary histamine is used to diagnose systemic mastocytosis
135. Zinc deficiency caused by total parenteral nutrition (TPN) can result in acrodermatitis herpetiformis
136. An elderly Jewish woman develops painful, flaccid, easily ruptured vesicles and bullae on the skin. They are
not itchy and were preceded by mouth lesions - pemphigus vulgaris
161
M Y Elamin
MBBS, DTM&H, MCTM, MRCPI 1& 2
137. Grave's disease, orange peel shin lesions - pretibial myxoedema
138. Isotretinoin may cause depression
139. Isotretinoin may cause dry mouth
140. Isotretinoin may cause epistaxis
141. Isotretinoin may cause photosensitivity
142. Large vessel vasculitis - Takayasu's arteritis
143. Psoriasis may be worsened by alcohol
144. Purple, pruritic, polygonal plaques - lichen planus
145. Seborrhoeic dermatitis (face and body) , treatment of choice: topical ketoconazole
146. Small vessel, immune complex vasculitis - Henoch-Schonlein purpura
147. Sulphonamides may cause Steven-Johnson syndrome
148. Ulcerative colitis, red papule --> necrotic ulcer - pyoderma gangrenosum
149. White, pruritic, vulval plaques - lichen sclerosus
150. Young adult with an abdominal herald patch, followed by erythematous, oval, scaly patches in a 'fir-tree'
distribution - pityriasis rosea
151. (SJS) is an immune-complex-mediated hypersensitivity complex that is a severe expression of erythema
multiforme. It typically involves the skin and mucous membranes. The drugs most closely associated with causing SJS
include antibacterials, sulfonamides, and anticonvulsants (oxicam).
152. A diagnosis of molluscum contagiosum is obvious when a child presents with pearly white hemispherical lesions
particularly if they are umbilicated over limbs, trunk or face in various stages of evolution.
153. A history of photosensitive eruption, hypertrichosis, and milia formation (keratin-containing cysts) are
characteristic of PCT.
154. A presentation of pruritic vesicles on elbows, knees, and buttocks which have not responded to topical steroids is
the classical presentation of dermatitis herpetiformis (DH).
155. Alopecia areata and vitiligo both have a similar autoimmune aetiology and often co-exist
156. Ankle-brachial pressure index is a simple way of assessing arterial disease in a primary care setting as a first line.
157. Anticentromere antibodies and antitopoisomerase I antibodies are the classic autoantibodies associated with
systemic sclerosis.
158. Anti-streptolysin-O titre (ASOT) is most likely to confirm post-streptococcal erythema nodosum.
159. Arthralgia and arthritis are the most common presenting manifestations of SLE typically affecting the small joints
of the hands, wrists, and knees. The symmetrical rash is the classical butterfly rash that occurs in a malar distribution.
Alopecia is common and may be diffuse or patchy.
160. Blue-grey discolouration of the legs is associated with minocycline use.
161. Botulinum toxin injection to each axilla is the most appropriate treatment for primary hyperhidrosis. This treatment
is licensed for use and would be the preferred treatment before aluminium salts, as antiperspirants have failed.
162. Bullous pemphigoid occurs as a result of IgG antibodies which bind to the skin basement membrane.
162
M Y Elamin
MBBS, DTM&H, MCTM, MRCPI 1& 2
163. Callus formation is an important predictor of ulceration in diabetic feet
164. Candida intertrigo is common amongst obese patients with diabetes, and oral intervention is preferred in those who
have significant skin infection.
165. Complications of pregnancy in SLE
166. Consider that dressings are absorbent and, if not properly maintained, can harbour infection.
167. Cutaneous anthrax is associated with a black eschar without pus, tends to be painless and to have widespread
oedema.
168. Cutaneous leishmaniasis is suggested by painless ulceration after return form an endemic area.
169. Cyclosporin is a well used drug in the treatment of atopic dermatitis (AD). It is usually used as a systemic therapy
when topical treatment with steroids, calcineurin inhibitors (tacrolimus, pimecrolimus) are ineffective. It is not
recommended for use in infants and young children but should be reserved in older children and adeloscents with very
severe disease with a significant negative impact on the quality of life.
170. Deranged liver enzymes and hyperlipidaemia are possible complications of treatment with oral isotretinoin.
171. Dermatitis herpetiformis characteristically has very intensely pruritic vesicles. It is not usually responsive to topical
steroids, but would respond well to dapsone. It is associated with gluten sensitivity and coeliac disease.
172. Dermatophytosis is common in athletes. The affliction on legs and thighs is termed tinea corporis.
173. Dermatophytosis is common in young adults. The affliction of legs and thighs is termed tinea corporis. It is an
occupational dermatoses in athletes and may be recurrent in them if due precautions are not taken.
174. Discoid lupus erythematosus is characterised by discrete plaque lesions, often erythematous, covered by scales that
extend into dilated hair follicles. These lesions most typically occur on the face, scalp, in the pinnae, behind the ears,
and on the neck. They can exist in areas not exposed to the sun.
175. Discoid lupus is photosensitive and so avoiding the sun with protective clothing and sunscreens, and changing
behaviour is important.
176. Eczema herpeticum is related to herpes simplex infection and IV acyclovir is the first-line treatment option.
177. Eisenmengers syndrome is the process in which a long-standing left-to-right cardiac shunt causes pulmonary
hypertension and eventual reversal of the shunt into a cyanotic right-to-left shunt.
178. Erythema induratum (EI) is a form of panniculitis characterised by chronic, recurrent, tender, subcutaneous, and
sometimes ulcerated nodules on the lower legs that may also appear elsewhere
179. Erythema multiforme can be caused by cerain antibiotics, including penicillins.
180. Erythema multiforme is associated with numerous drugs, allopurinol being the most common.
181. Erythema nodosum is a self-limiting condition and resolves on treatment of the underlying
condition/discontinuation of the responsible drug.
182. Erythroderma or exfoliative dermatitis, is a rare but life threatening condition that can occur with psoriasis and
atopic dermatitis, drug hypersensitivity reactions and rare conditions like Sezary syndrome (a leukaemic subtype of T
call lymphoma).
183. Eye toxicity is one of the most serious side effects of treatment with antimalarials and requires regular screening.
163
M Y Elamin
MBBS, DTM&H, MCTM, MRCPI 1& 2
184. First line treatment of asteatotic eczema is with emollients
185. G6PD deficiency is an absolute contraindication to treatment with dapsone as it can lead to severe haemolytic
anaemia.
186. Granuloma annulare is a benign inflammatory condition which leads to the formation of dermal papules, and where
lesions become larger, annular plaques.
187. Guttate psoriasis is classically seen two to three weeks after streptococcal infection, and usually resolves
spontaneously.
188. Hair dye contains substances which may induce an eczematous response in form of contact allergic dermatitis.
189. Hereditary haemorrhagic telangiectasia (HHT) is a multisystem vascular dysplasia characterised by the presence of
multiple arteriovenous malformations (AVMs) that lack intervening capillaries and result in direct connections between
arteries and veins.
190. Hypertension is a complication of therapy with ciclosporin. Renal toxicity and electrolyte abnormalities are other
possible complications.
191. In hypersensitive patients aspirin can cause angioedema, bronchospasm, and urticaria (skin rashes).
192. In malingering, the patient consciously fakes or claims to have a disorder in order to attain a specific gain (for
example, financial).
193. Increased skin fragility is seen in a number of disorders and is used as a clinical test in bullous disorders (Nikolsky's
sign).
194. Infliximab is associated with tuberculosis by reactivation of latent disease. Thus it is advised that patients who are
to be treated with infliximab are tuberculin tested and if required receive chest radiography.
195. Latex allergy is associated with hypersensitivity to other plants, particularly fruits (the latext-fruit syndrome) for
example, banana
196. Linear lesions are rarely caused by organic disease, and should make you query dermatitis artifacta especially when
the lesions are inaccessible locations such as the forearms.
197. Livedo reticularis is due to superficial capillary dilatation, resulting in characteristic mottling of the skin. It is mainly
idiopathic.
198. Löfgren's syndrome is a benign form of sarcoidosis. It presents with erythema nodosum, arthropathy, and bilateral
hilar lymphadenopathy syndrome.
199. Lupus as suggested by the indurated plaques on cheeks, the scarring alopecia and hyperkeratosis over the hair
follicles.
200. Methotrexate would be the only correct treatment for someone with erythrodermic psoriasis.
201. Neurofibromatosis with axillary freckling and neurofibromas is usually inherited as autosomal dominant although
it may arise from a sporadic new mutation of the NF1 gene.
202. Non-itchy, circular, raised, smooth-surfaced lesions on the elbows, extensor aspects of the forearms and knuckles
and the raised borders are suggestive of granuloma annulare.
203. Patients on long-term immunosuppression are at risk of malignancy; most commonly skin BCCs and SCCs.
164
M Y Elamin
MBBS, DTM&H, MCTM, MRCPI 1& 2
204. Patients with bullous pemphigoid typically present with tense blisters on arms, trunk and legs. They are otherwise
well and there is no mucosal involvement.
205. PCT causes blistering of the hands and the forehead which usually heal with small scar and milia formation.
206. Pellagra is aggravated by exposure to sunlight.
207. Pemphigoid, erythema multiforme, and herpes are the commonest causes of a blistering rash.
208. Pemphigus is classically associated with flaccid blistering with oral involvment , and often with immunoglobulin
(Ig)G antibodies.
209. Pigmented spots on the iris are a characteristic feature of neurofibromatosis type 1 (NF1).
210. Pityriasis rosea is characterised by flat scaly patches.
211. Pityriasis versicolor (also called tinea versicolor) is a skin lesion caused by a fungus called Malassezia furfur. The
treatment is topical selenium sulphide.
212. Pityriasis versicolor is treated initially with topical imidazole creams, such as ketoconazole cream.
213. Pityriasis versicolor usually presents as slightly scaly hypopigmented lesions. Growth is encouraged by an increase
in temperature and suntan oils, and is most commonly seen after a sun holiday.
214. Psoriatic arthropathy precedes cutaneous lesions in roughly 20% of cases.
215. Rapid onset of renal failure, coupled with a rash and eosinophilia is highly suspicious of a diagnosis of interstitial
nephritis
216. Regular washing of the hair with a ketoconazole based shampoo has been shown to reduce the severity of
seborrhoeic dermatitis on the scalp.
217. Scabies causes itchiness in the skin even where there is no obvious lesion to be seen.
218. Scabies is treated with permethrin, which should be repeated after seven days, and all close contacts should also be
treated.
219. Survival is strongly correlated with depth of melanoma at the point of diagnosis, with lesions over 4 mm thick being
associated with a particularly poor outcome.
220. Symmetry is suspicious characteristic of malignant melanoma.
221. T cells play a prominent role in the pathogenesis of psoriasis.
222. Telogen effluvium describes a state whereby stress or shock to the system causes as many as 70% of the scalp hairs
to be precipitated into a resting state, thus reversing the usual ratio.
223. Terbinafine is recommended as first line treatment for fungal nail infection (onychomycosis) because it is effective
against both dermatophytes and Candida species, whereas the '-azoles' (e.g. fluconazole) do not have as much efficacy
against dermatophytes
224. Tetracycline is a recognised cause of onycholysis together with eczema, psoriasis and thyrotoxicosis.
225. The barrier function of the skin is provided by the outermost layer, the stratum corneum.
226. The Koebner phenomenon refers to the appearance of lesions along the site of an injury or scar. It can occur in
psoriasis, molluscum contagiosum, lichen planus and other conditions.

165
M Y Elamin
MBBS, DTM&H, MCTM, MRCPI 1& 2
227. The mainstay of treatment of venous ulceration is compression therapy, which aims to improve venous return and
thereby reduce venous hypertension.
228. The most appropriate treatment for acne rosacea is a tetracycline.
229. The most common presenting sign of porphyria cutanea tarda (PCT) is fragility of sun exposed skin after mechanical
trauma, leading to erosions and bullae, worst on dorsal hands, forearms and face.
230. This patient had non-scarring alopecia due to invasion of hairs by dermatophytes, most commonly Trichophyton
tonsurans.
231. To diagnose neurofibromatosis, two of the following criteria must be present: six or more café-au-lait spots, two or
more neurofibromas, axillary freckling, optic glioma, lisch nodules or positive family history.
232. Topical antifungals are the treatment of choice for pityriasis versicolor; if the depigmentation is extensive, a
systemic compound like fluconazole may be used.
233. Toxic epidermal necrolysis (TEN) can be caused by carbamazepine therapy. TEN is a severe mucocutaneous
exfoliative disease with an uncertain pathogenesis and a high mortality rate.
234. Tuberous sclerosis, is inherited in autosomal dominant fashion, with responsible defects having been identified on
both chromosome 9 and chromosome 16.
235. Uniform pigmentation is not in itself a suspicious feature of a mole, but colour variegation and irregular border are
two of many suspicious features
236. Uritcaria can be classified into idiopathic, immune, or non-immune. Up to 50% of cases are idiopathic, and no
trigger is readily identifiable.
237. Urticaria is a common condition and usually responds very well to systemic antihistamines which are the correct
first line treatment.
238. Vitiligo is associated with a number of autoimmune conditions, the most common of which is thyroid disease.
239. Vitiligo is associated with thyroid disease, pernicious anaemia, and diabetes mellitus (in descending order).
240. Wheeze, stridor and urticaria is more in keeping with allergy than MSG syndrome following Chinese takeaway
241. Zinc deficiency can lead to acrodermatitis which presents with perioral dermatitis, acral involvement, and
sometimes alopecia and is common in patients following bowel resection due to lack of absorption from the small
bowel.
242. Zinc is a component of many of the enzymes responsible for wound healing.A plaque is a descriptive term for a
skin lesion that is raised and greater than 1 cm in diameter.
243. A single herald patch followed by multiple erythematous plaques is characteristic of Pityriasis rosea.
244. age
245. Alopecia areata is common in children as well as adults presenting with localised patches of non-scarring hair loss
and remaining hairs have a characteristic 'exclamation mark' appearance.
246. Angiosarcomas are malignant vascular tumours most commonly seen in elderly men on the scalp and forehead and
present with an infiltrative vascular patch or plaque with super-imposed nodules which may bleed with minor trauma
247. Anticentromere antibodies are strongly associated with CREST syndrome.
166
M Y Elamin
MBBS, DTM&H, MCTM, MRCPI 1& 2
248. Antithyroid antibodies are commonly associated with pre-tibial myxoedema.
249. Basal cell carcinoma presents with a typical 'rodent ulcer' with rolled pearly edges and small telangiectasias. It is
the commonest malignant skin tumour and most commonly occurs in elderly patients with sun-damaged skin.
250. Beau's lines, a benign nail condition that presents as a jagged transverse groove on the nail plate which can occur
during or after an episode of severe medical illness, such as sepsis.
251. Biological immune modifying agents licensed to treat moderate to severe psoriasis And should be used when first
line agents fail.
252. Botulinum toxin type A (or trade name Botox®) acts by decreasing transmission of acetylcholine across the
neuromuscular junction and can induce temporary flaccid paralysis.
253. Bullous pemphigoid is an autoimmune condition that primarily affects the over 60s, leading to the formation of
subepidermal blisters
254. bullous pemphigoid presents with multiple tense blisters that rupture with residual erosions and on skin biopsy will
reveal a subepidermal blister with an infiltrate of eosinophils.
255. Carbamezapine is a recognised cause of TEN.
256. Clinical features of Behcet's disease include the classical triad of: recurrent oral ulceration, recurrent genital
ulceration, and iritis.
257. Consider referring to an ear, nose, and throat specialist if guttate psoriasis is exacerbated by recurrent tonsillitis.
258. Cutaneous larva migrans is caused by hookworms and should be suspected in patients with suspicious lesions
following a beach holiday.
259. Cutaneous TB usually occurs due to spread from an endogenous source with most cases presenting on the face or
neck.
260. Dermatomyositis is very rarely seen in primary care but can be associated with lung, ovary and GI malignancies.
261. Discoid lupus erythematosus presents with scaly, hyperpigmented, erythematous plaques on sun-exposed areas and
may lead to severe scarring.
262. Eczema Herpeticum can be secondary infected with Staphylococcus and require additional antibiotics.
263. Eruptive xanthomata occur in hyperlipidaemic states associated with hypertriglyceridaemia.
264. Erysipelas is a Streptococcus pyogenes infection of the deep dermis and subcutis.
265. Erythema ab igne classically presents on the front of the legs due to the patient sitting too close to a fire or heater,
due to the lethargy and weight gain the next most important test is TFTs
266. Erythema multiforme typically presents with target lesions, cause may be multifactorial
267. Erythema nodosum is characterised by the presence of tender subcutaneous nodules, usually on the shins
268. Erythema nodosum is commonly idiopathic, but there should be a high index of suspicion for sarcoidosis.
269. Erythema nodosum lesions tend to last six to eight weeks.
270. First line treatment of scalp psoriasis includes topical corticosteroid preparations as well as medicated shampoos
containing zinc, coal tar and salicylic acid.
271. Gastric CA may progress from gastric ulcer and may present with Acanthosis nigricans
167
M Y Elamin
MBBS, DTM&H, MCTM, MRCPI 1& 2
272. Glucagonoma is associated with necrolytic migratory erythema
273. Group B Streptococcus has a predilection for cellulitis in diabetes
274. Heart block is a significant sequela of neonatal SLE
275. Hereditary haemorrhagic telangectasia is usually an autosomal dominant condition that may present with anaemia
due to bleeding from occult GI telangectasia
276. HZV distributes in a dermatomal fashion.
277. Immediate treatment of TEN is to stop the offending drug as well as Intensive care support and urgent Dermatology
review..
278. In extensive bullous pemphigoid oral corticosteroids are first line treatment.
279. In guttate psoriasis, if lesions are not widespread (<10% body surface area) and the person is not concerned about
the appearance (it is not having an impact on their physical, psychological, or social well being), then no treatment is
an option as the condition is self-limiting.
280. In guttate psoriasis, if lesions are widespread (greater than 10% body surface area), refer urgently to a dermatologist
for consideration of phototherapy.
281. In respect to Anthrax infection cutaneous disease is the commonest form of the infection in humans.
282. Kaposis sarcoma is a tricky and rare diagnosis in primary care that presents as red to purplish macules and raised
papules/nodules anywhere on the skin or mucous membranes
283. Keratacanthomas are characterised by very rapidly growing lesions with a central keratinised plug. They should be
referred under the 2 week wait as they can be difficult to distinguish from invasive SCC
284. Keratoacanthomas are well-differentiated variant of squamous cell carcinoma presenting as a rapidly enlarging
nodule with a central keratinous crateriform scab.
285. Lichen simplex chronicus (LSC) presents with hyperpigmented, scaly, lichenified plaques commonly seen on the
ankles.
286. Lichenification of the skin is due to epidermal thickening characterised by visible and palpable thickening of the
skin with accentuation of skin markings.
287. Manifestations and management of hereditary haemorrhagic telangectasia
288. Molluscum Contagiosum is caused by a DNA pox virus and are associated with HIV / AIDS
289. Mycobacterium marinum is a skin infection found in ornamental fish and is commonly seen in individuals who rear
fish as a hobby.
290. Mycosis fungoides (a cutaneous T cell lymphoma) presents as a pruritic eczematous rash and develops
telangiectasias and areas of 'cigarette paper' atrophy.
291. Necrobiosis lipoidica is a typically painless lesion assoicated with diabetes
292. Necrobiosis lipoidica is commonly associated with diabetes and is painless lesion due to collagen degeneration.
293. Necrobiosis lipoidica is commonly associated with diabetes. It is a painless lesion and secondary to collagen
degeneration.
294. Papules with central umbilication are associated with molluscum contagiosum.
168
M Y Elamin
MBBS, DTM&H, MCTM, MRCPI 1& 2
295. Patients presenting with a chancre of primary syphilis should have dark ground microscopy to confirm the diagnosis.
296. PCOS is associated with insulin resistance; acanthosis nigricans may be present.
297. PCT may be precipitated by alcohol, exposure to sunlight, iron and oestrogen
298. Polymorphous light eruption presents most commonly in young females and commonly occurs a few hours after
sun-exposure and resolves after a few days, without scarring.
299. Porphyria cutanea tarda (PCT) is associated with deficiency of hepatic uroporphyrinogen (URO) decarboxylase.
The major clinical feature is cutaneous photosensitivity. Bullae develop on sun-exposed areas and lesions heal slowly,
leaving scars.
300. Porphyria cutanea tarda (PCT) is associated with reduced hepatic uroporphyrinogen decarboxylase activity.
301. Porphyria cutanea tarda is usually seen in men who have damaged their liver by drinking too much alcohol.
302. Pustules are small elevations of the skin containing cloudy or purulent material, usually consisting of necrotic
inflammatory cells.
303. PUVA is an effective treatment for psoriasis and may be useful where compliance with topical therapy may be an
issue
304. Pyoderma gangrenosum presents as an expanding ulcer with a polycyclic or serpiginous outline and a characteristic
undermined bluish edge which is responsive to oral steroids.
305. Pyogenic granulomata are usually solitary lesions, appearing as a glistening red papule or nodule that is prone to
bleeding and ulceration developing rapidly over a few weeks.
306. Reduced DLCO is an early indication of pulmonary disease in systemic sclerosis.
307. Scabies is a highly contagious disease caused by the mite Sarcoptes scabiei which causes an intense pruritis and can
be treated with permethrin cream
308. Scurvy is a result of vitamin C deficiency and is important for collagen produciton. The characteristic rash is peri-
follicuar haemorrhage
309. Several drugs are known to exacerbate or trigger the onset of psoriasis, including beta blockers, lithium, NSAIDs
and anti-malarials.
310. Skin appearance and systemic manifestations of pseudoxanthoma elasticum
311. Skin lesions associated with gastrointetinal disease
312. Streptococcal infection can precede the onset of guttate psoriasis, which presents with an itchy rash and sore throat.
313. Symptoms of cutaneous worm infections include rash and itching. Treatment may be topical (thiabendzole) or oral
(albendazole)
314. The characteristic cutaneous features of dermatomyositis are heliotrope rash and Gottron's papules.
315. The clinical appearance of pemphigus vulgaris and laboratory tests used to confirm diagnosis
316. toxic epidermal necrolysis (TEN), a severe idiosyncratic drug reaction commonly caused by drugs such as anti-
epileptics, penicillins, NSAIDS and allopurinol. Nikolsky's sign is elicited when rubbing of the skin results in
desquamation.

169
M Y Elamin
MBBS, DTM&H, MCTM, MRCPI 1& 2
317. Treatment of erysipelas is with the penicillins, but macrolides and cephalosporins can also be used.
Chloramphenicol use is limited due to risk of aplastic anaemia.
318. Tuberous sclerosis presents with flesh coloured papules (angiofibromas) on the face and fibromas in the central
nervous system
319. Typical features of systemic sclerosis include facial telangiectasia, small mouth, and beaking of the nose.
320. Varicella pneumonia occurs in up to 20% of adults with chickenpox, appearing three to five days into the course of
the illness. In adults with pneumonitis, treatment with aciclovir is warranted.
321. Vitiligo is associated with autoimmune conditions such as autoimmune thyroiditis or diabetes.
322. Yellow nail syndrome is caused by hypoplastic lymphatics and is characterised by the triad of lymphoedema, pleural
effusions, and yellow discolouration of the nails.
323.

170
M Y Elamin
MBBS, DTM&H, MCTM, MRCPI 1& 2
ENDOCRINOLOGY
1. A 25-year-old woman with a history of irregular periods complains of hirsutism. Blood tests show an elevated
LH:FSH ratio and raised testosterone level - polycystic ovarian syndrome
2. A 2-month-old baby is noted to have hypotonia, macroglossia and a puffy face. They were treated at birth for
neonatal jaundice - congenital hypothyroidism
3. A 2-year-old is investigated for an abdominal mass and pallor. Investigations reveal raised urinary
vanillylmandelic acid (VMA) and homovanillic acid (HVA) levels - neuroblastoma
4. A 30-year-old woman is investigated for amenorrhoea, galactorrhoea and a low libido - prolactinoma
5. A 30-year-old woman presents with weight gain and irregular menstruation. Her blood pressure is elevated at
170/100 mmHg and there is evidence of proximal muscle weakness - Cushing's syndrome
6. A 30-year-old woman with a history of recurrent peptic ulcer disease is found to have hypercalcaemia on routine
bloods. - multiple endocrine neoplasia (type 1)
7. A 35-year-old woman is found to have a blood pressure of 180/110 mmHg. She complains of feeling tired and weak.
Routine bloods show hypokalaemia - primary hyperaldosteronism
8. A 40-year-old man presents with headaches and coarsening of his facial features. He says his wedding ring and
shoes no longer fit him - acromegaly
9. A 40-year-old patient with a history of hypertension presents with episodic palpitations, excessive sweating,
headaches and tremor - phaeochromocytoma
10. A 40-year-old woman presents with lethargy, weakness and weight loss. On examination her blood pressure is 80/50
mmHg and there is hyperpigmentation of the skin - Addison's disease
11. A 45-year-old woman with a history of retroperitoneal fibrosis is found to have hypothyroidism. On examination
a hard, fixed, painless goitre is noted - Riedel's thyroiditis
12. A 60-year-old obese, nulliparous woman presents with vaginal bleeding - endometrial cancer
13. A 60-year-old woman is investigated for heat intolerance and weight loss. The free T4 level is elevated and the TSH
is suppressed. Nuclear scintigraphy reveals an enlarged thyroid gland with patchy uptake - toxic multinodular
goitre
14. A 60-year-old woman presents lower abdominal discomfort, urinary frequency, bloating and abdominal
distension - ovarian cancer
15. A baby is born with ambiguous genitalia, exhibiting labioscrotal folds with clitoromegaly. At 13 years of age the
child undergoes virilization with facial hair and deepening of the voice - 5-alpha reductase deficiency
16. A man with a history of medullary thyroid cancer and phaeochromocytoma is noted to have a tall, thin,
'marfanoid' habitus. A number of small lumps are noted on his eyelids and lips - multiple endocrine neoplasia
(type 2B)
17. A middle aged woman presents with a tender goitre and symptoms of hyperthyroidism. Bloods show a suppressed
TSH and raised T4. There is a globally reduced uptake on iodine-131 scan - subacute thyroiditis
171
M Y Elamin
MBBS, DTM&H, MCTM, MRCPI 1& 2
18. A middle aged woman presents with thyrotoxicosis and a goitre. On examination there is exophthalmos and
pretibial myxoedema. Anti-TSH receptor stimulating antibodies are positive - Graves' disease
19. A middle-aged women presents with symptoms of hypothyroidism. There is a diffuse, non-tender goitre on
examination. TSH is raised, T4 is low, anti-TPO is positive - Hashimoto's thyroiditis
20. A newborn baby is noted to have ambiguous genitalia. At 2 weeks they develop severe vomiting, dehydration and
weight loss - congenital adrenal hyperplasia (21-hydroxylase deficiency)
21. A patient is diagnosed as having medullary thyroid cancer. Her past medical history includes phaeochromocytoma
and she is currently undergoing investigations for hypercalcaemia - multiple endocrine neoplasia (type 2A)
22. A patient is investigated for episodic confusion, sweating, dizziness and headches. The symptoms often improve
after eating - insulinoma
23. A patient presents with muscle cramping and perioral paraesthesia two months after having thyroid surgery -
primary hypoparathyroidism
24. A tall, slim 18-year-old man presents with hypogonadism. He also complains of anosmia. Gonadotrophin levels
are reduced - Kallman's syndrome
25. A tall, slim man presents with infertility. He is noted to have small, firm testes. Gonadotrophin levels are elevated
- Klinefelter's syndrome
26. A teenage girl is investigated for primary amenorrhoea, despite having developed secondary sexual
characteristics. On examination she has well developed breasts with scanty pubic hair and blind-ending vagina -
androgen insensitivity syndrome
27. ACE inhibitors causes hyperkalaemia & first dose hypertention
28. Acetazolamide causes hypokalaemia
29. Acromegaly - serum IGF-1 measurements
30. Acromegaly causes hypercalcaemia
31. Acute renal failure causes hyperkalaemia
32. Addison's disease - ACTH stimulation test
33. Addison's disease - hyperpigmentation of the palmar creases
34. Addison's disease causes hypercalcaemia
35. Addison's disease causes hyperkalaemia
36. Addison's disease causes hyponatraemia
37. Adenoma of rectum causes hyponatraemia
38. Adrenal adenoma - cortisol: not suppressed, ACTH: suppressed
39. An elderly patient presents with bone pain, depression, renal stones and recurrent peptic ulcer - primary
hyperparathyroidism
40. An overweight 25-year-old female presents with infertility. She has a history of irregular periods, hirsutism and
acne - polycystic ovarian syndrome

172
M Y Elamin
MBBS, DTM&H, MCTM, MRCPI 1& 2
41. An overweight 50-year-old man presents with lethargy and excessive thirst. Fundoscopy reveals blot
haemorrhages - type 2 diabetes mellitus
42. Anaplastic thyroid cancer - the worse prognosis of the thyroid cancers
43. Androgen insensitivity syndrome - end-organ resistance to testosterone
44. Angiotensin 2 receptor blockers causes hyperkalaemia
45. Bartter's syndrome - defect in the Na+ K+ 2Cl- cotransporter (NKCC2)
46. Bartter's syndrome associated with normotension
47. Bone metastases causes hypercalcaemia
48. Burns causes hyponatraemia
49. Carbimazole - blocks thyroid peroxidase from coupling and iodinating the tyrosine residues on thyroglobulin --
> reducing thyroid hormone production + no action on 5'-deiodinase
50. Carbimazole may cause myelosuppression/agranulocytosis
51. Cervical cancer - risk factors include: combined oral contraceptive pill use
52. Cervical cancer - risk factors include: early first intercourse
53. Cervical cancer - risk factors include: high parity
54. Cervical cancer - risk factors include: human immunodeficiency virus
55. Cervical cancer - risk factors include: human papilloma virus 16,18 & 33
56. Cervical cancer - risk factors include: lower socioeconomic status
57. Cervical cancer - risk factors include: many sexual partners
58. Cervical cancer - risk factors include: smoking
59. Chronic kidney disease causes hypocalcaemia
60. Ciclosporin causes hyperkalaemia
61. Corticosteroids may cause avascular necrosis of the femoral head
62. Corticosteroids may cause cataracts
63. Corticosteroids may cause Cushing's syndrome
64. Corticosteroids may cause depression
65. Corticosteroids may cause fluid retention
66. Corticosteroids may cause glaucoma
67. Corticosteroids may cause hirsutism
68. Corticosteroids may cause hyperlipidaemia
69. Corticosteroids may cause hypertension
70. Corticosteroids may cause impaired glucose tolerance
71. Corticosteroids may cause increased susceptibility to severe infection
72. Corticosteroids may cause insomnia
73. Corticosteroids may cause intracranial hypertension
74. Corticosteroids may cause mania
173
M Y Elamin
MBBS, DTM&H, MCTM, MRCPI 1& 2
75. Corticosteroids may cause osteoporosis
76. Corticosteroids may cause pancreatitis
77. Corticosteroids may cause peptic ulceration
78. Corticosteroids may cause proximal myopathy
79. Corticosteroids may cause psychosis
80. Corticosteroids may cause reactivation of tuberculosis
81. Corticosteroids may cause suppression of growth in children
82. Corticosteroids may cause weight gain
83. Cranial diabetes insipidus - starting plasma osmolality: high - urine osmolality after fluid deprivation: low - urine
osmolality after desmopressin: high
84. Cushing's syndrome - abdominal striae
85. Cushing's syndrome - overnight dexamethasone suppression test
86. Cushing's syndrome associated with hypertension
87. Cushing's syndrome causes hypokalaemia
88. Deep, laboured, gasping respiration - diabetic ketoacidosis
89. Dehydration causes hypercalcaemia
90. Dehydration causes hypernatraemia
91. Demeclocycline - ADH antagonist
92. Diabetes insipidus causes hypernatraemia
93. Diarrhoea associated with normotension
94. Diarrhoea causes hypokalaemia
95. Diarrhoea causes hyponatraemia
96. Diuretics associated with normotension
97. Ectopic source of ACTH - cortisol: not suppressed, ACTH: not suppressed
98. Endometrial cancer - risk factors include: diabetes mellitus
99. Endometrial cancer - risk factors include: early menarche
100. Endometrial cancer - risk factors include: hereditary non-polyposis colorectal carcinoma
101. Endometrial cancer - risk factors include: late menopause
102. Endometrial cancer - risk factors include: nulliparity
103. Endometrial cancer - risk factors include: obesity
104. Endometrial cancer - risk factors include: polycystic ovarian syndrome
105. Endometrial cancer - risk factors include: tamoxifen
106. Endometrial cancer - risk factors include: unopposed oestrogen
107. Exenatide - glucagon-like peptide-1 mimetic
108. Ezetimibe - inhibits cholesterol receptors on enterocytes, decreasing cholesterol absorption in the small intestine
109. Failure to thrive, polyuria, hypokalaemia - Bartter's syndrome
174
M Y Elamin
MBBS, DTM&H, MCTM, MRCPI 1& 2
110. Gitelman syndrome associated with normotension
111. Gitelman's syndrome - defect in the thiazide-sensitive Na+ Cl- transporter
112. Graves' disease - pretibial myxoedema
113. Hashimoto's thyroiditis - goitre
114. Heart failure causes hyponatraemia
115. Heat intolerance, tremor, palpitations - thyrotoxicosis
116. High LH and low testosterone - Klinefelter's syndrome
117. High LH and normal/high testosterone - androgen insensitivity syndrome
118. Hyperkalaemia, hyponatraemia, hypoglycaemia - Addison's disease
119. Hyperkalaemia, low aldosterone, diabetic patient on ACE-inhiibitor - type 4 renal tubular acidosis
120. Hyperosmolar non-ketotic diabetic coma causes hypernatraemia
121. Hypokalaemia, hypertension, alkalosis, family history of similar problems, low aldosterone - Liddle's syndrome
122. Hypokalaemia, hypertension, alkalosis, no similar family history, raised aldosterone - Conn's syndrome
123. Hypokalaemia, nephrocalcinosis - type 1 renal tubular acidosis
124. Hypokalaemia, osteomalacia - type 2 renal tubular acidosis
125. Hypoparathyroidism causes hypocalcaemia
126. Hypotension, hyperpigmentation, lethargy - Addison's disease
127. Hypothyroidism causes hyponatraemia
128. Insulinoma - supervised fasting
129. Intermittent headache, palpitations, hypertension - phaechromocytoma
130. Klinefelter's syndrome - elevated gonadotrophin levels
131. Klinefelter's syndrome - gynaecomastia
132. Klinefelter's syndrome - karyotype
133. Klinefelter's syndrome - small, firm testes
134. Klinefelter's syndrome - subfertility/infertility
135. Klinefelter's syndrome - tall stature
136. Lethargy, weight gain, cold intolerance - hypothyroidism
137. Liddle's syndrome - disordered sodium channels in the distal tubules leading to increased reabsorption of sodium
138. Liddle's syndrome , treatment of choice: amiloride
139. Liddle's syndrome associated with hypertension
140. Liver cirrhosis causes hyponatraemia
141. Low LH and high testosterone - testosterone-secreting tumour
142. Low LH and low testosterone - Kallman's syndrome
143. Magnesium deficiency causes hypocalcaemia
144. Magnesium deficiency causes hypokalaemia
145. Massive blood transfusion causes hyperkalaemia
175
M Y Elamin
MBBS, DTM&H, MCTM, MRCPI 1& 2
146. Medullary thyroid cancer - MEN-2
147. Medullary thyroid cancer, hypercalcaemia, phaeochromocytoma - multiple endocrine neoplasia type IIa
148. Medullary thyroid cancer, phaeochromocytoma, marfanoid body habitus - multiple endocrine neoplasia type IIb
149. Meglitinides - bind to an ATP-dependent K+(KATP) channel on the cell membrane of pancreatic beta cells
150. Meglitinides - increase pancreatic insulin secretion
151. Meglitinides may cause hypoglycaemia
152. Meglitinides may cause weight gain
153. MEN type I - parathyroid, pituitary, pancreas lesions
154. MEN type IIa - parathyroid, phaeochromocytoma lesions; medullary thyroid cancer
155. MEN type IIb - phaeochromocytoma lesions; medullary thyroid cancer; marfanoid body habitus; neuromas
156. Metabolic acidosis causes hyperkalaemia
157. Milk-alkali syndrome causes hypercalcaemia
158. MODY 2 - defect in the glucokinase gene
159. MODY 3 - defect in the HNF-1 alpha gene
160. Nephrogenic diabetes insipidus - starting plasma osmolality: high - urine osmolality after fluid deprivation: low -
urine osmolality after desmopressin: low
161. Neuropathic pain (diabetic) , treatment of choice: amitriptyline or duloxetine or gabapentin or pregabalin
162. Newly diagnosed T2DM, failed trial of lifestyle interventions - add metformin
163. Normal - starting plasma osmolality: normal - urine osmolality after fluid deprivation: high - urine osmolality
after desmopressin: high
164. Octreotide , uses include: acromegaly
165. Ovarian cancer - risk factors include: early menarche
166. Ovarian cancer - risk factors include: late menopause
167. Ovarian cancer - risk factors include: mutations of the BRCA1 gene
168. Ovarian cancer - risk factors include: mutations of the BRCA2 gene
169. Ovarian cancer - risk factors include: nulliparity
170. Oxybutynin - antimuscarinic
171. Oxybutynin , uses include: urinary incontinence
172. Papillary thyroid cancer - the best prognosis of the thyroid cancers
173. Papillary thyroid cancer - the most common type of thyroid cancer
174. Pegvisomant - growth hormone receptor antagonist
175. Pegvisomant , uses include: acromegaly
176. Peptic ulceration, galactorrhoea, hypercalcaemia - multiple endocrine neoplasia type I
177. Phaeochromocytoma - 24 hr urinary collection of metanephrines
178. Pioglitazone - agonist of the PPAR-gamma receptor
179. Pioglitazone - contraindicated by: heart failure
176
M Y Elamin
MBBS, DTM&H, MCTM, MRCPI 1& 2
180. Pioglitazone may cause bladder cancer
181. Pioglitazone may cause fluid retention
182. Pioglitazone may cause hepatotoxicity
183. Pioglitazone may cause increased risk of fractures
184. Pioglitazone may cause weight gain
185. Pituitary adenoma - cortisol: suppressed, ACTH: suppressed
186. Primary hyperaldosteronism associated with hypertension
187. Primary hyperaldosteronism causes hypokalaemia
188. Primary hyperparathyroidism causes hypercalcaemia
189. Primary polydipsia - starting plasma osmolality: low - urine osmolality after fluid deprivation: high - urine
osmolality after desmopressin: high
190. Propylthiouracil - blocks thyroid peroxidase from coupling and iodinating the tyrosine residues on thyroglobulin
--> reducing thyroid hormone production + inhibits 5'-deiodinase which reduces peripheral conversion of T4 to
T3
191. Pseudohypoparathyroidism causes hypocalcaemia
192. Psychogenic polydipsia causes hyponatraemia
193. Renal tubular acidosis (type 4) causes hyperkalaemia
194. Renal tubular acidosis (types 1 and 2) causes hypokalaemia
195. Renal tubular acidosis type 1 and 2 associated with normotension
196. Rhabdomyolysis causes hyperkalaemia
197. Rhabdomyolysis causes hypocalcaemia
198. Riedel's thyroiditis - goitre
199. Sarcoidosis causes hypercalcaemia
200. SIADH causes hyponatraemia
201. Sitagliptin - dipeptidyl peptidase-4 inhibitors
202. Spironolactone , uses include: primary hyperaldosteronism
203. Spironolactone causes hyperkalaemia
204. Squamous cell lung cancer causes hypercalcaemia
205. Subacute thyroiditis - globally reduced uptake on iodine-131 scan
206. Subacute thyroiditis - goitre
207. Subacute thyroiditis - raised ESR
208. Sulfonylureas - bind to an ATP-dependent K+(KATP) channel on the cell membrane of pancreatic beta cells
209. Sulfonylureas - increase pancreatic insulin secretion
210. Sulfonylureas may cause hepatotoxicity
211. Sulfonylureas may cause hypoglycaemia
212. Sulfonylureas may cause hyponatraemia
177
M Y Elamin
MBBS, DTM&H, MCTM, MRCPI 1& 2
213. Sulfonylureas may cause weight gain
214. Systemic illness, abnormal TFTs with low T3 - sick euthyroid syndrome
215. T2DM, inadequate control on metformin, at risk from consequences of hypoglycaemia - add a DPP-4 inhibitor
or a thiazolidinedione
216. T2DM, inadequate control on metformin, not at risk from hypoglycaemia - add a sulfonylurea
217. Thiazides causes hypercalcaemia
218. Thiazides causes hypokalaemia
219. Thiazides causes hyponatraemia
220. Thyroid lymphoma - Hashimoto's thyroiditis
221. Thyrotoxicosis causes hypercalcaemia
222. TSH: High, T4: Low - primary hypothyroidism
223. TSH: High, T4: Normal - poor compliance with thyroxine
224. TSH: High, T4: Normal - subclinical hypothyroidism
225. TSH: Low, T4: High - thyrotoxicosis
226. TSH: Low, T4: Low - secondary hypothyroidism
227. TSH: Low, T4: Low - sick euthyroid syndrome
228. Type 1 renal tubular acidosis complication - renal stones
229. Type 1 renal tubular acidosis may be caused by rheumatoid arthritis
230. Type 1 renal tubular acidosis may be caused by Sjogren's syndrome
231. Type 1 renal tubular acidosis may be caused by SLE
232. Type 2 renal tubular acidosis complication - osteomalacia
233. Type 2 renal tubular acidosis may be caused by cystinosis
234. Type 2 renal tubular acidosis may be caused by Fanconi syndrome
235. Type 2 renal tubular acidosis may be caused by Wilson's disease
236. Type 4 renal tubular acidosis may be caused by diabetes mellitus
237. Type 4 renal tubular acidosis may be caused by hypoaldosteronism
238. Vitamin D deficiency causes hypocalcaemia
239. Vitamin D intoxication causes hypercalcaemia
240. Vomiting causes hypokalaemia
241. Vomiting causes hyponatraemia
242. Alcohol results in exaggerated insulin secretion → hypoglycaemia. The mechanism is thought to be due to the
effect of alcohol on the pancreatic microcirculation → redistribution of pancreatic blood flow from the exocrine
into the endocrine parts → increased insulin sec retion
243. 11-beta hydroxylase deficiency associated with hypertension
244. 9 am cortisol between 100-500nmol/l is inconclusive and requires further investigation with a short synacthen
test
178
M Y Elamin
MBBS, DTM&H, MCTM, MRCPI 1& 2
245. PHaeochromocytoma - give PHenoxybenzamine before beta-blockers
246. A 10 g monofilament should be used to assess for diabetic neuropathy in the feet
247. A high glycaemic index food has a greater ability to raise blood glucose compared with glucose in normal
glucose-tolerant individuals
248. A normal short synacthen test does not exclude adrenocortical insufficiency due to pituitary failure
249. A plasma aldosterone/renin ratio is the first-line investigation in suspected primary hyperaldosteronism
250. Acetazolamide causes hypokalaemia
251. Acromegaly is caused by excessive growth hormone. Somatostatin directly inhibits the release of growth
hormone, and hence somatostatin analogues are used to treat acromegaly
252. Acromegaly: increased sweating is caused by sweat gland hypertrophy
253. Acute severe hyponatraemia can cause cerebral oedema
254. Addison's disease is associated with a metabolic acidosis
255. Alcohol results in exaggerated insulin secretion → hypoglycaemia. The mechanism is thought to be due to the
effect of alcohol on the pancreatic microcirculation → redistribution of pancreatic blood flow from the exocrine
into the endocrine parts → increased insulin secretion
256. Anaplastic thyroid cancer - aggressive, difficult to treat and often causes pressure symptoms
257. Anticholinergics for urge incontinence are associated with confusion in elderly people - mirabegron is a
preferable alternative
258. Bartter's syndrome is associated with normotension
259. Bartter's syndrome results from a defective NKCC2 channel in the ascending loop of Henle
260. Bilateral idiopathic adrenal hyperplasia is the most common cause of primary hyperaldosteronism
261. Carbimazole blocks thyroid peroxidase from coupling and iodinating the tyrosine residues on thyroglobulin
→ reducing thyroid hormone production
262. Causes of raised prolactin - the p's
a. Pregnancy
b. Prolactinoma
c. Physiological
d. Polycystic ovarian syndrome
e. Primary hypothyroidism
f. Phenothiazines, metoclopramide, domperidone
263. Cerebral oedema is an important complication of fluid resuscitation in DKA, especially in young patients
264. Cervical cancer: Human papillomavirus infection (particularly 16,18 & 33) is by far the most important risk
factor
265. Cinacalcet is a calcimimetic - a drug that 'mimics' the action of calcium on tissue by allosteric activation of the
calcium-sensing receptor
266. Congenital adrenal hyperplasia is most commonly due to 21-hydroxylase deficiency
179
M Y Elamin
MBBS, DTM&H, MCTM, MRCPI 1& 2
267. Cushing's syndrome - hypokalaemic metabolic alkalosis
268. Cushing's syndrome causes hypokalaemia with alkalosis
269. Diabetes melilites diagnosis: fasting > 7.0, random > 11.1 - if asymptomatic need two readings
270. Diabetes mellitus - HbA1c of 48 mmol/mol (6.5%) or greater is diagnostic
271. Diabetic ketoacidosis: the IV insulin infusion should be started at 0.1 unit/kg/hour
272. Diabetic ketoacidosis: isotonic saline should be used initially, even if the patient is severely acidotic
273. Digoxin can cause drug-induced gynaecomastia
274. DPP-4 inhibitors increase levels of incretins such as GLP-1 and
275. Duloxetine may be used in patients with stress incontinence who don't respond to pelvic floor muscle exercises
and decline surgical intervention
276. During Ramadan, one-third of the normal metformin dose should be taken before sunrise and two-thirds
should be taken after sunset
277. Excessive flatulence is an extremely common side effect of acarbose which is often poorly tolerated
278. Exenatide = Glucagon-like peptide-1 (GLP-1) mimetic
279. Exenatide causes vomiting
280. Familial hypercholesterolaemia is an autosomal dominant condition
281. Fibrates may increase the risk of venous thromboembolism
282. Fibrates work through activating PPAR alpha receptors resulting in an increase in LPL activity reducing
triglyceride levels
283. First line treatment in diabetic neuropathy is with amitriptyline, duloxetine, gabapentin or pregabalin
284. Gitelman's syndrome is due to a reabsorptive defect of the NaCl symporter in the DCT
285. Gitelman's syndrome: normotension, hypokalaemia + hypocalciuria
286. Gliflozins - SGLT2 inhibitors
287. Gliptins (DPP-4 inhibitors) reduce the peripheral breakdown of incretins such as GLP-1
288. Glitazones are agonists of PPAR-gamma receptors, reducing peripheral insulin resistance
289. Glucagon is the first hormone secreted in response to hypoglycaemia
290. Glucocorticoid treatment can induce neutrophilia
291. GnRH agonists (e.g. goserelin) used in the management of prostate cancer may result in gynaecomastia
292. Graves' disease is the most common cause of thyrotoxicosis
293. Graves' disease may present first or become worse during the post-natal period
294. Hashimoto's thyroiditis = hypothyroidism + goitre + anti-TPO
295. Hashimoto's thyroiditis is associated with the development of MALT lymphoma
296. Hashimoto's thyroiditis is associated with thyroid lymphoma
297. HRT: adding a progestogen increases the risk of breast cancer
298. HRT: unopposed oestrogen increases risk of endometrial cancer
299. Hungry bone syndrome is the result of a sudden drop in previously high parathyroid hormone levels
180
M Y Elamin
MBBS, DTM&H, MCTM, MRCPI 1& 2
300. Hypercholesterolaemia rather than hypertriglyceridaemia: nephrotic syndrome, cholestasis, hypothyroidism
301. Hypertonic saline is usually indicated in patients with acute, severe, symptomatic hyponatraemia (< 120
mmol/L)
302. Hypoglycaemia in patients with alcoholic liver disease does not respond to glucagon
303. Hypoglycaemia treatment - if the patient is conscious and able to swallow the first-line treatment is a fast-
acting carbohydrate by mouth i.e.. glucose liquids, tablets or gels
304. Hypokalaemia, nephrocalcinosis - type 1 renal tubular acidosis
305. If starting an SGLT-2 as initial therapy for T2DM then ensure metformin is titrated up first
306. In a very athletic woman, hypothalamic hypogonadism is a common cause of secondary amenorrhoea
307. In Cushing's disease, cortisol is not suppressed by low-dose dexamethasone but is suppressed by high-dose
dexamethasone
308. In gestational diabetes, if blood glucose targets are not met with diet/metformin then insulin should be added
309. In patients with T2DM, SGLT-2 should be introduced at any point they develop CVD, a high risk of CVD or
chronic heart failure
310. In the investigation of acromegaly, if a patient is shown to have raised IGF-1 levels, an oral glucose tolerance
test (OGTT) with serial GH measurements is suggested to confirm the diagnosis
311. In the primary prevention of CVD using statins aim for a reduction in non-HDL cholesterol of > 40%
312. In toxic multinodular goitre, nuclear scintigraphy reveals patchy uptake
313. In type 1 diabetics, a general HbA1c target of 48 mmol/mol (6.5%) should be used
314. Infertility in PCOS - clomifene is typically used first-line
315. Insulinoma is diagnosed with supervised prolonged fasting
316. Intravenous calcium gluconate is used for the acute management of hypocalcaemia
317. Iron / calcium carbonate tablets can reduce the absorption of levothyroxine - should be given 4 hours apart
318. IV fluid therapy is the first-line management in patients with hypercalcaemia
319. Kallman's syndrome - LH & FSH low-normal and testosterone is low
320. Klinefelter's - 47, XXY
321. Klinefelter's syndrome - elevated gonadotrophin levels
322. Klinefelter's? - do a karyotype
323. LH surge causes ovulation
324. Liddle's syndrome: hypokalaemia + hypertension
325. Medullary thyroid cancer, phaeochromocytoma, marfanoid body habitus - multiple endocrine neoplasia type
IIb
326. Meglitinides - bind to an ATP-dependent K+(KATP) channel on the cell membrane of pancreatic beta cells
327. Metformin is the first line therapy of choice for diabetes in pregnancy
328. Mirabegron is a beta-3 agonist
329. MODY is inherited in an autosomal dominant fashion so a family history is often present
181
M Y Elamin
MBBS, DTM&H, MCTM, MRCPI 1& 2
330. Myxoedemic coma is treated with thyroxine and hydrocortisone
331. Non-functioning pituitary tumours present with hypopituitarism and pressure effects
332. Orlistat works by inhibiting gastric and pancreatic lipase to reduce the digestion of fat
333. Oxybutynin should not be used in frail older women with urinary incontinence due to the risk of impairment
of daily functioning, confusion and acute delirium
334. Patients on insulin may now hold a HGV licence if they meet strict DVLA criteria
335. Patients on long-term steroids should have their doses doubled during intercurrent illness
336. Patients with acromegaly have an increased risk of colorectal carcinoma
337. Patients with established CVD should take atorvastatin 80mg on
338. Patients with type I diabetes and a BMI > 25 should be considered for metformin in addition to insulin
339. Peptic ulceration, galactorrhoea, hypercalcaemia - multiple endocrine neoplasia type I
340. Phaeochromocytoma: do 24 hr urinary metanephrines, not catecholamines
341. Pioglitazone - contraindicated by: heart failure
342. Pioglitazone may cause fluid retention
343. Primary hyperaldosteronism can present with hypertension, hypernatraemia, and hypokalemia
344. Prolactin - under continuous inhibition
345. Raised total T3 and T4 but normal fT3 and fT4 suggest high concentrations of thyroid binding globulin, which
can be seen during pregnancy
346. Refeeding syndrome causes hypophosphataemia
347. Riedel's thyroiditis is associated with retroperitoneal fibrosis
348. Serum IGF-1 levels are now the first-line test for acromegaly
349. SGLT-2 inhibitors reversibly inhibit sodium-glucose co-transporter 2 (SGLT-2) in the renal proximal
convoluted tubule
350. SGLT-2 inhibitors should be used in addition to metformin as initial therapy for T2DM if CVD, high-risk of
CVD or chronic heart failure
351. SGLT-2 inhibitors work by increasing urinary excretion of glucose
a. (Important as it is the cause of main side effects - increased urine output, weight loss, UTI)
352. SIADH - drug causes: carbamazepine, sulfonylureas, SSRIs, tricyclics
353. SIADH is treated with fluid restriction
354. Small cell lung cancer accounts 50-75% of case of ectopic ACTH
355. Subacute thyroiditis causes hyper- then hypothyroidism
356. Sulfonylureas - bind to an ATP-dependent K+(KATP) channel on the cell membrane of pancreatic beta cells
357. Sulfonylureas often cause weight gain
358. Sulfonyureas increase stimulation of insulin secretion by pancreatic B-cells and decrease hepatic clearance of
insulin

182
M Y Elamin
MBBS, DTM&H, MCTM, MRCPI 1& 2
359. Systemic glucocorticoids can cause drug-induced acne. This is characterised as monomorphic papular rash
without comedones or cysts. This does not respond to acne treatment but improves on drug discontinuation
360. T2DM on metformin, if HbA1c has risen to 58 mmol/mol then one of the following should be offered depending
on the individual clinical scenario:
a. DPP-4 inhibitor
b. pioglitazone
c. sulfonylurea
d. SGLT-2 inhibitor (if NICE criteria met)
361. The definitive management of primary hyperparathyroidism is total parathyroidectomy
362. The low-dose (overnight) dexamethasone suppression test is the best test to diagnosis Cushing's syndrome
363. The most common ECG change in hypocalcaemia is prolongation of the QTc interval
364. The presence of an elevated prolactin level along with secondary hypothyroidism and hypogonadism is
indicative of stalk compression is consistent with a non-functioning pituitary adenoma
365. The PTH level in primary hyperparathyroidism may be normal
366. The short synacthen test is the best test to diagnose Addison's disease
367. Thiazides cause hypercalcaemia
368. Thinning of pubic and axillary hair is seen in females with Addison's disease due to reduced production of
testosterones from the adrenal gland
369. Thyrotoxic storm is treated with beta blockers, propylthiouracil and hydrocortisone
370. Thyrotoxicosis with tender goitre = subacute (De Quervain's) thyroiditis
371. TSH antibodies are found in 90% of patients with Graves' disease and can help distinguish from other forms of
hyperthyroidism
372. Type 1 (distal) renal tubular acidosis may be caused by Sjogren's syndrome
373. Type 1 renal tubular acidosis (distal) complication - renal stones
374. Urinary incontinence - first-line treatment:
a. urge incontinence: bladder retraining
b. stress incontinence: pelvic floor muscle training
375. Uterine fibroids may rarely be associated with secondary polycythaemia due to autonomous production of
erythropoietin
376. Water deprivation test: cranial DI
a. urine osmolality after fluid deprivation: low
b. urine osmolality after desmopressin: high
377. Water deprivation test: nephrogenic DI
a. urine osmolality after fluid deprivation: low
b. urine osmolality after desmopressin: low
378. Water deprivation test: primary polydipsia
183
M Y Elamin
MBBS, DTM&H, MCTM, MRCPI 1& 2
a. urine osmolality after fluid deprivation: high
b. urine osmolality after desmopressin: high
379. Women who smoke are at a two-fold increased risk of developing cervical cancer than women who do not
380. Women with hypothyroidism may need to increase their thyroid hormone replacement dose by up to 50% as
early as 4-6 weeks of pregnancy
381. 9 am cortisol between 100-500nmol/l is inconclusive and requires further investigation with a short synacthen test
382. 11-beta hydroxylase deficiency associated with hypertension
383. Thyrotoxicosis is associated with high alkaline phosphatase secondary to increased bone turnover. High bone
turnover and osteoporosis may be associated with thyrotoxicosis.
384. Diabetic patients with acute stroke resulting in dysphagia should have their oral medication held, sugars
monitored and restarted when their swallow improves.
385. The subcutaneous insulin options are inappropriate because they may increase risk of hypoglycaemia and over
the longer term may drive a worsening in blood pressure and weight gain. Study evidence suggests intensive
insulinisation increases the risk of hypoglycaemia without improving stroke outcomes. Additional evidence from
diabetes trials also suggests that hypoglycaemia increases the risk of cardiac arrhythmia.
386. Episodes of hypokalaemic periodic paralysis occur in 10% of young Latin American or Asian men with
thyrotoxicosis (of whatever aetiology). Acute attacks respond to potassium administration. The periodic paralysis
resolves when the thyrotoxicosis is treated.
387. Temporary hypocalcaemia may affect up to 20% of patients after thyroidectomy as a result of trauma to the
parathyroid glands or their inadvertent excision in theatre. Acute hypocalcaemia usually presents 24-48 hours
postoperatively and may present with perioral tingling, twitching or tetany. This may progress to seizures and
ventricular arrhythmias. The ECG may show a prolonged QT interval (QTc is prolonged if >440 ms in men or
>460 ms in women.) is associated with increased risk of torsades de pointes and treatment for serum calcium levels
<2 mmol/L is urgent intravenous calcium (10 ml of 10% calcium gluconate) followed by an infusion if necessary.
388. Psuedohypoparathyroidism is associated with slipped epiphyseal plate in childhood. It is an autosomal dominant
condition and is due to defects in the gene (GNAS1) encoding the alpha subunit of the stimulatory G protein
(Gsa) contributing to at least three different forms of the disease: the severity of the condition may vary with
generations
389. Many drugs have been associated with gynecomastia, including phytoestrogens, oestrogens and drugs with
oestrogen-like properties, inhibitors of testosterone synthesis or action, and other agents with unknown mechanisms.
390. Tuberous sclerosis presents with flesh coloured papules (angiofibromas) on the face and fibromas in the central
nervous system
391. Klinefelter's is characterised by tall stature, small testes, azoospermia and gynaecomastia in a male. Typical
karyotype is 47XXY, though mosaics occur with 46XY/47XXY karyotype.
392. Klinefelters does not have a genetic pattern of inheritance
393. Psuedohypoparathyroidism is associated with slipped epiphyseal plate in childhood.
184
M Y Elamin
MBBS, DTM&H, MCTM, MRCPI 1& 2
394. CF is caused by mutations in a single large gene on chromosome 7 that encodes the cystic fibrosis transmembrane
conductance regulator (CFTR) protein
395. Huntington's disease is an autosomally inherited condition due to an expanded CAG trinucleotide repeat on the
short arm of chromosome 4. It is characterised by progressive dementia and worsening choreiform movements.
Symptoms typically appear between 30 and 50 years of age.
396. In X-linked dominant diseases, the affected gene follows the X sex chromosome. Fathers pass the gene on to all
their daughters but none of their sons. Mothers pass the gene on to 50% of their sons and daughters.
397. In X linked recessive diseases, the trait is expressed in hemizygous males and in homozygous females. A
heterozygous (carrier) female may not have the phenotype, but passes the gene to 50% of her children.
398. ADPKD cause mitral valve prolapse and cerebral aneurysm not aortic stenosis.
399. A normal male has 46XY chromosomes
400. 11-beta hydroxylase deficiency associated with hypertension
401. 5-alpha reductase deficiency - inability to convert testosterone to dihydrotestosterone
402. "Dyshormonogenesis" means "bad hormone creation". The most fitting answer, logically, is a defect in
iodine organification.
403. 1 mg prednisolone is equivalent to 4 mg hydrocortisone.
404. 1,25 dihydroxy vitamin D increases absorption of calcium from the intestine.
405. 24 hour urinary free cortisol is the initial investigation of choice where there is high clinical suspicion of
Cushing's syndrome.
406. 40% of acromegaly patients will have insulin resistance and impaired glucose tolerance, whilst 20% will
have diabetes melliitus.
407. 72 hour fast is the test of choice for diagnosis of insulinoma
408. 72 hour fast requires patient to reach glucose levels below 2 mmol/l with paired c-peptide and insulin
levels to be valid test.
409. 80% of malignant hypercalcaemia (including lung cancer) is due to PTH related peptide.
410. 90% of affected individuals with porphobilinogen (PBG) deaminase deficiency remain asymptomatic
throughout their lives.
411. A 72 hour supervised fast is the optimal investigation as well as good history taking for patients with
suspected recurrent hypoglycaemia.
412. A chromophobe adenoma refers to no uptake of dye within the tumourous specimen.
413. A family history of osteoporotic fracture is a known risk for osteoporosis. This may be related to
inherited differences in bone structure which put the patient at increased risk.
414. A glucose tolerance test with growth hormone measurement is useful with respect to screening as
growth hormone is usually suppressed by hyperglycaemia to a level below 0.3 mcg/L.

185
M Y Elamin
MBBS, DTM&H, MCTM, MRCPI 1& 2
415. A high or even normal PTH concentration in the presence of hypercalcaemia would support the
diagnosis of hyperparathyroidism.
416. A mildly elevated serum calcium, low phosphate and normal renal function makes primary
hyperparathyroidism the most likely diagnosis.
417. A non-functioning microadenoma of the pituitary should remain under surveillance with reassurance to
the patient.
418. A picture of low calcium with mild elevation in phosphate and normal alkaline phosphatase fits best
with hypoparathyroidism.
419. A raised amylase in the absence of frank pancreatitis is common in patients with diabetic ketoacidosis
(DKA).
420. A random cortisol concentration is not adequate to diagnose hypoadrenalism.
421. A reduction in prolactin levels occurs in around a third of people presenting with idiopathic
hyperprolactinaemia.
422. A significant percentage of growth hormone secreting tumours are thought to have a mutation in the
alpha sub-unit of the stimulatory guanosine triphosphate (GTP) binding protein, leading to persistent
elevation of cyclic adenosine monophosphate (AMP) and hence production of excess growth hormone.
423. A significant percentage of growth hormone secreting tumours are thought to have a mutation in the
alpha sub-unit of the stimulatory guanosine triphosphate (GTP) binding protein, leading to persistent
elevation of cyclic adenosine monophosphate (AMP) and hence production of excess growth hormone.
424. Abdominal MRI would exclude a diagnosis of phaeochromocytoma, which is crucial before considering
thyroidectomy.
425. Abnormal liver function, weight gain, and erectile disfunction point towards chronic alcoholism.
426. Acanthosis nigricans is found in obese patients, diabetes from insulin resistance, familial, some genetic
syndromes, from drug resistance and rarely in some malignancies (eg gastric adenocarcinomas).
427. According to NICE guidance (NG28), the recommended interval for re-checking HbA1c with each
treatment intensification is at 3/6 monthly intervals.
428. Achrondroplasia may be diagnosed radiographically at birth, or becomes obvious within the first year
with disparity between a large skull, normal trunk length, and short limbs.
429. Acromegaly is diagnosed by non-suppression of GH in the glucose tolerance test.
430. Acromegaly is treated with surgery first line.
431. Addisonian crisis should be treated with intravenous hydrocortisone in the first instance.
432. Addison's disease is associated with primary hypothyroidism in the complex of autoimmune
polyendocrine syndrome.
433. Addison's is associated with hyperkalaemic acidosis

186
M Y Elamin
MBBS, DTM&H, MCTM, MRCPI 1& 2
434. Adenomas larger than 1.5 cm frequently have suprasellar extension and may displace the optic chiasm.
435. ADH binds to V2 receptors which are found on the peritubular surface of cells in the distal convoluted
tubule and medullary collecting duct.
436. ADH causes increased Factor VIII production, so can be used in treating haemophilia B.
437. Adhesive capsulitis is a recognised musculoskeletal complication of diabetes.
438. AIS is associated with resistance to androgens. As such, patients have a spectrum of external genitalia
development which ranges from normal female, to female with cliteromegaly, to under-developed male,
(hypospadias). Cryptorchidism is common, as is the development of abdominal hernias. Unfortunately some
patients are still diagnosed relatively late when they present with primary amenorrhoea in their teenage
years.
439. Amiodarone can cause hypothyroidism by inhibiting the peripheral conversion of T4 to T3.
440. An absence of pigmentation in the presence of electrolyte disturbance consistent with adrenal
insufficiency helps point towards pituitary disease.
441. An association of Horner's syndrome and a thyroid nodule would suggest invasion of the sympathetic
chain and would suggest that this thyroid nodule is malignant.
442. An elevated testosterone level in isolation is suggestive of an androgen secreting tumour.
443. An incidental finding of non-functioning macroadenoma may be followed with serial imaging.
444. Anabolic steroid use can induce hypogonadism
445. Anaplastic carcinomas of the thyroid are those which are most likely to cause compression or infiltration
of the trachea.
446. Anaplastic thyroid cancer often has local infiltration and invasion at diagnosis.
447. Anaplastic thyroid carcinoma has worse outcome, followed by medullary (particularly in MEN2B).
448. Androgen insensitivity syndrome will produce an XY genotype with female phenotype.
449. Anorexia nervosa is associated with impaired glucose tolerance.
450. Anorexia nervosa results in hyponatraemia, hypokalaemia, and alkalosis. Albumin levels are classically
normal, although the biological mechanism is not fully understood.
451. Anosmia, failure to enter puberty, and low testosterone are typical of Kallmann's syndrome.
452. Anti-21-hydroxylase antibody is found in 80% of patients with autoimmune adrenalitis.
453. Anticholinergic agents decrease large intestinal motility.
454. Antiphospholipid syndrome predisposes to arterial and venous thrombosis, and can result in
hypoadrenalism due to adrenal infarcts.
455. Antithyroid drugs, usually carbimazole, should be administered immediately in thyroid storm, followed
quickly by betablockers and corticosteroids.
187
M Y Elamin
MBBS, DTM&H, MCTM, MRCPI 1& 2
456. Around 80% of patients treated with radioactive iodine will result in hypothyroidism.
457. Azoospermia may be due to obstruction or inadequate production.
458. Bariatric surgery is the only effective and enduring intervention for obesity in terms of weight loss and
comorbidity resolution.
459. Bartter syndrome is autosomal recessive condition associated with hypokalaemia.
460. Beta blockers can result in false positive results for the aldosterone/plasma renin activity ratio.
461. Bisphosphonates rarely cause jaw osteonecrosis.
462. Bisphosphonates work by inhibiting osteoclast activity, thus increasing mineralisation.
463. Block replace is the initial choice for managing thyrotoxicosis in patients with significant thyroid eye
disease.
464. Both thiazides and beta blockers have been shown to be associated with impaired glucose tolerance and
increased risk of type 2 diabetes howevere Risperidone has a higher risk of doing so.
465. Bulimia is associated with teeth erosion secondary to acidic vomit, and raised alkaline phosphatase
secondary to bone loss.
466. Bulimia nervosa can cause low urea with hyponatraemia and hypokalaemia secondary to vomiting.
467. C cell hyperplasia eventually undergoes malignant transformation, leading to medullary carcinoma of
the thyroid. If patients with MEN-2 are not identified by screening, often at the time of presentation
medullary thyroid carcinoma with metastases to cervical lymph nodes has already occurred.
468. C peptide is released with endogenous insulin so its absence in the setting of a high insulin and low
glucose suggests exogenous administration of insulin.
469. CAH can lead to premature epiphyseal closure and reduced final height.
470. CAH is treated with hydrocortisone with monitoring of 17 OHP, androstenidione and renin activity.
471. Calcitonin screening is strongly recommended in MEN-2 to identify medullary thyroid cancer as early
as possible.
472. Calcitonin-gene related peptide causes vasodilatation
473. Calcium and vitamin D are the most appropriate management for symptoms of osteomalacia.
474. Candida intertrigo is common amongst obese patients with diabetes, and oral intervention is preferred in
those who have significant skin infection.
475. Carbamazepine and gabapentin are first line agents for managing the condition.
476. Carcinoid syndrome can lead to niacin deficiency, and clinical manifestations of pellagra.
477. Carcinoid tumours of the lung rarely cause classical carcinoid syndrome, but can result in ACTH
secretion and Cushing's syndrome.

188
M Y Elamin
MBBS, DTM&H, MCTM, MRCPI 1& 2
478. Carcinoid tumours of the lung rarely cause classical carcinoid syndrome, but can result in GNRH
secretion and acromegaly.
479. Cardiovascular risk calculators apply a risk multiplier of 1.3 for triglycerides above 1.7 mmol/L.
480. Certain drugs are recognised to interfere with absorption of thyroxine, these include rifampicin, calcium
supplements, amiodarone, and ferrous sulphate.
481. Changes in libido are reported in patients with hyperthyroidism which resolves once euthyroid status is
attained.
482. Chronic renal failure cause hyperprolactinemia by inducing an increased level of prolactin, the exact
mechanism of which remains unclear, and a decreased metabolic clearance rate.
483. Chylomicronaemia is due to a circulating inhibitor of lipoprotein lipase (type 1c hyperlipidaemia),
which typically presents with eruptive xanthoma and abdominal colic.
484. Clinical studies have shown that metformin reduces insulin resistance and have demonstrated a fall in
serum androgens, luteinising hormone and weight in PCO
485. Combined oral contraceptive pill can help with hirsuitism in PCOS.
486. Congenital hypothyroidism may be asymptomatic.
487. Conn's syndrome (hyperaldosteronism) is characterised by hypertension, hypokalaemia, high
aldosterone, low renin.
488. Conn's syndrome will have hypertension and hypokaleamic alkalosis
489. Consider premature ovarian failure as a differential in young women presenting with prolonged
amenorrhoea.
490. Corticosteroids are recognised to inhibit osteoblast activity and increase osteoblast apoptosis leading to
reduced bone denisty.
491. Cranial irradiation can result in panhypopituitarism. Diagnosis should be aimed at detecting hormone
deficiencies.
492. Cushing's syndrome can be diagnosed with 24 hour urinary cortisol collection or ONDST.
493. Cushing's syndrome may rarely present with necrosis of the femoral head due to osteoporosis.
494. De Quervain's thyroiditis is associated with low uptake on radio-isotope scanning of the thyroid.
495. Decreased potassium would be consistent with both induced vomiting and abuse of diuretics.
496. Diabetes insipidus is confirmed with water deprivation testing.
497. Differential diagnosis of Charcot joint.
498. Domperidone is a dopamine agonist and may cause galactorrhoea.
499. Dopamine agonists are the treatment of choice in patients with micro/macroprolactinoma.

189
M Y Elamin
MBBS, DTM&H, MCTM, MRCPI 1& 2
500. DPPIV inhibitors reduce natural degradation of incretins, leading to increased levels of GLP-1. This
results in suppression of glucagon.
501. Drop in systolic blood pressure on standing, hyponatraemia, hypokalaemia, and increased urinary
sodium excretion all point towards a diagnosis of relative hypovolaemia in the presence of thiazide use
502. Drug related gynaecomastia is said to account for 10-25% of cases.
503. Drugs causing gynaecomastia include spironolactone, digoxin, and cimetidine.
504. Drugs that can cause gynaecomastia include cimetidine, spironolactone, digoxin, methyldopa,
gonadotrophins, and cyproterone acetate.
505. Duloxetine is recommended by NICE as therapy for painful diabetic neuropathy.
506. Elastase is an enzyme produced by the pancreas. It is secreted into the duodenum and not degraded
during guy transit, so measurement of faecal elastase allows the diagnosis or exclusion of pancreatic
exocrine insufficiency.
507. Elevated prolactin levels is a recognised association of acromegaly.
508. Elevated total T4 is a normal finding at this stage of pregnancy due to increased thyroid binding
globulin. Elevated thyroid hormones with suppression of TSH may be considered a normal variant in the
early stages of pregnancy if there is a history of hyperemesis.
509. Elevated total T4 is a normal finding at this stage of pregnancy due to increased thyroid binding
globulin. Elevated thyroid hormones with suppression of TSH may be considered a normal variant in the
early stages of pregnancy if there is a history of hyperemesis.
510. Elevated triglycerides are associated with disordered energy handling by adipocytes; this leads to
increased insulin requirements.
511. Enteroviruses may play a role in protection from and susceptibility to T1DM
512. Epidemiological analyses suggest that triglycerides above 1.7 are associated with a 30% relative risk
increase with respect to cardiovascular events.
513. Evaluation of hyponatraemia.
514. Exogenous calcitonin inhibits osteoclast activity, and is associated with a 1-2% increase in BMD which
is sustained for up to five years.
515. Factitious thyrotoxicosis should be a diagnosis of exclusion.
516. Faecal elastase is non-invasive and can be performed on a single stool sample. Low levels of faecal
elastase are indicative of pancreatic exocrine deficiency and predictive of the response to supplementation.
517. Failure of menarche by age 16 is a recognised sign of delayed puberty and should prompt targeted
examination and appropriate investigations. Both hypogonadotrophic causes such as low weight/malnutrition
and hypergonadotrophic causes, such as Turner’s syndrome, are recognised.

190
M Y Elamin
MBBS, DTM&H, MCTM, MRCPI 1& 2
518. Failure of menarche by age 16 is a recognised sign of delayed puberty and should prompt targeted
examination and appropriate investigations. Both hypogonadotrophic causes such as low weight/malnutrition
and hypergonadotrophic causes, such as Turner’s syndrome, are recognised.
519. Failure of menarche by age 16 is a recognised sign of delayed puberty and should prompt targeted
examination and appropriate investigations. Both hypogonadotrophic causes such as low weight/malnutrition
and hypergonadotrophic causes, such as Turner’s syndrome, are recognised.
520. Failure to lose weight after intensive lifestyle intervention should raise the possibility of considering
bariatric surgery in certain patients with diabetes.
521. Familial GRA is an autosomal dominant condition where a mutation leads to ACTH responsive
aldosterone production from the zona fasciculata rather than the zona glomerulosa.
522. Fasting gastrin levels >1000 with low pH is highly correlated with a diagnosis of ZE syndrome. Where
levels are less than 1000 and the diagnosis is suspected, then secretin stimulation testing, (where a rise >200
15 minutes after dosing is considered positive), or calcium stimulation testing, (where a rise >395 is
considered positive) may be useful.
523. Fasting gastrin levels >1000 with low pH is highly correlated with a diagnosis of ZE syndrome. Where
levels are less than 1000 and the diagnosis is suspected, then secretin stimulation testing, (where a rise >200
15 minutes after dosing is considered positive), or calcium stimulation testing, (where a rise >395 is
considered positive) may be useful.
524. Features of Graves' disease are exophthalmos, thyroid bruit, and preibial myxodema.
525. Features of hypercortisolism include femoral avascular necrosis, weight gain, hirsutism, hypertension,
and impaired glucose tolerance.
526. Felty's syndrome is a complication of rheumatoid arthritis (RA), defined as the triad of RA, neutropenia,
and splenomegaly.
527. Female patients preenting with elevated testosterone, and absence of acne or secondary sexual hair are
all pointers towards androgen insensitivity syndrome.
528. Fibrates increases HDL-C by 10-15% and reduces triglycerides by 15-20%.
529. FIHP is a rare condition characterised by an autosomal dominant mode of inheritance.
530. Finasteride inhibits the conversion of testosterone to active DHT.
531. Findings in hypercalcaemia.
532. First line treatment of acromegaly is pituitary surgery.
533. Flow volume loop is the best test to assess severity of obstruction for retrosternal goitre.
534. Flow volume loops are the best method to detect tracheal obstruction associated with retrosternal goitre.
535. Fludrocortisone replacement in Addison’s is guided by symptoms/measurement of postural hypotension
and serum electrolytes, evidence of hyponatraemia/hyperkalaemia. It is not usually increased in times of
illness.
191
M Y Elamin
MBBS, DTM&H, MCTM, MRCPI 1& 2
536. Flu-like symptoms and neck tenderness fit best with a diagnosis of sub-acute thyroiditis.
537. For large invasive tumours, gamma knife surgery or traditional adenomectomy are treatments of choice.
Monitoring of ACTH regularly, and follow up MRI pituitary at three to six months post adrenalectomy are
recommended to drive early intervention.
538. For patients with thyroid nodule, fine needle aspiration cytology case series indicate that satisfactory or
diagnostic samples are obtained in over 85% of cases.
539. Fragile X syndrome is the most common form of inherited learning difficulties with characteristic post
puberty phenotype.
540. G alpha subunit mutation is present in 30% of GH secreting pituitary tumours.
541. Gardener's syndrome is associated with intestinal tumours, lipomas and papillary thyroid cancer.
542. GDM should be initially treated with lifestyle modifications unless macrosomia on foetal scan.
543. GH deficiency is diagnosed by a peak GH response of less than 9 mU/L with insulin-induced
hypoglycaemia.
544. GH deficiency may present with lethargy and weight gain.
545. GH replacement therapy may cause gynaecomastia, hypertension, SUFE and idiopathic intracranial
hypertension.
546. GHRG/arginine test should be used when ITT contraindicated.
547. Gliclazide action can be potentiated predominantly by displacement of the drug from plasma proteins to
give more free (unbound) drug and interference with the hepatic metabolism of the drug.
548. GLP-1 is a hormone released by the gut in response to food intake that acts on the pancreatic beta cells
to trigger insulin secretion.
549. Glucagon is designed to increase provision of energy in a time of need. One of these energy sources may
indeed be fat, therefore glucagon stimulates lipolysis.
550. Gold standard test to confirm growth hormone excess is IGF-1 serum levels
551. Graves' thyrotoxicosis is associated with pre-tibial myxoedema, commonly described as orange peel
skin present on both shins.
552. Growth hormone therapy rarely may cause idiopathic intracranial hypertension.
553. Gynaecomastia is a central feature of Klinefelter's syndrome, which occurs as a result of an XXY
karyotype.
554. Gynaecomastia may be due to HCG secretion form a seminoma.
555. Haemachromatosis is associated with IDDM and hypogonadotrophic hypogonadism.

192
M Y Elamin
MBBS, DTM&H, MCTM, MRCPI 1& 2
556. Haemochromatosis is characterised by failure of gonadotrophins leads to testosterone deficiency and
erectile dysfunction, deposition in the endocrine pancreas leads to diabetes mellitus, and chondrocalcinosis
leads to joint pains.
557. Hair loss one to three months after pregnancy is normal and no treatment is required.
558. Hashimoto thyroiditis is the commenest cause of hypothyroidism independent of gender.
559. Hashimoto's disease carries a small increased risk of thyroid lymphoma.
560. Hashimoto's disease is associated with positive anti-TPO (or microsomal) antibodies.
561. HbA1c above 48 mmol/mol or 6.5% with symptoms is diagnostic of diabetes (WHO 2011).
562. HbA1c reflects three months of glucose control. Other methods should therefore be used to estimate
glucose control accurately in patients who have received a large blood transfusion within the past three
months.
563. Heavy metal poisoning causes acquired RTA and hence osteomalacia
564. Heavy proteinuria in the presence of hypertension may indicate renal cause of secondary hypertension.
565. High triglycerides and low high-density lipoprotein (HDL) cholesterol are the commonest lipid
abnormality seen in type 2 diabetes, and both are associated with increased cardiovascular risk.
566. Hirsuitism may be caused by drugs, e.g. minoxidil, phenytoin and cyclosporin.
567. History of headache and amenorrhoea suggest a prolactinoma or non-functioning tumour.
568. HRT increases CVD and stroke (see NICE guidance).
569. HRT should be used to prevent osteoporosis in the symptomatic menopausal woman.
570. Hydrocortisone replacement is urgent, whilst other hormones can be replaced afterwards.
571. Hyperaldosteronism is associated with hypertension, low potassium, low magnesium, alkalosis,
suppressed renin and low/normal sodium.
572. Hypercalcaemia above 3.5 mmol/L is known to be associated with shortening of the QT interval. This in
turn significantly increases the risk of cardiac arrhythmias.
573. Hypercalcaemia with a borderline low phosphate concentration but an inappropriately normal
parathyroid hormone (PTH) concentration suggests hyperparathyroidism
574. Hypercarotenaemia occurs in hypothyroidism due to reduced conversion of carotene to vitamin A.
575. Hypercholesterolaemia with hypertriglyceridaemia is associated with subclinic hypothyroidism.
576. Hyperprolactinaemia can be associated with PCOS but not greatly elevated levels.
577. Hypertriglyceridaemia and raised transaminases are suggestive of increased hepatic fat.
578. Hypoadrenalism is a medical emergency, which often presents with abdominal pain and a history of
weight loss.
193
M Y Elamin
MBBS, DTM&H, MCTM, MRCPI 1& 2
579. Hypogonadism is a common cause for osteoporosis in a young man.
580. Hypogonadism may be due to zinc deficiency.
581. Hypogonadotrophic hypogonadism in the presence of raised prolactin is likely secondary to
microprolactinoma
582. Hypogonadotrophic hypogonadism with normal growth and normal IQ is likely due to Kallman's
syndrome
583. Hypokalaemia in the presence of ACE inhibition and thiazide use is highly suspicious of primary
hyperaldosteronism. The incidence of primary aldosteronism is now thought to be much higher than
previously considered, (up to 4-5% of patients with hypertension). The next investigation of choice is the
aldosterone/renin ratio which will help differentiate between primary hyperaldosteronism and renovascular
disease as possible causes.
584. Hypomagnesaemia can result in QT prolongation, and may be caused by diarrhoea or
bendroflumethiazide use.
585. Hypomagnesaemia may be due to diuretic use.
586. Hyponatraemia with hypokalaemia is likely secondary to diuretic use.
587. Hypothyroidism is the commonest pre-existing endocrine disorders in pregnancy, with an incidence of 9
in 1000 pregnancies.
588. If cortisol is not suppressed by the low or high dose dexamethasone suppression test then either primary
adrenal Cushing's syndrome (low/undetectable ACTH) or ectopic ACTH is the cause (high ACTH). If
cortisol is not suppressed by low dose dexamethasone but is by high dose then Cushing's disease is the likely
cause.
589. If cortisol is not suppressed by the low or high dose dexamethasone suppression test then either primary
adrenal Cushing's syndrome (low/undetectable ACTH) or ectopic ACTH is the cause (high ACTH). If
cortisol is not suppressed by low dose dexamethasone but is by high dose then Cushing's disease is the likely
cause.
590. If TSH is within normal range thyroid replacement is adequate.
591. Impaired perception of colour implies acute progressive neuropathy and as such is a consideration for
urgent ophthalmological referral.
592. In HHS there is sufficient insulin to supress ketogenesis but not hyperglycaemia.
593. In lithium-induced hypothyroidism the lithium should not be stopped. Levothyroxine should be started.
594. In patients with osteomalacia, who are unable to increase their sun exposure, calcium and vitamin D
supplementation is the treatment of choice.
595. In phaeochromocytoma in pregnancy, plan for lower segment caesarean section and later surgery for the
tumour.
596. In poor compliance/absorption one would expect elevated TSH with normal range T4.
194
M Y Elamin
MBBS, DTM&H, MCTM, MRCPI 1& 2
597. Inappropriately elevated TSH in the presence of hyperthyroidism should lead to investigation of
pituitary adenoma.
598. Inferior petrosal sinus sampling is an extremely sensitive, specific and accurate test for differentiating
between Cushing's disease and ectopic ACTH production.
599. Infertility and gynaecomastia are two of the most common features resulting in a diagnosis of
Klinefelter's syndrome.
600. Inhibition of 11 beta hydroxysteroid dehydrogenase produces psuedohyperaldosteronism.
601. Insulin acts via a similar mechanism to cell surface receptors.
602. Insulinoma can be diagnosed with 72hr fast.
603. Insulinoma presents with sweating and weight gain commonly.
604. Intralesional corticosteroids are the initial management of choice in patients with limited alopecia.
605. Iodine deficiency is relatively common in parts of central Africa, where the diet is poor in iodine and
access to sea fish is relatively difficult.
606. Iodine deficiency occurs more commonly in central Africa. It may present in a euthyroid state, or with
severe deficiency, symptoms of hypothyroidism.
607. Iodine driven inhibition is the only option where amiodarone related events leading to hypothyroidism
are described
608. Irregular heavy periods, obesity, male pattern hisutism in women are some signs that point towards
PCOS.
609. Islet B cells produces amylin.
610. It is important to know the relative potencies of the glucocorticoids.
611. IV hydrocortisone is first line treatment in Addisonian crisis.
612. Kallmann syndrome has hypogonadotrophic hypogonadism, pubertal failure, anosmia, and usually
infertility.
613. Kallmann syndrome is due to KAL-1 or KAL-2 gene abnormalities.
614. Kallmann's syndrome is diagnosed by combination of anosmia and hypogonadotrophic hypogonadism.
615. Kallman's syndrome results in hypogonadotrophic hypogonadism with a low testosterone and a relative
low follicle-stimulating hormone (FSH) and luteinising hormone (LH).
616. Ketaconazole may be used in Cushing's disease for patients unfit for surgery.
617. Lack of sense of smell, which suggests failure of development of the olfactory bulb, suggests Kallman
syndrome
618. Leptin causes satiety via its acton on the hypothalamus.

195
M Y Elamin
MBBS, DTM&H, MCTM, MRCPI 1& 2
619. Levels of prolactin between around 1000 and 3000 mU/l are consistent with a microprolactinoma.
Levels lower than 1000 would be more consistent with exposure to drugs known to increase prolactin, or
greater than 3000 would raise suspicion of a macroprolactinoma.
620. LH/FSH is useful as an initial screening test for Cushing's syndrome.
621. Liddle's syndrome is typically associated with hypokalaemic hypertension and low renin and
aldosterone concentrations - the so-called pseudo-hyperaldosteronism.
622. Lithium can cause DI and hypercalcaemia.
623. Lithium can cause diabetes insipidus.
624. Lithium can cause nephrogenic DI.
625. Lithium is a recognised cause of nephrogenic DI.
626. Low HDL cholesterol is recognised to be associated with low levels of sex steroids and is a feature of
Klinefelter's syndrome.
627. Low magnesium (below 0.7 mmol/L) can be seen in protein malnutrition states which include anorexia
and eating disorders.
628. Low plasma bicarbonate is highly suggestive of acidosis.
629. Low serum sodium and osmolality with raised urinary osmolality and urinary sodium above 20 mmol/L
is diagnostic for SIADH.
630. Male sex has poor prognosis for thyroid cancer.
631. Markedly elevated alkaline phosphatase in the presence of normal liver enzymes, normal calcium and
normal ESR, and his history of lumbar spine and right hip pain points towards Paget's disease.
632. Medical management of choice for diarrhoea in VIPoma patient is with somatostatin analogue.
633. MEN-1 comprises of parathyroid dysfunciton, gastroenteropancreatic tract tumours, and anterior
pituitary dysfunction.
634. MEN-1 is associated with parathyroid hyperplasia and consequent hypercalcaemia in 80% of patients,
with pancreatic endocrine tumours occurring in around 70% of patients with MEN-1.
635. MEN1 is pancreatic NET, pituitary and parathyroid pathology.
636. MEN-2b is associated with the presence of mucosal neuromas which can occur anywhere in the GI tract,
leading to poor suckling if they occur in the mouth, or intermittent obstruction, particularly if they occur in
the small bowel.
637. Metabolic alkalosis is commonly seen in Cushing's.
638. Metformin and clomiphene can be used to increase fertility in PCOS.
639. Metformin can be used in gestational diabetes if cBG >7.8

196
M Y Elamin
MBBS, DTM&H, MCTM, MRCPI 1& 2
640. Metformin is an insulin sensitiser and although its actions are not fully understood its main role appears
to be through inhibition of hepatic gluconeogenesis.
641. Metformin is contraindicated immediately following MI due to tissue hypoxia which is a risk factor for
the development of lactic acidosis.
642. Metformin is used first line after lifestyle modification for type 2 diabetics with HbA1c >6.5%.
643. Methyldopa should be avoided in patients with deranged LFT.
644. Metoclopramide can cause hyperprolactinaemia.
645. Metoclopramide results in hyperprolactinaemia by blocking dopamine and therefore stimulating
prolactin release.
646. "Metyrapone inhibits 11-beta hydroxylase and as such inhibits cortisol production. It has a relatively
rapid onset of action and as such may be of value preoperatively in improving blood pressure and glycaemic
control without associated weight gain of other options such as insulin.
647. "
648. Metyrapone inhibits 11-beta hydroxylase and as such inhibits cortisol production. It has a relatively
rapid onset of action and as such may be of value preoperatively in improving blood pressure and glycaemic
control without associated weight gain of other options such as insulin.
649. "Metyrapone inhibits 11-beta hydroxylase and as such inhibits cortisol production. It has a relatively
rapid onset of action and as such may be of value preoperatively in improving blood pressure and glycaemic
control without associated weight gain of other options such as insulin.
650. "
651. Minimally elevated prolactin may be due to stalk effect from a pituitary adenoma.
652. Monogenic diabetes/MODY is associated with gene mutations, commonly glucokinase and HNF1a and
HNF4a.
653. Most thyroxine overdoses are managed symptomatically with propranolol.
654. MRI abdomen is the investigation that is likely to confirm the possibility of an adrenal adenoma.
655. Mutations in the alpha subunit lead to elevated levels of PTH, but a loss of function, resulting in
hypocalcaemia seen in pseudohypoparathyroidism.
656. Mutations in the gonadotrophin receptor are less common than those seen causing hypothyroidism.
657. NAFLD may be associated with deranged liver function, central obesity, hypertension, dyslipidaemia
and insulin resistance
658. NICE defines osteopenia as T score -1 to -2.5 in post-menopausal women.
659. NICE guidance on Urinary incontinence (CG171) recommends bladder training as the initial
intervention of choice for urge incontinence.

197
M Y Elamin
MBBS, DTM&H, MCTM, MRCPI 1& 2
660. NICE guidance recommends weight loss for PCOS as the initial step, even in women who are trying for
a pregnancy.
661. NICE guidelines advocate use of ACEi or ARB in diabetics with hypertension.
662. NICE guidelines now recommend amitriptyline, duloxetine, gabapentin or pregabalin for initial
treatment of neuropathic pain.
663. Nicotinic acid may increase blood glucose levels, although this effect is minimal for most diabetics.
664. Non-alcoholic steatohepatitis (NASH) is associated with insulin resistance, hyperlipidaemia and chronic
moderately elevated liver enzymes.
665. Normal visual acuity is seen in proliferative retinopathy
666. Octreotide is a somatostatin analogue that directly inhibits growth hormone secretion.
667. Octreotide is a somatostatin analogue which is effective in reducing symptoms of flushing, diarrhoea,
hypotension, and abdominal pain which are associated with carcinoid syndrome
668. Ocular paresis occurs in up to 80% of non-functioning adenoma patients, with third nerve palsy the
commonest finding.
669. On average, data referred to in the SIGN guidelines for the Diagnosis and Management of Chronic
Kidney Disease on change in GFR/year suggest in patients with proteinuria, the deterioration if BP is well
controlled is 4 ml/min/year.
670. Once pregnancy is established, bromocriptine is not necessarily required, and so most physicians
recommend stopping it for the duration.
671. One of the indications for parathyroidectomy in primary hyperparathyroidism is the presence of a T-
score less than -2.5 at any site.
672. Oral glucose tolerance test is the test of choice to diagnose acromegaly.
673. Osteomalacia is related to low levels of vitamin D and occurs more commonly in patients of South
Asian origin, particularly those who have a cultural tendency to spend more time inside.
674. Osteoporosis is a feature of Cushing's syndrome.
675. Ovarian failure is most likely to be associated with elevated FSH, but levels can vary and should be
repeated over four to eight weeks.
676. Over-perception of body image is typical of anorexia nervosa
677. Painful thyroid with no constitutional signs and euthyroidism is likely heamorrhage into thyroid cyst.
678. Papillary thyroid cancer is the commonest thyroid epithelial tumour, and may present with a discrete
thyroid swelling which is cold on isotope scanning.
679. Parathyroid adenoma is the commonest cause for primary hyperparathyroidism.

198
M Y Elamin
MBBS, DTM&H, MCTM, MRCPI 1& 2
680. Parathyroid hormone increases calcium directly by binding osteoblasts and via enhanced reabsorption in
the distal tubules of the kidney.
681. Parathyroid hormone reduces renal reabsorption of phosphate.
682. Parathyroidectomy should be offered to patients with primary hyperparathyroidism who have significant
hypercalcaemia, hypercalcuria, impaired renal function, osteoporosis, or who are aged less than 50 years.
683. Parvovirus B19 is known to be associated with aplastic crises in sickle cell anaemia which can
precipitate severe anaemia and subsequent cardiac failure.
684. Patient with high risk given his history of type 2 diabetes mellitus, and as such should be loaded with
both aspirin and clopidogrel
685. Patients presenting with insulinoma should lead you to suspect multiple endocrine neoplasia type 1
686. Patients with HONK are often sensitive to insulin replacement and doses can be much lower than those
required for DKA.
687. Patients with Hypoglycaemic episodes with poor HbA1c shoulde be transitioned to meal time boluses
with basal insulin at night.
688. PCOS is a result of increased pituitary LH secretion which leads to excess androgen production by the
ovary.
689. PCOS is associated with raised LH:FSH and androstenedione level
690. Pegvisomant is a genetically modified analogue of human growth hormone and is a highly selective
growth hormone receptor antagonist. It has been shown to normalise insulin-like growth factor-I (IGF-I)
levels in 90-100% of patients with a history of acromegaly. Growth hormone levels increase during
treatment and no decrease in tumour size is seen, as such the major use of pegvisomant is in patients who
have an inadequate response to surgery or radiotherapy.
691. Pelvic ultrasound scan is the investigation of choice for PCOS, and can be supported with serum
testosterone and LH:FSH ratio.
692. Pendred's syndrome is sensorineural deafness and goitre
693. Pernicious anaemia ias associated with other autoimmune disorders such as coelias disease or addison
694. Phaeochromocytoma is associated with hypertension, tachycardia and headache.
695. Phenoxybenzamine should be intiated first to treat phaeochromocytoma.
696. Physiologically, thyroid hormones enhance insulin sensitivity as well as increasing gluconeogenesis.
697. Pioglitazone can result in fluid retention of unknown aetiology which may cause a mild dilutional
anaemia (haemoglobin typically falls by 10 to 20 g/L) and ankle oedema. It is contraindicated in congestive
heart failure
698. Pioglitazone should not be used in patients with either a history or a high risk of developing of bladder
cancer.

199
M Y Elamin
MBBS, DTM&H, MCTM, MRCPI 1& 2
699. Pituitary apoplexy is due to haemorrhage/infarct into pituitary gland with associated endocrine and
neurological symptoms.
700. Pituitary disease would not be expected to be associated with virilisation or abnormal LFTs.
701. PPAR gamma is activated by pioglitazone not antagonised
702. Pregnancy causes elevated cortisol levels.
703. Presenting features of insulinoma are often non-specific, and include weakness, memory loss, sweating,
hunger, and weight gain.
704. Pre-tibial myxoedema may be seen in patients with Grave’s disease, and is thought to arise because of
glycosaminoglycans deposition by fibroblasts.
705. Primary hyperparathyroidism is associated with gastrinoma in MEN1
706. Primary hyperparathyroidism is common, and typically presents with symptoms and signs of
hypercalcaemia.
707. Primary management of acromegaly should be pituitary surgery.
708. Primary polydipsia is associated with low plasma osmolality
709. Prior to wound debridement when high risk injury for tetanus is suspected, it is crucial to give human
tetanus immunoglobulin.
710. Prolactin above 2000 mU/L is suggestive of prolatinoma.
711. Prolactin levels less than 1000 fit best with drug related hyperprolactinaemia.
712. Prolonged fasting is used to investigate suspected insulinoma.
713. Propylthiouracil may be used in pregnancy.
714. Proximal myopathy, easy bruising, and thin skin are clinical features suggestive of Cushing's syndrome
715. Psuedohypoparathyroidism is due to PTH receptor abnormality producing resistance.
716. Pulmonary emboli result in hypoxia, and therefore cyanosis can be seen.
717. Radioactive iodine treatment is associated with the induction of hypothyroidism in the majority of
subjects by three months (70%) with 10% failing at the first dose at about 18 months.
718. Raised creatinine, borderline low calcium, and raised phosphate are consistent with secondary
hyperparathyroidism.
719. Raised triglycerides are indicative of disordered free fatty acid handling, which is associated with
increased central fat. Consider metabolic syndrome.
720. Raloxifen is a selective oestrogen receptor modulator used in treatment of osteoporosis
721. Raloxifen is a SERM (selective oestrogen receptor modulator)
722. Reduced IGF1 is seen with starvation.
200
M Y Elamin
MBBS, DTM&H, MCTM, MRCPI 1& 2
723. Renal artery stenosis would result in elevated renin and aldosterone levels.
724. Reported complications vary according to case series, but transient hypoparathyroidism is said to occur
in around 8% of cases, with permanent hypoparathyroidism seen in 1-2% of patients undergoing
thyroidectomy. Thankfully, surgical juniors are well aware of the signs of hypocalcaemia and its treatment.
725. Reported complications vary according to case series, but transient hypoparathyroidism is said to occur
in around 8% of cases, with permanent hypoparathyroidism seen in 1-2% of patients undergoing
thyroidectomy. Thankfully, surgical juniors are well aware of the signs of hypocalcaemia and its treatment.
726. Rifampicin may be expected to reduce the efficacy of the oral contraceptive through liver enzyme
induction.
727. SCLC is a cause of clinical SIADH in up to 10% of patients with the condition.
728. Secondary thyroid failure would be a differential, but measurement of reverse T3 would help
differentiate between this and sick euthyroid syndrome.
729. Sheehan's syndrome is postpartum infarction of the pituitary, and growth hormone deficiency (among
multiple other hormone deficiencies) is expected.
730. SIADH is associated with acute intermittent porphyria.
731. Significant hypothyroidism can result in raised levels of TRH (thyrotropin-releasing factor), which can
act as prolactin-releasing factor and bind to the receptors resulting in the release of prolactin and
hyperprolactinaemia.
732. Significant problems with sickness may be driven by high levels of beta-human chorionic gonadotropin
(HCG) which has some activity on thyroid stimulation.
733. Sildenafil is contraindicated if the patient is taking nitrates and this includes nitrate derivatives such as
nicorandil
734. Simple bisphosphonates inhibit bone resorption through their actions on osteoclasts.
735. Sitagliptin and exenatide are not recommended in patients with this degree of renal impairment
736. Sitagliptin works by inhibiting the DPP-4 enzyme that destroys GLP and GIP hormones, allowing both
to function more effectively.
737. Small cell lung cancer typically produces ectopic ACTH.
738. Somatuline, like naturally occurring somatostatin, results in a marked reduction in mealtime related
increases in superior mesenteric artery and portal blood flow.
739. Spironolactone is a non-selective aldosterone receptor antagonist. As such it blocks both the effects of
aldosterone and testosterone, and is known to cause gynaecomastia.
740. SSRIs are a cause of SIADH.
741. Stalk compression by a non-secretory pituitary adenoma can result in hyperprolactinaemia due to
impaired dopamine transfer.

201
M Y Elamin
MBBS, DTM&H, MCTM, MRCPI 1& 2
742. Statins result in only a modest reduction in triglycerides versus their impact on LDL cholesterol.
743. Subclinical hyperthyroidism may be due to toxic solitary nodule.
744. Supported diet and physical activity modification is first-line treatment for obesity, which should be
assessed by waist circumference and BMI.
745. Suppression of T3 may be observed in patients with anorexia, although T4 and TSH may be normal.
746. Symptoms of hypocalcaemia include paraesthesia, tetany, muscle cramp and carpropedal spasm.
747. Testosterone is a steroid hormone that acts by binding intracellular receptors
748. Tetracyclines are known to induce nephrogenic diabetes insipidus so are an appropriate addition to
therapy for SIADH when fluid restriction alone is inadequate.
749. The 8 mg overnight dexamethasone suppression test and 48 hour high-dose dexamethasone test are
useful when baseline ACTH levels are equivocal. They can be very useful in determining whether a patient
has pituitary or ectopic ACTH production. Greater than 90% reduction in basal urinary free cortisol levels
supports the diagnosis of a pituitary adenoma; ectopic ACTH causes lesser degrees of suppression.
750. The 8 mg overnight dexamethasone suppression test and 48 hour high-dose dexamethasone test are
useful when baseline ACTH levels are equivocal. They can be very useful in determining whether a patient
has pituitary or ectopic ACTH production. Greater than 90% reduction in basal urinary free cortisol levels
supports the diagnosis of a pituitary adenoma; ectopic ACTH causes lesser degrees of suppression.
751. The approximate equivalent glucocorticoid action of prednisolone to hydrocortisone is 4:1.
752. The average daily energy consumption of a male is 2500 kcal and 2000 kcal for a female.
753. The best way to differentiate between insulinoma and exogenous insulin administration is glucose,
insulin, and C peptide, as with self administration of insulin, C peptide is low, but in contrast with
insulinoma it would be elevated.
754. The classic abnormalities seen on bloods in Bartter's syndrome are hypokalaemic alkalosis and elevated
renin and aldosterone levels.
755. The clinical picture here, whilst typical of polycystic ovarian syndrome, is confirmed as non-classical
congenital adrenal hyperplasia by the presence of raised levels of 17-OH progesterone. As such the treatment
likely to be most effective is an anti-androgen such as flutamide.
756. The commonest form of amiodarone induced hyperthyroidism is caused by thyroid cell destruction, and
is best treated with prednisolone.
757. The diagnosis of McCune-Albright is established on clinical grounds, with the presence of precocious
puberty, bony fibromas leading to possible pathological fractures, thyrotoxicosis, and café au lait spots all
supporting this as the underlying cause.
758. The DIGAMI 2 study did not support the use of acutely introduced long-term insulin treatment in type 2
diabetic patients following an MI, but did identify glucose level as a strong independent predictor of long-
term mortality.

202
M Y Elamin
MBBS, DTM&H, MCTM, MRCPI 1& 2
759. The DIGAMI study has demonstrated that there is a survival advantage in initially treating such patients
with elevated glucose concentrations with sliding scale insulin for 24 hours post-infarct, even if they are not
known to be diabetic
760. The equivalent ratio of prednisolone to hydrocortisone is 1:4
761. The GLP-1 mimetics are associated with weight loss and are a good choice in patients who are
significantly overweight.
762. The incretin response to an oral glucose challenge is lost in patients who develop type 2 diabetes. This
means that they lose the response generated by incretin hormones such as GLP-1 and GIP which enhances
insulin release and leads to satiety. Those patients who carry TCF7L2 mutations have a blunted incretin
response.
763. The incretin response to an oral glucose challenge is lost in patients who develop type 2 diabetes. This
means that they lose the response generated by incretin hormones such as GLP-1 and GIP which enhances
insulin release and leads to satiety. Those patients who carry TCF7L2 mutations have a blunted incretin
response.
764. The inferior thyroid artery runs closest to the recurrent laryngeal nerve.
765. The investigation most likely to contribute to making a diagnosis of adrenal insufficiency is a short
Synacthen test.
766. The lower limit of detectability for cortisol is 28 nmol/L. As such, a result of 30 after low dose
dexamethasone suppression test is consistent with a euadrenal state
767. The most appropriate therapy advocated by the National Osteoporosis Society for the prevention of
steroid-induced osteoporosis would be bisphosphonate therapy such as Didronel or alendronate.
768. The most appropriate treatment of de Quervain's thyroiditis is symptomatic control.
769. The most effective treatment for Charcot's ankle is a period of immobilisation in a specially made cast.
770. The NICE guidelines on criteria for Orlistat previously stated it should be used in patients who have
demonstrated dietary compliance with at least a 2.5 kg weight reduction prior to initiating orlistat.
771. The non-classical "mild" form of congenital adrenal hyperplasia can present in later childhood or
adolescence. 17-OH progesterone is elevated because of the enzyme deficiency.
772. The objective of treatment is to elevate calcium to within the normal range and so suppress excess PTH
levels. This is likely significantly to relieve the patient’s aches and pains and lethargy.
773. the patient has poor glycaemic control, but otherwise has no features that preclude him from driving
such as retinopathy, neuropathy or hypoglycaemic episodes
774. The presentation is typically that associated with sub-acute or De Quervain’s thyroiditis, where flu-like
illness is followed by transient hyperthyroidism, then hypothyroidism, then recovery. The gland is diffusely
tender, although pain responds to non-steroidal anti-inflammatory drugs. Anti-thyroid drugs have no value in
the management of the condition. It is associated with HLA-B35 and it is thought that a viral antigen binds to
HLA-B35 molecules on macrophages.
203
M Y Elamin
MBBS, DTM&H, MCTM, MRCPI 1& 2
775. The prevalence of hypokalaemic periodic paralysis is much higher in patients with thyrotoxicosis of
Chinese origin versus Caucasian.
776. The radioiodine scan in de Quervain's thyroiditis shows minimal or no uptake by the thyroid.
777. The risk of mortality in DKA is up to 5%, the majority of which is due to cerebral oedema.
778. The suspicion is that this woman has an underlying VIPoma. As such she is over-producing large
quantities of small bowel secretions, leading to bicarbonate and potassium loss. This leads to a normal anion
gap metabolic acidosis.
779. The symptoms of flu-like illness and arthritis, coupled with a rash which may well have represented a
previous tick bite are consistent with Lyme disease, caused by Borrelia burgdorferi.
780. The symptoms of hyperthyroidism are usually a result of increased beta-adrenergic tone, and
propranolol is therefore an effective treatment.
781. The synacthen stimulation test can evaluate adrenal gland function, and when 17-OHP levels are
measured concurrently, can help to distinguish between PCOS and non-classical CAH.
782. The three hallmarks of adhesive capsulitis are insidious shoulder stiffness, severe pain (even at night),
and near complete loss of passive and active external rotation of the shoulder.
783. Thiazide diuretics are associated with hypercalcaemia.
784. This patient has impaired fasting glucose and is therefore at significant risk of progression to type 2
diabetes and should be managed with aggressive diet and lifestyle intervention.
785. Thyroid hormone receptor is a nuclear receptor
786. Thyrotropin is glycosylated
787. Tinnitus is recognised as a rare side effect of bromocriptine therapy.
788. Transferrin physiology
789. Treatment with Thyroxine will often improve lipid abnormalities in hypothyroidism,
790. Trials of omega 3 supplementation suggest that it is associated with triglyceride reduction of up to 38%.
791. Trk is a proto-oncogene, mutation of which leads to activation of tyrosine kinase receptors linked to
thyroid papilary carcinoma.
792. TSH is the best measure for thyroid hormone replacement in primary hypothyroidism.
793. TSH receptor stimulating antibodies are specific to Graves' disease.
794. Turner syndrome is associated with essential hypertension (10%).
795. Turner's syndrome is associated with hypothyroidism in 24% patients.
796. Type 1 renal tubular acidosis is due to impaired secretion of hydrogen ions. Associated with
hypokalaemia and renal stones.

204
M Y Elamin
MBBS, DTM&H, MCTM, MRCPI 1& 2
797. Type 2 diabetics have a 40-60% increased risk of colon cancer.
798. Type 4 renal tubular acidosis is caused by hypoaldoseteronism, resulting in reduction in proximal
tubular ammonium excretion.
799. UFC is often recommended in the diagnosis of Cushing's syndrome and has a 95% specificity (85%
specificity in the obese) and a 98% sensitivity.
800. UKPDS (United Kingdom Prospective Diabetes Study) has shown that metformin is the preferable first
line drug therapy in type 2 diabetes.
801. Ultrasound is the initial investigation of choice for thyroid nodules; it can visualise cystic lesions 2 mm
or more in diameter, and solid lesions 3 mm or more in diameter.
802. Under-treatment with respect to blood glucose lowering therapy leads to hypertriglyceridaemia and
raised HbA1c.
803. Untreated acromegaly has 2-4 times increased mortality, the commonest cause being heart failure.
804. Urinary B-HCG will confirm pregnancy.
805. V2 ADH mutation is usually X linked recessive.
806. Viagra is contraindicated in patients taking nitrates
807. VIPoma is associated with elevated urea and calcium with hypokalaemic acidosis.
808. Vitamin D deficiency classically presents in the female Asian population whose clothing offers little
exposure to sunlight.
809. Vitamin D deficiency may occur after colectomy and suitable prophylaxis should be given.
810. Vitamin D has a number of actions; these include suppression of synthesis of type 1 collagen. This is
balanced by upregulation of osteocalcin, the balance of these changes is an increase in bone mineralisation.
811. Vitamin D is recognised to modulate cytokine production and may have a role in the treatment of
inflammatory disorders in the future.
812. Vitamin D-resistant rickets is X-linked dominant inheritance.
813. Vitiligo is associated with a number of autoimmune conditions, the most common of which is thyroid
disease.
814. Vitiligo is associated with thyroid disease, pernicious anaemia, and diabetes mellitus (in descending
order).
815. Weight gain is a potentially useful pointer to an underlying insulinoma.
816. Whilst duloxetine is first line therapy and amitriptyline second line therapy for peripheral diabetic
neuropathy, it is improved glycaemic control which has most effect on diabetic amyotrophy
817. WIth raised TSH and raised T4 in the presence of someone recently commenced on thyroid replacement
for hypothyroidism, consider compliance issues.
205
M Y Elamin
MBBS, DTM&H, MCTM, MRCPI 1& 2
818. 11 Beta-hydroxylase is responsible for conversion of 11-deoxycorticosterone and 11-deoxycortisol to
corticosterone and cortisol.
819. 40% of microadenomas will not be seen on imaging, therefore petrosal sinus sampling is necessary to
confirm pituitary source of cortisol (ACTH) excess.
820. A number of agents are known to interfere with thyroxine absorption, such as omeprazole.
821. A raised 17-OHP concentration is diagnostic for CAH
822. A TSHoma is rare. It usually presents with clinical hyperthyroidism in the context of raised TSH, T4
and T3.
823. Abnormal GTT with GH measurement is diagnostic of acromegaly.
824. Abuse of androgenic steroids can cause paranoid dellusions and aggression.
825. ACEi is first line blood pressure treatment in diabetes (but needs to be gradually titrated to avoid side-
effects)
826. Addison's disease (primary hypoadrenalism) is associated with low cortisol and aldosterone levels.
827. Addison's disease is associated with primary hypothyroidism in the complex of autoimmune
polyendocrine syndrome
828. Addison's disease is diagnosed by short synacthen test.
829. Addition of aldosterone antagonist therapy to ACE inhibition further impacts to reduce proteinuria in
diabetic renal disease.
830. Adrenal disease, Cushing’s syndrome.
831. Adrenal insufficiency in combination with T1 DM +/- premature ovarian failure is Schmidt's disease
(tpe 2 autoimmune polyendocrine syndrome)
832. Agranulocytosis (neutrophil count <0.5 ×109/L) associated with carbimazole is rare but life threatening
833. Agranulocytosis occurs in less than 1% of patients taking thionamides
834. Amiodarone can cause both hypothyroidism and thyrotoxicosis
835. Amiodarone inhibits peripheral conversion of T4 to T3.
836. Amiodarone-induced thyrotoxicosis is due to thyroiditis and can be treated with steroids
837. Amiodarone-induced thyrotoxicosis may be due to thyroiditis and can be treated with steroids or
withdrawal of amiodarone
838. Amitryptilline can also cause glaucoma
839. Anorexia nervosa may present with hypogonadotrophic hypogonadism and mildly elevated prolactin.
840. Antithyroid antibodies are commonly associated with pre-tibial myxoedema.
841. AntiTPO antibodies are present in 10% females without thyroid pathology
206
M Y Elamin
MBBS, DTM&H, MCTM, MRCPI 1& 2
842. Arterial blood gas is important to guide immediate management in acute pancreatitis
843. Bartter's syndrome is hypokalaemic metabolic acidosis with high urinary potassium loss.
844. Bisphosphonates are first line treatment in osteoporosis provided there are no contraindications and they
are tolerated.
845. Bisphosphonates are first line use for prevention of osteoporosis in post-menopausal women on steroids.
846. Bisphosphonates are proven to reduce symptoms of pain and serum alkaline phosphatase which acts as a
marker of disease activity in Paget's disease.
847. Bisphosphonates are the drug of choice in established osteoporosis
848. BP control shows greater reduction in CV risk than tight glyceamic control in UKPDS
849. Carbimazole may cause neutropenia (defined as neutrophils <0.5 ×109/L)
850. Carcinoid heart disease has a poor prognosis
851. Cerebral oedema in association with diabetic ketoacidosis (DKA) Is more common in children.
852. Cheiroarthropathy causes skin tightening in the hands resulting in contracture of the fingers
853. Chronic diabetes with nephropathy may result in hyporeninaemic hypoaldosteronism.
854. Coeliac disease is commonly associated with type 1 diabetes and will present with anaemia,
malabsorption and low albumin on blood testing.
855. Colour flow Doppler has been investigated as a useful tool to differentiate type 1 and type 2 amiodarone
induced thyroiditis.
856. Complaints of tiredness and weight gain in hypopituitary patients could be a sign of inadequate growth
hormone replacement.
857. Congenital GnRH deficiency (hypogonadotrophic hypogonadism) due to Kallmann's syndrome is
associated with anosmia, deafness, colour blindness and midline deformity.
858. Conn's is associated with hypokalaemic hypertension
859. Consider haemochromatosis if features of pigmentation, arthralgia, gynaecomastia and reduced libido
860. Craniopharyngioma causes an inferior bitemporal hemianopia and central diabetes insipidus.
861. Cushings is associated with acne, weight gain and mood disturbance, with dysmenorrhoea and low
oestrodiol.
862. Denosumab is a RANK-ligand inhibitor used in osteoporosis which works by reducing osteoclast
formation.
863. DeQuervain's thyroiditis is associated with painful gland, raised ESR and acute illness
864. DeQuervain's thyroiditis presents with elevated erythrocyte sedimentation rate (ESR), weight loss and
tender goitre and should be treated with analgesia..
207
M Y Elamin
MBBS, DTM&H, MCTM, MRCPI 1& 2
865. Diabetes insipidus
866. Diabetes insipidus is an important complication of neurosurgery; diagnosis requires an understanding of
urinary and serum sodium biochemistry.
867. Diabetic amyotrophy affects femoral nerve, lumbosacral plexus ad lumbar roots
868. Diabetic ketoacidosis is increasingly recognised in patents with type 1 diabetes who are prescribed an
SGLT-2 inhibitor.
869. Diabetic patients on insulin with a history of ischaemic heart disease should try to minimise
hypoglycemic episodes as this can precipitate ischaemia.
870. Diagnosis of acute pancreatitis
871. Dianette, a combination of cyproterone acetate and ethinyloestradiol is considered an effective option
for reducing hirsutism in patients with PCOS
872. Dopamine agonists are the treatment of choice in patients with micro/macroprolactinoma
873. Drugs that may cause SIADH include Selective serotonin reuptake inhibitors (SSRIs), Tricyclic
antidepressants, Sulphonylureas, Thiazides, and Carbamazepine.
874. During periods of illness, type 1 diabetics should test their blood sugar and ketone levels at least 4
hourly.
875. During periods of illness, type 2 diabetics should test their blood sugar levels at least 4 hourly.
876. Episodes of hypokalaemic periodic paralysis occur in 10% of young Latin American or Asian men with
thyrotoxicosis
877. Erythem ab igne is due to fire/hot-water bottle use and may be secondary to symptomatic
hypothyroidism
878. Erythromycine may potentiate the effect of carbimazole due to it's inhibition of liver enzyme activity
879. Failure to suppress cortisol below 50 nmol/L on a ODST test is highly suggestive of Cushing's disease
880. Features of neurofibromatosis Type 2 include cerebellar and retinal haemangioblastomas, café-au-lait
spots, and acoustic neuromas.
881. Females over the age of 75 who have a history of fragility fracture should be started on oral
bisphosphonate therapy
882. FHH is associated with raise serum calcium, normal PTH and low urinary calcium.
883. FHH is autosomal dominant
884. Fibrates are the primary medication for treatment of isolated hypertriglyceridaemia.
885. First-line therapy for PCOS has traditionally been the preparation Dianette, which contains
ethinyloestradiol (35 lg) in combination with cyproterone acetate (2 mg).

208
M Y Elamin
MBBS, DTM&H, MCTM, MRCPI 1& 2
886. Follicle-stimulating hormone (FSH) will be normal or low with PCOS while luteinisng hormone (LH)
will be elevated.
887. Follicular thyroid cancer is treated surgically.
888. Full alpha blockade prior to beta blockade with an agent such as phenoxybenzamine is recommended to
control blood pressure initially in suspected phaeochromocytoma.
889. Gastric CA may progress from gastric ulcer and may present with Acanthosis nigricans
890. Glucagon response to the mixed meal test is significantly disordered in patients with new onset Type 1
diabetes.
891. Glucagonoma is associated with necrolytic migratory erythema
892. Gonadal problems; interpretation of gonadotrophins and gonadal hormones.
893. Gonadotrophins; hormonal problems with gonads.
894. Grave's disease is associated with papillary carcinoma
895. Grave's disease is associated with vitiligo (7%)
896. Grave's disease with symptoms consistent with acute progressive optic neuropathy which occurs due to
stretching of the optic nerve requires urgent high dose steroids.
897. Grave's eye disease can occur in euthyroid, hypothyroid or hyperthyroid setting
898. Growth hormone deficiency occurs in abnormal body composition such as short stature of growth
retardation.
899. Hashimoto's disease is associated with positive anti-TPO (or microsomal) antibodies.
900. Heparin causes a false rise in free T4 due to displacement on assay
901. HOPE study showed increased benefit from ACEi in diabetes to reduce CV death.
902. Hypercalcaemia is initially treated with rehydration, IV normal saline
903. Hypercalcaemia would cause the PTH to be low due to negative feedback so a high calcium and PTH
suggests primary hyperparathyroidism
904. Hyperchloraemic metabolic acidosis is a documented complication of neobladder formation.
905. Hypoadrenalism is associated with hyponatreamia, hyperkalaemia, elevated TSH and mild
hypercalcaemia
906. Hypocalcaemia is an important complication of thyroid surgery, and you need to be able to recognise the
clinical presentation of this condition.
907. Hypogonadotrophic hypogonadism (low testosterone with inappropriately low or normal LH & FSH)
requires pituitary investigation primarily.

209
M Y Elamin
MBBS, DTM&H, MCTM, MRCPI 1& 2
908. Hypogonadotrophic hypogonadism in the presence of raised prolactin is likely secondary to
microprolactinoma
909. Hypogonadotrophic hypogonadism is common in obesity and diabetes, but sinister causes should be
excluded.
910. Hypokalaemic metabolic alkalosis is seen in Cushing's syndrome.
911. Hyponatraemia should be corrected slowly, except where seizures or significant neurological
dysfunction occurs.
912. Hyponatreamia with urine sodium <30, raised BUN and hypotension is likely multifactorial and should
be treated with Normal Saline
913. hypoparathyroidism leads to low PTH and low ionised calcium levels
914. Hypotestosteronaemia can present with reduced libido, tiredness and emotional problems and is not an
unusual finding in obese diabetic patients
915. Hypothyroidism and interpretation of thyroid function tests
916. Hypothyroidism can present as a medical emergency with myxoedema coma
917. Hypothyroidism is associated with mixed dyslipidaemia, hence increased triglycerides is the most
appropriate answer. Low HDL and a small increase in LDL are also seen.
918. In a case of hypoglycaemia, check paired insulin and c-peptide levels and blood sulphonyl urea levels.
919. In cases of thyrotoxicosis or thyroid storm the initial treatment is with beta blockers.
920. In Conn’s syndrome Aldosterone inhibition with spironolactone will bring the greatest additional
reduction in blood pressure.
921. In hypoadrenalism thyroxine can precipitate acute hypoadrenalism
922. In premature ovarian failure Patients should be counselled with respect to appropriate hormone
replacement to protect against osteoporotic fracture and about likely infertility.
923. In thyrotoxicosis factitia, uptake is globally reduced on a radioactive uptake scan.
924. In women with oopthrectomy unopposed oestrogen HRT is appropriate.
925. Inability to concentrate urine during the water deprivation test, which improves with DDAVP is central
DI
926. Inappropriately normal PTH in the face of hypercalcaemia is diagnostic of primary hyperparathyroidism
927. Increased sun exposure results in increased vitamin D production.
928. indapamide enhance potassium excretion and not contribute to oedema and so is useful in the
management of blood pressure control in chronic kidney disease
929. Infertility and gynaecomastia are two of the most common features resulting in a diagnosis of
Klinefelter's syndrome.
210
M Y Elamin
MBBS, DTM&H, MCTM, MRCPI 1& 2
930. Insulin is the safest agent to address hyperglycaemia in renal insufficiency.
931. Insulinoma can present with collapse/sweating relieved by carbohydrate
932. It is recognised that long term use of glitazone therapy increases the risk of osteoporotic fracture. (for
example pioglitazone or rosiglitazone)
933. JBDS DKA guideline - no evidence for IV bicrabonate with pH >7
934. Klinefelters does not have a genetic pattern of inheritance
935. Klinefelter's is characterised by tall stature, small testes, azoospermia and gynaecomastia in a male.
Typical karyotype is 47XXY, though mosaics occur with 46XY/47XXY karyotype.
936. Klinefelter's syndrome is suggested by hypergonadotrophic hypogonadism, gynaecomastia and low IQ.
937. Leaving both corticosteroid and mineralocorticoid dose unchanged is inappropriate in patients on long
term steroids who present with infections.
938. Legionnaire's disease may be assocaited with hyponatreamia secondary to SIADH
939. Lesion <1 cm in the pituitary and hyperprolactinaemia is MICROprolactinoma
940. Less than 1% of patients suffer leukopenia or neutropenia as a result of thionamides
941. Lipoprotein lipase deficiency will produce hyperchylomicronaemia/hypertriglycerideamia.
942. Liraglutide can only be used in patients of BMI >35 who are of European descent in combination with
metformin.
943. Long term exposure to tetracyclines can result in nephrogenic diabetes insipidus which can be confirmed
using a water deprivation test to asses the kidney's ability to concentrate urine.
944. Loss of first phase insulin response is closely associated with the development of diabetes, with 100% of
subjects with loss of first phase insulin response and anti-IA2 antibodies progressing to type 1 diabetes
within 2 years.
945. Low TSH with normal thyroid hormone levels is subclinical hyperthyroidism and is unlikely to cause
LVF
946. Macroadenoma witll cause visual field defects
947. Management of DKA in adults includes recommends replacement of 40 mmol/L where potassium is
between 3.5 and 5.5 mmol/L.
948. Management of T2DM
949. MEN 2 is made up of phaeochromocytoma, medullary thyroid carcinoma and parathyroid hyperplasia
950. MEN syndromes and screening.
951. MEN-1 is associated with pancreatic endocrine tumours, 60% of which are gastrinomas and 30% are
insulinomas.

211
M Y Elamin
MBBS, DTM&H, MCTM, MRCPI 1& 2
952. MEN2 - phaeochromocytoma, medullary thyroid cancer and parathyroid adenoma.
953. Metabolic bone disease, bone markers.
954. Metoclopramide is a pro-kinetic antiemetic and so is appropriate initial therapy for diabetic
gastroparesis.
955. Myxoedema coma is a medical emergency requiring treatment with intravenous thyroxine and
hydrocortisone.
956. Necrobiosis lipoidica is a typically painless lesion assoicated with diabetes
957. Nelson's syndrome effects 30% adrenalectomised patients with Cushing's disease
958. Neuroendocrine tumours usually respond well to somatostatin analogues
959. NICE recommend NPH insulin once or twice daily according to need as the first-line option when
initiating insulin in a type 2 diabetic.
960. Non-suppressed TSH with elevated alpha subunit is in keeping with TSH-oma
961. Normal free thyroid hormone levels but increased total levels indicate increased binding globulins as
seen in pregnancy
962. Normal saline is usually the first line in the management of hypercalcaemia even in the presence of
normal or slightly raised serum sodium levels.
963. Oestrogen therapy causes raised thyroxie binding globulin, effecting total thyroid hormone levels.
964. Oral diabetic agents can generally be continued when insulin is started
965. Osteopenia is T score -1 to -2.5
966. Paget’s disease; metabolic bone disease; calcium and parathyroid hormone.
967. Papillary carcinoma of the thyroid is the commonest type of thyroid cancer, large thyrocytes and
psammoma bodies are features typically seen on histology.
968. Patients on anti-obesity drugs who gain weight should have the drug withdrawn.
969. Patients taking OCP will have unrecordable ostradiol levels
970. Patients who have limited sun exposure or are fully covered for religious reasons should be screen for
Vitamin D deficiency.
971. Patients with Kleinfelters syndrome should receive testosterone treatment to reduce their risk of
osteoporotic fracture.
972. PCOS is associated with insulin resistance with mildly elevated tesosterone and increased LH:FSH
973. PCOS is associated with insulin resistance; acanthosis nigricans may be present.
974. Petrosal sinus sampling helps to differentiate pituitary from ectopic ACTH-dependent Cushing's
syndrome
212
M Y Elamin
MBBS, DTM&H, MCTM, MRCPI 1& 2
975. Pituitary apoplexy can cause compression of the occular nerves and trigeminal nerve within the
cavernous sinus.
976. Pituitary hormones; gonadal disease.
977. Pituitary hormones; prolactin.
978. Post-partum thyroiditis is usually transient, with hyperthyroidism 2-6 months post-partum and then
hypothyroidism. It rarely needs treatment.
979. pregnancy blood result show high oestrogen, low luteinising hormone (LH) and follicle-stimulating
hormone (FSH) and elevated prolactin and beta-HCG.
980. Pregnancy blood results show high oestrogen, low luteinising hormone (LH) and follicle-stimulating
hormone (FSH) and elevated prolactin and beta-HCG.
981. Pregnancy increases thyroid binding globulin
982. Presentation of metabolic acidosis in a patient with methanol toxicity
983. Pretibial myxodeama rarely require treatment.
984. Primary hypogonadism is associated with low testosterone and sperm count with appropriately elevated
LH & FSH.
985. Prolactin above 2000 mU/L is suggestive of prolatinoma
986. Prolactin levels are of no clinical use during prenancy as they will always be elevated
987. Propylthiouracil is best used in breast feeding mothers.
988. Propylthiouracil may be used in pregnancy
989. Proven interventions in the treatment of diabetic nephropathy include ACE inhibitors (greatest benefit),
low dietary protein and improved glycaemic control
990. Psuedocushing syndrome resolves rapidly with abstinence from alcohol
991. Psuedohypoparathyroidism is associated with slipped epiphyseal plate in childhood.
992. PTU is recommended in the first trimester and then carbimazole thereafter
993. Radioiodine therapy can worsen thyroid eye disease
994. Radio-iodine uptake scan will help differentiate amiodarone induced thyrotoxicosis type 1 from 2, as
they are treated differently.
995. Radioiodine uptake scan will show reduced uptake in De Quervain's thyroiditis
996. Raised insulin with low c-peptide indicates exogenous insulin
997. Raised oestradiol with raised prolactin and low LH/FSH is likley due to pregnancy
998. Raised TSH with elevated thyroid hormone levels in someone know to have hypothyroidism is likely
due to non-compliance
213
M Y Elamin
MBBS, DTM&H, MCTM, MRCPI 1& 2
999. Raised urinary cortisol with suppressed ACTH is indicative of Cushing's syndrome with adrenal source.
1000. Reduce insulin dose in steps of 20% if hypoglycaemia occurs.
1001. Reducing insulin requirements, weight loss and hypoglycaemia should alert you to autoimmune
hypoadrenalism if found in a type 1 diabetic
1002. Renin, aldosterone and Conn’s syndrome.
1003. Revise NICE guidance on type 2 diabetes and the introduction of insulin
1004. Serum ferritin is a useful screenig test for haemochromatosis
1005. SGLT-2 inhibition is associated with increased risk of bone fracture, this is thought to be associated with
a small but significant increase in PTH.
1006. Short synacthen test confirms diagnosis of Addison's disease
1007. Short synacthen test will confirm diagnosis of Addison's disease
1008. SIADH, ADH, normal regulation of osmolality.
1009. Sildenafil and other phosphodiesterase inhibitors may substantially reduce urine output in nephrogenic
diabetes insipidus.
1010. Small cell carcinoma may be associated with ectopic ACTH production, causing Cushing's syndrome
1011. Solitary toxic nodule will produce a minimal/subclinical hyperthyroidism.
1012. Somatostatin analogues are the treatment of choice for carcinoid symptoms
1013. Somatostatin analogues improve symptoms and prognosis in carcinoid syndrome
1014. Steroid replacement is the initial management of adrenal insufficiency.
1015. Symptoms of GH deficiency include lethargy, low mood and poor concentration
1016. Syndrome of inappropriate antidiuretic hormone (SIADH) is diagnosed with hyponatraemia and
elevated urine sodium, an osmolality towards the upper limit of normal, and low plasma osmolality.
1017. T3 toxicosis should be considered in patients with symptomsbut normal T4 and suppressed TSH levels
1018. TB adrenalitis can be diagnosed wih CT showing gland englargement predominantly
1019. Tendinous xanthomata is pathognomonic for type II(a) hyperlipidaemia (familial
hypercholesterolaemia)
1020. Teriparatide is the 1-34 active fragment of human parathyroid hormone used in osteoporosis when
bisphosphonate therapy is not tolerated and works by osteoblast activation.
1021. Testosterone replacement may improve bone mineral density
1022. The diagnosis of diabetes mellitus.
1023. The initial management of HHS is with IV rehydration (and later addition os insulin if required)
214
M Y Elamin
MBBS, DTM&H, MCTM, MRCPI 1& 2
1024. The initial treatment for impaired fasting glucose is diet and exercise advice.
1025. The main benefit of GLP-1 therapy in patients with established type 1 diabetes appears to be in reducing
body weight.
1026. The main priories in the management of sepsis should be high flow oxygen, fluid resuscitation, accurate
urine output monitoring, empiric antibiotics, blood cultures and the measurement of serum lactate.
1027. The mechanism by which lithium leads to diabetes insipidus, reduced GSK3 beta signalling, is well
established.
1028. The presence of breast development in the absence of secondary sexual hair, with a history of hernias as
a child is suggestive of a diagnosis of androgen insensitivity syndrome.
1029. The synacthen stimulation test can evaluate adrenal gland function, and when 17-OHP levels are
measured concurrently, can help to distinguish between PCOS and non-classical CAH.
1030. There is 50% chance of recurrence following treatment with antithyroid medications
1031. There is no evidence that insulin reduces CV death in primary prevention
1032. Thyroglobulin measurement helps differentiate factious thyrotoxicosis from other diagnosis.
1033. Thyroid disease and non-thyroidal illness, TFT interpretation.
1034. Thyroid ultrasound is the most useful investigation for characterising thyroid nodules.
1035. Thyroidal neck swellings can be due to goitre or nodules and can be present in hypothyroid, euthyroid or
hyperthyroid states.
1036. Thyrotoxicosis is associated with high alkaline phosphatase secondary to increased bone turnover
1037. Thyroxine may exacerbate/precipitate Addisonian crisis in undiagnosed patients
1038. TSH is elevated with normal thyroid hormone levels in patients with poor compliance with medication
1039. Type 2 amiodarone-induced thyrotoxicosis responds to withdrawal of amiodarone and steroids.
1040. Type 3 hyperlipidaemia is associated with tendon xanthoma, pancreatitis and premature cardiovascular
disease.
1041. UKPDS showed reduction in CV risk with BP control versus glycaemia control.
1042. Urinary free cortisol or 1 mg overnight dexamethasone suppression test has 95% sensitivity and
specificity for diagnosing Cushing's syndrome
1043. Use of bisphosphonates and GFR.
1044. Vivid dreams and lethargy may be due to nocturnal hypoglycaemia
1045. Weight loss and osmotic symptoms suggest insulinopenia and therefore insulin should be commenced if
glycaemic control is suboptimal.

215
M Y Elamin
MBBS, DTM&H, MCTM, MRCPI 1& 2
1046. When considering twice daily pre-mixed insulin: the morning dose is titrated according to the pre-lunch
and pre-evening meal blood glucose readings; the evening dose is titrated against the pre-bed and pre-
breakfast readings.
1047. When up-titrating insulin, increase in steps of 10%
1048. WIth raised TSH and raised T4 in the presence of someone recently commenced on thyroid replacement
for hypothyroidism consider compliance issues.
1049. Xendos study concludes orlistat and diet control reduces risk of diabetes in obese patients by 38%

216
M Y Elamin
MBBS, DTM&H, MCTM, MRCPI 1& 2
GASTROENTEROLOGY
1. 24hr oesophageal PH monitoring is gold standard investigation in GORD
2. A 10-day-old baby boy presents with abdominal distension and vomiting. He first passed meconium at 72 hours
and has not opened is bowels since. An abdominal x-ray shows a dilated colon - Hirschsprung's disease
3. A 12-month-old develops profuse watery diarrhoea associated with a low-grade fever. A number of other children
at his day-care centre have similar symptoms - rotavirus
4. A 15-year-old presents with pain in the right iliac fossa associated with anorexia and low-grade pyrexia -
appendicitis
5. A 1-year-old boy with a recent history of rectal bleeding presents with intestinal obstruction. During surgery he
is found to have a omphalomesenteric band - Meckel's diverticulum
6. A 20-year-old man of Iranian Jewish descent is investigated for jaundice and found to have an elevated conjugated
bilirubin level - Dubin-Johnson syndrome
7. A 20-year-old man presents with a 3-week history of bloody diarrhoea associated with tenesmus - ulcerative
colitis
8. A 20-year-old woman presents with diarrhoea. Over the past few months she has lost weight and suffered with
recurrent abdominal pain and mouth ulcers - Crohn's disease
9. A 20-year-old woman presents with recurrent episodes of abdominal pain associated with bloating. The pain is
relieved on defecation. She normal passes 3 loose stools with mucous in the mornings - irritable bowel syndrome
10. A 20-year-old woman with a history of emotional lability and depression presents with an asymmetrical tremor.
On examination she has early stigmata of chronic liver disease - Wilson's disease
11. A 30-year-old woman presents with foul smelling oily diarrhoea, abdominal bloating, fatigue and weight loss. On
examination she has papulovesicular lesions on the extensor aspects of her arms - coeliac disease
12. A 35-year-old man presents with fatigue, erectile dysfunction and arthralgia. On examination he 'bronze'
pigmentation of his skin and some early stigmata of chronic liver disease - haemochromatosis
13. A 35-year-old man with painful defecation associated with bright red, rectal bleeding - anal fissure
14. A 40-year-old alcoholic presents with haematemesis after repeated episodes of vomiting. An endoscopy two
months ago showed no evidence of varices - Mallory-Weiss syndrome
15. A 40-year-old man with a history of ulcerative colitis presents with fatigue, jaundice and pruritus and right upper
quadrant pain - primary sclerosing cholangitis
16. A 45-year-old alcoholic man presents with severe epigastric pain associated with vomiting. He has had a number
of similar episodes before - acute pancreatitis
17. A 45-year-old man is being investigated for diarrhoea, weight loss and arthralgia. Jejunal biopsy shows
deposition of macrophages containing PAS-positive granules - Whipple's disease
18. A 45-year-old woman presents with fatigue and pruritus. Blood tests show a raised bilirubin, ALP and IgM level
- primary biliary cholangitis
19. A 50-year-old man presents with epigastric pain relieved by eating - duodenal ulcer
217
M Y Elamin
MBBS, DTM&H, MCTM, MRCPI 1& 2
20. A 50-year-old man presents with epigastric pain worsened by eating - gastric ulcer
21. A 50-year-old obese man with a history of type 2 diabetes mellitus is noted to have mild hepatomegaly on
examination. He drinks one glass of wine per week. Blood tests show a slightly raised ALT and a liver ultrasound
shows increased echogenicity - non-alcoholic fatty liver disease
22. A 55-year-old presents with RUQ pain, fever and jaundice. His blood pressure is 98/60 mmHg and he appears
slightly confused - ascending cholangitis
23. A 60-year-old alcoholic man presents with severe chest pain after repeated episodes of vomiting. Subcutaenous
emphysema is noted on examination - Boerhaave syndrome (oesophageal rupture)
24. A 60-year-old woman presents with dysphagia and lethargy. On examination she has a smooth, swollen tongue
and pale conjunctivae - Plummer-Vinson syndrome:
a. Dysphagia (secondary to oesophageal webs)
b. Glossitis
c. Iron-deficiency anaemia
d. Treatment includes iron supplementation and dilation of the webs
25. A 60-year-old woman with a history of polycythaemia rubra vera presents with abdominal pain and distension.
She is found to have ascites and hepatomegaly on examination - Budd-Chiari syndrome
26. A 65-year-old man with a history of chronic hepatitis b infection presents with symptoms and signs of liver
cirrhosis. Alpha-fetoprotein is elevated. - hepatocellular carcinoma
27. A 65-year-old woman presents with jaundice, weight loss and passing clay-coloured stools. She also describes
recurrent bouts of colicky RUQ abdominal pain. On examination a mass is palpable in the RUQ -
cholangiocarcinoma
28. A 6-week-old baby boy presents with 'projectle' vomiting 30 minutes after feeding. On examination he is mildy
dehydrated and has a small palpable mass in the upper abdomen - pyloric stenosis
29. A 70-year-old man presents with anorexia, weight loss and painless jaundice - pancreatic cancer
30. A 75-year-old man presents with passing large amounts of a mucous-like diarrhoea. Bloods demonstrate
hypokalaemia - villous adenoma
31. A 75-year-old woman presents with colicky pain typically in the LLQ associated with diarrhoea and fever -
diverticulitis
32. A baby develops persistent, marked jaundice after birth. Bloods show a very high level of unconjugated bilirubin
- Crigler-Najjar syndrome (type 1)
33. A man who presents with intussusception is noticed to have multiple polyps on colonoscopy. These are later shown
to be hamartomatous. He also has pigmented lesions on his lips and palms - Peutz-Jeghers syndrome
34. A middle-aged man is investigated for episodic flushing, diarrrhoea and bronchospasm. His urinary 5-HIAA
levels are found to be elevated - carcinoid syndrome
35. A middle-aged patient presents with dysphagia of both liquids and solids from the start. They also complain of
regurgitation and a persistent cough - achalasia
218
M Y Elamin
MBBS, DTM&H, MCTM, MRCPI 1& 2
36. A patient who is known to have HIV presents with odynophagia - oesophageal candidiasis
37. A patient with a history of anxiety complains of having a 'lump' in her throat - globus hystericus
38. A patient with a history of heartburn presents with odynophagia. There no weight loss, vomiting or anorexia -
oesophagitis
39. A patient with a known history of Barrett's oesophagus presents with dysphagia and weight loss - oesophageal
cancer
40. A woman complains of lethargy, diplopia and dysphagia. She is noted to have a ptosis on examination -
myasthenia gravis
41. A young man is noted to have an elevated unconjugated bilirubin level on a fasting blood sample - Gilbert's
syndrome
42. Abdominal pain, ascites, tender hepatomegaly - Budd-Chiari syndrome
43. Abdominal pain, rectal bleeding, metabolic acidosis - mesenteric ischaemia
44. Acute variceal haemorrhage , treatment of choice: terlipressin
45. An elderly man complains of dysphagia, halitosis, regurgitation and cough - pharyngeal pouch
46. An elderly man presents with severe central abdominal pain radiating to the back associated with hypotension -
ruptured abdominal aortic aneurysm
47. An elderly man presents with vomiting, 'constipation' and abdominal distension. On examination he has 'tinkling'
bowel sounds - intestinal obstruction
48. An elderly patient presents with watery diarrhoea after being treated for pneumonia. Blood tests show a new,
marked neutrophilia - Clostridium difficile
49. An elderly patient who has a history of aortic stenosis presents with rectal bleeding. A colonoscopy shows multiple
small, flat, erythematous lesions with scalloped edges - angiodysplasia
50. An infant is investigated for persistent jaundice and is found to have a high level of unconjugated bilirubin. There
is some improvement with phenobarbital - Crigler-Najjar syndrome (type 2)
51. An obese 50-year-old woman presents with pain in the RUQ which radiates to the interscapular region. She is
apyrexial and not jaundiced - biliary colic
52. An obese 55-year-old presents with pain in the RUQ associated with fever. Palpation of the RUQ causes arrest of
respiration - acute cholecystitis
53. An overweight 45-year-old woman presents with recurrent episodes of right upper quadrant pain that is made
worse by eating a fatty meal - biliary colic
54. Aortic stenosis, iron-deficiency anaemia - angiodysplasia
55. Bile acid malabsorption - SeHCAT
56. Bile acid malabsorption , treatment of choice: cholestyramine
57. Budd-Chiari syndrome - thrombosis of the hepatic vein
58. Carcinoid syndrome - urinary 5-HIAA
59. CCK - decreases gastric emptying
219
M Y Elamin
MBBS, DTM&H, MCTM, MRCPI 1& 2
60. CCK - I cells in upper small intestine
61. CCK - increases gallbladder contraction
62. CCK - increases pancreatic secretions rich in digestive enzymes
63. Child, perianal itch - Enterobius vermicularis infection
64. Cholestyramine , uses include: hyperlipidaemia
65. Cholestyramine , uses include: treatment resistant diarrhoea in Crohn's disease
66. Cholestyramine may cause cholesterol gallstones
67. Cholestyramine may cause constipation
68. Cholestyramine may cause decreases absorption of fat-soluble vitamins
69. Chronic anal fissure , treatment of choice: topical glyceryl trinitrate
70. Coeliac disease - anti-endomyseal antibody
71. Coeliac disease - anti-tissue transglutaminase antibody
72. Coeliac disease complication - anaemia
73. Coeliac disease complication - hyposplenism
74. Coeliac disease complication - lactose intolerance
75. Coeliac disease complication - osteoporosis
76. Coeliac disease complication - small bowel lymphoma
77. Coeliac disease complication - subfertility
78. Crohn's disease - cobble-stone appearance on endoscopy
79. Crohn's disease - fistula
80. Crohn's disease - gallstones
81. Crohn's disease - granulomas
82. Crohn's disease - increased goblet cells
83. Crohn's disease - inflammation anywhere from the mouth to anus
84. Crohn's disease - inflammation in all layers from mucosa to serosa
85. Crohn's disease - intestinal obstruction
86. Crohn's disease - Kantor's string sign
87. Crohn's disease - mouth ulcers
88. Crohn's disease - oxalate renal stones
89. Crohn's disease - perianal disease
90. Crohn's disease - rose thorn ulcers
91. Crohn's disease - skip lesions
92. Crohn's disease - weight loss
93. Diarrhoea, flushing, bronchospasm - carcinoid syndrome
94. Diarrhoea, weight loss, lymphadenopathy, arthralgia, fever - Whipple's disease
95. Direct inguinal hernia low risk of strangulation
220
M Y Elamin
MBBS, DTM&H, MCTM, MRCPI 1& 2
96. Direct inguinal hernia much more common in males
97. Direct inguinal hernia passes medial to the inferior epigastric artery
98. Dubin-Johnson syndrome - mutation in the canalicular multidrug resistance protein 2 (MRP2) results in defective
hepatic excretion of bilirubin
99. Dysphagia (oesophageal webs), glossitis, iron-deficiency anaemia - Plummer-Vinson syndrome
100. Elderly patient, LIF pain, diarrhoea, fever - diverticulitis
101. Familial adenomatous polyposis - chromosome 5
102. Female, middle-aged, episodic RUQ pain - biliary colic
103. Femoral hernia high risk of strangulation
104. Femoral hernia more common in females
105. First episode of C. difficile infection - oral vancomycin
106. Gastrin - G cells in antrum of the stomach
107. Gastrin - increases gastric H+ secretion
108. Gastrin - increases gastric motility
109. Gastrin - increases pepsinogen secretion
110. Gastrin - stimulates parietal cell maturation
111. Crigler-Najjar syndrome (type 1) - absence of UDP-glucuronyl transferase
112. Gilbert's syndrome - mild deficiency of UDP-glucuronyl transferase
113. Haemochromatosis - arthritis
114. Haemochromatosis - bronze skin pigmentation
115. Haemochromatosis - chromosome 6
116. Haemochromatosis - diabetes mellitus
117. Haemochromatosis - dilated cardiomyopathy
118. Hamartomatous polyps in GI tract, pigmented lesions on mouth/palms/soles - Peutz-Jeghers syndrome
119. HBsAg negative, anti-HBs positive, IgG anti-HBc negative - previous immunisation
120. HBsAg negative, anti-HBs positive, IgG anti-HBc positive - previous infection, not a carrier
121. HBsAg positive, anti-HBs negative, IgG anti-HBc positive, IgM anti-HBc negative - chronic infection
122. HBsAg positive, anti-HBs negative, IgM anti-HBc positive - acute infection
123. Helicobacter pylori may cause B cell lymphoma of MALT tissue
124. Helicobacter pylori may cause gastric carcinoma
125. Hepatorenal syndrome - splanchnic vasodilation
126. Hepatorenal syndrome , treatment of choice: terlipressin
127. Indirect inguinal hernia low risk of strangulation
128. Indirect inguinal hernia may occur in infants
129. Indirect inguinal hernia passes lateral to the inferior epigastric artery
130. Isolated raised unconjugated bilirubin on fasting - Gilbert's syndrome
221
M Y Elamin
MBBS, DTM&H, MCTM, MRCPI 1& 2
131. Life-threatening C. difficile infection - oral vancomycin AND IV metronidazole
132. Liver cysts - Echinococcus granulosus infection
133. Liver disease, neuropsychiatric features - Wilson's disease
134. Low serum ceruloplasmin - Wilson's disease
135. Metoclopramide - D2 receptor antagonist
136. Metoclopramide may cause hyperprolactinaemia
137. Metoclopramide may cause oculogyric crisis
138. Metoclopramide may cause parkinsonism
139. Metoclopramide may cause tardive dyskinesia
140. Middle aged female, fatigue, pruritus, raised IgM - primary biliary cholangitis
141. Multiple colonic polyps, osteomas, epidermoid cysts - Gardner's syndrome
142. Omeprazole - irreversible blockade of H+/K+ ATPase
143. Painless jaundice, steatorrhoea - pancreatic cancer
144. Palpation in LLQ causes pain in RLQ - acute appendicitis
145. Pharyngeal pouch - barium swallow combined with dynamic video fluoroscopy
146. Primary biliary cirrhosis - anti-mitochondrial antibodies
147. Primary sclerosing cholangitis - an association with ulcerative colitis
148. Prophylaxis of variceal haemorrhage, treatment of choice: propranolol
149. Raised conjugated bilirubin, black liver - Dubin-Johnson syndrome
150. Raised unconjugated bilirubin, neonatal jaundice, kernicterus - Crigler-Najjar syndrome
151. Recent antibiotic use, profuse diarrhoea, high white cell count - Clostridium difficile infection
152. Recurrent episode of C. difficile infection within 12 weeks of symptom resolution - oral fidaxomicin
153. Right upper quadrant pain, fever, jaundice - ascending cholangitis
154. Secretin - decreases gastric H+ secretion
155. Secretin - increases pancreatic bicarbonate secretion
156. Secretin - S cells in upper small intestine
157. Severe alcoholic hepatitis , treatment of choice: prednisolone
158. Severe central abdominal pain, history of alcohol or gallstones - acute pancreatitis
159. Severe vomiting --> haematemesis - Mallory-Weiss syndrome
160. Somatostatin - D cells in the pancreas & stomach
161. Somatostatin - decreases gallbladder contraction
162. Somatostatin - decreases gastric emptying
163. Somatostatin - decreases gastric H+ secretion
164. Somatostatin - decreases pancreatic secretions
165. Somatostatin - decreases pepsinogen secretion
166. Strong family history of colorectal and endometrial cancer - hereditary non-polyposis colorectal carcinoma
222
M Y Elamin
MBBS, DTM&H, MCTM, MRCPI 1& 2
167. Sulphasalazine may cause headache
168. Sulphasalazine may cause Heinz body anaemia
169. Sulphasalazine may cause megaloblastic anaemia
170. Sulphasalazine may cause oligospermia
171. Sulphasalazine may cause pulmonary fibrosis
172. Sulphasalazine may cause Steven-Johnson syndrome
173. Tropheryma whippelii - jejunal biopsy shows periodic acid-Schiff positive macrophages
174. Ulcerative colitis - bloody diarrhoea
175. Ulcerative colitis - colorectal cancer
176. Ulcerative colitis - continuous disease
177. Ulcerative colitis - crypt abscesses
178. Ulcerative colitis - depletion of goblet cells
179. Ulcerative colitis - drainpipe colon
180. Ulcerative colitis - inflammation starting at the rectum and not spreading beyond the ileocaecal valve
181. Ulcerative colitis - loss of haustrations on barium enema
182. Ulcerative colitis - no inflammation beyond submucosa
183. Ulcerative colitis - primary sclerosing cholangitis
184. Ulcerative colitis - pseudopolyps seen on endoscopy
185. Ulcerative colitis - tenesmus
186. VIP - decreases gastric H+ secretion
187. VIP - increases secretion of water from pancreas and intestines
188. Vomiting --> severe chest pain, shock - Boerhaave syndrome
189. Wilson's disease - asterixis
190. Wilson's disease - basal ganglia degeneration
191. Wilson's disease - blue nails
192. Wilson's disease - chorea
193. Wilson's disease - chromosome 13
194. Wilson's disease - defect in the ATP7B gene
195. Wilson's disease - dementia
196. Wilson's disease - haemolytic anaemia
197. Wilson's disease - Kayser-Fleischer rings
198. Wilson's disease - renal tubular acidosis
199. Wilson's disease , treatment of choice: penicillamine
200. Achalasia Failure of oesophageal peristalsis and of relaxation of the lower oesophageal sphincter (LOS) due to
degenerative loss of ganglia from Auerbach's plexus case dysphagia of BOTH liquids and solids (heart burn,
regurgitation, cough and aspiration pneumonoia. Malignant chage small no of patients.
223
M Y Elamin
MBBS, DTM&H, MCTM, MRCPI 1& 2
201. A retrocardiac air-fluid level and wide mediastinum is sometimes seen in patients with achalasia xray for
Achalasia.
202. oesophageal manometry (excessive LOS tone which doesn't relax on swallowing) considered the most important
diagnostic test for Achalasia.
203. Achalasia Management, Pneumatic (balloon) dilation is increasingly the preferred first-line option, Surgery in
some patients, intra-sphincteric injection of botulinum toxin is sometimes, and drug therapy (e.g. nitrates, calcium
channel blockers) has a role but is limited by side-effects.
204. Acute appendicitis is the most common acute abdominal condition requiring surgery. It can occur at any age but
is most common in young people aged 10-20 years.
205. lymphoid hyperplasia or a faecolith → obstruction of appendiceal lumen → gut organisms invading the
appendix wall → oedema, ischaemia +/- perforation
206. laparoscopic appendicectomy is now the treatment of choice
207. administration of prophylactic intravenous antibiotics reduces wound infection rates
208. patients with perforated appendicitis (typical around 15-20%) require copious abdominal lavage
209. be wary in the older patients who may have either an underlying caecal malignancy or perforated sigmoid
diverticular disease.
210. Paracetamol overdose: necrosis affects the entire acinus (panacinar necrosis) resulting in liver failure.
211. A combination of liver and neurological disease points towards Wilson's disease
212. the apoptosis seen in patients with mild cases of viral hepatitis, resulting in the possibility of regeneration and
recovery of hepatocellular function
213. Acute liver failure,
a. Paracetamol overdose,
b. Alcohol,
c. Viral hepatitis (usually a or b)
d. Acute fatty liver of pregnancy
214. Features is common ('hepatorenal syndrome')
a. Jaundice,
b. Coagulopathy: raised prothrombin time,
c. Hypoalbuminaemia
d. Hepatic encephalopathy
e. Renal failure
215. A non-cardioselective B-blocker (NSBB) is used for the prophylaxis of oesophageal bleeding
216. A recurrent episode of C. Difficile within 12 weeks of symptom resolution should be treated with oral fidaxomicin
217. A severe flare of ulcerative colitis should be treated in hospital with IV corticosteroids
218. Acute pancreatitis is the most common complication of ERCP
219. Acute respiratory distress syndrome is a recognised complication of acute pancreatitis
224
M Y Elamin
MBBS, DTM&H, MCTM, MRCPI 1& 2
220. Amylase: breaks starch down to sugars
221. An isolated rise in bilirubin in response to physiological stress is typical of Gilbert's syndrome
222. Angiodysplasia is associated with aortic stenosis
223. Antibiotic prophylaxis reduces mortality in cirrhotic patients with gastrointestinal bleeding
224. Bezlotoxumab is a monoclonal antibody which targets C. Difficile toxin B
225. Brush border enzymes:
a. Maltase: glucose + glucose
b. Sucrase: glucose + fructose
c. Lactase: glucose + galactose
226. Budd–Chiari syndrome - ultrasound with Doppler flow studies is very sensitive and should be the initial
radiological investigation
227. Budd-Chiari syndrome is most likely due to a thrombophilia
228. Carcinoid syndrome can affect the right side of the heart. The valvular effects are tricuspid insufficiency and
pulmonary stenosis
229. Causes of villous atrophy (other than coeliacs):
a. Tropical sprue,
b. Whipple's,
c. Lymphoma,
d. Hypogammaglobulinaemia
230. CCK - I cells in upper small intestine
231. Cessation of smoking may trigger an ulcerative colitis flare
232. Charcot's cholangitis triad: fever, jaundice and right upper quadrant pain
233. Chronic anal fissure - topical glyceryl trinitrate
234. Clindamycin treatment is associated with a high risk of C. Difficile
235. Co-danthramer is genotoxic and should only be prescribed to palliative patients due to its carcinogenic potential
236. Coeliac disease - tissue transglutaminase antibodies are the first-line test
237. Coeliac disease has a strong association with HLA-DQ2 (present in 95% of patients)
238. Colorectal cancer screening - PPV of faecal occult blood = 5 - 15%
239. Constipation can be a trigger for liver decompensation in cirrhotic patients
240. Corticosteroids are used in the management of severe alcoholic hepatitis
241. Courvoisier's sign - a palpable gallbladder in the presence of painless jaundice is unlikely to be gallstones
242. CT pancreas is the preferred diagnostic test for chronic pancreatitis - looking for pancreatic calcification
243. Diarrhoea + hypokalaemia → villous adenoma
244. Diarrhoea - biospy shows pigment laden macrophages = laxative abuse
245. Diarrhoea, weight loss, skin, arthralgia, lymphadenopathy, ophthalmoplegia, skin or neurological change
?Whipple's disease
225
M Y Elamin
MBBS, DTM&H, MCTM, MRCPI 1& 2
246. Dietary modification and topical steroids are used in the management of eosinophilic oesophagitis
247. Dysphagia affecting both solids and liquids from the start - think achalasia
248. Dysphagia, aspiration pneumonia, halitosis → ?Pharyngeal pouch
249. Eosinophilic esophagitis typically presents in young men with symptoms of dysphagia and in patients with a
history of food allergy, eczema and asthma
250. ERCP/MRCP are the investigations of choice in primary sclerosing cholangitis
251. Faecal elastase is a useful test of exocrine function in chronic pancreatits
252. Familial adenomatous polyposis is due to a mutation in a tumour suppressor gene called adenomatous
polyposis coli gene (APC)
253. FAST scans can be used to assess the presence of fluid in the abdomen and thorax
254. Ferritin and transferrin saturation are used to monitor treatment in haemochromatosis
255. Flucloxacillin is a well recognised cause of cholestasis
256. Flushing, diarrhoea, bronchospasm, tricuspid stenosis, pellagra → carcinoid with liver mets - diagnosis:
urinary 5-HIAA
257. Gastric adenocarcinoma - signet ring cells
258. Gastric MALT lymphoma - eradicate H. Pylori
259. Gastrin - increases gastric motility
260. Gastrin increases H+ secretion by gastric parietal cells
261. Gastrin increases HCL production and gastrointestinal motility
262. Gastrin is produced by the G cells in the antrum of the stomach
263. Give 50% of normal energy intake in starved patients (> 5 days) to avoid refeeding syndrome
264. GORD is the single strongest risk factor for the development of Barrett's oesophagus
265. H. Pylori eradication:
a. PPI + amoxicillin + clarithromycin, or
b. PPI + metronidazole + clarithromycin
266. Haemochromatosis is autosomal recessive
267. Haemochromatosis is more common than cystic fibrosis
268. HBsAg negative, anti-HBs positive, IgG anti-HBc negative - previous immunisation
269. HbsAg negative, anti-HBs positive, IgG anti-HBc positive - previous infection, not a carrier
270. Positive anti-HBc IgG, negative anti-HBc IgM and negative anti-HBc in the presence of HBsAg implies chronic
HBV infection
271. HBV screening
a. Anti-HBs = (immunity) negative in chronic disease
b. HBsAg = infection
c. Anti-HBc (IgM acute or recent & IgG persists)
d. HbeAg = replication and infectivity (breakdown of core antigen from infected liver cells)
226
M Y Elamin
MBBS, DTM&H, MCTM, MRCPI 1& 2
272. HCV = viral RNA, the antibodies remain even after clearance of infection
273. Hepatocellular carcinoma
a. Hepatitis B most common cause worldwide
b. Hepatitis C most common cause in Europe
274. Hepatorenal syndrome is primarily caused by splanchnic vasodilation
275. High urea levels can indicate an upper GI bleed versus lower GI bleed
276. High-resolution CT scanning is the diagnostic investigation of choice for pancreatic cancer
277. HPV infection is the strongest risk factor for anal cancer
278. Hydrogen breath testing is an appropriate first line test for diagnosis of small bowel overgrowth syndrome
279. Hypophosphataemia is a characteristic biochemical sign in patients at risk of refeeding syndrome
280. Oral vancomycin is the first line antibiotic for use in patients with C. Difficile infection
281. If C. Difficile does not respond to first-line vancomycin , oral fidaxomicin should be used next, except in life-
threatening infections
282. If a first episode of C. Difficile doesn't respond to either vancomycin or fidaxomicin then oral vancomycin +/-
IV metronidazole should be tried
283. In life-threatening C. Difficile infection treatment is with ORAL vancomycin and IV metronidazole
284. If a mild-moderate flare of ulcerative colitis does not respond to topical or oral aminosalicylates then oral
corticosteroids are added
285. In a mild-moderate flare of ulcerative colitis extending past the left-sided colon, oral aminosalicylates should
be added to rectal aminosalicylates, as enemas only reach so far
286. Imaging is required before making a diagnosis of PBC to exclude extrahepatic biliary obstruction
287. In haemochromatosis, cardiomyopathy and skin pigmentation are reversible with treatment
288. In patients with non-alcoholic fatty liver disease, enhanced liver fibrosis (ELF) testing is recommended to aid
diagnosis of liver fibrosis
289. In suspected SBP- diagnosis is by paracentesis. Confirmed by neutrophil count >250 cells/ul
290. Insoluble sources of fibre such as bran and wholemeal should be avoided in IBS
291. Lactulose and rifaximin are used for the secondary prophylaxis of hepatic encephalopathy
292. Liver abscesses are generally managed with a combination of antibiotics & drainage
293. Liver failure following cardiac arrest think ischaemic hepatitis
294. Metabolic ketoacidosis with normal or low glucose: think alcohol
295. NICE recommend avoiding lactulose in the management of IBS
296. Obesity - NICE bariatric referral cut-offs
a. With risk factors (T2DM, BP etc): > 35 kg/m^2
b. No risk factors: > 40 kg/m^2
297. Obesity with abnormal LFTs - ? Non-alcoholic fatty liver disease
298. Oesophageal adenocarcinoma is associated with GORD or Barrett's
227
M Y Elamin
MBBS, DTM&H, MCTM, MRCPI 1& 2
299. Oesophageal/Gastric Cancer - Endoscopic ultrasound (EUS) is better than CT or MRI in assessing mural
invasion
300. Omeprazole can increase your risk of severe diarrhoea (Clostridium difficile infections)
301. Ongoing diarrhoea in Crohn's patient post-resection with normal CRP → cholestyramine
302. Patients must eat gluten for at least 6 weeks before they are tested
303. Patients with ascites (and protein concentration <= 15 g/L) should be given oral ciprofloxacin or norfloxacin
as prophylaxis against spontaneous bacterial peritonitis
304. Patients with ascites secondary to liver cirrhosis should be given an aldosterone antagonist
305. PEG insertion is not normally recommended in advanced dementia patients
306. People with coeliac disease receive the pneumococcal vaccine due to hyposplenism
307. Pernicious anaemia is an autoimmune disease caused by antibodies to intrinsic factor +/- gastric parietal cells
autoimmune destruction of gastroparietal cells
308. Peutz-Jeghers syndrome - autosomal dominant
309. PPis are a cause of microscopic colitis, which can present with chronic diarrhoea, colonoscopy and biopsy should
be considered when patients present in this way and are taking a PPI
310. PPis are a risk factor for C. Difficile infection
311. Primary biliary cholangitis - the M rule
a. IgM
b. Anti-Mitochondrial antibodies, M2 subtype
c. Middle aged females
312. Pyloric stenosis classically leads to hypochloraemic, hypokalaemic alkalosis
313. Percutaneous gastrostomy tube (PEG) is the preferred way to support nutrition in patents with motor neuron
disease
314. Screening for haemochromatosis
a. General population: transferrin saturation > ferritin
b. Family members: HFE genetic testing
315. Secretin - S cells in upper small intestine
316. Secretin increases secretion of bicarbonate-rich fluid from pancreas and hepatic duct cells
317. Sehcat is the investigation of choice for bile acid malabsorption
318. Sjogren's syndrome is common in patients with PBC
319. Somatostatin is produced by D cells in the pancreas & stomach
320. Speed of onset can help to differentiate the type of hepatorenal syndrome, Type 1 HRS is Rapidly progressive,
WHERE Type 2 HRS is Slowly progressive
321. Spontaneous bacterial peritonitis - treatment: intravenous cefotaxime
322. Spontaneous bacterial peritonitis: most common organism found on ascitic fluid culture is E. Coli
323. Sulphasalazine can cause oligospermia and infertility in men
228
M Y Elamin
MBBS, DTM&H, MCTM, MRCPI 1& 2
324. Sulphonylureas may cause cholestasis
325. Surgery is indicated in patients with ongoing acute bleeding despite repeated endoscopic therapy
326. T2DM with abnormal LFTs - ? Non-alcoholic fatty liver disease
327. Terlipressin - method of action = constriction of the splanchnic vessels
328. The 'double duct' sign may be seen in pancreatic cancer
329. The Alvarado score can be used to suggest the likelihood that a patient has acute appendicitis (It consists of eight
different criteria (symptoms, signs and laboratory results) and divides patients into appendicitis unlikely, possible,
probable and definite)
330. The Child-Pugh score assesses prognosis in liver cirrhosis.
331. The Glasgow score assesses severity of acute pancreatitis.
332. The MELD score assesses the severity of end-stage liver disease.
333. The Centor score assesses the likelihood that a pharyngitis is due to Streptococcus
334. The gold standard test for achalasia is oesophageal manometry
335. The oral contraceptive pill is associated with drug-induced cholestasis
336. The splenic flexure is the most commonly affected site in ischaemic colitis
337. Torsades-des-pointes secondary to hypomagnesaemia can result as a consequence of refeeding syndrome
338. TPMT activity should be assessed before offering azathioprine or mercaptopurine therapy in Crohn's disease
339. Transient elastography is now the investigation of choice to detect liver cirrhosis
340. Transjugular Intrahepatic Portosystemic Shunt commonly causes an exacerbation of hepatic encephalopathy
341. Treatment for Wilson's disease is currently penicillamine
342. Ulcerative colitis - depletion of goblet cells
343. Ulcerative colitis - the rectum is the most common site affected
344. Urea breath test - no antibiotics in past 4 weeks, no antisecretory drugs (e.g. PPI) in past 2 weeks
345. Urea breath test is the only test recommended for H. Pylori post-eradication therapy
346. Ursodeoxycholic acid is the first-line medication for primary biliary cholangitis
347. USS is the first line investigation for suspected cholangitis
348. Weight loss is the best first line management for NAFLD
349. While amylase is an important investigation in the diagnosis of pancreatitis, it does not offer prognostic value
350. Whilst dysphagia of solids and liquids is a classic history for achalasia, certain features such as significant weight
loss are not consistent and suggest cancer - 'pseudoachalasia'
351. Whilst hypercalcaemia can cause pancreatitis, hypocalcaemia is an indicator of pancreatitis severity
352. Whipple's disease is a rare multi-system disorder caused by Tropheryma whippelii infection. common in those
who are HLA-B27 positive and in middle-aged men.
353. Whipple's disease: jejunal biopsy shows deposition of macrophages containing Periodic acid-Schiff (PAS)
granules. Treated with oral co-trimoxazole for a year is thought to have the lowest relapse rate, sometimes preceded
by a course of IV penicillin.
229
M Y Elamin
MBBS, DTM&H, MCTM, MRCPI 1& 2
354. Wilson's disease - autosomal recessive
355. Wilson's disease: in the brain, most copper is deposited in the basal ganglia
356. Wilson's disease - serum caeruloplasmin is decreased
357. You cannot interpret TTG level in coeliac disease without looking at the IgA level
358. Zollinger-Ellison syndrome: epigastric pain and diarrhea
359. A blood transfusion is the most appropriate immediate action for patients with a major gastrointestinal
(GI) bleed.
360. A diagnosis of colon cancer, diverticular disease, or colonic inflammation is most readily excluded by
colonoscopy and biopsy and this is initially the most helpful investigation.
361. A false positive is a result that indicates that a given condition is present when it is not.
362. A history of abdominal bloating with intermittent diarrhoea and strong smelling bowel gas is typical of
giardiasis in the returning traveller .
363. A history of abdominal pain in the context of ischaemic heart disease and presence of AF is suspicious
for mesenteric ischaemia
364. A history of harder stools, fresh red blood and intense pain on defecation points to a diagnosis of anal
fissure.
365. A history of lethargy and itching, together with a blood picture consistent with obstructive liver disease
is typical of primary sclerosing cholangitis. It is associated with Ulcerative collitis and patients are at
increased risk of cholangiocarnioma.
366. A history of regurgitation of rotten food, coupled with chronic cough and a gurgling mass on
examination fits best with a pharyngeal pouch.
367. A longstanding history of dysphagia to both solids and liquids suggests a functional rather than
mechanical cause for the dysphagia.
368. A Mallory-Weiss tear can cause haematemesis.
369. A pH of less than 7.3 is a poor prognostic factor for patients who have taken an overdose of
paracetemol.
370. A pharyngeal pouch typically presents with dysphagia and regurgitation, and there may be a palpable
lump in the neck on examination.
371. A positive hydrogen breath test in the context of bloating, abdominal distension and diarrhoea is
diagnostic of bacterial overgrowth syndrome.
372. A recent change to more frequent stools in the last two months is a red flag which in this patient should
be investigated under urgent 2 week wait referal.
373. Abdominal pain is often only mild, or even absent in SBP, with patients often presenting with otherwise
unexplained hepatic decompensation.
374. Absorption of oral iron is improved by ascorbic acid.
230
M Y Elamin
MBBS, DTM&H, MCTM, MRCPI 1& 2
375. Achalasia is caused by dysfunction of the lower oesophageal sphincter, which can be treated with
endoscopic dilatation or surgical myotomy.
376. Achalasia presents often in the third to fifth decades with vague chest discomfort and nocturnal cough.
377. Achlorhydria is classically associated with VIPoma together with profuse diarrhoea, a hypokalaemic
acidosis and hyperglycaemia.
378. Acute amoebic dysentery is managed with a course of oral metronidazole or tinidazole, to be followed
by a ten-day course of diloxanide to eradicate colonisation of the gut.
379. Acute intermittent porphyria (AIP) is a rare disorder characterised by abdominal pain and
neuropsychiatric symptoms which usually presents in the 20-40 age group.
380. Acute intermittent porphyria is an autosomal disorder. Clinical features include abdominal pain and
constipation. Associated neurological manifestations such as polyneuropathy and epilepsy are also seen.
Urinary porphobilinogen is greater than four times the upper limit of normal. Management includes
analgesia – opiates are often required. High carbohydrate intake and in severe attacks haem infusion inhibit
haem production and thereby reduce porphyrin synthesis.
381. Alcohol is a risk factor for the development of acute pancreatitis, which presents with abdominal pain
and raised amylase.
382. Alpha-1-antitrypsin (A1AT) deficiency is an autosomal co-dominant disorder - both alleles contribute to
the phenotype.
383. An ascitic neutrophil count of >250/mm3 is indicative of spontaneous bacterial peritonitis which should
be treated with IV antibiotics.
384. Anticholinergic agents decrease large intestinal motility.
385. Anti-mitochondrial antibodies (AMA) are positive in approximately 95% of patients with PBC.
386. Anti-TTG antibodies are most likely to be present in patients with coeliac disease.
387. Aplastic crisis is a recognised complication of parvovirus B19 infection, particularly in patients with a
history of sickle cell anaemia or hereditary spherocytosis.
388. Approximately half of patients with primary sclerosing cholangitis will have normal examination
findings at the time of diagnosis.
389. Autoimmune hepatitis is often seen in individuals with other autoimmune disorders such as ulcerative
colitis.
390. Azathioprine can be used in pregnancy without significant risk to the fetus.
391. Banding is proven to eliminate oesophageal varices with fewer procedures and complications than
injection sclerotherapy.
392. Barret’s esophagus is treated with PPI as first line therapy.

231
M Y Elamin
MBBS, DTM&H, MCTM, MRCPI 1& 2
393. Barrett's oesophagus is a premalignant condition caused by chronic gastro-oesophageal reflux which
increases ones’ risk of oesophageal adneocacrinoma 50-100 fold.
394. Barrett's oesophagus is a premalignant state and surveillance endoscopies are recommended, with some
guidance suggesting two-yearly endoscopy.
395. Bilirubin conjugation is catalysed by a glucuronyl transferase.
396. Both calcium channel blockers (nifedipine) and nitrates relax the lower oesophageal sphincter.
397. Budesonide is efficacious in the management of microscopic colitis
398. Campylobacter infection is one of the commonest causes of inflammatory diarrhoea with pain often a
prominent feature of the illness frequently localising to the right iliac fossa. Symptoms may last a week or
longer.
399. Carcinoid syndrome is a paraneoplastic syndrome caused by serotonin secretion. It is diagnosed by a 24
hour urine collection testing for 5-HIAA.
400. Cases of non-alcoholic steatohepatitis, the diagnosis of which is made only by histology of liver biopsy
which shows lesions suggestive of ethanol intake in a patient known to consume less than 40 g of alcohol
per week.
401. Cholecystokinin is released by the small bowel and is the major hormone which stimulates contraction
of the gallbladder.
402. Cholera toxin activates adenylate cyclase with generation of cyclic adenosine monophosphate (cAMP).
403. Cholestyramine is an anion exchange resin and will interfere with the absorption of fat-soluble vitamins,
including vitamin D.
404. Cholestyramine may be effective for the treatment of bile acid diarrhoea.
405. Chronic laxative abuse (containing anthraquinones) may cause melanosis coli.
406. Ciprofloxacin is recommended for first-line antibiotic therapy in Escherichia coli when supportive
measures fail or when signs of sepsis are present.
407. Ciprofloxacin is the antibiotic of choice for the treatment of Salmonella - 500 mg bd for 10-14 days.
408. Class II of hypovolaemic shock by blood loss in adults is where there is 750-1500 ml blood loss with
15-30% loss of circulating blood volume.
409. Clostridium difficile diarrhoea is a common complication for frail patients treated with broad-spectrum
antibiotics.
410. Clostridium difficile, a Gram-positive anaerobic bacterium, is the cause of pseudomembranous colitis.
411. Co-amoxiclav (Augmentin) is notorious for causing drug-induced jaundice, often with a mixed
hepatitic/cholestatic picture. A four week delay in symptoms and signs is not unusual.
412. Coeliac disease results from small bowel inflammation and atrophy due to T-cell mediated
hypersensitivity reaction to the alpha-gliadin component of gluten.
232
M Y Elamin
MBBS, DTM&H, MCTM, MRCPI 1& 2
413. Coeliac disease symptoms may be mild and non-specific
414. Coelic disease has characteristic histological features.
415. Colonoscopy is the best screening test for suspected colon carcinoma.
416. Conjugated bilirubin passes into the enterohepatic circulation and the bilirubin which evades this system
is metabolised by bacteria, primarily in the large intestine, to urobilinogen, then stercobilinogen and
eventually oxidised to stercobilin.
417. Considering the potential complications of major abdominal surgery.
418. Contrast-enhanced transthoracic echocardiography is the best test to demonstrate intrapulmonary
vascular dilatation seen in hepatopulmonary syndrome.
419. Criteria for referral for urgent endoscopy include dyspepsia in a patient aged 55 or above with onset of
dyspepsia within one year and persistent symptoms.
420. Crohn's disease commonly presents with diarrhoea, abdominal pain and weight loss. It can affect the
whole gastrointestinal tract, the most common being ileocolitis.
421. Dermatitis herpetiformis is characterised by IgA at the dermo-epidermal junction and associated with
Coeliac disease.
422. Dexamethasone is usually the corticosteroid of choice in the management of liver capsule pain.
423. Diagnostic criteria for Behçet's syndrome include oral aphthous ulcers which are painful, recurrent and
non-scarring.
424. Distal intestinal obstruction syndrome occurs in 10-20% of patients with cystic fibrosis and incidence
increases with age with around 80% of cases presenting for the first time in adults
425. Dysphagia is the most common symptom in patients with achalasia, with the majority suffering from
dysphagia to solids.
426. Dysphagia with bad breath and marked weight loss in an elderly patient may be due to pharyngeal
pouch.
427. Early dumping is usually self-limiting and resolves within seven to 12 weeks.
428. Enterotoxigenic E coli is the commonest cause of traveller's diarrhoea and is usually a self-limiting
condition.
429. Episodic flushing is the clinical hallmark of carcinoid syndrome, occurring in 85% of patients.
430. Faecal elastase is the most appropriate screening test for diarrhoea resulting from pancreatic exocrine
insufficiency.
431. Familial adenomatous polyposis (FAP) is caused by the loss of the APC gene on the long arm of
chromosome 5.
432. Females are more commonly long term carriers than males of Salmonella typhi, as was the case with
Typoid Mary.
233
M Y Elamin
MBBS, DTM&H, MCTM, MRCPI 1& 2
433. Flexible sigmoidoscopy then barium enema should be offered for patients with major co-morbidity with
suspected colorectal carcinoma.
434. Folic acid deficency is associated with alcoholism and presents with macrocytic anaemia.
435. Following a confrimed total excision of adenocarcinoma, patients should continue to be reviewed with
colonoscopy annually for at least two years.
436. Forty percent of Down's syndrome babies have atrioventricular septal defects.
437. Fresh frozen plasma (FFP) contains more dilute clotting factors and therefore produces inferior
correction and should not be used in the management of life-threatening bleeding (unless prothrombin
complex concentrate is not available).
438. Gastrin levels above 1,000 are strongly indicative of a gastrinoma
439. Gastrin release release is stimulated by gastric luminal peptides.
440. Gastrin stimulates gastric motility, growth and acid secretion, and intestinal motility.
441. Giardia classically presents with a few weeks of diarrhoea with an offensive smell.
442. Gilbert's syndrome commonly causes mild isolated hyperbilirubinaemia.
443. Gilbert's syndrome is inherited in an autosomal recessive fashion and results from a defect in the UGT
1A1 promoter.
444. Granuloma formation is not classically seen in liver cirrhosis.
445. H. pylori eradication is indicated in long term healing of gastric and duodenal ulceration, but not reflux
disease.
446. Haemachromatosis can cause articular calcinosis
447. Haemochromatosis is a genetic metabolic disorder which leads to inappropriate intestinal absorption of
iron (9), that is, primary iron overload.
448. HBsAg appears in the serum one to 10 weeks following acute exposure to the hepatitis B virus, prior to
the onset of symptoms or a rise in serum ALT.
449. Hepatitis C infection is strongly associated with mixed essential cryoglobulinaemia which may produce
mesangiocapillary (also known as membranoproliferative) glomerulonephritis.
450. Hepatitis C is closely associated with previous IV drug abuse, and can present with deranged LFTs and
cryoglobulinaemia.
451. Hepatocellular failure is associated with hyperdynamic circulation and systemic vasodilatation with
increased vascular capacitance.
452. Hereditary haemorrhagic telangiectasia is an autosomal dominant condition resulting in telangiectasia of
the nasal and buccal mucosa and gastrointestinal tract, and visceral arteriovenous malformations.

234
M Y Elamin
MBBS, DTM&H, MCTM, MRCPI 1& 2
453. High viral load is associated with poor prognosis in hepatitis B related hepatic decompensation, and
entecavir or tenofovir are therefore indicated in the acute treatment.
454. Histological features of coeliac disease range from mild changes such as increased intraepithelial
lymphocytes to total mucosal atrophy and complete loss of villi.
455. Hpophosphataemia is a key feature of refeeding syndrome.
456. Hypercalcaemia with multiple duodenal ulcers in a young patient shoudl trigger alarm bells for MEN.
457. Hypertension is a common adverse effect of ciclosporin.(8)
458. Hypertriglyceridaemia is defined as a level greater than 1.7 mmol/L. A level above 11.2 mmol/L is
considered a risk factor for acute pancreatitis which is the third most common cause.
459. Hypophosphataemia is often seen with re-feeding syndrome.
460. If transferrin saturation >45% then genotyping should be considered to diagnose haemachromatosis.
461. Ileo-caecal TB is not associated with clubbing and would be very rare in a young Caucasian in the
United Kingdom.
462. In addition to radiographic evidence of colonic distension at least three of the following criteria should
be met: pyrexia above 38.6°C, tachycardia above 120 beats per minute, neutrophilia above 10.5 ×109/L,
and anaemia.
463. In cases of Campylobacter, appropriate fluid replacement and anti-emetics are initially indicated; most
units advocate no antibiotic treatment as the illness is usually self limiting.
464. In elderly patients with iron deficiency anaemia not adequately explained by upper GI endoscopy,
further imaging of the lower GI tract is mandatory.
465. In familial polyposis coli, the increased cancer risk is due to inheritance of a mutated tumour suppressor
gene.
466. In pancreatic cancer a lack of pancreatic enzymes results in increased fats in the small intestine, leading
to sequestration of calcium and malabsorption of calcium as a consequence.
467. In patients presenting with hereditary haemochromatosis (HHC), DNA analysis is an apporopriate
management step.
468. In patients with an insulinoma there is low fasting glucose due to high levels of insulin and C peptide is
elevated.
469. In patients with gastric carcinoma, endoscopic ultrasonography is superior to conventional CT scanning
for local tumour staging.
470. In patients with rheumatoid arthritis, chronic NSAID use can lead to complications such as iron
deficiency anaemia and the superficial ulceration on endoscopy.
471. In severe acute pancreatitis if enteral feeding is not achieved within 48 to 72 hours supplemental
parenteral nutrition should be provided.

235
M Y Elamin
MBBS, DTM&H, MCTM, MRCPI 1& 2
472. In the older age group, investigation of the lower gastrointestinal (GI) tract is vital to exclude a lower GI
malignancy in cases such as unexplained anaemia with bowel signs or symptoms. Colonoscopy would have
the greatest diagnostic yield in most settings.
473. In tylosis there is palmar and plantar keratosis, this is associated with oesophageal cancer.
474. Increased fluid intake restores fluid lost through the digestive tract, and also acts as a dilutional inhibitor
of crystal and stone formation.
475. Individuals with HNPCC should have colonoscopic surveillance every two years from the age of 25.
476. Initial treatment of spontaneous bacterial peritonitis is with broad-spectrum antibiotics such as
cefotaxime. Quinolones may be offered as prophylaxis in patients with cirrhosis and ascites in patients with
an ascitic fluid protein of 15g/L or less until the ascites has resolved. (NICE)
477. Islet B cells produces amylin.
478. IV dextrose administration can exhaust vitamin B reserves. B vitamins must be administered to all
alcoholic patients requiring dextrose.
479. Korsakoff's is associated with short term memory loss with subsequent compensatory confabulation by
the patient.
480. Lactose intolerance is best diagnosed with a methane breath test
481. Lactulose is used in patients with cirrhosis/hepatic encephalopathy to limit the proliferation of ammonia-
forming gut organisms and increase the clearance of protein load in the gut.
482. Laparoscopic fundoplication is the treatment of choice for patients with GORD refractory to, or
intolerant of,proton pump inhibitor therapy.
483. Laxative abuse, where not a function of chronic constipation, is usually closely linked to eating
disorders (bulimia and anorexia) and as such is typically over-represented in young, female patients.
484. Leptospirosis which presents as jaundice, acute kidney injury and bleeding is known as Weil’s Disease.
485. Loss of the terminal ileum frequently leads to bile salt malabsorption and treatment with the bile salt
chelator cholestyramine quickly relieves the problem.
486. Low-grade gastric MALT tumours associated with Helicobacter pylori infection respond in over 80% to
Helicobacter eradication as the primary mode of treatment.
487. Lymphomas restricted to the gastric mucosa usually disappear when H. pylori is eradicated.
488. Mackler's triad (vomiting, chest pain and surgical emphysema) is classical for oesophageal rupture but
absent in almost half the cases. A gastrograffin swallow is the recommended first-line investigation.
489. Macrocytic anaemia should promt checking serum B12 levels (and folate).
490. Macroglossia is a characteristic feature of AL amyloid.
491. Malabsorption of fat-soluble vitamins (A, D, K) is common in primary biliary cirrhosis.

236
M Y Elamin
MBBS, DTM&H, MCTM, MRCPI 1& 2
492. Management of acute bleeding should be with a vasoactive drug (such as somatostatin or terlipressin).
493. Melanosis coli is a benign condition associated with chronic laxative use, most commonly osmotic
laxatives.
494. Metformin can commonly cause gastrointestinal disturbances.
495. Microcytic anaemia should prompt investigation for occult GI malignancies
496. Microcytosis may be absent where there is combined iron and folate deficiency.
497. Mortality in severe acute pancreatitis is approximatley 20%
498. Most patients with Giardia respond to oral metronidazole 250-400 mg tds for five days.
499. NAFLD may be associated with deranged liver function, central obesity, hypertension, dyslipidaemia
and insulin resistance
500. NASH gives a hepatitic picture with echobrightness on USS
501. NASH is associated with increased prevalence of insulin resistance/type 2 diabetes.
502. New-onset weight loss with rectal bleeding and abdominal pain in a 52-year-old man must be assumed
to be colonic carcinoma until proven otherwise.
503. Ninety per cent of the population have normal or high enzyme activity, that is, are homozygous for the
wild-type allele.
504. Non-alcoholic steatohepatitis (NASH) is associated with insulin resistance, hyperlipidaemia and chronic
moderately elevated liver enzymes.
505. Of the options listed PiZZ is the genotype with the lowest level of plasma AAT.
506. OGD is indicated in patients with indigestion and weight loss, anaemia, or vomiting; and in the over 55s
with continuous dyspepsia of <1y duration .
507. Omeprazole is a liver enzyme inhibitor.
508. Oral and genital ulceration, colitis and scleritis are a classical descripton of Behçet's disease
509. Pain is the most common presenting feature of small bowel lymphoma with the ileum the site most
commonly affected.
510. Pancolitis would give a high cause for concern with respect to risk of colonic malignancy.
511. Pancreatitis is a rare adverse effect of azathioprine.
512. Parenteral iron replacement should be considered where iron replacement is necessary but an oral
preparation cannot be tolerated or absorbed
513. Paroxysms of pain, accompanied by episodes of bleeding are typical of anal fissure.
514. Patient has cystic fibrosis which has a disease prevalence of 1:3000 northern Europeans and gene
carriage 1:25.
237
M Y Elamin
MBBS, DTM&H, MCTM, MRCPI 1& 2
515. Patients who are asymptomatic with gallstones reuire no treatment.
516. Patients who fulfil the diagnostic criteria for irritable bowel syndrome should be screened for other
conditions with tissue transglutaminase (TTG) antibodies.
517. Patients with cirrhosis are frequently hyponatraemic. This is a function of an inability to excrete free
water.
518. Patients with recurrent vomiting may have hypochloraemic metabolic alkalosis due to persistent
vomiting of gastric hydrochloric acid and the metabolic disturbance becomes more severe with worsening
dehydration.
519. PBC usually occurs in middle aged women, and commonly presents with lethargy and pruritis before
prominent features of liver failure occur.
520. Peptic ulceration is the commonest cause of acute upper gastrointestinal (GI) haemorrhage.
521. Peutz-Jegher syndrome is inherited in an autosomal dominant fashion and may predispose individuals to
malignancy
522. Peutz-Jeghers syndrome is caused by a mutation of the STK11/LKB1 gene.
523. Phenobarbitone is a liver enzyme inducer.
524. Phenytoin can lead to gingival hyperplasia.
525. Plain AXR is useful for diagnosing toxic dilatation and would be the investigation of choice in patients
presenting with abdominal distension.
526. Plasma immunoelectrophoresis to look for an M band is the most appropriate next investigation
following blood tests in the diagnosis of myeloma.
527. Poor glycaemic control increases risk of both micro and macrovascular complications, as seen in
UKPDS and DCCT.
528. Primary biliary cirrhosis (PBC) is associated with HLA-DR8.
529. Progressive dysphagia to both solids and liquids in young adults should prompt search for achalasia.
530. Prolonged laxative use can result in melanosis coli.
531. Prostaglandins inhibit gastric acid secretion.
532. Protein, peptides, and amino acids are specific components which will stimulate gastrin release.
533. Pseudopolyps are seen in both ulcerative colitis and Crohn’s disease.
534. Refeeding syndrome can cause potentially fatal fluid and electrolyte imbalances.
535. Refeeding syndrome is associated with hypophosphataemia.
536. Relief with food suggests peptic, and specifically, duodenal ulceration.

238
M Y Elamin
MBBS, DTM&H, MCTM, MRCPI 1& 2
537. Resection of a significant portion of small intestine can result in malabsorption of fluid and nutrients,
presenting with profuse watery diarrhoea in the days following surgery.
538. Scombrotoxin food poisoning is caused by the ingestion of foods that contain high levels of histamine;
linked to spoilage of foods , particularly tuna or mahi-mahi.
539. Secretin is secreted from the small intestine when there is acid in the small intestine. It inhibits gastric
motility and acid production and stimulates bicarbonate secretion from the pancreas and liver.
540. Serologically, primary biliary cirrhosis PBC is characterised by antimitochondrial antibodies, which are
present in 90-95% of patients (often before clinical signs develop) and have a specificity of 98%.
541. Severe exacerbation of UC needs intravenous corticosteroids with or without cyclosporine.
542. Shigella is a cause of profuse bloody diarrhoea.
543. Smoking has been shown to be associated with earlier age of onset of disease among women with
Crohn’s disease but not men.
544. Somatostatin is released from delta cells in the pancreas and is a strong inhibitor of insulin and glucagon
secretion.
545. Spontaneous bacterial peritonitis (SBP) is diagnosed if the ascitic fluid neutrophil count is = 250
cells/mm3.
546. Spontaneous bacterial peritonitis is diagnosed by ascitic fluid examination which reveals a neutrophil
count of >250/ml, and is typically caused by aerobic gram negative bacteria.
547. Spontaneous bacterial peritonitis should be managed aggressively.
548. Staphylococci are skin organisms most commonly introduced during pacemaker insertion.
549. Subtotal villous atrophy is seen in a variety of conditions including Whipple's disease.
550. Symptoms of GORD do not correlate with the mucosal appearances at endoscopy.
551. Symptoms of Whipple's disease include chronic diarrhoea, weight loss, arthralgia and abdominal pain.
552. Tender hepatomegaly is one of the hallmarks of BCS.
553. Tender hepatosplenomegaly and fever are consistent with a diagnosis of alcoholic hepatitis, which
frequently occurs on a background of cirrhosis.
554. The anterior relations of the left kidney are the adrenal gland, the spleen, the stomach, pancreas,
jejunum, and colon.
555. The British Society of Gastroenterology guidelines suggest that where the serum sodium is =120
mmol/L diuretic therapy should be stopped and patients should receive volume expansion with colloid or
normal saline.
556. The final common pathway of hepatic fibrosis is mediated by the hepatic stellate cell in the space of
Disse.

239
M Y Elamin
MBBS, DTM&H, MCTM, MRCPI 1& 2
557. The first step in analysis of a spherocytic hemolytic anaemia is to determine whether the process is
hemolytic or not. The best way to do it is a direct antiglobulin test.
558. The gene involved in Wilson's disease is located on chromosome 13.
559. The incidence of patients with haemolytic disorders such as sickle cell disease and hereditary
spherocytosis forming pigment gallstones is approximately 50% due to an increase in bilirubin excretion.
560. The King's College Criteria are the most widely accepted prognostic tool for patients who present with
acute liver failure secondary to paracetamol overdose, and include arterial pH, INR, creatinine and
encephalopathy.
561. The most useful tumour marker for pancreatic cancer is CA 19-9, of which the sensitivity and specificity
for pancreatic carcinoma are 80% and 90% respectively.
562. The next step in in the managment of Chron's when steroids fail is a trial of azathioprine which is used
as a steroid-sparing agent.
563. The presence of skipping and full thickness inflammation with granuloma formation is typical of
Crohn's disease.
564. The presence of thrombocytopenia and evidence of haemolysis in association with bloody diarrhoea
should make you think of haemolytic uraemic syndrome (HUS).
565. The prevalence of coeliac disease in Europe varies widely and is in the region of between 1:100 and
1:300. It is more common in the Celtic population.
566. The Ranson criteria is a useful system for predicting the severity of acute pancreatitis.
567. The right and left gastroepiploic arteries supply the greater curvature of the stomach.
568. The right gastric artery arises from the hepatic artery or the left hepatic artery supplies the pylorus and
travels along the lesser curvature of the stomach, supplying it, and anastomosing with the left gastric artery.
569. The Rockall scoring system is based on: Age (the higher the age the worse the prognosis);
Comorbidities, e.g. ischaemic heart disease (IHD); Presence of shock; and Endoscopic abnormalities.
570. The sicca syndrome (xerostomia/dry eyes, keratoconjunctivitis sicca) may occur in cases of autoimmune
hepatitis.
571. The treatment of choice for large, symptomatic ascites is large volume therapeutic paracentesis.
572. The two main conditions causing pigmentation and chronic liver disease are primary biliary cirrhosis
(PBC) and haemochromatosis.
573. The urease breath test is the most specific investigation detecting current infection with Helicobacter
pylori.
574. The vomiting-type outbreaks of Bacillus cereus food poisoning have generally been associated with rice
products.
575. Tissue transglutaminase generates the antigenic epitopes present in alpha-gliadin.

240
M Y Elamin
MBBS, DTM&H, MCTM, MRCPI 1& 2
576. Transferrin physiology
577. Type 2 diabetics have a 40-60% increased risk of colon cancer.
578. Typically broad spectrum antibiotics are administered with a consequent risk of pseudomembranous
colitis.
579. Ulcerative colitis is characterised by crypt abscess formation and lymphocytic infiltration of the lamina
propria on colonic biopsy.
580. Ulcerative colitis is characterised by mucosal inflammation. In contrast, Crohn's disease is characterised
by transmural inflammation.
581. Ulcerative colitis only affects the bowel starting from the rectum, and may go as far as the terminal
ileum.
582. Ultrasound scan would confirm most moderate-sized to large liver abscesses and could guide a
diagnostic aspiration.
583. Up to half of patients presenting with Streptococcus bovis endocarditis have colorectal tumours.
584. Urea breath test is used to determine eradication after treatment for H. pylori.
585. Urea breath test used to confirm H. pylori eradication as it is non-invasive and has a high sensitivity (95-
98%) and specificity (98%).
586. VIPoma is associated with elevated urea and calcium with hypokalaemic acidosis.
587. VIPomas are endocrine tumours that secrete excessive amounts of VIP 32 which cause a distinct
syndrome characterised by large-volume watery diarrhoea, hypokalaemia, and dehydration.
588. Vitamin C deficiency, or scurvy, results in perifollicular haemorrhages and bleeding gums.
589. Vomiting of food from several meals ago suggests gastric stasis or gastric outflow obstruction.
590. Warfarin does not undergo extensive hepatic first-pass metabolism.
591. Whipple's disease is caused by Tropheryma whippleii.
592. Whipple's disease is rare and most commonly affects middle-aged males. It can affect any organ but is
dominated by involvement of small bowel, causing malabsorption.
593. Zinc deficiency can lead to acrodermatitis which presents with perioral dermatitis, acral involvement,
and sometimes alopecia and is common in patients following bowel resection due to lack of absorption
from the small bowel.A CT scan of the abdomen will show morphological abnormalities in the pancreas in
chronic pancreatitis, such as calcification.
594. A detailed knowledge of the causes of pancreatitis, and the indicators of each cause, are important for
MRCP and for clinical practice.
595. A normal Full blood count, erythrocyte sedimentation rate, C reactive protein and anti-tissue
transglutaminase antibodies make the diagnosis of IBS more likely.

241
M Y Elamin
MBBS, DTM&H, MCTM, MRCPI 1& 2
596. A rapid urease (for example, CLO) test is negative this does not exclude Helicobacter pylori infection
especially in the context of multiple ulcer on GI endoscopy and should prompt a blood antigen test.
597. A triad of fever, jaundice and RUQ pain points to cholangitis and prompt antibiotics should be started as
well as imaging to rule out an abscess or empyema.
598. ABCDE assessment if first line in all patients with reduced conscious level.
599. Abdominal pain, weight loss and jaundice should prompt investigations to rule out malignancy.
600. abdominal ultrasound is the first line investigation for obstructive jaundice.
601. Achalasia causes intermittent dysphagia, non-acid reflux and chest pain due to failure of the lower
oesophageal sphincter to relax and aperistalsis in the oesophageal body.
602. Achalasia typically causes absence of peristalsis in the body of the oesophagus whereas oesophageal
spasm produces repetative, high amplitude contractions on manometry.
603. Actinomycosis is caused by a filamentous, Gram positive bacterium and treatment is with
benzylpenicilin or surgical drainage.
604. Acute hepatitis C infection is asymptomatic in 60-70% of cases, infection is chronic in around 85% of
individuals.
605. Alcoholic liver disease typically causes a mildly raised ALT with an AST more than double of the raised
ALT.
606. Although hepatotoxic in high doses even in fairly advanced chronic liver disease paracetamol can be
used safely as long as doses do not exceed 2-3 g per day except in alcoholic liver disease.
607. Although not licensed in the UK, tricyclic antidepressants are a recognised second line agent in irritable
bowel syndrome.
608. An epigastric mass which is causing discomfort and dyspepsia should prompt urgent investigation.
609. An episode of spontaneous bacterial peritonitis carries a two year mortality rate of 50%.
610. An upper gastrointestinal endoscopy would be the first line investigation of choice however once
performed and negative, a full examination of the lower gastrointestinal tract would be most appropriate.
611. Anal fistulae cause chronic discharge and have an internal and external opening.
612. Anticentromere antibodies are strongly associated with CREST syndrome.
613. Anti-mitochondiral antibodies are positive in 95% of cases of primary biliary cirrhosis.
614. Antimitochondrial antibodies are the hallmark of primary biliary cirrhosis.
615. Antiviral treatment would not be indicated unless biopsy suggested significant disease due to HBV.
616. Aortic-enteric Fistula formation often occurs in the setting of infection of prosthetic material, such as
endovascular repair and results in massive gastrointestinal haemorrhage.
617. Arterial blood gas is important to guide immediate management in acute pancreatitis
242
M Y Elamin
MBBS, DTM&H, MCTM, MRCPI 1& 2
618. As well as the Kings College initial criteria for referral for liver transplant any degree of encephalopathy
48 hours after ingestion is also an indication.
619. Ascitic fluid analysis demonstrates a low serum albumin ascites gradient (SAAG) - that is, <11 g/L in
acute pancreatitis
620. Bacillus cereus infection present with profuse vomiting which occurs approximately one to five hours
after eating recooked rice.
621. Barium follow through will show loops of small bowel narrowing and stricture in Chron's disease,
especially in the terminal ileum.
622. Bechet's disease is associated with painful oral ulceration, painful genital ulceration and uveitis.
623. Bloods indicate a macrocytic anaemia and isolated B12 deficiency with autoimmune disease should
promt investigation for pernicious anaemia.
624. Bloody ascites with a significantly elevated neutrophilic white cell count should prompt CT abdomen
and urgent surgical review for perforated bowel.
625. Bloody ascitic fluid with a Gram stain revealing Gram negative bacilli and Gram positive cocci is
suggestive of bowel perforation.
626. British Society of Gastroenterology guidelines on the investigation and management of iron deficiency
anaemia advocate that following a negative coeliac screen, and in the absence of upper gastrointestinal
symptoms or a significant family history of bowel cancer, no further gastrointestinal investigation is
warranted and a trial of iron replacement should be instituted.
627. Budd-Chiari syndrome management varies depending on whether the disease process is acute or
chronic.
628. Budd-Chiari syndrome presents classically with the triad of abdominal pain, ascites and hepatomegaly.
It is caused by occlusion of the hepatic veins.
629. Candida oesophagitis requires an extended course of oral or IV therapy as opposed to oropharyngeal
candidiasis.
630. Carcinoid heart disease has a poor prognosis
631. Carcinoid syndrome is caused by excessive levels of serotonin secreted from a neuroendocrine tumour
arising from enterochromaffin cells in the gut or less frequently the lungs.
632. Causative organism in Chagas' disease
633. Change in bowel habit, weight loss and rectal bleeding should prompt investigation for colon cancer.
634. Cholangiocarcinoma appears hypovascular on CT imaging.
635. Choledochal cysts are an uncommon cause of biliary sepsis in adult life and are classified by the Todani
system.
636. Ciclosporin can be used to treat severe eczema in cases where conventional therapy has failed.

243
M Y Elamin
MBBS, DTM&H, MCTM, MRCPI 1& 2
637. Classification systems exist for polyp cancers; Kicuchi for sessile polyps and Haggitt for pedunculated
polyps.
638. Clinical signs of haemochromatosis include hepatomegaly, cirrhosis and splenomegaly as well as the
development of diabetes and hyperpigmentation of the skin.
639. Clopidogrel is associated with a much higher risk of bleeding than aspirin - stop 7 days prior to ERCP.
640. Cobblestone gastric mucosa is a clinical manifestation of Crohn's disease.
641. Coeliac disease is caused by a T cell mediated hypersensitivity reaction to gluten which causes intestinal
inflammation and atrophy.
642. Coeliac may present with iron deficiency or B12/folate deficiency
643. Colonoscopy is the diagnostic investigation of choice in inflammatory bowel disease.
644. Consider food impaction as a cause of acute dysphagia and vomiting with those with pre-existing stents
and acute symptoms.
645. Consider small bowel obstruction with distended small bowel loops on abdominal film, paucity of large
bowel gas and abdominal pain.
646. Constipation with intermittent rectal bleeding with no other worrying signs or symptoms is most likely
haemorrhoids or an anal fissure.
647. Cytomegalovirus may be a cause of a colitis flare and should be suspected in patients with steroid
refractory disease. The presence of multiple intranuclear inclusion bodies on colonic biopsy is highly
indicative.
648. Delirium tremens is the most severe form of alcohol withdrawal and should be treated with
benzodiazpines.
649. Delirium tremens presents three to seven days after cessation of chronic alcohol ingestion and is
characterised by visual hallucinations, autonomic instability and confusion.
650. Diagnosis of acute pancreatitis
651. Diarrhoea in the returning traveller is a common presentation. It is important to recognise symptoms of
the most frequent culprits. Giardiasis presents with abdominal symptoms as listed above.
652. Duodenal ulcers produce epigastic pain and dyspepsia which is better after eating.
653. Dyspepsia due to medications is cause by This is either irritation of the mucosaor relaxation of the lower
oesophageal sphincter increasing reflux.
654. Elevation of transaminases to more than 100 times the upper limit of normal generally occurs in only
two conditions - ischaemic hepatitis and paracetamol overdose.
655. Endometeriosis may cause tethering of the colon and lead to cyclical rectal bleeding.
656. Eradication therapy in H Pylori is indicated in the context of peptic ulcer disease as it increases healing
rates and reduces recurrence
244
M Y Elamin
MBBS, DTM&H, MCTM, MRCPI 1& 2
657. ERCP is diagnostic and demonstrates irregular stricturing and dilatation (beading) of the intrahepatic
ducts in primary sclerosing cholangitis.
658. Even in high suggestive history of IBD it is important to do a stool culture in a case of diarrhoea before
considering the other investigations.
659. Extremely high levels of AFP in cirrhotic liver should prompt investigation for malignancy.
660. Female sex, age less than 60 and a low probability of structural disease all increase the risk of post-
ERCP pancreatitis.
661. Flumenazil is used in cases of benzodiazepine toxicity.
662. Giardia lamblia is a common cause of diarrhoea even in the United Kingdom and symptoms include
bloating, abdominal pain and diarrhoea.
663. GISTs originate from the interstitial cells of Cajal and are most common in the stomach.
664. GISTs usually stain positive for CD117 (c-KIT).
665. Goodsall's rule states that anterior fistulas open internally at the same clock face as their external
opening while posterior fistulas open at 6 o'clock.
666. Gorlin syndrome causes gastric hamartomas, basal cell carcinomas, mandibular bone cysts, and
intracranial calcification as well as pits on the palms and soles.
667. H.pylori stool antigen testing needs to be delayed for 3 months after treatment to confirm eradication.
668. Haemetemesis in a young patient with epigastric pain in a young patient with a history of excessive
alcohol intake will require OGD to investigate for ulcers or varices.
669. Haemochromatosis has two mutations of the HFE gene (C282Y and H63D), which account for over
90% of cases in Europeans.
670. Haemorrhoid grading guides treatment. It is based on prolapse and whether or not this returns
spontaneously, requires manual reduction, or is permanent.
671. Haemorrhoids are the most common cause of painless rectal bleeding
672. Helicobacter eradication therapy (antibiotics and proton pump inhibition) is effective in producing
disease regression in around 75% of cases of low-grade GALT lymphoma or gastric MALToma associated
with infection
673. Hepatitis C genotypes 2 and 3 have a far better response to interferon treatment than genotype 1.
674. Hepatitis E infection does not result in a carrier state.
675. History UC is a risk factor for PSC.
676. Hydatid infection was previously endemic in sheep farming regions. Asymptomatic, calcified cystic
lesions in the liver are typical of hydatid cysts.

245
M Y Elamin
MBBS, DTM&H, MCTM, MRCPI 1& 2
677. Hypomagnasaemia should be correct first to reduce the risk of developing and worsening toxic dilatation
in exacerbations of ulcerative colitis
678. Identification of dilated oesophagus on CT
679. If there is a delay in surgery toxic dilatation of the colon carries the risk of perforation which has a high
mortality.
680. In any patient with a history of chronic ascites presenting with abdominal pain, spontaneous bacterial
peritonitis should be considered as a differential.
681. In bleeding duodenal ulcers the initial step in the management is adrenaline injection and heater probe
application when found at OGD.
682. In Budd-Chiari syndrome hepatomegaly is present due to venous congestion of the liver and the
transaminases are usually only mildly deranged.
683. In dyspepsia resistant to a double dose of proton pump inhibition an H2 receptor antagonist may be
added.
684. In high risk patients or those with multiple co-morbidities flexible sigmoidoscopy is safer for bowel
investigation than colonoscopy.
685. In low risk areas a combination of HBsAg positivity and features of acute hepatitis usually indicates
acute self-limiting hepatitis B infection.
686. In massive upper GI bleeds with signs of shock an ABC approach should be adopted alongside specialist
urgent referral for endoscopy.
687. In patients with mild gallstone pancreatitis, in the absence of cholangitis, there is no evidence to support
ERCP and stone extraction in the acute setting.
688. Increasing Rockall scores are strongly correlated with increasing risk of mortality and should prompt
urgent intervention.
689. Intractable ascites is an indication for a transjugular intrahepatic portosystemic shunt
690. Intravenous corticosteroid therapy is recommended in all cases of severe ulcerative colitis flare whereas
antibiotics aren't always recommended.
691. Iron chelation with desferrioxamine is sometimes required when venesection is not tolerated in severe
case of haemochromatosis.
692. Isolated anti-HBs positive suggests immunity to HBV secondary to vaccination.
693. Large volume diarrhoea is a feature of an acute graft versus host reaction whereas symptoms after 80
days is sign of chronic graft versus host disease.
694. Laxative or purgative abuse is most commonly seen in young female patients complaining of chronic
diarrhoea with hypokalaemia.
695. Lesions below the dentate line have lymphatic drainage via the inguinal nodes.

246
M Y Elamin
MBBS, DTM&H, MCTM, MRCPI 1& 2
696. Liver abscesses can be a complication of gallstones.
697. Liver biopsy is diagnostic of haemochromatosis and is the gold standard for diagnosis. In patients
homozygous for the HFE gene, a liver biopsy is indicated where there is abnormal liver biochemistry or
ferritin > 1000 microg/l. All liver biopsies should be stained for iron (Perls' stain).
698. Liver biopsy remains the gold standard diagnostic tool since it allows an estimation of the degree of iron
loading in haemochromatosis.
699. Liver transplantation is very effective in PBC and has a five year survival of greater than 80%.
700. Loperamide would be contraindicated in the clinical setting of c diff diarrhoea due to the risk of
precipitating toxic megacolon;
701. Magnetic resonance imaging is highly effective and sensitive in confirming the presence of common bile
duct stones
702. Manifestations and management of hereditary haemorrhagic telangectasia
703. Melanosis coli is found in patients who use and abuse anthraquinone laxatives.
704. Mercaptopurine is also inactivated by the enzyme xanthine oxidase. This enzyme is involved in the
production of uric acid and is inhibited by allopurinol.
705. MRCP easily identifies any gallstones and also the best tool for studying the distal CBD and its insertion
into duodenum.
706. Note characteristic appearances of duodenal ulcer with evidence of recent haemorrhage and a visible
vessel.
707. Oedematous looking sigmoid colon and areas of sparing on endoscopy are suggestive of ulcerative
colitis.
708. Oesophageal rupture presents with sever pain radiating to the back following eating with equal blood
pressures and normal pulses.
709. Once H.pylori has been eradicated and the patient is asymptomatic, no further confirmatory tests or
treatment is necessary unless there are any new red flag symptoms.
710. Pancrelipase are a collection of pancreatic enzymes used in treatment of chronic pancreatitis
711. Parenteral iron replacement should be considered where iron replacement is necessary but an oral
preparation cannot be tolerated or absorbed
712. Patients should stop their PPI therapy 14 days prior to urea breath testing for accurate results.
713. Patients who have taken staggered paracetamol overdoses should be started on N-acetylcysteine therapy
empirically whilst the extent and course of liver damage is evaluated.
714. Patients with alcoholic liver disease and ascites on diuretics should stop therapy when the serum sodium
is =120 mmol/L and should receive volume expansion with colloid or normal saline.

247
M Y Elamin
MBBS, DTM&H, MCTM, MRCPI 1& 2
715. Patients with alcoholic liver disease are often surprisingly sensitive to opiate analgesia which should
only be used with caution.
716. Patients with Barrett's oesophagus with high grade dysplasia should be discussed in an MDT for
consideration of more aggressive treatment endoscopically (for example, EMR) or even surgically.
717. Patients with Barrett's oesophagus with no signs of dysplasia require PPI only and follow up in 2 years.
718. Patients with Barrett's require biopsy to rule out dysplasia or malignancy.
719. Patients with Chrons disease are at risk of developing calcium oxalate stones when they have had ileal
resection.
720. Patients with elevated bilirubin and LDH who are from high risk areas for sickle cell disease should be
investigated with a blood film.
721. Patients with gastric ulceration tend to suffer from anorexia and weight loss while those with a duodenal
ulcer maintain or gain weight.
722. Patients with GORD should be started on omeprazole 20mg once a day for a month and then reviewed
to see if symptoms have improved.
723. patients with marked microcytic anaemia (<120 g/L in men and <100 g/L in women) should be referred
urgently for investigation of GI malignancy.
724. Patients with non-alcoholic fatty liver disease have raised ALT, AST and GGT where the ALT is higher
than the AST, which is the opposite of alcoholic liver disease.
725. Patients with PBC who are asymptomatic should be treated with ursodeoxycholic acid to improve liver
function.
726. Patients with Peutz-Jeghers syndrome require colonoscopy every two years after the age of 25 for
evaluation of the presence of polyps and polypectomy.
727. PBC is associated with non-specific symptoms such as fatigue and pruritis and is commonly associated
with autoimmune disorders
728. PBC is associated with other autoimmune disorders and increases ones risk of hepatocellular carcinoma.
729. Perinatal transmission is the most common route of hepatitis B infection.
730. Porphyria cutanea tarda (PCT) is associated with reduced hepatic uroporphyrinogen decarboxylase
activity.
731. Portal hypertension, haematuria and travel history from an at risk area this should raise the possibility of
schistosomiasis. Findings of peri-portal fibrosis and granulomata (related to schistosomal eggs) are
characteristic.
732. PPI's and H2 antagonists have not been researched enough in pregnancy and children, therefore the
safest option is antacid therapy.

248
M Y Elamin
MBBS, DTM&H, MCTM, MRCPI 1& 2
733. Preceding history of abdominal pain exacerbated by eating is suggestive of mesenteric ischaemia and in
a patient with significant risk factor should prompt investigation for ischamic bowel.
734. Presenation and autoantibodies in Addison's disease
735. Previous adenomas or elevated IGF-1 are high risk features for acromegalics developing colorectal
cancer.
736. Primary sclerosing cholangitis (PSC) has a strong association with ulcerative colitis and there is an
increased risk of cholangiocarcinoma.
737. Primary sclerosing cholangitis (PSC) is characterised by biliary stricturing and dilatation and associated
with inflammatory bowel disease.
738. Progressive worsening dysphagia with weight loss is suggestive of a stricture caused by malignancy.
739. Propranolol is used in the primary prophylaxis of variceal haemorrhage.
740. Pseudomonas aeruginosa is characterised by green coloration of the colonies.
741. Psueudomembranous colitis is a potential complication of clostridium difficile infection
742. Psychological factors that may impair ability to comply with immunosuppressive treatment are a
contraindication to liver transplantation and patients should be discussed thoroughly at MDT meetings prior
to referal.
743. Recent stricture places colitics in a high risk category for development of colorectal cancer.
744. Recurrent vomiting and haematemesis are red flag signs for urgent OGD.
745. Refeeding syndrome occurs when malnourished individuals are given nutritional support and leads to
electrolyte abnormalities.
746. Refeeding syndrome occurs when malnourished individuals are given nutritional support and requires
electrolyte replacement.
747. Rockall scores are used for known upper GI bleed patients with a completed endoscopy. Low risk
patients can be managed in community.
748. Rome III criteria describe functional GI diagnoses.
749. Sacroilitis is certainly associated with ulcerative colitis but the degree of sacroilitis is not directly related
to the underlying inflammatory bowel disease.
750. Salt poor albumin should be used in replacement of fluid following ascetic drainage in cirrhosis to
prevent retention of salt and fluid.
751. Serological testing for Coeliac's disease should be performed in cases of malabsorption prior to
endoscopy.
752. Several large randomised, controlled trials have shown that repeated large volume paracentesis (4-6 L)
is safer and more effective for the treatment of tense ascites compared with larger than usual doses of
diuretics.
249
M Y Elamin
MBBS, DTM&H, MCTM, MRCPI 1& 2
753. Sigmoid volvulus is differentiated from a caecal volvulus by its ahaustral wall and the lower end
pointing to the pelvis. Note appearance of the coffee bean sign.
754. Significant weight loss with dyspepsia in someone over the age of 60 should trigger 2 week wait referral
for endoscopy.
755. Sjögren's syndrome is an extrahepatic association of hepatitis C infection.
756. Skin lesions associated with gastrointetinal disease
757. Somatostatin analogues are the treatment of choice for carcinoid symptoms
758. Steatorrhoea and flatulence are classic presenting features of small bowel bacterial overgrowth.
759. Stricturing and beading of the bile ducts on ERCP are features classically seen in PSC.
760. Studies looking at dietary restriction followed by reintroduction suggest food intolerance in 30-60% of
patients with irritable bowel syndrome (IBS).
761. Surgical management should be considered where endoscopic intervention has failed
762. Surgical resection is the treatment of choice for localised pancreatic cancer and offers the possibility of
cure in up to 25% of patients.
763. Symptomology and exacerbating factors of irritable bowel syndrome are hugely variable from patient to
patient.
764. Tc-99m pertechnetate accumulates in gastric mucosa and is the study of choice for identifying ectopic
gastric mucosa in a Meckel's diverticulum.
765. Thankfully, late presenting paracetamol OD is rare, but it is important to be aware of the criteria for
specialist referral to the local liver unit.
766. The Bismuth-Corlette classification is used to describe the morphology of cholangiocarcinomas.
767. The combination of history and clinical findings are suggestive of malignancy as the likely cause
768. The inferior mesenteric artery (IMA) supplies the colon from the distal transverse colon to the upper
third of the rectum.
769. The key feature of increasing encephalopathy is depression of the conscious level.
770. The majority of cholangiocarcinomas are sporadic.
771. The management of post-ischaemic event renal failure is to treat the underlying cause and organ support.
772. The most common causes of unconjugated hyperbilirubinemia are bilirubin overproduction, Gilbert
syndrome, and neonatal jaundice.
773. The national bowel cancer screening programme is open to men and women between the ages of 60 and
74.
774. The passage of faeces resulting in relief of symptoms can support a diagnosis of irritable bowel
syndrome and is part of the Rome III criteria.
250
M Y Elamin
MBBS, DTM&H, MCTM, MRCPI 1& 2
775. The pattern of inheritance of Peutz-Jeghers syndrome is autosomal dominant.
776. The serum ascites albumin gradient (SAAG) is considered the most sensitive and specific method of
categorising ascites.
777. The serum ferritin is the most powerful marker of iron deficiency in the absence of inflammation, should
there be inflammation transferrin saturation should be used.
778. The test most likely to be diagnostic for the cause of ascitic fluid findings is laparoscopic peritoneal
biopsy and culture.
779. The treatment of choice for large, symptomatic ascites is large volume therapeutic paracentesis.
780. Theophylline lowers the pressure of the lower oesophageal sphincter
781. There is little evidence for the role of protein restriction in hepatic encephalopathy which may in fact
worsen the condition.
782. This lady has a personal history of autoimmune type disease. The history describes a peripheral
neuropathy. Bloods indicate a macrocytic anaemia and isolated B12 deficiency (folate is normal)
suggesting an isolated problem with B12 absorption. This makes pernicious anaemia a likely diagnosis.
Anti-intrinsic factor antibodies are diagnostic of the condition though may be absent in up to 50% of
patients with the condition.
783. Threadworm infection is common in children and institutions
784. Ulcerative colitis has two age peaks of incidence.
785. Upper GI endoscopy is frist line investigation for suspected cases of oesophageal malignancy with a
98% detection rate.
786. Urea breath test has 90% sensitivity and 96% specificity for H. pylori.
787. Ursodeoxycholic acid is used in the management of primary biliary cirrhosis and can help prevent or
delay liver damage.
788. Variceal hemorrhage is common in patients with alcoholic liver disease and portal hypertension.
789. Venous thromboembolism more than three months ago is considered a low risk clinical scenario for
temporary discontinuation of anticoagulation.
790. Vomiting is a recognised cause of metabolic alkalosis.
791. Wernicke's encephalopathy is a result of thiamine deficiency and should be treated with high dose
intravenous vitamin B
792. Wernicke's encephalopathy is a result of thiamine deficiency presenting with encephalopathy, ataxia and
oculomotor dysfunction (usually nystagmus).
793. Whipple's disease is caused by Tropheryma whippelei and should suspected in patients with
malabsorption, abdominal pain and cerebellar signs in patients who have previously spent time in high risk
areas.

251
M Y Elamin
MBBS, DTM&H, MCTM, MRCPI 1& 2
794. Wilson's disease typically causes reduced caeruloplasmin, raised urinary copper and raised free serum
copper.
795. Within the small bowel, the most common site of origin for carcinoid tumours is the distal ileum.

252
M Y Elamin
MBBS, DTM&H, MCTM, MRCPI 1& 2
GERIATRIC MEDICINE
1. A 55-year-old is investigated for a gradual change in personality. He has recently become more socially
inappropriate and aggressive. His wife reports that his memory seems intact - frontotemporal dementia
2. A 70-year-old man is noted to have progressive loss of short term memory associated with visual hallucinations.
On examination he is noted to have a resting tremor - Lewy body dementia
3. A 75-year-old man with a history of hypertension presents with memory problems. His wife reports that he has had
a number of sudden deteriorations over the past 2 years but then seems to stay the same - vascular dementia
4. Alzheimer's disease - cortical plaques due to deposition of type A-Beta-amyloid protein and intraneuronal
neurofibrillary tangles caused by abnormal aggregation of the tau protein
5. Cognitive impairment, parkinsonism, visual hallucinations - Lewy body dementia
6. Donepezil - acetylcholinesterase inhibitor
7. Donepezil , uses include: Alzheimer's disease
8. Frontotemporal dementia - spherical aggregations of tau protein
9. Galantamine - acetylcholinesterase inhibitor
10. Lewy body dementia - alpha-synuclein cytoplasmic inclusions in the substantia nigra, paralimbic and neocortical
areas
11. Memantine - NMDA receptor antagonist
12. Memantine , uses include: Alzheimer's disease
13. Rivastigmine - acetylcholinesterase inhibitor
14. Silver staining spherical aggregations of tau protein - Pick's disease
15. In acute psychosis associated with dementia NICE recommends using olanzapine, lorazepam, or haloperidol to
control symptoms.
16. Acute confusional state: if treating the underlying cause and environmental modification not working then
haloperidol sometimes used
17. Antipsychotics are associated with a significant increase in mortality in dementia patients
18. Constipation can cause delirium in the elderly
19. Delirium involves an impairment of conscious level and often involves psychotic symptoms
20. Donepezil - acetylcholinesterase inhibitor
21. Donepezil can cause insomnia
22. Donepezil is generally avoided (relative contraindication) in patients with bradycardia and is used with caution in
other cardiac abnormalities
23. Fluctuation of symptoms helps to differentiate delirium and dementia
24. Frontotemporal dementia presents with social disinhibition and often has a family history
25. Haloperidol is contraindicated in patients with Parkinson's disease
26. Memantine - NMDA receptor antagonist
27. Neuroimaging is required to diagnose dementia
253
M Y Elamin
MBBS, DTM&H, MCTM, MRCPI 1& 2
28. New surroundings can cause delirium in cognitively impaired patients
29. NICE guidelines do not support the use of memantine in mild dementia
30. Stepwise deterioration in cognitive function? - think vascular dementia
31. Tight control of vascular risk factors, rather than antidementia medication, is recommended by NICE in vascular
dementia
32. Typical antipsychotics should be avoided in delirious patients with a background of Parkinson's disease
33. Visual hallucinations with dementia = Lewy body dementia
34. Waterlow score - used to identify patients at risk of pressure sores
35. A number of mutations associated with early onset Alzheimer’s disease have now been i...
36. A number of mutations associated with early onset Alzheimer’s disease have now been identified, and
screening for them is available.
37. ACE- inhibitors have benefits in reducing mortality and morbidity in heart failure by preventing myocardial
remoddeling.
38. Citalopram is the antidepressant of choice in Alzheimer's disease due to it's effect ...
39. Common presentations and complications of chronic lymphocytic leukemia
40. Denosumab is a RANK ligand inhibitor, reducing osteoclast activity which is used when bisphosphonates
aren’t tolerated.
41. Experimental evidence supports a positive effect of metformin on cardiac function, both in models of
ischaemia and in models of heart failure
42. Flecainide is now considered an acceptable anti-arrhythmic in patients with WPW who d...
43. Guidance on use of ICD has changed, and where there is QRS prolongation in class III ...
44. Haemochromatosis has two mutations of the HFE gene (C282Y and H63D), which account for over 90% of
cases in Europeans.
45. Hypoglycaemia is common and mimics signs and symptoms of acute stroke; hence determination of blood
glucose and treatment of hypoglycaemia is part of the initial assessment of a patient with symptoms or
signs of acute stroke.
46. In acute psychosis associated with dementia NICE recommends using olanzapine, lorazepam, or
haloperidol to control symptoms.
47. It is imperative to recognize the syndrome of thrombotic thrombocytopenic purpura, and institute plasma
exchange with fresh frozen plasma as soon as possible to avoid the disease’s high mortality rate.
48. It is important to understand the issues surrounding the management of peri-operative assessment of
anticoagulation.
49. IV diuretic therapy is the initial intervention of choice for patients presenting wit...
50. Knowledge of some of the common chemotherapy related complications
254
M Y Elamin
MBBS, DTM&H, MCTM, MRCPI 1& 2
51. Macrocytosis can be caused by zidovudine.
52. Management of diabetes in MI
53. Management of HIV
54. Modern guidance recommends moving to early ablation in patients who suffer from paroxysmal AF,
because it is associated with a much greater chance of success when performed earlier.
55. Most cases of Parkinson’s disease occur in people with no family history of the disorder. The inheritance
pattern, if any, is unknown. Genetic screening is not offered in the absence of family history.
56. NICE now supports initiation of acetylcholinesterase inhibitors in mild Alzheimer’s disease.
57. Oseltamivir is the oral treatment of choice for H1N1 infection.
58. Patients under the age of 40 with less than 20% cardiovascular risk at 10 years should be initially offered
lifestyle interventions to deal with hypertension.
59. Patients with close contact of high risk patients should be offered influenza prophyl...
60. Patients with HIV have complex vaccination needs and live vaccines are contraindicated in patients with a
CD4 count of <200 cells/µL.
61. Plasmapheresis is a useful therapeutic modality with straightforward indications where it is imperative to
institute it as soon as possible once the pathology is identified and diagnosed.
62. Presentation of myelodysplastic syndrome (pancytopenia, blood film findings, and bone marrow findings)
63. Schistosomiasis is the third most devastating tropical disease in the world therefore consider in patients
who have travelled to endemic areas who have fever, lethargy and myalgia.
64. Sildenafil and other phosphodiesterase inhibitors may substantially reduce urine outp...
65. Sildenafil and other phosphodiesterase inhibitors may substantially reduce urine output in nephrogenic
diabetes insipidus.
66. Testing for heritable thrombophilia is not recommended in a patient with central venous catheter related
thrombosis, nor in central retinal vein thrombosis.
67. Tetracyclines can cause idiopathic intracranial hypertension
68. The mechanism by which lithium leads to diabetes insipidus, reduced GSK3 beta signalling, is well
established.
69. The prognosis of heart failure has been improved significantly with the use of angiotensin converting
enzyme (ACE) inhibitors and angiotensin receptor blockers that inhibit renin-angiotensin-aldosterone
system.
70. This management of essential thrombocythaemia is based on cytoreduction with hydroxycarbamide and an
antiplatelet agent such as aspirin to decrease the thrombotic risk.
71. This question relates to the management of relapsed Hodgkin lymphoma.
255
M Y Elamin
MBBS, DTM&H, MCTM, MRCPI 1& 2
72. Ticagrelor is associated with superior efficacy versus clopidogrel in patients who have suffered a STEMI.

256
M Y Elamin
MBBS, DTM&H, MCTM, MRCPI 1& 2
INFECTIOUS DISEASES AND STIS
1. > 60 years age group are at risk from meninigitis caused by Listeria monocytogenes
2. > 60 years age group are at risk from meninigitis caused by Neisseria meningitidis
3. > 60 years age group are at risk from meninigitis caused by Streptococcus pneumoniae
4. 0 - 3 months age group are at risk from meninigitis caused by group B Streptococcus
5. 0 - 3 months age group are at risk from meninigitis caused by E. coli
6. 0 - 3 months age group are at risk from meninigitis caused by Listeria monocytogenes
7. 3 months - 6 years age group are at risk from meninigitis caused by Neisseria meningitidis
8. 3 months - 6 years age group are at risk from meninigitis caused by Streptococcus pneumoniae
9. 6 years - 60 years age group are at risk from meninigitis caused by Neisseria meningitidis
10. 6 years - 60 years age group are at risk from meninigitis caused by Streptococcus pneumoniae
11. A 10-year-old presents with fever and a sore throat. Today they have a developed a fine, erythematous, 'sand-
paper' rash which is more prominent in flexural areas. - Streptococcus pyogenes
12. A 14-year-old immigrant from the Ukraine presents with a sore throat, fever and dysphagia. On examination she
is noted to have bulky cervical lymphadenopathy and a grey membrane on her pharynx - Corynebacterium
diphtheriae
13. A 15-year-old boy develops fever and tachypnoea one week after recovering from influenza - Staphylococcus
aureus pneumonia
14. A 15-year-old presents with a mouth ulceration and fever. On examination he has severe gingivostomatitis - herpes
simplex virus-1
15. A 1-month-old baby is admitted following a seizure. They have been irritable, feeding poorly and having fever.
The baby was born at 35 weeks following premature rupture of the membranes - group B Streptococcus
16. A 20-year-old man presents in summer with gradually worsening flu-like symptoms and a dry cough. On
examination he is noted to have erythema multiforme - mycoplasma pneumonia
17. A 20-year-old man presents with bloody diarrhoea. He also has abdominal pain and tenesmus. Yesterday he had
flu-like symptoms - Campylobacter
18. A 20-year-old man with no past medical history presents with a tender, red swelling on his back. A pointing,
erythematous abscess is found on examination - Staphylococcus aureus
19. A 20-year-old woman develops high fever, hypotension and a desquamanating rash, particularly of the palms and
soles. She has recently finished her period for which she uses tampons - Staphylococcus aureus
20. A 25-year-old man develops a painless pustule that later erodes to form an ulcer. By the time he presents he has
painful inguinal lymphadenopathy and proctocolitis - lymphogranuloma venereum
21. A 2-year-old child develops perianal itching, which is particularly bad at night - Enterobius vermicularis
22. A 2-year-old child is admitted with watery diarrhoea, vomiting and dehydration - rotavirus
23. A 30-year-old man develops fever, myalgia and lethargy associated with a dry cough and sore throat - influenza
virus
257
M Y Elamin
MBBS, DTM&H, MCTM, MRCPI 1& 2
24. A 30-year-old man who has recently returned from working in west Africa presents with a number of painful
genital ulcers associated with unilateral, painful inguinal lymphadenopathy. The ulcers have sharply defined,
ragged, undermined border - chancroid
25. A 30-year-old man with no past medical history presents with a productive cough, fever and chest pain. On
examination he has herpes labialis and signs consistent with consolidation in the right lower zone - pneumococcal
pneumonia
26. A 30-year-old presents with a two-day history of vomiting and watery, non-bloody diarrhoea. This is associated
with colicky abdominal pain. Temperature is normal. - gastroenteritis
27. A 30-year-old woman complains of thin, white, 'fishy' smelling discharge. Clue cells are seen on microscopy -
bacterial vaginosis
28. A 35-year-old intravenous drug user presents with decompensated liver disease. They do not drink alcohol. On
examination a number of purpura are found - hepatitis C
29. A 35-year-old man who has recently emigrated to the country presents with back pain associated with night sweats.
He has also lost 5kg in weight over the past month - Pott's disease
30. A 35-year-old woman presents with loin pain associated with fever. She has blood+++, leucocytes +++ on the urine
dipstick - pyelonephritis
31. A 3-year-old develops oral ulcers associated with a mild fever and vesicles on the palms and soles - coxsackie A
virus
32. A 40-year-old renal transplant recipient on immunosuppressants presents with fever and fatigue. A blood film
shows atypical lymphocytes, abnormal LFTs but a negative heterophil antibody test - cytomegalovirus
33. A 40-year-old with advanced HIV (CD4 count < 50) presents with blurred vision. Fundoscopy shows multiple
retinal haemorrhages - cytomegalovirus retinitis
34. A 40-year-old woman complains of an offensive, yellow/green, frothy vaginal discharge. On examination the cervix
is erythematous - Trichomonas vaginalis
35. A 5-year-old child develops fever, cough and conjunctivtis. On examination the child is irritable and has a blanching
erythematous rash on the face and upper chest. Small white spots can also be seen on the inside of the cheeks -
measles virus
36. A 5-year-old develops fever, pharyngitis and conjunctivitis - adenovirus
37. A 60-year-old man presents with flu-like symptoms and a dry cough. On examination he has a relative bradycardia
and appears slightly confused. His blood tests show a low sodium - Legionnaires' disease
38. A 6-year-old girl who has sickle cell disease develops pallor and fatigue shortly after a viral illness characterised by
an erythematous rash affecting her cheeks - parvovirus B19
39. A child develops a pink maculopapular rash, initially on the face before spreading to the whole body. Suboccipital
and postauricular lymphadenopathy is present - rubella
40. A child develops fever, malaise and myalgia associated with bilateral parotitis - mumps

258
M Y Elamin
MBBS, DTM&H, MCTM, MRCPI 1& 2
41. A child presents a sore throat and low-grade fever. On examination there are vesicles in the mouth and on the palms
and soles of the feet - hand, foot and mouth disease
42. A child presents with fever associated with an itchy, macular rash which then becomes vesicular - chickenpox
43. A child presents with fever, conjunctivitis and being irritable. He has also developed a maculopapular rash which
started behind ears before spreading and becoming blotchy & confluent - measles
44. A child presents with fever, malaise and tonsillitis associated with a 'strawberry' tongue and a fine punctate
erythematous rash sparing the area around the mouth - scarlet fever
45. A child presents with lethargy, fever and headache associated with a 'slapped-cheek' rash spreading to the
proximal arms and extensor surfaces - erythema infectiosum
46. A farmer presents with an atypical pneumonia after having a fever, headache and rigors for two weeks - Coxiella
burnetti
47. A man develops abdominal pain after a holiday where he was walking barefoot in northern Africa. Bloods show an
iron deficiency anaemia - Ancylostoma duodenale
48. A man develops cholangitis after eating some undercooked fish - Clonorchis sinensis
49. A man develops episodic angioedema whilst in west Africa. On one occasion he noticed a 'worm moving across the
left eye' - Loa loa
50. A man develops fever, periorbital oedema and myositis after eating raw pork - Trichinella spiralis
51. A man develops haematuria and frequency after visiting Malawi - Schistosoma haematobium
52. A man develops hyperpigmented skin and blindness after being bitten by a fly - Onchocerca volvulus
53. A man develops pneumonia after eating some undercooked crabmeat - Paragonimus westermani
54. A man develops profuse 'rice water' diarrhoea after visiting a disaster relief zone. He is vomiting but has no
abdominal pain. On examination he is severely dehydrated - cholera
55. A man develops visceral larva migrans. During the work-up he is noted to have developed some retinal granulomas
- Toxocara canis
56. A man develops who has recently arrived from Sudan develops high fever, jaundice and haematemesis. A liver
biopsy 3 months later shows Councilman bodies - yellow fever virus
57. A man presents with diarrhoea and abdominal pain, associated with papulovesicular lesions on the soles of his feet
and an urticarial rash - Strongyloides stercoralis
58. A man presents with severe vomiting. Earlier in the day he reheated some rice from a takeaway - Bacillus cereus
59. A man who works in the wool industry develops a black skin lesion surrounded by oedema. It initially started as a
small red papule - Bacillus anthracis
60. A middle-aged man who sustains an open fracture of his leg develops painful, oedematous, bronze coloured skin
around the fracture site. Haemorrhagic blebs and bullae are noted along with crepitus and tenderness on palpation
of the affected skin - C. perfringens
61. A patient develops gradual onset bloody diarrhoea after travelling to Egypt. His symptoms have now lasted four
weeks. He has occasional abdominal pain and tenesmus - amoebiasis
259
M Y Elamin
MBBS, DTM&H, MCTM, MRCPI 1& 2
62. A patient develops persistent foul-smelling watery diarrhoea and weight loss after travelling to India. His symptoms
have now lasted two weeks. He has no abdominal pain, nausea or fever - giardiasis
63. A patient develops spasms in his jaw which spread to his back and abdomen. The spasms become more severe
causing arching of the back. One week ago he had a deep cut to his leg - C. tetani
64. A patient develops watery diarrhoea 5 days after arriving in Mexico - Escherichia coli (enterotoxigenic)
65. A patient is admitted with blurred vision, a 'droopy face' and weakness in his arms. Yesterday he had eaten some
food that he had canned himself previously - C. botulinum
66. A patient presents with a 3 day history of bloody diarrhoea, fever and abdominal pain - shigella
67. A patient presents with fever, headache and a rash after visiting North Carolina. The rash started on the wrist and
ankles before spreading - Rickettsia ricketsii
68. A patient presents with severe vomiting 4 hours after eating at a restuarant - Staphylococcus aureus
69. A patient who had an intravenous catheter inserted 5 days ago becomes toxic. Erythema is noted around the
catheter insertion site - Staphylococcus epidermidis
70. A patient who is known to have HIV presents gradually worsening speech and behavioural problems associated
with coordination difficulties. A MRI shows multifocal non-enhancing lesions - JC virus
71. A patient who is known to have HIV presents with watery diarrhoea - Cryptosporidium
72. A recent immigrant from Latin America presents to the Emergency Department following a seizure. A CT head
shows multiple cystic lesions - Taenia solium
73. A sheep farmer is found to have hepatic cysts on ultrasound - Echinococcus granulosus
74. A traveller who has recently returned from South America develops megaoesophagus, megacolon and myocarditis
- Chagas' disease
75. A traveller who has recently returned from West Africa develops a painless subcutaneous nodule associated with
posterior cervical lymphadenopathy and intermittent fever. He later develops a headache and lethargy - African
trypanosomiasis
76. A west African man is admitted to hospital with intestinal obstruction. He has recently been suffering from
haemoptysis and wheezing - Ascaris lumbricoides
77. A young man from the north-east of the USA presents with fever, rash and arthralgia. The rash is a 5 cm
erythematous oval area with central clearing - Borrelia burgdorferi
78. A young man presents with a severe sore throat and fever. Cervical lymphadenopathy and splenomegaly are noted.
The heterophil antibody test is positive - Epstein-Barr virus
79. Aciclovir - guanosine analog, phosphorylated by thymidine kinase which in turn inhibits the viral DNA
polymerase
80. Aciclovir may cause crystalline nephropathy
81. Actinomyces - Gram-positive rod
82. Acute epiglottitis - Haemophilus influenzae
83. Acute prostatitis - quinolone or trimethoprim
260
M Y Elamin
MBBS, DTM&H, MCTM, MRCPI 1& 2
84. Acute pyelonephritis - broad-spectrum cephalosporin or quinolone
85. Acute sinusitis - phenoxymethylpenicillin
86. African sleeping sickness - Trypanosoma gambiense and Trypanosoma rhodesiense
87. Amantadine - inhibits uncoating (M2 protein) of virus in cell. Also releases dopamine from nerve endings
88. Amantadine may cause ataxia
89. Amantadine may cause confusion
90. Aminoglycosides may cause nephrotoxicity
91. Aminoglycosides may cause ototoxicity
92. Amphotericin B - binds with ergosterol forming a transmembrane channel that leads to monovalent ion (K+, Na+,
H+ and Cl) leakage
93. Amphotericin B may cause nephrotoxicity
94. An African farmer develops grossly swollen, oedematous legs associated with a massively enlarged scrotum
- Wuchereria bancrofti
95. An elderly patient presents with a painful, 'burning' rash affecting the right T8 dermatome - varicella zoster virus
96. An elderly patient presents with fever and a cough productive of 'rusty' coloured sputum. On examination there is
dullness at the right base of the lung and bronchial breathing - Streptococcus pneumoniae
97. Animal or human bite - co-amoxiclav
98. Argyll-Robertson pupil - tertiary syphilis
99. Ascending aortic aneurysm - tertiary syphilis
100. Aspergillosis - CD4 50-100
101. Atypical lymphocytes - infectious mononucleosis
102. Azithromycin , uses include: prophylaxis of Mycobacterium avium complex in HIV
103. Bacillus anthracis - Gram-positive rod
104. Back pain, night sweats, kyphosis - Pott's disease
105. Bacterial vaginosis - Gardnerella vaginalis
106. Bacterial vaginosis - oral or topical metronidazole or topical clindamycin
107. BCG - live attenuated
108. Black necrotic eshar on face, diabetic - mucormycosis
109. Blunted upper incisor teeth, keratitis, saber shins, saddle nose and deafness - syphilis
110. Buccal ulceration - secondary syphilis
111. Campylobacter enteritis - clarithromycin
112. Campylobacter jejuni - Gram-negative rod
113. Cannot be vaccinated against - hepatitis C
114. Cat scratch disease - Bartonella henselae
115. Causes the most morbidity in IVDUs - hepatitis C
116. Cellulitis (near the eyes or nose) - co-amoxiclav
261
M Y Elamin
MBBS, DTM&H, MCTM, MRCPI 1& 2
117. Cellulitis (NOT near the eyes or nose) - flucloxacillin
118. Chagas' disease - Trypanosoma cruzi
119. Chancre - primary syphilis
120. Chancroid - Haemophilus ducreyi.
121. Child with fever, barking cough at night, stridor - croup
122. Child with fever, conjunctivitis, white spots on buccal mucosa, rash starting behind ears - measles
123. Child with fever, fine punctate erythematous rash with 'sandpaper' texture, strawberry tongue - scarlet fever
124. Chlamydia - doxycycline or azithromycin
125. Clear appearance, glucose 70% of plasma, protein 0.3 g/l, WCC 2 per mm^3 (neuts) - normal sample
126. Clostridium - Gram-positive rod
127. Clostridium difficile (first episode) - vancomycin
128. Clostridium difficile (Second or subsequent episodes) - fidaxomicin
129. Cloudy appearance, glucose 25% of plasma, protein 1.5 g/l, WCC 2,000 per mm^3 (neuts) - bacterial
meningitis
130. Cloudy appearance, glucose 70% of plasma, protein 0.5 g/l, WCC 500 per mm^3 (lymphs) - viral meningitis
131. Common cold - rhinovirus
132. Community-acquired pneumonia - Streptococcus pneumoniae
133. Condylomata lata - secondary syphilis
134. Corynebacterium diphtheriae - Gram-positive rod
135. Croup - parainfluenza viruses
136. Cryptococcal meningitis - CD4 50-100
137. Cryptococcus neoformans - stains with India ink
138. Cryptosporidiosis - CD4 100-200
139. Cryptosporidium spp. - stains with Ziehl-Neelsen stain
140. Cutaneous larva migrans - Ancyclostoma Braziliense
141. Cytomegalovirus retinitis - CD4 < 50
142. Cytomegalovirus retinitis , treatment of choice: intravenous ganciclovir
143. Dental abscess - amoxicillin
144. Diarrhoea, pruritic rash, chest symptoms - Strongyloides stercoralis infection
145. Dilated cardiomyopathy, megaoesophagus, megacolon - Chagas' disease
146. Diphtheria - toxoid
147. Enterococci - Gram-positive cocci
148. Epstein-Barr virus may cause Burkitt's lymphoma
149. Epstein-Barr virus may cause HIV-associated central nervous system lymphomas
150. Epstein-Barr virus may cause Hodgkin's lymphoma
151. Epstein-Barr virus may cause nasopharyngeal carcinoma
262
M Y Elamin
MBBS, DTM&H, MCTM, MRCPI 1& 2
152. Erysipelas - flucloxacillin
153. Erythema infectiosum - slapped-cheek rash
154. Escherichia coli - Gram-negative rod
155. Exacerbation of bronchiectasis - Haemophilus influenzae
156. Exacerbations of chronic bronchitis - amoxicillin or tetracycline or clarithromycin
157. Foscarnet - pyrophosphate analog which inhibits viral DNA polymerase
158. Foscarnet may cause nephrotoxicity
159. Ganciclovir - guanosine analog, phosphorylated by thymidine kinase which in turn inhibits the viral DNA
polymerase
160. Ganciclovir may cause myelosuppression/agranulocytosis
161. Gardnerella vaginalis - clue cells are visible on microscopy
162. Gonorrhoea - intramuscular ceftriaxone
163. Granuloma inguinale - Klebsiella granulomatis
164. Granulomatous lesions of the skin and bones - tertiary syphilis
165. Gummas - tertiary syphilis
166. Haemophilus - subunit/conjugate
167. Haemophilus influenzae - Gram-negative rod
168. Hairy leukoplakia - CD4 200-500
169. Has a particularly high mortality rate in pregnant women - hepatitis E
170. Hepatitis A - inactivated preparations
171. Hepatitis B - subunit/conjugate
172. Hepatitis C complication - chronic hepatitis
173. Hepatitis C complication - cryoglobulinaemia
174. Hepatitis C complication - hepatocellular cancer
175. Hepatitis C complication - liver cirrhosis
176. Heterophile antibodies - infectious mononucleosis
177. HIV dementia - CD4 100-200
178. HIV, neuro symptoms, headache, CSF India ink positive - CNS cryptococcal infection
179. HIV, neuro symptoms, multiple brain lesions with ring enhancement - toxoplasmosis
180. HIV, neuro symptoms, single brain lesions with homogenous enhancement - CNS lymphoma
181. HIV, neuro symptoms, widespread demyelination - progressive multifocal leukoencephalopathy
182. HIV, purple/red skin lesions - Kaposi sarcoma
183. Human papillomavirus - subunit/conjugate
184. Hutchinson's teeth - congenital syphilis
185. Impetigo (mild) - topical hydrogen peroxide
186. Increases the risk of hepatocellular cancer - hepatitis B
263
M Y Elamin
MBBS, DTM&H, MCTM, MRCPI 1& 2
187. Increases the risk of hepatocellular cancer - hepatitis C
188. Infectious mononucleosis - atypical lymphocytes
189. Infectious mononucleosis - cold agglutins (IgM)
190. Infectious mononucleosis - deranged liver function tests
191. Infectious mononucleosis - haemolytic anaemia
192. Infectious mononucleosis - palatal petechiae
193. Infectious mononucleosis - splenomegaly
194. Influenza (intramuscular) - inactivated preparations
195. Influenza (intranasal) - live attenuated
196. Initial empirical therapy of meningitis (aged < 3 months) - intravenous cefotaxime + amoxicillin
197. Initial empirical therapy of meningitis (aged > 50 years) - intravenous cefotaxime + amoxicillin
198. Initial empirical therapy of meningitis (aged 3 months - 50 years) - intravenous cefotaxime
199. Interferon-alpha - human glycoproteins which inhibit synthesis of mRNA
200. Interferon-alpha may cause flu-like symptoms
201. Interferon-alpha may cause myelosuppression/agranulocytosis
202. Is spread by the faecal-oral route - hepatitis A
203. Is spread by the faecal-oral route - hepatitis E
204. Is spread by the parenteral, sexual and vertical transmission route - hepatitis B
205. Is spread by the parenteral, sexual and vertical transmission route - hepatitis C
206. Kaposi sarcoma - CD4 200-500
207. Kaposi's sarcoma - human herpes virus 8
208. Legionella pneumophilia - deranged liver function tests
209. Legionella pneumophilia - relative bradycardia
210. Legionella pneumophilia - confusion
211. Legionella pneumophilia - hyponatraemia
212. Linear scars at the angle of the mouth - congenital syphilis
213. Listeria monocytogenes - Gram-positive rod
214. Lower urinary tract infection - trimethoprim or nitrofurantoin
215. Lyme disease - cranial nerve palsy
216. Lyme disease - ELISA test for antibodies to Borrelia burgdorferi
217. Lyme disease - erythema chronicum migrans
218. Lyme disease - heart block
219. Lyme disease (early) , treatment of choice: doxycycline
220. Lyme disease (pregnant) , treatment of choice: amoxicillin
221. Lymphogranuloma venereum - Chlamydia trachomatis
222. May cause chronic hepatitis - hepatitis B
264
M Y Elamin
MBBS, DTM&H, MCTM, MRCPI 1& 2
223. May cause chronic hepatitis - hepatitis C
224. May cause cryoglobulinaemia - hepatitis B
225. May cause cryoglobulinaemia - hepatitis C
226. May cause polyarteritis nodosa - hepatitis B
227. Measles complication - subacute sclerosing panencephalitis
228. Meningococcus - subunit/conjugate
229. Metronidazole may cause disulfiram-like reaction with alcohol
230. MMR - live attenuated
231. Moraxella catarrhalis - Gram-negative cocci
232. Mulberry molars - congenital syphilis
233. Multiple painful genital ulcers, sexually active - genital herpes
234. Mycobacterium avium-intracellulare infection - CD4 < 50
235. Mycoplasma pneumoniae - cold agglutins (IgM)
236. Mycoplasma pneumoniae - erythema multiforme
237. Mycoplasma pneumoniae - haemolytic anaemia
238. Mycoplasma pneumoniae - bullous myringitis
239. Mycoplasma pneumoniae - may cause a positive cold agglutination test
240. Neisseria gonorrhoeae - Gram-negative cocci
241. Neisseria meningitidis - Gram-negative cocci
242. Non-falciparum malaria (acute infection) , treatment of choice: chloroquine
243. Non-falciparum malaria (preventing relapse) , treatment of choice: primaquine
244. Non-gonococcal urethritis - doxycycline or azithromycin
245. Oesophageal candidiasis - CD4 50-100
246. Oral polio - live attenuated
247. Oral rotavirus - live attenuated
248. Oral thrush - CD4 200-500
249. Oral typhoid - live attenuated
250. Oseltamivir - inhibits neuraminidase
251. Otitis externa (severe) - flucloxacillin
252. Otitis media (requiring antibiotics) - amoxicillin
253. Painless genital pustule --> ulcer --> painful inguinal lymphadenopathy --> proctocolitis - lymphogranuloma
venereum
254. Painless genital ulcer - syphilis
255. Painless ulcer - primary syphilis
256. Painless, warty lesions on the genitalia - secondary syphilis

265
M Y Elamin
MBBS, DTM&H, MCTM, MRCPI 1& 2
257. Pelvic inflammatory disease - oral ofloxacin + oral metronidazole or intramuscular ceftriaxone + oral
doxycycline + oral metronidazole
258. Pertussis - toxoid
259. Pharyngitis, grey-white membrane on pharynx - diphtheria
260. Plasmodium falciparum - causes the most severe disease
261. Plasmodium falciparum - commonest cause of malaria overall
262. Plasmodium malariae - classically causes a cyclical fever every 72 hours
263. Plasmodium malariae - is associated with nephrotic syndrome
264. Plasmodium ovale - classically causes a cyclical fever every 48 hours
265. Plasmodium ovale - primaquine should be given following acute treatment to destroy liver hypnozoites and
prevent relapse
266. Plasmodium vivax - primaquine should be given following acute treatment to destroy liver hypnozoites and
prevent relapse
267. Plasmodium vivax - classically causes a cyclical fever every 48 hours
268. Plasmodium vivax - the most common type of non-falciparum malaria
269. Pneumococcus - subunit/conjugate
270. Pneumocystis jiroveci - stains with silver stain
271. Pneumocystis jiroveci pneumonia , treatment of choice: co-trimoxazole
272. Pneumocystis jiroveci pneumonia complication - pneumothorax
273. Pneumocystis jirovecii pneumonia - CD4 100-200
274. Pneumonia possibly caused by atypical pathogens - clarithromycin
275. Pneumonia post-influenza infection - Staphylococcus aureus
276. Primary CNS lymphoma - CD4 50-100
277. Progressive multifocal leukoencephalopathy - CD4 100-200
278. Protease inhibitors may cause hyperlipidaemia
279. Protease inhibitors may cause impaired glucose tolerance
280. Protease inhibitors may cause lipodystrophy
281. Pseudomonas aeruginosa - Gram-negative rod
282. Q fever - Coxiella burnetti
283. Rabies - inactivated preparations
284. Rash on palms and soles - secondary syphilis
285. Requires the hepatitis B virus to propagate - hepatitis D
286. Ribavirin - guanosine analog which inhibits inosine monophosphate (IMP) dehydrogenase, interferes with the
capping of viral mRNA
287. Ribavirin may cause haemolytic anaemia
288. Rocky Mountain spotted fever - Rickettsia ricketsii
266
M Y Elamin
MBBS, DTM&H, MCTM, MRCPI 1& 2
289. Saddle nose - congenital syphilis
290. Salmonella (non-typhoid) - ciprofloxacin
291. Salmonella sp. - Gram-negative rod
292. Scarlet fever - strawberry tongue
293. Sensorineural deafness, congenital cataracts, patent ductus arteriosus, purpuric skin lesions and 'salt and
pepper' chorioretinitis - rubella
294. Severe falciparum malaria , treatment of choice: intravenous artesunate
295. Shigella sp. - Gram-negative rod
296. Shigellosis - ciprofloxacin
297. Shingles - CD4 200-500
298. Slightly cloudy appearance with fibrin web, glucose 25% of plasma, protein 4 g/l, WCC 500 per mm^3
(lymphs) - tuberculous meningitis
299. Staphylococci - Gram-positive cocci
300. Stippled vaginal epithelial cells - Gardnerella vaginalis
301. Streptococci - Gram-positive cocci
302. Strongyloides stercoralis , treatment of choice: ivermectin
303. Sulphonamides may cause Steven-Johnson syndrome
304. Syphilis - benzathine benzylpenicillin or doxycycline or erythromycin
305. Syphilis - benzylpenicillin
306. Syphilis - Treponema pallidum
307. Tabes dorsalis - tertiary syphilis
308. Terbinafine - inhibits squalene epoxidase
309. Tetanus - toxoid
310. Tetracyclines may cause discolouration of teeth
311. Tetracyclines may cause intracranial hypertension
312. Tetracyclines may cause photosensitivity
313. Throat infections (requiring antibiotics) - phenoxymethylpenicillin
314. Trichomonas vaginalis - motile trophozoites on wet mount
315. Trichomonas vaginalis , treatment of choice: metronidazole
316. Uncomplicated community-acquired pneumonia (not penicillin allergic) - amoxicillin
317. Uncomplicated falciparum malaria , treatment of choice: oral artemisinin-based combination therapies
318. Urinary frequency, haematuria - Schistosoma haematobium infection
319. Yellow fever - live attenuated
320. Crofloxacin is the first line treaatment for Salmonela Typhii. Resistance to fluoroquinolones is developing in
South Asia, where the drug to consider would be Azithromycin.

267
M Y Elamin
MBBS, DTM&H, MCTM, MRCPI 1& 2
321. sexually acquired reactive arthritis (SARA) is probably an immune mediated phenomenon linked to the immune
response to uro-genital micro-organisms. It affects men more than women (10:1) and the HLA B27 gene increases
susceptibility by up to 50 times. Typically it affects one to six joints asymmetrically in lower limbs, especially feet,
ankles and knees. It may be associated with conjunctivitis, iritis, circinate balanitis, tenosynovitis or enthesitis
(especially affecting Achilles tendon and plantar fascia attachment to calcaneum as in this case). Rarely cardiac
lesions including tachycardia, LV dilatation, pericarditis or aortic valve disease can occur.
322. West Nile encephalitis may present with seizures, reduced level of consciousness and a flaccid paralysis and
requires interferon-alpha treatment urgently.
323. The typical clinical presentation of vertebral artery dissection is with severe occipital headache followed by
focal neurological signs
324. A cerebral perfusion pressure of 61-80 mmHg is recommended in SAH to prevent watershed infarcts as well
as worsening of the bleed.
325. Diabetic patients with acute stroke resulting in dysphagia should have their oral medication held, sugars
monitored and restarted when their swallow improves.
326. 6 years - 60 years age group are at risk from meninigitis caused by Streptococcus pneumoniae
327. Legionella pneumophilia is best diagnosed by the urinary antigen test
328. Schistosomiasis is a risk factor for Squamous cell bladder cancer
329. Bacillus cereus characteristically occurs after eating rice that has been reheated
330. Campylobacter infection is often self-limiting but if severe then treatment with clarithromycin may be indicated
331. Chlamydia - treat with doxycycline
332. Clostridium botulinum presents with flaccid paralysis, whereas Clostridium tetani presents with spastic
paralysis
333. Clostridium - Gram-positive rod
334. Coxiella burnetti is the causative organism in Q fever
335. Coxiella presents may present with culture-negative endocarditis
336. E. coli is the most common cause of travellers' diarrhoea
337. Eikenella is notable as a cause of infections following human bites
338. HIV-1/2 Ab/Ag Immunoassay (fourth generation). This fourth-generation assay should be used to screen
individuals for HIV as it will detect HIV-1 and HIV-2 antibodies (Ab) and antigens (Ag) with high sensitivity within
a short window period (approximately 10 days after infection).
339. Listeria monocytogenes - Gram-positive rod
340. M. tuberculosis can cause hypoadrenalism
341. Moraxella catarrhalis - Gram-negative cocci
342. Mycoplasma pneumonia - treat with doxycycline or a macrolide
343. Neisseria gonorrhoeae - Gram-negative cocci

268
M Y Elamin
MBBS, DTM&H, MCTM, MRCPI 1& 2
344. P. knowlesi has the shortest erythrocytic replication cycle, leading to high parasite counts in short periods of
time
345. Rickettsia ricketsii is the causative organism for rocky mountain spotted fever
346. Salmonella typhi infection can cause rose spots on the abdomen
347. Schistosoma haematobium can be treated with praziquantel
348. Staph aureus is a coagulase positive Staph
349. Staphylococcus aureus is a gram+ve bacterium, catalase +ve, coagulase +ve organism
350. Staphylococcus aureus is associated with cavitating lesions when it causes pneumonia
351. Staphylococcus saprophyticus can commonly cause UTI in sexually active young women
352. Strongyloides stercoralis gains access to the body by penetrating the skin
353. A ‘hot stool’ (a stool examined within 15 minutes of passage, or kept warm) is required to make a microscopic
diagnosis of intestinal amoebiasis as once cooled Entamoeba histolytic returns to its cystic state and becomes
indistinguishable from the non-pathogen Entamoeba dispar
354. A history of Intravenous drug use coupled with a descending paralysis, diplopia and bulbar palsy is
characteristic of infection with Clostridium botulinum
355. Aciclovir - guanosine analog, phosphorylated by thymidine kinase which in turn inhibits the viral DNA
polymerase
356. Aciclovir is much more specific for viral than mammalian DNA polymerase
357. Acute epiglottitis is caused by Haemophilus influenzae type B
358. Acute schistosomal infection can present with a syndrome known as Katayama Fever
359. Acute toxoplasmosis in the immunocompetent patient can mimic acute EBV infection (low-grade fever,
generalised lymphadenopathy with prominent cervical lymph nodes and malaise) and should be suspected with
negative EBV serology. Pregnancy testing and counselling is paramount due to the risk of congenital toxoplasmosis
360. Aerosolized pentamidine is an alternative treatment for Pneumocystis jiroveci pneumonia but is less effective
with a risk of pneumothorax
361. Alcohol gels are less effective than handwashing in preventing the spread of norovirus
362. All patients with a CD4 count lower than 200/mm3 should receive prophylaxis against Pneumocystis jiroveci
pneumonia
363. Amantadine - inhibits uncoating (M2 protein) of virus in cell. Also releases dopamine from nerve endings
364. Amoebiasis should be considered in the presentation of dysentery after a long incubation period
365. Amoxicillin is an alternative to treat early Lyme disease if doxycycline is contraindicated such as in pregnancy
366. Amphotericin B binds with ergosterol, a component of fungal cell membranes, forming pores that cause lysis
of the cell wall and subsequent fungal cell death
367. An aspergilloma may arise in a lung cavity that developed secondary to previous tuberculosis
368. Animal bite - co-amoxiclav
369. Animal bites are generally polymicrobial but the most common isolated organism is Pasteurella multocida
269
M Y Elamin
MBBS, DTM&H, MCTM, MRCPI 1& 2
370. Anthrax presents with a black eschar that is typically painless; it is treated with ciprofloxacin
371. Anti-retroviral therapy for HIV is now started at the time of diagnosis, rather than waiting for the CD4 count to
drop to a particular level
372. Antiretroviral therapy should generally be started as soon as HIV is diagnosed
373. Antivirals are of no benefit in the treatment of confirmed viral meningitis
374. Aspergilloma on x-ray may show a fungal ball cavity with a crescent sign
375. Asymptomatic bacteriuria should not be treated except in pregnancy, children younger than 5 years or
immunosuppressed patients due to the risk of complications
376. Atypical lymphocytes - ?glandular fever
377. Azithromycin, erythromycin or amoxicillin may be used to treat Chlamydia in pregnancy
378. Bacterial vaginosis - overgrowth of predominately Gardnerella vaginalis
379. Bacterial vaginosis: oral metronidazole
380. Benznidazole is used in the acute phase of Chagas' disease to manage the illness
381. Bilateral conjunctivitis, bilateral calf pains and high fevers in a sewage worker suggests leptospirosis
382. Botulinum toxin inhibits the release of acetylcholine at synapses
383. Campylobacter infection is characterised by a prodrome, abdominal pain and bloody diarrhoea
384. Cardiomyopathy is the most frequent and most severe manifestation of chronic Chagas' disease
385. Cat scratch disease - caused by Bartonella henselae
386. Cephalosporins act by inhibiting cell wall formation
387. Cephalosporins are a type of beta-lactam
388. Chancroid causes painful genital ulcers
389. Chickenpox exposure in pregnancy - first step is to check antibodies
390. Chickenpox exposure in pregnancy <= 20 weeks - if not immune give VZIG
391. Chikungunya can present with debilitating joint pain
392. Ciprofloxacin promotes acquisition of MRSA
393. Cryptosporidium can be diagnosed by modified Ziehl-Neelsen staining of stool to reveal red cysts
394. Cutaneous leishmaniasis acquired in South or Central America merits treatment due to the risk of
mucocutaneous leishmaniasis whereas disease acquired in Africa or India can be managed more conservatively
395. Cysticercosis can be treated with bendazoles
396. Clostridioides difficile: spreads via the faecal-oral route by ingestion of spores
397. HBsAg negative, anti-HBs positive, IgG anti-HBc negative - previous immunization. An equivocal (or low-level
positive) HBV surface antigen may occasionally occur if the patient has received a recent HBV vaccination.
398. Pneumonia, peripheral blood smear showing red blood cell agglutination → Mycoplasma pneumoniae
399. Negative HCV RNA means there is no current HCV infection and positive anti-HCV antibodies means there
has been a previously cleared HCV infection

270
M Y Elamin
MBBS, DTM&H, MCTM, MRCPI 1& 2
400. The BCG vaccine is unreliable in protecting against pulmonary tuberculosis. It is thought to prevent 60 - 80% of
cases of TB meningitis in children who received BCG as neonates
401. The treatment of botulism - supportive care and botulism antitoxin
402. Vancomycin is a useful antibiotic to treat MRSA infections
403. Yellow fever typically presents with flu like illness → brief remission→ followed by jaundice and
haematemesis
404. Dexamethasone improves outcomes (by reducing neurological sequelae) in the treatment of bacterial meningitis
405. Diabetes is the strongest risk factor for the development of melioidosis
406. Disseminated gonococcal infection triad = tenosynovitis, migratory polyarthritis, dermatitis
407. Doxycycline is typically used in the management of typhus
408. E.coli O157: H7 is the strain causing haemolytic uraemic syndrome
409. EBV: associated malignancies:
a. Burkitt's lymphoma
b. Hodgkin's lymphoma
c. nasopharyngeal carcinoma
410. ELISA is the first-line investigation for suspected Lyme disease in patients with no history of erythema migrans
411. Enterococci - Gram-positive cocci
412. Exchange transfusion should be considered in cases of severe parasitaemia (>10%)
413. False positive VDRL/RPR: 'SomeTimes Mistakes Happen':
a. pregnancy
b. SLE, anti-phospholipid syndrome
c. tuberculosis
d. leprosy
e. malaria
f. HIV
414. Fever and the presence of a eschar in a patient returning from South East Asia is strongly suggestive of scrub
typhus (caused by Orientia tsutsugamushi) and necessitates urgent treatment with doxycycline
415. First line treatment for early Lyme disease is a 14-21 day course of oral doxycycline
416. Fish tank granuloma is caused by Mycobacterium marinum
417. Following treatment for syphilis: TPHA remains positive, VDRL becomes negative
418. Genital ulcers
a. painful: herpes much more common than chancroid
b. painless: syphilis more common than lymphogranuloma venereum
419. Genital wart treatment
a. multiple, non-keratinised warts: topical podophyllum
b. solitary, keratinised warts: cryotherapy
271
M Y Elamin
MBBS, DTM&H, MCTM, MRCPI 1& 2
420. Genital warts - 90% are caused by HPV 6 & 11
421. Gonorrhoea is a gram-negative diplococci that can be identified on gram staining
422. Granuloma inguinale - Klebsiella granulomatis
423. Hepatitis C - 55-85% become chronically infected
424. Heterophile antibodies - infectious mononucleosis
425. HIV antibody testing is most reliable 3 months post exposure
426. HIV drugs, rule of thumb:
a. NRTIs end in 'ine'
b. PIs: end in 'vir'
c. NNRTIs: nevirapine, efavirenz
427. HIV is an RNA retrovirus
428. HIV, neuro symptoms, multiple brain lesions with ring enhancement - toxoplasmosis
429. HIV, neuro symptoms, single brain lesions with homogenous enhancement - CNS lymphoma
430. HIV, neuro symptoms, widespread demyelination - progressive multifocal leukoencephalopathy
431. HIV: anti-retrovirals - P450 interaction
a. nevirapine (a NNRTI): induces P450
b. protease inhibitors: inhibits P450
432. Hookworms may cause an iron deficiency anaemia in patients returning from travel to endemic areas e.g. the
Indian subcontinent
433. HPV vaccination should be offered to men who have sex with men under the age of 45 to protect against anal,
throat and penile cancers
434. Human bites, like animal bites, should be treated with co-amoxiclav
435. If a patient has had 5 doses of tetanus vaccine, with the last dose < 10 years ago, they don't require a booster
vaccine nor immunoglobulins, regardless of how severe the wound is
436. If a sexually active patient presents with genital chlamydia and bowel symptoms, LGV proctocolitis should be
considered
437. Immune reconstitution inflammatory syndrome can occur in HIV positive patients when starting anti-
retrovirals; this is an immune phenomenon that results in the clinical worsening of a pre-exisiting opportunistic
infection
438. Immunocompetent patients with toxoplasmosis don't usually require treatment
439. Immunocompromised patients with toxoplasmosis are treated with pyrimethamine plus sulphadiazine
440. In the UK all HIV positive women should be advised not to breastfeed
441. Infectious mononucleosis is generally a self-limiting condition
442. Integrase inhibitors ('gravirs') - blocks the enzyme that inserts the viral genome into the DNA of the host cell
443. Intramuscular ceftriaxone is the treatment of choice for Gonorrhoea

272
M Y Elamin
MBBS, DTM&H, MCTM, MRCPI 1& 2
444. IV ceftriaxone should be used as first-line treatment of Lyme disease with disseminated or central nervous
system involvement
445. Ivermectin is the drug of choice for treatment of strongyloidiasis
446. Japanese encephalitis can present with Parkinsonism- this is a sign of basal ganglia involvement
447. Kaposi's sarcoma - caused by HHV-8 (human herpes virus 8)
448. Kaposi's sarcoma is caused by HHV-8 infection in HIV positive individuals
449. Lassa fever is contracted by contact with the excreta of infected African rats (Mastomys rodent) or by person-
to-person spread
450. Latent syphilis (i.e asymptomatic syphilis) can occur at an early and a late stage and requires the same antibiotic
treatment
451. Leishmaniasis is caused by the intracellular protozoa Leishmania, which are spread by the bites of sandflies
452. Leprosy leads to skin hypopigmentation
453. Leptospirosis - give penicillin or doxycycline
454. Live attenuated vaccines
a. BCG
b. MMR
c. oral polio
d. yellow fever
e. oral typhoid
455. Live vaccines given by injection may be either given concomitantly or a minimum interval of 4 weeks apart to
prevent risk of immunological interference
456. Lymphogranuloma venereum - Chlamydia trachomatis
457. Macrolides inhibit the 50S subunit of ribosomes
458. Macrolides such as clarithromycin are used to treat Legionella
459. Man returns from trip abroad with maculopapular rash and flu-like illness - think HIV seroconversion
460. Management of necrotising fasciitis revolves around immediate surgical debridement and IV antibiotics
461. Measles complication - pneumonia
462. Measles is characterised by prodromal symptoms, Koplik spots. maculopapular rash starting behind the ears
and conjunctivitis
463. Measles is spread by aerosol transmission
464. Most common organism found in central line infections - Staphylococcus epidermidis
465. Mucocutaneous ulceration following travel? - Leishmania brasiliensis
466. Mumps meningitis is associated with a low CSF glucose
467. Mycoplasma is associated with erythema multiforme
468. Mycoplasma? - serology is diagnostic
469. Negative non-treponemal test + positive treponemal test is consistent with successfully treated syphilis
273
M Y Elamin
MBBS, DTM&H, MCTM, MRCPI 1& 2
470. Nitazoxanide may be used to treat immunocompromised patients with cryptosporidiosis
471. Non specific (non gonococcal) urethritis is a common presentation where inflammatory cells but no gonococcal
bacteria are seen on swab; it requires treatment with doxycycline or azithromycin
472. Non-falciparum malaria (acute infection) , treatment of choice: artemisinin-based combination therapy (ACT)
or chloroquine
473. Nucleic acid amplification tests (NAATs) are the investigation of choice for Chlamydia
474. One of the sequelae of diphtheria is cardiovascular disease; notably heart block
475. Oral ciprofloxacin or rifampicin is used as prophylaxis for contacts of patients with meningococcal meningitis
476. p24 testing can be used 4 week after an exposure and is often used in combination with the HIV antibody test
in clinical practice
477. Painless black eschar - anthrax
478. Parasitaemia > 2% is a feature of severe malaria
479. Parvovirus is a common cause of fetal hydrops during pregnancy and can be treated with fetal transfusion
480. Patients who are immunosuppressed secondary to long-term steroids or methotrexate should receive VZIG if
they are exposed to chickenpox and have no antibodies to varicella
481. Patients with an uncertain tetanus vaccination history should be given a booster vaccine + immunoglobulin,
unless the wound is very minor and < 6 hours old
482. Patients with hyposplenism should be vaccinated against pneumococcal, Haemophilus type B and
meningococcus type C
483. Patients with peritonsillar abscesses can develop Lemierre's syndrome (thrombophlebitis of the IJV)- this can
present with neck pain, and can result in septic pulmonary embolism
484. Pneumocystis jiroveci penumonia is treated with co-trimoxazole, which is a mix of trimethoprim and
sulfamethoxazole
485. Pneumonia caused by Streptococcus pneumoniae is associated with cold sores
486. Pneumonia in an alcoholic - Klebsiella
487. Pneumonia, peripheral blood smear showing red blood cell agglutination → Mycoplasma pneumoniae
488. Perianal itching in children, possibly affecting other family members → Enterobius vermicularis (threadworms)
489. Post-exposure prophylaxis for HIV: oral antiretroviral therapy for 4 weeks
490. Pott's disease (spinal TB) is an important differential in the setting of chronic back pain, fevers and old TB
491. Pregnant women with a UTI: nitrofurantoin is first-line unless the woman is close to term
492. Primaquine is used in non-falciparum malaria to destroy liver hypnozoites and prevent relapse
493. Pruritic rash on the buttocks or soles ?Strongyloides stercoralis
494. Pubic lice can be treated with either malathion or permethrin
495. Quinine is no longer recommended as a first-line treatment for complicated/severe falciparum malaria
496. Rabies - following possible exposure give immunglobulin + vaccination

274
M Y Elamin
MBBS, DTM&H, MCTM, MRCPI 1& 2
497. Recurrent herpes outbreaks in pregnancy should be treated with suppressive therapy; risk of transmission to
the baby is low and aciclovir is safe to use in pregnant women
498. Renal transplant + infection ?CMV
499. Retro-orbital headache, fever, facial flushing, rash, thrombocytopenia in returning traveller → ?dengue
500. Ribavirin - guanosine analog which inhibits inosine monophosphate (IMP) dehydrogenase, interferes with the
capping of viral mRNA
501. Rifampicin inhibits RNA synthesis
502. Schistosoma haematobium causes haematuria
503. Schistosomiasis is treated with praziquantel
504. Schizonts and late stages of trophozoites are typically sequested in Plasmodium falciparum infection and their
presence on the peripheral smear indicates severe disease
505. Severe falciparum malaria - intravenous artesunate
506. Severe hepatitis in a pregnant woman - think hepatitis E
507. Severe manifestations of enteric fever include bowel perforation and neurological complication. If these occur
it is typically in the third week of illness
508. Shigella infection is usually self limiting and does not require antibiotic treatment; antibiotics are indicated for
people with severe disease, who are immunocompromised or with bloody diarrhoea
509. Staphylococcal toxic shock syndrome is characterised by fever, hypotension and a rash → desquamation
510. Stereotypical features of Legionella include flu-like symptoms and a dry cough, relative bradycardia and
confusion. Blood tests may show hyponatraemia
511. Supportive therapy is the mainstay of treatment in Cryptosporidium diarrhoea
512. Tampon use is a risk factor for staphylococcal toxic shock syndrome
513. Telbivudine is a synthetic thymidine nucleoside analogue
514. Terbinafine inhibits the fungal enzyme squalene epoxidase, causing cellular death
515. Tetanus toxin (tetanospasmin) blocks the release of the inhibitory neurotransmitters GABA and glycine
resulting in continuous motor neuron activity
516. Tetracyclines can cause photosensitivity
517. Tetracyclines inhibit the 30S subunit of ribosomes
518. The BCG vaccine is unreliable in protecting against pulmonary tuberculosis
519. The Jarisch-Herxheimer reaction is a known phenomenon following syphilis treatment that does not require
any specific treatment or investigations other antipyretics
520. The Jarisch-Herxheimer reaction, unlike an anaphylactic reaction, will not present with hypotension and
wheeze
521. The most common causes of viral meningitis in adults are enteroviruses
522. The treatment of botulism - supportive care and botulism antitoxin
523. The viral load determines the risk of HIV transmission following a needle stick injury
275
M Y Elamin
MBBS, DTM&H, MCTM, MRCPI 1& 2
524. Thick blood films check for parasite burden, thin films allow for speciation
525. Treatment for invasive amoebiasis should be followed by a luminal amoebicide (diloxanide furoate,
iodoquinol, and paromomycin) to eradicate the cystic stage which is resistant to metronidazole and tinidazole
(which are used against the invasive stage)
526. Treatment of pelvic inflammatory disease: oral ofloxacin + oral metronidazole or intramuscular ceftriaxone
+ oral doxycycline + oral metronidazole
527. Trimethoprim and co-trimoxazole should be avoided in patients on methotrexate
528. Trypanosomiasis: African-form causes sleeping sickness and American-form causes Chagas' disease
529. Tuberculoid leprosy is suggested by a limited number of skin lesions, hypaesthesia and hair loss
530. URTI symptoms + amoxicillin → rash ?glandular fever
531. Vancomycin is a useful antibiotic to treat MRSA infections
532. Yellow fever - live attenuated
533. Yellow fever typically presents with flu like illness → brief remission→ followed by jaundice and haematemesis
534. West Nile encephalitis may present with seizures, reduced level of consciousness and a flaccid paralysis and
requires interferon-alpha treatment urgently.
535. A Heaf test diameter greater than 15 mm raises the possibility of active tuberculosis infection.
536. A history of a mild constitutional illness coupled with painful and blistering genital ulceration and
neuropathic type pain fits best with herpes simplex infection.
537. A history of abdominal bloating with intermittent diarrhoea and strong smelling bowel gas is typical of
giardiasis in the returning traveller .
538. A history of dyspnoea and desaturation on exercise in an HIV-positive patient would suggest PCP. On
examination, the chest is typically clear in PCP.
539. A history of night sweats, haemoptysis and miliary shadowing in an alcoholic is highly suggestive of
TB.
540. A history of severe exudative pharyngitis in a person who has recently travelled to an endemic area is
highly suggestive of diphtheria. In developed countries, it is seen more as a result of the waning of vaccine
efficacy or inadequate immune response to the vaccine.
541. A raised eosinophil count supports a diagnosis of schistosomiasis in the retirning traveler with a new
rash.
542. Abdominal bloating, diarrhoea, and excess production of gas. and consistent with Giardia lamblia
infection in the returning traveler.
543. Aciclovir acts through inhibition of viral deoxyribonucleic acid (DNA) polymerase but it is a pro-drug
and first requires phosphorylation by thymidine kinase.
544. Aciclovir is a synthetic purine nucleotide analogue and as such is a specific inhibitor of herpes virus
DNA polymerase.

276
M Y Elamin
MBBS, DTM&H, MCTM, MRCPI 1& 2
545. Acute amoebic dysentery is managed with a course of oral metronidazole or tinidazole, to be followed
by a ten-day course of diloxanide to eradicate colonisation of the gut.
546. Acute EBV typically presents with a history of one to two weeks of fatigue and malaise, fever,
pharyngitis, and symmetrical, bilateral lymphadenopathy.
547. Acute HIV can present with fever, malaise and generalised lymphadenopathy. The incubation period is
generally between 3-6 weeks.
548. Acute human immunodeficiency virus (HIV) presents two weeks to three months after exposure to the
virus; the illness typically consists of: fever, arthritis, rash, and lymphadenopathy.
549. Acute human immunodeficiency virus (HIV) seroconversion illness should be suspected where there has
been a risk of exposure.
550. Acute illness on return from a holiday with no evidence of a preceding illness such as diarrhoea, and
involvement of the larger joints, that is, knees, with effusions - suggests gonorrhoea.
551. Acute retroviral syndrome is said to occur in 60-80% of patients between two and 12 weeks following
exposure to HIV.
552. All patients diagnosed with Streptococcus bovis infective endocarditis should undergo evaluation
including investigation of the gastrointestinal tract.
553. All tuberculosis patients should have pre-treatment LFTs.
554. Although other organisms can cause meningitis, N. meningitidis is the likeliest cause in infants and
young children.
555. Although seldom seen in countries in which a vaccine is available, measles is a major health problem in
refugee camps in Africa. Failure to vaccinate is the single biggest cause of measles outbreaks.
556. American ('New World') cutaneous leishmaniasis presents with single or multiple lesions (usually in sun
exposed areas), or mucosal disease.
557. An ascitic neutrophil count of >250/mm3 is indicative of spontaneous bacterial peritonitis which should
be treated with IV antibiotics.
558. Antibiotics such as Doxycycline have been shown to reduce fluid loss and hasten clearance of the
organism from the gut in cases of cholera.
559. Aplastic crisis in sickle cell anaemia (SSA) is caused by infection with the parvovirus B19.
560. Atazanavir causes hyperbilirubinaemia with normal transaminases and alkaline phosphatase (mimicking
Gilbert's syndrome).
561. B cells activated in the primary immune response initially produce IgM
562. Bacillus cereus food poisoning occurs around six to eight hours after ingestion of reheated rice.
563. BAL is more sensitive than sputum collection at detecting TB, either by smear or culture.

277
M Y Elamin
MBBS, DTM&H, MCTM, MRCPI 1& 2
564. Blood cultures are the most important investigation for enteric fevers caused by Salmonella typhi or S.
paratyphi.
565. Campylobacter infection is one of the commonest causes of inflammatory diarrhoea with pain often a
prominent feature of the illness frequently localising to the right iliac fossa. Symptoms may last a week or
longer.
566. Campylobacter is a common cause of bloody diarrhoea in the UK, and is characterised by fever,
abdominal cramps and vomiting.
567. Campylobacter is a common cause of food poisoning and presents after 1-2 days with bloody diarrhoea
and right iliac fossa pain.
568. Cephalic tetanus causes severe dysphagia.
569. Chemoprophylaxis is not normally indicated for close contacts of those with pneumococcal meningitis
570. Chicken pox pneumonitis should be recognised promptly, with IV acyclovir the intervention of choice.
571. Chlamydia psittaci is endemic in birds including psittacine birds, canaries, finches, pigeons and poultry.
Person to person transmission occurs, especially in a hospital environment.
572. Chronic lymphocytic leukaemia (CLL) is commonly complicated by panhypogammaglobulinaemia.
Although intravenous immunoglobulin prevents recurrent infections it does not prolong survival.
573. Ciprofloxacin is recommended for first-line antibiotic therapy in Escherichia coli when supportive
measures fail or when signs of sepsis are present.
574. Clindamycin is a bacteriostatic antibiotic, it acts by switching off protein synthesis within bacteria; this,
in turn, will lead to decreased exotoxin expression, thereby removing the mediators of TSS.
575. Close contacts of cases of meningitis C should be offered an extra dose of vaccine and ciprofloxacin
chemoprophylaxis.
576. Close contacts of Haemophilus influenzae meningitis should receive rifampicin
577. Co-amoxiclav is recommended as first-line treatment for mild-to-moderate infections following a dog,
cat, or human bite.
578. Contaminated air conditioning units are often to blame for outbreaks of Legionnaire's disease and
obtaining a good history, as ever, is paramount in making a diagnosis
579. Cutaneous anthrax is associated with a black eschar without pus, tends to be painless and to have
widespread oedema.
580. Cutaneous diphtheria presents with non-healing ulcers covered with a grey membrane, which can
develop bacterial co-infection.
581. Cutaneous leishmaniasis is suggested by painless ulceration after return form an endemic area.
582. Deficiencies of complement C5-9 predispose to disseminated Neisseria infections.

278
M Y Elamin
MBBS, DTM&H, MCTM, MRCPI 1& 2
583. Dengue fever is caused by an arthropod-borne Flavivirus with an incubation period of seven days. The
new WHO classification (2009) is Dengue without warning signs (fever, headache, nausea, rash,
leukopaenia), Dengue with warning signs (pain abdomen, persistent vomiting, mucosal bleed,
hepatomegaly, raised haematocrit and low platelets) and severe Dengue (shock from capillary leakage).
584. Diagnosis and Management of TB and its complications
585. Diagnosis and Management of TB and its complications.
586. Diazepam, neuromuscular blockade, and intubation may all be required during the acute phase of tetanus
infection.
587. DIC is associated with an elevated D-dimer as well as decreased platelets and fibrinogen with normal
clotting factors.
588. Diphtheria, coxsackie virus, Chagas disease, and toxoplasmosis are all associated with myocarditis.
589. Doxycycline is the drug of choice for NGU.
590. During pregnancy, treatment options for chlamydia infection are erthromycin or amoxicillin or
azithromycin
591. Early treatment of herpes zoster with antivirals can control acute symptoms and reduce the risk of longer
term complications
592. EBV infection, infectious mononucleosis, which may be confirmed by the presence of immunoglobulin
(Ig)M to EBV.
593. EBV is a common cause of pharyngitis in teenagers, and antibody testing may be negative in up to 25%
of individuals during the first week of the illness.
594. EBV-associated lymphoproliferative disease may occur in individuals with inherited or acquired
immunodeficiency syndromes.
595. Effective and regular hand washing is vital in preventing spread of infection within hospitals.
596. Empirical treatment of uncomplicated MRSA skin and soft tissue infections should be with oral
doxycycline or fusidic acir or trimethoprim combined with rifampicin.
597. Emtricitabine causes hyperpigmentation of the skin, including palmar creases, in 8% of black patients.
598. Ensuring meat products are thoroughly cooked is effective in helping reducing transmission of
Escherichia coli O157:H7
599. Enterotoxigenic E coli is the commonest cause of traveller's diarrhoea and is usually a self-limiting
condition.
600. Enterovirus is the commonest cause of viral meningitis in the adult population.
601. Enteroviruses and mumps are the commonest causes of viral meningitis.
602. Females are more commonly long term carriers than males of Salmonella typhi, as was the case with
Typoid Mary.
279
M Y Elamin
MBBS, DTM&H, MCTM, MRCPI 1& 2
603. Finding multiple ring-enhanced lesions on CT scan needs further investigations.
604. First line management of needlestick injuries includes immediate washing of the affected area under
running water
605. Flexor sheath infection is an emergency and treatment is washout of the flexor sheath and intravenous
antibiotics.
606. Flucloxacillin and gentamicin is the regime of choice for methicillin-sensitive Staphylococcus aureus,
with linezolid an appropriate alternative in MRSA.
607. Fluid resuscitation is the mainstay of therapy for cholera.
608. Food poisoning with colicky abdominal pain, diarrhoea, no vomiting, and an incubation period between
9-13 hours is typical of Clostridium perfringens.
609. G. lamblia is usually acquired by the faeco-oral route.
610. Giardia classically presents with a few weeks of diarrhoea with an offensive smell.
611. Gonorrhoea is a common STD, concomitant treatment with a penicillin and doxycycline can be useful.
612. Gonorrhoea is the second most common bacterial STI in the UK after chlamydia with primary infection
is symptomatic in 90-95% of men, but only 50% of women.
613. Griseofulvin is implicated in causing Stevens-Johnson syndrome.
614. Guillain-Barré syndrome is often preceded by an episode of infection such as viral (cytomegalovirus
[CMV]) or bacterial (Campylobacter).
615. Haemolytic uraemic syndrome (HUS) presents with acute renal failure, microangiopathic haemolytic
anaemia, and thrombocytopenia with normal clotting.
616. Haemorrhagic lesions of the temporal lobe are typical for Herpes simplex virus infection.
617. Haemosiderosis does not lead to chronic liver disease, Haemosiderosis usually arises due to parenteral
iron overload, for example, in patients with aplastic anaemia after multiple transfusions
618. Hepatitis C infection is strongly associated with mixed essential cryoglobulinaemia which may produce
mesangiocapillary (also known as membranoproliferative) glomerulonephritis.
619. Hepatitis E is viral hepatitis transmitted via the faeco-oral route, and is not protected against by the
current vaccination program.
620. Hepatotoxicity is a feature of antituberculous treatment. The Joint Tuberculosis Committee of the British
Thoracic Society recommend that liver function should be checked before treatment for clinical cases.
621. Herpes simplex encephalitis (HSE) often presents subacutely over several days with declining cerebral
function.
622. Herpes simplex encephalitis produces a typical EEG pattern with lateralised periodic discharges at 2 Hz.

280
M Y Elamin
MBBS, DTM&H, MCTM, MRCPI 1& 2
623. Herpes simplex is a sexually transmitted disease and the commonest cause of multiple painful genital
lesions.
624. HPV 16 is oncogenic and causes squamous cell carcinomas in the oral cavity, cervix, anus and penis.
625. Human herpes virus 8 is also associated with primary effusion lymphoma (a rare lymphoma of serous
cavities) and Castleman's disease.
626. Hypoglycaemia is an important side effect of quinine therapy and should be monitored in those having
intravenous quinine.
627. If pre-existing psoriasis flares up for no apparent reason or middle-aged people develop psoriasis for the
first time, one should exclude underlying HIV infection in those patients.
628. IgA nephropathy is the most common form of glomerulonephritis globally, and is most often seen in
young men following an upper respiratory tract infection. It classically presents with rash and arthritis.
629. IgG comprises the majority of circulating antibody in serum.
630. In cases of Campylobacter, appropriate fluid replacement and anti-emetics are initially indicated; most
units advocate no antibiotic treatment as the illness is usually self limiting.
631. In disseminated gonococcal infection, leading to monoarthritis, IV therapy for at least 48 hours followed
by oral antibiotics is the most appropriate management.
632. In herpes simplex encephalitis, MRI brain normally shows changes in the temporal lobes.
633. In herpes simplex encephalitis, MRI brain normally shows changes in the temporal lobes.
634. In HIV, focal symptoms eventually appear as the disease progresses if left untreated.
635. In humans, anthrax most commonly causes a painless, black, indurated eschar.
636. In meningeal TB at least 12 months of anti-tuberculous therapy, (including an initial 2 month period of 4
drugs) is usually required.
637. In patients unable to produce sputum, undertaking a bronchoscopy with bronchial washings for
microscopy staining and culture is the investigation of choice.
638. In patients with with community-acquired pneumonia, the presence of a respiratory rate equal or
>30/min will significantly increased risk of death.
639. In tuberculosis of the mediastinal lymph nodes, corticosteroids are effective in reducing lymph node
enlargement and inflammation and hence will help the stridor and breathlessness.
640. Increased neutrophils are found in CMV polyradiculomyelopathy but not in Guillain-Barré syndrome.
641. Individuals with blood group O are more susceptible than other individuals to severe cholera, although
the mechanism underlying this association is unknown.
642. Infection with Japanese encephalitis has been reported in travellers who have spent only short periods in
endemic areas, and transplacental transmission can occur.

281
M Y Elamin
MBBS, DTM&H, MCTM, MRCPI 1& 2
643. Initial treatment of spontaneous bacterial peritonitis is with broad-spectrum antibiotics such as
cefotaxime. Quinolones may be offered as prophylaxis in patients with cirrhosis and ascites in patients with
an ascitic fluid protein of 15g/L or less until the ascites has resolved. (NICE)
644. IV normal saline is the immediate treatment for patients presenting with dengue fever.
645. JC virus causes PML in immunocompromised patients especially when the CD4 count is below 100
cells/mm3.
646. Joint aspiration is essential in cases of suspected septic arthritis, in order to establish a microbiological
diagnosis that will guide appropriate treatment.
647. Keratitis due to varicella zoster virus (VZV) occurs in upto two-third of patients with herpes zoster
ophthalmicus and may subsequently lead to iridocyclitis and secondary glaucoma.
648. Klebsiella pneumonia characteristically affects the upper lobes, and can result in abscess formation.
649. Koplik's spots are seen only in measles; they are diagnostic.
650. Leptospirosis, or Weil's disease, is transmitted to man by animals, including rodents (rat urine and
faeces) and characteristed by jaundice, fever, headaches and myalgia
651. Lipodystrophy, lipoatrophy, and alterations in serum lipid values have been observed in patients with
human immunodeficiency virus (HIV) disease who are taking highly active antiretroviral therapy.
652. Maculopapular rash following introduction of amoxicillin is considered almost pathognomonic of
glandular fever.
653. Magnetic resonance imaging (MRI) is the gold standard method for diagnostic imaging of osteomyelitis.
654. Measles and mumps cause only mild maternal infection and do not pose a serious problem to the fetus.
655. Most patients with Giardia respond to oral metronidazole 250-400 mg tds for five days.
656. MRI is indicated urgently when there is suspicion of spinal cord compression.
657. Multiple myeloma is a cause of isolated B-cell immune deficiency
658. Multiple painless umbilicated papular lesions are typical of molluscum contagiosum and are caused by
pox virus.
659. Multiple ring-enhanced lesions are commonly seen in patients with cerebral toxoplasmosis and these
patients are normally immunosuppressed.
660. Multiple violaceous painless lesions are typical of Kaposi's sarcoma in Caucasians. This is associated
with HHV 8.
661. Mycobacterium avium causes disseminated infection in patients with advanced HIV, typically when the
CD4 count is less than 50 cells/mm3. This disseminated infection usually causes symptoms of fatigue,
weight loss and fevers.
662. Myocarditis can have features similar to cardiomyopathy and one of the most likely organisms is
Coxsackie B virus.
282
M Y Elamin
MBBS, DTM&H, MCTM, MRCPI 1& 2
663. Neisseria gonorrhoeae bacteria are highly fastidious Gram-negative cocci, that is, they require special
nutrients to survive. These cocci typically appear in pairs (diplococci).
664. Neisseria gonorrhoeae occurs in young adults and is often preceded by a migratory tendonitis or
arthritis. Gram stain is positive in 25% and culture positive in 50%.
665. Neisseria gonorrhoeae typically occurs in young adults. Presentation is either as a bacteraemic form
(classic triad of migratory polyarthritis, tenosynovitis, and dermatitis) which is usually polyarticular or a
septic arthritis.
666. Neutrophillia is indicative of bacterial infection.
667. Nevirapine can cause acute hepatitis and skin rash as a part of hypersensitive reaction especially when
the CD4 count is over 250 cells/ml in women and over 400 cells/ml in men. Nevirapine should not be
prescribed in those conditions.
668. NICE guidance on Prophylaxis against infective endocarditis (CG64) suggests that prophylaxis is not
required during dental procedures.
669. Non-bacterial thrombotic endocarditis (marantic endocarditis) is due to platelet-fibrin thrombi that are
prone to embolising.
670. Normal cerebrospinal fluid, a CSF lymphocytosis, an increased opening pressure, and a raised CSF
protein are typical of viral meningitis.
671. Norovirus is highly contagious, and person to person spread can occur from aerosols of projectile vomit
or faecal material.
672. Oesophageal candidiasis is a sign of advanced HIV disease.
673. Oesophageal candidiasis is an AIDS-defining illness but oral candidiasis is not.
674. One should think of common infections rather than any opportunistic infections in HIV patients with
good CD4 counts. (More than 400 cells/mm3 is not immunocompromised.)
675. Only untreated smear-positive pulmonary TB is likely to be infectious. Non-sputum producing patients
are non-infectious.
676. Oral hairy leucoplakia is a sign of immunodeficiency. It is due to reactivation of Epstein-Barr virus
infection.
677. Organ failure is a common finding, being as common as bleeding in DIC, and is likely to be due to fibrin
deposition within the organ.
678. Oseltamivir (Tamiflu) may be used in the prophylactic treatment of healthcare workers during flu
epidemics.
679. P. jiroveci can rarely present in extra pulmonary locations.
680. P24 antigen testing may detect HIV infection one to three weeks after the event but may need to be
retesteted if negative 6 weeks after the initial exposure.

283
M Y Elamin
MBBS, DTM&H, MCTM, MRCPI 1& 2
681. Patients should be vaccinated with an appropriate pneumococcal vaccination at latest two weeks prior to
surgery to allow the maximal humoral immune response.
682. Patients who undergo splenectomy should be vaccinated against pneumoncoccus and meningococcus,
ideally 2 weeks prior to surgery.
683. Patients with C3 deficiency, be it absolute, relative, genetically determined (autosomal dominant or
recessive), or due to properdin deficiency, are predisposed to recurrent infection with encapsulated proteins,
particularly N. meningitidis.
684. Patients with chronic obstructive airways disease (COPD) should not, in general, receive more than 24-
28% oxygen without arterial blood gas monitoring.
685. Patients with diphtheria classically present with severe pharyngitis and the presence of a
pseudomembrane over the tonsils.
686. Pharyngeal diphtheria presents with fever, sore throat, cervical lymphadenopathy and an adherent,
greyish pharyngeal membrane.
687. Pityriasis versicolor (also called tinea versicolor) is a skin lesion caused by a fungus called Malassezia
furfur. The treatment is topical selenium sulphide.
688. Pleural tuberculosis (TB) typically presents with pleurisy, pleuritic chest pain, cough, fever and a small
to moderate unilateral pleural effusion. Investigation should include a chest radiograph and sputum cultures
initially but the investigation most likely to yield a positive diagnosis is pleural biopsy.
689. Pleural tuberculosis is the most likely cause in an HIV-positive African man with a two-month history of
weight loss, chest symptoms and night sweats.
690. Pneumococcal meningitis is commoner in older patients.
691. Pneumococcal meningitis is the second commonest cause of bacterial meningitis (commonest in the
elderly) and is associated with the highest mortality (20%) and deafness which may occur in 50%
692. Pneumocystis jiroveci is a eukaryotic microorganism. In immunosuppressed patients, it can cause
pneumonia, which is most recognised in patients with AIDS but can also be seen in those with organ
transplants or when undergoing chemotherapy. A CD4 count of less than 200 is associated with significant
risk.
693. Pneumocystis jiroveci presents in immunocompromised patients with fever, dry cough, weight loss and
night sweats. Trimethoprim-sulfamethoxazole is the treatment of choice.
694. Pneumocystis jirovecii may be identified on microscopy after methenamine silver staining, giemsa
staining or fluorescence-tagged monoclonal antibody.
695. Pneumocystis jirovecii is confined to the alveolar space of the lung and produces debris and cysts in the
alveolar space with interstitial infiltration of lymphocytes and plasma cells.
696. Pneumonia caused by Legionella pneumophila is associated with hyponatraemia
697. Pneumothorax is a well-known complication of pneumocystis pneumonia (PCP)

284
M Y Elamin
MBBS, DTM&H, MCTM, MRCPI 1& 2
698. Post-splenectomy patients are at risk of infections due to capsulated organisms.
699. Postural drainage is the cornerstone to treating bronchiectasis and should be undertaken at least once per
day and more frequently during exacerbations.
700. Primary pulmonary tuberculosis is often asymptomatic consisting of primary complex.
701. Progressive multifocal leukoencephalopathy (PML) is a demyelinating disease seen in advanced
HIV/AIDS and caused by the JC virus.
702. Progressive multifocal leukoencephalopathy (PML), is a condition that can develop in
immunocompromised patients, such as those with AIDS.
703. Prosthetic valve endocarditis arising within two months of valve surgery is generally the result of
intraoperative contamination of the prosthesis or a bacteraemia postoperative complication.
704. Pulmonary tuberculosis in HIV-positive patients is a likley indicator of a diagnosis of AIDS.
705. Quinupristin and dalfopristin are a synergistic combination of a streptogramin A and B respectively.
706. Reactive arthritis classically presents with the combination of urethritis, conjunctivitis and arthritis, with
NSAIDs the initial treatment of choice.
707. Reverse transcriptase-PCR is used to amplify ribonucleic acid (RNA).
708. Roth's spots are the fundoscopic hallmark of bacterial endocarditis. They are white-centred retinal
haemorrhages, caused by capillary fragility. In addition to subacute bacterial endocarditis, they can also be
seen in leukaemia and retinal ischaemia.
709. Salmonella osteomyelitis is seen in patients with sickle cell anaemia.
710. Scabies causes itchiness in the skin even where there is no obvious lesion to be seen.
711. Scabies is treated with permethrin, which should be repeated after seven days, and all close contacts
should also be treated.
712. Scombrotoxin food poisoning is caused by the ingestion of foods that contain high levels of histamine;
linked to spoilage of foods , particularly tuna or mahi-mahi.
713. Severe PCP should be treated with high percentage oxygen and the drug of choice is high dose IV co-
trimoxazole (trimethoprim-sulphamethoxazole).
714. Shigella is a cause of profuse bloody diarrhoea.
715. Shigella is spread by faeco-oral contact via infected food or water, often during travel, especially
overcrowded facilites - or in long-term care facilities such as daycare centres or nursing homes.
716. Shock, fever, confusion, and rash are typical of staphylococcal toxic shock syndrome (TSS).
717. Signs and Symptoms of common sexually transmitted diseases.
718. Signs and Symptoms of Congenital Renal Pathologies

285
M Y Elamin
MBBS, DTM&H, MCTM, MRCPI 1& 2
719. Signs of raised intracranial pressure, normal glucose on CSF sampling and positive India ink test are all
strong pointers to the diagnosis of cryptococcal meningitis.
720. Small intestine biopsy is the best investigation to confirm Giardia as a diagnosis should repeated
cultures produce negative results.
721. Spontaneous bacterial peritonitis is diagnosed by ascitic fluid examination which reveals a neutrophil
count of >250/ml, and is typically caused by aerobic gram negative bacteria.
722. Spores of Saccharopolyspora rectivirgula found in moldy hay/straw are responsible for farmer's lung.
723. Staphylococcal pneumonia is known to cause cavitation and occurs with increased frequency after
influenza.
724. Staphylococci are skin organisms most commonly introduced during pacemaker insertion.
725. Streptococcus bovis is a normal commensal of the gastrointestinal (GI) tract. However, S. bovis
bacteraemia and endocarditis have a strong association with GI malignancy.
726. Streptococcus pneumoniae is one of the most common causes of bacterial meningitis in persons over 50
years of age
727. Streptococcus viridans is NOT associated with a worse prognosis of infective endocarditis.
728. Strongyloides is associated with a larval stage, the pulmonary migration of which can lead to symptoms
of bronchospasm with severe wheeze and shortness of breath.
729. Subacute bacterial endocarditis (Streptococcus viridans) has a better prognosis in cases of infective
endocarditis.
730. T cells recognise antigen only when presented by (self) MHC molecules on an antigen presenting cell.
731. The CD4 count can help direct the most likely diagnosis in HIV.
732. The confrimatory diagnostic test for PCP pneumonia is Sputum immunofluorescence.
733. The differential diagnosis of ring-enhancing lesions on CT in a patient with AIDS include:cerebral
toxoplasmosis, abcesses, metastases, and atypicalCNS lymphoma.
734. The drug of choice is trimethoprim-sulfamethoxazole for the initial therapy of nocardiosis.
735. The Duffy blood group on the red blood cells acts as a receptor for P. vivax.
736. The EEG in sporadic CJD may show significant abnormalities involving deep brain areas such as the
thalami.
737. The history is typical of cutaneous leishmaniasis and the diagnosis is confirmed with the finding of
amastigotes on a thin smear of dermal scrapings.
738. The incubation time for Salmonella enteritidis is 12-48 hours and the likely sources are poultry and
eggs.
739. The indications for varicella zoster immunoglobulin (VZIG).
286
M Y Elamin
MBBS, DTM&H, MCTM, MRCPI 1& 2
740. The Mantoux test measures the T cell-mediated immune response to TB antigen.
741. The MMR vaccine should not be administered to persons who are immunosuppressed.
742. The most appropriate antibiotic therapy in dog bites associated with cellulitis is co-amoxiclav.
743. The most appropriate treatment for legionellosis is clarithromycin.
744. The most likely organism as a cause of a vaginal discharge with few other symptoms, is Candida.
Vulvovaginal candidiasis (VVC) is a clinical diagnosis based on typical features supported by laboratory
confirmation of Candida sp. from a vaginal sample.
745. The pertussis vaccine is rarely associated with a hemiseizure-hemiplegia syndrome, probably related to
hyperpyrexia rather than direct neurological toxicity.
746. The presence of skipping and full thickness inflammation with granuloma formation is typical of
Crohn's disease.
747. The treatment of choice for chicken pox pneumonitis is initial IV aciclovir.
748. The treatment of diarrhoea is ceftriaxone 500 mg IM STAT with azithromycin 1 g PO STAT.
749. The vomiting-type outbreaks of Bacillus cereus food poisoning have generally been associated with rice
products.
750. There is a higher prevalence of HBV in the Far East which can lead to hepatoma development if left
untreated.
751. There is evidence to support the use of oseltamivir as a prophylactic agent against influenza
752. This patient had non-scarring alopecia due to invasion of hairs by dermatophytes, most commonly
Trichophyton tonsurans.
753. T-lymphocytes compose the majority of lymphocytes in plasma (60-80%).
754. Toxoplasmosis can present with fits in patients with AIDS
755. Treatment for community acquired pneumonia should be empirical until and organism is identified.
756. Treatment of chlamydia in a pregnant woman with erythromycin, amoxicillin or azithromycin first line
is advised prior to getting test results if strongly suspected clinically
757. Treatment of community-acquired pneumonia is guided by the CURB-65 severity score.
758. Treatment of G-6-PD deficiency revolves around avoidance of all known precipitating factors.
759. Trichomonas vaginalis causes itchy, frothy, greenish vaginal discharge.
760. Tryptase is a neutral protease stored in mast cell secretory granules. Elevated serum levels are consistent
with a diagnosis of anaphylaxis.
761. Ultrasound scan would confirm most moderate-sized to large liver abscesses and could guide a
diagnostic aspiration.

287
M Y Elamin
MBBS, DTM&H, MCTM, MRCPI 1& 2
762. Up to half of patients presenting with Streptococcus bovis endocarditis have colorectal tumours.
763. Urea above 7 mmol/l is an indicator of a worse prognosis associated with community-acquired
pneumonia.
764. Urological procedures, such as transurethral resection of the prostate (TURP), may cause a transient E.
faecalis bacteraemia.
765. Vaccinations should be given atleast two weeks before elective splenectomy.
766. Vancomycin-resistant enterococci may be found in healthy community volunteers not recently
hospitalised .
767. Varicella zoster infection causes herpes zoster and chickenpox.
768. Visceral leishmaniasis (kala-azar) is an endemic disease in several regions of India and sub-Saharan
Africa. It causes splenomegaly and fever.
769. Visceral leishmaniasis is fatal if left untreated
770. West Nile virus is a mosquito-borne zoonotic arbovirus belonging to the genus Flavivirus, which is
usually asymptomatic, but can results in a mild febrile syndrome or rarely a neuro-invasive disease.
771. With a recent history of travel to Nigeria, cerebral malaria caused by P. falciparum should be
considered.
772. With regards to Syphillis staging the absence of symptoms, late latent infection is more likely than early
latent infection.
773. With respect to HIV testing in the unconscious patient, it may be performed in the event that it is
considered to be in the best interests of the patient with respect to targeting appropriate intervention.
774. Yellow fever is an RNA virus transmitted by female mosquitoes, which results in an acute haemorrhagic
disease. It is found in tropical and subtropical regions of South America and Africa, but not in Asia.
775. Yellow fever vaccines should be avoided in patients with severe egg allergy.
776. Zidovudine causes increased pigmentation of the nails in black patients.
777. A positive rubella haemagglutination inhibition (HAI) combined with a negative rubella IgM is
consistent with early acute infection with rubella, previous vaccination, or previous rubella infection.
778. A urea raised more than 7 mmol/l in the context of normal renal function is a poor prognostic sign in
pneumonia.
779. Actinomycosis is caused by a filamentous, Gram positive bacterium and treatment is with
benzylpenicilin or surgical drainage.
780. Active Hepatitis B infection shows positive Anti-HBc antibody and negative HBs antigen and anti-HBs
antibody.
781. Acute CXR changes such as pneumonia can persist up to six weeks after resolution of symptoms
clinically.
288
M Y Elamin
MBBS, DTM&H, MCTM, MRCPI 1& 2
782. Acute HIV disease (seroconversion illness) occurs two to 12 weeks following exposure to HIV.
783. Acute onset liver and renal failure should prompt investigation for leptospirosis in patients recently
exposed to tropical water.
784. African trypanosomiasis has two recognised forms both of which are treated with Suramin
785. Aminoglycosides may impair neuromuscular transmission and should not be given to patients with
myasthenia gravis
786. Amphotericin is used to treat patients with Aspergillus fumigatus.
787. Anaerobic and gram negative organisms are common in abscesses following aspiration. An unconscious
patient is at risk of aspiration.
788. Antiretroviral therapies are frequent causes of anaemia in HIV-seropositive patients.
789. Arthritis, conjuctivitis and urethritis in a sexually active person is strongly suggestive of gonococcal
infection
790. Ascaris lumbricoides is a large roundworm and is the most common nematode parasite of humans with
patients being largely asymptomatic.
791. Aspergillus is a cause of a haemoptysis.
792. Bacillus cereus infection present with profuse vomiting which occurs approximately one to five hours
after eating recooked rice.
793. Bacterial vaginosis may be treated with oral metronidazole 400 mg tds seven days
794. Bacterial vaginosis microscopy shows clue cells and vaginal ph changes.
795. Blood film that shows ring forms within erythrocytes; some erythrocytes contain two to three parasites
per cell - are typical of falciparum; other forms of malaria seldom have more than one parasite per red cell.
796. Bronchiectasis associated with cystic fibrosis frequently results in recurrent infections
with Pseudomonas.
797. Bronchoalveolar lavage is the gold standard for diagnosis of PCP pneumonia.
798. Brucellosis is cause by a small Gram negative aerobic coccobacilli which presents with fever, arthralgia,
anorexia, weight loss and lethargy.
799. Cacnhroid is caused by Haemophilus ducreyi and presents with painful lymphadeonapthy and
erythematous papules.
800. Candida oesophagitis requires an extended course of oral or IV therapy as opposed to oropharyngeal
candidiasis.
801. Causative organism in Chagas' disease
802. CMV retinitis is commonly associated with HIV. It presents with a 'pizza' appearance
803. Coeliac may present with iron deficiency or B12/folate deficiency
289
M Y Elamin
MBBS, DTM&H, MCTM, MRCPI 1& 2
804. Common variable immunodeficiency (CVID) is the most prevalent of the primary immunodeficiency
diseases and patients are deficient in both IgG and IgA.
805. Complications of common conditions in the immunocompromised host.
806. Contact tracing should be offered following all confirmed sexual transmitted disease and can be done
discretely.
807. Crofloxacin is the first line treaatment for Salmonela Typhii.
808. Cryptococcal meningitis is an AIDS defining illness
809. Cryptococcus meningo-encephalitis is an AIDS-defining illness and should be treated with IV
amphotericin B.
810. Cryptococcus neoformans skin lesions occur in HIV patients with very low viral loads and are treated
with fluconazole.
811. CURB 65 score of 2 require oral dual antibiotic therapy.
812. Cutaneous TB usually occurs due to spread from an endogenous source with most cases presenting on
the face or neck.
813. CVC blood stream infection is an important cause of morbidity and mortality in critical care patients.
Considerable research has been undertaken to address this.
814. Cytomegalovirus (CMV) infection is responsible for a substantial fraction of the morbidity and mortality
following organ transplantation.
815. Decisions about PEP must be made by the clinicianin neddlestick injury if patients cannot consent to
HIV testing.
816. Dendritic ulcers are caused by herpes simplex virus
817. Dendritic ulcers are caused by herpes simplex virus. They are treated with aciclovir eye drops, which
should be continued for three days after the ulcer has healed.
818. Diagnosis and Management of TB and its complications
819. Diagnosis is by demonstration of C. neoformans in the cerebrospinal fluid, shown on an India ink stain:
the thick polysaccharide capsule is highlighted around the cell
820. Diagnosis of legionella pneumonia should be made via clinical history, x-ray and urinary antigen.
821. Diarrhoea in the returning traveller is a common presentation. It is important to recognise symptoms of
the most frequent culprits. Giardiasis presents with abdominal symptoms as listed above.
822. Different serovars of Chlamydia trachomatis cause either LGV or urethral chlamydia infection.
823. early recognition and intubation of acute epiglottitis is essential management
824. EBV is associated with oral hairy leukoplakia and occurs in HIV patients with CD4 counts between 200
and 500/mm3.
290
M Y Elamin
MBBS, DTM&H, MCTM, MRCPI 1& 2
825. efavirenz is a strong enzyme inducer and reduces the efficacy of the contraceptive pill increasing the risk
of pregnancy.
826. Endocarditis from IV use is most commonly associated with Staphylococcus aureus and treated with
Gentamicin and Vancomycin.
827. Epididymitis is extermely painful and risk factor for development include long term catheters,
underlying urinary tract pathology or recent instrumentation.
828. Epilepsy and alcohol and drug abuse are risk factors for aspiration pneumonia.
829. Erysipelas is a Streptococcus pyogenes infection of the deep dermis and subcutis.
830. Erythema multiforme typically presents with target lesions, cause may be multifactorial
831. Erythematous burrows in the web spaces of the fingers are almost pathognomonic of scabies. Treatment
is with permethrin cream.
832. Fenofibrate is used to treat isolated hypertriglyceridaemia, though it should not be prescribed routinely
for primary or secondary prevention of hypercholesterolaemia.
833. First choice treatment for Mycoplasma pneumoniae is with macrolide antibiotic (clarithromycin or
erythromycin); alternatives include tetracycline or doxycycline.
834. First line treatment for Legionella pneumonia is with macrolides or quinolones.
835. Fitz-Hugh-Curtis syndrome is a long term complication in women with untreated chlamydia.
836. G6PD deficiency is the commonest inherited genetic abnormality worldwide and treatment is based on
avoidance of precipitants.
837. Genital herpes can be transmitted when lesions aren't present by asymptotic shedding of the lesion.
838. Gentamicin is bactericidal.
839. Giarda is diagnosed on stool microscopy and is treated with oral metronidazole.
840. Gonorrhoea is a common STD, concomitant treatment with a penicillin and doxycycline can be useful
841. Group B Streptococcus has a predilection for cellulitis in diabetes
842. Gynaecomastia is one of the less common side effects of efavirenz together with the more common
neuropsychiatry side effects.
843. Haemolysis is associated with the presence of IgM antibodies (cold agglutinins) directed against the I
antigen of the erythrocyte membrane.
844. haemolytic anaemia is associated with atypical pneumonia
845. Haemophilus influenzae is found in increased frequency in smokers and patients with COPD as well as
patients on ventilation in ITU.
846. Haemoptysis in patients on immunosuppression patients should be investigated with CT thorax and
covered for fungal infection.
291
M Y Elamin
MBBS, DTM&H, MCTM, MRCPI 1& 2
847. Have a low threshold for testing for HIV, be aware of the clinical indicator diseases and that HIV testing
does not require extensive pre-test counselling.
848. Herpes zoster usually has a prodrome pain before the vesicles appear and will then have a dermatomal
distribution.
849. HIV patients who develop TB should have their HAART reviewed prior to starting TB drugs as multiple
interactions can occur.
850. HIV-positive patients are more prone to herpes zoster regardless of their CD4 count.
851. How to draw BC in suspected IE
852. HSV encephalitis presents with symptoms of fever, acute confusion and seizures with MRI showing
temporal lobe change.
853. Hydatid infection was previously endemic in sheep farming regions. Asymptomatic, calcified cystic
lesions in the liver are typical of hydatid cysts.
854. HZV distributes in a dermatomal fashion.
855. Identification of dilated oesophagus on CT
856. If the cryptococcal antigen ratios do not decrease or increase there is a high chance of relapse in
cryptococcal meningitis.
857. In cases of suspected TTP urgent blood film is required to look for red cell fragments.
858. In countries of low risk for drug resistance to anti-malarials the best prophylaxis is doxycyline.
859. In endocarditis isolation of the offending organism is crucial in order to tailor antibiotic therapy
860. In HIV infection, KS lesions occur most commonly on the face; however lesions may be widely
disseminated in skin, bronchial tree and gastrointestinal tract.
861. In low risk areas a combination of HBsAg positivity and features of acute hepatitis usually indicates
acute self-limiting hepatitis B infection.
862. In questions with heart pressure tables is important not to miss another non-cardiac diagnosis.
863. In respect to Anthrax infection cutaneous disease is the commonest form of the infection in humans.
864. Interstitial nephritis may be caused by numerous aetiologies such as: infection, autoimmunity or
glomerular disease as well as hypersensitivity to medicines.
865. Ireland is among the countries with the highest reported prevalence of sarcoidosis worldwide. Biopsy of
granuloma is diagnostic for the disease.
866. Isoiazid can cause a demyelinating peripheral neuropathy, particularly in patients with pre-existing risk
factors.
867. Isoniazid toxicity is best treated with high-dose pyridoxine and, where possible, stopping or reducing the
dose of isoniazid.
292
M Y Elamin
MBBS, DTM&H, MCTM, MRCPI 1& 2
868. Isoniazid-induced hepatitis occurs in ~1% of individuals and is much commoner in people more than
35-years-old (risk of hepatitis is less than 0.3% in patients under 20 years; 2-3% risk in individuals over 50
years).
869. Jaccoud's arthropathy may be seen in SLE, RA, Parkinson's disease and hypocompementaemic urticarial
vasculitis
870. Kaposi's sarcoma (KS) lesion is a red-purple mass arising from the conjunctiva. It is not limited to the
sclera (which could suggest a subconjunctival haemorrhage) and it protrudes, which also favours a
malignant growth.
871. Kaposis sarcoma is a tricky and rare diagnosis in primary care that presents as red to purplish macules
and raised papules/nodules anywhere on the skin or mucous membranes
872. Lactic acidosis, rhabdomyolysis and lipodystrophy are thought to be side-effect common to all
nucleoside analogues.
873. Lipodystrophy, lipoatrophy and alterations in serum lipid values have been observed in patients with
human immunodeficiency virus (HIV) disease taking highly active antiretroviral therapy. Elevated serum
lipid levels have been associated with premature coronary artery disease.
874. Listeria meningitis should always be considered in patients with meningitis associated with brain stem
involvement, and in immunosuppressed patients. The treatment of choice is gentamicin and ampicillin.
875. Low CURB-65 scores in penicillin allergic patients should be treated with oral doxycycline.
876. Macrocytosis can be caused by zidovudine.
877. Management of common respiratory conditions and their complications.
878. Management of endocarditis and its common pathogens
879. Management of HIV
880. Management of HIV in pregnancy.
881. meningococcal meningitis is caused by Neisseria meningitidis which is a Gram negative diplococci.
882. Meningococcal septicaemia presents with a rapidly progressive, non blanching rash, purpuric in nature.
Treatment is with cephalosporin.
883. Molluscum Contagiosum is caused by a DNA pox virus and are associated with HIV / AIDS
884. Monotherapy in a case of aspiration pneumonia is insufficient. Co Amoxiclav covers aerobes, whilst
metronidazole targets anaerobes
885. Most of the viruses spread by rodents and arthropods cause similar symptoms and signs, but the case
above is dengue fever which is diagnosed by dengue fever serology. Dengue fever has an incubation period
of two to seven days.
886. Multiple ring enhancing lesions on imaging of the brain in an immunosuppressed patient may indicate
cerebral toxoplasmosis.

293
M Y Elamin
MBBS, DTM&H, MCTM, MRCPI 1& 2
887. Mycoplamsa Pneumonia can present with haemolytic anaemia
888. NAAT has sensitivities of around 90-95%. It can also run many samples easily.
889. Necrotising fasciitis is a rapidly fatal condition that requires emergency treatment.
890. Neisseria meningitidis are Gram negative cocci.
891. Non-invasive ventilation is an open circuit system.
892. Note ethambutol induced optic neuropathy is a well known complication resulting from the use of
ethambutol. Dimness of vision is the main symptom.
893. Oseltamivir is the oral treatment of choice for H1N1 infection.
894. P. jiroveci is occasionally found in induced sputum, however bronchoalveolar lavage increases the
chance of diagnosis.
895. Pancrelipase are a collection of pancreatic enzymes used in treatment of chronic pancreatitis
896. Papules with central umbilication are associated with molluscum contagiosum.
897. Parvovirus B19 is caused by HHV 6 and can result in a small joint arthropathy.
898. Patients co-infected with HIV and hepatitis B can easily have both viruses treated with the right
antiretroviral regime and all patients with HIV should be immunised against Hepatitis B.
899. Patients on antibiotics who develop diarrhoea should be investigated for C. diff.
900. Patients with a normal CD4 count can have a mild pneumonia treated in the community or as an
outpatient.
901. patients with an intermediate risk for a viral haemorrhagic fever should be isolated in a side room
immediately.
902. Patients with HIV have a 4 fold increased risk of cervical dykaryosis and it is currently recommended
that they are monitored with a yearly cervical smear.
903. Patients with HIV have complex vaccination needs and live vaccines are contraindicated in patients with
a CD4 count of <200 cells/µL.
904. Patients with neutropaenic sepsis with no obvious source should be initiated on broad spectrum
antibiotics immediately.
905. Patients with sickle cell disease have a predisposition to develop osteomyelitis due to Salmonella
species.
906. PCP is an opportunistic infection which affects patients with HIV with significantly low CD4 counts.
907. PCP is more common in patients with CD4 count less than 200 and diagnosis is made on induced
sputum or BAL.
908. PCP pneumonia characteristically causes oxygen desaturation during exercise.

294
M Y Elamin
MBBS, DTM&H, MCTM, MRCPI 1& 2
909. Pharyngeal diphtheria presents with fever, sore throat, cervical lymphadenopathy and an adherent,
greyish pharyngeal membrane.
910. Praziquantel is the treatment of choice for all Schistosoma species.
911. Progressive multifocal leukoencephalopathy (PML) is a demyelinating disease of the central nervous
system characterised by widespread lesions and associated with both HIV-1 and HIV-2.
912. Psueudomembranous colitis is a potential complication of clostridium difficile infection
913. Q fever is a rickettsial zoonotic disease caused by Coxiella burnetii which is usually self limiting.
914. Rapid cognitive decline in a young person with myoclonus is strongly suggestive of Creutzfeldt-Jakob
disease (CJD) which shows a characteristic EEG patern.
915. RCT is the gold standard for evaluation of efficacy for therapeutic and preventative measures.
916. Rifampicin may be expected to reduce the efficacy of the oral contraceptive through liver enzyme
induction.
917. Ring enhancing lesions may be associated with: cerebral mets, toxoplamosis, histioplasmosis, brain
abscess
918. Risk factors for ATLS include high grade tumours and large tumour burden. Treatment is with
raspuricase.
919. Salient features of pneumocystis jirovecii pneumonia (PCP) are several days/weeks of increasing
dyspnoea, dry cough, marked oxygen desaturation with exercise.
920. Sarcoidosis is a disease of unknown cause characterised by the presence histologically of non-caseating
granulomata.
921. Scabies is a highly contagious disease caused by the mite Sarcoptes scabiei which causes an intense
pruritis and can be treated with permethrin cream
922. Scabies should be treated with topical Malathion as well as hot washes of all bed clothes and linen to
prevent reinfection.
923. Schistosomiasis is the third most devastating tropical disease in the world therefore consider in patients
who have travelled to endemic areas who have fever, lethargy and myalgia.
924. Schuffner's nodes are characteristic on a thin blood film of P. Viva malaria
925. Secondary cold agglutinin disease typically presents with anaemia and haemoglobinuria due to
intravascular haemolysis two to three weeks following infection
926. Sepsis may be complicated by disseminated intravascular coagulation
927. Septic shock presents with hypothermia, tachycardia and hypotension, and often with decreased
consciousness in the elderly.
928. sexually acquired reactive arthritis (SARA) is likely an immune mediated phenomenon linked to the
immune response to uro-genital micro-organisms.
295
M Y Elamin
MBBS, DTM&H, MCTM, MRCPI 1& 2
929. Signs and Symptoms of common sexually transmitted diseases
930. Signs and Symptoms of Congenital Renal Pathologies
931. Staphylococcus aureus is Gram positive, in irregular clusters, produces catalase and coagulase.
932. Staphylococcus is the most common organism to cause post operative infection of a prosthetic joint
933. Streptococcal infection can precede the onset of guttate psoriasis, which presents with an itchy rash and
sore throat.
934. Streptococcus bovis has a strong association with underlying colonic carcinoma and patients should be
promptly investigated by colonoscopy.
935. Symptoms of acute HIV infection include fever, sore throat, lymphadenopathy and a widespread
macular rash.
936. Symptoms of cutaneous worm infections include rash and itching. Treatment may be topical
(thiabendzole) or oral (albendazole)
937. TB meningitis should be treated aggressively with antibiotics and dexemethasone.
938. Tetanus is not a live vaccine and can be given in HIV patients not on treatment.
939. Tetracyclines can cause idiopathic intracranial hypertension
940. The correct management of DGI is IV ceftriaxone 1 g od for seven days followed by oral abx.
941. The importance of early recognition of clostridium difficile colitis/diarrhoea and the need to start
treatment urgently
942. The metabolism of corticosteroids is increased by rifampicin and should therefore lead to increased
steroid doses in patients on them long term.
943. The most common causes of unconjugated hyperbilirubinemia are bilirubin overproduction, Gilbert
syndrome, and neonatal jaundice.
944. The most common clinical signs prior to a cardiac arrest are respiratory distress (RR >25/min) and
altered mental state.
945. The most important treatment for neutropaenic sepsis is IV broad spectrum antibiotics.
946. The MRI in HSV encephalitis demonstrates a temporoparietal dense lesion
947. The order of removing personal protective equipment is important to reduce contamination.
948. The resistant organisms produce PBPs that have a low affinity for binding beta-lactamase antibiotics.
Other organisms which do the same are Pneumococci and Enterococci.
949. The treatment of choice in acquired methaemoglobinaemia is a 1% solution of methylene blue.
950. The usual causative organism of acute epiglottitis is Haemophilus influenzae type b.
951. The yellow fever vaccine is safe, especially if the patient's CD4+ is above 200 cells/mm3.
296
M Y Elamin
MBBS, DTM&H, MCTM, MRCPI 1& 2
952. This patient has septic shock from cellulitis, intravenous antibiotics must be used initially and the patient
re-assessed after 24 hours
953. Threadworm infection is common in children and institutions
954. Toxocara canis a parasite found globally, carried by dogs.
955. Toxoplasmosis in association with HIV often involves focal neurological deficits with signs of raised
intracranial pressure. MRI shows ring enhancing lesions.
956. Toxoplasmosis is the commonest space occupying lesion in HIV/AIDS.
957. Treatment of erysipelas is with the penicillins, but macrolides and cephalosporins can also be used.
Chloramphenicol use is limited due to risk of aplastic anaemia.
958. Treatment of PCP includes co trimoxazole, or iv pentamidine, or clindamycin & primaquine. Steroids
should be started in conjunction.
959. Treponema pallidum haemagglutination test (TPHA) uses specific treponemal antigens and therefore are
more specific.
960. Trichomonas infection is a curable sexually transmitted infection, caused by a protozoon called
Trichomonas vaginalis. TV can infect the vagina, urethra (water passage), and underneath the foreskin.
961. Tropical eosinophilia is an allergic reaction to microfilaria of Wuchereria bancrofti and presents with
myalgia, fatigue, weight loss, cough and dyspnoea with wheeze.
962. TSS is caused by toxins produced by S. aureus, the patient deteriorates rapidly and presents in shock a
differential for this may be meningitis.
963. Tuberculosis is common in intravenous drug users.
964. Varicella pneumonia occurs in up to 20% of adults with chickenpox, appearing three to five days into
the course of the illness. In adults with pneumonitis, treatment with aciclovir is warranted.
965. West Nile encephalitis may present with seizures, reduced level of consciousness and a flaccid paralysis
and requires interferon-alpha treatment urgently.
966. Where Ebola is suspected individuals should be isolated in a side room immediately.

297
M Y Elamin
MBBS, DTM&H, MCTM, MRCPI 1& 2
NEPHROLOGY
1. A 10-year-old boy presents after passing 'brown' urine. Two weeks ago he had a severe sore throat. On examination
his blood pressure is high for his age and there is periorbital oedema - Streptococcus pyogenes
2. A 15-year-old presents with nephrotic syndrome. Their blood pressure and renal blood tests are normal - minimal
change disease
3. A 25-year-old man presents with visible haematuria. He also complains of having a bad sore throat at the current
time - IgA nephropathy
4. A 30-year-old man presents with haematuria. He is already known to have bilateral sensorineural hearing loss
and visual problems - Alport's syndrome
5. A 30-year-old man presents with haemoptysis and renal failure. A renal biopsy shows linear IgG deposits along
the basement membrane - Goodpasture's syndrome
6. A 30-year-old man who is known to have HIV presents with nephrotic syndrome - focal segmental
glomerulosclerosis
7. A 30-year-old woman with a history of migraine which she treats with paracetamol and aspirin develops
haematuria, flank pain and renal impairment - renal papillary necrosis
8. A 35-year-old man presents with severe, episodic loin pain which radiates round to the front. He has blood+++,
leucocytes ++ on the urine dipstick - renal colic
9. A 35-year-old man who has a history of hypertension and subarachnoid haemorrhage presents with recurrent
UTIs and episodic haematuria - autosomal dominant polycystic kidney disease
10. A 3-year-old child is noted to have haematuria. On examination there is a mass in the left loin - Wilms' tumour
11. A 40-year-old man presents with nephrotic syndrome. Renal biopsy demonstrates sub-epithelial immune complex
deposition in the glomerulus - membranous glomerulonephritis
12. A 45-year-old woman with nephrotic syndrome is noted to have marked loss of subcutaneous tissue from the face
- membranoproliferative glomerulonephritis (type II)
13. A 5-year-old boy develops a palpable, erythematous rash on the extensor surfaces of the arms and legs associated
with abdominal pain. He is later found to have haematuria and mild renal failure - Henoch-Schonlein purpura
14. A 5-year-old child develops acute renal failure shortly after being admitted with bloody diarrhoea and fever. Bloods
show renal failure, thrombocytopaenia and anaemia - haemolytic uraemic syndrome
15. A 6-year-old boy presents with passing dark urine 10 days after having had a sore throat. On examination he has
periorbital oedema - post-streptococcal glomerulonephritis
16. A newborn baby is noted to have large palpable masses in the loins and dysmorphic features consistent with
Potter's syndrome. Further investigations show renal impairment and liver fibrosis - autosomal recessive
polycystic kidney disease
17. A patient presents with polyuria and polydipsia. Their blood glucose is normal and the urine osmolality is 220
mOsm/kg - diabetes insipidus
18. Acetazolamide - normal anion gap metabolic acidosis
298
M Y Elamin
MBBS, DTM&H, MCTM, MRCPI 1& 2
19. Acute interstitial nephritis - a recent history of penicillin use
20. Acute interstitial nephritis - eosinophilia
21. Acute tubular necrosis - brown granular casts in urine
22. Acute tubular necrosis - fractional sodium excretion > 1%
23. Acute tubular necrosis - normal serum urea:creatinine ratio
24. Acute tubular necrosis - poor response to fluid challenge
25. Acute tubular necrosis - urine osmolality < 350 mOsm/kg
26. Acute tubular necrosis - urine sodium > 40 mmol/L
27. Addison's disease - normal anion gap metabolic acidosis
28. Alcohol poisoning - raised anion gap metabolic acidosis
29. Alport's syndrome - abnormality in type IV collagen
30. Alport's syndrome - longitudinal splitting of the lamina densa of the glomerular basement membrane, resulting
in a 'basket-weave' appearance on electron microscopy
31. Alport's syndrome - nephritits
32. Alport's syndrome - sensorineural hearing loss
33. Ammonium chloride injection - normal anion gap metabolic acidosis
34. Amyloidosis - apple-green birefringence on Congo red staining
35. An elderly male smoker presents with visible haematuria associated with loin pain. On examination a loin mass
can be found on the same side - renal cell carcinoma
36. Autosomal dominant polycystic kidney disease type 1 - chromosome 16
37. Autosomal dominant polycystic kidney disease type 2 - chromosome 4
38. Chronic pyelonephritis - a history of recurrent renal stones
39. Chronic pyelonephritis - a history of vesicoureteral reflux
40. Chronic pyelonephritis - corticomedullary scarring with atrophy of tubules
41. Cystinuria - cyanide-nitroprusside test
42. Diabetes insipidus - water deprivation test
43. Diabetic ketoacidosis - raised anion gap metabolic acidosis
44. Diarrhoea - normal anion gap metabolic acidosis
45. Diffuse proliferative glomerulonephritis - granular appearance on immunofluorescence
46. Diffuse proliferative glomerulonephritis - subendothelial immune complex deposits
47. Diffuse proliferative glomerulonephritis - 'wire-loop' appearance on light microscopy
48. Episodic haematuria at time of sore throats, normal complement levels - IgA nephropathy
49. Fanconi syndrome - generalised disorder of renal tubular transport in the proximal convoluted tubule
50. Fistula - normal anion gap metabolic acidosis
51. Goodpasture's syndrome - anti-glomerular basement membrane (anti-GBM) antibodies
52. Goodpasture's syndrome - linear IgG deposits along basement membrane
299
M Y Elamin
MBBS, DTM&H, MCTM, MRCPI 1& 2
53. Henoch-Schonlein purpura - an association with IgA nephropathy
54. Henoch-Schonlein purpura - IgA mediated small vessel vasculitis
55. Henoch-Schonlein purpura - palpable purpuric rash over buttocks
56. Hydrocele - failure of the processus vaginalis to close
57. Hypoxia - raised anion gap metabolic acidosis
58. IgA nephropathy - a recent history of a respiratory tract infection
59. IgA nephropathy - mesangial hypercellularity, positive immunofluorescence for IgA & C3
60. Membranoproliferative glomerulonephritis (type I) - subendothelial immune complex deposits with 'tram-
track' appearance on electron microscopy
61. Membranoproliferative glomerulonephritis (type II) - C3b nephritic factor
62. Membranoproliferative glomerulonephritis (type II) - intramembranous immune complex deposits with 'dense
deposits' on electron microscopy
63. Membranous glomerulonephritis - thickened basement membrane with subepithelial electron dense deposits
creating a 'spike and dome' appearance
64. Minimal change disease - effacement of foot processes on electron microscopy
65. Minimal change disease - normal findings on light microscopy
66. Nephritis, sensorineural deafness - Alport's syndrome
67. Post-streptococcal glomerulonephritis - granular appearance on immunofluorescence
68. Post-streptococcal glomerulonephritis - immune complex (IgG, IgM and C3) deposition in the glomeruli
69. Post-streptococcal glomerulonephritis - subepithelial 'humps' caused by lumpy immune complex deposits on
electron microscopy
70. Prerenal disease - fractional sodium excretion < 1%
71. Prerenal disease - good response to fluid challenge
72. Prerenal disease - normal urine sediment
73. Prerenal disease - raised serum urea:creatinine ratio
74. Prerenal disease - urine osmolality > 500 mOsm/kg
75. Prerenal disease - urine sodium < 20 mmol/L
76. Prophylaxis against calcium renal stones , treatment of choice: a thiazide
77. Purpuric rash over buttocks, abdominal pain, arthralgia, haematuria - Henoch-Schonlein purpura
78. Renal cell cancer - polycythaemia secondary to erythropoietin secretion
79. Renal colic - non-contrast CT scan
80. Renal failure - raised anion gap metabolic acidosis
81. Renal papillary necrosis - a history of long-term NSAID use
82. Renal papillary necrosis - a history of sickle cell anaemia
83. Renal tubular acidosis - normal anion gap metabolic acidosis
84. Salicylate poisoning - raised anion gap metabolic acidosis
300
M Y Elamin
MBBS, DTM&H, MCTM, MRCPI 1& 2
85. Schistosomiasis may cause squamous cell carcinoma of the bladder cancer
86. Shock - raised anion gap metabolic acidosis
87. Sore throat two weeks before symptoms start, proteinuria > haematuria, low complement levels - post-
streptococcal glomerulonephritis
88. Spironolactone - aldosterone antagonist
89. Spironolactone may cause gynaecomastia
90. Spironolactone may cause hyperkalaemia
91. Type 2 (proximal) renal tubular acidosis, rickets/osteomalacia, polyuria - Fanconi syndrome
92. Ureterosigmoidostomy - normal anion gap metabolic acidosis
93. The mechanism by which lithium leads to diabetes insipidus, reduced GSK3 beta signalling, is well established.
94. Sildenafil and other phosphodiesterase inhibitors may substantially reduce urine output in nephrogenic diabetes
insipidus.
95. Goodpasture's syndrome
a. IgG deposits on renal biopsy
b. Anti-GBM antibodies
96. Stag-horn calculi
a. Composed of Struvite (ammonium magnesium phosphate, triple phosphate)
b. Form in alkaline urine (ammonia producing bacteria such as Ureaplasma urealyticum and Proteus therefore
predispose)
97. Proteus mirabilis infection predisposes to struvite kidney stones
98. AA amyloidosis, formerly known as secondary or reactive systemic amyloidosis, is a complication of chronic
inflammatory and infective diseases in which there is a sustained acute-phase response with overproduction of
serum amyloid A (SAA) protein, a very sensitive and dynamic major acute-phase protein.
99. AA amyloidosis can occur due to chronic HIV infection and can result in nephrotic syndrome. However, it is
exceedingly rare in comparison to FSGS. Furthermore, amyloid deposition usually occurs in other organs such as
the gastrointestinal tract, heart, and nerves.
100. A common complication of plasma exchange is hypocalcaemia
101. ADPKD is associated with hepatomegaly (due to hepatic cysts)
102. ADPKD is associated with mitral valve prolaspe
103. ADPKD type 1 = chromosome 16 = 85% of cases
104. ADPKD type 2 = chromosome 4 = 15% of cases
105. ADPKD is associated with mitral valve prolapse
106. The clinical features of pulmonary haemorrhage, and nephritic syndrome, are highly suggestive of
Goodpasture's syndrome. Anti-GBM antibodies are most specific for this condition.
107. Alcohol bingeing can lead to ADH suppression in the posterior pituitary gland subsequently leading to polyuria
108. Aldosterone antagonists acts on the cortical collecting ducts as a diuretic
301
M Y Elamin
MBBS, DTM&H, MCTM, MRCPI 1& 2
109. Alfacalcidol is used as a vitamin D supplement in end-stage renal disease because it does not require activation
in the kidneys
110. Alport's syndrome - sensorineural hearing loss
111. Alport's syndrome - type IV collagen defect
112. Alport's syndrome - X-linked dominant (in the majority)
113. Amyloidosis biopsy findings - Congo red stain shows apple-green birefringence under polarised light
114. Antimuscarinic drugs are useful in patients with an overactive bladder
115. Arteriovenous fistulas are the preferred method of access for haemodialysis
116. Ascites - use spironolactone
117. ATN or prerenal uraemia? In prerenal uraemia think of the kidneys holding on to sodium to preserve volume
118. Bicalutamide is an androgen recept blocker
119. Calciphylaxis lesion are intensely painful, purpuric patches with an area of black necrotic tissue that may form
bullae, ulcerate, and leave a hard, firm eschar
120. Calcium resonium results in removal of potassium from the body, rather than shifting potassium between fluid
compartments in the short-term
121. Chronic Kidney Disease often leads to anaemia due to reduced levels of erythropoietin
122. CKD on haemodialysis - most likely cause of death is IHD
123. CKD: only diagnose stages 1 & 2 if supporting evidence to accompany eGFR
124. CMV infection is one of the most important in transplant receptors - clinically is characterized by fever, deranged
transaminases, leukopenia and thrombocytopenia. Diagnosed by PCR and treated with ganciclovir
125. Coagulase-negative Staphylococcus is the most common cause of peritonitis secondary to peritoneal dialysis
126. Consider fibromuscular dysplasia in young female patients who develop AKI after the initiation of an ACE
inhibitor
127. Contrast-induced nephropathy occurs 2 -5 days after administration
128. Cytomegalovirus is the most common and important viral infection in solid organ transplant recipients
129. Diabetes insipidus in patients taking lithium mechanism: lithium desensitizes the kidney's ability to respond to
ADH in the collecting ducts
130. Diffuse proliferative glomerulonephritis is the most common and severe form of renal disease in SLE patients
131. Diffuse proliferative glomerulonephritis, causes:
a. Post-streptococcal
b. SLE
132. eGFR variables - CAGE - Creatinine, Age, Gender, Ethnicity
133. Eplerenone can be used in patients with troublesome gynaecomastia on spironolactone
134. Fanconi syndrome is a reabsorptive defect in PCT where there is increased excretion of nearly all amino acids,
glucose, bicarbonate and phosphate
135. Finasteride treatment of BPH may take 6 months before results are seen
302
M Y Elamin
MBBS, DTM&H, MCTM, MRCPI 1& 2
136. Flash pulmonary oedema, U&Es worse on ACE inhibitor, asymmetrical kidneys → renal artery stenosis - do
MR angiography
137. Gentamicin causes an intrinsic AKI
138. Guidelines continue to recommend the use of IM diclofenac in the acute management of renal colic
139. Haemolytic uraemic syndrome - classically caused by E coli 0157:H7
140. HCG is associated with testicular seminomas
141. Heroin use is a risk factor for focal segmental glomerulosclerosis
142. HIV-associated nephropathy (HIVAN) causes collapsing FSGS and usually presents as nephrotic syndrome
143. Idiopathic membranous glomerulonephritis is related to anti-phospholipase A2 antibodies
144. In a patient with hypercalciuria and renal stones, calcium excretion and stone formation can be decreased by the
use of thiazide diuretics
145. In AKI, hyperkalaeamia which is refractory to medical management is an indicator for renal replacement therapy
146. Membranoproliferative glomerulonephritis (mesangiocapillary)
147. Membranous nephropathy is frequently associated with malignancy
148. Mesangiocapillary glomerulonephritis (membranoproliferative)
a. Type 1: cryoglobulinaemia, hepatitis C
b. Type 2: partial lipodystrophy
149. Micturating cystography is the investigation of choice for reflux nephropathy
150. Minimal change disease is the most common cause of nephrotic syndrome in a child
151. Minimal change glomerulonephritis - prednisolone
152. Nephrogenic diabetes insipidus may be caused genetic mutations:
a. The more common form affects the vasopression (ADH) receptor
b. The less common form results from a mutation in the gene that encodes the aquaporin 2 channel
153. The mechanism by which lithium leads to diabetes insipidus, reduced glycogen synthase kinase type 3 beta
(GSK3 beta) signalling, is well established, resulting in dysfunction of the aquaporin-2 water channel.
154. Sildenafil and other phosphodiesterase inhibitors may substantially reduce urine output in congenital x-linked
nephrogenic diabetes insipidus. congenital nephrogenic DI is generally unresponsive to exogenous ADH.
155. Nephrotic syndrome - malignancies cause membranous glomerulonephritis
156. Nephrotic syndrome is associated with a hypercoagulable state due to loss of antithrombin III via the kidneys
157. Non-seminoma germ cell testicular tumours (e.g. Teratomas) are associated with raised HCG and AFP
158. NSAIDS and ACE-inhibitors/ARB cause prerenal acute kidney injury by decreasing the glomerular filtration
159. NSAIDS should be stopped in AKI except aspirin at cardio-protective dose
160. Patients who are high-risk for contrast-induced nephropathy should have metformin withheld for a minimum of
48 hours and until the renal function has been shown to be normal
161. Patients who have received an organ transplant are at risk of skin cancer (particularly squamous cell carcinoma)
due to long-term use of immunosuppressants
303
M Y Elamin
MBBS, DTM&H, MCTM, MRCPI 1& 2
162. Prevention of contrast-induced nephropathy: volume expansion with 0.9% saline
163. PSGN develops 1-2 weeks after URTI. IgA nephropathy develops 1-2 days after URTI
164. Rapidly progressive glomerulonephritis, causes:
a. Goodpasture's
b. ANCA positive vasculitis
165. Renal cell carcinoma can cause liver dysfunction in particular cholestasis and hepatosplenomegaly
166. Renal cell carcinoma can metastasise to the lungs, and remains an important differential in the setting of
hypertension, hypercalcaemia and haematuria
167. Renal stones on x-ray
a. Cystine stones: semi-opaque
b. Urate + xanthine stones: radio-lucent
168. Renal transplant HLA matching - DR is the most important
169. Renal tubular acidosis causes a normal anion gap
170. Rhabdomyolysis can cause parenchymal acute kidney injury and is characterised by elevated plasma creatine
kinase (CK)
171. Rhabdomyolysis should always be considered in the setting of lactic acidosis, hyperkalaemia and features of acute
tubular necrosis
172. Stag-horn calculi are composed of struvite and form in alkaline urine (ammonia producing bacteria therefore
predispose)
173. Sterile pyuria and white cell casts in the setting of rash and fever should raise the suspicion of acute interstitial
nephritis, which is commonly due to antibiotic therapy
174. The mainstay of rhabdomyolysis treatment is rapid IV fluid rehydration
175. The presence of upper respiratory tract signs points towards granulomatosis with polyangiitis in a patient with
rapidly progressive glomerulonephritis
176. The time taken for an arteriovenous fistula to develop is 6 to 8 weeks
177. Tolvaptan is a vasopressin receptor 2 antagonist
178. Tremor is a common tacrolimus‐associated side effect
179. Ultrasound is the screening test for adult polycystic kidney disease
180. Ureterosigmoidostomy - normal anion gap metabolic acidosis
181. Uric acid nephrolithiasis are radiolucent, requiring ultrasonography or CT KUB (without contrast)
182. Urine dip can be used to differentiate acute tubular necrosis from acute interstitial nephritis in AKI
183. Urine output of < 0.5 ml/kg/hr over 6 consecutive hours constitutes an acute kidney injury
184. Use of 0.9% Sodium Chloride for fluid therapy in patients requiring large volumes = risk of hyperchloraemic
metabolic acidosis
185. When prescribing fluids, the potassium requirement per day is 1 mmol/kg/day
186. With MAHA, once TTP and HUS are excluded, atypical HUS needs to be considered and treated
304
M Y Elamin
MBBS, DTM&H, MCTM, MRCPI 1& 2
187. Young female, hypertension and asymmetric kidneys → fibromuscular dysplasia
188. " Recombinant human erythropoeitin is effective in anaemia of chronic renal failure, in patients receiving
platinum-based chemotherapy and the prevention of anaemia in premature babies with low birth weight. "
189. 0.9% saline represents the physiological concentration.
190. A high serum urate concentration can lead to gout. As such, patients with high levels should be given
lifestyle advice about reducing intake of substances containing high purines (including some alcohol and
red meat).
191. A long history of urinary tract infections with underlying reflux scarring would predispose patients to
developing pyelonephritis in pregnancy.
192. A past history of excess alcohol use is associated with IgA nephropathy.
193. A ten minute period of hyperventilation will normally be expected to lead to an increased rate of
bicarbonate excretion in urine in correctly functioning kidneys.
194. A transferrin saturation less than 20% is used in conjunction with serum ferritin to assess need for iron
replacement in the management of anaemia in chronic kidney disease.
195. Abdominal ultrasound has a sensitivity approaching 100% for autosomal dominant polycystic kidney
disease (ADPKD) patients above 20 years of age.
196. Accelerated hypertension is not a complication of nephrotic syndrome.
197. ACE inhibitors are contraindicated in pregnancy and blood pressure managment is with with labetalol,
methyldopa or nifedipine,.
198. ACE inhibitors should be held in patients with hypovolaemic kidney injury
199. Acetazolamide is a carbonic anhydrase inhibitor and has a weak diuretic activity.
200. Acute interstitial nephritis is characterised by interstitial inflammation and oedema. Left untreated this
results in interstitial fibrosis. A definitive diagnosis is established by renal biopsy, although eosinophiluria
and gallium 67 scanning are also suggestive.
201. Acutely unwell patients with sepsis should be treated with IV insulin infusion initially
202. ADH causes insertion of aquaporin 2 channels in the collecting duct, allowing increased water
reabsorption.
203. ADH is a nonapeptide released from the posterior pituitary.
204. Administration of albumin (1.5 g per kilogram of body weight at diagnosis and 1.0 g per kilogram 48 hours
later), in addition to antibiotics, has been shown in randomised controlled trial to markedly reduce the risk
of hepatorenal syndrome.
205. ADPKD is autosomal dominant and offspring of a parent with the disease have a 50% chance of having the
condition
206. Adult polycystic kidney disease is associated with polycythaemia.
305
M Y Elamin
MBBS, DTM&H, MCTM, MRCPI 1& 2
207. Age-related muscle loss in the elderly is due to a decline in glomerular filtration rate and plasma flow rate.
208. Alport's syndrome is a disorder of type 4 collagen assembly and is inherited as an X-linked disorder in 85%
of cases.
209. Alport's syndrome is associated with sensorineural deafness.
210. Alport's syndrome is typically associated with sensorineural hearing loss and progressive renal failure. It is
an X-linked condition.
211. Ammonium magnesium phosphate is the most likely composition for staghorn calculus.
212. Amyloid deposits composed of ß2 microglobulin as the major constituent protein are mainly localised in
joints and periarticular bone and lead to destructive arthropathy which tends to develop five to ten years
after the initiation of dialysis.
213. An elevated serum creatinine can be due to due to rhabdomyolysis, especially in patients found
unconscious, hypothermic, and likely to have sustained muscle injury or who have been lying on the floor
for a prolonger period of time.
214. An ultrasound of the renal tract may not be appropriate for patients under 20, given that cysts may not
become apparent until the age of 20.
215. Anaemia associated with diabetes is often due to reduced erythropoietin release due to chronic renal failure.
216. Anaemia in chronic kidney disease is very common, and as a result haemoglobin should be monitored at
least annually.
217. Anaemia is a common cause of fatigue in dialysis patients, and can be treated with erythropoietin.
218. Anaemia is common in chronic kidney disease patients, and serum ferritin and transferrin saturation should
be checked to ensure the patient is iron replete prior to the initiation of recombinant erythropoiesis-
stimulating agents.
219. Anion gap is calculated as (Na + K) - (Cl + HCO3).
220. Anticholinergic side effects, including acute urinary retention, are common with amitriptyline.
221. Antidiuretic hormone is synthesised in the hypothalamus and released from the posterior pituitary.
222. Approximately 50% of subjects with focal segmental glomerulosclerosis (FSGS) do not respond to steroid
therapy but angiotensin-converting enzyme (ACE) inhibitors are a recognised strategy to slow the
progression of renal disease.
223. Approximately a drop of 1 mmol in serum potassium is equivalent to a total body loss of 200-300 mmol of
potassium.
224. Asymmetric reduction in renal size should lead you to suspect renal artery stenosis, for which MR
angiography is the investigation of choice.
225. Bartter syndrome is autosomal recessive condition associated with hypokalaemia.
226. Both ACE inhibitor and angiotensin-receptor blocker are contraindicated in pregnancy.
306
M Y Elamin
MBBS, DTM&H, MCTM, MRCPI 1& 2
227. Both thiazide diuretics and potassium citrate can be used to reduce urinary excretion of calcium. Potassium
citrate is generally preferred as it has fewer side effects, and is therefore better tolerated.
228. C3 complement component is consumed during an acute attack of systemic lupus erythematosus and is
therefore, the best marker of acute disease activity.
229. Calcium and infection stones are radio-opaque, cystine stones are weakly radio-opaque and urate stones are
radiolucent.
230. Calcium oxalate stones are the most frequent in the general population.
231. Casts containing erythrocytes (red cell casts) are an indication of renal bleeding and are typically found
when there is acute glomerular inflammation caused by glomerulonephritis or vasculitis.
232. Catheter removal is strongly recommended in Staphylococcus aureus bloodstream infection given the high
risk of recurrence in these patients.
233. CD4+ CD25+ are thought to be the most important T regulatory cell population. They are thought to play
an important role in regulating immune responses after invading organisms have been tackled and
preventing the development of autoimmunity.
234. Chromosome 16 contains the PKD-1 gene mutation.
235. Chronic hypertension in pregnancy is defined as blood pressure of at least 140 mmHg systolic or 90 mmHg
diastolic before pregnancy, or for women who first present for care during pregnancy as in this case, before
20 weeks of gestation.
236. Chronic kidney disease results in reduced levels of 1,25(OH)2 vitamin D, which in turn results in
hypocalcaemia.
237. Chronic renal failure leads to low levels of hydroxylated vitamin D, and hence to hypocalcaemia and
therefore secondary hyperparathyroidism.
238. Chronic renal failure managed with dialysis is the commonest cause of secondary oxalosis (acute arthritis
of small joints with digital calcific deposits).
239. Churg-Strauss syndrome is a rare form of small-vessel vasculitis, characterised by asthma, allergic rhinitis
and prominent peripheral blood eosinophilia.
240. Cinacalcet is a calcimimetic.
241. Conn's syndrome (hyperaldosteronism) is characterised by hypertension, hypokalaemia, high aldosterone,
low renin.
242. Contrast induced nephropathy peaks in incidence three to five days after the administration of intravenous
contrast.
243. Current UK guidelines with regard to chronic kidney disease recommend referral for further investigation
of atherosclerotic renal artery stenosis when there is fall of GFR >15% over 12 months
244. Cystinuria is characterised by recurrent cystine urolithiasis.

307
M Y Elamin
MBBS, DTM&H, MCTM, MRCPI 1& 2
245. Cystinuria, cystinosis, urate uropathy and hyperoxaluria are autosomal recessive conditions.
246. Cystoscopy is the best initial investigation in cases of suspected bladder tumour.
247. Demeclocycline blocks the action of antidiuretic hormone at the distal renal tubules and increases free
water clearance, thereby increasing serum sodium concentration.
248. Deranged coagulation associated with nephrotic syndrome is a consequence of AT III deficiency, increased
fibrinogen and increased factor VIIIc.
249. Diabetic nephropathy has greater incidence in type 1 diabetes than type 2.
250. Diabetic nephropathy typically presents with renal impairment, proteinuria and normal or small kidneys on
ultrasound.
251. Dialysis is able to remove about 2700 mg of phosphate per week in a patient who is anuric. This is half of
the normal kidney's ability.
252. Different formulations of ciclosporin have different pharmacokinetic properties and it is essential that
ciclosporin is prescribed by brand and not generically.
253. Differential diagnosis of SLE nephritis
254. E.coli 157 causes haemolytic uraemic syndrome, characterised by renal dysfunction, anaemia, raised
bilirubin, proteinuria and haematuria in combination with diarrhoea.
255. Eighty-five percent of cases of autosomal dominant APKD-1 are due to the defect in PKD-1 locus on
chromosome 16p13.3.
256. Eighty-five percent of untreated subjects with granulomatosis with polyangiitis will have circulating anti-
neutrophil cytoplasmic antibody (cANCA) and those with limited disease are less likely to have positive
serology.
257. Elevated amino transferase (AST) (from muscle) suggest a diagnosis of rhabdomyolysis.
258. Escherichia coli is the most likely responsible pathogen for haemolytic uraemic syndrome with a diarrhoea
prodrome.
259. Excessive urinary arginine excretion is a feature of cystinuria.
260. Familial hypocalciuric hypercalcaemia is an autosomal dominant condition
261. First line treatment in scleroderma is a gradual reduction in blood pressure (10-15 mmHg per day) with an
ACE inhibitor until the diastolic pressure reaches 85-90 mmHg.
262. Fluvastatin has a <0.04% risk of rhabdomyolysis.
263. focal segmental glomerulosclerosis (FSGS) leads to chronic renal failure in half of cases. This accounts for
approximately 20% of cases of nephrotic syndrome in children and 40% in adults.
264. Gadolinium-induced nephrogenic systemic fibrosis (NSF) is a debilitating and irreversible disease.
265. Genetic linkage studies can exclude or make the diagnosis of polycystic kidney disease in younger patients.
308
M Y Elamin
MBBS, DTM&H, MCTM, MRCPI 1& 2
266. Granulomatosis with polyangiitis is a multi-system disorder characterised by necrotising granulomas of
medium and small blood vessels.
267. Granulomatosis with polyangiitis is most often associated with the anti-cytoplasmic ANCA (c-ANCA)
positivity.
268. Haemodialysis is the treatment of choice for patients with aspirin overdose when the plasma-salicylate
concentration is greater than 700 mg/litre (5.1 mmol/litre) or in the presence of severe metabolic acidosis as
recommended within the British National Formulary (BNF) poisons section.
269. Haemolytic uraemic syndrome (HUS) presents with acute renal failure, microangiopathic haemolytic
anaemia, and thrombocytopenia with normal clotting.
270. Heavy proteinuria in the presence of hypertension may indicate renal cause of secondary hypertension.
271. Henoch-Schönlein purpura usually settles between four to six weeks without sequelae if kidney
involvement is mild.
272. Histologically, a number of different types of renal disease are recognised in SLE, with immune-complex
mediated glomerular disease being the most common
273. Hypertension is a frequent problem associated with erythropoietin and may induce seizures.
274. Hypertensive disorders during pregnancy occur in women with pre-existing primary or secondary
hypertension, and in women who develop new-onset hypertension (defined as those cases occurring after
20 weeks).
275. Hyperuricaemia can occur in association with enhanced cell destruction, particularly leukaemias.
276. Hypocomplementaemia is associated with a non-vasculitic process such as subacute bacterial endocarditis
(SBE).
277. Ideally, before starting EPO in renal patients you should get their haematinics (iron, B12, folate) to ensure
they are replete of all.
278. IgA nephropathy is most common in the 2nd and 3rd decades of life
279. IgA nephropathy is the most common form of glomerulonephritis globally, and is most often seen in young
men following an upper respiratory tract infection. It classically presents with rash and arthritis.
280. IgA nephropathy usually presents in the age range of 20-40 years.
281. Immediate treatment in the context of a life threatening arrhythmia associated with acute renal failure,
hyperkalaemia, and palpitations, would be calcium gluconate.
282. Immunosuppression in the form of alternating steroids and cyclophosphamide is indicated in those patients
with symptomatic nephrotic syndrome caused by membranous nephropathy.
283. In cases of suspected renal cell carcinomas, ultrasound scan of the renal tract would be the first
investigation of choice, as it is able to pick up 95% of renal cell carcinomas greater than 1 cm in diameter.

309
M Y Elamin
MBBS, DTM&H, MCTM, MRCPI 1& 2
284. In hyper/hypocalcaemia, always check the corrected calcium to make sure that the "abnormality" in the
total serum calcium levels is not due to abnormal albumin concentrations.
285. In particular tomato and banana have high potassium content. Interestingly, a low intake of potassium has
been found to be associated with high blood pressure, increased stroke risk and chronic kidney disease.
286. In patients presenting with blood and protein in the urine and abackground of impaired renal function and
shrunken kidneys, the best investigation would be to perform a renal biopsy.
287. In patient's with Goodpasture's syndrome, the prognosis is drastically improved with the removal of antigen
through plasmapheresis, immunosuppression with corticosteroids and cyclophosphamide.
288. In the presence of pericarditis haemodialysis is required.
289. Increased fluid intake restores fluid lost through the digestive tract, and also acts as a dilutional inhibitor of
crystal and stone formation.
290. Increasing the duration of dialysis sessions can improve patient fatigue and quality of life.
291. Intravenous doses of aciclovir, which has a relatively low solubility, can lead to deposition of aciclovir
crystals in the renal tubules, resulting in intratubular obstruction and foci of interstitial inflammation.
292. Ischaemia, typically in hypotensive hospitalised patients, is the most frequent antecedent to acute tubular
necrosis. Blood pressure should be maintained in cardiogenic shock with fluids and/or inotropic agents.
293. IV iron transfusion is an appropriate step for haemodialysis patients.
294. IV sodium bicarbonate can be used in severe acidosis prior to dialysis treatment.
295. Key diagnostic features of nephrotic syndrome, minimal change disease as most common subtype in
children.
296. Kimmelstiel-Wilson lesions depict nodular glomerulosclerosis which is seen in diabetic kidney disease.
297. Lithium can cause diabetes insipidus.
298. Living related kidney donation generally results in a better match and better graft survival as compared to
unrelated donation.
299. Loop diuretics have the potential side effect of ototoxicity. Of the diuretics listed, furosemide is a loop
diuretic and is the most likely candidate.
300. Loss of acid secretions from the stomach renders this patient deficient of acid in her blood; hence her acid
base status will be reflective of metabolic alkalosis. One may also expect a lower serum potassium and
chloride which are also lost in gastric secretions.
301. Low albumin and elevated cholesterol would suggest nephrotic syndrome.
302. Medullary sponge kidney is typically not inherited but is a congenital condition.
303. Membranous glomerulonephritis presents with proteinuria, and is caused by immune complex deposition in
the subepithelial aspect of the capillary loop.

310
M Y Elamin
MBBS, DTM&H, MCTM, MRCPI 1& 2
304. Membranous nephropathy is characterised by thickened basement membranes and monotonous granular
deposits of IgG and C3 distributed in the epimembranous space of virtually all glomerular capillaries. It is
typically seen in the over 40 age group with a male predominance of 2 to 1.
305. Membranous nephropathy is the second most common cause of nephrotic syndrome in adults in western
countries, accounting for 22-33% of cases and coming in just behind focal segmental glomerulosclerosis. It
is primary (idiopathic) in two-thirds of cases.
306. Mesangial IgA deposits are the most typical features of HSP on renal biopsy.
307. Metformin should be stopped if creatinine is above 150 µmol/L or eGFR less than 30 mL/1.73m2
308. Metformin, a biguanide, increases the risk of contrast induced nephropathy.
309. Microalbuminuria is defined as 30-300 mg albumin/24 hrs or >2.5 mg/mmol (men) or >3.5 mg/mmol
(women) albumin:creatinine ratio
310. Microscopic polyangiitis classically presents with worsening renal function, haemoptysis and a purpuric
rash affecting the lower limbs and is associated with MPO ANCA positivity.
311. Minimal change disease is characterised by heavy proteinuria and is treated primarily with high-dose
prednisolone.
312. Minimal change disease, the glomerular basement membrane is normal on electron microscopy
313. Mononeuritis multiplex, fever, hypertension, and nephritic renal involvement which is most consistent with
a diagnosis of polyarteritis nodosa.
314. MR angiography (MRA) should be considered the optimum non-invasive screening test for renovascular
disease.
315. Nephrogenic diabetes inspidius can be caused by doxycycline
316. Nephrotic range proteinuria is defined as a protein:creatinine ratio of 300-350 mg/mmol.
317. Nephrotic syndrome is the main presentation of membranous glomerulonephritis and associated with
malignancy.
318. Nephrotoxicity is the most frequent side effect of ciclosporin.
319. Of the answers listed IgA nephropathy is associated with a normal C3.
320. Of the answers listed, MRA is the best choice of investigation for renal artery stenosis.
321. Only USS screening at age 30 years or older can definitively rule out ADPKD type 1.
322. Osmotic cerebral changes precipitated by urinary sodium loss can cause of drowsiness.
323. Patchy tubular necrosis on renal biopsy is seen following cardiogenic shock and subsequent acute kidney
injury.
324. Patients on long-term immunosuppression are at risk of malignancy; most commonly skin BCCs and SCCs.

311
M Y Elamin
MBBS, DTM&H, MCTM, MRCPI 1& 2
325. Patients on maintenance fluids should be prescribed approximately 1mmol/kg/day of potassium, sodium
and chloride.
326. Patients who undergo long-term haemodialysis suffer from increasing arterial calcification, which is
associated with both increased risk of myocardial infarction and stroke, but the greatest absolute increase is
in MI rates.
327. Peritoneal cavity integrity and (relatively) intact peritoneal membrane are essential for effective peritoneal
dialysis.
328. Peritonitis is a serious side effect of peritoneal dialysis therapy. Treatment should be guided by local
protocols, and usually involves intraperitoneal antibiotics.
329. Plasma protein electrophoresis would be the investigation of choice in patients with suspected myeloma.
330. Post-transplant patients should be regularly screened for different malignancies, especially those of the
skin.
331. Primary hyperaldosteronism (Conn's syndrome) typically presents with hypokalaemic alkalosis.
332. Psuedohypoparathyroidism is due to PTH receptor abnormality producing resistance.
333. PTH production is stimulated by hypocalcaemia and hyperphosphataemia.
334. Quadruple therapy containing a proton pump inhibitor, bismuth, metronidazole, and tetracycline, has been
shown in meta-analysis of comparative randomised controlled trials to achieve a similar eradication rate to
clarithromycin-based triple therapy.
335. Rapid onset of renal failure, coupled with a rash and eosinophilia is highly suspicious of a diagnosis of
interstitial nephritis
336. Rapidly worsening symptoms of asthma, accompanied by hypertension, raised creatinine and blood and
proteinuria are features of Churg-Strauss.
337. Red cell casts present in acute glomerulonephritis, renal vasculitis, accelerated hypertension and interstitial
nephritis.
338. Regimes comprising chlorambucil or cyclophosphamide, either alone or with steroids, are more effective
than symptomatic treatment for membranous nephropathy.
339. Renal biopsies show any changes of glomerulonephritis along with renal scarring from longstanding
hypertension or urinary tract infections.
340. Renal biopsy will show mesangial IgA deposition on immunofluorescence and light microscopy will show
mesangial hypercellularity and matrix expansion.
341. Renal clearance is the main factor that determines the choice of loading dose of digoxin.
342. Renal cysts are usually found on ultrasound and CT. Tissue may be needed to differentiate between
malignant and benign cysts, but this is obtained via aspiration rather than renal parenchymal biopsy.
343. Renal failure from ATN occurs in 25% of patients with severe hepatic damage.

312
M Y Elamin
MBBS, DTM&H, MCTM, MRCPI 1& 2
344. Renal impairment is unrelated to immune complex; it occurs mostly as a result of direct tubular damage
from excess protein load, dehydration, hypercalcaemia, and the use of nephrotoxic medications.
345. Renal scintigraphy with DMSA involves administration of radioactive isotope which is avidly taken up by
the renal parenchyma.
346. Renal size asymmetry in the presence of hypertension and renal impairment should prompt the search for
renovascular disease.
347. Renal size on ultrasound scan (USS) can help distinguish between acute and chronic renal failure.
348. Renal transplat patients at high risk of an acute episode of pyelonephritis in the transplanted kidney, due to
the immunosuppression, the neuropathic bladder and self-catheterisation. This would present like an acute
rejection episode, with a tender swollen graft, low-grade pyrexia, and deteriorating graft function.
349. Renal ultrasound allows one to assess kidney size, the presence of hydronephrosis, cysts or tumours and
any developmental abnormalities. Doppler traces also allow renal blood flow to be assessed.
350. Renovascular disease is due to disease affecting the arterial supply of the kidney(s). The resulting renal
hypoperfusion leads to hyperactivation of the renin-angiotensin-aldosterone axis, causing hypertension.
351. Reversible acute tubular necrosis after aminoglycoside reflects a concurrent impairment in the
concentrating ability, and most patients are non-oliguric.
352. S. aureus and S. epidermidis are the two most common pathogens identified in cases of CAPD peritonitis.
353. Serum creatinine and potassium should be checked one to two weeks after commencement of ACE-
inhibitors.
354. Serum cystatin c represents a novel marker to estimate renal function.
355. Severe decrease in GFR, with or without other evidence of renal damage is reflective of stage IV chronic
kidney disease.
356. SIADH results in excess ADH production with subsequent excess water reabsorption at the distal tubules,
resulting in low serum osmolality and high urine osmolality.
357. Spironolactone is a potassium-sparing diuretic, which works at the sodium/potassium co-transporter of the
distal convoluted tubule.
358. Staphylococcus aureus, Staphylococcus epidermidis, Pseudomonas aeruginosa, and Escherichia coli are the
commonest causes of peritonitis in peritoneal dialysis patients. Severe infections may be polymicrobial.
359. Studies report less nephrotoxicity and equal efficacy when aminoglycosides are given once daily
(supratherapeutic doses) rather than in conventional divided doses.
360. Tenofovir is associated with acute and chronic renal impairment and of the answers given is the most likely
cause of the acute renal impairment.
361. The acute rejection of a kidney after transplantatioon is recognised and due to anti-IgG antibodies to the
human leukocyte antigen (HLA) incompatible tissues with primary activation of T cells.

313
M Y Elamin
MBBS, DTM&H, MCTM, MRCPI 1& 2
362. The anion gap is calculated by the sum of the positive ions in the blood (mainly sodium and potassium)
from which is subtracted the sum of the negative ions (chloride and bicarbonate).
363. The arthritis of Henoch-Schönlein purpura is usually transient or migratory, involves one to four joints, and
is non-deforming.
364. The best drugs for Autosomal dominant APKD-1 are ACE inhibitors (e.g. captopril, enalapril, or lisinopril)
or angiotensin II receptor antagonist blockers (e.g. telmisartan, losartan, irbesartan, or candesartan).
365. The biochemical features of rhabdomyolysis are raised creatine kinase, hypocalcaemia (especially early
after injury), hyperkalaemia and acute kidney injury.
366. The biochemical features of rhabdomyolysis are raised creatine kinase, hypocalcaemia, hyperkalaemia and
acute kidney injury.
367. The classical pentad of clinical features of thrombotic thrombocytopenic purpura comprises
microangiopathic haemolytic anaemia, acute kidney injury, thrombocytopenia, fever and neurological
symptoms.
368. The clues to the diagnosis are longstanding chronic inflammatory disease, normal sized kidneys and
proteinuria.
369. The combination of haemoptysis (with radiological findings consistent with pulmonary haemorrhage) and
red cell casts in the urine points to a diagnosis of Goodpastures syndrome
370. The diagnosis of Churg-Strauss syndrome is supporting by the finding of extravascular eosinophils on
vascular biopsy.
371. The fact that chronic rejection is rare in transplants between HLA-identical siblings suggests that HLA-
antigen dependent immunological factors are important. The presence of anti-HLA antibodies is a risk
factor for rejection.
372. The 'Fick principle' can be used to estimate RBF through clearance.
373. The gold standard for establishing the diagnosis of renal artery stenosis is renal arteriography and this is
commonly performed with magnetic resonance angiography.
374. The inhibition of angiotensin II production reduces aldosterone levels resulting in a reduction in potassium
secretion in the distal tubule.
375. The laboratory diagnostic criteria for PD peritonitis include a PD fluid white cell count of greater than 100
mm3 or a differential WCC of greater than 50% neutrophils.
376. The mist common side-effect of erythropoietin, when used in chronic kidney disease is hypertension.
377. The most frequent class of antibody implicated in Goodpasture’s disease is IgG.
378. The normal adult kidney size on ultrasound is between 9-12 cm.
379. The PCR value of 100 mg/mmol approximates a 24 hour urinary protein collection of 1 g

314
M Y Elamin
MBBS, DTM&H, MCTM, MRCPI 1& 2
380. The renal manifestations of SLE are highly variable, ranging from mild asymptomatic proteinuria and/or
haematuria to rapidly progressive uraemia.
381. The use of serum creatinine to estimate the glomerular filtration rate can be misleading when the patient
has abnormal muscle mass (such as leg amputation).
382. The vast majority of cases of minimal change nephropathy, in adults and children, remit following
treatment with oral glucocorticoids.
383. There is no evidence to support re-introducing ACE inhibition or substituting an angiotensin receptor
blocker in atherosclerotiv renal disease
384. Thiazide diuretics act on the cortical diluting segment of the nephron, predominantly in the early distal
tubule.
385. This vignette depicts a case of post-streptococcal GN. Circulating antibody-antigen complexes deposit on
the glomerular basement resulting in the hump appearance in sub-epithelial space.
386. This vignette describes a case of IgA nephropathy. IgA mesangial deposition is the correct biopsy finding.
387. Transforming growth factor-beta is strongly implicated in renal scarring
388. Treatment with corticosteroids in IgA nephropathy is usually reserved for those patients with hypertension
and a rising creatinine
389. Triple phosphate stones are classically produced by urea splitting organisms such as Klebsiella or Proteus.
390. Twenty per cent of the cardiac output goes towards the kidney.
391. Type 1 renal tubular acidosis is due to impaired secretion of hydrogen ions. Associated with hypokalaemia
and renal stones.
392. Ultrasound imaging is a safe, non-invasive means rapidly to exclude a correctable cause of renal
impairment. It is readily available in most hospitals and can be performed by a sonographer or radiologist.
393. Up to 25% of patients with polycystic kidney disease may have some degree of mitral valve prolapse.
394. Uraemia leads to exudation of fibrin onto the epicardial and pericardial surfaces.
395. Uric acid renal stones are a possible side effect of thiazide diuretics.
396. Urine pH is affected by diet, with vegetarians having more alkaline urine when compared with omnivores.
397. Urine sodium concentration is typically above 30 mmol/L due to tubular injury. The fractional excretion of
sodium distinguishes ATN from pre-renal diseases as a cause of AKI.
398. Very few patients with minimal change disease actually progress to end stage renal disease.
399. A complication of reflux nephropathy is reccurent UTI's and can lead to renal scarring and renal failure.
400. A rise in serum creatinine more than 20% above the baseline after starting an angiotensin-converting
enzyme inhibitor (ACEI) should prompt the clinician to hold the drug.

315
M Y Elamin
MBBS, DTM&H, MCTM, MRCPI 1& 2
401. ACEi is first line blood pressure treatment in diabetes (but needs to be gradually titrated to avoid side-
effects)
402. Acetazolamide can cause metabolic acidosis with a normal anion gap.
403. Acute interstitial nephritis is commonly caused by drugs such as NSAIDS and antibiotics.
404. Acute interstitial nephritis is inflammation of the renal tubulo-interstitium and commonly occurs after a
prolonged course of NSAIDs due to hypersensitivity.
405. Acute kidney injury can develop as a result of Upper GI bleed and should be treated with IV fluids.
406. ADPK disease progresses to renal replacement therapy after 40-60 years
407. ADPKD cause mitral valve prolapse and cerebral aneurysm not aortic stenosis.
408. Aetiology of acute kidney injury
409. All patients with diabetes and microalbuminuria should be offered therapy with an ACE inhibitor or
angiotensin receptor blocker irrespective of whether they have hypertension.
410. Alport syndrome can be autosomal dominant, recessive or a new spontaneous mutation. However in 85%
of cases it is X linked.
411. Anaemia associated with diabetes is often due to reduced erythropoietin release due to chronic renal failure.
412. Anaemia is not an indication for commencing renal replacement therapy.
413. ANCA positive vasculitis
414. Approximately half of patients with ADPKD will reach end stage renal failure by the age of 60 and require
renal replacement therapy.
415. ATIII deficiency occurs in nephrotic syndrome due to loss of clotting factors through the kidney.
416. Bartter's syndrome is a rare autosomal recessive disorder, caused by one of three mutations of the ion
transporter/channel present in the thick ascending limb of the loop of Henle.
417. Bartter's syndrome is hypokalaemic metabolic acidosis with high urinary potassium loss.
418. Beta 2 microglobulin is the most accurate marker of disease progression in haematological syndromes such
as myeloma.
419. Calcium gluconate is given in hyperkalaemia to protect the myocardium but insulin and dextrose is used to
treat and lower the potassium.
420. Causes of acute interstitial nephritis
421. Causes of acute kidney injury.
422. Causes of acute urinary obstruction
423. Chronic diabetes with nephropathy may result in hyporeninaemic hypoaldosteronism.
424. Chronic lead exposure leads to hypochromic microcytic anaemia with basophilic stippling on the film.
316
M Y Elamin
MBBS, DTM&H, MCTM, MRCPI 1& 2
425. Ciclosporin has many side efects, however Dupytren's contracture is not one.
426. Colcicine is safe to use in acute exacerbations of gout with renal failure.
427. Complications associated with long-term dialysis
428. Complications of chronic immunosuppression
429. Complications of thyroidectomy
430. Cough secondary to ACE inhibitors is due to accumulation of bradykinin.
431. Cytomegalovirus (CMV) infection is responsible for a substantial fraction of the morbidity and mortality
following organ transplantation.
432. Dairy products are high in phosphate content and patients with chronic kidney disease should be counselled
by a dietitian on suitable foods.
433. Dialysis should be considered in patients with acute kidney injury, particularly when fluid overloaded and
hyperkalaemic
434. Diffuse Proliferative GN is the commonest form of GN in SLE, treatment is with prednisolone and
intravenous cyclophosphamide
435. Drug causes of hyponatraemia.
436. Drug induced hypocalcaemia
437. Drug induced nephrotoxicity.
438. Drug interaction with fluconazole.
439. Fibromuscular dysplasia, a rare cause of hypertension and hypokalaemia is more common in women which
causes hyperreninaemic hyperaldosteronism.
440. Gold is a recognised cause of interstitial nephritis.
441. Goodpasture's syndrome with potentially life threatening pulmonary haemorrhage and profound renal
impairment requires plasmapheresis.
442. Gradual progressive increase in serum creatinine is not unusual in the treatment of diabetic nephropathy
and it is unlikely to be related to any other pathology or medication.
443. HOPE study showed increased benefit from ACEi in diabetes to reduce CV death.
444. Hyperkalaemia treatment is initiated when over 6.5 or 6 with ECG changes.
445. Hypertension and proteinuria should be treated with an ACE-inhibitor as first line.
446. Hypertension in APKD is induced due to increased production of erythropoetin.
447. Hypertension, diabetes and the presence of proteinuria are well-recognised and accepted risk factors for the
progression of chronic kidney disease (CKD).
448. Hypertensive nephropathy can present with a raised creatinine and proteinuria.
317
M Y Elamin
MBBS, DTM&H, MCTM, MRCPI 1& 2
449. Idiopathic hypercalciuria is often familial, the most common cause being increased gastrointestinal
absorption of calcium which causes calcium oxalate stones.
450. IgA nephropathy commonly presents as nephritic syndrome.
451. IgA nephropathy typically presents as recurrent haematuria in young men, often precipitated by upper
respiratory tract infections.
452. In acute flares of Goodpasteur's disease, removing the autoantibody by plasmaphoresis is first line
treatment.
453. In ATN urinay sodium losses are usually greater than 60 mmol/L
454. In cases of acute kidney injury nephrotoxics should be stopped and intravenous fluids prescribed as well as
strict fluid monitoring and catheterisation if patients cannot reliably do this.
455. In cases of minimal change disease, initial treatment should be steroid, even in cases which relapse.
456. In cases of suspected TTP urgent blood film is required to look for red cell fragments.
457. In hyperkalaemia the first step in treatment must be to administer calcium gluconate in order to stabilise the
myocardium.
458. In membranous GN forty per cent remit without treatment, 30% develop endstage renal failure (ESRF).
459. In order to maximise the efficacy of erythropoietin in the treatment of the anaemia of chronic kidney
disease it is important to make sure that patients' iron statuses are adequately optimised.
460. In patients with chronic kidney disease maintaining the haemoglobin level within the normal range is
associated with adverse outcomes.
461. In quality improvement terms, adherence to the checklist is a type of process measure which gives you an
indication as to whether the core processes are being carried out.
462. In the setting of diabetes and stable renal function the albumin:creatinine ratio is considered the most
appropriate test to detect and quantify proteinuria.
463. Indications for dialysis include acid base disturbance, pulmonary oedema, symptomatic uraemia or resistant
hyperkalaemia.
464. Intensive care trainees should be aware of the indications for emergency renal replacement therapy.
465. Interstitial nephritis may be caused by numerous aetiologies such as: infection, autoimmunity or glomerular
disease as well as hypersensitivity to medicines.
466. Investigation and treatment of renal anaemia.
467. Investigation of glomerulo-nephritis
468. IV furosemide not effective in HD patients
469. IV iron tends to work much more effectively in patients with CKD.
470. K-DOQI guidelines suggest maintaining Hb >110 g/L in CRF
318
M Y Elamin
MBBS, DTM&H, MCTM, MRCPI 1& 2
471. Management of acute kidney injury secondary to pneumonia
472. Membranoproliferative glomerulonephritis (also known as mesangiocapillary glomerulonephritis) is the
characteristic histological finding on biopsy in cryoglobulinaemia secondary to hepatitis C chronic
infection.
473. Mesangial widening, inflammation and mesangial cell proliferation are common features on renal biopsy in
IgA nephropathy.
474. methyl pred for 3 days, then a steroid sparing drug for long term
475. Microscopic polyangiitis is most frequently associated with perinuclear anti-neutrophil cytoplasmic
antibody (pANCA) antibodies and raised MPO (myeloperoxidase) titre.
476. Mild hyperkalaemia with no evidence of cardiac toxicity require monitoring only.
477. Minimal change disease will improve rapidly with oral steroids.
478. MRSA requires specific extended spectrum antibiotics for effective treatment.
479. Nephrogneic systemic fibrosis can be induced by Gadolinium used in MRI in ESRF patients.
480. Nephrotic syndrome in young adults.
481. Obesity alone should not be used as a risk factor For developing CKD.
482. Omeprazole is principally dependent upon hepatic clearance and safe even with marked renal impairment.
483. Once obstructive uropathy has been resolved supplementation with IV fluid is crucial to prevent AKI
484. Osmolalities will correct following a water deprivation test in psychogenic polydipsia.
485. Oxalate stones commonly occur in syndromes of malabsorption.
486. Pamidronate is second line treatment for hypercalcaemia after rehydration has failed
487. Patients on long term immunosuprresion are at increased risk of non-melanoma skin cancers.
488. Patients with adult polycystic kidney disease should only have screening for cerebral aneurysms if high risk
(ie previous ruptured aneurysm, neurological symptoms or positive FHx).
489. Patients with APKD commonly suffer from cyst rupture and should be managed with basic analgesia
provided there is no hemorrhage.
490. Patients with Chrons disease are at risk of developing calcium oxalate stones when they have had ileal
resection.
491. Patients with HSP and hypertension plus proteinuria should be treated with an ACE-inhibitor.
492. Patients with idiopathic hypercalciuria should aim for a daily urinary output in excess of 2000 ml.
493. Patients with large myocardial infarctions can develop profuse acute heart failure and the mainstay
treatment is revascularisation by PCI.
494. Patients with rhabomyolysis commonly require urgent correction of hyperkalaemia.
319
M Y Elamin
MBBS, DTM&H, MCTM, MRCPI 1& 2
495. PD peritonitis fluid will be cloudy and is an emergency that requires prompt broad spectrum antibiotic
therapy delivered through the intra-peritoneal catheter.
496. Planning for vascular access in patients with stage 4 kidney disease is essential to avoid the need for
emergency dialysis.
497. Polycythaemia is a well recognised complication of hypernephroma due to erythropoietin secretion.
498. Proven interventions in the treatment of diabetic nephropathy include ACE inhibitors (greatest benefit), low
dietary protein and improved glycaemic control
499. Recurrent UTIs in a diabetic can be reduced with good glycaemic control
500. Relative contraindication to erythropoietin therapy
501. Renal bioopsy suggestive of HIV nephropathy shows focal segmental glomerulosclerosis.
502. Renal biopsy can help to determine the underlying disease process in a patient presenting with nephrotic
syndrome
503. Renal ultrasound screening is the test of choice and has 100% sensitivity for PKD1, false negatives may
occur below the age of 20.
504. Renal Vein Thrombosis is often clinically silent. Association with hypercoagulable state, peripheral leg
oedema and flank pain in a patient presenting with AKI are all pertinent clues.
505. SCA can lead to renal complications such as papilary necrosis which should be treated conservatively
unless there is overwhelming haematuria causing anaemia.
506. Sepsis driven hypovolaemic acute kidney injury should first be treated with IV fluids.
507. Sevelemer is a non-aluminium containing phosphate binder which is suitable for using in patients with end
stage renal failure and raised serum phosphate which lowers phosphate and increases calcium
508. Steriod responsiveness is near 100% in minimal change but only about 40% in FSGS.
509. Systolic or diastolic blood pressures above the target ranges are associated with increased risk of a doubling
in serum creatinine, end-stage renal failure and death in CKD.
510. Tertiary hyperparathyroidism is characterised by raised calcium, raised (or sometimes normal) phosphate
and grossly elevated parathyroid hormone levels.
511. The ASTRAL trial showed that medical management or RAS is superior to interventional treatment.
512. The commonest cause of nephrotic syndrome in children is minimal change disease, which usually
responds to a course of high dose corticosteroids.
513. The diagnosis of ATN is based on the clinical context and the tubular dysfunction leads to low urinary
sodium osmolality.
514. There are certain criteria according to the BHS which require irradiated blood, such as transplant or
Hodgkins lymphoma.

320
M Y Elamin
MBBS, DTM&H, MCTM, MRCPI 1& 2
515. This patient has an ESBL urine infection and is symptomatic. Meropenem has a broad spectrum of activity
and is the right initial course of antibiotics in this case pending full sensitivities of the cultured organism.
516. Treatment of PD-related peritonitis
517. Typical ultrasound changes in renal artery stenosis are asymmetrical kidneys; the affected kidney >2 cm
smaller than the unaffected kidney.
518. Ultrasound scanning should be performed first line in cases of deteriorating renal function to rule out
obstruction.
519. Urine osmolality comparison with serum osmolality will help to distinguish between renal and extra-renal
loss of fluid/
520. Use of NSAIDs impairs kidney function by interstitial nephritis and prostaglandin inhibition.
521. Vomiting is a recognised cause of metabolic alkalosis.
522. When treating emergency high blood pressure the target for lowering is 25% to prevent end organ damage.
523. Wire-loop lesions are seen on renal biopsy in post-infective glomerulonephritis.
524. With the history of prolonged time on the floor, rhabdomyolysis should be sought as a possible cause of
acute kidney injury.
525.

321
M Y Elamin
MBBS, DTM&H, MCTM, MRCPI 1& 2
NEUROLOGY
1. A 15-year-old is noted to have a midline neck lump that is painless and cystic in nature. It moves upwards on
protrusion of the tongue - thyroglossal cyst
2. A 15-year-old presents with leg weakness and frequent sprained ankles. On examination areflexia, pes cavus, and
distal lower extremity weakness and atrophy are found - Charcot-Marie-Tooth disease
3. A 1-year-old is noted to have a swelling on the left side of the neck. The mass is soft, non-tender and transilluminates
- cystic hygroma
4. A 20-year-old man presents with tender, swollen 2cm mass in the right submandibular region. His temperature is
38.2degC and he is noted to have poor dental hygiene - reactive lymphadenopathy
5. A 20-year-old presents with a painless swelling on the lateral aspect of the neck. On examination it is smooth, non-
tender and fluctuant - branchial cyst
6. A 25-year-old is investigated for ataxia. He is found to have a cerebellar and retinal haemangiomas. His blood
pressure is 190/115 mmHg - von Hippel-Lindau syndrome
7. A 25-year-old man with muscle weakness is reviewed in clinic. On shaking his hand he has difficultly loosening his
grip. On examination he has frontal balding, bilateral ptosis and cataracts - myotonic dystrophy
8. A 2-year-old boy whose mother has been treating him with aspirin for a fever presents with confusion & seizures.
Found to have hepatomegaly & hypoglycaemia on admission - Reye's syndrome
9. A 30-year-old man presents with progressive weakness of his hands. On examination you notice wasting of the small
muscles of the hand. Also pain and temperature sensation is reduced but light touch is preserved. - syringomyelia
10. A 30-year-old presents with a slowly enlarging lump in the submandibular area. It is non-tender and firm on
examination. Splenomegaly is also noted - lymphoma
11. A 30-year-old woman presents with loss of vision in her right eye associated with pain on eye movement. Six months
ago she presented with tingling and numbness in her left hand - multiple sclerosis
12. A 30-year-old woman who has recently developed an upper respiratory tract infection presents with vertigo and
vomiting. Her hearing is also affected. The symptoms came on suddenly this morning - viral labyrinthitis
13. A 30-year-old woman who is recovering from an upper respiratory tract infections presents with recurrent attacks
of vertigo associated with nausea and vomiting. There is no hearing loss or tinnitus - vestibular neuritis
14. A 35-year-old presents with jerky, random and uncontrollable movements of his arms. These have been getting
progressively worse for the past few months. His wife also says he is increasingly irritable and moody - Huntington's
disease
15. A 35-year-old woman is noted to have a firm swelling at the bottom of her neck. She has been suffering from
weakness and numbness of her arm on the same side - cervical rib
16. A 3-year-old boy is investigated for repeated sinopulmonary infection and abnormal gait and head movements.
Dilated capillaries are noted in his eye - ataxic telangiectasia
17. A 40-year-old man presents with recurrent episodes of vertigo associated with tinnitus and hearing loss. A feeling
of aural fullness often precedes the attacks - Meniere's disease
322
M Y Elamin
MBBS, DTM&H, MCTM, MRCPI 1& 2
18. A 40-year-old woman presents with a tremor of both hands. This is worse when she is stressed and relieved by
alcohol. There is a family history of similar problems - essential tremor
19. A 45-year-old man is investigated for unilateral hearing loss. He also complains of tinnitus and some problems with
his balance. There is an absent corneal reflex on examination - acoustic neuroma
20. A 55-year-old presents with fever, headache, confusion and aphasia. A CT shows petechial haemorrhages in the
temporal lobe - herpes simplex encephalitis
21. A 55-year-old woman presents with recurrent episodes of vertigo that are often triggered by a change in head
position. Each episode lasts around 10-20 seconds and is associated with nausea - benign paroxysmal positional
vertigo
22. A 60-year-old man present with muscle fasciculation associated with wasting of the small muscles of the hand. On
examination there are LMN signs in the arms and UMN signs in the legs - motor neuron disease
23. A 65-year-old man is investigated for rapid onset memory problems. On examination he is noted to have cerebellar
signs and myoclonic jerks - sporadic Creutzfeldt-Jakob disease
24. A 65-year-old man with lung cancer develops muscle weakness in his legs. He finds that repeated muscle
contractions lead to increased muscle strength. Hyporeflexia and weakness are found on examination - Lambert-
Eaton syndrome
25. A 65-year-old presents recurrent falls and cognitive impairment. He also complains of difficulty reading and
descending stairs. On examination he is found to have a stiff, broad-based gait and bradykinesia - progressive
supranuclear palsy
26. A 6-week-old baby is noted to have a 1*1.5cm swelling in the left axilla. They were given the BCG vaccine at birth
- reactive lymphadenopathy
27. A 75-year-old man presents with dysphagia, regurgitation and chronic cough. He is noted to have halitosis but
otherwise examination is normal - pharyngeal pouch
28. A male smoker presents with recurrent episodes of severe pain around one eye which typical lasts around 1 hour.
These episodes have occured daily for the past 6 weeks and are associated with ocular erythema and lacrimation -
cluster headache
29. A man loses consciousness, falls to the ground and starts to stiffen and straighten his limbs alternatively. During
the seizure he bites his tongue - tonic-clonic seizure
30. A man presents with a severe occipital headache and vomiting. It came on suddenly and reached maximum intensity
within a minute. On examination there is neck stiffness - subarachnoid haemorrhage
31. A man suddenly stares into space and is non-responsive. He then repeatedly smacks his lip and appears to be
chewing. This lasts for around 1 minute - focal impaired awareness seizure
32. A man who has recently been treated for herpes simplex encephalitis presents with a sudden increase in appetite
and sexual desire - Kluver-Bucy syndrome

323
M Y Elamin
MBBS, DTM&H, MCTM, MRCPI 1& 2
33. A middle-aged man presents with a head injury after falling down some stairs. After losing consciousness he quickly
recovers but complains of a headache. Over the next few hours he becomes more confused and has one seizure -
extradural (epidural) haematoma
34. A patient is noted to have laboured, non-fluent, halting speech. He does however retain his ability to understand
what people are saying to him - Broca's aphasia
35. A patient is noted to have no speech and appears to understand little of what is said to him - global aphasia
36. A patient is noted to speak fluently but have difficulty repeating phrases. He is aware of the errors he is making
and comprehension remains normal - conduction aphasia
37. A patient is noted to speak fluently but makes little sense with mulitple word substitutions and neologisms. He also
has difficulty understanding what people are saying to him - Wernicke's aphasia
38. A patient presents with a constant headache associated with nausea. The headache is worse in the morning and when
coughing - raised intracranial pressure
39. A patient with a long history of 'migraines' presents with regular headaches despite using sumatriptan on a daily
basis - medication overuse headache
40. A patient with Huntington's disease is noted to have slurred speech which is difficult to comprehend. He retains the
ability to understand what is being said to him - dysarthria
41. A teenage boy presents with clumsy walking. On examination he has gait ataxia, an intention tremor and loss of
lower limb reflexes - Friedreich's ataxia
42. A woman suddenly falls to the ground then lays motionless - atonic seizures
43. A woman suddenly has the sensation of smelling roses whilst at work. She is conscious throughout - focal aware
seizure
44. A young boy is noted to have occasional periods where he stares blankly in class - absence seizure
45. A young man is noted to have multiple pigmented spots on his torso. There is also some freckling around the axillary
and groin region - neurofibromatosis
46. A young man presents with an acute headache associated with unilateral periorbital edema. On examination there is
a lateral gaze palsy - cavernous sinus thrombosis
47. A young man presents with weakness of his legs shortly after having a severe bout of gastroenteritis. Sensation in
the legs is normal although there is a loss of the deep tendon reflexes - Guillain-Barre syndrome
48. A young man with a history of epilepsy and learning difficulties is noted to areas of depigmented skin on his torso.
He also has roughened patches of skin over the lumbar spine - tuberous sclerosis
49. Absence seizures (male patient) - ethosuximide
50. Acute migraine treatment , treatment of choice: triptan + NSAID or triptan + paracetamol
51. Amygdala lesions may cause Kluver-Bucy syndrome
52. Amyotrophic lateral sclerosis , treatment of choice: riluzole
53. An elderly alcoholic man presents with a persistent headache after falling over the previous day. On examination he
has a fluctuating level of consciousness - subdural haemorrhage
324
M Y Elamin
MBBS, DTM&H, MCTM, MRCPI 1& 2
54. An elderly man presents with a resting tremor affecting his right hand. You also notice a shuffling gait and a general
slowness of movement - Parkinson's disease
55. An elderly man presents with gradual worsening dementia and bradyphrenia associated with a slow, shuffling gait.
He has also recently suffered with urinary incontinence - normal pressure hydrocephalus
56. An elderly man presents with parkinsonism atonic bladder, postural hypotension and cerebellar signs - multiple
system atrophy
57. An elderly man presents with severe pain around his right eye, associated with nausea, redness of the eye, 'misty
vision' and a semi-dilated pupil - acute narrow-angle glaucoma
58. An elderly patient is admitted with acute onset ataxia, nystagmus and sensory loss. On examination he has sensory
loss affecting the left side of the face and right arm - posterior inferior cerebellar artery occlusion (lateral medullary
syndrome)
59. An overweight teenage girl presents with a headache and blurred vision. On examination she is noted to have
papilloedema - idiopathic intracranial hypertension
60. Apomorphine - dopamine receptor agonist
61. Ataxic telangiectasia - defective ATM gene which encodes for DNA repair enzymes
62. Ataxic telangiectasia - dilated capillaries in the eye
63. Ataxic telangiectasia - IgA deficiency
64. Ataxic telangiectasia - recurrent sinopulmonary infections
65. Becker muscular dystrophy - non-frameshift insertion in the dystrophin gene
66. Benign paroxysmal positional vertigo - Dix-Hallpike manoeuvre
67. Benign paroxysmal positional vertigo , treatment of choice: Epley manoeuvre
68. Benign rolandic epilepsy - a child has partial seizures that occur at night
69. Benzhexol - antimuscarinic
70. Benzotropine - antimuscarinic
71. Bilateral arm pain and temperature loss, fine touch normal - syringomyelia
72. Bilateral spastic paresis and loss of pain and temperature sensation - anterior spinal artery occlusion
73. Bilateral spastic paresis, loss of proprioception and vibration sensation and an intention tremor - Friedrich's
ataxia
74. Bilateral spastic paresis, loss of proprioception and vibration sensation and limb ataxia - subacute combined
degeneration of the spinal cord
75. Bilateral vestibular schwannomas - neurofibromatosis type 2
76. Bitemporal inferior quadrantanopia, endocrine dysfunction - craniopharyngioma
77. Bitemporal superior quadrantanopia, endocrine dysfunction - pituitary tumour
78. Bromocriptine - dopamine receptor agonist
79. Bromocriptine may cause hypotension
80. Bromocriptine may cause pulmonary, retroperitoneal and cardiac fibrosis
325
M Y Elamin
MBBS, DTM&H, MCTM, MRCPI 1& 2
81. Cabergoline - dopamine receptor agonist
82. Cafe-au-lait spots, axillary freckles, iris harmatomas, phaeochromacytoma - neurofibromatosis type 1
83. Carbamazepine - binds to sodium channels increases their refractory period
84. Carbamazepine may cause ataxia
85. Carbamazepine may cause diplopia
86. Carbamazepine may cause dizziness
87. Carbamazepine may cause headache
88. Carbamazepine may cause hyponatraemia
89. Carbamazepine may cause lethargy/drowsiness
90. Carbamazepine may cause myelosuppression/agranulocytosis
91. Carbamazepine may cause Steven-Johnson syndrome
92. Cerebellar/retinal haemangioma, renal cysts, phaeochromocytoma - von Hippel-Lindau syndrome
93. Cerebellopontine lesion - schwannoma
94. Chorea, personality changes, no liver disease - Huntington's disease
95. Combination of both upper and lower motor neuron signs - amyotrophic lateral sclerosis
96. Conjugate horizonal gaze palsy, nystagmus in abducting eye - multiple sclerosis
97. Contralateral hemiparesis and sensory loss with the lower extremity being more affected than the upper -
anterior cerebral artery
98. Contralateral hemiparesis and sensory loss with the upper extremity being more affected than the lower,
contralateral homonymous hemianopia and aphasia - middle cerebral artery
99. Contralateral homonymous hemianopia with macular sparing and visual agnosia - posterior cerebral artery
100. Craniopharyngioma - bitemporal hemianopia (lower quadrant defect > upper quadrant defect)
101. Creutzfeldt-Jakob disease - prion proteins induce the formation of amyloid folds resulting in tightly packed
beta-pleated sheets
102. Dantrolene , uses include: neuroleptic malignant syndrome
103. Duchenne muscular dystrophy - frameshift mutation in the dystrophin gene
104. Edrophonium - acetylcholinesterase inhibitor
105. Edrophonium , uses include: the diagnosis of myasthenia gravis
106. Elderly patient dizzy on extending neck - vertebrobasilar ischaemia
107. Elderly patient with a two week history of unilateral headache associated with pain on chewing and brushing
her hair - temporal arteritis
108. Elderly, alcoholic, head injury, insidiuous onset symptom - subdural haematoma
109. Entacapone - COMT inhibitor
110. Epilepsy, depigmented skin, roughened patches of skin over the lumbar spine - tuberous sclerosis
111. Episodes of vertigo, tinnitus and hearing loss - Meniere's disease
112. Episodic severe facial pain, lasts 1 minute - trigeminal neuralgia
326
M Y Elamin
MBBS, DTM&H, MCTM, MRCPI 1& 2
113. Episodic severe unilateral headache, lasts 1 hour, red eye, lacrimation - cluster headache
114. Essential tremor, treatment of choice: propranolol
115. Ethosuximide - blocks T-type calcium channels in thalamic neurons
116. First seizure : DVLA group 1 rules - cannot drive for 6 months
117. Flacid paresis of the intrinsic hand muscles with loss of pain and temperature sensation - syringomyelia
118. Focal seizures (male patient) - lamotrigine or levetiracetam
119. Foot drop, pes cavus, distal muscle weakness/atrophy, stork leg deformity - Charcot-Marie-Tooth disease
120. Friedreich's ataxia - diabetes mellitus
121. Friedreich's ataxia - GAA triplet repeats causing reduced levels of frataxin
122. Friedreich's ataxia - hypertrophic obstructive cardiomyopathy
123. Friedreich's ataxia - kyphoscoliosis
124. Friedreich's ataxia - lateral corticospinal, spinocerebellar tract and dorsal column degeneration
125. Frontal headache which has developed following an upper respiratory tract infection, worse on leaning
forwards - sinusitis
126. Frontal lobe lesions may cause disinhibition
127. Frontal lobe lesions may cause expressive (Broca's) aphasia
128. Frontal lobe lesions may cause inability to generate a list
129. Frontal lobe lesions may cause perseveration
130. Generalised tonic-clonic seizures (male patient) - sodium valproate
131. Head injury, lucid interval - extradural (epidural) haematoma
132. Huntington's disease - CAG triplet repeats causing degeneration of cholinergic and GABAergic neurons in
the striatum of the basal ganglia
133. Huntington's disease - chromosome 4
134. Hypersexuality, hyperorality, increased appetite, visual agnosia - Kluver-Bucy syndrome
135. Intense pain around one eye. Attacks occurs once a day, each episode lasting 1 hour for the past 8 weeks.
Associated with a red and watery eye and a constricted pupil - cluster headache
136. Ipsilateral oculomotor palsy and contralateral weakness of the upper and lower extremity - branches of the
posterior cerebral artery that supply the midbrain
137. Lambert-Eaton myasthenic syndrome - antibodies directed against pre-synaptic voltage gated calcium
channel in the peripheral nervous system
138. Lamotrigine may cause Steven-Johnson syndrome
139. Lethargy, optic neuritis, paraesthesia, spastic weakness - multiple sclerosis
140. Levodopa may cause anorexia
141. Levodopa may cause dry mouth
142. Levodopa may cause dyskinesia
143. Levodopa may cause hypotension
327
M Y Elamin
MBBS, DTM&H, MCTM, MRCPI 1& 2
144. Levodopa may cause lethargy/drowsiness
145. Levodopa may cause palpitations
146. Levodopa may cause psychosis
147. Locked-in syndrome - basilar artery
148. Loss of proprioception and vibration sensation. No motor deficit - neurosyphilis
149. Medial thalamus and mammillary bodies of the hypothalamus lesions may cause Wernicke and Korsakoff
syndrome
150. Metronidazole may cause peripheral neuropathy
151. Migraine prophylaxis (not a woman of childbearing age) , treatment of choice: topiramate or propranolol
152. Migraine prophylaxis (woman of childbearing age) , treatment of choice: propranolol
153. Multiple TIAs over short period of times : DVLA group 1 rules - cannot drive for 3 months
154. Muscle weakness, difficulty relaxing grip, frontal balding, ptosis - myotonic dystrophy type 1
155. Myoclonic seizures (male patient) - sodium valproate
156. Myotonic dystrophy type 1 - CTG trinucleotide repeat of the DMPK gene on chromosome 19
157. Neurofibromatosis - axillary/groin freckles
158. Neurofibromatosis - bilateral vestibular schwannomas
159. Neurofibromatosis - cafe-au-lait spots
160. Neurofibromatosis - Lisch nodules in the iris
161. Neurofibromatosis - phaeochromocytoma
162. Neurofibromatosis type 1 - chromosome 17
163. Neurofibromatosis type 2 - chromosome 22
164. Neuropathic pain (not diabetic) , treatment of choice: amitriptyline or duloxetine or gabapentin or
pregabalin
165. Nitrofurantoin may cause peripheral neuropathy
166. Occipital cortex lesion - homonymous hemianopia with macula sparing
167. Occipital lobe lesions may cause cortical blindness
168. Occipital lobe lesions may cause homonymous hemianopia (with macula sparing)
169. Occipital lobe lesions may cause visual agnosia
170. Ondansetron - 5-HT3 antagonist
171. Ondansetron , uses include: chemotherapy related nausea
172. Ongoing vertigo, tinnitus and hearing loss, absent corneal reflex - acoustic neuroma
173. Onset < 5 years old, pseudohypertrophy of calf muscles, trouble standing from sitting, learning disability,
dilated cardiomyopathy - Duchenne muscular dystrophy
174. Onset > 10 years old, pseudohypertrophy of calf muscles, trouble standing from sitting, no learning disability
- Becker muscular dystrophy
175. Optic neuritis, myelitis, vomiting, aquaporin 4 positive antibody positive - neuromyelitis optica
328
M Y Elamin
MBBS, DTM&H, MCTM, MRCPI 1& 2
176. Parietal lobe lesion - inferior homonymous quadrantanopia
177. Parietal lobe lesions may cause acalculia
178. Parietal lobe lesions may cause alexia
179. Parietal lobe lesions may cause apraxias
180. Parietal lobe lesions may cause astereognosis
181. Parietal lobe lesions may cause Gerstmann's syndrome
182. Parietal lobe lesions may cause inferior homonymous quadrantanopia
183. Parietal lobe lesions may cause sensory inattention
184. Parkinsonism, autonomic disturbance, cerebellar signs - multiple system atrophy
185. Parkinson's disease - loss of neurons in the substantia nigra
186. Phenytoin - binds to sodium channels increases their refractory period
187. Phenytoin may cause ataxia
188. Phenytoin may cause coarsening of facial features
189. Phenytoin may cause diplopia
190. Phenytoin may cause dizziness
191. Phenytoin may cause fever
192. Phenytoin may cause gingival hyperplasia
193. Phenytoin may cause hepatotoxicity
194. Phenytoin may cause hirsutism
195. Phenytoin may cause lethargy/drowsiness
196. Phenytoin may cause lymphadenopathy
197. Phenytoin may cause megaloblastic anaemia
198. Phenytoin may cause myelosuppression/agranulocytosis
199. Phenytoin may cause nystagmus
200. Phenytoin may cause osteomalacia
201. Phenytoin may cause peripheral neuropathy
202. Phenytoin may cause slurred speech
203. Phenytoin may cause toxic epidermal necrolysis
204. Pituitary tumour - bitemporal hemianopia (upper quadrant defect > lower quadrant defect)
205. Postural instability, impairment of vertical gaze, parkinsonism, frontal lobe dysfunction - progressive
supranuclear palsy
206. Prevention of vasospasm after subarachnoid haemorrhage , treatment of choice: nimodipine
207. Procyclidine - antimuscarinic
208. Proximal muscle weakness (repeated contractions lead to increased strength), hyporeflexia, autonomic
symptoms - Lambert-Eaton syndrome
209. Pyridostigmine - acetylcholinesterase inhibitor
329
M Y Elamin
MBBS, DTM&H, MCTM, MRCPI 1& 2
210. Pyridostigmine , uses include: the treatment of myasthenia gravis
211. Ramsay Hunt syndrome , treatment of choice: prednisolone + aciclovir
212. Recent viral infection, ongoing vertigo and nausea - viral labyrinthitis
213. Recurrent brief vertigo triggered by change in head movement - benign paroxysmal positional vertigo
214. Recurrent, non-disabling, bilateral headache, described as a 'tight-band' - tension headache
215. Recurrent, severe headache which is unilateral and throbbing in nature. Associated with nausea and
photosensitivity - migraine
216. Resting tremor, shuffling gait, slowness of movement - Parkinson's disease
217. Restless legs syndrome , treatment of choice: ropinirole
218. Riluzole - prevents stimulation of glutamate receptors
219. Rinne result: AC > BC bilaterally; Weber result: Lateralises to left ear - right sensorineural hearing loss
220. Rinne result: AC > BC bilaterally; Weber result: Lateralises to midline - normal hearing
221. Rinne result: AC > BC bilaterally; Weber result: Lateralises to right ear - left sensorineural hearing loss
222. Rinne result: BC > AC in left ear, AC > BC in right ear; Weber result: Lateralises to left - left conductive
hearing loss
223. Rinne result: BC > AC in right ear, AC > BC in left ear; Weber result: Lateralises to right - right conductive
hearing loss
224. Ropinirole - dopamine receptor agonist
225. Selegiline - MAO-B inhibitor
226. Slowly progressive weakness, sensory ataxia, paresthesias, extensor plantars, brisk knee reflexes, absent
ankle jerks - subacute combined degeneration of spinal cord
227. Sodium valproate may cause alopecia
228. Sodium valproate may cause ataxia
229. Sodium valproate may cause hepatotoxicity
230. Sodium valproate may cause hyponatraemia
231. Sodium valproate may cause pancreatitis
232. Sodium valproate may cause thrombocytopaenia
233. Sodium valproate may cause tremor
234. Sodium valproate may cause weight gain
235. Striatum (caudate nucleus) of the basal ganglia lesions may cause Huntington chorea
236. Stroke or TIA : DVLA group 1 rules - cannot drive for 4 weeks
237. Substantia nigra of the basal ganglia lesions may cause Parkinson's disease
238. Subthalamic nucleus of the basal ganglia lesions may cause hemiballism
239. Sudden occipital headache - subarachnoid haemorrhage
240. Sudden onset severe occipital headache, like being 'kicked in the back of the head' - subarachnoid
haemorrhage
330
M Y Elamin
MBBS, DTM&H, MCTM, MRCPI 1& 2
241. Sudden onset vertigo and vomiting, dysphagia, ipsilateral facial pain and temperature loss, contralateral
limb pain and temperature loss and ataxia - posterior inferior cerebellar artery
242. Sudden onset vertigo and vomiting, ipsilateral facial paralysis and deafness - anterior inferior cerebellar
artery
243. Sumatriptan - 5-HT1B and 1D agonist
244. Sumatriptan - contraindicated by: cerebrovascular disease
245. Sumatriptan - contraindicated by: ischaemic heart disease
246. Temporal lobe lesion - superior homonymous quadrantanopia
247. Temporal lobe lesions may cause auditory agnosia
248. Temporal lobe lesions may cause prosopagnosia (difficulty recognising faces)
249. Temporal lobe lesions may cause superior homonymous quadrantanopia
250. Temporal lobe lesions may cause Wernicke's aphasia
251. Tonic or atonic seizures (male patient) - sodium valproate
252. Transient unilateral loss of vision described as a 'curtain descending' - ophthalmic artery
253. Tremor on outstretched hands, titubation, family history - essential tremor
254. Trigeminal neuralgia , treatment of choice: carbamazepine
255. Tuberous sclerosis - adenoma sebaceum
256. Tuberous sclerosis - cafe-au-lait spots
257. Tuberous sclerosis - depigmented 'ash-leaf' spots
258. Tuberous sclerosis - epilepsy
259. Tuberous sclerosis - learning difficulties
260. Tuberous sclerosis - renal cysts
261. Tuberous sclerosis - retinal hamartomas
262. Tuberous sclerosis - rhabdomyomas of the heart
263. Tuberous sclerosis - roughened patches of skin over lumbar spine (Shagreen patches)
264. Tuberous sclerosis - subungual fibromata
265. Unexplained syncope : DVLA group 1 rules - cannot drive for 6 months
266. Unilateral spastic paresis and loss of proprioception/vibration sensation with loss of pain and temperature
sensation on the opposite side - Brown-Sequard syndrome
267. Urinary incontinence, cognitive impairment, gait abnormality - normal pressure hydrocephalus
268. Von Hippel-Lindau syndrome - cerebellar haemangiomas
269. Von Hippel-Lindau syndrome - chromosome 3
270. Von Hippel-Lindau syndrome - endolymphatic sac tumours
271. Von Hippel-Lindau syndrome - phaeochromocytoma
272. Von Hippel-Lindau syndrome - renal cysts
273. Von Hippel-Lindau syndrome - retinal haemangiomas
331
M Y Elamin
MBBS, DTM&H, MCTM, MRCPI 1& 2
274. Wernicke-Korsakoff syndrome , treatment of choice: thiamine
275. Yellow tinged CSF - subarachnoid haemorrhage
276. Pilocarpine , uses include: glaucoma
277. 'Fasciculations' - think motor neuron disease
278. 5-HT3 antagonists such as ondansetron can predispose to prolonged QT interval and increased risk of
polymorphic VT
279. Chorea is caused by damage to the basal ganglia, in particular the Caudate nucleus
280. Dystrophia myotonica - DM1
a. Distal weakness initially
b. Autosomal dominant
c. Diabetes
d. Dysarthria
281. A combination of thrombolysis AND thrombectomy is recommend for patients with an acute ischaemic stroke
who present within 4.5 hours
282. A facial palsy caused by an upper motor neuron lesion 'spares' the upper face i.e. forehead
283. A rare but recognised adverse effect of lamotrigine therapy is Stevens-Johnson syndrome
284. Absence seizures - good prognosis: 90-95% become seizure free in adolescence
285. Alcohol is a common trigger for cluster headaches
286. All patients with a Bell's palsy should be given oral prednisolone within 72 hours of onset
287. Always replace vitamin B12 before folate - giving folate to a patient deficient in B12 can precipitate subacute
combined degeneration of the cord
288. Anti-NMDA receptor encephalitis is a paraneoplastic syndrome which presents with prominent psychiatric
features
289. Antiplatelets
a. TIA: clopidogrel
b. Ischaemic stroke: clopidogrel
290. Apomorphine - dopamine receptor agonist
291. Asymmetrical symptoms suggests idiopathic Parkinson's
292. Ataxic telangiectasia is characterised by cerebellar ataxia and telangiectasia, onset is in childhood
293. Baclofen and gabapentin are first-line for spasticity in multiple sclerosis
294. Baclofen is an agonist of GABA receptors that acts in the central nervous system
295. Bilateral spastic paresis and loss of pain and temperature sensation - anterior spinal artery occlusion
296. Bitemporal hemianopia: lesion of optic chiasm
a. upper quadrant defect > lower quadrant defect = inferior chiasmal compression, commonly a
pituitary tumour

332
M Y Elamin
MBBS, DTM&H, MCTM, MRCPI 1& 2
b. lower quadrant defect > upper quadrant defect = superior chiasmal compression, commonly a
craniopharyngioma
297. Benign paroxysmal positional vertigo (BPPV)
a. Dix-Hallpike manoeuvre is diagnostic
b. Epley manoeuvre is for treatment
298. Brachial neuritis is characterized by acute onset unilateral severe pain followed by shoulder and scapular
weakness several days later
299. Brain abscess: IV 3rd-generation cephalosporin + metronidazole
300. Breast feeding is acceptable with nearly all anti-epileptic drugs
301. Broca's dysphasia: speech non-fluent, comprehension normal, repetition impaired
302. Brown-Sequard syndrome is a result of lateral hemisection of the spinal cord
303. Brown-Sequard syndrome: ipsilateral weakness, loss of proprioception and vibration sensation, contralateral
loss of pain and temperature sensation
304. CADASIL is a rare cause of multiple cerebral infarctions
305. Carbamazepine is known to exhibit autoinduction. hence when patients start carbamazepine they may see a
return of seizures after 3-4 weeks of treatment
306. Carbamazepine induces its own metabolism via the hepatic microsomal enzyme system CYP3A4 system levels
fall significantly (by about 50%) after several weeks of treatment, which may result in seizure recurrence within
this period of auto-induction. For this reason, the dose should be increased every 2 weeks.
307. Carbamazepine is contraindicated in absence seizures
308. Cardiovascular disease is a contraindication to triptan use
309. Cerebellar abscesses are most commonly caused by otogenic diseases like mastoiditis and sinusitis infections
310. Cerebellar stroke patients can present like they are 'drunk'
311. Cervical spondylitic myelopathy is caused by advanced osteoarthritis of the cervical spine causing
compression of the cervical cord
312. Charcot-Marie-Tooth disease can affect both motor and sensory peripheral nerves
313. Charcot-Marie-Tooth is a cause for distal muscle wasting
314. Chiari malformations are often associated with syringomyelia due to disturbed cerebrospinal fluid flow at the
foramen magnum
315. Cluster headache - acute treatment: subcutaneous sumatriptan + 100% O2
316. Common peroneal nerve lesion can cause weakness of foot dorsiflexion and foot eversion
317. Conduction dysphasia: speech fluent, but repetition poor. Comprehension is relatively intact
318. Confusion, gait ataxia, nystagmus + ophthalmoplegia are features of Wernicke's encephalopathy
319. Consider glycopyrronium bromide to manage drooling of saliva in people with Parkinson's disease
320. Contralateral hemiparesis and sensory loss with the lower extremity being more affected than the upper -
anterior cerebral artery
333
M Y Elamin
MBBS, DTM&H, MCTM, MRCPI 1& 2
321. Creutzfeldt-Jakob disease is characterised by rapid onset dementia and myoclonus
322. CT head showing temporal lobe changes - think herpes simplex encephalitis
323. Dix–Hallpike test: rapidly lower a patient to the supine position with an extended neck. A positive test
recreates the symptoms of benign paroxysmal positional vertigo
324. Drugs for neuropathic pain are typically used as monotherapy, i.e. if not working then drugs should be
switched, not added
325. DVLA advice post multiple TIAs: cannot drive for 3 months
326. DVLA advice post stroke or TIA: cannot drive for 1 month
327. Epilepsy + pregnancy = 5mg folic acid
328. Epilepsy medication for males:
a. generalised seizure: sodium valproate
b. focal seizure: lamotrigine or levetiracetam
329. Episodic eye pain, lacrimation, nasal stuffiness occurring daily - cluster headache
330. Essential tremor is an AD condition that is made worse when arms are outstretched, made better by alcohol
and propranolol
331. Extensor plantars + absent ankle jerk → mixed UMN + LMN signs- motor neuron disease, subacute combined
degeneration of cord, syringomyelia
332. Extradural or subdural haemorrhage? Extradural = lucid period, usually following major head injury.
Subdural = fluctuating consciousness, often following trivial injury in the elderly or alcoholics
333. Facioscapulohumeral muscular dystrophy is an austosomal dominent disorder
334. Facioscapulohumeral muscular dystrophy: facial weakness (e.g. difficulty closing eyes, smiling, blowing),
winged scapula and weakness of the shoulder, upper arm and hip girdle muscles
335. Fluctuating confusion/consciousness? - subdural haematoma
336. Fluctuating consciousness = subdural haemorrhage
337. For thrombectomy in acute ischaemic stroke, an extended target time of 6-24 hours may be considered if there
is the potential to salvage brain tissue, as shown by imaging such as CT perfusion or diffusion-weighted MRI
sequences showing limited infarct core volume
338. Frontal lobe lesions may cause perseveration
339. FVC is used to monitor respiratory function in Guillain-Barre syndrome
340. Focal seizures not responding to first-line drug - try lamotrigine or levetiracetam (i.e. the first-line drug not
already tried) and if neither help then carbamazepine
341. Percutaneous gastrostomy tube (PEG) is the preferred way to support nutrition in patents with motor neuron
disease
342. Gingival hyperplasia: phenytoin, ciclosporin, calcium channel blockers and AML
343. Guillain-Barre syndrome is classically triggered by Campylobacter jejuni infection
344. Head injury, lucid interval - extradural (epidural) haematoma
334
M Y Elamin
MBBS, DTM&H, MCTM, MRCPI 1& 2
345. Headache, fever and focal neurology and suggestive of a brain abscess
346. Hemiballism is caused by damage to the subthalamic nucleus
347. Hemiballism results in involuntary, sudden, jerking movements which occur contralateral to the side of the
lesion
348. Hypertension should not be treated in the initial period following a stroke
349. If subarachnoid haemorrhage is suspected but a CT head done within 6 hours of symptom onset is normal, do
not do an LP, consider an alternative diagnosis instead
350. In status epilepticus, a phenytoin infusion should be given if not responding to benzodiazepines
351. In subacute combined degeneration of the spinal cord, the dorsal columns and lateral corticospinal tracts are
affected
352. Inferior homonymous quadrantanopias are caused by lesions of the superior optic radiations in the parietal
lobe
353. Internuclear ophthalmoplegia is due to a lesion in the medial longitudinal fasciculus
354. Intravenous phenytoin can cause hypotension
355. IV lorazepam is the first-line treatment in patients with early status epilepticus
356. IVIG is generally used first-line for Guillain-Barre syndrome
357. Juvenile myoclonic epilepsy is classically associated with seizures in the morning/following sleep deprivation
358. Klumpke's paralysis: T1 nerve root damage
359. Lambert-Eaton syndrome - antibodies against the voltage-gated Ca2+ channels
360. Lambert-Eaton syndrome or myasthenia gravis? Weakness in Lambert Eaton improves after exercise, unlike
myasthenia gravis; which worsens after exercise
361. Large artery acute ischaemic stroke? Consider mechanical clot retrieval
362. Lateral medullary syndrome - PICA lesion - cerebellar signs, contralateral sensory loss & ipsilateral Horner's
363. Lateral medullary syndrome can be caused by PICA strokes
364. Laughter → fall/collapse ?cataplexy
365. Leg crossing, squatting or kneeling may cause a foot drop secondary to a common peroneal neuropathy
366. Lethargy is a very common early symptom of multiple sclerosis
367. Levodopa should be offered for patients with newly diagnosed Parkinson's who have motor symptoms affecting
their quality of life
368. Lip smacking + post-ictal dysphasia are localising features of a temporal lobe seizure
369. Loss of corneal reflex - think acoustic neuroma
370. Medication overuse headache
a. Simple analgesia + triptans: stop abruptly
b. Opioid analgesia: withdraw gradually
371. Migraine
a. Acute: triptan + NSAID or triptan + paracetamol
335
M Y Elamin
MBBS, DTM&H, MCTM, MRCPI 1& 2
b. Prophylaxis: topiramate or propranolol
372. Miller Fisher syndrome - areflexia, ataxia, ophthalmoplegia
373. Motor neuron disease - riluzole
374. Motor neuron disease - treatment: NIV is better than riluzole
375. Multiple sclerosis diagnosis that requires demyelinating lesions that are separated in space and time
376. Multiple sclerosis is a cause of spastic paraparesis
377. Multiple sclerosis patient with bladder dysfunction - get an ultrasound first to assess bladder emptying
378. Myasthenia gravis - antibodies against acetylcholine receptors
379. Myoclonic seizures: sodium valproate is first-line for males
380. Narcolepsy is associated with low orexin (hypocretin) levels
381. Natalizumab can cause reactivation of the JC virus causing progressive multifocal leukoencephalopathy (PML)
382. Neurofibromatosis type 1 - chromosome 17
383. Neurofibromatosis type 2 is associated with bilateral vestibular schwannomas
384. Nimodipine is used to prevent vasospasm in aneurysmal subarachnoid haemorrhages
385. Non-contrast CT head scan is the first line radiological investigation for suspected stroke
386. Obese, young female with headaches / blurred vision think idiopathic intracranial hypertension
387. On CT imaging, a chronic subdural haematoma will appear as a hypodense (dark), crescentic collection
around the convexity of the brain
388. Ondansetron is a 5-HT 3 serotonin antagonist
389. Otosclerosis is characterised by conductive hearing loss, tinnitus and positive family history
390. Ovarian teratoma is associated with Anti-NMDA receptor encephalitis
391. Painful third nerve palsy = posterior communicating artery aneurysm
392. Parietal lobe lesions may cause acalculia
393. Parietal lobe lesions may cause astereognosis
394. Parietal lobe lesions may cause Gerstmann's syndrome
395. Parkinson's disease - most common psychiatric problem is depression
396. Patients cannot drive for 6 months following a first unprovoked or isolated seizure if brain imaging and EEG
normal
397. Patients with an intracranial extradural haematoma may experience a lucid interval in which they briefly
regain consciousness after the injury before progressing into a coma
398. Patients with von Hippel-Lindau syndrome may develop retinal haemangiomas which present as a vitreous
haemorrhage
399. Percutaneous gastrostomy tube (PEG) is the preferred way to support nutrition in patents with motor neuron
disease
400. Phenytoin induces vitamin K metabolism, which can cause a relative vitamin K deficiency, creating the potential
for heamorrhagic disease of the newborn
336
M Y Elamin
MBBS, DTM&H, MCTM, MRCPI 1& 2
401. Phenytoin use is a cause of the cerebellar syndrome
402. Posterior circulation stroke must always be considered as a differential in a patient presenting with acute vertigo
403. Procyclidine - antimuscarinic
404. Progressive supranuclear palsy: postural instability, impairment of vertical gaze, parkinsonism, frontal lobe
dysfunction
405. Pyridostigmine is a long-acting acetylcholinesterase inhibitor that reduces the breakdown of acetylcholine in
the neuromuscular junction, temporarily improving symptoms of myasthenia gravis
406. Raised ICP can cause a third nerve palsy due to herniation
407. Restless leg syndrome - management includes dopamine agonists such as ropinirole
408. Restless legs syndrome - ferritin is the single most important blood test
409. Ropinirole - dopamine receptor agonist
410. Selegiline - MAO-B inhibitor
411. Sensorineural hearing loss is the most common complication following meningitis
412. SIADH is a common consequence of subarachnoid haemorrhage
413. Sodium valproate may cause tremor
414. Sodium valproate may cause weight gain
415. Spastic paraparesis can be caused by transverse myelitis
416. Stroke and TIA are associated with sudden-onset 'negative' symptoms, migraine is more commonly associated
with 'positive' symptoms
417. Stroke thrombolysis - only consider if less than 4.5 hours and haemorrhage excluded
418. Subarachnoid haemorrhage: if rebleeding is suspected (e.g. sudden worsening of neurological symptoms) →
repeat CT
419. Subthalamic nucleus of the basal ganglia lesions may cause hemiballism
420. Superior homonymous quadrantanopias are caused by lesions of the inferior optic radiations in the temporal
lobe
421. Syringomyelia - spinothalamic sensory loss (pain and temperature)
422. Syringomyelia classically presents with cape-like loss of pain and temperature sensation due to compression of
the spinothalamic tract fibres decussating in the anterior white commissure of the spine
423. Syringomyelia classically presents with cape-like loss of pain and temperature sensation due to compression
of the spinothalamic tract fibres decussating in the anterior white commissure of the spine
424. Temporal lobe lesions may cause auditory agnosia
425. The Barthel index is a scale that measures disability or dependence in activities of daily living in stroke patients
426. The medial longitudinal fasciculus is located in the paramedian area of the midbrain and pons
427. The most common pattern for progression of multiple sclerosis is relapsing-remitting
428. The standard target time for thrombectomy in acute ischaemic stroke is 6 hours
429. There is a repeat expansion of CAG trinucleotide in Huntington's disease
337
M Y Elamin
MBBS, DTM&H, MCTM, MRCPI 1& 2
430. To detect a subarachnoid haemorrhage the LP should be done at least 12 hours after the start of the headache
431. Topiramate can precipitate acute angle closure glaucoma
432. Transient global amnesia - acute onset of anterograde amnesia, patient may keep asking the same questions,
resolves within 24 hours
433. Transverse myelitis can be caused by viral infection - such as varicella, herpes simplex, EBV and HIV
434. Treatment of neuroleptic malignant syndrome - dantrolene
435. Treatment of Ramsay Hunt syndrome consists of oral aciclovir and corticosteroids
436. Trigeminal neuralgia - carbamazepine is first-line
437. Triptans are serotonin 5-HT1B and 5-HT1D receptor agonists
438. Tuberous sclerosis patients develop renal angiomyolipomata which are prone to haemorrhage
439. Typical BPPV history:
a. Vertigo triggered by change in head position (e.g. Rolling over in bed or gazing upwards)
b. May be associated with nausea
c. Each episode typically lasts 10-20 seconds
440. Uhthoff ’s phenomenon where neurological symptoms are exacerbated by increases in body temperature is
typically associated with multiple sclerosis
441. Uraemic polyneuropathy is predominantly sensory
442. Urinary incontinence + gait abnormality + dementia = normal pressure hydrocephalus
443. Use MRI FLAIR sequence in diagnosis of multiple sclerosis vs. MRI STIR in flares of thyroid eye disease
444. V for Vigabatrin - V for Visual field defects
445. Verapamil is used for long-term prophylaxis of cluster headaches
446. Visual field defects:
a. left homonymous hemianopia means visual field defect to the left, i.e. lesion of right optic tract
b. homonymous quadrantanopias: PITS (Parietal-Inferior, Temporal-Superior)
c. incongruous defects = optic tract lesion; congruous defects= optic radiation lesion or occipital cortex
447. Von Hippel-Lindau syndrome is associated with the development of clear-cell renal cell carcinoma
448. Wernicke's dysphasia: speech fluent, comprehension abnormal, repetition impaired
449. 47% of patients with epilepsy achieve seizure control with a single agent. An additional 14% achieve
control with a second or third drug.
450. A burning sensation is typical of a neuropathy affecting the small unmyelinated and thinly myelinated
nerve fibres.
451. A cerebral abscess can present with focal neurological deficits, fevers and reduced consciousness.
452. A classical presentation of Guillain-Barre would be the gradual development of ascending weakness
with autonomic involvement.
453. A clinical presentation of ipsilateral weakness and a loss of position and vibration below the lesion with
contralateral loss of pain and temperature is linked to a diagnosis of a left sided thoracic spinal cord lesion.
338
M Y Elamin
MBBS, DTM&H, MCTM, MRCPI 1& 2
454. A complicated migraine is one which results in hemi sensory or hemi motor findings associated with a
typical migraine presentation.
455. A family history is especially associated with depressive illness and schizophrenia.
456. A family history of polycystic renal disease would suggest a diagnosis of subarachnoid haemorrhage
rather than bacterial meningitis.
457. A history of sleep paralysis, excessive daytime somnolence and cataplexy are suggestive of narcolepsy.
458. A left lacunar infarct will cause right sided motor symptoms.
459. A lesion of the occipital cortex can cause a quadrantic hemianopia.
460. A patent foramen ovale can result in a transient ischaemic attack or stroke when thrombus travels from a
DVT - a 'paradoxical embolus'.
461. A recent history of unsteady gait of sudden onset associated with vomiting and headache, followed by
drowsiness is typical of cerebellar haemorrhage.
462. A reduced rate of paracetemol absorption is caused by gastrointestinal stasis and reduced rate of gastric
emptying.
463. A right homonymous hemianopia is an expected feature of a left posterior cerebral artery occlusion.
464. A TACI comprises higher cortical dysfunction, contralateral homonymous hemianopia and contralateral
motor or sensory deficits.
465. A triad of memory loss, gait difficulties and urinary incontinence will lead towards the diagnosis of
normal pressure hydrocephalus.
466. A unilateral parietal lobe lesion, left or right, causes a contralateral hemihypesthesia, mild hemiparesis,
parietal ataxia, homonymous hemianiopia or inferior quadrantanopia and unilateral impairment of
optokinetic nystagmus.
467. Absent ankle jerks may occur in conditions associated with neuropathy (B12 deficiency, systemic lupus
erythematosus [SLE], cerebrotendinous xanthomatosis) and dorsal root disease (tabes dorsalis).
468. Acid maltase deficiency typically presents with insidious onset of proximal myopathy and early
respiratory muscle weakness.
469. Acoustic neuromas are benign neoplasms that can be resected with a good prognosis.
470. Acute back pain followed by weakness of dorsiflexion of the left foot are assocaited with sensory loss in
the dorsum of the foot which suggests L5 radiculopathy.
471. Acute cerebellar infarct or haemorrhage is associated with acute onset of ataxia, headache and vomiting
and eventually drowsiness
472. Acute intermittent porphyria (AIP) is a rare disorder characterised by abdominal pain and
neuropsychiatric symptoms which usually presents in the 20-40 age group.
473. Adenomas larger than 1.5 cm frequently have suprasellar extension and may displace the optic chiasm.
339
M Y Elamin
MBBS, DTM&H, MCTM, MRCPI 1& 2
474. Adie's tonic pupil, is characteristically seen in young women, and may occur after an episode of zoster
infection. Initially the pupil is large, but over time becomes small and poorly reactive. It is diagnosed with
weak pilocarpine eye drops.
475. Alcohol abuse and diabetes are the commonest causes of peripheral neuropathy in the United Kingdom.
476. Alcohol is a common cause of hypoglycaemia, which can be rapidly fatal if left untreated.
477. Altered cerebral function is the key differentiator between meningitis and encephalitis, though some
features overlap in meningoencephalitis where there are features of parenchymal and meningeal involvement
in some patients.
478. Alzheimer's disease is the most common form of dementia
479. Amyotrophic lateral sclerosis (ALS) causes degeneration of upper (UMN) and lower motor neurones
(LMN), therefore giving a mixed picture with fasciculations and spasticity, weakness and hypo- or
hyperreflexia.
480. An Adie pupil is reactive to accommodation but not to light.
481. An aortic aneurysm may stretch the phrenic nerve and cause a palsy.
482. An inability to generate a list rapidly is typical of frontal lobe dysfunction
483. An incongruous homonymous hemianopia is caused by an optic tract lesion prior to the geniculate body.
484. An insidious onset of memory loss and a CT head demonstrating hippocampal atrophy points to a
diagnosis of Alzheimer's dementia.
485. An intermittent history of confusion and disturbance of physical function in the elderly is common in
chronic subdural haemaorrhage
486. Analgesic misuse headache is one of the commonest causes of chronic daily headache and is caused by
analgesics such as codeine phosphate and paracetemol. Treatment is based on gradually withdrawing
analgesics.
487. Analgesic misuse headaches typically respond to analgesics but then return within a few hours.
488. Angioedema is a side effect of angiotensin-converting enzyme inhibitor (ACEi) and to a lesser extent
angiotensin receptor blocker (ARB). It does not necessarily occur as soon as the medication is started.
489. Ankylosing spondylitis typicaly presents with stiffness, first thing in the morning and after inactivity,
and lower back pain radiating into the buttocks.
490. Anomic aphasia or nominal aphasia results in word finding difficulties.
491. Anterior spinal artery occlusion develops suddenly with symptoms of sudden onset weakness of his legs
associated with urinary retention.
492. Anti GQib antibodies are present in Miller Fisher syndrome.
493. Anticholinergic agents decrease large intestinal motility.

340
M Y Elamin
MBBS, DTM&H, MCTM, MRCPI 1& 2
494. Anticholinergic drugs such as benzhexol remain the treatment of choice in parkinsonian tremor.
495. Anticholinergic treatment (for example, benzhexol) is the treatment of choice for tremor predominantly
Parkinson's disease.
496. Aphemia is a type of aphasia in which there is severe dysarthria and impairment of verbal output. There
is intact comprehension. It is believed to be the result of pars opercularis, inferior pre-Rolandic gyrus or
subcortical lesions.
497. Approximately 50% of epileptic patients achieve seizure control with a single first-line agent.
498. As a general rule, pregnant patient and fetus are at far more risk from uncontrolled seizures than from
any potential teratogenic effect of carbamazepine
499. Atonic seizure, cataplexy, vertebrobasilar TIA, and vestibular pathologies (Meniere disease and
aminoglycoside toxicity) are potential causes of drop attacks.
500. Baclofen causes hallucinations when withdrawn.
501. Becker's muscular dystrophy is an X linked condition and so 50% of males will be affected in a family.
502. Bickerstaff's encephalitis affects the brainstem causing drowsiness, ophthalmoparesis, ataxia and brisk
reflexes.
503. Bilateral upgoing plantar responses would make the diagnosis of idiopathic intracranial hypertension
(IIH) unlikely.
504. Botulinum toxin is the treatment of choice for focal dystonia (such as torticollis, and hemi-facial spasm)
and focal dystonia.
505. BPPV occurs due to otolith detachment into the semicircular canals of the inner ear, and results in
nausea, vertigo and nystagmus.
506. Brain herniation often causes false localising signs due to compression of various areas of the brain
507. Brainstem signs would be expected with a brainstem CVA and impaired conscious level.
508. Broad based gait is associated with cerebellar syndrome.
509. CADASIL is a diffuse disease of small arteries, predominantly in the brain; it may present with
migraine.
510. Carbon monoxide poisoning and ethylene glycol may result in dilated pupils
511. Caroticocavernous fistula is a high pressure shunt of blood between the intracavernous carotid artery and
the cavernous sinus.
512. Carotid artery atherosclerosis is an important cause of iscahemic stroke. Left-sided neurological signs
indicate the symptomatic carotid is on the right side.
513. Carotid artery dissection can cause pulsatile tinnitus, syncope and amaurosis fugax.
514. Cataracts is an expected feature in dystrophia myotonica.
341
M Y Elamin
MBBS, DTM&H, MCTM, MRCPI 1& 2
515. Caudate nucleus pathology is associated with chorea (for example, in Huntington's disease).
516. Central pontine myelinolysis is a common consequence of over-rapid correction of hyponatraemia.
517. Central scotoma is likely with retrobulbar neuritis and optic atrophy seen in multiple sclerosis
518. Cephalic tetanus causes severe dysphagia.
519. Cerebral aneurysms may be associated with polycystic kidney disease
520. Cerebral sinus thrombosis is a diagnosis easy to miss. Multiple small neurological findings may push
towards it.
521. Cervical Myopathy is common in patients with rheumatoid arthritis.
522. Chloride concentration >60mmol/L with sodium concentration lower than that of chloride on two
separate occasions is diagnostic of cystic fibrosis.
523. Chorea is a form of dystonia, which results in involuntary, irregular, flowing movements ('dance-like').
It has multiple causes which can be primary or secondary. The underlying pathology is generally poorly
understood but likely involves an imbalance in the basal ganglia pathways.
524. Chronic inflammatory demyelinating polyneuropathy (CIDP) is a cause of demyelinating
polyneuropathy.
525. Chronic subdural haemorrhage usually presents with progressive symptoms including confusion and
deteriorating mobility.
526. Ciclosporin can cause coarse tremor, in a dose-dependent fashion.
527. CIDP shows progressive weakness and sensory function in the upper and lower limbs.
528. Classic symptoms of Guillain Barre syndrome are weakness, paraesthesiae and hyporeflexia, which
usually reach a peak 2 weeks following onset, and improve by 5 weeks. Marked muscle wasting is therefore
not a usual feature.
529. Clinical guidelines recommend that patients who are under 60 years of age with large cerebral
infarctions arising in the MCA territory should be considered for decompressive hemicraniectomy.
530. Cluster headache has a trigeminal distribution with Ipsilateral autonomic features.
531. Cluster headaches are associated with lacrimation, ptosis, pupil constriction, nasal congestion, redness of
eye, swelling of eyelid.
532. Cluster headaches typically involve unilateral excruciating pain which occurs as the name suggest in
clusters for a relatively short period of time. There is usually associated lacrimation.
533. COCP like many oral contraceptive, may be associated with idiopathic intracranial hypertension (IIH).
534. Common peroneal nerve palsy results in weakness of ankle dorsiflexion and eversion, and reduced
sensation over the lateral aspect of the leg and dorsum of the foot.

342
M Y Elamin
MBBS, DTM&H, MCTM, MRCPI 1& 2
535. Compartment syndrome is a surgical emergency manifest by severe pain exacerbated by stretching the
compartmental muscles and a tense fascial compartment. Loss of pulses is a late sign.
536. Conduction aphasia has poor repetition and naming but intact comprehension and fluent verbal output.
537. Consider a left parietal lobe insult if a patient is found to have difficulty with perception, interpretation
of sensory information and the formation of motor responses.
538. Consider cerebral venous thrombosis in a pregnant patient presenting with headache, nausea and
vomiting. CVT may progress to a reduction in conscious level and eventual coma.
539. Consider meningioma in a patient presenting with slowly progressive weakness associated with a
sensory level.
540. Conus medularis syndrome presents with mixed upper and lower motor neurone signs.. These include
bilateral distal weakness with increased tone and hyper-reflexia, fasciculation.
541. Core symptoms of Ménière's disease are vertigo, tinnitus and fluctuating hearing loss with a sensation of
aural pressure
542. Correlation of brain anatomy and function
543. Cortical blindness is caused by a lesion of the occipital lobe
544. Cortical vein thrombosis typically presents with headache, seizures and focal neurological deficit and is
more common in the postpartum period.
545. CT head is the initial investigation of choice in patients with suspected subdural haematoma.
546. Current UK guidelines recommend carotid endarterectomy for symptomatic patients with greater than
70% stenosis.
547. Damage to the radial nerve results in wrist drop and impaired sensation over the dorsum of the thumb,
548. De Quervain's tenosynovitis is a cause of dorsoradial wrist pain
549. Degenerative changes affecting the intervertebral discs, vertebrae, facet joints, and ligamentous
structures encroach on the cervical spinal canal and damage the cord, especially in patients with a
congenitally small canal.
550. Delerium tremens (DT) starts 48-96 hours after the last drink. Anything more rapid is unlikely to be DT,
however severe.
551. Delirium should be treated with short-term haloperidol or olanzapine, if verbal or behavioural
techniques are not successful.
552. Despite many years of research, the cause of Parkinson's disease is not fully understood. The relative
contribution of genetic and environmental factors still remains unclear.
553. Diabetes insipidus is confirmed with water deprivation testing.
554. Diabetes is a cause of optic neuritis which is usually associated with headache.

343
M Y Elamin
MBBS, DTM&H, MCTM, MRCPI 1& 2
555. Diabetes is the commonest cause of small fibre painful peripheral sensory neuropathy.
556. Differential diagnosis in dementia based on clinical history, including identification of potentially
reversible causes.
557. Disc prolapse typically results in pain, paraesthesia and weakness in the distribution of the nerve root
affected.
558. Dopa-decarboxylase inhibitors help to reduce side effects of levodopa.
559. Dopamine agonists such as ropinirole are the treatment of choice for restless legs syndrome.
560. Downbeat nystagmus (fast phase downwards) suggests a lesion in the lower part of the medulla. It is
therefore typical of the Arnold-Chiari malformation.
561. Doxazosin is an alpha-adrenoceptor blocker. Its side effects include dry mouth, syncope, drowsiness,
oedema, and gastrointestinal (GI) disturbances.
562. Drugs with anti-muscarinic effects may cause urinary retention.
563. DVT can result in a cerebrovascular accident as a result of an atrial or ventricular septal defect
(paradoxical embolus).
564. Early initiation of immunosuppressive treatment has been shown to lead to a good prognosis in cases of
Guillan-Barré syndrome.
565. Early onset of cognitive impairment in association with parkinsonian features is suggestive of Lewy
body disease
566. ECHO and thrombophillia screen shoudl be performed on all young patients with stroke.
567. Edrophonium (Tensilon) is sometimes used in the diagnosis of myasthenia gravis and would be expected
temporarily to improve symptoms whereas plasmapheresis would treat the disease in an aim to put the
disease into remission.
568. Elevated prolactin levels is a recognised association of acromegaly.
569. Enteroviruses and mumps are the commonest causes of viral meningitis.
570. Entrapment of the median nerve by pronator teres causes a median nerve neuropathy, which is worse
during pronation of the forearm.
571. Essential tremor is the commonest cause of head tremor, and can be reduced with propranolol.
572. Facial pain or sensory disturbance in the trigeminal nerve distribution can occur with a petrous apex
lesion.
573. Features of autonomic dysfunction, with postural hypotension and urinary retention, coupled with
parkinsonism point towards a diagnosis of multi-system atrophy.
574. Features of relief with alcohol, postural tremor and a slight and rather benign deterioration over five
years suggests a diagnosis of benign essential tremor.

344
M Y Elamin
MBBS, DTM&H, MCTM, MRCPI 1& 2
575. Fibromyalgia is becoming a recognised medical diagnosis, and is based on the presence of pain in all
four quadrants of the body, as well as tenderness in 11 of 18 anatomically defined trigger areas.
576. Finger flexors and extensors are supplied by C8.
577. Fingolimod is a new agent for multiple sclerosis. It is the only oral medication available to date.
Fingolimod has been reported to be a cannabinoid receptor antagonist as well as a ceramide synthase
inhibitor.
578. Fortification spectra (jagged lines resembling battlements) and teichopsia (flashes) are common features
of migraine.
579. Foster Kennedy's syndrome is a combination of optic atrophy and central scotoma, contralateral
papilloedema and anosmia. It is caused by olfactory and optic nerve compression and raised ICP.
580. Frontal lobe brain damage is classically associated with personality change and deterioration in intellect,
but perseveration may also occur.
581. Frontal lobe syndrome usually presents with personality changes, urinary and faecal incontinence and
anosmia
582. Frontotemporal dementia (FTD) appears to begin in the orbitofrontal cortex and anterior cingulate
regions of the frontal lobes, along with the anterior insula.
583. FVC is the best way to monitor respiratory function in any neurological disorders that can affect the
respiratory muscles
584. Gabapentin is excreted renally; those with poor kidney function are at risk of side effects due to raised
plasma concentrations.
585. Gelastic seizures are typically the result of a hypothalamic harmatoma.
586. Gelastic seizures should be suspected in cases of erratic laughing or crying. It can be hard to identify in
young children but there is usually associated automatisms such as fidgeting or lip smacking or change in
sensorium.
587. Giant cell arteritis is a clinical emergency. High doses of steroids (1 mg/kg) should be initiated as soon
as possible to prevent visual loss, which is usually irreversible.
588. Giant cell arteritis often features elderly patients with headaches and ocular symptoms.
589. Global aphasia presents with the inability to comprehend, produce fluent speech or repeat what is being
asked. It is typically the result of a perisylvian insult resulting in damage of Broca’s, Wernicke’s and the
arcuate fasiculus.
590. Guillain-Barre syndrome is an immune-mediated acute inflammatory progressive demyelinating
polyneuropathy which has the potential to affect the respiratory muscles; therefore vital capacity should be
monitored.
591. Guillain-Barré syndrome is often preceded by an episode of infection such as viral (cytomegalovirus
[CMV]) or bacterial (Campylobacter).

345
M Y Elamin
MBBS, DTM&H, MCTM, MRCPI 1& 2
592. Haemorrhagic lesions of the temporal lobe are typical for Herpes simplex virus infection.
593. Haloperidol is contraindicated in Lewy Body dementia due to risk of severe neuroleptic reactions
594. Hashimoto's encephalopathy can result in altered mental state, myoclonus and ataxia, and responds to
steroids.
595. Hemiballismus is a characteristic clinical feature of contralateral subthalamic nucleus infarction.
596. Hereditary motor sensory neuropathy results in a demyelinating polyneuropathy.
597. Herpes simplex encephalitis (HSE) often presents subacutely over several days with declining cerebral
function.
598. Herpes simplex encephalitis produces a typical EEG pattern with lateralised periodic discharges at 2 Hz.
599. High CK, proximal muscle weakness and skin changes suggest dermatomyositis.
600. Hippus is papillary athetosis. It is typically a benign finding. It is a spasmodic rhythmical dilation and
contraction of the pupil.
601. History of headache and amenorrhoea suggest a prolactinoma or non-functioning tumour.
602. HIV infection can cause dementia that progresses over a longer time period than progressive multifocal
leukoencephalopathy (PML).
603. Holmes-Adie and 3rd nerve palsies result in a dilated pupil
604. Homonymous inferior quadrantanopia is caused by a lesion of parietal lobe.
605. HSV encephalitis typically presents with headache, and impairment of consciousness. Temporal lobe
seizures are also possible.
606. Huntington's disease is a neurodegenerative genetic disorder that is autosomal dominant.
607. Huntington's disease is an autosomal dominant genetic condition, resulting in progressive chorea
preceded by a prodromal phase of altered behaviour.
608. Idiopathic Intracranial Hypertension (IIH) often present in obese women of child-bearing group who
present with headache, transient visual obscurations, intracranial noises (pulsatile tinnitus). Other symptoms
may be photopsia, back pain, retrobulbar pain, diplopia and sudden visual loss.
609. Idiopathic intracranial hypertension (IIH) which maybe associated with reduction of vision with
manoeuvres that raise intracranial pressure and is associated with sixth nerve palsies.
610. Idiopathic intracranial hypertension normally presents with headache, blurred vision, dizziness,
horizontal diplopia and transient visual loss.
611. If visual symptoms are present in oatients with giant cell arteritis (GCA), intravenous
methylprednisolone should be given wihhout delay to preserve vision.
612. Impairment of adduction in both eyes signifies an internuclear ophthalmoplegia.

346
M Y Elamin
MBBS, DTM&H, MCTM, MRCPI 1& 2
613. In Broca's or non-fluent or expressive aphasia the patient is unable to name objects with poor
comprehension and repetition.
614. In cases of suspected acoustic neuroma, an MRI brain is the investigation of choice.
615. In herpes simplex encephalitis, MRI brain normally shows changes in the temporal lobes.
616. In herpes simplex encephalitis, MRI brain normally shows changes in the temporal lobes.
617. In idiopathic Parkinson's disease, there is a loss of more than 50% of the dopaminergic neurones in the
substantia nigra when motor symptoms begin to develop.
618. In optic neuropathy, accommodaton response is unaffected.
619. Indications for carotid artery surgery versus anti-platelet therapy alone following TIA/amarousis fugax
620. Indications for carotid endarterectomy
621. Indications for carotid endarterectomy
622. Indications for carotid endarterectomy
623. Indications for carotid endarterectomy
624. Indications for carotid endarterectomy
625. Indications for carotid endarterectomy.
626. Infarction of the posterior inferior cerebellar artery results in lateral medullary syndrome.
627. Insoluble alpha-synuclein accumulation is the underlying pathology in dementia with Lewy bodies.
628. Interestingly, postconcussion syndrome is more often described after mild traumatic brain injury (TBI),
but may happen after moderate and sever TBIs as well as after whiplash injury. Mild TBI is defined by a
GCS of 13-15 30 minutes after a blunt force producing non-penetrating head trauma.
629. Internuclear ophthalmoplegia is due to a lesion at the medial longitudinal fasciculus and in men the most
likely explanation is multiple sclerosis (MS).
630. Ipsilateral signs of right sided thoracic spinal cord lesion include pyramidal weakness and dorsal column
dysfunction (joint position and light touch) and contralateral signs include spinothalamic dysfunction
(pinprick and temperature).
631. It is well recognised that carbamazepine is a P450 enzyme inducer but it is less well appreciated that it
causes auto-induction and so would require increase in dose to maintain the same therapeutic concentration.
632. IV dextrose administration can exhaust vitamin B reserves. B vitamins must be administered to all
alcoholic patients requiring dextrose.
633. IV mannitol is the treatment of choice for raised intracranial pressure; it leads to reduced intracranial
pressure and improves cerebral blood flow.
634. IV normal saline is the immediate treatment for patients presenting with dengue fever.

347
M Y Elamin
MBBS, DTM&H, MCTM, MRCPI 1& 2
635. Korsakoff's is associated with short term memory loss with subsequent compensatory confabulation by
the patient.
636. Levetiracetam (Keppra) is rapidly absorbed orally, it does not affect hepatic enzymes but dose reduction
is required in renal failure.
637. Listeria meningitis produces neurodeficits in addition to signs of meningism and affects extremes of age.
CSF may show pleocytosis; from 100% polymorphs to 100% mononuclear cells.
638. Low serum caeruloplasmin (<0.1 g/L) with worsening neurology is highly suggestive of Wilson's
disease.
639. Lumbar puncture (LP) is not usually required unless the history is suggestive and the CT is normal.
640. Lyme disease is spread by the bite of ticks of the genus Ixodes that are infected with Borrelia
burgdorferi.
641. Magnetic resonance imaging (MRI) of the brain in vCJD may show signal hyperintensity in the pulvinar
(pulvinar sign) or in both pulvinar and dorsomedial thalamus (hockey stick sign).
642. Major depression following traumatic brain injury has been associated with reduced gray matter volume
in the lateral left prefrontal cortex.
643. Maple syrup urine disease may present with feeding problems, poor growth and developmental delay
during infancy, or may present much later in life with learning difficulties.
644. Meralgia paraesthetica is a consequence of lateral cutaneousnerve of the thigh entrapment.
645. Metoclopramide is a dopamine receptor antagonist that can induce parkinsonism.
646. Minimal right facial weakness, impaired elevation of the right shoulder, with relatively preserved right
hand strength, and global weakness in the right leg which appears to be maximal in the foot, is a classical
presentation of anterior cerebral artery stroke.
647. Mitochondria has its own self-replicating DNA. It replicates using proteins found in the nucleoids which
also house the DNA.
648. Mononeuritis multiplex, fever, hypertension, and nephritic renal involvement which is most consistent
with a diagnosis of polyarteritis nodosa.
649. Most cases of optic neuritis are retrobulbar and hence there are no abnormalities on fundoscopy.
650. Motor neuron disease is the most common cause of presentations with fasciculation, muscle wasting and
weakness, but no sensory impairment.
651. MRI is the investigation which will show periventricular white matter plaques of different ages and in
different locations, to help in the diagnosis of MS.
652. Multiple myeloma often presents with bone pain and hypercalcaemia, with or without renal impairment
(usually secondary to cast nephropathy).

348
M Y Elamin
MBBS, DTM&H, MCTM, MRCPI 1& 2
653. Multi-system atrophy includes three syndromes that usually overlap: Strionigral degeneration leading to
parkinsonism, autonomic failure, and olivopontocerebellar degeneration.
654. Muscles of facial expression are controlled by the seventh nerve.
655. Mutations in either the parkin gene or UCHL1 lead to impaired protein degradation.
656. Myasthenia gravis is well known to be associated with other autoimmune diseases such as pernicious
anaemia, thyroid disease and rheumatoid arthritis.
657. Myotonic dystrophy is an autosomal dominant inherited progressive multi-system condition which
classically presents with myotonia, frontal baldness and ptosis.
658. Nasal regurgitation, coughing and choking episodes during meals, dysphagia that is worse with liquids
than solids and dysarthria indicate neurogenic dysphagia.
659. Neuralgic amyotrophy is a brachial plexopathy (usually upper brachial plexus) usually preceded by an
infective picture.
660. Neurodegenerative disease
661. New variant Creutzfeldt-Jakob disease (CJD) usually presents in the twenties or thirties, with psychiatric
and painful sensory symptoms in the lower limbs.
662. NICE guidelines state that an oral triptan and an NSAID or paracetamol should be used for the acute
treatment of migraine.
663. Non-steroidals and antiemetics are the acute treatment of choice for hemiplegic migraine.
664. Normal pressure hydrocephalus causes the triad of dementia, gait abnormality and urinary incontinence.
665. Oculogyric crisis should be treated with procyclidine (usually IV or IM) or benztropine.
666. Of the Plasmodium species, only Plasmodium falciparum invades the central nervous system, causing
cerebral malaria.
667. Opercular syndrome is a facio-labio-pharyngo-masticatory syndrome with automato-voluntary
dissociation.
668. Optic ataxia describes an inability to control hand-eye coordination.
669. Optic atrophy is not a feature of MND.
670. Optic chiasm lesions characteristically produce a bitemporal hemianopia
671. Optic neuritis can be associated with multiple sclerosis, and can result in central scotoma, reduced visual
acuity and papilloedema.
672. Other than reassurance, no specific therapy for transient global amnesia is required.
673. Pancoast's tumour causes pain in the C8 and T1 distribution and Horner's syndrome.
674. Paracetamol absorption is reduced during migraine attacks and reduced absorption is associated with
increased nausea.
349
M Y Elamin
MBBS, DTM&H, MCTM, MRCPI 1& 2
675. Paraneoplastic syndromes are a result of antibody generation from or against malignant cells attacking
normal tissue. Examples include antineuronal antibodies directed against the Purkinje cells of the cerebellum
leading to cerebellar syndrome.
676. Paraspinal abscesses can develop after endocarditis or infection of an indwelling line, and result in
neurological deficits in the distribution of the nerve roots affected.
677. Patients receiving phenytoin may develop pseudolymphoma or, rarely, malignant lymphoma and
mycosis-fungoides-like lesions.
678. Patients with a history of drug abuse are at high risk of developing seizures, which occur
characteristically within 48 hours of alcohol discontinuation, usually in the morning.
679. Patients with a hypercoagulable state (pregnancy) and papilloedema with neurological signs should be
investigated for cerebral venous thrombosis by MRI or Ct with contrast.
680. Patients with a TIA should receive a CT/MRI head and carotid duplex within a week. If carotid
endarterectomy is deemed necessary then surgery should be performed within two weeks.
681. PFO can result in paradoxical emboli, which is a cause of cerebral infarcts in the young.
682. Pharyngeal diphtheria presents with fever, sore throat, cervical lymphadenopathy and an adherent,
greyish pharyngeal membrane.
683. Phenytoin is well known to cause neurological side effects such as peripheral sensory neuropathy and
cerebellar ataxia.
684. Phenytoin toxicity manifests as CNS dysfunction, particularly confusion, nystagmus, and ataxia.
685. Pituitary apoplexy is due to haemorrhage/infarct into pituitary gland with associated endocrine and
neurological symptoms.
686. Post head injury neurogenic diabetes insipidus is caused by reduced antidiuretic hormone secretion.
687. Posterior communicating artery aneurysms can present with an oculomotor nerve palsy with pupillary
involvement.
688. Post-stroke seizures should be managed as per idiopathic epilepsy, with careful consideration of drug
interactions. Sodium valproate or lamotrigine can be used for generalised seizures.
689. Progressive multifocal leukoencephalopathy (PML) can be diagnosed via CSF PCR for the JC virus.
690. Progressive multifocal leukoencephalopathy (PML) is a demyelinating disease seen in advanced
HIV/AIDS and caused by the JC virus.
691. Progressive multifocal leukoencephalopathy (PML), is a condition that can develop in
immunocompromised patients, such as those with AIDS.
692. Progressive supranuclear palsy (PSP) is a tauopathy similar to Alzheimer's disease.
693. Prolonged fasting is used to investigate suspected insulinoma.

350
M Y Elamin
MBBS, DTM&H, MCTM, MRCPI 1& 2
694. Prolonged neuromuscular junction (NMJ) blockade may be exacerbated by both corticosteroids and
magnesium.
695. Randomised controlled trials have shown that human immunoglobulins and plasma exchange improve
outcome in Guillain-Barré Syndrome (GBS).
696. Recognition of a stroke syndrome and the associated neuroanatomical area affected
697. Rett syndrome is a neurodevelopmental disorder mostly affecting girls. There is repetitive hand
movements such as hand wringing syndrome related to the MECP2 gene on the X chromosome.
698. Right sided pathology affecting the IX, X and XI cranial nerves is suggestive of a lesion at the right
jugular foramen.
699. Ropinirole and rotigotine are some of the effective drugs in the treatments of restless legs, whilst
gabapentin may be effective as a second line alternative
700. Sciatic nerve palsy is a known complication of a total hip replacement; it causes global weakness of the
ankle due to the involvement of both of its branches: tibial nerve (plantaflexion and inversion) and common
peroneal nerve (dorsiflexion and eversion).
701. Single fibre electromyography (EMG) is the most sensitive test for myasthenia gravis. It simultaneously
records the variability in potentials of two muscle fibres innervated by an individual axon: jitter.
702. Sodium valproate is associated with an increased risk of neural tube defects when used in pregnancy
703. Sodium valproate is associated with weight gain, tremor, hair loss, POD and teratogenicity.
704. Sodium valproate, lamotrigine, and topiramate are the treatments of choice for absences, generalized
tonic clonic seizures and myoclonus in primary generalized epilepsy.
705. Subacute combined degeneration of the cord is secondary to B12 deficiency.
706. Subdural haematoma can be associated with gradual onset of headaches, memory loss, personality
change, dementia, confusion and drowsiness (all of which can be fluctuating).
707. Sudden onset of painless monocular visual loss in patients aged 50 or more is commonly due to
ischaemic optic neuropathy.
708. Sudden onset of visual loss is often due to a vascular cause
709. Sweating over the cheek is unaffected by a lesion of the facial nerve in the internal auditory meatus.
710. Syringomyelia is associated with headaches, motor and bladder symptoms. Normal pressure
hydrocephalus does not present with headache but memory loss.
711. Syringomyelia typically causes loss of reflexes, spinothalamic sensory loss, and weakness. It can be
asymmetrical initially.
712. Temporal lobe epilepsy can present with gastric rising and repetitive mouth movements.
713. Temporal lobe lesions result in a contralateral homonymous superior quadrantanopia.

351
M Y Elamin
MBBS, DTM&H, MCTM, MRCPI 1& 2
714. Tetrabenazine works as a VMAT-inhibitor (vesicular monoamine transporter 2), involved in
transportation of monoamines.
715. The Achilles reflex tests the S1/2 nerve root, and sciatic nerve.
716. The best way to test short term memory is to ask the patient to recall new information in the next few
minutes.
717. The carotid artery bifurcates at C4.
718. The causes of absent ankle reflexes and extensor plantars include subacute combined degeneration of the
cord (posterior column signs, positive Romberg's sign, anaemia, splenomegaly).
719. The classic story of a lower brachial plexus injury is a sudden upward movement of the abducted arm.
720. The classic triad of symptoms of carotid dissection are unilateral (ipsilateral) headache, ipsilateral
Horner's syndrome and contralateral hemisphere dysfunction (aphasia, neglect, visual disturbance,
hemiparesis).
721. The classical clinical features of Horner's syndrome are ptosis, miosis, and anhidrosis.
722. The combination of diplopia, hemiparesis and a lower motor neurone facial nerve lesion should lead you
to consider a pontine stroke.
723. The common peroneal nerve gives sensory supply to the lateral part of the lower leg and the dorsum of
the foot
724. The common peroneal nerve is involved when presented with acute onset of left foot drop along with
weakness of ankle dorsiflexion and eversion.
725. The common peroneal nerve supplies sensory innervation to the dorsum of the foot, but the 5th toe is
spared.
726. The commonest cause of a 'paradoxical embolus' is a patent foramen ovale. Although atrial septal
defects are also a cause, they are more rare
727. The cumulative probability of developing MS by 15 years after onset of optic neuritis is 50%.
728. The Duffy blood group on the red blood cells acts as a receptor for P. vivax.
729. The EEG in sporadic CJD may show significant abnormalities involving deep brain areas such as the
thalami.
730. The femoral nerve supplies quadriceps femoris muscle and therefore the knee-jerk reflex
731. The Glasgow Coma Scale assesses a patient's level of consciousness by assessing their eye opening (out
of 4), their verbal responses (out of 5) and their motor responses (out of 6).
732. The Holmes-Adie pupil usually occurs after a herpes zoster infection, which initially leads to an
abnormally large pupil poorly reactive to light with a normal near reflex.
733. The incidence of Down syndrome is roughly 1 in 1200 births for women under the age of 30 rising to 1
in 60 by the age of 42.
352
M Y Elamin
MBBS, DTM&H, MCTM, MRCPI 1& 2
734. The L5/S1 disc can compress the S1 nerve root leading to symptoms of sciatica, loss of plantarflexion
and impaired ankle jerk.
735. The most appropriate investigation for a suspected subdural haematoma would be CT head scan.
736. The most common features of myasthenia gravis include ptosis, diplopia and ophthalmoplegia.
737. The most common presenting symptom of chronic SDH is headache.
738. The most common symptoms of cerebellar haemorrhage are severe nausea and vomiting, and ataxia.
Headache may be severe. Patients with cerebellar haemorrhage can rapidly become comatose within hours
after the onset from herniation, because of its limited space in the posterior fossa.
739. The most likely localisation of the lesion is around the optic chiasm spreading up the right optic nerve
hence a bitemporal visual field defect with involvement of the right optic nerve.
740. The most widely recognised limitation of the Barthel index is floor and ceiling effects.
741. The NICE guidelines on stroke suggest that we use terminology to describe the type of impairment in
accordance with the WHO International Classification of Functioning, Disability and Health.
742. The palmar cutaneous branch of the median nerve lies superficial to the flexor retinaculum and is spared
in carpal tunnel syndrome.
743. The posterior cerebral artery supplies the occipital lobe, which contains most of the visual cortex.
744. The posterior inferior cerebellar artery is affected in lateral medullary syndrome.
745. The presence of a swollen optic disc suggests ischaemic optic neuropathy.
746. The presentation of myopathy is characterised by proximal weakness with normal reflexes and sensation
and the absence of fasciculations.
747. The root supply to the peroneal muscles (which control eversion of the foot and which also participate in
the reflex arc of the ankle jerk reflex) is S1 via the tibial and superficial peroneal nerves
748. The specific test for neuromyelitis optica is the NMO antibody which is against aquaporin 4.
749. The spinothalamic tracts are supplied principally by the anterior spinal artery.
750. The subarachnoid space is in between the arachnoid mater and the pia mater. Vascular malformations
and aneurysms typically bleed in the subarachnoid space.
751. The suggestion of bilateral leg pain, weakness and reduced knee reflexes with an impaired fasting
glucose concentration suggests a diagnosis of diabetic amyotrophy.
752. The three hallmarks of adhesive capsulitis are insidious shoulder stiffness, severe pain (even at night),
and near complete loss of passive and active external rotation of the shoulder.
753. The thumb and index finger are within the C6 dermatome.
754. The treatment of choice for Neuroleptic malignant syndrome is dantrolene and bromocriptine.
755. The treatment of choice for neuropathic pain is amitriptyline/gabapentin.
353
M Y Elamin
MBBS, DTM&H, MCTM, MRCPI 1& 2
756. The tremor of Parkinson's disease is a rest tremor with low to moderate frequency 3 to 6 HZ.
757. The tremor of Parkinson's disease is typically asymmetrical, a feature which can help distinguish it from
other causes of tremor.
758. The triad of parkinsonism, vertical gaze palsy and cognitive impairment suggests progressive
supranuclear palsy (PSP).
759. The typical tremor associated with Parkinson's disease is 4-6 Hz, although in a minority, the tremor may
be faster at 8 Hz.
760. The ulnar nerve supplies all of the intrinsic hand muscles except for those of the thenar eminence and
the first and second lumbricals which are innervated by the median nerve.
761. The upper part of the visual field flows through the temporal lobe, whilst the inferior part flows through
the parietal lobe. This can be remembered with the mnemonic, PITS: Parietal Inferior, Temporal Superior.
762. The use of AED in patients receiving warfarin requires careful elimination of possible drug interactions.
763. The usual source of somatosensory seizures is the parietal lobe, with symptoms of an intermittent
tingling sensation.
764. The vagus nerve supplies the palatal muscles.
765. The VIth cranial nerve provides motor supply to the lateral rectus muscle of the eye.
766. There is an association between myasthenia gravis and thyroid disease, pernicious anaemia, systemic
lupus erythematosus and rheumatoid arthritis.
767. There is an increased risk of neural tube defects associated with valproate.
768. Third nerve palsy typically presents with a dilated, unreactive pupil and an eyeball that is displaced
"down and out"
769. Thrombolysis with alteplase can be given up to 4.5 from the known onset of symptoms as long as a
haemorrhagic stroke is excluded and there are no contraindications to thrombolysis.
770. Tilt table test is a useful test to support the diagnosis of vasovagal syncope.
771. Tourette syndrome presents before 18 years of age. Diagnosis criteria requires diagnosis require multiple
motor and one or more vocal tics, showing themselves over a year, with not more than three consecutive
months tic free.
772. Transcortical sensory aphasia is characterised by impaired auditory comprehension with intact repetition
and fluent speech, and is caused by damage to the temporal lobes.
773. Transcortical sensory aphasia is similar to transcortical motor aphasia in that there is good repetition but
comprehension and fluency are poor.
774. Transient global amnesia lasts less than 24 hours and normal cognition and an awareness of personal
identity is retained

354
M Y Elamin
MBBS, DTM&H, MCTM, MRCPI 1& 2
775. Transverse myelitis describes a heterogeneous group of conditions that are characterised by acute or
subacute motor, sensory and autonomic spinal cord dysfunction.
776. Treatment of Bell's palsy
777. Tuberous sclerosis, is inherited in autosomal dominant fashion, with responsible defects having been
identified on both chromosome 9 and chromosome 16.
778. Two-thirds of cases of Cheyne-Stokes breathing appear to have heart failure.
779. Ulnar nerve damage at the wrist results in wasting of the intrinsic hand muscles (other than the lateral
two lumbricals) and the hypothenar eminence. This produces the classic claw hand.
780. Ulnar neuropathy causes ring and little finger paraesthesia, hypothenar wasting, and weak thumb
adduction.
781. Up to 12% of patients taking sodium valproate report significant hair loss in clinical trials.
782. Urgent CT will confirm diagnosis in 95% of patients with subarachnoid haemorrhage
783. Variant CJD is a transmissible spongiform encephalopathy, associated with the accumulation of an
abnormal isoform of prion protein in the CNS.
784. Vasovagal syncope is common during dental procedures, mainly induced by pain (as the dentist started
drilling).
785. Vasovagal syncope occurs when standing, with very minimal abnormal movements and a rapid
recovery.
786. Venous sinus thrombosis is associated with the oral contraceptive pill, the post partum period and other
hypercoagulable states.
787. Venous sinus thrombosis is more common in young women and it may present with headaches or
symptoms of raised intracranial pressure.
788. Vitamin B12 deficiency can result in a variety of neurological symptoms due to peripheral neuropathy
and subacute combined degeneration of the cord.
789. Voltage gated calcium channels antibodies is the investigation most likely to confirm Lambert-Eaton
syndrome.
790. Wallenberg's syndrome/lateral medullary syndrome is caused by the occlusion of the posterior inferior
cerebellar artery.
791. Wallerian degeneration is degeneration of the portion of the nerve distal to the injury. It occurs
following axonal injury in both the peripheral and central nervous systems and usually begins within 24-36
hours of injury.
792. Wasting of the muscles, fasciculation, and hyperreflexia, with no loss of sensation, are found in motor
neurone disease
793. Weakness of finger flexion would be the result of a spinal lesion at the level of C8.

355
M Y Elamin
MBBS, DTM&H, MCTM, MRCPI 1& 2
794. Weakness of wrist extension with wrist drop and weakness of finger extension are typical of a radial
nerve lesion.
795. When back pain is caused by a L5/S1 disc prolapse, the S1 nerve root may be affected
796. Whipple's disease is caused by Tropheryma whippleii.
797. Wilson's disease is a rare disorder of copper metabolism which is inherited as an autosomal recessive
disease.
798. Wilson's disease is an autosomal recessive condition which causes build up of copper in the body.
799. Wilson's disease is associated with parkinsonian features due to copper deposits within the substantia
nigra.
800. 60% of patients with MERRF syndrome present with generalised myoclonus.
801. A brain biopsy is the definitive diagnostic test for PML.
802. A cerebral perfusion pressure of 61-80 mmHg is recommended in SAH to prevent watershed infarcts as
well as worsening of the bleed.
803. A history of progressive weakness and loss of tendon reflexes associated with a history of recent
infection suggests Guillain-Barré syndrome (post-infectious polyradiculopathy).
804. A lumbar puncture is needed to confirm high intracranial pressure (greater than 25 cm of H2O) in the
diagnosis of IIH as well as normal CSF analysis.
805. a well circumscribed high attenuation lesion with some surrounding oedema on a non contrast CT is
consistent with an intracranial haematoma.
806. ABCD2 score aims to identify those at high risk of stroke following a TIA.
807. Acoustic neuromas are the commonest cerebellopontine angle tumour.
808. Acute monocular painless visual loss is a recognised sign of cholesterol embolism.
809. Acute onset bilateral paraplegia is characteristic of spinal artery infarction.
810. Acute onset of weakness with areflexia and high CSF protein with normal cell count are typical features
of GBS.
811. Acute subdural haemorrhage is characterised by a convex peripheral high attenuation area with
effacement of the sulci and mass effect.
812. ADEM presents with a short history of flu-like illness, with subsequent development of focal neurology,
fever and encephalopathy
813. Adenocarcinomas of the cervix, lung, ovaries, pancreas, bladder, and stomach account for
approximately 70% of the cancers associated with inflammatory myopathies.
814. ALS is a mixture of lower motor neurone signs in the upper arms and upper motor neurone signs in the
legs.
356
M Y Elamin
MBBS, DTM&H, MCTM, MRCPI 1& 2
815. Aminoglycosides may impair neuromuscular transmission and should not be given to patients with
myasthenia gravis
816. An acute history of unsteadiness, diplopia, dizziness and drowsiness in a patient with access to anti-
epileptics suggest phenytoin toxicity.
817. An obese female patient with headaches and papilloedema should prompt investigation for IIH
818. Anion gap = (Na + K) - (Cl + HCO3); normal range 7-17 mmol/L.
819. Anterior cord syndrome causes UMN and LMN signs acutely.
820. Anti-social personality disorder presents with a history of violence, impulsive acts and detachment from
friends or family.
821. Arnold-Chiari malformation is the commonest cause of down beating nystagmus
822. Asymmetrical upper and lower motor neuron signs with the absence of sensory impairment points
towards Motor Neuron Disease
823. Atrial septal defect with a deep vein thrombosis (DVT) can lead to a paradoxical embolus.
824. Auras in migraine can be negative or positive.
825. Autonomic dysreflexia (AD) is a syndrome of massive imbalanced reflex sympathetic discharge
occurring in patients with spinal cord injury (SCI)
826. Basilar migraine is a self terminating stroke mimic.
827. Bechet's disease will produce a positive pethergy test.
828. Beta-interferon can reduce the relapse rate by approximately one third for the first two years of
treatment in relapse remitting MS.
829. Bilateral cerebral infarcts with angioid streaks on slit light examination are suggestive of PXE.
830. Botulinum toxin type A (or trade name Botox®) acts by decreasing transmission of acetylcholine across
the neuromuscular junction and can induce temporary flaccid paralysis.
831. Botulism is caused by a neurotoxin that binds irreversibly to the presynaptic membranes of peripheral
neuromuscular and autonomic nerve junctions.
832. Brachial plexus injury is common following trauma and leads to motor and sensory disturbance across
multiple dermatomes and myotomes
833. Brain imaging will be needed prior to LP to exclude a space occupying lesion.
834. Brain stem death testing confirms absence of respiratory effort and tests a series of cranial nerve
responses.
835. Brainstem features include diplopia, dysarthria and dizziness.
836. Calcified phakomas in the periventricular white matter on CT scanning combined with neurological
stigmata is suggestive of tuberous sclerosis.
357
M Y Elamin
MBBS, DTM&H, MCTM, MRCPI 1& 2
837. Cancer and metastatic spread are a recognised cause of plexopathy.
838. Carbamezapine is a recognised cause of TEN.
839. Carotid artery dissection can occur spontaneously and presents with neck, facial or head pain, ipsilateral
Horner's syndrome (miosis and ptosis) and contralateral weakness.
840. Carotid endarterectomy is not indicated if stenosis is less than 70% in CVA or TIA.
841. Carpal tunnel syndrome (CTS) is a result of pressure on the median nerve in the carpal tunnel.
842. Cases of mono-neuritits in a young female patient should prompt investigation for multiple sclerosis.
843. Cerebellar pontine angle tumours affect cranial nerves VI, VII, and VIII, which emanate from the pons.
844. Cerebral ischaemia from cerebral vasospasm is the most common cause of death and disability
following aneurysmal subarachnoid hemorrhage (SAH).
845. Cerebrospinal fluid (CSF) analysis in neurosarcoidosis shows intrathecal IgG synthesis and a
lymphocytosis
846. Cervical myopathy is a common complication of rheumatoid arthritis.
847. CHADSVASC scoring estimates ones risk of stroke in cases of AF
848. Characteristic features of dementia with Lewy bodies are fluctuation of awareness from day-to-day,
signs of parkinsonism and visual hallucinations or delusions occur frequently. Falls are also common. They
have pronounced visuoperceptual impairment.
849. Characteristic features of Lewy body dementia
850. Chronic paroxysmal hemicrania almost invariably responds to indomethacin.
851. Chronic subdural haematoma should be considered in patients with fluctuating neurology on blood
thinners and minor trauma.
852. CIDP shows progressive weakness and sensory function in the upper and lower limbs.
853. Clinical presentation of cauda equina syndrome is varied, but can include low back pain with unilateral
or bilateral lower limb motor and/or sensory abnormality and bowel and/or bladder dysfunction with saddle
and perineal anaesthesia.
854. Clinical signs of papilloedema and gait ataxia suggest cerebellar pathology which may represent
metastatic spread of malignancy.
855. Cluster headaches are described as an attack of severe pain localised to the unilateral orbital, supra-
orbital, and/or temporal areas; lasting from 15 minutes to 3 hours. they occur from once every other day to 8
times per day.
856. Coeliac disease can cause neurological symptoms such as peripheral neuropathy and ataxia.
857. Common peroneal nerve palsy results in weakness of ankle dorsiflexion and eversion, and reduced
sensation over the lateral aspect of the leg and dorsum of the foot.
358
M Y Elamin
MBBS, DTM&H, MCTM, MRCPI 1& 2
858. Compression of the optic chiasm from above causes a bilateral inferior quadrantinopia.
859. Concomitant treatment with lithium or anticholinergics may increase the risk of NMS. It is manifested
by fever, rigidity, altered mental status and autonomic dysfunction. Treatment includes withdrawal of the
offending agent, reduction of body temperature with antipyretics. Dantrolene, bromocriptine or levodopa
preparations may be beneficial.
860. Concomitant treatment with lithium or anticholinergics may increase the risk of NMS. Serum creatine
phosphokinase is the most important investigation to be performed.
861. Consider alcohol withdrawal as the cause of seizures in a patient with heavy alcohol intake.
Management is with benzodiazepines, not anti-epileptics.
862. Consider neuroleptic malignant syndrome in a patient taking antipsychotic medications and with fever,
muscle rigidity, confusion and autonomic dysfunction.
863. Consider parasagittal meningioma with lower limb weakness, cortical sensory loss, bladder dysfunction
and raised protein in CSF.
864. Consider serotonin syndrome in a patient who suddenly develops neuromuscular hyperactivity, fever,
confusion and muscle rigidity.
865. Consider syringomyelia in context of progressive pain and weakness in back, shoulders, arms or legs
with dissociated sensory disturbances and bowel/bladder dysfunction.
866. Copper studies in Wilson's disease show low serum copper, low serum caeruloplasmin and increased
urinary copper.
867. Cryptococcal meningitis is an AIDS defining illness
868. Cyproheptadine treats serotonin syndrome
869. Damage to the oculomotor nerve during its course results in ipsilateral ptosis.
870. Derealisation at the time of a suspect seizure is likely a symptom of a functional disorder.
871. Descending weakness with autonomic dysfunction and normal EMG is characteristic of botulism.
872. Dexamethasone helps to reduce swelling around malignant lesions and can help to relieve compression
symptoms.
873. Diabetic patients with acute stroke resulting in dysphagia should have their oral medication held, sugars
monitored and restarted when their swallow improves.
874. Diagnosis is by demonstration of C. neoformans in the cerebrospinal fluid, shown on an India ink stain:
the thick polysaccharide capsule is highlighted around the cell
875. Differential diagnoses of non-convulsive status include: metabolic encephalopathy, transient ischaemia,
migraine, infection, psychiatric causes.
876. Differential diagnosis for autonomic neuropathy includes DM, CRF, AIDS, AL, Porphyria

359
M Y Elamin
MBBS, DTM&H, MCTM, MRCPI 1& 2
877. Diffuse Lewy body disease is characterised by fluctuating cognitive impairment, visual hallucinations
and parkinsonism.
878. Digital subtraction angiography may be helpful in identifying an acutely ruptured aneurysm in SAH.
879. Dissection and cardioembolism are the commonest causes of stroke in young people.
880. Dix-Hallpike manoeuvre is a helpful bedside test in eliciting vertigo in benign positional vertigo.
881. Drug induced myastheic syndromes can occur due to penacillamine toxicity.
882. Duodenal biopsy in Whipples disease shows PAS positive foamy macrophages throughout the lamina
propria and the bacillus Tropheryma whippelii
883. Early sets of blood cultures remain the most important investigation in a suspected case of infective
endocarditis.
884. Eighty per cent of patients will maintain mobility if ambulatory function is good at presentation of
malignant spinal cord compression.
885. Elderly and falls cay result in sudural haematoma, symptoms may be present subacutely
886. Encephalopathy, gait ataxia and oculomotor dysfunction with a background of alcohol misuse is
suggestive of Wernicke's encephalopathy.
887. Even though rare, its important to be aware of the genetic mitochondrial disorders. Red ragged fibres are
seen on muscle biopsy of Kearns Sayre syndrome.
888. Falx calcification if big enough can cause neurological symptoms.
889. Fasiculations can present for a number of reasons and they are most commonly benign and of no
concern.
890. Fat embolism syndrome is a clinical diagnosis secondary to the presence of fat globules in the lung
parenchyma and circulation that are released, typically following long bone fractures.
891. Fatigue on repetitive movements or tasks such as number counting or tapping feet is a sign of
Myasthenia.
892. Features of myotonic dystrophy include frontal baldness in men, atrophy of temporalis, masseters, and
facial muscle, and bilateral ptosis.
893. Features of NEAD are eye closure, red, or normal colouring, prolonged attacks and post-ictal crying.
894. Features of neurofibromatosis Type 2 include cerebellar and retinal haemangioblastomas, café-au-lait
spots, and acoustic neuromas.
895. Femoral neuropathy causes weakness predominantly of the quadriceps.
896. Fifty per cent of patients with pituitary apoplexy have a negative CT.
897. Fist choice options for initial pharmacotherapy in people with early Parkinson’s disease are levodopa,
dopamine agonists and MAO-B inhibitors.
360
M Y Elamin
MBBS, DTM&H, MCTM, MRCPI 1& 2
898. Following a first, unprovoked seizure DVLA guidelines advise that a patient must have six months off
driving unless there are factors which suggest an acceptably high risk of a further seizure.
899. Friedreich's ataxia is an autosomal recessive condition which affects the posterior columns, corticospinal
and spinocerebellar tracts.
900. Friedreich's ataxia is an autosomal recessive inherited spinocerebellar disorder which presents with
progressive ataxia and parasthesia.
901. Friedreich's ataxia is an autosomal recessive trinucleotide repeat disorder leading to gross
neurodegeneration.
902. Frontotemporal dementia is more common under the age of 65. It presents with changes in behaviour
and loss of insight more commonly than memory problems. Patients are disinhibited, apathetic and can be
compulsive overeaters. They may have a normal MMSE, visuospatial and language function but they fail on
tasks of verbal fluency and executive function.
903. GBS may present as a variant and affect purely motor neurons.
904. GBS presents with a short history of ascending symmetrical progressive weakness and numbness
affecting initially the legs and extending to involve the upper limb.
905. Giant cell arteritis can lead to amaurosis fugax.
906. Glomus jugulare tumours will cause cranial nerve palsies affecting the 9th-12th nerve.
907. Guidelines for CT head post-trauma are indicated by NICE and comprehensive and clear.
908. Guidelines on criteria for CT head post injury should be used and followed with all patients who have
had head trauma.
909. Guillain-Barré syndrome cause ascending sensory neuropathy with absent reflexes commonly following
a flu-like illness.
910. Guillain-Barré syndrome is investigated using CSF analysis, NCS (F wave) and by monitoring FVC.
911. Hallpike manoeuvre reproduces the symptoms of benign positional vertigo.
912. Hereditary haemorrhagic telangiectasia presents with epistaxis, recurrent haemoptysis and cerebral and
pulmonary arteriovenous malformations.
913. History of optic neuritis and cerebellar/pyramidal signs in a young patient should prompt investigation
for MS.
914. Holmes' tremor or rubral tremor is secondary to a lesion in the red nucleus and classically produces an
irregular low frequency tremor .
915. HSMN has various types. Genetic testing usually reveals the diagnosis and EMG and NCS help
distinguish between the types.

361
M Y Elamin
MBBS, DTM&H, MCTM, MRCPI 1& 2
916. Huntington's disease is an autosomally inherited condition due to an expanded CAG trinucleotide repeat
on the short arm of chromosome 4. It is characterised by progressive dementia and worsening choreiform
movements. Symptoms typically appear between 30 and 50 years of age.
917. Hyperchloraemic metabolic acidosis is a documented complication of neobladder formation.
918. Hypothyroidism can sometimes mimic parkinsonian disorders.
919. Idiopathic intracranial hypertension is more commonly associated with obesity, use of the oral
contraceptive/tetracycline and pregnancy.
920. Idiopathic intracranial hypertension presents with bilateral papilloedema.
921. Idiopathic Intracranial Hypertension requires MR venogram and LP in order to confirm diagnosis
922. Immediate high dose steroids are recommended in the management of suspected malignant spinal cord
compression
923. Impulse control disorder is an uncommon disorder in those using dopaminergic medication. Features
include: hypersexuality, pathological gambling, and binge eating.
924. In cases of status epilepticus which do not terminate after 30 minutes a parenteral anti-epileptic agent
should be started.
925. In later Parkinson’s disease, first choice options for adjuvant pharmacotherapy are: dopamine agonists,
COMT inhibitors, and MAO-B inhibitors.
926. In Venous sinus thrombopsis anticoagilation should be continued even if there is evidence of bleeding
intracranially.
927. In vertigo, timing is a distinguishing feature. Remember: Duration of seconds-minutes: BPPV. Minutes-
hours: Menieres disease. Days-weeks: vestibular neuronitis.
928. In Wernicke's encephalopthy patients should receive IV thiamine and B12.
929. Indications for carotid endarterectomy based on doppler findings
930. Indications for carotid endarterectomy following TIA/amarousis fugax
931. InKearns-Sayre syndrome most cases have large mitochondrial DNA mutations and red ragged fibres on
muscle biopsy.
932. Intravenous immunoglobulins (0.4 g/kg for five days) and plasma exchange are the treatment of choice
in Guillain-Barré.
933. Intravenous thiamine should be given immediately in cases of Wernicke's encephalopathy.
934. Isolated sensory dermatomal distribution is usually suggestive of a cervical disc prolapse at a certain
level.
935. Jaccoud's arthropathy may be seen in SLE, RA, Parkinson's disease and hypocompementaemic urticarial
vasculitis

362
M Y Elamin
MBBS, DTM&H, MCTM, MRCPI 1& 2
936. Juvenile myoclonic epilepsy is associated with absences, myoclonic jerks and tonic-clonic seizures
which tend to occur within an hour of waking.
937. Juvenile myoclonic epilepsy is the most common generalised epileptic disorder in childhood/teenage
years.
938. Knowledge of classical presentations of stroke based on an understanding of functional neuroanatomy
939. Korsakoff syndrome is a chronic disorder caused by severe thiamine deficiency. It is characterised by
memory problems, confabulation and apathy.
940. Korsakoff's syndrome is associated with heavy alcohol use and is characterised by amnesia and
confabulation with lack of ability to reason.
941. Lambert-Eaton syndrome and myasthenia gravis are important examples of how disease pathologies can
affect the biochemistry of the neuromuscular junction.
942. Lambert-Eaton syndrome is a paraneoplastic disorder usually associated with small cell lung carcinoma
943. Leber's optic atrophy usually affects young men and causes sequential painless optic neuropathies in
days to weeks.
944. Lewy body dementia classically involves complex visual hallucinations, Parkinsonism and a subcortical
pattern of dementia.
945. Lewy body dementia usually appears five to ten years after the onset of parkinsonism and is presents
with sensitivity to neuroleptics.
946. Listeria meningitis should always be considered in patients over 50 years presenting with infective
meningitis
947. Listeria monocytogenes is a Gram-positive bacillus which affects immunosuppressed patients, or
patients at extremes of age.
948. Listeria meningitis should always be considered in patients with meningitis associated with brain stem
involvement, and in immunosuppressed patients. The treatment of choice is gentamicin and ampicillin.
949. Loss of consciousness and muscle tone following strong emotions is a sign of cataplexy.
950. Low dose phenytoin can be used for grip problems.
951. Lyme disease will produce an lumbar puncture which reveals a lymphocytic pleocytosis with intrathecal
oligoclonal band production.
952. Median nerve palsy affects the muscles of the thenar eminence, the abductor pollicis (C7, C8), flexor
pollicis brevis, opponens pollicis, and the lateral two lumbricals
953. Median nerve supplies the LOAF muscles: Lateral 2 lumbricals, Opponens pollicis, ABductor pollicis
brevis, Flexor pollicis brevis
954. Melanoma commonly spreads to the brain and lesions can cavitate causing neurological decline.

363
M Y Elamin
MBBS, DTM&H, MCTM, MRCPI 1& 2
955. Memantine is recommended for use in severe Alzhemier's or moderate disease where AChe inhibitors
are not tolerated or contraindicated.
956. Meningiomas are confined to the meninges, whilst primary CNS lymphomas have a predilection to
periventricular white matter.
957. Meningococcal septicaemia presents with a rapidly progressive, non blanching rash, purpuric in nature.
Treatment is with cephalosporin.
958. Migraine is more common where there is a family history of it.
959. MMN is an autoimmune neuropathy associated in 80% of cases with an elevated anti-GM1 ganglioside
antibody.
960. MND has signs of both upper and lower motor lesions.
961. MND produces and EMG with fasicualtions in clinically normal and abnormal muscles with preserved
motor conduction and sensory studies.
962. MND will present with a mix of both upper and lower motor neuron signs without sensory disturbance.
963. modafinil is first line for the treatment of excessive daytime sleepiness.
964. MRI changes in B12 deficiency progress with demyelination involving the dorsal columns. Usually it is
in the lower cervical and upper thoracic regions. It may eventually involve the entire dorsal columns.
965. MRI findings in HSV encephalitis classically shows high signal in the temporal lobes.
966. MS investigations will show intrathecal production of oligoclonal bands and multiple white matter
lesions on T2-weighted MRI scan some of which appear gadolinium enhancing.
967. Multiple ring enhancing lesions on imaging of the brain in an immunosuppressed patient may indicate
cerebral toxoplasmosis.
968. Multiple system atrophy is a degenerative disorder characterised by parkinsonian features with
autonomic insufficiency.
969. Myalgia, myositis and myopathy are all recognised effects of HMG-CoA reductase inhibitors (statins).
There is an increased incidence when statins are co-administered with a fibrate or when given to patients on
immunosuppressants.
970. Myasthenia Gravis can present with negative acetyl choline receptor antibodies.
971. Myasthenic crisis requires plasma phoresis and transfer to ITU.
972. Myotonic dystrophy is the most common adult muscular dystrophy. It is inherited as autosomal
dominant, the gene is on chromosome 19 and it is a trinucleotide repeat disease, which is a repeat of CTG,
that exhibits genetic anticipation.
973. Myotonic dystrophy presents with distal weakness more so than proximal weakness as well as myotonia
at an early age.
974. Nerve root lesions will present with sign and symptoms affecting a specific dermatome and myotome.
364
M Y Elamin
MBBS, DTM&H, MCTM, MRCPI 1& 2
975. Neuralgic amyotrophy usually is preceded by a URTI and the primary symptoms is pain around the
shoulder which resolves on its own.
976. Neuroimaging usually shows a unilateral atrophy of the frontal lobe in Pick's disease.
977. Neuroleptic malignant syndrome can be precipitated by Olanzapine.
978. Neurological signs and symptoms which vary and lack a pattern of distribution may be suggestive of a
functional disorder.
979. Neurology which presents 24 hours after minor head trauma is suggestive of a SDH.
980. NICE guidance suggest managing cluster headache with subcutaneous or intranasal sumatriptan as first
line.
981. NICE guidelines on head injury (CG176) recommend that patients with known coagulopathy or on
treatment with warfarin should have imaging by CT, with results analysed within one hour of request.
982. Normal pressure hydrocephalus characterised by a triad of urinary incontinence, gait disturbance and
dementia.
983. Normal pressure hydrocephalus is treated with repeated therapeutic lumbar puncture.
984. Normal pressure hydrocephalus presents clasically with the triad of cognitive impairment, gait apraxia,
and urinary incontinence
985. Occlusion of the middle cerebral artery produces contralateral hemiparesis and hemisensory loss with
speech disturbance if the dominant hemisphere is involved.
986. OCD may occur following an environmental trigger which causes great anxiety or grief.
987. OCP can cause stroke like episodes, in patients suffering with migraine.
988. Oligoclonal bands which are present in the cerebrospinal fluid and matched in the serum is a non-
specific biochemical finding.
989. One seizure of unknown aetiology yet to be investigated does not require treatment with anti-epileptics.
990. Only life threatening or function changes in MS require high dose pulsed methylprednisolone.
991. Orbital apex syndrome
992. Paraesthesias accompany weakness in more than 80% of patients with Guillain-Barré syndrome.
993. Paranoid schizophrenia is dominated by relatively stable, often paranoid delusions, usually accompanied
by hallucinations, particularly of the auditory variety, and perceptual disturbances.
994. Parkinson's disease is a progressive disorder and evidence of sustained remission excludes the disease.
995. partially treated bacterial meningitis can lead to subdural empyema.
996. Patients that are inattentive and unaware of their surrounding with preserved brain stem functions is
suggestive of permanent vegatative state.

365
M Y Elamin
MBBS, DTM&H, MCTM, MRCPI 1& 2
997. Patient's who present with signs and symptoms of stroke and pain after minor trauma should raise
suspicions of a vertebral artery dissection.
998. Patients with adult polycystic kidney disease should only have screening for cerebral aneurysms if high
risk (ie previous ruptured aneurysm, neurological symptoms or positive FHx).
999. Patients with cerebral venous sinus thrombosis (CVST) require IV heparin infusion which should be
continued even in case of haeorrhagic transformation.
1000. Patients with dementia who worsen on donepizil should be treated with memantine and stop donepezil.
1001. Patient's with GBS who have reduced vital capacity should be discussed with ITU for airway support.
1002. Patients with manual labour jobs or jobs which require repetitive physical actions may develop focal
root neuropathy.
1003. Patients with multiple cranial nerve palsies should be considered for investigation of skull base
osteomyelitis.
1004. Patients with multiple nerve palsies who have recently been in an endemic area on holiday should
prompt investigation for Neuroborreliosis (Lyme disease).
1005. Patients with oral and genital ulcers, of Mediterranean origin and arthralgia is classical for Bechet's
disease.
1006. Patients with syncope can commonly have jerking of the limbs when they are unconscious and this does
not mean they have had a seizure. Tilt table testing is useful to support the diagnosis of vasovagal syncope.
1007. Pharyngeal diphtheria presents with fever, sore throat, cervical lymphadenopathy and an adherent,
greyish pharyngeal membrane.
1008. Phenytoin toxicity commonly causes cerebellar symptoms such as nystagmus and liver dysfunction.
1009. Pick's disease is a rare form of dementia characterised by degeneration of the frontal and temporal lobes.
1010. Pick's disease is also known as frontotemporal dementia.
1011. Pituitary apoplexy can cause compression of the occular nerves and trigeminal nerve within the
cavernous sinus.
1012. Positive voltage-gated calcium antibodies and electromyogram (EMG) are needed to confirm LEMS.
1013. Primary axonopathy is seen in vasculitis.
1014. Progressive multifocal leukoencephalopathy (PML) is a demyelinating disease of the central nervous
system characterised by widespread lesions and associated with both HIV-1 and HIV-2.
1015. Progressive multifocal leukoencephalopathy (PML) is the result of infection of oligodendrocytes with
JC virus and common in patients with HIV and low CD4 counts.
1016. Progressive multifocal neurology involving cerebral cortex, cerebellum and meninges may be a sing of
cerebral angiitis.

366
M Y Elamin
MBBS, DTM&H, MCTM, MRCPI 1& 2
1017. Progressive proximal and distal weakness associated with wasting in absence of reflex changes or
sensory disturbance is characteristic of inclusion body myositis.
1018. PSP presents with symmetrical parkinsonism, postural instability and a supranuclear palsy.
1019. PSP presents with symmetrical stiffness and bradykinesia associated with poor postural reflexes and
impaired down-gaze.
1020. Quadriparesis, spinothalamic sensory loss (with sparing of posterior columns) and sphincter dysfunction
is suggestive of an anterior spinal cord syndrome.
1021. Radial nerve palsy can result from a pressure palsy when drunk called a "Saturday night palsy".
1022. Rapid cognitive decline in a young person with myoclonus is strongly suggestive of Creutzfeldt-Jakob
disease (CJD) which shows a characteristic EEG patern.
1023. Recognition of a stroke syndrome and the associated neuroanatomical area affected
1024. Resolving neurological signs are a contraindication to thrombolysis.
1025. Restless leg syndrome is associated with Fe deficiency, folate def, magnesium def, parkinsons diesase,
venous insufficiency and COPD.
1026. Rhabdomyolysis and associated renal failure are signs of MDMA overdose.
1027. Ring enhancing lesions may be associated with: cerebral mets, toxoplamosis, histioplasmosis, brain
abscess
1028. Rivastigmine is effective in treating cognitive decline in dementia with Lewy bodies. It may be effective
in reducing anxiety, apathy and hallucinations.
1029. Seizures can be divided into partial or complex and with careful examination the focus of the seizure can
be found.
1030. Selective serotonin reuptake inhibitor (SSRI) discontinuation syndrome occurs following abrupt
withdrawal of SSRIs and symptoms occur after days
1031. SIADH occurs when water retention accompanies normal water intake, leading to plasma
hyponatraemia and hypo-osmolality
1032. Small cell lung cancer commonly presents with paraneoplastic syndromes such as limbic encephalitis.
1033. Strokes presenting within 6 hours, confirmed infarct with no CIs should be thrombolysed
1034. Subacute cerebellar syndrome is present in paraneoplastic syndromes due to anti-yo antibodies in breast
cancer.
1035. Subacute cerebellum syndrome is one of the commonest paraneoplastic syndromes and is accompanied
by the presence of anti-Hu antibody.
1036. Subacute combined cord degeneration is due to vitamin B12 deficiency and is characterised by absent
reflexes and extensor plantar response.

367
M Y Elamin
MBBS, DTM&H, MCTM, MRCPI 1& 2
1037. Subclavian steal syndrome occurs when there is an occlusion proximal to the origin of the left vertebral
artery and blood is redirected from the right vertebral artery causing basilar insufficiency.
1038. Subdural haematomas are as a result of blood within the outermost meningeal layer between the dura
and arachnoid mater and results from tearing of the bridging veins.
1039. Subdural is not intraparenchymal
1040. Sudden onset comatose and quadraparesis is suggestive of a pons lesion.
1041. Sudden onset of anterior cord syndrome causing bilateral paresis is suggestive of an anterior spinal
artery lesion.
1042. Symptoms of botulism are due to neuromuscular junction blockade.
1043. Syndrome of inappropriate antidiuretic hormone (SIADH) is diagnosed with hyponatraemia and
elevated urine sodium, an osmolality towards the upper limit of normal, and low plasma osmolality.
1044. Systolic blood pressures above 220 mmHg require treatment even if no end organ damage is present.
1045. Tetrabenezine is used in the management of hemiballismus
1046. Tetracyclines can cause idiopathic intracranial hypertension
1047. TGA will resole spontaneously within 1-8 hours usually.
1048. The combination of upper and lower motor signs with the absence of sensory involvement is suggestive
of MND.
1049. The commonest cause of unilateral internuclear opthalmoplegia in a young person in the United
Kingdom is multiple sclerosis
1050. The connection between the lateral ventricles and third ventricle is via the foramen of Monro
1051. The diagnosis of sCJD is confirmed with a combination of EEG, lumbar puncture and MR scan findings.
1052. The Dix -Hallpike manoeuvre is used to make the diagnosis of BPPV whereas The Epley and Semont
manoeuvres used in the treatment.
1053. The ECG is a quick tool to categorise a presentation of syncope into high and low risk.
1054. The Hunt and Hess scale grades subarachnoid haemorrhage
1055. The majority of patients with TGA recover within 24 hours and do not get further such episodes.
1056. The majority of people with Transient global amnesia resolve spontaneously within 24 hours.
1057. The most common cause of an isolated deep intracerebral haemorrhage in the basal ganglia is
hypertension.
1058. The presence of optic atrophy on one side with contralateral papilloedema is characteristic of Foster
Kennedy syndrome.

368
M Y Elamin
MBBS, DTM&H, MCTM, MRCPI 1& 2
1059. The sciatic nerve (L4-S3) supplies both anterior and posterior compartment muscles and also the
hamstrings.
1060. The third (oculomotor) nerve nucleus complex lies in the midbrain.
1061. The treatment of choice during acute attacks of cluster headache is either high flow oxygen or
subcutaneous sumatriptan
1062. The typical clinical presentation of vertebral artery dissection is with severe occipital headache followed
by focal neurological signs
1063. The typical presentation of vertebral artery dissection is a young person with severe occipital headache
and neck pain following a recent trauma which is often deemed trivial.
1064. The ultimate decision to consent a dead relative to organ “harvest” following the brain-stem death of a
patient, rests with the family.
1065. The young age of the patient, rapid onset of dementia and presence of myoclonic jerks and EEG are
characteristic of Creutzfeldt-Jakob disease.
1066. There are multiple causes of bilateral facial weakness which include: Guillain-Barré syndrome, Bell's
palsy, lyme disease and sarcoidosis
1067. There is a mixture of lower motor neurone signs in the upper arms and upper motor neurone signs in the
legs. This patient has motor neurone disease of the amyotrophic lateral sclerosis type.
1068. Treatment for chorea secondary to strep infection is with tetrabenazine or sulpiride and a course of
penicillin for acute infection.
1069. Treatment of Guillain-Barré syndrome includes monitoring FVC regularly, and iv immunoglobulin.
Consider plasma exchange.
1070. Treatment of JME includes lamotrigine, sodium valproate and topiramate
1071. Treatment of malignant syndrome following an overdose of his antipsychotic medication is with
dantrolene or bromocriptine.
1072. Treatment of SBP is with IV broad spectrum antibiotics such as cephalosporins.
1073. Treatment of stroke secondary to polycythaemia includes venesection.
1074. Treatment of TA includes high dose steroids followed by biopsy to confirm diagnosis
1075. treatment with IV immunoglobulin or plasmaphoresisin myasthenia gravis are reserved for patients with
severe weakness with respiratory involvement.
1076. TTP is treated with daily plasma exchange and high dose methylprednisolone for 3 days.
1077. Tuberous sclerosis presents with flesh coloured papules (angiofibromas) on the face and fibromas in the
central nervous system
1078. Ultrasound still needs to be compared to other treatments for CTS

369
M Y Elamin
MBBS, DTM&H, MCTM, MRCPI 1& 2
1079. Upto date trials in stroke managent include CHADS2VASC risk scoring and PROGRESS
1080. Venous sinus thrombosis is a complication anti-phospholipid syndrome being a hypercoaguable state.
1081. VHL tumours consist of renal cell carcinoma, islet cell tumours and adenomas.
1082. Violent shaking, resistance to passive eye opening and normal vital signs usually indicate a
pseudoseizure.
1083. Visual anosognosia, or the denial of loss of vision, associated with confabulation in the setting of
obvious visual loss and cortical blindness is known as Anton's syndrome.
1084. Visual disturbances can occur with anti-epileptic medication. However only vigabatrin causes visual
field loss.
1085. Visual loss is the single threatening complication of idiopathic intracranial hypertension (IIH).
1086. Vitamin A is a well known cause of IIH
1087. Vivid dreams and lethargy may be due to nocturnal hypoglycaemia
1088. Wernicke-Korsakoff syndrome is as a result of chronic alcohol use and is due to thiamine (vitamin B1)
deficiency.
1089. When considering hypertensive disorders of pregnancy, pre-eclampsia and eclampsia must be
distinguished from pregnancy induced hypertension.
1090. Wilson's disease is an autosomal recessive disorder resulting in copper deposition in organs such as the
liver and the basal ganglia.
1091. Wilsons disease presents with KF rings and have abnormal brain MR findings.
1092. Yoga or other stretching exercises where injury may be trivial and not recalled the patient can be a risk
factor for developing vertebral artery dissection.
1093. Young female patients with multiple cases of neuritis is suggestive of multiple sclerosis.
1094. Young patients with intermittent headaches and cranial nerve lesions should be investigated for
Cavernous sinus thrombosis.
1095.

370
M Y Elamin
MBBS, DTM&H, MCTM, MRCPI 1& 2
PALLIATIVE MEDICINE AND END OF LIFE CARE
1. Benzydamine hydrochloride mouthwash or spray may be useful in reducing the discomfort associated with a
painful mouth that may occur at the end of life
2. Breakthrough dose = 1/6th of daily morphine dose
3. Buprenorphine or fentanyl are the opioids of choice for pain relief in palliative care patients with severe renal
impairment, as they are not renally excreted and therefore are less likely to cause toxicity than morphine
4. Codeine to morphine - divide by 10
5. Divide by two for oral to subcutaneous morphine conversion
6. Harsh inspiratory breath sounds in the last hours of life are caused by respiratory secretions, and can be treated
with hyoscine hydrobromide
7. Headache caused by raised intracranial pressure due to brain cancer (or metastases) can be palliated with
dexamethasone
8. Hiccups in palliative care - chlorpromazine or haloperidol
9. Hyoscine butylbromide or Hyoscine hydrobromide (Buscopan) is an antimuscarinic is generally used first-line to
manage secretions in a palliative care setting
10. Glycopyrronium bromide is an alternative antimuscarinic for excessive secretions in palliative care but is not
normally used first-line. However, It is considered first-line for sialorrhoea (excessive drooling) in, for example,
patients with neurodegenerative disorders.
11. In palliative patients increase morphine doses by 30-50% if pain not controlled
12. Levomepromazine is indicated for either nausea and vomiting or terminal agitation in palliative care.
13. Metastatic bone pain may respond to analgesia, bisphosphonates or radiotherapy
14. Metoclopramide is used for nausea and vomiting, bowel obstruction or hiccups in palliative care
15. Midazolam is used as an anxiolytic or sedative for agitation or breathlessness in palliative care.
16. Opioids may cause lethargy/drowsiness
17. Opioids may cause pruritus
18. Opioids may cause respiratory depression
19. Oxycodone is preferred to morphine in palliative patients with mild-moderate renal impairment
20. Syringe drivers: respiratory secretions & bowel colic may be treated by hyoscine hydrobromide, hyoscine
butylbromide, or glycopyrronium bromide
21. Neuropathic pain can be treated with amitriptyline 10mg first line alongside pregablin.
22. Pain management – characteristics of different pains causes, assessment and management of GI symptoms –
intestinal obstruction
23. Opioids may cause lethargy/drowsiness
24. Opioids may cause lethargy/drowsiness
25. Opioids may cause pruritus
26. Opioids may cause respiratory depression
371
M Y Elamin
MBBS, DTM&H, MCTM, MRCPI 1& 2
27. Hyoscine butylbromide (Buscopan) is an antispasmodic agent which can be given subcutaneously, which makes
it an excellent choice of analgesic in bowel obstruction
28. Patients on large doses of morphine still in pain may benefit from switching to a transdermal preparation such as
fentanyl.
29. he WHO analgesic ladder was developed specifically with cancer pain in mind. It recommended that analgesia be
given promptly in the following order until the patient is free from pain:
a. Simple analgesics (for example, paracetamol, aspirin, ibuprofen)
b. Mild opioids (for example, codeine)
c. Strong opioids (for example, diamorphine).
30. Patients must inform the DVLA as soon as possible after receiving a diagnosis of a brain tumour.
31. Hypercalcaemia is the most common life-threatening metabolic disorder associated with malignancy and should
be treated as an oncological emergency.
32. Ten per cent of cancer patients develop hypercalcaemia, most of whom have disseminated disease and 80% die
within a year.
33. The cancers most frequently associated with hypercalcaemia are breast cancer, lung cancer, renal cell
carcinoma and (most commonly) myeloma
34. Dexamethasone given for spinal cord compression can be given via any available route. Giving it intravenously
offers no significant advantage over giving it orally. 16 mg dexamethasone should be offered to all patients with
MSCC as soon as possible after assessment.
35. neuroleptic malignant syndrome (NMS). The classic tetrad is of: altered mental state muscle rigidity
hyperthermia, and autonomic instability .These can be accompanied by other neurological abnormalities such as:
tremor, dysphagia, chorea, and dysarthria
36. The most common signs associated with cardiac tamponade are: pulsus paradoxus (fall in blood pressure on
inspiration) Kussmaul's sign (rise in JVP on inspiration) tachycardia, muffled heart sounds, hypotension, and
impalpable apex beat.
37. MST 40 mg b.d. is approximately equivalent to a 35 µg/hr buprenorphine patch or a 25 µg/hr fentanyl patch.
38. Bisphosphonates have been shown to be effective in reducing skeletal complications in malignancy,
39. The WHO analgesic ladder is as follows:
a. Step 1 - Non-opioid +/- adjuvants (e.g. paracetamol/NSAIDs)
b. Step 2 - Weak opioid + non-opioid +/- adjuvants (e.g. co-codamol 30/500)
c. Step 3 - Strong opioid + non-opioid +/- adjuvants (e.g. morphine, fentanyl, oxycodone).
40. Naproxen (1 g daily) is associated with a lower thrombotic risk.
41. Low dose ibuprofen (1.2 g daily) would also be a relatively safe choice for patient as it has not been linked to an
increased risk of myocardial infarction.
42. If maintenance therapy via a syringe driver is required to prevent seizures midazolam 20-30 mg should be used
in end of life care.
372
M Y Elamin
MBBS, DTM&H, MCTM, MRCPI 1& 2
43. The conversion for oral morphine to subcutaneous diamorphine is one third. So 20 mg MST bd is a total of 40
mg oral morphine in 24 hours and one third of this is 13.3 mg diamorphine. This can be rounded to 15 mg.
44. Assessment involves the observation that all brain stem reflexes are absent (for example, corneal reflex) and this
assessment must be carried out by two medical practitioners either together or separately. These practitioners
must be registered for more than five years and at least one (but not both) must be a consultant.
45. MRI spine is the investigation of choice for investigating for potential metastatic spread of cancer.
46. Haloperidol is the first choice antiemetic for opiate induced nausea in the palliative care setting. Haloperidol acts
as a central dopamine antagonist.
47. Anxiolytics such as benzodiazepines can be used in patients pre-chemotherapy to prevent nausea and vomiting.
48. Malignant spinal cord compression is an emergency, managed in the initial stages with high dose steroids
49. Non-convulsive seizures can present as abnormal behaviour in a patient with a brain tumour. The likely diagnosis
is increased seizure activity and a trial of increasing anti-epileptics is the most appropriate choice.
50. Patients stable on high doses of morphine (or other opioids) are at risk of side effects if they develop renal
impairment, as the metabolites accumulate and effectively lead to an overdose. This patient's morphine should be
stopped and urgent U&Es arranged to assess renal function. Myoclonic jerks are a side effect of morphine
51. Certain medications do not mix together in solution and problems with precipitation can be encountered in
palliative care prescribing with respect to syringe drivers (Cyclizine & dexamethasone)
52. Spinal cord compression occurs in 5% of all cancer patients and 10% of those with spinal metastases.
53. CT thorax is the imaging modality of choice in suspected cases of SVCO.
54. The signs described are of superior vena caval (SVC) obstruction secondary to mediastinal lymphadenopathy - this
is an oncological emergency, especially in a young patient with potentially curable disease.
55. SVCO Treatment includes high dose steroids, chemotherapy if the tumour is likely to be very chemosensitive (for
example, small cell lung cancer or lymphoma), radiotherapy to the mediastinum and SVC stenting.
56. Lambert-Eaton myasthenic syndrome (LEMS) - a paraneoplastic disorder associated with small cell lung cancer.
57. LEMS is not treated with whole brain radiotherapy and rather with plasma exchange, steroids and
chemotherapy.
58. LEMS As the syndrome is autoimmune, mediated treatments such as IV immunoglobulin, plasma exchange and
high dose steroids may help, as well as treating the underlying small cell lung cancer, which is often very
responsive to appropriate chemotherapy
59. Fentanyl lozenges provide rapid onset analgesia and can be used safely in renal impairment. Fentanyl is a selective
µ receptor agonist. It has extensive first-pass metabolism so is not especially effective orally. However, buccal
absorption is good so lozenges are an effective mode of administration and have a rapid onset of action (five
minutes).
60. Patients with malignancy are at higher risk of deep venous thrombosis (DVT) and pulmonary embolism (PE) and
complications of anticoagulation. This makes management decisions tricky. Daily low molecular weight heparin
has been demonstrated to be efficacious in this setting, with low bleeding risks and is acceptable to most patients.
373
M Y Elamin
MBBS, DTM&H, MCTM, MRCPI 1& 2
It avoids regular blood tests and has fewer interactions. Therefore in the palliative care setting, daily low molecular
weight heparin is the treatment of choice for most palliative patients.
61. There are many well recognised side effects of opioid medication including nausea and constipation, rarer
symptoms of neurotoxicity (for example, hallucinations, myoclonus and delirium) and are the cause of much
anxiety amongst prescribers and Myoclonus
62. Lactulose (an osmotic laxative) is usually avoided in palliative care as it can cause abdominal cramps and excessive
flatulence
63. Polyethylene glycol (Movicol) would seem the best choice in p Constipation. It has an osmotic action and helps to
retain water in the gut to aid faecal passage. It is generally better tolerated than some other oral laxatives and has
been shown to be more effective than lactulose in the management of chronic constipation.
64. Palliation of breathlessness involves use of opioids, other medications, physiotherapy and psychological support.
Opioids are very effective agents to reduce the sensation of breathlessness - they reduce inappropriate respiratory
drive. They rarely cause respiratory depression when used correctly.
65. The patient's with the possibility of brain metastases and therefore cyclizine would be the most appropriate first
line agent. It targets the dopamine and cholinergic receptors and is widely accepted as the best antiemetic for
nausea associated with cerebral disease.
66. Morphine is thought to cause pruritus in some patients and change to oxycodone may be required.
67. Morphine accumulates in renal failure. This does not occur with fentanyl which therefore is useful in this situation.
Fentanyl has a rapid onset of action.
68. Eighty per cent of patients will maintain mobility if ambulatory function is good at presentation of malignant
spinal cord compression.
69. malignant spinal cord compression, The definitive investigation in this case is an MRI of the vertebral column to
look for vertebral collapse or other vertebral disease.
70. malignant spinal cord compression, Steroids should be commenced on clinical suspicion of spinal cord
compression; diagnosis may be delayed, and steroid treatment may minimise permanent damage.
71. malignant spinal cord compression, Urgent radiotherapy is the definitive treatment, although neurosurgical
opinion should be sought in order to ensure that surgical decompression is not required. Treatment is effective in
90% of patients if the diagnosis is made early.
72. Lymphoedema is a collection of excessive interstitial fluid which, in cancer patients, tends to be due to blockage
of the lymphatic system by malignancy or fibrosis. Management tends to centre around manual lymphatic drainage,
multilayer lymphoedema bandaging and skin care. Exercise can play an important part in encouraging lymphatic
drainage, although specialist physiotherapy is rarely appropriate.
73. Superior vena cava obstruction is a complication of malignant mesothelioma and can be treated with high dose
steroids in the first instance. Radiotherapy can be very effective for SVCO (if the tumour is radiosensitive) but
usually only in a patient with a better performance status and after medical therapy has been tried.

374
M Y Elamin
MBBS, DTM&H, MCTM, MRCPI 1& 2
74. Dexamethasone should be given to all patients when first diagnosed (to reduce inflammation around the cord)
and radiotherapy to the large, clinically significant lesion.
75. Standard management of malignant hypercalcaemia involves administration of intravenous fluids followed by
an infusion of a bisphosphonate. Pamidronate (30-90 mg) will restore normocalcaemia in 70-75% of cases at five
to seven days and its effect should last for approximately 2 ½ weeks.
76. The liver is contained within a collagenous capsule and any swelling of the liver (including that caused by metastatic
disease) causes that capsule to stretch, causing pain. Dexamethasone
77. MRI spine is the investigation of choice for investigating for potential metastatic spread of cancer.
78. Certain medications do not mix together in solution and problems with precipitation?
79. CT thorax is the imaging modality of choice in suspected cases of SVCO.
80. Spinal cord compression occurs in 5% of all cancer patients and 10% of those with spinal metastases.
81. 16 mg dexamethasone should be offered to all patients with MSCC as soon as possible after assessment and
MRI should be within 24 hours.
82. ALS is characterised by both upper and lower motor neurone symptoms (typically LMN in the arms and
UMN in the legs) and intact sensation.
83. Anxiolytics such as benzodiazepines can be used in patients pre-chemotherapy to prevent nausea and
vomiting.
84. Bisphosphonates have been shown to be effective in reducing skeletal complications in malignancy,
85. Causes, assessment and management of GI symptoms : constipation
86. Causes, assessment and management of neurological and psychiatric problems – MCA and MHA
87. Causes, assessment and management of skin problems – lymphoedema.
88. Certain medications do not mix together in solution and problems with precipitation can be encountered in
palliative care prescribing with respect to syringe drivers.
89. CT thorax is the imaging modality of choice in suspected cases of SVCO.
90. Eighty per cent of patients will maintain mobility if ambulatory function is good at presentation of
malignant spinal cord compression.
91. Fentanyl lozenges provide rapid onset analgesia and can be used safely in renal impairment.
92. Fentanyl should be used in pain management in renal failure for palliative care prescribing.
93. If maintenance therapy via a syringe driver is required to prevent seizures midazolam 20-30 mg should be
used in end of life care.
94. LEMS is not treated with whole brain radiotherapy and rather with plasma exchange, steroids and
chemotherapy.
95. Malignant spinal cord compression is an emergency, managed in the initial stages with high dose steroids

375
M Y Elamin
MBBS, DTM&H, MCTM, MRCPI 1& 2
96. Management of anaemia and coagulopathies
97. Management of nausea and vomiting in palliative medicine.
98. Management options for malignant hypercalcaemia and of a palliative care emergency.
99. Members of the transplant team are not permitted to be involved in the diagnosis of brain stem death
100. Morphine accumulates in renal failure. This does not occur with fentanyl which therefore is useful in
this situation. Fentanyl has a rapid onset of action.
101. Morphine is thought to cause pruritus in some patients and change to oxycodone may be required.
102. MRI spine is the investigation of choice for investigating for potential metastatic spread of cancer.
103. MRI whole spine is the imagin modality of choice in cases of suspected malignant spinal cord
compression
104. MST 40 mg b.d. is approximately equivalent to a 35 µg/hr buprenorphine patch or a 25 µg/hr fentanyl
patch.
105. Nausea is common in a palliative care setting and choice of antiemetic is important. Haloperidol is the
first choice antiemetic for opiate induced nausea.
106. Neuropathic pain can be treated with amitriptyline 10mg first line alongside pregablin.
107. New side effects from a patient previously stable on doses of morphine should raise the suspicion of
renal injury. Myoclonic jerks are a side effect of opioids.
108. Non-convulsive seizures can present as abnormal behaviour in a patient with a brain tumour.
109. Opioids are very useful in the treatment of breathlessness in palliative care. Oxygen therapy is mainly
used for the hypoxic patient.
110. Pain management – characteristics of different pains causes, assessment and management of GI
symptoms – intestinal obstruction
111. Pain management – drug treatment (WHO)
112. Pain management – drug treatment.
113. Pain management – opioids, opioid switching (dose conversions)
114. Pain management : opioids and side effect management.
115. Patients must inform the DVLA as soon as possible after receiving a diagnosis of a brain tumour.
116. Patients on large doses of morphine still in pain may benefit from switching to a transdermal preparation
such as fentanyl.
117. Patients with malignancy are at higher risk of DVT and PE and complications of anticoagulation. Low
molecular weight heparin is the treatment of choice for PEs and DVTs in the palliative care setting.

376
M Y Elamin
MBBS, DTM&H, MCTM, MRCPI 1& 2
118. Patients with pain should be appropriately titrated up the WHO pain adder provided they have no
barriers to administration such as being Nil by mouth.
119. Presentation and management of emergencies – anaphylaxis
120. Presentation and management of emergencies – cardiac tamponade.
121. Presentation and management of emergencies – hypercalcaemia disease process and management –
common malignancies.
122. Principles of pain and symptom management – drugs pain management – drug treatment.
123. Principles of pain and symptom management – Drugs presentation and management of emergencies –
acute dystonia
124. Principles of pain and symptom management – drugs presentation and management of emergencies –
neuroleptic malignant syndrome
125. Refeeding syndrome occurs as a result of shifts in fluid and electrolytes in malnourished patients
receiving artificial nutrition.
126. Spinal cord compression is a palliative care emergency. Management depends on location of
compression. Lesions below L1 have a lower risk of permanent dysfunction.
127. Spinal cord compression occurs in 5% of all cancer patients and 10% of those with spinal metastases.
128. Superior vena cava obstruction is a complication of malignant mesothelioma and can be treated with
high dose steroids in the first instance.
129. The majority of patients with TGA recover within 24 hours and do not get further such episodes.

377
M Y Elamin
MBBS, DTM&H, MCTM, MRCPI 1& 2
OPHTHALMOLOGY
1. A 30-year-old man presents with visual problems. His vision is much worse in the dark and he has now started to
lose peripheral vision. He has a family history of similar problems - retinitis pigmentosa
2. A man who has a history of syphilis presents with bilateral small, irregular pupils which respond to accomodation
but not to light - Argyll-Robertson pupil
3. A man who is on treatment for HIV presents with a painful, vesicular rash on the right side of his face around the
eye. Fluorescein staining reveals multiple small defects on the right cornea - herpes zoster ophthalmicus
4. A patient presents with a 'droopy eyelid'. On examination they have unilateral miosis, ptosis and narrow palpebral
aperture giving the appearance of enophthalmos - Horner's syndrome
5. A patient presents with an acute, painful red eye associated with photophobia and epiphora. Fluorescein staining
reveals a ragged area on the cornea - herpes simplex keratitis
6. A woman is noted to have a unilateral mydriatic pupil which is minimally reactive to light - Holmes-Adie pupil
7. A woman with a history of rheumatoid arthritis presents after developing a red right eye. There is no pain,
discharge or photophobia although the eye is tearing. Visual acuity is normal - episcleritis
8. A woman with a history of rheumatoid arthritis presents with severe, constant pain in her right eye. On
examination the right eye is red and there is a degree of photophobia. Visual acuity is normal - scleritis
9. A young man presents with an acute, painful red eye associated with photophobia and blurred vision. On
examination the pupil is small and irregular - anterior uveitis
10. An elderly female smoker presents with reduced visual acuity, complaining of 'blurred' vision. On examination
there is a central scotoma and fundoscopy reveals multiple drusen - macular degeneration
11. An elderly man presents an acute, painful red eye associated with decreased visual acuity. His symptoms are
worse in the dark. On examination he has a semi-dilated non-reacting pupil - acute angle closure glaucoma
12. An elderly man with a long history of diabetes presents with sudden visual loss in one eye. For the past few days
he had been experiencing floaters and 'cobwebs'. Visual acuity is reduced to sensing light - vitreous haemorrhage
13. An elderly patient with a history of atrial fibrillation presents with a sudden painless loss of vision in one eye.
Fundoscopy reveals a 'cherry red' spot on a pale retina - central retinal artery occlusion
14. An elderly patient with a history of chronic glaucoma and hypertension presents with a sudden painless loss of
vision in one eye. Fundoscopy reveals multiple flame-shaped haemorrhages and optic disc oedema - central retinal
vein occlusion
15. An elderly patient with a history of hypertension presents with decreasing visual acuity and peripheral visual field
loss. Fundoscopy reveals optic disc cupping - primary open-angle glaucoma
16. An elderly short-sighted man presents with a floater on the temporal field of vision. Visual acuity is normal for
the patient - posterior vitreous detachment
17. An elderly short-sighted man presents with flashing lights which are worse on eye movement and are located in
the temporal visual field. He also describes some upper visual field loss 'like a curtain coming down' - retinal
detachment
378
M Y Elamin
MBBS, DTM&H, MCTM, MRCPI 1& 2
18. Pilocarpine - muscarinic receptor agonist
19. 1st line of treatment for blepharitis is hot compresses
20. Drusen = Dry macular degeneration
21. A central scotoma is a feature of optic neuritis
22. A relative afferent pupillary defect is when the affected and normal eye appears to dilate when light is shone on
the affected eye
23. Acute angle closure glaucoma is associated with hypermetropia, whereas primary open-angle glaucoma is
associated with myopia
24. Alongside eye drops, IV acetazolamide is used in the initial emergency medical management of acute angle-
closure glaucoma
25. Angioid retinal streaks are a feature of pseudoxanthoma elasticum
26. Beta blockers such as timolol work in primary open-angle glaucoma by reducing aqueous production
27. Bevacizumab is a monoclonal antibody directed against vascular endothelial growth factor (VEGF)
28. Central retinal vein occlusion - sudden painless loss of vision, severe retinal haemorrhages on fundoscopy
29. Dorzolamide - carbonic anhydrase inhibitor
30. Flashes + floaters are most commonly caused by a posterior vitreous detachment
31. Flashes and floaters - vitreous/retinal detachment
32. Fundoscopy reveals end organ damage in hypertension
33. Holmes adie = dilated pupil, females, absent leg reflexes
34. Horner's syndrome - anhydrosis determines site of lesion:
a. Head, arm, trunk = central lesion: stroke, syringomyelia
b. Just face = pre-ganglionic lesion: Pancoast's, cervical rib
c. Absent = post-ganglionic lesion: carotid artery
35. Latanoprost is a prostaglandin analog used in glaucoma. It works by increasing uveoscleral outflow
36. Macular degeneration - smoking is risk factor
37. Monocular transient painless loss of vision (amaurosis fugax) should be treated as a TIA
38. Patients with orbital cellulitis require admission to hospital for IV antibiotics due to the risk of cavernous sinus
thrombosis and intracranial spread
39. Pilocarpine is a muscarinic receptor agonist
40. Red eye - glaucoma or uveitis?
41. Glaucoma: severe pain, haloes, 'semi-dilated' pupil
42. Uveitis: small, fixed oval pupil, ciliary flush
43. Relative afferent pupillary defect indicates an optic nerve lesion or severe retinal disease
44. Retinal detachment is a cause of sudden painless loss of vision. It is characterised by a dense shadow starting
peripherally and progressing centrally
45. Retinitis pigmentosa - night blindness + tunnel vision
379
M Y Elamin
MBBS, DTM&H, MCTM, MRCPI 1& 2
46. Scleritis is painful, episcleritis is not painful
47. Vitamin A toxicity is a rare cause of papilloedema
48. Vitreous haemorrhage is a cause of sudden painless loss of vision in the context of diabetic retinopathy
49. A chronic progressive limitation of eye movements points to a likely diagnosis of chronic progressive
external ophthalmoplegia (CPEO).
50. A lesion of the occipital cortex can cause a quadrantic hemianopia.
51. A posterior communicating artery aneurysm results in anisocoria, headaches and diplopia.
52. Adie's tonic pupil, is characteristically seen in young women, and may occur after an episode of zoster
infection. Initially the pupil is large, but over time becomes small and poorly reactive. It is diagnosed with
weak pilocarpine eye drops.
53. Behcet's disease is a systemic vasculitis. It is commonly associated with mucocutaneous manifestations
(oro-genital ulceration, erythema nodosum), ocular disease, gastrointestinal involvement and neurological
features.
54. Bilateral internal carotid artery displacement can cause binasal incongruous hemianopia if the optic nerves
are compressed.
55. Carbon monoxide poisoning and ethylene glycol may result in dilated pupils
56. Causes of an acute red eye include closed angle glaucoma, anterior uveitis, corneal ulcers, conjunctivitis
and scleritis
57. CMV retinitis is a complication of acute untreated HIV infection.
58. congenital hypertrophy of retinal pigment epithelium (CHRPE) is associated with autosomal dominantly
inherited familial adenomatous polyposis (FAP).
59. Delays in latencies indicate demyelination in the anterior visual pathways.
60. Diabetes is a cause of optic neuritis which is usually associated with headache.
61. Ectopia lentis/subluxation of the lens is associated with Ehlers-Danlos syndrome, Marfan's syndrome,
Weill-Marchesani syndrome, , Refsum's disease.
62. Ethambutol is associated with retrobulbar neuritis, generalised cutaneous reactions, hepatitis, and
peripheral neuropathy.
63. Friedreich's ataxia is asociated with optic atrophy. The major clinical manifestations are neurologic
dysfunction, cardiomyopathy, and diabetes mellitus. Because the disease is progressive, the full clinical
picture may not be seen until several years after presentation.
64. History of headache and amenorrhoea suggest a prolactinoma or non-functioning tumour.
65. Holmes-Adie and 3rd nerve palsies result in a dilated pupil

380
M Y Elamin
MBBS, DTM&H, MCTM, MRCPI 1& 2
66. Idiopathic Intracranial Hypertension (IIH) often present in obese women of child-bearing group who
present with headache, transient visual obscurations, intracranial noises (pulsatile tinnitus). Other
symptoms may be photopsia, back pain, retrobulbar pain, diplopia and sudden visual loss.
67. Idiopathic intracranial hypertension normally presents with headache, blurred vision, dizziness, horizontal
diplopia and transient visual loss.
68. If visual symptoms are present in oatients with giant cell arteritis (GCA), intravenous methylprednisolone
should be given wihhout delay to preserve vision.
69. IHD is a risk factor for Age Related Wet Macular Degeneration.
70. In optic neuropathy, accommodaton response is unaffected.
71. Inhaled steroids can cause cataracts.
72. Interpretation of neurological eye signs can give aeitiological clues
73. Iritis is associated with conditions such as Reiter's, Behcet's, psoriatic arthropathy (about 20%) and
inflammatory bowel disease.
74. Kearns-Sayre Syndrome is a mitochondrial inherited disease, and as such is only passed on by mothers to
offspring.
75. Miller-Fisher is classically described as a triad of external ophthalmoplegia, ataxia, and areflexia
following a diarrhoeal illness.
76. Normal visual acuity is seen in proliferative retinopathy
77. Optic chiasm lesions characteristically produce a bitemporal hemianopia
78. Optic neuritis can be associated with multiple sclerosis, and can result in central scotoma, reduced visual
acuity and papilloedema.
79. Osmotic changes in the lens can result in blurred vision in patients with diabetes and poor glycaemic
control.
80. Posterior communicating artery aneurysms can present with an oculomotor nerve palsy with pupillary
involvement.
81. Pseudoxanthoma elasticum with angioid streaks is also known as Grönblad-Strandberg syndrome.
82. Red eye of an acute painful dull nature, with photophobia and reduced vision in a hypermetropic person is
highly indicative of acute angle closure glaucoma.
83. Red ragged fibres found in mitochondrial myopathy are found in Kearns-Sayre syndrome; mitochondrial
myopathy, lactic acidosis and stroke-like episodes (MELAS); and Leber's optic atrophy.
84. Rheumatoid arthritis is associated with both scleritis and episcleritis. Episcleritis is acute in onset, with
mild pain or discomfort / grittiness, can be unilateral or bilateral, with localised or diffuse red eye.
85. Scleritis is commonly associated with connective tissue disorders, in particular rheumatoid arthritis.

381
M Y Elamin
MBBS, DTM&H, MCTM, MRCPI 1& 2
86. Sudden onset of visual loss is often due to a vascular cause
87. Temporal lobe lesions result in a contralateral homonymous superior quadrantanopia.
88. The best long term management for idiopathic intracranial hypertension is weight reduction, which can
improve her symptoms.
89. The fourth cranial nerve can be affected in such a situation resulting in superior oblique palsy.
90. The most likely localisation of the lesion is around the optic chiasm spreading up the right optic nerve
hence a bitemporal visual field defect with involvement of the right optic nerve.
91. Third nerve palsy typically presents with a dilated, unreactive pupil and an eyeball that is displaced "down
and out"
92. Urgent referral to an ophthalmologist is required if there is proliferative retinopathy or there is evidence of
clinically significant macular oedema.
93. Venous beading, loops and soft exudates (cotton wool spots) are characteristic of the ischaemia associated
with preproliferative diabetic retinopathy.A pale optic disc is a sign of atroph which can be caused by
increased pressure, demyelination or external compression.
94. A recognised cause of cataracts i oral steroids.
95. Acute monocular painless visual loss is a recognised sign of cholesterol embolism.
96. An obese female patient with headaches and papilloedema should prompt investigation for IIH
97. Angioid Streaks are breaks in Bruch's membrane. They may be associated with pseudoxanthoma
elasticum, Ehlers-Danlos syndrome, sickle cell disease, acromegaly (and other pituitary disorders), and
Paget's disease.
98. angioid streaks are caused by breaks in Bruch's membrane
99. Bechet's disease is characterised by oral and genital ulcer and more common in people from the eastern
Mediterranean and the Middle East.
100. Brain imaging will be needed prior to LP to exclude a space occupying lesion.
101. Chediak-Higashi syndrome is autosomal recessive and patients may have a silvery sheen to their skin, and
blue to brown irises.
102. CMV retinitis is commonly associated with HIV. It presents with a 'pizza' appearance
103. Craniopharyngioma causes an inferior bitemporal hemianopia and central diabetes insipidus.
104. CRVO presents with a painless loss of vision, it is common in the elderly. Diffuse retinal haemorrhage
and disc oedema are present.
105. Cupping of the optic disc which is typical of glaucoma.
106. Dendritic ulcers are caused by herpes simplex virus

382
M Y Elamin
MBBS, DTM&H, MCTM, MRCPI 1& 2
107. Dendritic ulcers are caused by herpes simplex virus. They are treated with aciclovir eye drops, which
should be continued for three days after the ulcer has healed.
108. Grave's eye disease can occur in euthyroid, hypothyroid or hyperthyroid setting
109. Hyphaema is blood in the anterior chamber. It is commonly seen after trauma to the eye
110. In bilateral optic disc drusen in diabetes the investigation of choice is ultrasound orbits
111. in presentations of Gaucher disease, the skin may show a grey-brown discolouration, especially around
the forehead, hands and pre-tibial regions. Characteristic yellow or yellow-brown papules (pingueculae)
develop at the sclerocorneal junctions.
112. Indications for carotid endarterectomy based on doppler findings
113. Intermittent headaches with neurology affecting multiple cranial nerves is suggestive of Cavernous Sinus
Thrombosis.
114. Kaposi's sarcoma (KS) lesion is a red-purple mass arising from the conjunctiva. It is not limited to the
sclera (which could suggest a subconjunctival haemorrhage) and it protrudes, which also favours a
malignant growth.
115. Loss of night vision is a features of retinitis pigmentosa and retinal appearance at the periphery appears as
if there are "black bony spicules"
116. Macular degeneration will show small pale dots over the macluar on slit lamp examination known as
drusen.
117. Nystagmus and photophobia are common findings in albinos.
118. Ptosis and variable diplopia, without proptosis or injection of the eyes is suggestive of ocular myasthenia.
119. Radioiodine therapy can worsen thyroid eye disease
120. Subconjunctival haemorrhage is an alarming adverse effect of aspirin therapy (and other antiplatelets).
121. Tetracyclines can cause idiopathic intracranial hypertension
122. The use of oral contraceptives, corticosteroids, tetracyclines, and vitamin A have all been linked with
increased likelihood of developing idiopathic intracranial hypertension (IIH)
123. Toxocara canis a parasite found globally, carried by dogs.
124. Visual loss is the single threatening complication of idiopathic intracranial hypertension (IIH).
125.

383
M Y Elamin
MBBS, DTM&H, MCTM, MRCPI 1& 2
PSYCHIATRY
1. A car crash survivor feels like he is 'watching a movie' when trying to recall details of the episode - dissociative
disorder
2. A female librarian comes for advice. Her colleagues find her inflexible in her approach to her work. She easily
becomes annoyed if her 'systems' are interfered with and generally likes to work by herself, using lists and rules
to structure her day - obsessive-compulsive personality disorder
3. A man asks for help with social anxiety. He prefers to be alone and doesn't like to share his beliefs, which other
people think are odd. He has a strong interest in the paranormal and talks in an high-pitched voice when talking
about his 'spirit-guide' - schizotypal personality disorder
4. A middle-aged male manager comes in for review after having trouble at work. His colleagues find him arrogant,
'cut-throat' and lacking empathy. He seems to exaggerate his own importance to the company and seems
preoccupied with success - narcissistic personality disorder
5. A middle-aged man with a history of depression presents with multiple unrelated physical symptoms over the
past 5 years - somatisation disorder
6. A middle-aged patient is convinced he has an underlying malignancy despite multiple investigations and
reassurance - illness anxiety disorder
7. A middle-aged woman asks for help after splitting up with her partner, who apparently felt she was too 'needy'.
She describes him as 'my world' and thinks she won't be able to cope by herself and wants your advice on how to
find a new partner quickly - dependent personality disorder
8. A patient feigns chronic lower back pain to allow him to claim government benefits - malingering
9. A patient is suspected of injecting her mothers insulin to cause repeated episodes of hypoglycaemia - factitious
disorder
10. A woman presents to her male family doctor wearing a low-cut top and a short skirt. She tries to flirt with the
doctor. The consultation is filled with drama and she becomes annoyed with the centre of attention shifts from her
- histrionic personality disorder
11. A young athlete presents with parlaysis of his right leg before athletics meet. Neurological examination is
inconsistent - conversion disorder
12. A young man is arrested after crashing his car into a pedestrian. He shows little remorse and repeatedly lies to
try and avoid prosecution. He is known to police after being involved in repeated fights - antisocial personality
disorder
13. A young man is seen with his mother. She is concerned that he is socially withdrawn. He is bright and is doing well
in his job as a engineer. During the consultation he seems emotionally cold and has little interest in either praise
or criticism - schizoid personality disorder
14. A young woman comes for relationship advice. She is constantly questioning the loyalty of her partner and
regularly accuses him of having affairs for no reason. She also regularly falls out with her female friends as she
thinks they are belittling her - paranoid personality disorder
384
M Y Elamin
MBBS, DTM&H, MCTM, MRCPI 1& 2
15. A young woman complains of feeling lonely. She has stopped seeing her old friends as she is worried about not
being liked or criticised - avoidant personality disorder
16. A young woman takes a paracetamol overdose after splitting with her boyfriend. Two days later she is in a new
relationship which is troubled by her repeated outbursts of anger - borderline personality disorder
17. Acute stress disorder , treatment of choice: trauma-focused cognitive-behavioural therapy
18. An elderly man with a history of macular degeneration presents with visual hallucinations but reports no other
psychiatric symptoms - Charles Bonnet syndrome
19. Antipsychotics may cause akathisia (severe restlessness)
20. Antipsychotics may cause blurred vision
21. Antipsychotics may cause constipation
22. Antipsychotics may cause corneal deposits
23. Antipsychotics may cause dry mouth
24. Antipsychotics may cause dyskinesia
25. Antipsychotics may cause hyperlipidaemia
26. Antipsychotics may cause hyperprolactinaemia
27. Antipsychotics may cause impaired glucose tolerance
28. Antipsychotics may cause increased risk of ischaemic stroke
29. Antipsychotics may cause increased risk of venous thromboembolism
30. Antipsychotics may cause lengthens QT interval
31. Antipsychotics may cause lethargy/drowsiness
32. Antipsychotics may cause myelosuppression/agranulocytosis
33. Antipsychotics may cause neuroleptic malignant syndrome
34. Antipsychotics may cause oculogyric crisis
35. Antipsychotics may cause parkinsonism
36. Antipsychotics may cause reduced seizure threshold
37. Antipsychotics may cause slurred speech
38. Antipsychotics may cause tardive dyskinesia
39. Antipsychotics may cause torticollis
40. Antipsychotics may cause tremor
41. Antipsychotics may cause urinary retention
42. Antipsychotics may cause weight gain
43. Benzodiazepines - enhance the effect of the inhibitory neurotransmitter gamma-aminobutyric acid (GABA)
44. Benzodiazepines may cause dependence
45. Benzodiazepines may cause respiratory depression
46. Depression (post-myocardial infaction) , treatment of choice: sertraline
47. Generalised anxiety disorder , treatment of choice: sertraline
385
M Y Elamin
MBBS, DTM&H, MCTM, MRCPI 1& 2
48. Korsakoff's syndrome - haemorrhage to the mammillary bodies of the hypothalamus and the medial thalamus
49. Lithium may cause diabetes insipidus
50. Lithium may cause hypothyroidism
51. Lithium may cause intracranial hypertension
52. Lithium may cause nephrotoxicity
53. Lithium may cause tremor
54. Lithium may cause weight gain
55. Post-traumatic stress disorder , treatment of choice: trauma-focused cognitive-behavioural therapy (CBT) or eye
movement desensitisation and reprocessing (EMDR) therapy
56. Tricyclic antidepressants may cause blurred vision
57. Tricyclic antidepressants may cause constipation
58. Tricyclic antidepressants may cause dry mouth
59. Tricyclic antidepressants may cause lengthens QT interval
60. Tricyclic antidepressants may cause lethargy/drowsiness
61. Tricyclic antidepressants may cause urinary retention
62. Normal pressure hydrocephalus presents clasically with the triad of cognitive impairment, gait apraxia, and
urinary incontinence
63. A male with a history of alcohol or drug abuse and deliberate self harm should be considered to be at high risk of
suicide
64. Acute dystonia - sustained muscle contraction such as torticollis or oculogyric crisis
65. Age-related macular degeneration is associated with Charles-Bonnet syndrome
66. Alcohol withdrawal
a. Symptoms: 6-12 hours
b. Seizures: 36 hours
c. Delirium tremens: 72 hours
67. An obsession is an intrusive, unpleasant and unwanted thought.
68. A compulsion is a senseless action taken to reduce the anxiety caused by the obsession
69. Anorexia features
a. Most things low
b. G's and C's raised: growth hormone, glucose, salivary glands, cortisol, cholesterol, carotinaemia
70. Antidepressants should be continued for at least 6 months after remission of symptoms to decrease risk of relapse
71. Antipsychotics in the elderly - increased risk of stroke and VTE
72. Antipsychotics may cause akathisia (severe restlessness)
73. Atypical antipsychotics commonly cause weight gain
74. Benzodiazepines enhance the effect of GABA, the main inhibitory neurotransmitter

386
M Y Elamin
MBBS, DTM&H, MCTM, MRCPI 1& 2
75. Borderline personality disorder is associated with impulsivity, feelings of emptiness, fear of abandonment and
unstable self image
76. Citalopram is the antidepressant of choice in Alzheimer's disease due to it's effect on mood and well-being. Dose
limitation to 20 mg is generally recommended in the elderly because of a risk of QT prolongation.
77. Charles-Bonnet syndrome - peripheral visual impairment is a risk factor
78. Charles-Bonnet syndrome causes unpleasant visual hallucinations in a third of sufferers
79. Chlordiazepoxide or diazepam are used in the treatment of delirium tremens/alcohol withdrawal
80. Common features of PTSD
a. Re-experiencing e.g. Flashbacks, nightmares
b. Avoidance e.g. Avoiding people or situations
c. Hyperarousal e.g.hypervigilance, sleep problems
81. Conversion disorder - typically involves loss of motor or sensory function. May be caused by stress
82. Cotard syndrome is associated with severe depression
83. Cotard syndrome is characterised by a person believing they are dead or non-existent
84. Dosulepin - avoid as dangerous in overdose
85. Duloxetine mechanism of action = serotonin and noradrenaline reuptake inhibitor
86. Elderly patients with depression are less likely to complain of low mood and instead may present with health
anxiety, agitation and sleep disturbance
87. Erotomania (De Clerambault's syndrome) is the presence of a delusion that a famous is in love with them, with the
absence of other psychotic symptoms
88. Family history is the strongest risk factor for psychotic disorders
89. Gastrointestinal side-effects such as diarrhoea are seen in SSRI discontinuation syndrome
90. Histrionic personality disorder is characterised by inappropriate sexual seductiveness, suggestibility and intense
relationships
91. Hyperprolactinaemia is uncommon with certain atypical antipsychotics - consider other causes such as
prolactinoma if prolactin raised
92. If CBT or EMDR therapy are ineffective in PTSD, the first line drug treatments are venlafaxine or a SSRI
93. Illness anxiety disorder (hypochondriasis) is the persistent belief in the presence of an underlying serious disease,
e.g. Cancer
94. Korsakoff’s syndrome is a complication of Wernicke’s encephalopathy. It's features include: anterograde
amnesia, retrograde amnesia, and confabulation
95. Lithium levels should be checked every 3 months once a stable dose has been achieved
96. Lofepramine - the safest TCA in overdosage
97. Lying or exaggerating for financial gain is malingering, for example someone who fakes whiplash after a road
traffic accident for an insurance payment

387
M Y Elamin
MBBS, DTM&H, MCTM, MRCPI 1& 2
98. Narcisstic personalities lack empathy, have a sense of entitlement and take advantage of others to achieve their
own need
99. Olanzapine has a higher risk than other atypicals for dyslipidemia and obesity
100. Paranoid personality disorder may be diagnosed in patients who are overly sensitive and can be unforgiving if
insulted, question loyalty of those around them and are reluctant to confide in others
101. Paroxetine - higher incidence of discontinuation symptoms
102. Patient with a fixed, false belief (delusion) that they are infested by 'bugs' → delusional parasitosis
103. Patients with a history of complex withdrawals from alcohol (i.e. Delirium tremens, seizures, blackouts) should
be admitted to hospital for monitoring until withdrawals stabilised
104. Patients with antisocial personality disorder often fail to conform to social norms, and show lack of remorse,
deception and irresponsibility
105. Patients with avoidant personality disorder are fearful of criticism, being unliked, rejection and ridicule
106. Patients with Charles-Bonnet syndrome experience persistent or recurrent complex visual or auditory
hallucinations however generally have full insight into their condition
107. Patients with dependant personality disorder require excessive reassurance from others, seek out relationships
and require others to take responsibility for major life decisions
108. Patients with obsessive-compulsive personality can be rigid with respect to morals, ethics and values and often
are reluctant to surrender work to others
109. PTSD management - trauma-focused cognitive behavioural therapy or EMDR
110. Sertraline is the SSRI of choice post myocardial infarction
111. Severe depression can mimic dementia but gives a pattern of global memory loss rather than short-term memory
loss - this is called pseudodementia
112. SSRI + NSAID = GI bleeding risk - give a PPI
113. Ssris are associated with hyponatraemia
114. Ssris are the first-line pharmacological therapy for generalised anxiety disorder
115. Tardive kinesia can present as chewing, jaw pouting or excessive blinking due to late onset abnormal
involuntary choreoathetoid movements in patients on conventional antipsychotics
116. Triptans should be avoided in patients taking a SSRI
117. Unexplained symptoms
a. Somatisation = Symptoms
b. Hypochondria = Cancer
118. When stopping a SSRI the dose should be gradually reduced over a 4 week period
119. Although the cognitive disturbances are primarily frontal in origin, gait ataxia is extremely unusual in fronto-
temporal dementia, prompting a search for an alternate diagnosis.
120. Although, hypertension, the cognitive changes are not in keeping with the step-wise deterioration found in
vascular dementia.
388
M Y Elamin
MBBS, DTM&H, MCTM, MRCPI 1& 2
121. Alzheimer's disease produces a cortical dementia with apraxias, agnosia and visuospatial disturbances
122. Subcortical dementias, cause mental slowness, bradyphrenia and executive dysfunction.
123. CT head of the brain is the investigation of choice showing enlarged ventricles disproportionate to the amount
of cerebral atrophy.
124. Diffuse Lewy body disease is characterised by fluctuating cognitive impairment, visual hallucinations and
parkinsonism.
125. Diffuse Lewy body disease are very sensitive to neuroleptics which therefore should be avoided.
126. The treatment of choice for Diffuse Lewy body disease is rivastigmine, which improves both the visual
hallucinations, and cognitive impairment.1
127. Paranoid schizophrenia is dominated by relatively stable, often paranoid delusions, usually accompanied by
hallucinations, particularly of the auditory variety, and perceptual disturbances.
128. In addition Paranoid schizophrenia, he has delusions which have a persecutory content or jealousy theme or
associated with bodily change. He also appears to be having some hallucinations of smell, has had changes in weight
and neglect of personal hygiene.
129. The clinical picture of Paranoid schizophrenia is dominated by delusions which are fixed ideas, with less features
of mood change; hence psychotic depression and bipolar disease are less likely diagnoses.
130. Schizoid personality disorder is characterised by emotional coldness, detachment, limited capacity to express
emotion which subsequently leads to few friends or close relationships.
131. Normal pressure hydrocephalus presents with the triad of:
a. Dementia
b. Gait disturbance and
c. Incontinence.
132. Normal pressure hydrocephalus is associated with episodes of increased pressure related to impaired
cerebrospinal fluid (CSF) absorption and therefore may benefit from CSF shunting.
133. Although Normal pressure hydrocephalus, the ventricular enlargement described in this case may derive from
global atrophy and need not represent hydrocephalus the other features of the case strongly point to the diagnosis.
134. In equivocal the Normal pressure hydrocephalus cases therapeutic CSF drainage via lumbar puncture is
performed to identify the patients likely to benefit from permanent drainage.
135. Intracranial pressure monitoring to demonstrate the episodes of increased pressure is a more invasive
alternative. treated with repeated therapeutic lumbar puncture.
136. The patient is suffering from panic disorder - sudden, discrete attacks of intense anxiety or fear accompanied
by physical symptoms, for example, palpitations and a feeling of suffocation. To distinguish it from a specific
phobia, some of the attacks must occur without an environmental trigger.
137. Somatisation disorder describes a condition of multiple symptoms and health seeking. It is a chronic condition
and does not occur in discrete bursts.

389
M Y Elamin
MBBS, DTM&H, MCTM, MRCPI 1& 2
138. An adjustment disorder patients should describe any antecedent event bringing on her symptoms (excessive
response to the event or change within three months of it happening).
139. Abnormal discharge from the locus caeruleus in the midbrain has been implicated in panic attacks. The locus
caeruleus is the origin of most brain noradrenergic pathways.
140. the atypical antipsychotics such as olanzapine/risperidone/clozapine have been associated with
hyperglycaemia and insulin resistance. The mechanism remains obscure, but withdrawal of the medication may
produce resolution of the diabetes.
141. The atypical agents are increasingly popular as they have the advantage of less extrapyramidal and
anticholinergic side effects compared with traditional antipsychotics such as haloperidol.
142. Haloperidol has been associated with hypoglycaemia.
143. Lithium is associated with DI and deranged thyroid function.
144. Phenelzine, a mono-amine oxidase inhibitor is associated with dry mouth and constipation.
145. Valproate can be used in the treatment of manic disorders and side effects include leukopenia, deranged liver
function tests (LFTs) and weight gain.
146. borderline personality disorder are idealisation and rejection of others manifest by apparent criticism of
previous care and staff, and idealisation of the current doctor. In addition there are sudden unwarranted
aggressive outbursts and probable self-harm, with a background of substance abuse. All these features are found
in those exhibiting borderline personality disorders.
147. Antisocial personality disorder manifests as aggression, violence with criminal tendencies and an inability to
sustain loving relationships.
148. Narcissistic personalities have a grandiose sense of self-importance or uniqueness, preoccupied with fantasies
of unlimited success, power or brilliance. They are often arrogant and display haughty behaviour.
149. Anxious personality disorders are often shy and feel inadequate and have a tendency to avoid responsibilities.
150. Affective personality disorders have lifelong abnormalities in mood regulation.
151. syndrome of inappropriate antidiuretic hormone (SIADH) as suggested by the hyponatraemia with high urine
sodium and osmolality.
152. Drugs that may cause SIADH include:
a. Selective serotonin reuptake inhibitors (SSRIs) (fluoxetine)
b. Tricyclic antidepressants
c. Sulphonylureas
d. Thiazides, and
e. Carbamazepine.
153. Other causes of SIADH include:
a. Pneumonias
b. Meningitis
c. Guillain-Barré
390
M Y Elamin
MBBS, DTM&H, MCTM, MRCPI 1& 2
d. Trauma, and
e. Malignancy.
154. Lithium would cause diabetes insipidus (DI).
155. Neuroleptic malignant syndrome (NMS), can occur at any time during the treatment of antipsychotic
medications.
156. Concomitant treatment with lithium or anticholinergics may increase the risk of NMS. It is manifested by:
a. Fever
b. Rigidity
c. Altered mental status, and
d. Autonomic dysfunction.
157. NMS Treatment includes withdrawal of the offending agent and reduction of body temperature with
antipyretics. Dantrolene, bromocriptine or levodopa preparations may be beneficial.
158. Benztropine, procyclidine and lithium may all precipitate NMS.
159. Consider neuroleptic malignant syndrome in a patient taking antipsychotic medications and with fever, muscle
rigidity, confusion and autonomic dysfunction.
160. The following characterise neuroleptic malignant syndrome:
a. Confusion/drowsiness
b. Autonomic dysfunction
c. Pyrexia, and
d. Extrapyramidal signs.
161. Discontinuation of the neuroleptic and initiation of fluids and anti-pyretics are essential.
162. Lithium toxicity can cause diarrhoea, but may also be precipitated by dehydration. All of the symptoms may
be due to lithium toxicity (diarrhoea, tremor and dysarthria). Lithium levels should be taken, but may be of
limited value in the acute setting (rapid result may not be available; levels not always reliable especially with sustained
release preparations).
163. The management of lithium toxicity is largely supportive. The first step is to establish renal function and
correct serum electrolytes. Renal function will determine the patient's ability to excrete lithium.
164. bipolar I disorder which is characterised by severe alterations in mood (mania and depression) that are usually
episodic and recurrent.
165. Bipolar II disorder is characterised by one or more major depressive episodes (which this patient did not have),
at least one hypomanic and no manic episodes.
166. Patients with bipolar disorder, particularly those with bipolar II disorder who do not exhibit overt symptoms
of mania, are frequently misdiagnosed as having unipolar depression.
167. Establishing the diagnosis of bipolar disorder is essential, in order to appropriately treat acute episodes and
provide maintenance therapy.

391
M Y Elamin
MBBS, DTM&H, MCTM, MRCPI 1& 2
168. Mixed state refers to the presence of both depressive and mood elevated (manic or hypomanic) symptoms
simultaneously. It may thus occur with bipolar I or bipolar II disorder.
169. In dysthymic disorder, the patient's mood is chronic depression with never a manic or hypomanic episode, for
at least two years.
170. Sodium valproate and carbamazepine are efficacious as first line treatment in the prophylaxis of manic and
depressive episodes in bipolar I disorder. Lithium may be used if these anticonvulsants are ineffective.
171. However, in the initial stages of manic episodes, the addition of drugs with potent sedative effects are often
required, for example, clonazepam, lorazepam and haloperidol. These drugs can be tapered and then discontinued
as soon as the initial phase of the manic episode has subsided and the effects of the anticonvulsants or lithium are
seen clinically.
172. panic disorder - sudden, discrete attacks of intense anxiety or fear accompanied by physical symptoms, for
example, palpitations and a feeling of suffocation. To distinguish it from a specific phobia, some of the attacks
must occur without an environmental trigger.
173. Somatisation disorder describes a condition of multiple symptoms and health seeking. It is a chronic condition
and does not occur in discrete bursts.
174. An adjustment disorder is describe any antecedent event bringing on her symptoms.
175. Abnormal discharge from the locus caeruleus in the midbrain has been implicated in panic attacks.
176. The locus caeruleus is the origin of most brain noradrenergic pathways.
177. SSRIs are less cardiotoxic than tricyclics in overdose. Apart from that, SSRIs are better than monoamine-oxidase
inhibitors (MAOIs) because they are more effective and do not show the dangerous interactions with some foods
and have fewer dangerous interactions with drugs than are characteristic of the traditional MAOIs.
178. Citalopram is the safest antidepressant to use in patients who are also prescribed warfarin.
179. Warfarin has a narrow therapeutic index which is affected by a number of drugs, primarily through the
cytochrome P450 isoenzyme system.
180. Selective serotonin re-uptake inhibitors (SSRIs) may enhance the anticoagulant effect of warfarin, as may the
serotonin and noradrenaline re-uptake inhibitor (SNRI) venlafaxine.
181. Current data suggests that sertraline and citalopram appear to be the safest antidepressants to prescribe with
warfarin. Fluvoxamine and fluoxetine appear to pose the highest potential risk. The remaining antidepressants
appear to lie somewhere in between.
182. St John's wort (Hypericum perforatum), a herbal antidepressant available over the counter in the United
Kingdom, reduces the anticoagulant effect of warfarin and concomitant use should be avoided. In cases of true
depression, an antidepressant is indicated.
183. Tricyclic antidepressants are used much less frequently now, as safer antidepressants have been introduced.
Their use in cardiac patients is associated with a number of potential complications, due to antimuscarinic and
quinidine-like properties. There is a dose-dependent increase in the plasma half-life of warfarin.

392
M Y Elamin
MBBS, DTM&H, MCTM, MRCPI 1& 2
184. Venlafaxine has been reported to enhance warfarin, and it must be used with caution in patients with cardiac
disease. As with tricyclics, NICE recommends that it should not be used in patients with a high risk of serious
cardiac arrhythmias or a recent myocardial infarction.
185. Consider serotonin syndrome in a patient who suddenly develops neuromuscular hyperactivity, fever,
confusion and muscle rigidity.
186. Serotonin syndrome is a reversible monoamine oxidase inhibitor (MAOI). It is well known to cause serotonin
syndrome. The syndrome has many common features with neuroleptic malignant syndrome.
187. Serotonin syndrome is often misdiagnosed as NMS, but the two can easily be distinguished on the basis of
history, examination findings, and clinical course.
188. NMS develops over days to weeks, whereas serotonin syndrome develops over 24 hours.
189. serotonin syndrome is characterized by neuromuscular hyperreactivity (tremor, hyperreflexia, myoclonus),
while NMS involves sluggish neuromuscular responses (rigidity, bradyreflexia).
190. Hyperreflexia and myoclonus are rare in NMS. Also, resolution of NMS typically requires an average of nine
days, compared with less than 24 hours (usually) for resolution of serotonin syndrome
191. Hyperthermia, altered mental status, muscle rigidity, leukocytosis, elevated creatine phosphokinase, elevated
hepatic transaminases, and metabolic acidosis are seen in severe cases of both conditions (serotonin syndr &
NMS), which highlights the necessity of a thorough history and physical examination.
192. A cyclothymic disorder is characterised by the presence of numerous periods of both depression (but not major
depressive episodes) and hypomania for at least two years.
193. The crucial feature of a major depressive disorder is a severe dysphoric mood and persistent loss of interest or
pleasure in all usual activities.
194. Bipolar I disorder is characterised by severe alterations in mood (mania and depression) that are usually
episodic and recurrent.
195. In dysthymic disorder, the patient's mood is chronic depression with never a manic or hypomanic episode, for
at least two years.
196. Bipolar II disorder is characterised by one or more major depressive episodes, at least one hypomanic episode
and no manic episodes.
197. A bipolar I disorder is characterised by severe alterations in mood (mania and depression) that are usually
episodic and recurren
198. Abuse of androgenic steroids can cause paranoid dellusions and aggression.
199. Bromocriptine should be used in the treatment of NMS.
200. Lithium toxicity can cause diarrhoea, but may also be precipitated by dehydration.
201. Normal pressure hydrocephalus is treated with repeated therapeutic lumbar puncture.
202. Normal pressure hydrocephalus presents clasically with the triad of cognitive impairm
203. SSRIs are less cardiotoxic than tricyclics in overdose.
204. The locus Caeruleus is involved in the development and trigger of panic disorders
393
M Y Elamin
MBBS, DTM&H, MCTM, MRCPI 1& 2
205. A diagnosis of depersonalization disorder requires repeated episodes of feeling in some way unreal or
detached from one’s body.
206. A family history is especially associated with depressive illness and schizophrenia.
207. A history of polyuria and polydipsia, with only a marginally reduced sodium make psychogenic
polydipsia a likely diagnosis.
208. A patient who has driven to an isolated area to make a suicide attempt, where there was little chance of
being found, suggests a serious attempt at suicide which predicts future suicidal intent.
209. A presentation of early morning waking, low mood and suicidal intent should make you consider
depression.
210. A serious side effect of clozapine is agranulocytosis. Leukopaenia is an indication to permanently stop
clozapine treatment.
211. Adjustment disorder occurs within three months of an identifiable stressor and lasts six months from the
withdrawal of the stressor.
212. Akathisia (restlessness, or an inability to sit still) is a typical side effect associated with the use of the
atypical antipsychotic olanzapine.
213. Akathisia is a recognised side effect of antipsychotics, and should initially be treated by dose reduction
of antipyschotics. Benzodiazepines can be effective if dose reduction is not possible
214. Alcohol withdrawal can lead to anxiety, agitation, fever, sweats and tremor, peaking after 72 hours and
usually resolved by 3 weeks.
215. Alcohol withdrawal in an habitual abuser is a well recognised cause of fits together with the altered
behaviour.
216. Alcoholic hallucinosis can appear 12-24 hours after stopping alcohol and includes visual, auditory and
tactile hallucinations
217. Alzheimer's disease is characterised early in the disease by short term memory loss.
218. Alzheimer's disease is the most common form of dementia
219. Amphetamine use can lead to a clinical presentation mimicking schizophrenia
220. An inability to generate a list rapidly is typical of frontal lobe dysfunction
221. Anorexia nervosa is associated with impaired glucose tolerance.
222. Auditory hallucinations associated with schizophrenia typically are located by the patient in external
space and have the same quality as real perceptions.
223. Auditory hallucinations suggestive of schizophrenia include hearing voices discussing oneself in the
third person.
224. Biological symptoms of depression include early morning wakening, and loss of appetite and libido.

394
M Y Elamin
MBBS, DTM&H, MCTM, MRCPI 1& 2
225. Borderline personality disorder is characterised by, amongst other things, excessive attempts to avoid
perceived abandonment.
226. Carbamazepine has been shown to be as effective as oxazepam in treating alcohol withdrawal
227. CBT can be effective at reducing the severity of symptoms in chronic fatigue syndrome.
228. Citalopram is the safest antidepressant to use in patients who are also prescribed warfarin.
229. Conversion to section 2 (assessment order) or 3 (treatment order) is permitted in a section 5(2).
230. DaTscan™ (Ioflupane, 123-I FP-CIT) can be used to diagnose Lewy body dementia and Parkinson's
disease.
231. Delirium should be treated with short-term haloperidol or olanzapine, if verbal or behavioural
techniques are not successful.
232. Delusions of reference and thought insertion support a diagnosis of schizophrenia.
233. Dementia is an organic illness which is not induced by ECT.
234. Depersonalisation is not exclusively seen in schizophrenics.
235. Depersonalization is an unpleasant experience that oneself is in some way no longer real or fully human,
and is most often associated with anxiety.
236. Deprivation of Liberty Safeguarding (DOLS) legislation is designed to fill the 'Bournewood gap'
whereby the consent of a mentally incapacitated adult to remain in a given safe or therapeutic environment
is in doubt.
237. Disulfiram irreversibly inhibits the oxidation of acetaldehyde, the subsequent increased levels of which
are thought to produce the typically unpleasant side effects
238. Dopamine agonists are a significant cause of psychotic symptoms in Parkinson’s Disease, and should be
withdrawn if this occurs.
239. Drug interaction between lithium and diuretics may precipitate lithium toxicity.
240. Echolalia (repetition of what another individual has just said) is a recognised feature of Tourette’s
syndrome.
241. ECT is best avoided within three months of a cerebrovascular event
242. ECT is indicated in patients who derive no benefit from antidepressant medication with psychotic or
sever depression.
243. Emotional instability is a core feature of borderline personality disorder.
244. Essential tremor is the commonest cause of head tremor, and can be reduced with propranolol.
245. Frontal lobe brain damage is classically associated with personality change and deterioration in intellect,
but perseveration may also occur.
246. Globus hystericus is part of the anxiety disorders and thought to be due to somatisation.
395
M Y Elamin
MBBS, DTM&H, MCTM, MRCPI 1& 2
247. Grandiose delusions, flight of ideas and pressure of speech fit best with a diagnosis of mania.
248. Haloperidol is recommended by NICE guidelines for the treatment of delirium where non-
pharmacological interventions have not succeeded.
249. Heavy cannabis use is associated with a seven fold increase in the risk of developing schizophrenia.
250. Hyperventilation is often associated with distal paraesthesia
251. In cases of suspected neuroleptic malignant syndrome, a CK level is the best investigation.
252. In malingering, the patient consciously fakes or claims to have a disorder in order to attain a specific
gain (for example, financial).
253. In situations involving attempted suicide, you need to assess whether the patient has capacity.
254. In TCA overdose, 50 ml of 8.4% sodium bicarbonate should be given if the pH is less than 7.1, QRS
interval is more than 0.16 s, or there are cardiac arrhythmias or hypotension.
255. In the case of the Mental Capacity Act, the specific tests applied are that the individual must show an
ability to understand, weigh up and retain the relevant information.
256. In the west, six months is considered average for grief, most authorities would consider greater than 12
months abnormal.
257. Incongruity of affect is consistent with a diagnosis of schizophrenia
258. IV sodium bicarbonate can be used in severe acidosis prior to dialysis treatment.
259. Korsakoff's is associated with short term memory loss with subsequent compensatory confabulation by
the patient.
260. Lewy body dementia is a form of dementia that is closely associated with Parkinson's disease, and is
characterised by clumps of alpha-synuclein and ubiquitin protein in neurons (Lewy bodies).
261. Linear lesions are rarely caused by organic disease, and should make you query dermatitis artifacta
especially when the lesions are inaccessible locations such as the forearms.
262. Lithium toxicity occurs at levels above 1.4 mmol/L and can result in anorexia, diarrhoea, vomiting,
ataxia, nystagmus, dysarthria, confusion and seizures.
263. Making an effort to conceal an overdose suggests a serious intent to complete suicide.
264. MDMA (ecstasy) can present with a variety of features in overdose including cerebral oedema, DIC,
hyperpyrexia, serotonin syndrome and renal failure.
265. MDMA overdose can present as a serotonergic syndrome, which is characterised by agitation,
hyperreflexia, hyperthermia, tachycardia and hypertension.
266. Mitochondria has its own self-replicating DNA. It replicates using proteins found in the nucleoids which
also house the DNA.

396
M Y Elamin
MBBS, DTM&H, MCTM, MRCPI 1& 2
267. Monothematic delusions, such as belief in healing powers are commonly seen in patients with bipolar
disorder, both in manic and depressive episodes.
268. Mutism is a symptom associated with a functional disorder.
269. Neuroleptic malignant syndrome (NMS) is characterised by fever, muscular rigidity, altered mental
status, and autonomic dysfunction.
270. NICE guidelines on Depression in adults (CG90) support the use of trauma focussed cognitive
behavioural therapy as the mainstay of therapy for PTSD.
271. Nurture in an institution does not predispose individuals to schizophrenia.
272. Oculogyric crisis should be treated with procyclidine (usually IV or IM) or benztropine.
273. Over-perception of body image is typical of anorexia nervosa
274. Parkinson symptoms, coupled with memory loss and visual hallucinations, are all pointers towards an
underlying diagnosis of Lewy body dementia.
275. Patients with obsessional neurosis (obsessive compulsive disorder) usually maintain good insight into
their condition
276. Patients with somatisation disorder make slower progress if they think that their Doctor doesn't believe
them. Empathy, not persuasion is the key to management.
277. Prochlorperazine is first line in management of psychotic symptoms during acute attacks of acute
intermittent porphyria.
278. Puerperal psychosis is a mood disorder with features of loss of contact with reality, hallucinations,
thought disorder and abnormal behaviour. Prognosis is good.
279. Quetiapine and Clozapine are significantly less likely to cause extrapyramidal side effects in PD patients
and are the treatment of choice where an antipsychotic is required.
280. Rapid eye movement (REM) sleep behaviour disorder demonstrates increased electromyographic tone
and dream enactment, often associated with the violent re-enacting of dreams and occuring when the
normal atonicity of REM sleep is lost.
281. Recent bereavement has a highly significant impact on suicide risk, with SMRs for females over 100
times higher for those in the first week after losing a spouse.
282. Relaxation and counselling are appropriate ways of managing exam stress and preferable to the use of
benzodiazepines
283. Risperidone is the only antipsychotic licensed for treating challenging behaviour in dementia.
Antipsychotics are associated with increased mortality in patients with dementia and should be avoided
unless absolutely necessary.
284. Somatisation disorder is characterised by multiple recurring pains and gastrointestinal, sexual, and
pseudo-neurologic symptoms that occur over a period of years.

397
M Y Elamin
MBBS, DTM&H, MCTM, MRCPI 1& 2
285. Somatisation disorder is characterised by multiple, recurrent and changing symptoms for which no
physical cause can be found. These typically last for over two years, and can be associated with multiple
medical admissions.
286. SSRIs are first line drug treatment for generalised anxiety disorder.
287. Subdural haematoma can be associated with gradual onset of headaches, memory loss, personality
change, dementia, confusion and drowsiness (all of which can be fluctuating).
288. Suppression of T3 may be observed in patients with anorexia, although T4 and TSH may be normal.
289. Symptoms of anxiety neurosis include headache, hyperventilation (tetany and tingling) and palpitations,
related to the stressor
290. The best evidence-based treatment for chronic fatigue syndrome is graded exercise therapy.
291. The Committee on Safety of Medicines (CSM) have reported that hyponatraemia is associated with all
types of antidepressants.
292. The symptom of thought broadcasting is one of Schneider's first rank symptoms, which are widely
regarded as specific to schizophrenia.
293. The use of antidepressants has been linked with suicidal thoughts and behaviour. Where necessary
patients should be monitored for suicidal behaviour, self harm or hostility, particularly at the beginning of
treatment or when the dose is changed.
294. Third person auditory hallucinations with running commentary, and attempts to prevent 'thought
broadcasting' are first rank symproms of schizophrenia.
295. Tourette's syndrome can present with either vocal or motor tics. Vocal tics may be simple, such as
grunting, or complex, such as phrase recognition (palilalia).
296. Tricyclic anti-depressants are likely to carry significantly more risk than selective serotonin reuptake
inhibitors (SSRIs) in overdose.
297. Unemployment and low socio-economic groups are consistently associated with a higher risk of suicide
298. Unlike the tricyclic antidepressants, many of the SSRIs are safe in overdose and cause very few effects.
299. Urinary incontinence would be particularly unusual in association with a depressive episode and would
be far more typical of dementia.
300. Urinary retention is a known side effect of amitriptyline and similar tricyclic antidepressants.
301. When there are no abnormal physical signs in a paitent with repeated episodes of breathlessness and
palpitations lasting a short period of time and resolving gradually, the likley diagnosis is panic attacks
302. A bipolar I disorder is characterised by severe alterations in mood (mania and depression) that are
usually episodic and recurrent.
303. A cyclothymic disorder is characterised by the presence of numerous periods of both depression (but not
major depressive episodes) and hypomania for at least two years.

398
M Y Elamin
MBBS, DTM&H, MCTM, MRCPI 1& 2
304. A panic disorder is different from a specific phobia when some of the attacks occur without an
environmental trigger.
305. Abuse of androgenic steroids can cause paranoid dellusions and aggression.
306. Atypical antipsychotics such as olanzapine/risperidone/clozapine have been associated with
hyperglycaemia and insulin resistanc.
307. Bromocriptine should be used in the treatment of NMS.
308. Citalopram is the safest antidepressant to use in patients who are also prescribed warfarin.
309. Consider neuroleptic malignant syndrome in a patient taking antipsychotic medications and with fever,
muscle rigidity, confusion and autonomic dysfunction.
310. Consider serotonin syndrome in a patient who suddenly develops neuromuscular hyperactivity, fever,
confusion and muscle rigidity.
311. Diffuse Lewy body disease is characterised by fluctuating cognitive impairment, visual hallucinations
and parkinsonism.
312. Drugs that may cause SIADH include Selective serotonin reuptake inhibitors (SSRIs), Tricyclic
antidepressants, Sulphonylureas, Thiazides, and Carbamazepine.
313. In psychotic depression there may be hallucinations, typically accusing or derogatory voices.
314. Lithium toxicity can cause diarrhoea, but may also be precipitated by dehydration.
315. Neuroleptic malignant syndrome (NMS) is characterised by fever, muscular rigidity, altered mental
status, and autonomic dysfunction.
316. Normal pressure hydrocephalus is treated with repeated therapeutic lumbar puncture.
317. Normal pressure hydrocephalus presents clasically with the triad of cognitive impairment, gait apraxia,
and urinary incontinence
318. Paranoid schizophrenia is dominated by relatively stable, often paranoid delusions, usually accompanied
by hallucinations, particularly of the auditory variety, and perceptual disturbances.
319. SSRIs are less cardiotoxic than tricyclics in overdose.
320. The locus Caeruleus is involved in the development and trigger of panic disorders
321.

399
M Y Elamin
MBBS, DTM&H, MCTM, MRCPI 1& 2
RESPIRATORY MEDICINE
1. A 30-year-old man of African origin presents with a non-productive cough and dyspnoea. He also complains of
multiple joint pains. A chest x-ray shows bilateral hilar lymphadenopathy - sarcoidosis
2. A 30-year-old man with a history or repeated sinus and chest infections is investigated for infertility. On examination
heart sounds are difficult to auscultate - Kartagener's syndrome
3. A 45-year-old man is investigated for renal impairment. Over the past few weeks he has suffered from chronic
sinusitis and haemoptysis - Granulomatosis with polyangiitis
4. A 50-year-old alcoholic presents with fever. He is found to have an right upper lobe pneumonia - Klebsiella
pneumonia
5. A 50-year-old farmer presents with recurrent episodes of dyspnoea, cough and fever. A chest x-ray taken during one
of these episodes shows mid-zone fibrosis - extrinsic allergic alveolitis
6. A 50-year-old man who has smoked for the past 30 years presents with dyspnoea on exertion and a cough productive
of small amounts of sputum in the morning. Spirometry shows a FEV1/FVC ratio of 60% - chronic obstructive
pulmonary disease
7. A 50-year-old man with a history of sinusitis presents with haemoptysis and renal impairment. A renal biopsy shows
crescentic glomerulonephritis - Granulomatosis with polyangiitis
8. A 50-year-old man with late-onset asthma presents with malaise and weight loss. He also complains of recurrent
bouts of sinusitis. Bloods show a marked eosinophilia - Churg-Strauss syndrome
9. A 60-year-old man presents with a 4-week history of a productive cough, foul-smelling sputum and night sweats -
lung abscess
10. A 60-year-old patient with a history of COPD presents with an infective exacerbation characteristed by a productive
cough and consolidation on the chest x-ray - Haemophilus influenzae pneumonia
11. A 6-month-old develops mild fever, a dry cough and increased work of breathing during winter. On auscultation
there is some wheezing and fine inspiratory crackles - respiratory syncytial virus
12. A 75-year-old man who used to work in a foundry presents with progressive dyspnoea. A chest x-ray shows upper
zone interstitial lymph nodes with 'egg-shell' calcification of the hilar lymph nodes - silicosis
13. Acetazolamide - carbonic anhydrase inhibitor
14. Adult with asthma not controlled by a SABA - add a low-dose ICS
15. Adult with asthma not controlled by a SABA + low-dose ICS - add a leukotriene receptor antagonist
16. Adult with asthma not controlled by a SABA + low-dose ICS + leukotriene receptor antagonist - add a LABA
17. Adult with asthma not controlled by a SABA + low-dose ICS +/- leukotriene receptor antagonist + LABA - switch
ICS/LABA for a MART regime with a low-dose ICS
18. Adult with asthma not controlled by a SABA +/- leukotriene receptor antagonist + MART regime with a low-dose
ICS - switch to either a MART regime with a moderate-dose ICS or to separate moderate-dose ICS and LABA
inhalers

400
M Y Elamin
MBBS, DTM&H, MCTM, MRCPI 1& 2
19. Adult with asthma not controlled by a SABA +/- leukotriene receptor antagonist + MART regime with a moderate-
dose ICS - trial of either high-dose ICS as part of fixed-dose regime, additional drug (e.g. LAMA or theophylline)
or referral
20. Alcoholic/diabetic, 'red-currant jelly' sputum - Klebsiella pneumoniae pneumonia
21. Allergic bronchopulmonary aspergillosis , treatment of choice: prednisolone
22. Alpha-1 antitrypsin deficiency - panacinar emphysema
23. Alpha-1 antitrypsin deficiency is autosomal recessive
24. Altitude - respiratory alkalosis
25. An 80-year-old man who used to work in ship building presents with progressive shortness-of-breath. A chest x-ray
shows multiple pleural plaques and bilateral lower zone interstitial shadowing - asbestosis
26. Anaemia causes a lower TLCO
27. Anxiety leading to hyperventilation - respiratory alkalosis
28. Asthma causes a raised TLCO
29. Benzodiazepines overdose - respiratory acidosis
30. Bronchiectasis, infertility, situs inversus - Kartagener syndrome
31. Bronchiolitis - immunofluorescence of nasopharyngeal secretions
32. Bronchiolitis - respiratory syncytial virus
33. Bupropion - contraindicated by: epilepsy
34. Churg-Strauss syndrome - asthma
35. Churg-Strauss syndrome - chronic sinusitis
36. Churg-Strauss syndrome - eosinophilia
37. Churg-Strauss syndrome - positive pANCA serology
38. CNS disorders e.g. Stroke, subarachnoid haemorrhage, encephalitis - respiratory alkalosis
39. COPD - respiratory acidosis
40. COPD - spirometry
41. COPD not controlled with SABA/SAMA + LABA + ICS, asthmatic/steroid responsive features present - add a
LAMA
42. COPD not controlled with SABA/SAMA, asthmatic/steroid responsive features present - add a LABA + ICS
43. COPD not controlled with SABA/SAMA, no asthmatic/steroid responsive features - add a LABA + LAMA
44. Cystic fibrosis - chromosome 7
45. Cystic fibrosis - delayed puberty
46. Cystic fibrosis - diabetes mellitus
47. Cystic fibrosis - malabsorption
48. Cystic fibrosis - meconium ileus
49. Cystic fibrosis - nasal polyps
50. Cystic fibrosis - rectal prolapse
401
M Y Elamin
MBBS, DTM&H, MCTM, MRCPI 1& 2
51. Cystic fibrosis - recurrent chest infections
52. Cystic fibrosis - short stature
53. Cystic fibrosis - subfertility/infertility
54. Dyspnoea, cough, painful shin lesions, bilateral hilar lymphadenopathy - sarcoidosis
55. Emphysema causes a lower TLCO
56. Exercise causes a raised TLCO
57. Expiratory reserve volume - maximum volume of air that can be expired at the end of a normal tidal expiration
58. Fever, renal impairment, haemoptysis, palpable purpura, no sinus symptoms - microscopic polyangiitis
59. Granulomatosis with polyangiitis - cANCA antibodies
60. Granulomatosis with polyangiitis - chronic sinusitis
61. Granulomatosis with polyangiitis - epithelial crescents in Bowman's capsule
62. Granulomatosis with polyangiitis - haemoptysis
63. Granulomatosis with polyangiitis - positive cANCA serology
64. Granulomatosis with polyangiitis - rapidly progressive glomerulonephritis
65. Granulomatosis with polyangiitis - saddle-shape nose deformity
66. Hypercalcaemia secondary to sarcoidosis - macrophages cause an increased conversion of vitamin D to its active
form (1,25-dihydroxycholecalciferol)
67. Hyperkinetic states causes a raised TLCO
68. Idiopathic pulmonary fibrosis - high-resolution CT scan
69. Infective exacerbation of COPD - Haemophilus influenzae
70. Inspiratory capacity - equals tidal volume + inspiratory reserve volume
71. Inspiratory reserve volume - maximum volume of air that can be inspired at the end of a normal tidal inspiration
72. Ipratropium - antimuscarinic
73. Left-to-right cardiac shunts causes a raised TLCO
74. Life-threatening asthma (decompensated) - respiratory acidosis
75. Life-threatening asthma bradycardia, dysrhythmia or hypotension
76. Life-threatening asthma exhaustion, confusion or coma
77. Life-threatening asthma oxygen sats < 92%
78. Life-threatening asthma PEFR < 33% best or predicted
79. Life-threatening asthma silent chest, cyanosis or feeble respiratory effort
80. Low cardiac output causes a lower TLCO
81. Lower zones lung fibrosis: amiodarone
82. Lower zones lung fibrosis: asbestosis
83. Lower zones lung fibrosis: bleomycin
84. Lower zones lung fibrosis: idiopathic fibrosing alveolitis
85. Lower zones lung fibrosis: methotrexate
402
M Y Elamin
MBBS, DTM&H, MCTM, MRCPI 1& 2
86. Lower zones lung fibrosis: rheumatoid arthritis
87. Lower zones lung fibrosis: SLE
88. Magnesium sulphate, uses include: acute asthma
89. Male gender causes a raised TLCO
90. Metabolic acidosis - ↓ ph - Normal CO2 - ↓ HCO3-
91. Metabolic acidosis with respiratory compensation - ↓ ph - ↓ CO2 - ↓ HCO3-
92. Metabolic alkalosis - ↑ ph - Normal CO2 - ↑ HCO3-
93. Metabolic alkalosis with respiratory compensation - ↑ ph - ↑ CO2 - ↑ HCO3-
94. Moderate asthma - PEFR 50-75% best or predicted
95. Moderate asthma pulse < 110 bpm
96. Moderate asthma RR < 25 / min
97. Moderate asthma speech normal
98. Monteleukast - blocks leukotriene receptors
99. Neuromuscular disease - respiratory acidosis
100. New diagnosis of COPD - add a SABA or a SAMA
101. Newly diagnosed adult with asthma (symptoms < 3 times/weeks) - add a SABA
102. Obesity hypoventilation syndrome - respiratory acidosis
103. Occupational asthma - serial peak flows at home and at work
104. Opiate overdose - respiratory acidosis
105. Pneumonia causes a lower TLCO
106. Polycythaemia causes a raised TLCO
107. Pregnancy - respiratory alkalosis
108. Pulmonary emboli causes a lower TLCO
109. Pulmonary embolism - respiratory alkalosis
110. Pulmonary fibrosis causes a lower TLCO
111. Pulmonary haemorrhage causes a raised TLCO
112. Pulmonary oedema causes a lower TLCO
113. Residual volume - equals fuctional residual capacity - expiratory reserve volume
114. Residual volume - increases with age
115. Residual volume - volume of air remaining after maximal expiration
116. Respiratory acidosis - ↓ ph - ↑ CO2 - Normal HCO3–
117. Respiratory acidosis with metabolic compensation - ↓ / ↔ ph - ↑ CO2 - ↑ HCO3-
118. Respiratory alkalosis - ↑ ph - ↓ CO2 - Normal HCO3-
119. Respiratory alkalosis with metabolic compensation - ↑ / ↔ ph - ↓ CO2 - ↓ HCO3-
120. Salbutamol - beta receptor agonist
121. Salicylate poisoning (initial stages) - respiratory alkalosis
403
M Y Elamin
MBBS, DTM&H, MCTM, MRCPI 1& 2
122. Severe asthma can't complete sentences
123. Severe asthma PEFR 33 - 50% best or predicted
124. Severe asthma pulse > 110 bpm
125. Severe asthma RR > 25/min
126. Shifts the oxygen dissociation curve to the left - alkalosis
127. Shifts the oxygen dissociation curve to the left - carboxyhaemoglobin
128. Shifts the oxygen dissociation curve to the left - HBF
129. Shifts the oxygen dissociation curve to the left - hypothermia
130. Shifts the oxygen dissociation curve to the left - low 2,3-DPG
131. Shifts the oxygen dissociation curve to the left - low pCO2
132. Shifts the oxygen dissociation curve to the left - methaemoglobin
133. Shifts the oxygen dissociation curve to the right - acidosis
134. Shifts the oxygen dissociation curve to the right - hypercapnia
135. Shifts the oxygen dissociation curve to the right - pyrexia
136. Shifts the oxygen dissociation curve to the right - raised 2,3-DPG
137. Sinusitis, asthma, haematuria, eosinophilia - Churg-Strauss syndrome
138. Sinusitis, haemoptysis, haematuria - granulomatosis with polyangiitis
139. Small cell lung carcinoma - ACTH secretion
140. Small cell lung carcinoma - hyponatraemia secondary to ADH secretion
141. Small cell lung carcinoma - Lambert-Eaton syndrome
142. Squamous cell lung carcinoma - hypercalcaemia secondary to PTH-rp
143. Squamous cell lung carcinoma - hypertrophic pulmonary osteoarthropathy
144. Theophylline - non-specific inhibitor of phosphodiesterase resulting in an increase in cAMP
145. Tidal volume - 500ml in males, 350ml in females
146. Tidal volume - volume inspired or expired with each breath at rest
147. Total lung capacity - equals vital capacity + residual volume
148. Upper zones lung fibrosis: ankylosing spondylitis
149. Upper zones lung fibrosis: coal worker's pneumoconiosis
150. Upper zones lung fibrosis: histiocytosis
151. Upper zones lung fibrosis: hypersensitivity pneumonitis
152. Upper zones lung fibrosis: sarcoidosis
153. Upper zones lung fibrosis: silicosis
154. Upper zones lung fibrosis: tuberculosis
155. Vital capacity - 4,500ml in males, 3,500 mls in females
156. Vital capacity - equals inspiratory capacity + expiratory reserve volume
157. Vital capacity - maximum volume of air that can be expired after a maximal inspiration
404
M Y Elamin
MBBS, DTM&H, MCTM, MRCPI 1& 2
158. In a secondary pneumothorax
a. rim was > 2cm and/or the patient is short of breath then patient should be treated with chest drain (not
aspiration) as first-line.
b. Aspiration would be attempted if the patient was not short of breath and the rim was 1-2 cm.
c. if the pneumothorax is less the 1cm then the BTS guidelines suggest giving oxygen and admitting for 24
hours
159. Aspergillus clavatus causes malt workers' lung, a type of Extrinsic allergic alveolitis (EAA)
160. Chlamydia psittaci is a cause of pneumonia in bird keepers
161. Chlamydia psittaci is treated with tetracyclines
162. Saccharopolyspora rectivirgula causes farmer's lung, a type of Extrinsic allergic alveolitis (EAA)
163. A normal pco2 in a patient with acute severe asthma is an indicator that the attack may classified be life-
threatening
164. Acetazolamide (a carbonic anhydrase inhibitor) can be used to prevent acute mountain sickness
165. Acute exacerbation of COPD: BiPaP is indicated if type 2 respiratory failure develops despite medical therapy
166. Adult with asthma not controlled by a SABA - add a low-dose ICS
167. Adults with suspected asthma should have both a feno test and spirometry with reversibility
168. Alpha-1 antitrypsin deficiency: PiMZ Genotype = carrier and unlikely to develop emphysema if a non-smoker
169. Asbestosis causes pulmonary fibrosis predominantly affecting the lower zones
170. Asthmatic features/features suggesting steroid responsiveness in COPD:
a. Previous diagnosis of asthma or atopy
b. A higher blood eosinophil count
c. Substantial variation in FEV1 over time (at least 400 ml)
d. Substantial diurnal variation in peak expiratory flow (at least 20%)
171. Basal atelectasis should be suspected in the presentation of dyspnoea and hypoxaemia 72 hours post
operatively
172. Before starting azithromycin do an ECG (to rule out prolonged QT interval) and baseline liver function tests
173. Bronchiectasis: most common organism = Haemophilus influenzae
174. Bupropion should not be used in a patient with epilepsy as it reduces seizure threshold
175. Calcification in lung metastases is uncommon except in the case of chondrosarcoma or osteosarcoma
176. Catamenial pneumothorax is the cause of 3-6% of spontaneous pneumothoraces occurring in menstruating
women
177. Chronic infection with Burkholderia cepacia is an important CF-specific contraindication to lung
transplantation
178. Churg-Strauss syndrome - positive pANCA serology
179. Confusion in an asthma attack is a life-threatening feature

405
M Y Elamin
MBBS, DTM&H, MCTM, MRCPI 1& 2
180. Consider granulomatosis with polyangiitis when a patient presents with ENT, respiratory and kidney
involvement
181. Contact with camels (including camel products such as milk) is a significant risk factor for MERS-cov
182. Contraindications to lung cancer surgery include SVC obstruction
a. FEV < 1.5
b. MALIGNANT pleural effusion
c. And vocal cord paralysis
183. COPD - LTOT if 2 measurements of po2 < 7.3 kpa
184. COPD - reason for using inhaled corticosteroids - reduced exacerbations
185. COPD - still breathless despite using SABA/SAMA and asthma/steroid responsive features → add a LABA +
ICS
186. COPD - still breathless despite using SABA/SAMA and no asthma/steroid responsive features → add a LABA
+ LAMA
187. COPD is the most common cause of secondary pneumothorax
188. Erythema nodosum is associated with a good prognosis in sarcoidosis
189. Extrinsic allergic alveolitis is associated with upper/mid zone lung fibrosis
190. Flow volume loop is the investigation of choice for upper airway compression
191. Following NICE 2017, patients with asthma who are not controlled with a SABA + ICS should first have a
LTRA added, not a LABA
192. Following weight loss, CPAP is the first-line treatment for moderate/severe obstructive sleep apnoea
193. Functional residual capacity = Expiratory reserve volume + Residual volume
194. Hypercalcaemia + bilateral hilar lymphadenopathy → ?Sarcoidosis
195. If a secondary pneumothorax > 2cm and/or the patient is short of breath then patient should be treated with
chest drain (not aspiration) as first-line
196. In around 10% of patients subsequently diagnosed with lung cancer the chest x-ray was reported as normal
197. Indications for corticosteroid treatment for sarcoidosis are:
a. Parenchymal lung disease,
b. Uveitis,
c. Hypercalcaemia and
d. Neurological or cardiac involvement
198. Isocyanates are the most common cause of occupational asthma
199. Klebsiella most commonly causes a cavitating pneumonia in the upper lobes, mainly in diabetics and alcoholics
200. Klebsiella pneumonia-> commonly due to aspiration
201. Kyphoscoliosis (e.g. Ankylosing spondylitis) can cause a restrictive lung defect on spirometry
202. Leukotriene receptor antagonists may trigger eosinophilic granulomatosis with polyangiitis (Churg-Strauss
syndrome)
406
M Y Elamin
MBBS, DTM&H, MCTM, MRCPI 1& 2
203. Light's criteria: Effusion LDH level greater than 2/3rds the upper limit of serum LDH points to exudate
204. Lower zones lung fibrosis:
a. Amiodarone
b. Methotrexate
205. Lung volume reduction surgery can be used in the treatment of alpha-1 antitrypsin deficiency
206. Management in primary pneumothorax without shortness of breath, and <2cm in size, is discharge and review
207. Management of high altitude cerebral edema (HACE) is with descent + dexamethasone
208. NICE only recommend giving oral antibiotics in an acute exacerbation of COPD in the presence of
a. Purulent sputum or
b. Clinical signs of pneumonia
209. NIV (biPAP) is indicated in respiratory acidosis or rising paCO2 resistant to best medical management during
an acute exacerbation of COPD
210. NIV should be considered in all patients with an acute exacerbation of COPD in whom a respiratory acidosis
(paCO2>6kpa, ph <7.35 ≥7.26) persists despite immediate maximum standard medical treatment
211. Oral glucocorticoids are the treatment of choice for allergic bronchopulmonary aspergillosis
212. Over rapid aspiration/drainage of pneumothorax can result in re-expansion pulmonary oedema
213. Shifts Oxygen dissociation curve to Left
a. Lower oxygen delivery –
b. Lower acidity, temp, 2-3 DPG - also HBF,
c. Carboxy/methaemoglobin
d. Low pCO2
214. Shifts Oxygen dissociation curve to Right
a. Raised oxygen delivery –
b. Raised acidity, temp, 2-3 DPG
c. Hypercapnia
215. Painful shin rash + cough → ?Sarcoidosis
216. Paraneoplastic features of lung cancer
a. Squamous cell: PTHrp, clubbing, HPOA
b. Small cell: ADH, ACTH, Lambert-Eaton syndrome
c. Adenocarcinoma: HPOA, gynaecomastia
217. Pleural plaques are benign and do not undergo malignant change. They, therefore don't require any follow-
up.
218. Pneumothorax -> life long ban on deep sea diving
219. Polysomnography is diagnostic for obstructive sleep apnoea
220. Pthrp is a paraneoplastic syndrome associated with squamous cell lung cancer

407
M Y Elamin
MBBS, DTM&H, MCTM, MRCPI 1& 2
221. Pulmonary arterial hypertension is defined as an elevated pulmonary arterial pressure of greater than
25mmhg at rest or 30mmhg after exercise
222. Recurrent chest infections + subfertility - think primary ciliary dyskinesia syndrome (Kartagener's syndrome)
223. Smoking cessation is very important in patients with asbestos-related lung disease as the risk of lung cancer in
smokers is very high
224. Sarcoidosis CXR
a. 1 = BHL
b. 2 = BHL + infiltrates
c. 3 = infiltrates
d. 4 = fibrosis
225. Serial peak flow measurements at work and at home are used to detect occupational asthma
226. Shifts the oxygen dissociation curve to the left - low pCO2
227. Shifts the oxygen dissociation curve to the right - hypercapnia
228. Sleep apnoea causes include obesity and macroglossia
229. Small cell lung carcinoma secreting ACTH can cause Cushing's syndrome
230. SVC obstruction is a contraindication to surgery in patients with non-small cell lung cancer
231. Symptom control in non-CF bronchiectasis - inspiratory muscle training + postural drainage
232. The majority of patients with sarcoidosis get better without treatment
233. The most common organism causing infective exacerbations of COPD is Haemophilus influenzae
234. The triangle of safety for chest drain insertion involves the base of the axilla, lateral edge pectoralis major, 5th
intercostal space and the anterior border of latissimus dorsi
235. Transfer factor
a. Raised: asthma, haemorrhage, left-to-right shunts, polycythaemia, hyperkinetic states male gender,
exercise
b. Low: everything else, pulmonary fibrosis pneumonia pulmonary emboli pulmonary oedema emphysema
anaemia low cardiac output
236. KCO also tends to increase with age. Some conditions may cause an increased KCO with a normal or reduced
TLCO
a.
237. Unmasking of Churg-Strauss syndrome: Montelukast
238. Upper zones lung fibrosis: hypersensitivity pneumonitis
239. Vital capacity - 4,500ml in males, 3,500 mls in females
240. Vital capacity - maximum volume of air that can be expired after a maximal inspiration
241. 2011 guidelines have since been updated in 2017 which state that patients should switch to a MART
regime if not deriving benefit from a LABA and ICS

408
M Y Elamin
MBBS, DTM&H, MCTM, MRCPI 1& 2
242. A combination of obesity and hypoxia should lead you to consider obstructive sleep apnoea (OSA) as a
diagnosis.
243. A contrast CT thorax can help differentiate between benign and malignant disease and also guide further
investigations if an exudae effusion is suspected.
244. A Heaf test diameter greater than 15 mm raises the possibility of active tuberculosis infection.
245. A high resolution CT scan of the chest will show typical changes assocaited with idiopathic pulmonary
fibrosis.
246. A history of dyspnoea and desaturation on exercise in an HIV-positive patient would suggest PCP. On
examination, the chest is typically clear in PCP.
247. A history of night sweats, haemoptysis and miliary shadowing in an alcoholic is highly suggestive of
TB.
248. A Mantoux test resulting in a reaction over 15mm in size is suggestive of TB infection, and patients
should be referred for further investigation and treatment.
249. A normal perfusion scan has a sensitivity of 98% but a specificity of only 40%.
250. A peak flow diary with multiple measurements at different times in the day and involving both
weekdays and weekends, is the most useful tool in diagnosing workplace asthma.
251. A post-bronchodilator FEV1 of >1.5 litres is needed for a lobectomy, > 2 litres for a pneumonectomy.
252. A post-bronchodilator FEV1 of >1.5 litres is sufficient if a lobectomy is all that is required. If the
tumour necessitates a pneumonectomy, the post-bronchodilator FEV should be more than 2 litres.
253. A post-bronchodilator FEV1 of >1.5 litres is sufficient if a lobectomy is all that is required. If the
tumour necessitates a pneumonectomy, the post-bronchodilator FEV should be more than 2 litres.
254. A reduced transfer factor and CXR evidence of hyperexpanded lung fields points towards a
predominantly emphysematous picture.
255. A rise in cardiac troponin compared with previous one accompanied with a good history for cardiac
chest pain would support the presence of myocardial ischaemia
256. A specific subtype of intermediate care is Hospital at Home (HaH), where active treatment is provided
by healthcare professionals in the patient's home for a condition that otherwise would require hospital care.
257. A tumour size of greater than 3 cm (providing there is no lymphadenopathy or evidence of distant
spread) may still be amenable to surgery as it may fall within stages IB and IIB.
258. A1AT deficiency is associated with a number of malignancies including hepatocellular cancer, lung
cancer, bladder cancer and lymphoma.
259. Abnormalities of the anterior/superior mediastinum may relate to the thymus, thyroid.
260. Air travel advice in COPD.

409
M Y Elamin
MBBS, DTM&H, MCTM, MRCPI 1& 2
261. All patients presenting with possible PE or venous thromboembolism (VTE) should have the clinical
probability assessed. Therefore it is important to note the major and minor risk factors.
262. All tuberculosis patients should have pre-treatment LFTs.
263. Allergic bronchopulmonary aspergillosis is caused by Aspergillus fumigatus, which can present with
asthma and eosinophilia.
264. Allergic bronchopulmonary aspergillosis predominantly affects patients with asthma, cystic fibrosis and
bronchiectasis causing fever, wheeze, cough, chronic sinusitis and malaise.
265. Alpha-1 antitrypsin deficiency results in the very early development of obstructive lung disease, often
with co-existent liver disease.
266. Ambulatory ECG monitoring to discharge the risk of arrhythmia. is the appropriate investigation for
rythm disturances, or carotid sinus hypersensitivity.
267. Amyl nitrite ingestion typically results in methaemoglobinaemia, which is treated with methylene blue.
268. An aortic aneurysm may stretch the phrenic nerve and cause a palsy.
269. Anaplastic carcinomas of the thyroid are those which are most likely to cause compression or infiltration
of the trachea.
270. Anticoagulation with subcutaneous heparin is recommended in most guidelines for treatment of
pulmonary embolism. IV heparin and warfarin are contraindicated in pregnancy.
271. ARDS causes leakage of protein rich fluid into the alveoli, with reduced lung compliance and normal
pulmonary capillary wedge pressure.
272. As both parents are carriers of the CF gene then the chance of another child being affected
(homozygote) is 1 in 4 (25%).
273. Asbestos exposure can cause pulmonary fibrosis.
274. Aspergillus precipitins can contribute significantly to confirming the diagnosis of allergic
bronchopulmonary aspergillosis (ABPA).
275. Assessing a patient’s performance status is important when evaluating the most appropriate treatment
options. It is commonly used by cancer MDTs, but has a role in assessing patients with chronic illnesses
including COPD.
276. Assessment of suspected OSA
277. At high respiratory rates, with severe air flow limitation (for example, acute asthma) there is an
increased and sudden negative intrathoracic pressure on inspiration and this will enhance the normal fall in
blood pressure.
278. Atopy is consistently associated with sensitization to high-molecular-weight (HMW) agents (animals,
fish and shellfish, flour and cereals, enzymes, natural rubber latex).
279. Awareness of causes of cough and side effects of drugs.

410
M Y Elamin
MBBS, DTM&H, MCTM, MRCPI 1& 2
280. Awareness of treatment algorithm for pneumothorax.
281. Bleeding into alveolar space can be manifested as haemoptysis, and together with renal disease or
glomerulonephritis, should prompt an evaluation for systemic vasculitis.
282. BP of 80/50 is an indication for thrombolysis.
283. British thoracic guidelines for pneumothorax.
284. Bronchiolitis obliterans, with or without organising pneumonia, can be a fatal complication of
rheumatoid arthritis (especially in women taking penicillamine).
285. Bronchoscopy identifies up to 80% of carcinoid tumours in the main bronchi.
286. BTS guidelines suggest that adding in a long-acting beta 2 agonist (LABA) is more appropriate than
further increasing inhaled corticosteroids.
287. BTS has issued clear guidelines on the most appropriate antibiotic regime to treat community-acquired
pneumonia, according to the severity, which is often based on the CURB-65 score.
288. Cadmium is a cause of emphysema.
289. Carbon monoxide transfer factor is reduced in severe bullous emphysema.
290. Carcinoid tumour of the lung originate from K cells
291. Carcinoid tumour usually occur in the major bronchi, 85% can be seen bronchoscopically.
292. Carcinoid tumours of the lung rarely cause classical carcinoid syndrome, but can result in GNRH
secretion and acromegaly.
293. Causes of cavitating lung lesions include malignancy (of which squamous cell carcinoma is the most
common), infection (S. aureus, TB, Klebsiella, PCP), infarction, granulomatosis with polyangiitis and
rheumatoid nodules.
294. Chest radiograph should be repeated 6 weeks following treatment for pneumonia, to ensure complete
resolution of radiographic findings.
295. Chlamydia psittaci is endemic in birds including psittacine birds, canaries, finches, pigeons and poultry.
Person to person transmission occurs, especially in a hospital environment.
296. Chloride concentration >60mmol/L with sodium concentration lower than that of chloride on two
separate occasions is diagnostic of cystic fibrosis.
297. Chronic bronchitis is one of the most common respiratory diseases due to cigarette smoking.
298. Chronic obstructive airways disease and subsequent cor pulmonale can lead to right heart failure.
299. Chronic obstructive pulmonary disease (COPD) results in a spirometry pattern suggestive of obstructive
nature
300. Chronic respiratory acidosis presents with raised bicarbonate as compensation has occured.
301. Clarithromycin is a potent 3A4 inhibitor.
411
M Y Elamin
MBBS, DTM&H, MCTM, MRCPI 1& 2
302. Combination chemo-radiotherapy is the recommended first-line treatment for limited stage SCLC.
303. Conservative management with review in the clinic in two to three weeks is most appropriate
manbagement for a small pneumothorax
304. Consider granulomatosis with polyangiitis in a patient with alveolar haemorrhage and epistaxis.
305. Consider the severity of the allergic reaction when selecting antibiotics in penicillin allergic patients.
The chance of cross reactivity with beta-lactams is 10%.
306. Consolidation in the left lower lobe obliterates the diaphragm whilst lingular consolidation will
obliterate the left heart border.
307. Constipation even after the initial few days of life during which meconium ileus occurs is common in
patients with cystic fibrosis.
308. Constrictive pericarditis can be caused by prior tuberculous infection.
309. Contrast-enhanced transthoracic echocardiography is the best test to demonstrate intrapulmonary
vascular dilatation seen in hepatopulmonary syndrome.
310. COPD is the commonest cause of type 2 respiratory failure.
311. Debate exists as to what constitutes a cough event and what period of symptoms is needed to define a
chronic cough. However guidelines have been produced and knowledge of these is required for effective
management of patients.
312. Deranged coagulation associated with nephrotic syndrome is a consequence of AT III deficiency,
increased fibrinogen and increased factor VIIIc.
313. Desaturation with patchy infiltration on CXR suggests a diagnosis of amiodarone-induced lung disease.
314. Diabetes is an indicator of poor prognosis in pneumonia. Remember, each component of the CURB-65
score are also poor prognostic factors.
315. Diagnosis of narcolepsy
316. Differentiation between different suppurative lung diseases
317. Distal intestinal obstruction syndrome occurs in 10-20% of patients with cystic fibrosis and incidence
increases with age with around 80% of cases presenting for the first time in adults
318. Diurnal variation of peak expiratory flow rate (PEFR) greater than 20% is one of the diagnostic criteria
for asthma
319. DLCO measures the lung diffusion capacity for carbon monoxide. As such, smoking within around eight
hours of the test can significantly limit interpretation of the results.
320. Dosulepin is a cause of pulmonary fibrosis.
321. Eaton-Lambert syndrome is a paraneoplastic syndrome with seventy percent of cases occuring in
association with small cell carcinoma.

412
M Y Elamin
MBBS, DTM&H, MCTM, MRCPI 1& 2
322. Eaton-Lambert syndrome is characterised by proximal muscle weakness, depressed or absent tendon
reflexes, and autonomic features.
323. EGPA is a rare form of small vessel vasculitis, characterised by asthma, allergic rhinitis and prominent
peripheral blood eosinophilia.
324. Eosinophilic Granulomatosis with Polyangiitis (EGPA) is a rare form of small-vessel vasculitis,
characterised by asthma, allergic rhinitis and prominent peripheral blood eosinophilia. Rarely, it can cause
either an anterior or a posterior ischaemic optic neuropathy, which presents with visual loss.
325. Eosinophilic granulomatosis with polyangiitis (EGPA) is characterised by asthma, allergic rhinitis and
prominent peripheral blood eosinophilia.
326. Examples of endothelin receptor antagonists include bosentan and ambrisentan. They significantly
reduce pulmonary artery pressure, but adverse effects include peripheral oedema and liver function test
monitoring is recommended.
327. Exposure to beryllium is seen in the nuclear power, telecommunications, semi-conductor and electronics
industries; it results in a similar clinical picture to that of sarcoidosis.
328. Familial primary pulmonary hypertension (PPH) is recognised to account for 15-20% of cases of PPH
and is due to an inherited mutation in the BMPR2 gene which codes for a receptor in the TGF beta family.
329. Fat emboli can develop following long bone fractures, and are characterised by dyspnoea, confusion and
a petechial rash.
330. Fat embolism is thought to occur as a result of release of lipid globules from damaged bone marrow fat
cells.
331. Fibrosing alveolitis associated with rheumatoid arthritis is indistinguishable from idiopathic pulmonary
fibrosis.
332. Flow volume loop is the best test to assess severity of obstruction for retrosternal goitre.
333. Flow volume loops are the best method to detect tracheal obstruction associated with retrosternal goitre.
334. Flu-like symptoms, coupled with SIADH and low alkaline phosphatase, with pleural effusion on CXR
point towards possible Legionella.
335. FVC is the best way to monitor respiratory function in any neurological disorders that can affect the
respiratory muscles
336. General advice for patients with pneumothorax after discharge.
337. Grade I refers to someone very fit. Grade V refers to someone who is breathless at rest. Think of it as a
scale; if you can’t remember the details estimate where the patient would be in relation to I and V.
338. Guidelines from BTS/SIGN on the diagnosis and management of asthma suggest that the presence of
wheeze, breathlessness, chest tightness or cough, particularly if symptoms are worse at night or in the early
morning, after exercise are suggestive of the diagnosis.

413
M Y Elamin
MBBS, DTM&H, MCTM, MRCPI 1& 2
339. Heerfordt's syndrome is an acute presentation of sarcoidosis, which presents with fever, uveitis, swelling
of the parotid and other salivary and lacrimal glands.
340. Hepatotoxicity is a feature of antituberculous treatment. The Joint Tuberculosis Committee of the British
Thoracic Society recommend that liver function should be checked before treatment for clinical cases.
341. High CO2 can help distinguish COPD from other causes of respiratory failure.
342. High dose oxygen (40-60%) should be used in severe asthma attack, together with steroids and
nebulised bronchodilators.
343. High doses of parenteral and nebulised salbutamol are associated with hypokalaemia. This effect may
have been potentiated by concomitant treatment with prednisolone
344. High resolution CT chest is most likely to establish the diagnosis of pulmonary fibrosis.
345. High severity community acquired pneumonia (CURB 3-5) should be treated as soon as possible with
co-amoxiclav 1.2g TDS and clarithromycin 500mg BD.
346. HRCT in IPF may reveal ground glass changes, suggestive of inflammation.
347. Hypercalcaemia in absence of bony metastases occurs in about 15% of squamous cell lung carcinoma
from parathyroid hormone related protein (PTHrP) production.
348. Hypoxia and hypercapnia are a result of acute exacerbation of COPD, the respiratory centre is solely
stimulated by hypoxia. hence respiratory effort became less and the condition worsened when high
concentration oxygen is introduced depriving the paitent of hypoxic drive.
349. If the CURB score is 1-2 then risk of death is increased and hospital admission should be considered.
350. Immediate (type I) reactions occur in virtually all patients with ABPA following intradermal injections
of A. fumigatus extracts, with only 16% developing delayed (type IV) reactions.
351. Immotile cilia syndrome, also known as primary ciliary dyskinesia and includes Kartagener's syndrome,
is an inherited condition where the cilia lining the airways fail to function or function ineffectively.
352. Immunosuppressed patients are at risk of severe manifestations to vaccines such as disseminated
infection with bacillus Calmette-Guérin (BCG).
353. In 30% of cases of haemoptysis, the initial CXR is reported as normal. CT would be most logical as
opposed to proceeding straight to bronchoscopy.
354. In a chest x ray, you are most likelyto see an 'azygous loop' i the right upper zone. It has little clinical
significance.
355. In a patient who presents with shortness of breath and a background of an extensive smoking history,
COPD is an important diagnosis is consider.
356. In asthma, teaching diaphragmatic breathing, (as opposed to thoracic breathing which is practised by
many asthmatics), may significantly impact on symptoms.

414
M Y Elamin
MBBS, DTM&H, MCTM, MRCPI 1& 2
357. In cases of acute severe asthma, ß2-agonists should be administered as soon as possible, preferably
nebulised driven by oxygen.
358. In cases of suspected lung carcinaoma following chest radiograph, contrast-enhanced CT chest
(including the liver and adrenals) should be carried out. If the CT demonstrates a peripheral lung lesion, CT
or ultrasound-guided transthoracic needle biopsy should be offered.
359. In cases of suspected mesothelioma, tissue can be obtained from thoracoscopy with biopsy from
abnormal looking areas
360. In cases, of acute sever asthma, ß2-agonists should be administered as soon as possible, preferably
nebulised driven by oxygen.
361. In CF the bacteria present depends on the age of the patient.
362. In chronic PE warfarinisation is the optimal long term strategy to reduce the risk of further clots.
363. In Farmers Lung, crackles are typically heard at the lung bases.
364. In hypersensitivity pneumonitis, a short course of oral steroids may improve symptoms but optimal
management is removal of exposure to the antigen.
365. In life-threatening asthma ß2-agonists should be administered as soon as possible, preferably nebulised
driven by oxygen
366. In meningeal or cerebral TB, a four drug regime is continued for 12 months (use of steroids is also
recommended) to ensure adequate brain penetration and to prevent cranial nerve compression by meningeal
scarring.
367. In patients unable to produce sputum, undertaking a bronchoscopy with bronchial washings for
microscopy staining and culture is the investigation of choice.
368. In patients under 50 with COPD who present with chest pain, the guidelines advise to aspirate and admit
for 24 hours observation in respect to management of their pneumothorax.
369. In patients with myocardial infarction and arrhythmia, high pulmonary pressures would be expected
after this arrest scenario, as the pulmonary arterioles constrict in response to hypoxia.
370. In patients with with community-acquired pneumonia, the presence of a respiratory rate equal or
>30/min will significantly increased risk of death.
371. In respiratory acidosis with compensation the bicrbonate will be raised.
372. In restrictive lung disease due to respiratory muscle weakness the lung itself can function normally but
muscle weakness will result in grossly low lung volumes including forced expiratory volume in one second
(FEV1), forced vital capacity (FVC) and total lung capacity (TLC).
373. In silicosis, progressive massive fibrosis (PMF), or conglomerate nodules is due to exposure to dust of
high silicon content and hence PMF is more likely with higher silicon exposure than in simple coal
worker's lung.

415
M Y Elamin
MBBS, DTM&H, MCTM, MRCPI 1& 2
374. In those children thought to have a high probability of asthma, it is recommended that clinicians move
straight to a trial of treatment, and reserve further testing for those with a poor response.
375. In tuberculosis of the mediastinal lymph nodes, corticosteroids are effective in reducing lymph node
enlargement and inflammation and hence will help the stridor and breathlessness.
376. Increased alkaline phosphatase is an adverse prognostic feature in cases of small cell lung cancer.
377. Indication of BiPAP in COPD
378. Indication of hypoxic challenge and indication of inflight oxygen requirement.
379. Indications for LTOT in the UK are a pO2 of <7.3 kPa on air, with no evidence of hypercapnia with
oxygen therapy. A pO2 of less than 8 is an indication for LTOT if there is pulmonary hypertension or
polycythaemia.
380. Infertility occurs in 98% of adult men secondary to maldevelopment of the vas deferens or to other
forms of obstructive azoospermia.
381. Interpretation of blood gases is a key element of training. A logical approach is required.
382. IVC filters are indicated in the treatment of emboli where there are contraindications to systemic
anticoagulation.
383. Key to prognosis is removal from the source of carbon monoxide as quickly as possible, and instigation
of high flow oxygen treatment.
384. Klebsiella pneumonia characteristically affects the upper lobes, and can result in abscess formation.
385. Legionnaire's disease should be treated with a fluoroquinolone (such as levofloxacin) or a newer
macrolide (such as azithromycin or clarithromycin).
386. Life threatening haemoptysis is a medical emergency often associated with lung malignancy;
embolisation of this vessel(s) will immediately stem the bleeding whne performed by interventional
radiology.
387. Löfgren's syndrome is a benign form of sarcoidosis. It presents with erythema nodosum, arthropathy,
and bilateral hilar lymphadenopathy syndrome.
388. Long term oxygen therapy reduces mortality from secondary vascular complications but does not affect
the progression of the airways disease
389. Low albumin and elevated cholesterol would suggest nephrotic syndrome.
390. Low serum sodium and osmolality with raised urinary osmolality and urinary sodium above 20 mmol/L
is diagnostic for SIADH.
391. Low severity CAP (CURB 0-1) can be treated with amoxicillin 500mg TDS PO. CURB 2 CAP should
be treated with amoxicillin 500 mg-1 g TDS and clarithromycin 500 mg BD.
392. LTOT is indicated if the pO2 is between 7.3 and 8.0 and there is secondary polycythaemia (as here), or
pulmonary hypertension present.

416
M Y Elamin
MBBS, DTM&H, MCTM, MRCPI 1& 2
393. Lung volume reduction surgery is a palliative treatment which can be used in advanced COPD to
remove the least functional part of the lungs.The upper cut off for referral for lung reduction surgery for
pCO2 is 7.3.
394. Lupus pernio is a chronic, raised, hard skin lesion associated with sarcoid, and is an adverse prognostic
factor.
395. Management of recurrent pneumothorax in the young.
396. Massive PEs can lead to right ventricular strain, which can lead to a troponin rise.
397. Mediastinal radiotherapy leads to symptomatic relief in 80% of patients with superior vena cava
obstruction (SVCO), although case studies have shown this does not always correlate to patency of the
superior vena cava.
398. Metabolic acidosis causes an effort of respiratory compensation
399. Metals causing occupational asthma include isocyanate cobalt, aluminium, chrome, manganese, nickel,
zinc and platinum.
400. Methaemoglobinaemia is the accumulation of reversibly oxidised methaemoglobin causing reduced
oxygen affinity of the Hb molecule with consequent cyanosis. It can occur due to an inherited condition or
as a consequence of drugs such as nitrates.
401. Methotrexate is a recognised cause of pulmonary fibrosis. However, it is sometimes used in the
treatment of idiopathic pulmonary fibrosis as a steroid sparing agent.
402. Methotrexate is a well recognised cause of acute pneumonitis and interstitial lung disease.
403. Methotrexate is recognised to be associated with pulmonary fibrosis, it is however extremely effective in
control of rheumatoid and psoriatic arthritis.
404. Methylprednisolone and cyclophosphamide is the treatment of choice for induction of remission in
Granulomatosis with polyangiitis
405. Miliary TB is the progressive haematological spread of Mycobacterium tuberculosis.
406. Mycobacterium avium causes disseminated infection in patients with advanced HIV, typically when the
CD4 count is less than 50 cells/mm3. This disseminated infection usually causes symptoms of fatigue,
weight loss and fevers.
407. Mycoplasma presents with a slow-onset of symptoms (sometimes over weeks) with prominent
constitutional upset, including headache, chest pain, myalgia and malaise. There is classically a persistent
dry cough.
408. Nebulised tobramycin is a recognised treatment for Pseudomonas colonisation of the lower respiratory
tract
409. Non small cell lung cancer can give rise to gynaecomastia, which may be painful and associated with
testicular atrophy
410. Obesity is a cause of restrictive lung defect on spirometry
417
M Y Elamin
MBBS, DTM&H, MCTM, MRCPI 1& 2
411. Offer LTOT if PaO2 less than 7.3 kPa when stable, or greater than 7.3 and less than 8 kPa when stable
and with COPD complications.
412. One Australian study suggested that colonisation rates for Aspergillus in patients with cystic fibrosis
approach 19%. Rates of ABPA however are much lower at around 5%.
413. One of the CNS signs of type 2 respiratory failure is extensor plantar reflexes.
414. Only primary pneumothoraces (less than 2 cm) which are not associated with dyspnoea should be
managed with discharge and instructions to return if they become dyspnoeic.
415. Only untreated smear-positive pulmonary TB is likely to be infectious. Non-sputum producing patients
are non-infectious.
416. Optimal treatment for acute altitude sickness is to descend to lower altitude if possible.
417. Overall 5 year survival of lung carcinoma is approximately 14%
418. Pancoast's tumour causes pain in the C8 and T1 distribution and Horner's syndrome.
419. Paraneoplastic syndromes are a result of antibody generation from or against malignant cells attacking
normal tissue. Examples include antineuronal antibodies directed against the Purkinje cells of the
cerebellum leading to cerebellar syndrome.
420. Pathogenesis in spontaneous pneumothorax.
421. Patients at risk of type II respiratory failure should be provided with oxygen alert cards to give to
medical professionals in the event of becoming unwell.
422. Patients with chronic obstructive airways disease (COPD) should not, in general, receive more than 24-
28% oxygen without arterial blood gas monitoring.
423. Patients with occupational asthma are characteristically better when on holiday.
424. Patients with potentially operable tumours who are either too unfit for surgery, or who refuse surgery
may be suitable for radical radiotherapy.
425. Patients with Pneumocystis pneumonia often present with signs and symptoms of respiratory distress,
although on examination of the chest there is often no abnormality detected.
426. Peak flow less than 33% predicted is recognised as being a feature of life-threatening asthma.
427. Pentalaminar X bodies (Birbeck granules) found on BAL are considered diagnostic of pulmonary
histiocytosis X and so would not be expected with sarcoidosis.
428. Pleural effusions with a pH of 7.2 or less represent empyemas, and must be managed with a chest drain.
429. Pneumococcal vaccine is an essential vaccine for the elderly.
430. Pneumocystis jiroveci is a eukaryotic microorganism. In immunosuppressed patients, it can cause
pneumonia, which is most recognised in patients with AIDS but can also be seen in those with organ
transplants or when undergoing chemotherapy. A CD4 count of less than 200 is associated with significant
risk.
418
M Y Elamin
MBBS, DTM&H, MCTM, MRCPI 1& 2
431. Pneumocystis jirovecii is confined to the alveolar space of the lung and produces debris and cysts in the
alveolar space with interstitial infiltration of lymphocytes and plasma cells.
432. Polysomnography is the diagnostic test of choice for the diagnosis of obstructive sleep apnoea.
433. Positive predictive value refers to the percentage of people having a positive test who actually have the
disease
434. Postural drainage is the cornerstone to treating bronchiectasis and should be undertaken at least once per
day and more frequently during exacerbations.
435. Prednisolone is the mainstay of initial treatment for sarcoid, continued for 12 months or more in those
patients who respond, but tapered to the minimal effective dose.
436. Presence of spherocytes or fragmented red blood cells on the film is suggestive of haemolytic anaemia.
437. Primary pulmonary tuberculosis is often asymptomatic consisting of primary complex.
438. Primary TB is usually asymptomatic.
439. Primary treatment for acute hypersensitivity pneumonitis is avoidance of the antigen and systemic
corticosteroids.
440. Psittacosis is an infection caught from parrots. It presents as an atypical pneumonia. Complications
include joint pain, myocarditis, endocarditis, hepatitis and neutropenia.
441. Pulmonary emboli result in hypoxia, and therefore cyanosis can be seen.
442. Pulmonary embolism presents as respiratory alkalosis with type 1 respiratory failure.
443. Pulmonary hypertension occurs as a result of emphysema secondary to long term cigarette smoking.
444. Pyloric stenosis is the commonest neonatal surgical condition and hence a detailed understanding of the
pathophysiology is essential.
445. Raised CO2 or the need for mechanical ventilation with raised pressures is indicative of near fatal
asthma.
446. Recognition of atypical pneumonia and legionnaires' disease
447. Recognition of lung disease in HIV
448. Recognition of tension pneumothorax and immediate course of action.
449. Red cell mass studies can distinguish between depleted plasma volume and raised red cell mass, and are
important in the investigation of polycythaemia.
450. Remember that ptosis and constriction of the pupil can arise as a consequence of a pulmonary lesion
451. Replacement alpha-1 antitrypsin is available in A1AT deficiency, although its cost effectiveness and
long term clinical effectiveness is yet to be established.
452. Risk factors for MDR TB

419
M Y Elamin
MBBS, DTM&H, MCTM, MRCPI 1& 2
453. Sarcoid is common in subjects of Caribbean origin and is associated with better prognosis.
454. Sarcoidosis is a multisystem disorder although it predominately affects the respiratory system.
Radiological staging is based on chest x ray appearance.
455. Serum precipitating antibodies to Saccharopolyspora rectivirgula is the most useful diagnosic test for
farmer's lung.
456. Sleep apnoea is an independent risk factor for stroke.
457. Sleep apnoea is commonly associated with obesity, but myxoedema, acromegaly and retro/micrognathia
are also recognised causes.
458. Small cell lung cancer typically produces ectopic ACTH.
459. Smoking cessation is a vital element of management of many respiratory conditions and there has been
much research into the different techniques.
460. Sometimes patients require stenting of the SVCO for symptomatic relief prior to more definitive
treatment.
461. Specifically asking about the timing and location of occurrence of symptoms can help in raising a
suspicion of occupational asthma
462. Spirometry before and after exercise is the most appropriate investigation to diagnose exercise-induced
asthma.
463. Spores of Saccharopolyspora rectivirgula found in moldy hay/straw are responsible for farmer's lung.
464. Staphylococcal pneumonia is known to cause cavitation and occurs with increased frequency after
influenza.
465. Strongyloides is associated with a larval stage, the pulmonary migration of which can lead to symptoms
of bronchospasm with severe wheeze and shortness of breath.
466. Subjects acclimatised to life at high altitudes often have increased pulmonary artery pressure in an effort
to oxygenate more blood.
467. Sulphasalazine pulmonary toxicicity typically presents with new-onset dyspnoea and pulmonary
infiltrates visible on chest radiograph. There may be peripheral blood eosinophilia.
468. Surfactant is produced by type 2 pneumocytes and is responsible for the ability of the air-filled alveoli to
expand without collapse.
469. Surgical resection of a lung tumour may be expected to have increased success with limitation of disease
spread.
470. SVCO can result in hoarseness, due to recurrent laryngeal nerve palsy, Horner's syndrome, facial and
periorbital oedema and distended chest wall veins. Treatment is dependent on the underlying cause.
471. Systemic embolisation occurs in 20-50% of cases of infective endocarditis and can involve the lungs,
central nervous system , coronary arteries, spleen, bowel and extremities.

420
M Y Elamin
MBBS, DTM&H, MCTM, MRCPI 1& 2
472. Systemic sclerosis is a chronic autoimmune disease characterised by increased fibroblast activity and
fibrosis in a number of different organ systems. 90-95% of patients have positive antinuclear antibodies.
473. TB is a notifable disease and as such public health need to be notified and will screen contacts.
474. Tetracyclines should not be given to children under the age of 12 or to pregnant or breastfeeding
women.
475. The blood gas demonstrates respiratory alkalosis with type I respiratory failure. This question requires
an understanding of the physiological acid balance, and the causes of disturbances to it.
476. The blood gas demonstrates respiratory alkalosis. This question requires an understanding of the
physiological acid balance, and the causes of disturbances to it.
477. The combination of haemoptysis (with radiological findings consistent with pulmonary haemorrhage)
and red cell casts in the urine points to a diagnosis of Goodpastures syndrome
478. The commonest cause of type 2 respiratory failure is chronic obstructive airway disease.
479. The commonest causes of upper lobe pulmonary fibrosis are silicosis, sarcoidosis, coal worker
pneumoconiosis, ankylosing spondylitis and radiation.
480. The CURB criteria are based on the presence of urea >7
481. The CURB score is calculated by assessment of core adverse prognostic features which are used in
assessment of severity of pneumonia, one of which is blood urea.
482. The CURB score is calculated by assessment of core adverse prognostic features which are used in
assessment of severity of pneumonia.
483. The CURB-65 score can be used to help guide a patient's treatment in cases of community-acquired
pneumonia.
484. The diagnosis of Churg-Strauss syndrome is supporting by the finding of extravascular eosinophils on
vascular biopsy.
485. The diagnosis of phrenic nerve palsy is suspected when on the chest radiograph the diaphragmatic
leaflet is elevated and is confirmed fluoroscopically by observing paradoxical diaphragmatic motion on
sniff and cough.
486. The diagnosis of sleep apnoea syndrome is established by polygraphic recording of sleep showing at
least 30 of 10 or more seconds' duration in seven hours of sleep of apnoea associated with a fall in arterial
oxygen saturation.
487. The indictations for LTOT are paO2 < 7.3 kPa (55 mmHg) or paO2 < 8.0 kPa (60 mmHg) with evidence
of pulmonary hypertension, peripheral oedema or polycythaemia.
488. The latest guidelines now suggest 30:2 as the correct ratio of chest compressions to non-intubated
ventilations
489. The Mantoux test measures the T cell-mediated immune response to TB antigen.

421
M Y Elamin
MBBS, DTM&H, MCTM, MRCPI 1& 2
490. The mechanism by which it has its effect is not fully understood, but it is thought that low magnesium
levels in bronchial smooth muscle favour bronchoconstriction.
491. The most appropriate first step for a suspected pneumothorax in a stable patient would be confirmation
with chest x ray followed by chest drain insertion.
492. The most appropriate treatment for legionellosis is clarithromycin.
493. The most common example of allergic alveolitis is farmer's lung. This is caused by dust from mouldy
hay contaminated with thermophilic Actinomyces bacteria.
494. The optimal management of patients at significant risk of airways obstruction is intubation and
ventilation.
495. The paired t test is generally used when measurements are taken from the same subject before and after
some manipulation, such as injection of a drug.
496. The pathology of chronic obstructive airways disease is centrilobular or panacinar with loss of elastic
tissue.
497. The pertussis vaccine is rarely associated with a hemiseizure-hemiplegia syndrome, probably related to
hyperpyrexia rather than direct neurological toxicity.
498. The pulmonary circulation is a low pressure, high flow, high compliance system.
499. The RCOG advise that any pregnant female presenting with shortness of breath should be reviewed by a
senior member of the obstetric team.
500. The risk of mesothelioma is not affected by smoking but smoking and asbestos exposure greatly
increase the risk of lung cancer.
501. The treatment of choice for chicken pox pneumonitis is initial IV aciclovir.
502. The typical problem associated with sleep apnoea syndrome is excessive daytime somnolence.
503. The use of extracorporeal membrane oxygenation (ECMO) in patients with ARDS who are hypoxic on
maximal ventilatory therapy has been shown to promote a significant increase in survival without
significant disability.
504. There is a 50% chance that a child will be homozygous for CF and a 50% chance that the child will be a
carrier if one parent is a carrier and the other has the disease.
505. There is a role for fluids in PE in those patients with rapidly falling blood pressure but this should
preferably be done (if there is time) with central pressure monitoring.
506. There is no definitive consensus on antibiotic choice for Legionella pneumophilia, but a fluoroquinolone
or newer macrolide is usually used.
507. This gentleman is receiving adequate ipratropium and oxygen, and repeating/increasing these are
unlikely to help the situation markedly. Failure to respond to these treatment steps warrants the use of
intravenous magnesium sulphate

422
M Y Elamin
MBBS, DTM&H, MCTM, MRCPI 1& 2
508. This lady has a severe pneumonia on a background of immunosuppression secondary to her renal
transplant. As a result she is at risk from atypical and opportunistic organisms.
509. This patient is immunocompromised and given the clinical history and radiological findings, the likely
diagnosis in this lady is PCP (Pneumocystis pneumonia). She is also pregnant which affects treatment
choice.
510. This question addresses the diffuse parenchymal (interstitial) lung diseases (DPLD) and in partial
focuses on idiopathic interstitial pneumonitis (IIP).
511. This question addresses the management of type II respiratory failure in the context of an acute
exacerbation of COPD (AECOPD).
512. This question demands an understanding of the pathophysiology of atopic asthma.
513. This question requires an understanding of A1AT deficiency and the genetic variations of this condition.
514. This question requires an understanding of A1AT deficiency and the genetic variations of this condition.
515. This question requires an understanding of cystic fibrosis and the conditions associated with it.
516. This question requires an understanding of cystic fibrosis and the conditions associated with it.
517. This question requires an understanding of pulmonary vasculitis and the diagnostic antibodies involved.
518. This question requires an understanding of the investigations used to diagnose COPD, and the
measurements used to predict severity.
519. This question requires an understanding of the normal mean pulmonary arterial pressure and the
parameters at which PH may be diagnosed.
520. This question requires an understanding of the pathophysiology of respiratory failure.
521. This question requires an understanding on the TMN staging classification of NSCLC (non-small cell
lung cancer).
522. This question requires knowledge of both staging for lung cancer and the treatment modalities available.
523. Thrombolytic therapy is indicated for pulmonary embolism when circulation is severely compromised,
and is as effective delivered peripherally.
524. Transbronchial biopsy is the definitive route to confirming a diagnosis of sarcoidosis.
525. Treatment for community acquired pneumonia should be empirical until and organism is identified.
526. Treatment of community-acquired pneumonia is guided by the CURB-65 severity score.
527. Treatment of MDR-TB is complex and time consuming. Treatment must be continued for a minimum of
18 months, with at least 9 months of treatment after the patient becomes culture-negative.
528. Treatment with anti-TNF-alpha increases the risk of opportunistic infections and in particular, there is a
significant increase in the risk of TB reactivation in conjunction with infliximab.
529. Tuberculosis is a direct pulmonary cause of ARDS
423
M Y Elamin
MBBS, DTM&H, MCTM, MRCPI 1& 2
530. Under NICE guidelines a hoarse voice for three weeks or more is an indication for investigation to
exclude malignancy.
531. Up to 99% of children with cystic fibrosis have sweat chloride and sodium levels above 70 and 60 mM
respectively.
532. Varenicline is a non-nicotine drug that is a partial agonist of the alpha4beta2 nicotinic receptor
533. Varenicline is recommended as a possible treatment to help smokers who have said they want to stop
smoking
534. Wheeze, stridor and urticaria is more in keeping with allergy than MSG syndrome following Chinese
takeaway
535. A clicking noise on chest auscultation that is synchronised with the heart beat is a sign of pneumothorax.
536. A high-PEEP (open-lung) ventilator strategy will reduce atelectotrauma.
537. A raised PaCO2 in acute asthma is an important sign that the patient is deteriorating and intubation may
be required.
538. A urea raised more than 7 mmol/l in the context of normal renal function is a poor prognostic sign in
pneumonia.
539. ABG interpretation
540. Achalasia can cause megaoesophagus which can be seen on CXr as a wide mediastinum with absent
gastric bubble.
541. Acute onset shortness of breath without signs of infection should prompt investigation for PE.
542. Amphotericin is used to treat patients with Aspergillus fumigatus.
543. An understanding of the clinical features of the pathogens that cause respiratory illness is very
important.
544. Anaerobic and gram negative organisms are common in abscesses following aspiration. An unconscious
patient is at risk of aspiration.
545. Anaphylactic shock requires urgent treatment with adrenaline, which is absorbed at a faster rate
intramuscularly than subcutaneously.
546. ARDS is a recognised complication of burns and presents with bilateral haziness on CXR.
547. ARDS is a severe lung syndrome characterised by acute onset of bilateral infiltrates and severe
hypoxaemia in the absence of cardiac failure.
548. Aspergillus is a cause of a haemoptysis.
549. Assessing a patient’s performance status is important when evaluating the most appropriate treatment
options. It is commonly used by cancer MDTs, but has a role in assessing patients with chronic illnesses
including COPD.

424
M Y Elamin
MBBS, DTM&H, MCTM, MRCPI 1& 2
550. Asthma is potentially an acute life threatening event which requires careful assessment, treatment and
monitoring.
551. Breathless patient, greater than 50 years and pnx >2 cm, for chest drain.
552. Bronchial carcinoid tumours usually arise from the large bronchi and secrete serotonin.
553. Bronchiectasis associated with cystic fibrosis frequently results in recurrent infections
with Pseudomonas.
554. Bronchoalveolar cell carcinoma classically presents with progressive breathlessness and the production
of large amounts of sputum and account for up to 1% of all bronchial carcinomas.
555. BTS guidelines recommend that all patients with acute exacerbations of asthma receive steroids in
appropriate doses.
556. BTS guidelines recommend the use of amoxicillin and clarithromycin as the first line antibiotic regime
in the treatment of moderate severity CAP.
557. BTS has issued guidelines on the management of acute exacerbations of asthma.
558. Carbon monoxide poisoning will present with normal oxygen saturation on pulse oximetry.
559. Carbon monoxide poisoning will produce normal oxygen saturation on pulse oximetry.
560. CF is caused by mutations in a single large gene on chromosome 7 that encodes the cystic fibrosis
transmembrane conductance regulator (CFTR) protein
561. Clinical features and findings in tuberculosis infection
562. CO is an odorlous, colourless and non-irritating gas, it commonly presents in the winter period with
symptoms of altered mental status, treatment is with hyperbaric oxygen.
563. Common variable immunodeficiency (CVID) is the most prevalent of the primary immunodeficiency
diseases and patients are deficient in both IgG and IgA.
564. Consider Legionella infection with exposure to air conditioning systems, respiratory tract infections,
SIADH and deranged liver function tests.
565. COPD produce an obstructive picture on spirometry.
566. COPD will not show any improvement on bronchodilator medication and there will be progressive
decline of lung function.
567. Co-trimoxazole is the treatment of first choice for Pneumocystis jirovecii pneumonia. In patients with a
pO2 <60 mmHg, oral prednisolone is added to reduce the inflammatory effect.
568. Criteria for long term oxygen therapy in COPD patients
569. CT chest is the first investigation following CXR in hilar lymphadenopathy as it can assist in biopsy
planning.

425
M Y Elamin
MBBS, DTM&H, MCTM, MRCPI 1& 2
570. Ct guided biopsy is the diagnostic investigation of choice when appropriate and safe in cases of
suspected lung malignancy.
571. CT guided or thorascopic biopsy of pleural plaques has a sensitivity of 90-95%.
572. CURB-65 scoring is a quick method of assessing a patient's disease severity and using this to initiate
appropriate management:
573. Dermatomyositis is very rarely seen in primary care but can be associated with lung, ovary and GI
malignancies.
574. Diagnosis of legionella pneumonia should be made via clinical history, x-ray and urinary antigen.
575. early recognition and intubation of acute epiglottitis is essential management
576. Emphysema produces an obstructive picture on spirometry with a reduced FEV1.
577. Epilepsy and alcohol and drug abuse are risk factors for aspiration pneumonia.
578. Erythema nodosum is characterised by the presence of tender subcutaneous nodules, usually on the shins
579. Erythema nodosum is commonly idiopathic, but there should be a high index of suspicion for
sarcoidosis.
580. Erythromycin decreases theophylline clearance.
581. Exogenous PTH secretion and clubbing is associate with squamous cell carcinoma of the lung.
582. Extra-pulmonary restriction produces a characteristic pattern where Kco is greater than normal with a
normal or slight reduction in Tlco
583. First choice treatment for Mycoplasma pneumoniae is with macrolide antibiotic (clarithromycin or
erythromycin); alternatives include tetracycline or doxycycline.
584. First line investigation for a diagnosis of asthma is peak flow charts and diaries.
585. First line treatment for Legionella pneumonia is with macrolides or quinolones.
586. For obesity to cause a reduced total lung capacity (TLS) the weight (kg) / height (cm) must generally
exceed 1.
587. GCS less than 8 is not compatible with the ability to safely protect ones airway.
588. Granulomatosis with polyangiitis can be diagnosed via a positive c-ANCA, although biopsies are often
also required.
589. Haemolysis is associated with the presence of IgM antibodies (cold agglutinins) directed against the I
antigen of the erythrocyte membrane.
590. haemolytic anaemia is associated with atypical pneumonia
591. Hamman's sign is a clicking or crunching sound in time with the heart beat audible on the left chest, in
the context of pneumothorax or pneumomediastinum.

426
M Y Elamin
MBBS, DTM&H, MCTM, MRCPI 1& 2
592. Hereditary haemorrhagic telangectasia is usually an autosomal dominant condition that may present with
anaemia due to bleeding from occult GI telangectasia
593. History of a recent TIA, along with the clinical and ECG findings, are consistent with an atrial septal
defect.
594. HIV-positive patients are more prone to herpes zoster regardless of their CD4 count.
595. Hypercalcaemia and elevated parathyroid hormone related peptide levels are most commonly associated
with squamous cell carcinomas.
596. Hypercapnia and signs of fatigue are indications for immediate intubation and ventilation in acute
asthma.
597. Hypersensitivity pneumonitis (HP) is an inflammatory reaction to inhaled organic dusts.
598. Hypersensitivity pneumonitis may be acute, subacute or chronic.
599. Hypoadrenalism is associated with hyponatreamia, hyperkalaemia, elevated TSH and mild
hypercalcaemia
600. Hyponatreamia with urine sodium <30, raised BUN and hypotension is likely multifactorial and should
be treated with Normal Saline
601. If pleural fluid protein is 25-35 g/L then Light's criteria are more accurate in determining whether the
effusion is an exudate or transudate
602. ILD presents with a restrictive pattern of spirometry with reduced diffusion capacity.
603. illicit drug use can cause pulmonary hypertension
604. In membranous GN forty per cent remit without treatment, 30% develop endstage renal failure (ESRF).
605. In primary pneumothorax if the patient is not breathless and the rim of air is less than 2 cm they can be
considered for early discharge and repeat chest x ray in two weeks.
606. Initial treatment of PE is with low molecular weight heparin befor initiating warfarin or a DOAC.
607. Interpreting blood gases is essential in the management of acutely unwell patients.
608. IPF shows a restrictive pattern on spirometry with both Tlco and Kco reduced.
609. Ireland is among the countries with the highest reported prevalence of sarcoidosis worldwide. Biopsy of
granuloma is diagnostic for the disease.
610. Knowledge of neurological syndrome resulting from lung carcinoma
611. Legionnaire's disease may be assocaited with hyponatreamia secondary to SIADH
612. Lung fibrosis presents with a restrictive lung function test, together with reduced TLCO and KCO
613. Management of (submassive) pulmonary embolus
614. management of pneumothorax
427
M Y Elamin
MBBS, DTM&H, MCTM, MRCPI 1& 2
615. Marfan's syndrome is a risk factor for spontaneous pneumothorax.
616. Massive volumes of clear frothy sputum is produced by patients with bronchioalveolar cell carcinoma.
617. Moderate airways obstruction with norma/high transfer factor or transfer co-efficient is suggestive of
asthma.
618. Monotherapy in a case of aspiration pneumonia is insufficient. Co Amoxiclav covers aerobes, whilst
metronidazole targets anaerobes
619. Mucous plugging is not uncommon in asthmatic patients and can occasionally cause lobar collapse and
should be treated with mucolytics and chest physio
620. Mycoplamsa Pneumonia can present with haemolytic anaemia
621. Needle thoracostomy (NT) is the most rapid method of achieving lifesaving access to the pleural space if
a tension pneumothorax is rapidly progressing.
622. Non-caseating granulomas are seen in berylliosis and sarcoidosis.
623. Occupational asthma can be diagnosed using a peak flow diary.
624. OSA is diagnosed by overnight oximetry showing repetitive apnoeas and desaturations.
625. Oseltamivir is the oral treatment of choice for H1N1 infection.
626. Oxygen titration in COPD based on arterial blood gas interpretation
627. Pancoast tumours arise in the apex of the lung and infiltrates locally leading to symptoms such as
Horner's syndrome.
628. Panic attacks may present with deranged ABG result, including respiratory alkalosis.
629. Patients with type 2 respiratory failure with signs of acidosis require treatment with NIV.
630. PCP is an opportunistic infection which affects patients with HIV with significantly low CD4 counts.
631. PCP pneumonia characteristically causes oxygen desaturation during exercise.
632. People with significant smoking history associated with a persistent cough and weight loss should
prompt investigation for malignancy.
633. Pleural aspiration is only recommended in secondary pneumothoraces if the pneumothorax is less than 2
cm, patient has minimal breathlessness and the patient is less than 50-years-old.
634. Pleural Plaques are usually benign and associated with asbestos exposure
635. Pleural plaques suggest previous exposure to asbestos.
636. Pleural thickening produces a restrictive picture on spirometry with a raised Kco.
637. Pneumothorax management depends on whether it is primary or secondary and the size and degree of
respiratory compromise.

428
M Y Elamin
MBBS, DTM&H, MCTM, MRCPI 1& 2
638. Post-extubation stridor occurs frequently in extubated patients and may be associated with respiratory
failure due to airway obstruction requiring reintubation.
639. Presentation of an aspergilloma may be similar to many other chronic respiratory conditions, but key
points include haemoptysis, weight loss, sweating and characteristic CXR findings.
640. Prevention of cross infection is much more likely in patients who are smear positive in TB..
641. Primary ciliary dyskinesia is associated in 50% of cases with dextrocardia and situs inversus when it is
called Kartagener's syndrome.
642. Primary pneumothoraces over 4cm are treated by needle aspiration. Second intercostal space,
midclavicular line.
643. Prognosis in pneumonia based on clinical features (CURB65 score)
644. Pseudomonas aeruginosa is characterised by green coloration of the colonies.
645. Pseudomonas infection secondary to bronchiectasis is common in CF and should be treated with
Ceftazadime.
646. Psittacosis is an atypical pneumonia contracted from birds and is treated with tetracyclines.
647. Restrictive lung defects on spirometry with pleural plaques and clubbing is likely to be caused by
asbestos exposure.
648. Salient features of pneumocystis jirovecii pneumonia (PCP) are several days/weeks of increasing
dyspnoea, dry cough, marked oxygen desaturation with exercise.
649. Sarcoidosis is a disease of unknown cause characterised by the presence histologically of non-caseating
granulomata.
650. Severe Granulomatosis with polyangiitis, or in patients with primarily renal involvement needing
dialysis, plasmapheresis is used to rapidly remove the immune complexes.
651. Should a patient develop a pneumothorax on NIV they should continue NIV while a drain is inserted.
652. Silicosis is a fibrotic lung disease associated with the inhalation of silicon dioxide (silica) and leads to
eggshell calcification on a chest x Ray.
653. Smokers presenting with persistent cough and clubbing should prompt investigation for malignancy.
654. Specialist referral should be made if montelukast tablets fail to control asthma in patients.
655. Specialist referral should be made if montelukast tablets fail to control asthma.
656. Stage 3b tumours are not amenable to surgery
657. Symptomatic spontaneous pneumothoraces of more than 2 cm size should not be treated conservatively.
658. The combination of Horner's syndrome and the patient history should prompt further investigation for
apical lung tumour
659. The diagnosis is a mesothelioma
429
M Y Elamin
MBBS, DTM&H, MCTM, MRCPI 1& 2
660. The diagnosis of ABPA should be confirmed by using eitherserum IgE levels of aspergillus, skin prick
test for aspergillus or a radioallergosorbent test with lung function tests showing an obstructive pattern.
661. The diagnosis of cystic fibrosis can be made by performing a sweat test. A sweat concentration of
greater than 60 mmol/L is diagnostic.
662. The first line test for correct placement of a nasogastric tube is a pH check of gastric aspirate using pH
indicator paper. There is no place for the 'whoosh' test.
663. The Henderson-Hasselbalch equation can be useful in determining lab error on blood gasses.
664. The lower lobes are the usual site of aspiration when the patient is upright and foreign bodies should be
removed via bronchoscopy.
665. The metabolism of corticosteroids is increased by rifampicin and should therefore lead to increased
steroid doses in patients on them long term.
666. The most appropriate point for chest drain insertion for a pneumothorax is in the "safe triangle" in the
mid-axillary line.
667. The most common cause of SVCO is primary lung cancer but biopsy should be performed to exclude
other causes.
668. The side effects of amiodarone are well recognised, and include pneumonitis and pulmonary fibrosis.
669. The usual causative organism of acute epiglottitis is Haemophilus influenzae type b.
670. The Wells score is the most commonly used method to predict probability of pulmonary embolism.
671. This lady is immunocompromised and has the classical features associated with aspergillosis.
672. This man has a severe pneumonia with a history highly suggestive of underlying immunosuppression
from HIV. As a result he is at risk from atypical and opportunistic organisms
especially Pneumocystis pneumonia.
673. This patient is immunocompromised as a result of his homelessness and alcohol abuse; as a result he is
at risk from tuberculosis.
674. This question looks at the histological diagnosis of respiratory illnesses in immunocompromised hosts
675. This question requires an understanding of the CURB-65 scoring system for community-acquired
pneumonia.
676. This question requires an understanding of the different pulmonary function tests and how they relate to
respiratory pathology.
677. This question requires interpretation of arterial gas measurements and spirometry.
678. Thrombocytosis is associated with increased mortality compared to thrombocytopaenia or normal
platelet levels in pneumonia.
679. Thrombolysis is only indicated in massive PE or PE with signs of shock

430
M Y Elamin
MBBS, DTM&H, MCTM, MRCPI 1& 2
680. TNM staging is used in the management of lung cancer.
681. Transbronchial lung biopsy will provide positive histology in about 80% of patients and provided it can
be done safely is the gold standard for diagnosis of sarcoidosis.
682. Treatment of patients with OSA include weight loss, reducing alcohol intake and CPAP
683. Type 2 respiratory failure with severe acidosis (PH< 7.25) should have ITU review with scope to
intubate and ventilate.
684. Unilateral pleural calcification can be seen in TB or post haemothorax.
685. Unilateral pleural effusion in the context of pleural thickening should raise the possibility of
mesothelioma.
686. Unintentional weight loss may be a sing of underlying malignancy.
687. Varicella pneumonia occurs in up to 20% of adults with chickenpox, appearing three to five days into
the course of the illness. In adults with pneumonitis, treatment with aciclovir is warranted.
688. VATS biopsy is the diagnostic tool of choice as it provides the highest diagnostic yield In respect to
mesothelioma.
689. VATS procedure for pneumothorax
690. Venous thromboembolism is a relatively common complication after major pelvic/orthopaedic surgery
691. When treating severe Guillain-Barré syndrome it is necessary to measure the forced vital capacity using
spirometry to monitor for signs of respiratory muscle deterioration.
692. Where there is a tension pneumothorax neither oxygen alone nor needle aspiration are the definitive
treatment and a chest tube must be inserted. The definitive treatment is the one that will lead to resolution
of the pathology and a needle aspiration (or thoracocentesis) will not be big enough to allow the tension
pneumothorax to resolve.
693. Yellow nail syndrome is caused by hypoplastic lymphatics and is characterised by the triad of
lymphoedema, pleural effusions and yellow discolouration of the nails.
694. Yellow nail syndrome is caused by hypoplastic lymphatics and is characterised by the triad of
lymphoedema, pleural effusions, and yellow discolouration of the nail

431
M Y Elamin
MBBS, DTM&H, MCTM, MRCPI 1& 2
RHEUMATOLOGY
1. 20-year-old man is investigated for recurrent fractures following minor trauma. Calcium and phosphate levels
are normal. A skeletal survery shows sclerosis at the base of the skull – osteopetrosis.
2. A 25-year-old man develops a painful and swollen knee associated with dysuria and conjunctivitis - reactive
arthritis.
3. A 25-year-old man is admitted following a shoulder dislocation. He has a history of joint hypermobility and is
noted to have elastic, 'stretchy' skin on examination with multiple bruises - Ehler-Danlos syndrome.
4. A 25-year-old man is admitted to hospital following a pneumothorax. On examination he is noted to be tall with
arachnodactyly and pectus excavatum - Marfan's syndrome.
5. A 25-year-old woman presents with joint pains in her hands. On examination she has erythema and swelling
around the 2nd and 3rd metacarpophalangeal joints - rheumatoid arthritis
6. A 30-year-old Turkish man presents with recurrent attacks of oral and genital ulceration. In the past he has
been treated for anterior uveitis and thombophlebitis - Behcet's syndrome
7. A 35-year-old woman complains of a constant dry mouth and sore, dry eyes. She also describes painful hands
and feet - Sjogren's syndrome
8. A 40-year-old presents with severe pain and swelling over his right 1st metatarsophalangeal joint - gout
9. A 4-year-old boy fractures his femur after a minor fall. On examination he is noted to have blue sclera. He has
recently been referred to an audiologist for hearing tests - osteogenesis imperfecta
10. A 55-year-old man who has had fever and malaise for the past few weeks develops a common peroneal nerve
palsy. Bloods show the presence of hepatitis B surface antigen - polyarteritis nodosa
11. A 60-year-old man presents with an acutely painful and swollen knee. Joint aspiration demonstrates weakly-
positively birefringent rhomboid shaped crystals - pseudogout
12. A 65-year-old woman presents with aching and morning stiffness in the proximal limb muscles. She is having
difficulty getting out of a chair - polymyalgia rheumatica
13. A 6-year-old boy presents with progressive hip pain for the past few weeks associated with a limp - Perthes
disease
14. A 70-year-old man presents with a persistent headache associated with jaw pain whilst chewing. He also
complains of lethargy and generalised myalgia - temporal arteritis
15. A boy with a history of learning difficulties and self-mutilation presents with recurrent episodes of gout - Lesch-
Nyhan syndrome
16. A female immigrant from the Indian subcontinent presents with 'bone pain', muscle weakness and anorexia.
Bloods show a decreased calcium and phosphate level - osteomalacia
17. A middle-aged woman with a history of Raynaud's phenomenon complains of dysphagia. She is noted to have
multiple telangectasia on examination - systemic sclerosis
18. A sporty teenage boy presents with pain, tenderness and swelling over the tibial tubercle - Osgood-Schlatter
disease
432
M Y Elamin
MBBS, DTM&H, MCTM, MRCPI 1& 2
19. A sporty teenager presents with knee pain after exercise associated with intermittent swelling and locking -
osteochondritis dissecans
20. A teenage girl presents with anterior knee pain which is worse when going down stairs and rising from
prolonged sitting - chondromalacia patellae
21. A teenage girl presents with medial knee pain following activity. The knee has given way on occasion - patellar
subluxation
22. An elderly man is investigated for 'bone pains'. He is known to be deaf. Bloods show a raised ALP and a skull
x-ray shows a thickened vault - Paget's disease of the bone
23. An elderly woman fractures her neck of femur after tripping over - osteoporosis
24. An obese 12-year-old boy with pain in the hip and distal thigh. On examination there is loss of internal rotation
- slipped upper femoral epiphysis
25. Apremilast - phosphodiesterase type IV inhibitor
26. Azathioprine - metabolised to the active compound mercaptopurine, a purine analogue that inhibits purine
synthesis
27. Azathioprine may cause myelosuppression/agranulocytosis
28. Azathioprine may cause pancreatitis
29. Azathioprine may cause toxicity in patients with thiopurine methyltransferase deficiency
30. Bamboo spine - ankylosing spondylitis
31. Behcet's syndrome - anterior uveitis
32. Behcet's syndrome - genital ulcers
33. Behcet's syndrome - oral ulcers
34. Behcet's syndrome - thrombophlebitis and deep vein thrombosis
35. Bilateral inflammed 2nd + 3rd MCPs - rheumatoid arthritis
36. Bisphosphonates - pyrophosphate analog which inhibits osteoclast activity
37. Bisphosphonates may cause oesophagitis
38. Bisphosphonates may cause osteonecrosis of the jaw
39. Bone pain, bone thickening, raised ALP - Paget's disease
40. Bone pain, muscle weakness, Asian female - osteomalacia
41. Child, bone pain in skull/proximal femur, cutaneous nodules, recurrent otitis media/mastoiditis - Langerhans
cell histiocytosis
42. Chondrocalcinosis - pseudogout
43. Chronic fatigue syndrome , treatment of choice: graded exercise therapy
44. Churg-Strauss syndrome - p-ANCA
45. Colchicine - inhibits microtubule polymerization by binding to tubulin, interfering with mitosis. Also inhibits
neutrophil motility and activity
46. Denosumab - inhibits RANK ligand, which in turn inhibits the maturation of osteoclasts
433
M Y Elamin
MBBS, DTM&H, MCTM, MRCPI 1& 2
47. Dermatomyositis - anti-Mi-2 antibodies
48. Diffuse cutaneous systemic sclerosis - ANA
49. Diffuse cutaneous systemic sclerosis - anti-scl-70 antibodies
50. Discoid lupus erythematous (not responding to topical steroids) , treatment of choice: hydroxychloroquine
51. 'double/soap bubble' x-ray appearance around epiphysis of femur - giant cell tumour of bone
52. Dry eyes, dry mouth, arthralgia - Sjorgren's syndrome
53. Duchenne muscular dystrophy - calf pseudohypertrophy
54. Duchenne muscular dystrophy - child uses arms to stand up from a squatted position
55. Ehler-Danlos syndrome - abnormality in type III and V collagen
56. Etanercept - acts as a decoy receptor for TNF-alpha
57. Etanercept may cause demyelination
58. Etanercept may cause reactivation of tuberculosis
59. Hydralazine may cause drug-induced lupus
60. Hydroxychloroquine may cause retinopathy
61. Inflammed 1st MTP - gout
62. Infliximab - binds to TNF-alpha and prevents it from binding with TNF receptors
63. Infliximab may cause reactivation of tuberculosis
64. Lesch-Nyhan syndrome - hypoxanthine-guanine phosphoribosyl transferase (HGPRTase) deficiency
65. Limited cutaneous systemic sclerosis - ANA
66. Limited cutaneous systemic sclerosis - anti-centromere antibodies
67. Low serum calcium, low serum phosphate, raised ALP and raised PTH - osteomalacia
68. Low serum calcium, raised serum phosphate, raised ALP and raised PTH - chronic kidney disease leading to
secondary hyperparathyroidism
69. Male, gout, renal failure, neuropsychiatric features, self-mutilation - HGPRT deficiency
70. Marfan's syndrome - blue sclera
71. Marfan's syndrome - defect in the fibrillin-1 gene
72. Marfan's syndrome - dilation of the aortic sinuses
73. Marfan's syndrome - dural ectasia
74. Marfan's syndrome - mitral valve prolapse
75. Marfan's syndrome - upwards lens dislocation
76. Methotrexate may cause hepatotoxicity
77. Methotrexate may cause mucositis
78. Methotrexate may cause myelosuppression/agranulocytosis
79. Methotrexate may cause pneumonitis
80. Methotrexate may cause pulmonary fibrosis
81. Microscopic polyangiitis - c-ANCA
434
M Y Elamin
MBBS, DTM&H, MCTM, MRCPI 1& 2
82. Microscopic polyangiitis - p-ANCA
83. Multiple painful genital ulcers, not sexually active - Behcet's disease
84. Muscle pain and stiffness following exercise, myoglobinuria - McArdle's disease
85. Needle shaped negatively birefringent crystals - gout
86. Normal serum calcium, normal serum phosphate, normal ALP and normal PTH - osteopetrosis
87. Normal serum calcium, normal serum phosphate, normal ALP and normal PTH - osteoporosis
88. Normal serum calcium, normal serum phosphate, raised ALP and normal PTH - Paget's disease
89. Osteogenesis imperfecta - abnormality in type 1 collagen due to decreased synthesis of pro-alpha 1 or pro-alpha
2 collagen polypeptides
90. Osteogenesis imperfecta - blue sclera
91. Osteogenesis imperfecta - deafness
92. Osteomyelitis (history of sickle cell anaemia) - Salmonella
93. Osteomyelitis (no history of sickle cell anaemia) - Staphylococcus aureus
94. Osteopetrosis - defective osteoclast function resulting in failure of normal bone resorption
95. Paget's disease of the bone - excessive osteoclastic resorption followed by increased osteoblastic activity
96. Paget's disease of the bone , treatment of choice: a bisphosphonate
97. Polyarteritis nodosa - hypertension
98. Polyarteritis nodosa - positive hepatitis B serology
99. Polymyositis - anti-Jo-1 antibodies
100. Procainamide may cause drug-induced lupus
101. Pseudogout - deposition of calcium pyrophosphate dihydrate crystals
102. Raised serum calcium, low serum phosphate, raised ALP and raised PTH - primary hyperparathyroidism
103. Raloxifene - selective oestrogen receptor modulator
104. Raynaud's disease , treatment of choice: nifedipine
105. Recent STI, waxy yellow/brown papules on palms and soles - keratoderma blenorrhagica
106. Recurrent joint dislocation, easy bruising, elastic skin - Ehler-Danlos syndrome
107. Repeated fractures, false allegation of child abuse, blue sclera - osteogenesis imperfecta
108. Rheumatoid arthritis - anti-cyclic citrullinated peptide antibody
109. Sjogren's syndrome - ANA
110. Sjogren's syndrome - anti-Ro
111. Sjogren's syndrome - autoimmune destruction of the exocrine glands
112. Sjogren's syndrome - rheumatoid factor
113. Sjogren's syndrome - Schirmer's test
114. SLE – ANA
115. SLE - anti-dsDNA
116. SLE - anti-La
435
M Y Elamin
MBBS, DTM&H, MCTM, MRCPI 1& 2
117. SLE - anti-Ro
118. SLE - anti-Smith
119. STI --> arthritis, urethritis, conjunctivitis - reactive arthritis
120. Strontium ranelate - promotes differentiation of pre-osteoblasts to osteoblasts + inhibits osteoclasts
121. Tall, long fingered, aortic pathology, pneumothoraces - Marfan's syndrome
122. Temporal arteritis - an association with polymyalgia rheumatica
123. Temporal arteritis - jaw claudication
124. Weakly-positively birefringent rhomboid shaped crystals - pseudogout
125. Young female, facial rash, neuropsychiatric features, Raynaud's - systemic lupus erythematosus
126. TNF-α inhibitors may reactivate TB
127. Staphylococcus aureus is the most common cause of osteomyelitis
128. Adult-onset Still’s disease is a diagnosis of exclusion and can only be diagnosed if rheumatoid factor and
anti-nuclear antibody are negative
129. An asymmetrical presentation suggests psoriatic arthritis rather than rheumatoid
130. Ankylosing spondylitis - x-ray findings: subchondral erosions, sclerosis and squaring of lumbar vertebrae
131. Ankylosing spondylitis features - the 'a's
a. Apical fibrosis
b. Anterior uveitis
c. Aortic regurgitation
d. Achilles tendonitis
e. AV node block
f. Amyloidosis
132. Anti-CCP antibodies are a marker of poor prognosis in rheumatoid arthritis
133. Anti-cyclic citrullinated peptide antibodies are associated with rheumatoid arthritis
134. Anti-ribonuclear protein (anti-RNP) (Anti U1) = mixed connective tissue disease
135. Antiphospholipid syndrome: (paradoxically) prolonged APTT + low platelets
136. Antiphospholipid syndrome: arterial/venous thrombosis, miscarriage, livedo reticularis
137. Apremilast is a phosphodiesterase type-4 (PDE4) inhibitor which can be used to treat active psoriatic arthritis
(in combination with disease-modifying antirheumatic drugs or alone) in patients who have had an inadequate
response or who have been intolerant to a prior disease-modifying antirheumatic drug therapy.
138. As well as ear problems, relapsing polychondritis may present with nasal chondritis, respiratory tract
involvement and arthralgia
139. Azathioprine - check thiopurine methyltransferase deficiency (TPMT) before treatment
140. Azathioprine is metabolised to the active compound mercaptopurine, a purine analogue that inhibits purine
synthesis
141. Azathioprine is safe to use in pregnancy
436
M Y Elamin
MBBS, DTM&H, MCTM, MRCPI 1& 2
142. Behcet's syndrome is associated with HLA B51
143. Bisphosphonates are associated with an increased risk of atypical stress fractures
144. Bisphosphonates can cause a variety of oesophageal problems
145. Bisphosphonates inhibit osteoclasts
146. Bone pain, tenderness and proximal myopathy (→ waddling gait) → ?Osteomalacia
147. Burning thigh - ? Meralgia paraesthetica - lateral cutaneous nerve of thigh compression
148. cANCA = granulomatosis with polyangiitis; pANCA = eosinophilic granulomatosis with polyangiitis + others
149. Carpometacarpal and distal interphalangeal joint involvement is characteristic of hand osteoarthritis
150. Chondrocalcinosis helps to distinguish pseudogout from gout
151. Colchicine may be helpful in Familial Mediterranean Fever
152. Cubital tunnel syndrome is caused by compression of the ulnar nerve and can present with tingling/numbness
of the 4th and 5th finger
153. Denosumab - RANKL inhibitor
154. Dermatomyositis antibodies: ANA most common, anti-Mi-2 most specific
155. Dermatomyositis is associated with the anti-Jo-1 antibody (anti-synthetase antibodies)
156. anti-Jo-1 antibodies (anti-synthetase antibodies) are seen in pattern of Polymyositis disease associated with lung
involvement, Raynaud's and fever
157. DEXA scans: the T score is based on bone mass of young reference population
158. Diabetes mellitus is a risk factor for developing adhesive capsulitis
159. Diagnosis of ankylosing spondylitis can be best supported by sacro-ilitis on a pelvic X-ray
160. Discoid lupus erythematous - topical steroids → oral hydroxychloroquine
161. Ehlers-Danlos syndrome is most commonly associated with a defect in type III collagen
162. Epoprostenol (amongst other prostaglandins) can be used in the treatment of Raynaud's phenomenon
163. Exercise regimes and NSAIDS are the 1st line management for ankylosing spondylitis
164. External rotation (on both active and passive movement) is classically impaired in adhesive capsulitis
165. Familial Mediterranean fever (FMF) is an autosomal recessive, hereditary inflammatory disorder characterised
by reoccurring episodes of abdominal pain, fever, arthralgia, and chest pain
166. Fever/back pain with pain on extension of the hip → iliopsoas abscess
167. Golimumab is a TNF inhibitor
168. Haemochromatosis is a risk factor for pseudogout
169. Hydroxychloroquine - may result in a severe and permanent retinopathy
170. Hydroxychloroquine is the treatment of choice for SLE
171. In patients with a new diagnosis of dermatomyositis, urgent malignancy screen is needed
172. Inflammatory arthritis involving DIP swelling and dactylitis points to a diagnosis of psoriatic arthritis
173. Inflammatory back pain (e.g. Ankylosing spondylitis) typically improves with exercise
174. Isoniazid can cause drug-induced lupus
437
M Y Elamin
MBBS, DTM&H, MCTM, MRCPI 1& 2
175. Juxta-articular osteoporosis/osteopenia is an early x-ray feature of rheumatoid arthritis
176. L5 lesion features = loss of foot dorsiflexion + sensory loss dorsum of the foot + big toe
177. Langerhans cell histiocytosis is characterized by Birbeck granules on electron microscopy
178. Lateral epicondylitis: worse on resisted wrist extension/suppination whilst elbow extended
179. Leflunomide may cause hypertension
180. Limited (central) systemic sclerosis = anti-centromere antibodies
181. Marfan's syndrome - dural ectasia
182. Marfan's syndrome - upwards lens dislocation
183. Marfan's syndrome is associated with dilation of the aortic sinuses which may predispose to aortic dissection
184. Marfan's syndrome is caused by a mutation in a protein called fibrillin-1
185. May be able to see apical fibrosis on chest x-ray in later ankylosing spondylitis
186. Methotrexate works by inhibiting dihydrofolate reductase
187. Mycophenolate mofetil inhibits of inosine-5'-monophosphate dehydrogenase which is needed for purine
synthesis
188. NICE recommend co-prescribing a PPI with NSAIDS in all patients with osteoarthritis
189. NICE guidelines support starting bisphosphonates without waiting for a DEXA scan in such scenarios
190. Nifedipine is a pharmacological option for Raynaud's phenomenon
191. Offer allopurinol to all patients after their first attack of gout
192. Oral ulcers + genital ulcers + anterior uveitis = Behcet's
193. Osteoarthritis - paracetamol + topical nsaids (if knee/hand) first-line
194. Osteogenesis imperfecta occurs due to an abnormality in type 1 collagen
195. Osteomalacia
a. Low: calcium, phosphate
b. Raised: alkaline phosphatase
196. Osteomyelitis: MRI is the imaging modality of choice
197. Osteoporosis in a man - check testosterone
198. Paget's disease - increased serum and urine levels of hydroxyproline
199. Paget's disease - old man, bone pain, raised ALP
200. Paget's disease of the bone is treated with bisphosphonates
201. Pain on the radial side of the wrist/tenderness over the radial styloid process? De Quervain's tenosynovitis
202. Patients who are allergic to aspirin may also react to sulfasalazine
203. Patients with osteopetrosis have normal calcium, phosphate, ALP and PTH levels
204. Patients with Sjogren's syndrome have an increased risk of lymphoid malignancies
205. Polyarteritis nodosa can cause a mononeuritis multiplex syndrome
206. Previous chemotherapy is a significant risk factor for avascular necrosis
207. Pseudogout - weakly positively birefringent rhomboid-shaped crystals
438
M Y Elamin
MBBS, DTM&H, MCTM, MRCPI 1& 2
208. Pseudoxanthoma elasticum is associated with mitral valve prolapse and increased risk of ischaemic heart
disease
209. Radial tunnel syndrome presents similarly to lateral epicondylitis however pain is typically distal to the
epicondyle and worse on elbow extension/forearm pronation
210. Raloxifene may actually decrease the risk of breast cancer
211. Raynaud's disease (i.e. Primary) presents in young women with bilateral symptoms
212. Reactive arthritis: develops after an infection where the organism cannot be recovered from the joint
213. Repeated cramping and myoglobinuria after short bouts of exercise can point towards mcardle's disease
214. Rheumatoid arthritis - TNF is key in pathophysiology
215. Rheumatoid arthritis: patients have an increased risk of IHD
216. Rheumatoid factor is an IgM antibody against IgG
217. Rickets can present as widening of the wrist joints due to an excess of non-mineralized osteoid at the growth
plate
218. S1 lesion features = Sensory loss of posterolateral aspect of leg and lateral aspect of foot (small toe), weakness
in plantar flexion of foot, reduced ankle reflex, positive sciatic nerve stretch test
219. Septic arthritis - most common organism: Staphylococcus aureus
220. Septic arthritis: IV flucloxacillin
221. SLE - antibodies associated with congenital heart block = anti-Ro
222. SLE: ANA is 99% sensitive - anti-Sm & anti-DsDNA are 99% specific
223. SLE: complement levels are usually low during active disease - may be used to monitor flares
224. Spinal stenosis is the most likely diagnosis in a patient with gradual onset leg and back pain, weakness and
numbness which is brought on by walking (with a normal clinical examination)
225. Sulfasalazine is a disease-modifying anti-rheumatic drug which is safe in both pregnancy and breastfeeding
226. Syndesmophytes (ossification of outer fibres of annulus fibrosus) are a feature of ankylosing spondylitis
227. Synovial fluid sampling is the key investigation in patients with suspected septic arthritis
228. The concurrent use of methotrexate and trimethoprim containing antibiotics may cause bone marrow
suppression and severe or fatal pancytopaenia
229. The major target for PANCA is myeloperoxidase (MPO)
230. The most common cause of death in systemic sclerosis is respiratory involvement: interstitial lung disease and
pulmonary arterial hypertension
231. The vast majority of gout is due to decreased renal excretion of uric acid
232. Thiazide diuretics can precipitate an attack of gout
233. Trochanteric bursitis presents with isolated lateral hip/thigh pain with tenderness over the greater trochanter
234. Urethritis + arthritis + conjunctivitis = reactive arthritis
235. Aortic stenosis occurs frequently in the population suffering proximal femoral fractures.
236. In women with oopthrectomy unopposed oestrogen HRT is appropriate.
439
M Y Elamin
MBBS, DTM&H, MCTM, MRCPI 1& 2
237. Characteristic deformity and symptoms seen in active Paget's disease of bone
238. Septic arthritis needs to be ruled out in cases of an acute, hot, swollen, painful joint.
239. Positive ANA is present in nearly all cases of SLE
240. Infiltrative lymphocytic syndrome (DILS) can present in many ways and occurs in patients who are HIV
positive.
241. SLE is a cause of splenomegaly and should be considered when there is arthralgia and inflammatory change
242. Biopsy in Inclusion Body Myositis (IBM) shows intranuclear or cytoplasmic tubofilaments on electron
microscopy.
243. Behçet's syndrome is a systemic vasculitis with an unknown aetiology, which affects small and large vessels
(venous and arterial)
244. Ankylosing spondylitis is associated with positive HLA B27
245. Anticentromere antibodies are strongly associated with CREST syndrome.
246. Aortic stenosis occurs frequently in the population suffering proximal femoral fractures.
247. Arterial and venous thromboses are well recongised in Behcets disease
248. Causes of calcium pyrophosphate deposition disease (CPPD) “pseudogout” at young age. usually have some
underlying risk factor, such as: haemochromatosis, hyperparathyroidism, low magnesium, low phosphate
acromegaly, Wilson's disease
249. Characteristic deformity and symptoms seen in active Paget's disease of bone; 5% of patients are symptomatic
isolated raised ALP, bone pain (e.g. pelvis, lumbar spine, femur) classical, untreated features of bowing of tibia,
bossing of skull.
250. Erythema nodosum lesions tend to last six to eight weeks.
251. Escalating treatment for pain is paracetamol, NSAIDs,
252. High output cardiac failure can be a complication of Pagets disease
253. Infiltrative lymphocytic syndrome (DILS) can present in many ways and occurs in patients who are HIV
positive.
254. Joint swelling is bony in nature in osteoarthritis.
255. Patients with HAO are at risk of developing SLE, due to low C4.
256. Peri articular erosion is seen in RA
257. Positive ANA is present in nearly all cases of SLE
258. Risk of venous thromboembolism in those with anti-phospholipid antibody syndrome.
259. Septic arthritis needs to be ruled out in cases of an acute, hot, swollen, painful joint.
260. Side effects of hydroxychloroquine optic neuritis (Bulls eye retina)
261. Systemic sclerosis – renal complications.
262. "Intention to treat" helps reduce bias by keeping sunjects in treatment allocaiton groups even if they do
not recieve the treatment.

440
M Y Elamin
MBBS, DTM&H, MCTM, MRCPI 1& 2
263. 80% of patients with primary Sjögren's syndrome have hypergammaglobulinaemia, with ANA and
anti-Ro antibodies commonly seen.
264. A combination of lower back pain, stiffness and reduced spinal mobility makes ankylosing spondylitis
the most likely diagnosis..
265. A combination of thrombophilia, recurrent miscarriage, thrombocytopenia and leucopenia is sugestive
of a diagnosis of antiphospholipid syndrome.
266. A diagnosis of adult onset Still's disease is mainly one of exclusion.
267. A fibrillar appearance under electron micrography is a pathological feature of amyloidosis.
268. A history of recent surgery and diuretic use can point to gout in cases presenting with acute
monoarthritis.
269. Acute CPP crystal synovitis (pseudogout) diagnosis
270. Acute sarcoidosis usually resolves spontaneously over six to eight weeks.
271. Adult onset Still's disease (AOSD) IS characterised by high spiking fever, evanescent salmon pink
rash, polyarthralgia, lymphadenopathy, hepatosplenomegaly and neutrophilic leucocytosis. There is
often an accompanying sore throat and myalgia.
272. An acute attack of gout can follow the initiation of antihyperuricaemic (allopurinol) therapy.
273. An allergy to co-trimoxazole would be a contraindication to the use of sulphasalazine.
274. Antibodies commonly associated with APS are anticardiolipin antibodies.
275. Anti-CCP antibodies are highly specific and sensitive for rheumatoid arthritis and their titre correlates
with erosive disease.
276. Anticentromere antibodies and antitopoisomerase I antibodies are the classic autoantibodies associated
with systemic sclerosis.
277. Anticentromere antibodies and antitopoisomerase I antibodies are the classic autoantibodies associated
with systemic sclerosis.
278. Anticyclic citrullinated protein (CCP) antibodies are the most specific biomarkers associated with the
diagnosis of rheumatoid arthritis.
279. Anti-Ro antibodies can result in neonatal lupus, with heart block, in babies born to mothers with SLE.
280. Anti-Ro antibody is associated with congenital complete heart block (CHB) accounting for the vast
majority of cases of CHB.
281. Approach to a red hot joint.
282. Around 20% of patients with polymyositis have anti-Jo 1 antibodies.

441
M Y Elamin
MBBS, DTM&H, MCTM, MRCPI 1& 2
283. Arthralgia and arthritis are the most common presenting manifestations of SLE typically affecting the
small joints of the hands, wrists, and knees. The symmetrical rash is the classical butterfly rash that
occurs in a malar distribution. Alopecia is common and may be diffuse or patchy.
284. Arthritis associated with inflammatory bowel disease IBD.
285. Articular erosions in rheumatoid arthritis occuring early on in the course of the disease especially
within the first six months of presentation indicate a poor prognosis. Over time joint damage will relate
to disability.
286. As soon as a giant cell arteritis (GCA) diagnosis is suspected, high dose corticosteroids should be
given to preserve vision.
287. At least 38 drugs currently in use can cause DILE. However, most cases have been associated with
procainamide (15-20%), hydralazine (7-13%), or quinidine.
288. Avascular necrosis of the femoral head can occur as a consequence of acute lymphoblastic leukaemia,
or the treatment for the disease.
289. Azathioprine can be continued in pregnancy.
290. Azathioprine interacts with Allopurinol and toxicity can cause pancytopaenia
291. Behçet's syndrome is a multisystem disorder characterised by recurrent oral and genital ulceration, eye
lesions, and skin lesions.
292. Behçet's syndrome is a systemic vasculitis with an unknown aetiology, which affects small and large
vessels (venous and arterial); venous thrombosis is a common complication.
293. Bisphosphonates are generally considered first line treatment for the prevention of steroid induced
osteoporosis.
294. Bisphosphonates are indicated in those <65 who take steroids if they are osteoporotic or have a
fragility fracture.
295. Bisphosphonates work by inhibiting osteoclast activity, thus increasing mineralisation.
296. Bone pains with elevated alkaline phosphatase and normal calcium concentrations suggest a diagnosis
of Paget's disease.
297. Bronchiolitis obliterans, with or without organising pneumonia, can be a fatal complication of
rheumatoid arthritis (especially in women taking penicillamine).
298. Bronchoalveolar lavage is the best choice for diagnosing Pneumocystis jiroveci (formerly
Pneumocystis carinii) pneumonia
299. Carpal tunnel syndrome is one of the most common entrapment neuropathies, and is a recognised
occupational disease. Intermittent or sustained high pressure on the carpal tunnel produces ischaemia
of the median nerve, resulting in the classical symptoms or paraesthesia and pain.
300. Celecoxib has a lower level of anti-platelet activity than naproxen.

442
M Y Elamin
MBBS, DTM&H, MCTM, MRCPI 1& 2
301. Classically, osteoporosis in the absence of fracture, does not cause pain. Many patients with
osteoporosis have concomitant disorders such as osteomalacia and osteoarthritis which cause bone
pain.
302. Clinicians are recommended to check FBC fortnightly until 6 weeks after the last dose increase.
Provided it is stable, it can be checked monthly thereafter until the dose and disease is stable for one
year.
303. Complications of Paget’s disease.
304. Complications of pregnancy in Sjogren’s syndrome.
305. Consider drug-induced lupus where there is lupus-like symptomatology in a patient where a new
medication has been started.
306. Consider granulomatosis with polyangiitis in a patient with alveolar haemorrhage and epistaxis.
307. Constrictive pericarditis is the commonest cardiac complication of rheumatoid arthritis, and presents
with dyspnoea, atrial fibrillation, lower limb oedema, ascites, raised JVP, and biatrial enlargement with
normal systolic ventricular function.
308. Current United Kingdom guidance suggests that during the first three months of treatment with
sulfasalazine, full blood count (FBC) should be monitored monthly for the first three months.
309. Cushing's syndrome with hypercortisolism can promote osteoporosis.
310. Cytoplasmic (cANCA): a coarse, clumpy, granular cytoplasmic staining of neturophils, which is
associated with anti-proteinase-3 (PR3) antibodies in 85% of cases of Granulomatosis with polyangiitis
311. De Quervain’s tenosynovitis results in thickening of the synovial sheaths surrounding extensor pollicis
brevis and abductor polices longus, leading to pain and reduced movement.
312. De Quervain's tenosynovitis is a cause of dorsoradial wrist pain
313. Degenerative changes affecting the intervertebral discs, vertebrae, facet joints, and ligamentous
structures encroach on the cervical spinal canal and damage the cord, especially in patients with a
congenitally small canal.
314. Destructive septic arthritis must be excluded in patients that present with acute onset monoarthritis.
315. Diagnosis of Behçet’s disease.
316. Diagnosis of complications of hepatitis C.
317. Diagnosis of SAPHO syndrome
318. Diagnosis of sarcoidosis
319. Differential diagnoses of haemarthrosis.
320. Differential diagnosis of ANCA associated vasculitis
321. Differential diagnosis of Charcot joint.
443
M Y Elamin
MBBS, DTM&H, MCTM, MRCPI 1& 2
322. Differential diagnosis of coagulopathy
323. Differential diagnosis of haematuria.
324. Differential diagnosis of knee OA.
325. Differential diagnosis of low back pain.
326. Differential diagnosis of polymyalgia rheumatica
327. Differential diagnosis of psoriatic arthritis
328. Differentiate OA from other arthropathies.
329. Differentiate: Reactive arthritise, Behcet’s disease, arthropathy in inflammatory bowel disease,
Whipple’s disease and sarcoidosis
330. Discoid lupus erythematosus is characterised by discrete plaque lesions, often erythematous, covered
by scales that extend into dilated hair follicles. These lesions most typically occur on the face, scalp, in
the pinnae, behind the ears, and on the neck. They can exist in areas not exposed to the sun.
331. EGPA is a rare form of small-vessel vasculitis, characterised by asthma, allergic rhinitis and prominent
peripheral blood eosinophilia.
332. Erythema nodosum is commonly idiopathic, but there should be a high index of suspicion for
sarcoidosis.
333. Evidence of osteoarthritis, calcification of the articular cartilage and no growth on synovial culture
would be a typical presentation of pseudogout.
334. Exogenous oestrogen administration, HLA-DR3 and B8 and congenital deficiency of C2 are thought to
predispose to the development of C2, whereas breastfeeding is thought to be protective.
335. Features of drug induced lupus, list of drugs causing lupus, auto antibody association in
rheumatological conditions.
336. Fibromyalgia is becoming a recognised medical diagnosis, and is based on the presence of pain in all
four quadrants of the body, as well as tenderness in 11 of 18 anatomically defined trigger areas.
337. First line treatment in scleroderma is a gradual reduction in blood pressure (10-15 mmHg per day) with
an ACE inhibitor until the diastolic pressure reaches 85-90 mmHg.
338. For asymptomatic patients with isolated slightly elevated urate is lifestyle advice with an appropriately
reduced purine diet.
339. Functional limitation joint crepitus on active motion, morning stiffness lasting less than 30 minutes,
and an age over 45 years point towards patellofemoral osteoarthritis.
340. Gastrointestinal (GI), rectal and subcutaneous fat biopsies are the procedures of choice in diagnosing
systemic amyloidosis.
341. Gonorrhoea is the second most common bacterial STI in the UK after chlamydia with primary
infection is symptomatic in 90-95% of men, but only 50% of women.
444
M Y Elamin
MBBS, DTM&H, MCTM, MRCPI 1& 2
342. Gout diagnosis
343. Gout treatment.
344. Guidance recommends the use of disease modifying anti-rheumatic drugs (DMARDs), such as
methotrexate early in the treatment of rheumatoid arthritis.
345. Haemachromatosis can cause articular calcinosis
346. HAO is characterised by deficiency of C1 esterase inhibitor and leads to low C4 levels.
347. HSP is more commonly seen in infants, but may also be seen in older individuals.
348. IBD-associated arthropathy is considered a subtype of seronegative spondyloarthropathy with a variety
of joint involvement.
349. Identify nerve root affected in cervical radiculopathy.
350. If aspartate aminotransferase (AST) or alanine aminotransferase (ALT) is between two and three times
the upper limit of normal, and the leflunomide dose is more than 10 mg daily, the dose should be
reduced to 10 mg and LFTs rechecked weekly until normalised.
351. In cases of suspected sarcoidosis, a chest x ray may show bilateral hilar lymphadenopathy to guide
towards a diagnosis.
352. In patients with rheumatoid arthritis, chronic NSAID use can lead to complications such as iron
deficiency anaemia and the superficial ulceration on endoscopy.
353. In Raynaud's phenomenon, episodes lasting in excess of one hour suggest an underlying connective
tissue disease.
354. In Raynaud's, conservative management before embarking on potentially long term medication with
risks of side effects is important.
355. In rheumatoid arthritis, the presenting features may not fully satisfy the ACR criteria, but this is not
unusual in clinical practice.
356. In the circumstances of infection one should consider temporarily stopping methotrexate as it is an
immunosuppressant.
357. Inclusion body myositis (IBM) is an inflammatory condition that affects the over 50s. Muscle biopsy
with electron microscopy is most likely to give a definite diagnosis.
358. Infection in RA.
359. Infliximab is given with methotrexate and is associated with the development of tuberculosis. Other
side effects include injection site reactions, infusion reactions, neutropenia, demyelinating disease and
heart failure.
360. Infliximab is recommended by NICE for adults with rheumatoid arthritis who have active rheumatoid
arthritis, and who have undergone trials of two disease-modifying anti-rheumatic drugs including
methotrexate (unless contraindicated).

445
M Y Elamin
MBBS, DTM&H, MCTM, MRCPI 1& 2
361. Interpretations of anti-nuclear antibody result
362. Intra-articular corticosteroids is a good initial treatment for patients with gout, where NSAIDs are
contraindicated.
363. Investigation of low back pain.
364. Iritis is associated with conditions such as Reiter's, Behcet's, psoriatic arthropathy (about 20%) and
inflammatory bowel disease.
365. Jaw claudication increases the likliehood of a positive temporal artery biopsy.
366. Joint aspiration is essential in cases of suspected septic arthritis, in order to establish a microbiological
diagnosis that will guide appropriate treatment.
367. Joint aspiration is the best option in cases of suspected destructive septic arthritis.
368. Joint aspiration is the mopst appropriate investigation to differentiate monoarthritis from destructive
septic arthritis, gout and pseudogout.
369. Lateral epicondylitis occurs due to inflammation of the extensor forearm muscle origins, and causes
lateral elbow and upper forearm pain and tenderness.
370. Lymphoma and other lymphoproliferative disorders can have cold agglutinin disease as a
paraneoplastic phenomenon.
371. Macroglobulins are a central feature of Waldenstrom's macroglobulinaemia, where proliferation of
lymphocytes cause an excess of IgM.
372. Management of giant cell arteritis
373. Management of psoriatic arthritis.
374. Marked consumption of C4 together with a strongly positive rheumatoid factor strongly suggests
cryoglobulinaemia.
375. Methotrexate is a chemotherapeutic agent; it acts through inhibition of dihydrofolate reductase thus
depleting folate concentrations.
376. Methotrexate is a recognised cause of pulmonary fibrosis. However, it is sometimes used in the
treatment of idiopathic pulmonary fibrosis as a steroid sparing agent.
377. Methotrexate is a well recognised cause of acute pneumonitis and interstitial lung disease.
378. Methotrexate is teratogenic and, according to the British National Formulary (BNF), the manufacturers
advise effective contraception during, and for at least three months, after stopping methotrexate (both
males and females).
379. Methotrexate is teratogenic and, according to the British National Formulary (BNF), the manufacturers
advise effective contraception during, and for at least three months, after stopping methotrexate.
380. Methotrexate may be associated with haematopoietic suppression

446
M Y Elamin
MBBS, DTM&H, MCTM, MRCPI 1& 2
381. Minocycline can account for polyarthritis and antinuclear antibody (ANA), due to its ability to cause
drug-induced lupus erythematosus.
382. Minocycline has been well documented as a cause of drug-induced SLE.
383. Monoarticular Paget’s often affects the hip, and is associated with raised alkaline phosphatase in the
presence of normocalcaemia.
384. Monoclonal IgM causes hyperviscosity syndrome seen in Waldenström's macroglobulinaemia and
other IgM clonal disorders.
385. Mononeuritis multiplex, fever, hypertension, and nephritic renal involvement which is most consistent
with a diagnosis of polyarteritis nodosa.
386. Most GIT manifestations result from dysmotility secondary to infiltration of the intestinal wall with
fibrous tissue, and can cause life-threatening malabsorption and malnutrition.
387. Nailfold capillaroscopy can indicate an underlying connective tissue disorder.
388. Neisseria gonorrhoeae typically occurs in young adults. Presentation is either as a bacteraemic form
(classic triad of migratory polyarthritis, tenosynovitis, and dermatitis) which is usually polyarticular or
a septic arthritis.
389. Neonatal lupus is an uncommon condition associated with the transplacental passage of maternal anti-
Ro and/or anti-La autoantibodies.
390. NICE guidance recommends denosumab, a monoclonal antibody targeted against the nuclear factor-
kappa ligand (RANKL) involved in osteoclast activation, when bisphosphonates are contraindicated.
391. NICE guidelines recommend formulating individualised management plans for patients with
osteoarthritis.
392. Non-steroidal anti-inflammatory drugs (NSAIDs) may cause a deterioration in renal function and
would be associated with an increased risk of bleeding in the elderly.
393. Oesteoarthritis can be associated with Heberden's and Bouchard's nodes, patellofemoral joint
dysfunction and lower back pain.
394. Osteoarthritis is characterised by joint pain, crepitus, stiffness after mobility, and limitation of motion,
commonly affecting the knees, hips and small joints of the hand. The CMC joint is classically
involved, and involved in gripping and twisting. Joint swelling is bony in nature, unlike the boggy
swelling which occurs in inflammatory arthritis. Thenar wasting occurs in OA of the first CMC joint
due to disuse.
395. Osteoarthritis is one of the most common joint diseases, and its incidence is increasing with the age
and weight of the population.
396. Osteomalacia results in hypocalcaemia and hypophosphataemia and is treated with vitamin D
supplementation.

447
M Y Elamin
MBBS, DTM&H, MCTM, MRCPI 1& 2
397. Osteomyelitis represents an ongoing infection that produces marked bone deformity, not just joint
narrowing.
398. Osteoporosis prophylaxis with corticosteroids.
399. Paget's disease of bone is a localised disorder of bone remodelling. There are increased numbers of
giant osteoclasts, which increase bone resorption with subsequent increase in new bone formation and
altered bone architecture.
400. Pain on resisted extension of the wrist is suggestive of lateral epicondylitis (Tennis elbow)
401. Papilloedema without visual loss would suggest raised intracranial pressure, not giant cell arteritis
(GCA).
402. Paracetamol and/or topical NSAIDs (for knee or hand OA) should be offered before considering oral
NSAIDs. Rubefacients, intra-articular hyaluronan, electro-acupuncture and chondroitin and
glucosamine products are not recommended.
403. Paracetamol is the initial therapy recommended for the treatment of OA of the hip and knee. Studies
have shown that the short term and long term efficacy of paracetamol is comparable with that of
ibuprofen and naproxen in people with knee osteoarthritis.
404. Patients often present with ankylosing spondylitis (AS) in their 20s and 30s with a history of chronic
back pain and stiffness which improves with exercise.
405. Patients should be assessed for TB prior to commencing on anti-TNF alpha treatment, and
chemoprophylaxis given to those in whom latent TB is suspected.
406. Patients with rheumatoid arthritis often present with synovitis which may affect a variety of joints
before developing the classic features of a symmetrical, inflammatory polyarthritis affecting the small
joints of the hands and feet.
407. Patients with Sjogren's syndrome are at a higher risk of developing lymphoma (non-Hodgkin's
lymphoma [NHL] B cell), and should be monitored for this.
408. Polyarteritis nodosa (PAN) is a necrotising vasculitis affecting small to medium sized muscular
arteries.
409. Polymyalgia rheumatica is an inflammatory disease which typically presents with pain and stiffness of
the the shoulder and pelvic girdle muscles (classically symmetrical).
410. Polymyalgia rheumatica is an inflammatory disease which typically presents with pain and stiffness of
the the shoulder and pelvic girdle muscles. It can have either an acute or subacute onset and is
associated with a systemic inflammatory response and therefore constitutional symptoms such as fever,
anorexia, weight loss and malaise.
411. Polymyalgia rheumatica typically presents with pain and stiffness of the the shoulder and pelvic girdle
muscles with fever, anorexia, weight loss and malaise.
412. Polymyositis classically presents with relatively painless progressive proximal muscle weakness.

448
M Y Elamin
MBBS, DTM&H, MCTM, MRCPI 1& 2
413. Psoriatic arthritis has been subclassified according to different patterns of arthritis. Symmetrical
polyarthritis ('rheumatoid pattern') - affects wrists, hands, feet and ankles. The distal interphalangeal
joints are more commonly affected than the metacarpophalangeal joints, which helps to distinguish it
from rheumatoid arthritis
414. Pulmonary fibrosis is associated with anti-Scl-70 antibodies in up to 70% of cases.
415. Pulmonary hypertension is a typical late complication of systemic sclerosis.
416. Reactive arthritis is the triad of conjunctivitis, urethritis and arthritis which occurs one to three months
after an initiating infection.
417. Renal cysts are usually found on ultrasound and CT. Tissue may be needed to differentiate between
malignant and benign cysts, but this is obtained via aspiration rather than renal parenchymal biopsy.
418. Salmonella osteomyelitis is seen in patients with sickle cell anaemia.
419. Sarcoidosis is a multisystem disorder, and often presents with chest symptoms in combination with
erythema nodosum, palpable lymphadenopathy and hypercalcaemia.
420. Sarcoidosis is a multisystem granulomatous disorder. It is relatively uncommon in the United Kingdom
as a whole, but is more prevalent and more severe in Afro-Caribbean populations.
421. Septic arthritis is a potential catastrophic complication of intra-articular steroid injection, although it is
rare.
422. Septic arthritis must be excluded for all patients presenting with an acute painful monoarthritis.
423. Serum urate concentrations exceeding 7 mg/dl are associated with increased risk of gout.
424. Severe osteoporosis: hip BMD 2.5 SD or more below the young adult reference mean in the presence
of one or more fragility fractures (T-score =-2.5 PLUS fracture).
425. Simple bisphosphonates inhibit bone resorption through their actions on osteoclasts.
426. Six monthly denosumab is effective in managing patients with severe osteoporosis intolerant or with
contraindicaitons to Alendronate or Zolendronic acid.
427. Spinal/neurological involvement in RA.
428. Studies have shown that paracetamol 1 g combined with codeine at dose of 60 mg has the best
analgesic outcomes.
429. Systemic sclerosis, mixed connective tissue diseases
430. Takayasu’s arteritis presents most commonly in young women of Japanese origin, with symptoms of
proximal arterial disease.
431. The combination of back pain, weight loss and osteosclerotic lesions makes prostatic adenocarcinoma
the most likely diagnosis in elderly male patients.
432. The commonest subtype HLA associations with Ankylosing Spondylitis are HLA B*2705
(Caucasians), B*2704 (Chinese, Japanese) and B*2702 (Mediterranean)
449
M Y Elamin
MBBS, DTM&H, MCTM, MRCPI 1& 2
433. The current recommendation for methotrexate monitoring is that FBC, U&Es and LFTs are checked at
baseline and then two-weekly until the dose and monitoring is stable for 6 weeks. These should then be
checked monthly until the dose and disease is stable for a year.
434. The first step in long term management of patients with rheumatoid arthritis is DMARD therapy
(disease modifying anti-rheumatic drugs).
435. The Koebner phenomenon refers to the appearance of lesions along the site of an injury or scar. It can
occur in psoriasis, molluscum contagiosum, lichen planus and other conditions.
436. The L5 nerve root supplies sensation to the lateral aspect of leg and dorsum of foot (except for the
lateral border which is supplied by S1). In addition, it supplies hip extensors, knee flexors (with S1),
ankle dorsiflexors (with L4), and toe dorsiflexors.
437. The majority of cases of congenital heart block are due to the presence of anti-Ro/SSA antibodies in
the maternal serum.
438. The majority of research evidence is for cognitive behavioural therapy (CBT) and/or graded exercise
therapy and these should be offered to all people with mild or moderate CFS.
439. The mechanism of action of bisphosphonates involves the inhibition of osteoclasts
440. The mechanism of action of farnesyl diphosphate synthase within osteoclasts. In doing this they
interfere with geranylgeranylation (attachment of the lipid to regulatory proteins), which causes
osteoclast inactivation.
441. The occurrence of even a single thrombotic event in a patient with antiphospholipid syndrome warrants
lifelong anticoagulation, as the risk of recurrence is 20-70%.
442. The patient has multiple myeloma. The bone marrow needle was in a lytic lesion filled with plasma
cells.
443. The presence of anti-Sm antibodies is the most SPECIFIC for SLE than the other options.
444. The rheumatoid factor test utilises the patient's serum, to agglutinate cells coated with antibody.
Rheumatoid factor (RF) is an antibody whose specificity is directed to a domain situated within the Fc
portion of IgG. The rheumatoid factor may be of IgM, IgG or IgA class.
445. The serum levels of anti-dsDNA antibodies appear to correlate with disease activity in many patients
and often levels will rise just before a flare of disease.
446. The triad of fever, arthralgia and rash in a woman of childbearing age should suggest the diagnosis of
systemic lupus erythematosus (SLE).
447. The typical presenting features of haemochromatosis are diabetes, bronzing of the skin, hepatomegaly
and arthropathy.
448. There are five patterns of psoriatic arthritis, which is typically predated by the classical rash by a
number of years.

450
M Y Elamin
MBBS, DTM&H, MCTM, MRCPI 1& 2
449. There is an association between myasthenia gravis and thyroid disease, pernicious anaemia, systemic
lupus erythematosus and rheumatoid arthritis.
450. TNF-alpha is a pro-inflammatory cytokine.
451. Tocilizumab is a humanised anti-IL 6 receptor monoclonal antibody. It is licensed for the treatment of
moderate to severe rheumatoid arthritis (RA) which has responded inadequately to DMARDs or TNF
antagonists.
452. Treatment of ankylosing spondylitis (AS).
453. Treatment of knee OA.
454. Treatment of Paget’s disease.
455. Type II (mixed essential) cryoglobulinaemia is closely associated with persistent hepatic infection
(transaminitis, inferred IVDU) most commonly hepatitis C and to a lesser extent hepatitis B.
456. Under NICE guidelines a hoarse voice for three weeks or more is an indication for investigation to
exclude malignancy.
457. Vasculitic rash over shins, thighs and buttocks together with lower limb joint pains and renal
involvement are suggestive of Henoch-Schönlein purpura (HSP).
458. Weight loss is a recognised feature of polymyalgia rheumatica.
459. Whilst the first carpometacarpal joint can be affected in rheumatoid arthritis and psoriatic arthritis it is
rarely in isolation, whereas this is a frequent site of osteoarthritis in post menopausal women.
460. Zolendronic acid has been linked to the development of osteonecrosis of the jaw, with a statistically
significant association to dental extraction surgery as a precipitant.
461.
462.
463. 80% of patients with primary Sjögren's syndrome have hypergammaglobulinaemia, with ANA and
anti-Ro antibodies commonly seen.
464. a sign of 'mechanics hand' is a sign of a subtype of polymyositis called antisynthetase syndrome or Jo-1
syndrome.
465. Allopurinol hypersensitivity
466. Angioid Streaks are breaks in Bruch's membrane. They may be associated with pseudoxanthoma
elasticum, Ehlers-Danlos syndrome, sickle cell disease, acromegaly (and other pituitary disorders), and
Paget's disease.
467. Ankylosing spondylitis is associated with positive HLA B27
468. Anticentromere antibodies are strongly associated with CREST syndrome.

451
M Y Elamin
MBBS, DTM&H, MCTM, MRCPI 1& 2
469. Anti-neutrophil cytoplasmic antibodies (ANCA) should always be done in suspected systemic
vasculitis.
470. Anti-TNF a agent increases the risk of tuberculosis. This risk is most pronounced in the first three
months of treatment.
471. Aortic stenosis occurs frequently in the population suffering proximal femoral fractures.
472. Approach to septic arthritis + septic shock
473. Arterial and venous thromboses are well recongised in Behcets disease
474. Behçet's syndrome is a systemic vasculitis with an unknown aetiology, which affects small and large
vessels (venous and arterial).
475. Biologics mechanisms of action
476. Biopsy in IBM shows intranuclear or cytoplasmic tubofilaments on electron microscopy.
477. Bisphosphonates are first line use for prevention of osteoporosis in post-menopausal women on
steroids.
478. Causes of calcium pyrophosphate deposition disease (CPPD) at young age
479. Characteristic deformity and symptoms seen in active Paget's disease of bone
480. Chronic tophaceous gout has the features of asymmetrical joint swellings, tophi and progressive
cartilage and bone destruction. This condition can occur after several years of persistent gout.
481. Clinical features of Behcet's disease include the classical triad of: recurrent oral ulceration, recurrent
genital ulceration, and iritis.
482. Clinical presentation and radiological features of osteomyelitis
483. Complications of AS
484. Consider drug-induced lupus erythematosus in a patient 'newly started on a drug' with arthritis, a
photosensitive rash, serositis and myalgia.
485. Cutaneous lupus erythematosus
486. Diagnosis of cutaneous manifestation of sarcoidosis
487. Diagnosis of respiratory manifestation of sarcoidosis
488. Differential diagnosis of Charcot joint
489. Differential diagnosis of coagulopathy
490. Differential diagnosis of myositis
491. Differential diagnosis of polymyositis
492. Differential diagnosis of SLE nephritis

452
M Y Elamin
MBBS, DTM&H, MCTM, MRCPI 1& 2
493. Differentiate causes of acute back pain
494. Drug induced lupus
495. Early fluid resuscitation is the most important measure in the prevention of acute kidney injury
secondary to rhabdomyolysis. Large volume depletion occurs due to sequestration of water by injured
muscle. Most studies target urine outputs of 3 ml per kilogram per hour or >300 ml per hour.
496. Early menopause is a risk factor for osteoporotic bone fractures.
497. Erythema nodosum is commonly idiopathic, but there should be a high index of suspicion for
sarcoidosis.
498. Erythema nodosum lesions tend to last six to eight weeks.
499. Escalating treatment for pain is paracetamol, NSAIDs, opiates.
500. Eye involvement in RA
501. Familial Mediterranean fever
502. Febuxostat mechanism of action
503. Fracture of the midshaft of the humerus can cause a radial nerve palsy giving rise to wrist drop
504. Gout classically affects the metatarsal bones with punched out extra-articular lesions on x-ray.
505. Gout typically presents with pain, joint swelling, erythema and tenderness. Needle-shaped crystals
which are negatively birefringent are demonstrated on joint aspiration.
506. High output cardiac failure can be a complication of Pagets disease
507. Hyperuricaemia management
508. If pleural fluid protein is 25-35 g/L then Light's criteria are more accurate in determining whether the
effusion is an exudate or transudate
509. In ankylosing spondylitis the inflammatory pain is typically worse in the morning and gets better with
activity
510. In women with oopthrectomy unopposed oestrogen HRT is appropriate.
511. Infiltrative lymphocytic syndrome (DILS) can present in many ways and occurs in patients who are
HIV positive.
512. It is important to screen for pulmonary TB prior to commencing Infliximab and other monoclonal
antibodies, as they may reactivate latent TB
513. IV methylprednisolone may be commenced prior to renal biopsy if clinically necessary in suspected
vasculitis.
514. Jaccoud's arthropathy may be seen in SLE, RA, Parkinson's disease and hypocompementaemic
urticarial vasculitis

453
M Y Elamin
MBBS, DTM&H, MCTM, MRCPI 1& 2
515. Joint swelling is bony in nature in osteoarthritis in comparison to boggy swelling which occurs in
inflammatory arthritis.
516. Long term sequel of cyclophosphamide
517. Lyme disease transmission
518. Marked consumption of C4 with a strongly positive rheumatoid factor is suggestive of
cryoglobulinaemia.
519. Methotrexate is associated with hepatotoxicty, which requires immediate drug discontinuation.
520. Methotrexate pneumonitis is an idiosyncratic acute interstitial pneumonitis. It has an incidence of 3.3%
in RA. Most patients present in the first few months of starting methotrexate.
521. Most patients with granulomatosis with polyangiitis develop cANCAs. Proteinase-3 is the major
cANCA antigen.
522. Necrobiosis lipoidica is commonly associated with diabetes. It is a painless lesion and secondary to
collagen degeneration.
523. Neutropenia in RA.
524. Parvovirus B19 is caused by HHV 6 and can result in a small joint arthropathy.
525. Patients with HAO are at risk of developing SLE.
526. Peri articular erosion is seen in RA
527. Pharmacokinetics of azathioprine
528. Polymyalgia rheumatica/temporal arteritis may be associated with: predominantly polymyalgia
symptoms, for example, proximal muscle pain and stiffness, or arteritis symptoms, for example,
headaches, scalp tenderness and jaw claudication and systemic features, for example, fever, malaise
and weight loss.
529. Positive ANA is present in nearly all cases of SLE
530. Proximal muscle stiffness but not weakness in an elderly patient is suggestive of polymyalgia
rheumatica.
531. Pseudoxanthoma Elasticum presents with a 'plucked chicken skin' appearance of skin folds in the neck
532. Psoriatic arthritis can follow a number of patterns, and should be suspected where there is a personal or
close family history of psoriasis.
533. Pyoderma gangrenosum presents as an expanding ulcer with a polycyclic or serpiginous outline and a
characteristic undermined bluish edge which is responsive to oral steroids.
534. Rapid development of severe pain, swelling, and tenderness that reaches its maximum within 6-12
hours with overlying oedema is characteristic of crystal arthropathy.

454
M Y Elamin
MBBS, DTM&H, MCTM, MRCPI 1& 2
535. Reactive arthritisis classically a clinical triad of urethritis, conjunctivitis and seronegative arthritis,
mainly affecting the large weight-bearing joints (usually knee and ankle). However, not all three
symptoms are always seen in a patient.
536. Reduced DLCO is an early indication of pulmonary disease in systemic sclerosis.
537. Relapsing polychondritis (RP) is a severe, episodic, and progressive inflammatory condition involving
cartilaginous structures, predominantly those of the ears and nose.
538. Risk of venous thromboembolism in those with anti-phospholipid antibody syndrome.
539. Sarcoidosis is a disease of unknown cause characterised by the presence histologically of non-
caseating granulomata.
540. Screening for malignancy in myositis
541. Septic arthritis needs to be ruled out in cases of an acute, hot, swollen, painful joint.
542. Serious side effects of cyclosporine include hypertension, and renal impairment. Difficult to control
hypertension and an increase in creatinine by more than 30% from baseline are indications for stopping
cyclosporine treatment.
543. Shoulder pain differential diagnosis
544. Side effects of hydroxychloroquine
545. SLE is a cause of splenomegaly and should be considered when there is arthralgia and inflammatory
change
546. Soft tissue rheumatism around the wrist
547. Spondyloarthritis is a group of related but phenotypically distinct disorders including ankylosing
spondylitis.
548. Steroids and cyclophosphamide both increase the risk of P. jirovecii pneumonia
549. Symptoms of Behçet's syndrome include fever, central nervous system symptoms, and seronegative
arthritis.
550. Systemic involvement is common in polymyalgia rheumatica/temporal arteritis, for example, fever,
malaise and weight loss. Typically the Erythrocyte sedimentation rate (ESR) is very high.
551. Systemic sclerosis – PAH
552. Systemic sclerosis – renal complications
553. The British Society of Rheumatology guidelines state that the plasma urate should be maintained
below 300 µmol/L in the management of gout.
554. The combination of bone fractures, hypocalcaemia, hypophosphataemia, elevated alkaline phosphatase
and parathyroid hormone levels points to a diagnosis of osteomalacia, of which vitamin D deficiency is
a major cause in the elderly population.

455
M Y Elamin
MBBS, DTM&H, MCTM, MRCPI 1& 2
555. The evanescent rash, often called a 'salmon pink' maculopapular rash is quite a distinct feature of adult
onset Still’s diseas
556. The majority of cases of congenital heart block are due to the presence of anti-Ro/SSA antibodies in
the maternal serum.
557. The upper respiratory tract involvement suggested by the nosebleeds makes the diagnosis of
Granulomatosis with polyangiitis likely.
558. Thyrotoxicosis is associated with high alkaline phosphatase secondary to increased bone turnover
559. To consider steroid sparing therapy where reductions in corticosteroid have resulted in repeated
relapses of PMR symptoms
560. Treatment of knee OA
561. Typical features of RA include symmetrical joint involvement and morning stiffness which improves
with exercise, and involvement of MCPs and PIPs of hands and feet.
562. Typical features of systemic sclerosis include facial telangiectasia, small mouth, and beaking of the
nose.
563. Whilst the first carpometacarpal joint can be affected in rheumatoid arthritis and psoriatic arthritis it is
rarely in isolation, whereas this is a frequent site of osteoarthritis in post menopausal women
564. A

456
M Y Elamin
MBBS, DTM&H, MCTM, MRCPI 1& 2
EMERGENCY
1. The management of tachyarrhythmias is dependent on the presence of adverse features. When adverse
features are present, synchronised DCCV under GA or conscious sedation is the treatment of choice. These
include:
a. Myocardial ischaemia
b. Shock
c. Syncope, and
d. Heart failure.
2. This patient has a tachyarrhythmia (probable atrial fibrillation [AF]) with signs of heart failure therefore the
most appropriate management is DCCV.
3. If adverse features are not present, it is appropriate to try drugs first. For AF (complicated by heart failure)
digoxin or amiodarone are indicated.
4. Baclofen toxicity leads to respiratory depression and diaphragm contraction which requires intubation and
ventilation. Treatment is usually supportive and often requires intensive care.
5. His CO2 retention is likely to be due to central nervous system depression and reduction in diaphragmatic
contraction secondary to baclofen toxicity. Onset of toxicity is rapid and its effect can last up to 35-40 hours post
ingestion. Features include: Drowsiness, Coma, Respiratory depression, Hyporeflexia, Hypotonia, Hypothermia,
and Hypotension. Bradycardia with first degree heart block and prolongation of Q-T interval can occur.
6. NIPPV would be an option but not in a patient with a GCS 8/15. Patients with a high risk of aspiration pneumonia
are a contraindication to non-invasive ventilation.
7. Ventricular pre-excitation commonly masquerades as other conditions, such as bundle branch block or
ischaemia. The ECG shows a shortened PR interval (in this case the PR interval is approximately 0.08 s - two
small squares NR 0.12-0.2 s) and a delta wave. In this case it is probably best seen in the lateral chest leads V3 -
V6.
8. Ventricular pre-excitation (if there were a history of tachycardia it would be Wolff-Parkinson-White syndrome)
commonly masquerades as other conditions, such as bundle branch block or ischaemia.
9. Intracranial haemorrhage can cause changes in the ECG which are typically deep symmetrical T-wave
inversion and prolonged QT interval.
10. Acute pulmonary embolism is unlikely, as there are none of the transient features that one would expect to see
(S1Q3T3 pattern, RV strain and tachycardia).
11. Syndrome of inappropriate antidiuretic hormone (SIADH) as suggested by the hyponatraemia with high urine
sodium and osmolality.
12. Drugs that may cause SIADH include: Selective serotonin reuptake inhibitors (ssris) (fluoxetine) Tricyclic
antidepressants, Sulphonylureas, Thiazides, and Carbamazepine.
13. Other causes of SIADH include: Pneumonias, Meningitis, Guillain-Barré, Trauma, and Malignancy.
14. Lithium would cause diabetes insipidus (DI).
457
M Y Elamin
MBBS, DTM&H, MCTM, MRCPI 1& 2
15. In patients who have taken an overdose of paracetamol, INR is the best indicator of the degree of hepatic
impairment.
16. Paracetamol overdose is a common question. The essentials of management are:
a. Check paracetamol level four hours after ingestion, check levels against the Rumack-Matthew
nomogram.
b. Gastric lavage if large dose ingested (more than 7.5 g) and/or presenting within 24 hours of ingestion;
consider oral charcoal.
c. Give N-acetylcysteine or methionine.
d. Hourly bms monitored.
e. Check INR 12 hourly.
17. Signs associated with poor prognosis (and indicating need for transfer to a liver unit) include:
a. INR greater than 2.0 within 48 hours or greater than 3.5 within 72 hours of ingestion
b. Creatinine greater than 200 µmol/L
c. Blood ph less than 7.3 Signs of encephalopathy
d. Hypotension (SBP less than 80 mmhg).
e. Liver enzymes are a poor marker of the degree of hepatocellular damage; synthetic function (as
determined by INR or PT) is the best indicator.
18. Erythem ab igne is due to fire/hot-water bottle use and may be secondary to symptomatic hypothyroidism It
frequently occurs on the front of the shins or lower back, the latter especially associated with the use of a hot water
bottle.
19. Meningococcal septicaemia presents with a rapidly progressive, non blanching rash, purpuric in nature.
Treatment is with cephalosporin. Explanation The skin shows a purpuric rash of meningococcal septicaemia.
However, administration of intravenous antibiotics is the greatest priority, regardless of whether cultures have
been sent.
20. Myalgia, myositis and myopathy are all recognised effects of HMG-coa reductase inhibitors (statins). There is an
increased incidence when statins are co-administered with a fibrate or when given to patients on
immunosuppressants.
21. Rhabdomyolysis may ensue often associated with acute renal failure secondary to myoglobinuria.
22. Inclusion body myositis is an inflammatory myopathy producing distal and asymmetric weakness.
23. Mcardle's disease tends to present in children with painful muscle cramps and myoglobinuria after intense
exercise. It is an autosomal recessive condition resulting myophosphorylase deficiency which leads to an inability
to utilise glucose. There are no extra neurological features to suggest a mitochondrial disorder and the plasma
lactate is normal.
24. Neuroleptic malignant syndrome is caused by anti-psychotic medication.
25. The rapid detoxification programmes now used are excellent for acute withdrawal from alcohol but should not
be used in patients who are encephalopathic (as suggested by his confusion and flapping tremor).
458
M Y Elamin
MBBS, DTM&H, MCTM, MRCPI 1& 2
26. Flumazenil reverses the effects of benzodiazepines but care must taken in its usage due to a relatively short half
life when compared to benzodiazepines and the risk of provoking seizures when administered.
27. Causes of his encephalopathy should be sought, for example, spontaneous bacterial peritonitis, GI bleed or
infection.
28. Methanol toxicity early signs of toxicity are due to methanol. Later signs are due to its metabolite, formic acid.
Early signs include: Nausea, Vomiting, Headache, and Confusion. Formic acid later produces a metabolic
acidosis and retinal injury.
29. Anion gap = (Na + K) − (Cl + HCO3); normal range 7-17 mmol/L. Although elevated, the lactate level does not
account for the anion gap.
30. Methanol toxicity diagnosis can be made early by measuring the serum methanol and serum formate levels.
Treatment is aimed at:
a. Eliminating formic acid (alkaline diuresis or haemodialysis).
b. Correcting acidosis with IV bicarbonate.
c. Preventing metabolism of methanol to formic acid by administering IV ethanol.
31. Aspiration pneumonia causes a combination of bacterial and chemical pneumonitis. Organisms involved include
Staph aureus & Strep pneumoniae, but also anerobes e.g. Peptostreptococci and Fusobacterium.
32. Monotherapy in a case of aspiration pneumonia is insufficient. Co Amoxiclav covers aerobes, whilst
metronidazole targets anaerobes the only appropriate treatment would be co-amoxiclav and metronidazole.
They are required in conjunction to offer optimal cover. In cases of serious side effects, the regime would need to be
re-considered
33. The most common clinical signs prior to a cardiac arrest are respiratory distress (RR >25/min) and altered
mental state.
34. The following can all be used to predict critical illness:
a. Abnormalities in heart rate
b. Temperature
c. Respiratory rate
d. Urine output, and
e. Conscious level.
f. All of these have a low specificity but a high sensitivity in detecting underlying physiological disturbance.
35. Most critically ill patients have signs of organ dysfunction that are usually due to underlying tissue hypoxia. The
most common clinical signs prior to a cardiac arrest are respiratory distress (RR >25/min) and altered mental
state.
36. Over 50% of patients requiring ICU present with with metabolic acidosis many with a raised blood lactate
concentration. A raised lactate in associated with a higher mortality.
37. Base excess has been shown to be superior to ph in evaluating metabolic acidosis and in predicting subsequent
complications.
459
M Y Elamin
MBBS, DTM&H, MCTM, MRCPI 1& 2
38. A base excess of <-4 mmol/L has been associated with a mortality of 50-60%.
39. CO poisoning, CO is an odorlous, colourless and non-irritating gas, it commonly presents in the winter period
with symptoms of altered mental status, treatment is with hyperbaric oxygen.
40. CO poisoning, The history is suggestive of carbon monoxide (CO) poisoning due to poor ventilation in his
caravan.
41. CO poisoning, CO binds with high affinity to haemoglobin, forming carboxyhaemoglobin. CO also binds
myoglobin and mitochondrial cytochrome oxidase.
42. CO poisoning causes tissue hypoxia, anaerobic metabolism and lactic acidosis.
43. CO poisoning, Elevated carboxyhaemoglobin levels document exposure, but do not correlate with severity.
44. CO poisoning, Treatment is stabalisation using 100% oxygen via face mask, after which transfer to a centre with
hyperbaric oxygen should be considered.
45. VT, Independent atrial activity is seen with VT but not with an SVT
46. VT, The ECG shows a wide complex tachycardia with a rate of about 200 with marked left axis deviation.
47. VT, On careful examination of the rhythm strip there is evidence of independent atrial activity; P waves can be
seen 'marching through' the QRS complexes.
48. VT, Even without demonstrating independent P waves, the QRS width, axis deviation and rate all suggest a
ventricular origin rather than a supraventricular origin.
49. Torsades as there is characteristic twisting about the isoelectric line, and VF since there is no regular pattern to
the QRS complexes. Ventricular flutter but this would normally have an even higher rate.
50. Atropine is the first step in the management of bradycardia secondary to beta blocker or calcium channel
blocker overdose
51. Overdose of beta blockers or calcium channel blocker:
a. Can lead to significant bradycardia.
b. If taken within one hour of presentation, activated charcoal should be tried.

c. If there is symptomatic bradycardia atropine should be used in the first instance.


d. Glucagon can be effective but this should be tried after atropine.
e. Pacing may be necessary if these drug treatments fail.
f. High-dose insulin is being used more readily to treat beta blocker overdose in those who are
haemodynam- ically unstable. It can be titrated to maintain a stable blood pressure but typically
requires doses much higher than those used to treat diabetic ketoacidosis.
52. Intravenous lipid infusion has been used in various drug overdoses and has particular relevance in local
anaesthetic toxicity.
53. Toxbase recommends consider- ing intralipid in patients with a history of overdose of beta blockers, who have
cardiotoxic symptoms not responding to conventional treatment.
54. Therefore the correct answer here, in a patient who has symptomatic bradycardia more than one hour after a
probable beta blocker overdose is atropine.
460
M Y Elamin
MBBS, DTM&H, MCTM, MRCPI 1& 2
55. Hyphaema is blood in the anterior chamber. It is commonly seen after trauma to the eye. It is usually caused by
trauma - often small objects (champagne corks, squash balls) hitting the eye. Aspiration may be required to prevent
loss of vision.
56. In UGI haemorrhage, Surgical management should be considered where endoscopic intervention has failed. Urgent
surgery and oversewing of the bleeding vessel. Interventional radiology for embolisation of the vessel would be a
potential alternative route that could be considered depending upon availability of the service and the patient's
operative risk.
57. In UGI haemorrhage,, It would be appropriate to use omeprazole postoperatively to promote ulcer healing.
Omeprazole will not stop bleeding from a visible vessel.
58. In UGI haemorrhage, Terlipressin and octreotide decrease portal blood pressure and have an established role in
variceal haemorrhage, but not in peptic ulcer disease.
59. Refractory VF It is worth considering other management options if repeated shocks are ineffective. Also it is always
worth repeatedly reviewing the "4 Hs and 4 Ts" for any possible reversible causes.
a. Magnesium sulphate IV is recommended for the treatment of refractory VF, if there is anything to
suggest the patient may be hypomagnesaemic (such as on medications which might cause this, that is,
thiazides).
b. There is no indication for increasing doses of adrenaline or amiodarone, or increasing shock energy.
c. Amiodarone can be given again but this should be at the reduced dose of 150 mg.
d. Lidocaine is only recommended if amiodarone is unavailable, and/or has not already been given.
e. Therefore the most appropriate response here is magnesium sulphate IV.
60. Theophylline slow intravenous infusion (100-200 mg) is indicated for the management of bradycardia in patients
who have had cardiac transplantation. Therefore there is no place for atropine, even at higher doses. Temporary
pacing would also be an appropriate course of action but only if there is little delay
61. Glycopyrrolate is an antimuscirinc agent and has a similar mechanism of action to atropine.
62. Glucagon can be useful if there is a suggestion of betablocker overdose.
63. QT prolongation can lead to torsade de pointes and this should be treated in the first instance (if no adverse
signs) with IV magnesium
64. Drugs which prolong the QT interval (including amiodarone) can lead to torsade. This is a malignant
arrhythmia with a significant risk of deteriorating into ventricular fibrillation (VF) and needs prompt
treatment.
65. The initial management of torsade with no adverse features present is:
a. Stop all drugs which prolong QT
b. Correct any electrolyte abnormalities
c. Give IV magnesium (2 g IV over 10 minutes)
66. A blood transfusion is the most appropriate immediate action for patients with a major gastrointestinal (GI)
bleed.
461
M Y Elamin
MBBS, DTM&H, MCTM, MRCPI 1& 2
67. Acute aortic dissection presents with collapse, preceded by severe chest pain radiating to the back.
68. An overdose of amitriptyline and diazepam may lead to QRS widening, so an ECG would be an appropriate
initial investigation on presentation.
69. Anion gap is calculated as (Na + K) - (Cl + HCO3).
70. Anticoagulation with subcutaneous heparin is recommended in most guidelines for treatment of pulmonary
embolism. IV heparin and warfarin are contraindicated in pregnancy.
71. Brain herniation often causes false localising signs due to compression of various areas of the brain
72. Cholinergic features with a relative bradycardia, small pupils and increased salivation are highly suggestive
of organophosphorus poisoning which is an anticholinesterase, thus prolonging the effects of acetylcholine.
73. Considering the potential complications of major abdominal surgery.
74. Elevated amino transferase (AST) (from muscle) suggest a diagnosis of rhabdomyolysis.
75. Features of ecstasy overdose include hyperthermia, hypertension, and hyponatraemia.
76. Features of the amphetamine MDMA abuse include hyponatraemia, tachycardia, hyperventilation,
hyperthermia, and pyrexia.
77. Findings in hypercalcaemia.
78. Flecainide is the drug most likely to restore sinus rhythm in atrial fibrillation.
79. Fresh frozen plasma (FFP) contains more dilute clotting factors and therefore produces inferior correction
and should not be used in the management of life-threatening bleeding (unless prothrombin complex
concentrate is not available).
80. Haemodialysis is the treatment of choice for patients with aspirin overdose when the plasma-salicylate
concentration is greater than 700 mg/litre (5.1 mmol/litre) or in the presence of severe metabolic acidosis as
recommended within the British National Formulary (BNF) poisons section.
81. Hypotension in paracetamol overdose is unusual but is feasible in a massive overdose and usually associated
with lactic acidosis. IV fluids are the most appopriate immediate step in patients presenting with vomiting
and hypotension following overdose.
82. If VT is proven patients should be on amiodarone and the indication for an automated implantable
cardioverter/defibrillator strongly considered if the overall LV function is reduced.
83. IM adrenaline is the immediate treatment of anaphylaxis.
84. Immediate treatment in the context of a life threatening arrhythmia associated with acute renal failure,
hyperkalaemia, and palpitations, would be calcium gluconate.
85. In a chest x ray, you are most likelyto see an 'azygous loop' i the right upper zone. It has little clinical
significance.
86. In cases, of acute sever asthma, ß2-agonists should be administered as soon as possible, preferably nebulised
driven by oxygen.
462
M Y Elamin
MBBS, DTM&H, MCTM, MRCPI 1& 2
87. In patients with myocardial infarction and arrhythmia, high pulmonary pressures would be expected after
this arrest scenario, as the pulmonary arterioles constrict in response to hypoxia.
88. Increased output of noradrenaline is responsible for the initial tachycardia seen after blood loss.
89. IV hydrocortisone is first line treatment in Addisonian crisis.
90. Magnesium has been shown significantly to reduce maternal mortality in eclampsia and a favourable
outcome may also be expected in pre-eclampsia.
91. Malignant hyperpyrexia (MH) is characterised by increased temperature and muscle rigidity during
anaesthesia, which results from abnormal skeletal muscle contraction and increased metabolism. Muscle
biopsies may appear histologically normal.
92. Management of acute bleeding should be with a vasoactive drug (such as somatostatin or terlipressin).
93. Methotrexate would be the only correct treatment for someone with erythrodermic psoriasis.
94. Mortality from DKA has remained at 2% since 1970's.
95. N-acetylcysteine should be given to all patients who present with staggered paracetamol overdoses.
96. Pancytopenia is commonly seen in patients undergoing azathioprine therapy.
97. Paracetamol is conjugated to glucuronic acid and sulphate under normal conditions.
98. Pneumococcal meningitis is the second commonest cause of bacterial meningitis (commonest in the elderly)
and is associated with the highest mortality (20%) and deafness which may occur in 50%
99. Post head injury neurogenic diabetes insipidus is caused by reduced antidiuretic hormone secretion.
100. Prothrombin time is the best determinant of risk at 72 hours in cases of paracetemol overdose.
101. Pulmonary emboli result in hypoxia, and therefore cyanosis can be seen.
102. Sarin is an organophosphorus and pralidoxime reactivates the acetyl cholinesterase enzyme. It should be
used in the first few hours.
103. Several drugs, including chlormethiazole, can cause hypersalivation.
104. Severe cases of lithium overdose require haemodialysis.
105. Supportive measures are first line for beta-blocker overdose.
106. The most common symptoms of cerebellar haemorrhage are severe nausea and vomiting, and ataxia.
Headache may be severe. Patients with cerebellar haemorrhage can rapidly become comatose within hours
after the onset from herniation, because of its limited space in the posterior fossa.
107. The most likely explanation in a patient with a prior inferior myocardial infarct is mitral valve prolapse
due to papillary muscle rupture.
108. There is a role for fluids in PE in those patients with rapidly falling blood pressure but this should
preferably be done (if there is time) with central pressure monitoring.

463
M Y Elamin
MBBS, DTM&H, MCTM, MRCPI 1& 2
109. Transient global amnesia lasts less than 24 hours and normal cognition and an awareness of personal
identity is retained
110. Unlike the tricyclic antidepressants, many of the SSRIs are safe in overdose and cause very few effects.
111. Urinary B-HCG will confirm pregnancy.
112. Urine sodium concentration is typically above 30 mmol/L due to tubular injury. The fractional excretion
of sodium distinguishes ATN from pre-renal diseases as a cause of AKI.
113. Use of current guidance for paracetamol overdose management - measure level at four hours.
114. Very high osmolality with normal anion gap and acid base balance reflects the presence of ethanol.
115. Wallerian degeneration is degeneration of the portion of the nerve distal to the injury. It occurs
following axonal injury in both the peripheral and central nervous systems and usually begins within 24-36
hours of injury.
116. Wheeze, stridor and urticaria is more in keeping with allergy than MSG syndrome following Chinese
takeaway
117. When bleeding is the major problem in a pregnancy, the aim is to maintain the prothrombin and
activated thromboplastin time at a ratio of 1.5 times of the control and the fibrinogen level above 1 g/L.
118. A 'ferning' or 'arborescent' rash is pathognomonic of a lightning strike, also known as Lichtenberg
figures.
119. A rise in serum creatinine more than 20% above the baseline after starting an angiotensin-converting
enzyme inhibitor (ACEI) should prompt the clinician to hold the drug.
120. ABG interpretation
121. Addison's disease is diagnosed by short synacthen test.
122. Anion gap = (Na + K) - (Cl + HCO3); normal range 7-17 mmol/L.
123. Atropine is the first step in the management of bradycardia secondary to beta blocker or calcium channel
blocker overdose
124. Baclofen toxicity leads to respiratory depression and diaphragm contraction which requires intubation
and ventilation.
125. Bisphosphonates are the drug of choice in established osteoporosis
126. CO is an odorlous, colourless and non-irritating gas, it commonly presents in the winter period with
symptoms of altered mental status, treatment is with hyperbaric oxygen.
127. Consider neuroleptic malignant syndrome in a patient taking antipsychotic medications and with fever,
muscle rigidity, confusion and autonomic dysfunction.
128. CRVO presents with a painless loss of vision, it is common in the elderly. Diffuse retinal haemorrhage
and disc oedema are present.

464
M Y Elamin
MBBS, DTM&H, MCTM, MRCPI 1& 2
129. Dendritic ulcers are caused by herpes simplex virus
130. Dendritic ulcers are caused by herpes simplex virus. They are treated with aciclovir eye drops, which
should be continued for three days after the ulcer has healed.
131. Descending weakness with autonomic dysfunction and normal EMG is characteristic of botulism.
132. Diagnosis is by demonstration of C. neoformans in the cerebrospinal fluid, shown on an India ink stain:
the thick polysaccharide capsule is highlighted around the cell
133. Drugs that may cause SIADH include Selective serotonin reuptake inhibitors (SSRIs), Tricyclic
antidepressants, Sulphonylureas, Thiazides, and Carbamazepine.
134. Echocardiography can be used in the periarrest diagnose (or exclude) pericardial tamponade.
135. Endotracheal tube is no longer recommended as a route of administration for drugs
136. Erythem ab igne is due to fire/hot-water bottle use and may be secondary to symptomatic
hypothyroidism
137. Features of myotonic dystrophy include frontal baldness in men, atrophy of temporalis, masseters, and
facial muscle, and bilateral ptosis.
138. Fibrinogen consumption in DIC should be replaced with cryoprecipitate.
139. Flecainide is the treatment of choice for paroxysmal AF with aberrant conduction due to accessory
pathway (Wolff- Parkinson- White syndrome)
140. Flumenazil is used in cases of benzodiazepine toxicity.
141. Guidelines for CT head post-trauma are indicated by NICE and comprehensive and clear.
142. Hyphaema is blood in the anterior chamber. It is commonly seen after trauma to the eye
143. Immediately after defibrillation, chest compressions should be restarted and pulse and rhythm reassessed
after two minutes
144. In ATN urinay sodium losses are usually greater than 60 mmol/L
145. In bleeding duodenal ulcers the initial step in the management is adrenaline injection and heater probe
application when found at OGD.
146. In complete heart block P waves show no relation to the QRS complexes.
147. In patients who have taken an overdose of paracetamol, INR is the best indicator of the degree of hepatic
impairment.
148. Independent atrial activity is seen with VT but not with an SVT
149. Intermittent headaches with neurology affecting multiple cranial nerves is suggestive of Cavernous
Sinus Thrombosis.
150. Intravenous thiamine should be given immediately in cases of Wernicke's encephalopathy.

465
M Y Elamin
MBBS, DTM&H, MCTM, MRCPI 1& 2
151. Knowledge of broad complex tachycardia algorithm (Advanced Life Support)
152. Magnesium sulphate IV is recommend for the treatment of refractory VF, if there is anything to suggest
the patient may be hypomagnasaemic
153. management of pneumothorax
154. Mangement of arrhythmias
155. Meningococcal septicaemia presents with a rapidly progressive, non blanching rash, purpuric in nature.
Treatment is with cephalosporin.
156. Methanol has a large concentration of cations and hence causes a raised anion gap.
157. Mobitz type II heart block post MI is an indication for transvenous cardiac pacing.
158. Monotherapy in a case of aspiration pneumonia is insufficient. Co Amoxiclav covers aerobes, whilst
metronidazole targets anaerobes
159. Myalgia, myositis and myopathy are all recognised effects of HMG-CoA reductase inhibitors (statins).
There is an increased incidence when statins are co-administered with a fibrate or when given to patients on
immunosuppressants.
160. Myxoedema coma is a medical emergency requiring treatment with intravenous thyroxine and
hydrocortisone.
161. Orbital apex syndrome
162. Posteromedial papillary muscle is twice as likely to rupture as the anerolateral papillary muscle
163. Presentation of metabolic acidosis in a patient with methanol toxicity
164. QT prolongation can lead to torsade de pointes and this should be treated in the first instance (if no
adverse signs) with IV magnesium
165. Ricin's toxicity lies in its ability to inhibit protein synthesis, via the endonuclease activity of its A-chain.
166. Sepsis may be complicated by disseminated intravascular coagulation
167. Short synacthen test will confirm diagnosis of Addison's disease
168. Subconjunctival haemorrhage is an alarming adverse effect of aspirin therapy (and other antiplatelets).
169. Surgical management should be considered where endoscopic intervention has failed
170. The Adult Advanced Life Support guidelines 2010 emphasise the importance of minimal interruption to
chest compressions.
171. The most common clinical signs prior to a cardiac arrest are respiratory distress (RR >25/min) and
altered mental state.
172. Theophylline is indicated for the management of bradycardia in patients who have had cardiac
transplantation

466
M Y Elamin
MBBS, DTM&H, MCTM, MRCPI 1& 2
173. There are new cardiopulmonary resuscitation guidelines and these emphasise the importance of chest
compressions for as long as possible during the arrest situation
174. Thyroxine may exacerbate/precipitate Addisonian crisis in undiagnosed patients
175. Treatment of PCP includes co trimoxazole, or iv pentamidine, or clindamycin & primaquine. Steroids
should be started in conjunction.
176. Treatment of SBP is with IV broad spectrum antibiotics such as cephalosporins.
177. Ventricular pre-excitation commonly masquerades as other conditions, such as bundle branch block or
ischaemia.
178. Where there is a tension pneumothorax neither oxygen alone nor needle aspiration are the definitive
treatment and a chest tube must be inserted. The definitive treatment is the one that will lead to resolution of
the pathology and a needle aspiration (or thoracocentesis) will not be big enough to allow the tension
pneumothorax to resolve.

467
M Y Elamin
MBBS, DTM&H, MCTM, MRCPI 1& 2
RADIOLOGY

1. A global reduction in movement affecting the right hip, and point tenderness over the trochanter indicate
trochanteric bursitis.
2. A normal perfusion scan has a sensitivity of 98% but a specificity of only 40%.
3. Adenomas larger than 1.5 cm frequently have suprasellar extension and may displace the optic chiasm.
4. Asymmetric reduction in renal size should lead you to suspect renal artery stenosis, for which MR
angiography is the investigation of choice.
5. Causes of cavitating lung lesions include malignancy (of which squamous cell carcinoma is the most
common), infection (S. aureus, TB, Klebsiella, PCP), infarction, granulomatosis with polyangiitis and
rheumatoid nodules.
6. Constrictive pericarditis can be caused by prior tuberculous infection.
7. De Quervain's thyroiditis is associated with low uptake on radio-isotope scanning of the thyroid.
8. Excessive urinary arginine excretion is a feature of cystinuria.
9. In cases of suspected renal cell carcinomas, ultrasound scan of the renal tract would be the first investigation
of choice, as it is able to pick up 95% of renal cell carcinomas greater than 1 cm in diameter.
10. In HIV, focal symptoms eventually appear as the disease progresses if left untreated.
11. In silicosis, progressive massive fibrosis (PMF), or conglomerate nodules is due to exposure to dust of high
silicon content and hence PMF is more likely with higher silicon exposure than in simple coal worker's
lung.
12. In the older age group, investigation of the lower gastrointestinal (GI) tract is vital to exclude a lower GI
malignancy in cases such as unexplained anaemia with bowel signs or symptoms. Colonoscopy would have
the greatest diagnostic yield in most settings.
13. Klebsiella pneumonia is associated with cavitation on the chest x ray
14. Patients with potentially operable tumours who are either too unfit for surgery, or who refuse surgery may
be suitable for radical radiotherapy.
15. Salivary gland pleomorphic adenomas are the most common salivary gland tumour and are benign.
16. Unilateral chronic thigh pain lasting for many months, accompanied by raised ALP, in an adolescent is
consistent with a diagnosis of osteosarcoma.Achalasia can cause megaoesophagus which can be seen on
CXr as a wide mediastinum with absent gastric bubble.
17. Anaerobic and gram negative organisms are common in abscesses following aspiration. An unconscious
patient is at risk of aspiration.
18. Barium follow through will show loops of small bowel narrowing and stricture in Chron's disease,
especially in the terminal ileum.
468
M Y Elamin
MBBS, DTM&H, MCTM, MRCPI 1& 2
19. Causative organism in Chagas' disease
20. Characteristic deformity and symptoms seen in active Paget's disease of bone
21. Chronic subdural haematoma should be considered in patients with fluctuating neurology on blood thinners
and minor trauma.
22. Clinical signs of papilloedema and gait ataxia suggest cerebellar pathology which may represent metastatic
spread of malignancy.
23. Conservative management of Aortic dissection is with IV metoprolol to lower blood pressure.
24. Diagnosis of legionella pneumonia should be made via clinical history, x-ray and urinary antigen.
25. Elderly and falls cay result in sudural haematoma, symptoms may be present subacutely
26. Endometeriosis may cause tethering of the colon and lead to cyclical rectal bleeding.
27. Epilepsy and alcohol and drug abuse are risk factors for aspiration pneumonia.
28. Falx calcification if big enough can cause neurological symptoms.
29. First line treatment for Legionella pneumonia is with macrolides or quinolones.
30. Gout classically affects the metatarsal bones with punched out extra-articular lesions on x-ray.
31. In high risk patients or those with multiple co-morbidities flexible sigmoidoscopy is safer for bowel
investigation than colonoscopy.
32. Ireland is among the countries with the highest reported prevalence of sarcoidosis worldwide. Biopsy of
granuloma is diagnostic for the disease.
33. Kawasaki disease is a childhood febrile illness, resulting in arterial aneurysms, which may present with
cardiac symptoms later in life.
34. Knowledge of neurological syndrome resulting from lung carcinoma
35. Lesion <1 cm in the pituitary and hyperprolactinaemia is MICROprolactinoma
36. management of pneumothorax
37. Melanoma commonly spreads to the brain and lesions can cavitate causing neurological decline.
38. Multiple ring enhancing lesions on imaging of the brain in an immunosuppressed patient may indicate
cerebral toxoplasmosis.
39. On ultrasound, fibroids appear as well-defined, solid masses with a whorled appearance usually of similar
echogenicity to the myometrium, but sometimes may be hypoechoic. They cause the uterus to appear bulky
or may cause an alteration of the normal uterine contour
40. PCP is an opportunistic infection which affects patients with HIV with significantly low CD4 counts.
41. PCP pneumonia characteristically causes oxygen desaturation during exercise.
42. Pleural plaques suggest previous exposure to asbestos.
469
M Y Elamin
MBBS, DTM&H, MCTM, MRCPI 1& 2
43. Polycythaemia is a well recognised complication of hypernephroma due to erythropoietin secretion.
44. Pubic diastasis is a common injury amongst horse riders who present following a forwards fall onto the
saddle of the horse where their anterior pelvis takes the brunt of the force. The resulting trauma causes
dislocation of the ligaments and X ray shows widening of the joint space.
45. Ring enhancing lesions may be associated with: cerebral mets, toxoplamosis, histioplasmosis, brain abscess
46. Salient features of pneumocystis jirovecii pneumonia (PCP) are several days/weeks of increasing dyspnoea,
dry cough, marked oxygen desaturation with exercise.
47. Stricturing and beading of the bile ducts on ERCP are features classically seen in PSC.
48. The added sound is the pericardial 'knock' rather than a third/fourth heart sound in constrictive pericarditis.
49. The diagnosis is a mesothelioma
50. The MRI in HSV encephalitis demonstrates a temporoparietal dense lesion
51. There is a soft tissue mass destroying a large portion of the scapula. In a patient of this age a metastasis is
the most likely cause.
52. Unintentional weight loss may be a sing of underlying malignancy.
53. Where there is a tension pneumothorax neither oxygen alone nor needle aspiration are the definitive
treatment and a chest tube must be inserted. The definitive treatment is the one that will lead to resolution of
the pathology and a needle aspiration (or thoracocentesis) will not be big enough to allow the tension
pneumothorax to resolve.

470
M Y Elamin
MBBS, DTM&H, MCTM, MRCPI 1& 2
PHOTOGRAPHY

1. A 'ferning' or 'arborescent' rash is pathognomonic of a lightning strike, also known as Lichtenberg figures.
2. A single herald patch followed by multiple erythematous plaques is characteristic of Pityriasis rosea.
3. Abnormal GTT with GH measurement is diagnostic of acromegaly.
4. Achalasia can cause megaoesophagus which can be seen on CXr as a wide mediastinum with absent gastric
bubble.
5. Actinomycosis is caused by a filamentous, Gram positive bacterium and treatment is with benzylpenicilin or
surgical drainage.
6. Addison's disease (primary hypoadrenalism) is associated with low cortisol and aldosterone levels.
7. Addison's disease is diagnosed by short synacthen test.
8. African trypanosomiasis has two recognised forms both of which are treated with Suramin
9. Amphotericin is used to treat patients with Aspergillus fumigatus.
10. Anaerobic and gram negative organisms are common in abscesses following aspiration. An unconscious
patient is at risk of aspiration.
11. Angioid Streaks are breaks in Bruch's membrane. They may be associated with pseudoxanthoma elasticum,
Ehlers-Danlos syndrome, sickle cell disease, acromegaly (and other pituitary disorders), and Paget's disease.
12. angioid streaks are caused by breaks in Bruch's membrane
13. Anticentromere antibodies are strongly associated with CREST syndrome.
14. Antithyroid antibodies are commonly associated with pre-tibial myxoedema.
15. AntiTPO antibodies are present in 10% females without thyroid pathology
16. Aortic regurgitation presents with dyspnoea and an EDM
17. APML M3 is associated with a t(15;17) translocation and treatment is with all-trans retinoic acid.
18. Arterial and venous thromboses are well recongised in Behcets disease
19. Ascaris lumbricoides is a large roundworm and is the most common nematode parasite of humans with
patients being largely asymptomatic.
20. Aspergillus is a cause of a haemoptysis.
21. Blood film showing basophilic (blue) stippling of the red cells is typical of lead poisoning
22. Blood film that shows ring forms within erythrocytes; some erythrocytes contain two to three parasites per
cell - are typical of falciparum; other forms of malaria seldom have more than one parasite per red cell.

471
M Y Elamin
MBBS, DTM&H, MCTM, MRCPI 1& 2
23. Bullous pemphigoid is an autoimmune condition that primarily affects the over 60s, leading to the formation
of subepidermal blisters
24. Causative organism in Chagas' disease
25. Characteristic deformity and symptoms seen in active Paget's disease of bone
26. Chediak-Higashi syndrome is autosomal recessive and patients may have a silvery sheen to their skin, and
blue to brown irises.
27. Chronic lymphocytic leukaemia (CLL) is a monoclonal disorder characterised by a progressive
accumulation of functionally incompetent lymphocytes.
28. Clinical features of Behcet's disease include the classical triad of: recurrent oral ulceration, recurrent genital
ulceration, and iritis.
29. Clinical signs of papilloedema and gait ataxia suggest cerebellar pathology which may represent metastatic
spread of malignancy.
30. CMV retinitis is commonly associated with HIV. It presents with a 'pizza' appearance
31. Cobblestone gastric mucosa is a clinical manifestation of Crohn's disease.
32. Complaints of tiredness and weight gain in hypopituitary patients could be a sign of inadequate growth
hormone replacement.
33. CRVO presents with a painless loss of vision, it is common in the elderly. Diffuse retinal haemorrhage and
disc oedema are present.
34. Cryptococcal meningitis is an AIDS defining illness
35. Cutaneous TB usually occurs due to spread from an endogenous source with most cases presenting on the
face or neck.
36. Dendritic ulcers are caused by herpes simplex virus
37. Dendritic ulcers are caused by herpes simplex virus. They are treated with aciclovir eye drops, which should
be continued for three days after the ulcer has healed.
38. Dermatomyositis is very rarely seen in primary care but can be associated with lung, ovary and GI
malignancies.
39. Diagnosis is by demonstration of C. neoformans in the cerebrospinal fluid, shown on an India ink stain: the
thick polysaccharide capsule is highlighted around the cell
40. Diagnosis of legionella pneumonia should be made via clinical history, x-ray and urinary antigen.
41. Diarrhoea in the returning traveller is a common presentation. It is important to recognise symptoms of the
most frequent culprits. Giardiasis presents with abdominal symptoms as listed above.
42. Differential diagnosis for anaemia and hepatosplenomegaly, and blood film findings in myelofibrosis
43. E. coli 0157 can result in HUS and TTP, characterised by uraemia, anaemia, thrombocytopenia and MAHA.
472
M Y Elamin
MBBS, DTM&H, MCTM, MRCPI 1& 2
44. Endometeriosis may cause tethering of the colon and lead to cyclical rectal bleeding.
45. Epilepsy and alcohol and drug abuse are risk factors for aspiration pneumonia.
46. Eruptive xanthomata occur in hyperlipidaemic states associated with hypertriglyceridaemia.
47. Erysipelas is a Streptococcus pyogenes infection of the deep dermis and subcutis.
48. Erythem ab igne is due to fire/hot-water bottle use and may be secondary to symptomatic hypothyroidism
49. Erythema multiforme typically presents with target lesions, cause may be multifactorial
50. Erythema nodosum is characterised by the presence of tender subcutaneous nodules, usually on the shins
51. Erythema nodosum is commonly idiopathic, but there should be a high index of suspicion for sarcoidosis.
52. Erythema nodosum lesions tend to last six to eight weeks.
53. Failure to suppress cortisol below 50 nmol/L on a ODST test is highly suggestive of Cushing's disease
54. Falx calcification if big enough can cause neurological symptoms.
55. Features of myotonic dystrophy include frontal baldness in men, atrophy of temporalis, masseters, and facial
muscle, and bilateral ptosis.
56. First line treatment for Legionella pneumonia is with macrolides or quinolones.
57. Fracture of the midshaft of the humerus can cause a radial nerve palsy giving rise to wrist drop
58. Gastric CA may progress from gastric ulcer and may present with Acanthosis nigricans
59. Giardia lamblia is a common cause of diarrhoea even in the United Kingdom and symptoms include
bloating, abdominal pain and diarrhoea.
60. Glucagonoma is associated with necrolytic migratory erythema
61. Grave's eye disease can occur in euthyroid, hypothyroid or hyperthyroid setting
62. Group B Streptococcus has a predilection for cellulitis in diabetes
63. HE linical manifestations range from asymptomatic carriage to severe haemolytic anaemia and is diagnosed
on blood film showing cigar-shaped elliptocytes.
64. Hereditary haemorrhagic telangectasia is usually an autosomal dominant condition that may present with
anaemia due to bleeding from occult GI telangectasia
65. HIV-positive patients are more prone to herpes zoster regardless of their CD4 count.
66. Howell-Jolly bodies are red cells seen on the peripheral blood film in which the nuclear remnant is still seen
and caused by splenic atrophy or splenectomy.
67. Hyphaema is blood in the anterior chamber. It is commonly seen after trauma to the eye
68. Hyposplenism is a cause of Howell-Jolly bodies on a blood film.

473
M Y Elamin
MBBS, DTM&H, MCTM, MRCPI 1& 2
69. HZV distributes in a dermatomal fashion.
70. Imatanib is first line treatment for CML.
71. In HIV infection, KS lesions occur most commonly on the face; however lesions may be widely
disseminated in skin, bronchial tree and gastrointestinal tract.
72. in presentations of Gaucher disease, the skin may show a grey-brown discolouration, especially around the
forehead, hands and pre-tibial regions. Characteristic yellow or yellow-brown papules (pingueculae)
develop at the sclerocorneal junctions.
73. In respect to Anthrax infection cutaneous disease is the commonest form of the infection in humans.
74. Independent atrial activity is seen with VT but not with an SVT
75. Ireland is among the countries with the highest reported prevalence of sarcoidosis worldwide. Biopsy of
granuloma is diagnostic for the disease.
76. Jaccoud's arthropathy may be seen in SLE, RA, Parkinson's disease and hypocompementaemic urticarial
vasculitis
77. Kaposi's sarcoma (KS) lesion is a red-purple mass arising from the conjunctiva. It is not limited to the sclera
(which could suggest a subconjunctival haemorrhage) and it protrudes, which also favours a malignant
growth.
78. Kaposis sarcoma is a tricky and rare diagnosis in primary care that presents as red to purplish macules and
raised papules/nodules anywhere on the skin or mucous membranes
79. Kawasaki disease is a childhood febrile illness, resulting in arterial aneurysms, which may present with
cardiac symptoms later in life.
80. Klinefelter's is characterised by tall stature, small testes, azoospermia and gynaecomastia in a male. Typical
karyotype is 47XXY, though mosaics occur with 46XY/47XXY karyotype.
81. Klinefelter's syndrome is suggested by hypergonadotrophic hypogonadism, gynaecomastia and low IQ.
82. Knowledge of neurological syndrome resulting from lung carcinoma
83. Liver biopsy remains the gold standard diagnostic tool since it allows an estimation of the degree of iron
loading in haemochromatosis.
84. Loss of night vision is a features of retinitis pigmentosa and retinal appearance at the periphery appears as if
there are "black bony spicules"
85. Macroadenoma witll cause visual field defects
86. Macular degeneration will show small pale dots over the macluar on slit lamp examination known as
drusen.
87. management of pneumothorax
88. Manifestations and management of hereditary haemorrhagic telangectasia

474
M Y Elamin
MBBS, DTM&H, MCTM, MRCPI 1& 2
89. Many drugs have been associated with gynecomastia, including phytoestrogens, oestrogens and drugs with
oestrogen-like properties, inhibitors of testosterone synthesis or action, and other agents with unknown
mechanisms.
90. Melanoma commonly spreads to the brain and lesions can cavitate causing neurological decline.
91. Meningococcal septicaemia presents with a rapidly progressive, non blanching rash, purpuric in nature.
Treatment is with cephalosporin.
92. Molluscum Contagiosum is caused by a DNA pox virus and are associated with HIV / AIDS
93. Multiple ring enhancing lesions on imaging of the brain in an immunosuppressed patient may indicate
cerebral toxoplasmosis.
94. Mycosis fungoides (a cutaneous T cell lymphoma) presents as a pruritic eczematous rash and develops
telangiectasias and areas of 'cigarette paper' atrophy.
95. Myotonic dystrophy is the most common adult muscular dystrophy. It is inherited as autosomal dominant,
the gene is on chromosome 19 and it is a trinucleotide repeat disease, which is a repeat of CTG, that exhibits
genetic anticipation.
96. Necrobiosis lipoidica is a typically painless lesion assoicated with diabetes
97. Necrobiosis lipoidica is commonly associated with diabetes and is painless lesion due to collagen
degeneration.
98. Necrobiosis lipoidica is commonly associated with diabetes. It is a painless lesion and secondary to collagen
degeneration.
99. Non-caseating granulomas are seen in berylliosis and sarcoidosis.
100. Non-suppressed TSH with elevated alpha subunit is in keeping with TSH-oma
101. Nystagmus and photophobia are common findings in albinos.
102. Papules with central umbilication are associated with molluscum contagiosum.
103. Patients with Barrett's require biopsy to rule out dysplasia or malignancy.
104. PCOS is associated with insulin resistance; acanthosis nigricans may be present.
105. PCP is an opportunistic infection which affects patients with HIV with significantly low CD4 counts.
106. PCP pneumonia characteristically causes oxygen desaturation during exercise.
107. PCT may be precipitated by alcohol, exposure to sunlight, iron and oestrogen
108. Pituitary apoplexy can cause compression of the occular nerves and trigeminal nerve within the
cavernous sinus.
109. Pleural plaques suggest previous exposure to asbestos.

475
M Y Elamin
MBBS, DTM&H, MCTM, MRCPI 1& 2
110. Porphyria cutanea tarda (PCT) is associated with deficiency of hepatic uroporphyrinogen (URO)
decarboxylase. The major clinical feature is cutaneous photosensitivity. Bullae develop on sun-exposed
areas and lesions heal slowly, leaving scars.
111. Porphyria cutanea tarda (PCT) is associated with reduced hepatic uroporphyrinogen decarboxylase
activity.
112. Porphyria cutanea tarda is usually seen in men who have damaged their liver by drinking too much
alcohol.
113. Presenation and autoantibodies in Addison's disease
114. Presentation of myelodysplastic syndrome (pancytopenia, blood film findings, and bone marrow
findings)
115. Pseudomonas aeruginosa is characterised by green coloration of the colonies.
116. Pseudoxanthoma Elasticum presents with a 'plucked chicken skin' appearance of skin folds in the neck
117. Psueudomembranous colitis is a potential complication of clostridium difficile infection
118. Pyoderma gangrenosum presents as an expanding ulcer with a polycyclic or serpiginous outline and a
characteristic undermined bluish edge which is responsive to oral steroids.
119. Radioiodine therapy can worsen thyroid eye disease
120. Reduced DLCO is an early indication of pulmonary disease in systemic sclerosis.
121. Ring enhancing lesions may be associated with: cerebral mets, toxoplamosis, histioplasmosis, brain
abscess
122. Salient features of pneumocystis jirovecii pneumonia (PCP) are several days/weeks of increasing
dyspnoea, dry cough, marked oxygen desaturation with exercise.
123. Sarcoidosis is a disease of unknown cause characterised by the presence histologically of non-caseating
granulomata.
124. Short synacthen test confirms diagnosis of Addison's disease
125. Skin appearance and systemic manifestations of pseudoxanthoma elasticum
126. Staphylococcus is the most common organism to cause post operative infection of a prosthetic joint
127. Streptococcal infection can precede the onset of guttate psoriasis, which presents with an itchy rash and
sore throat.
128. Stricturing and beading of the bile ducts on ERCP are features classically seen in PSC.
129. Subconjunctival haemorrhage is an alarming adverse effect of aspirin therapy (and other antiplatelets).
130. Symptoms of cutaneous worm infections include rash and itching. Treatment may be topical
(thiabendzole) or oral (albendazole)
131. Target cells and Howell Jolly bodies are commonly seen in hyposplenism on a blood film.
476
M Y Elamin
MBBS, DTM&H, MCTM, MRCPI 1& 2
132. Tc-99m pertechnetate accumulates in gastric mucosa and is the study of choice for identifying ectopic
gastric mucosa in a Meckel's diverticulum.
133. Tendinous xanthomata is pathognomonic for type II(a) hyperlipidaemia (familial
hypercholesterolaemia)
134. Tetracyclines can cause idiopathic intracranial hypertension
135. The added sound is the pericardial 'knock' rather than a third/fourth heart sound in constrictive
pericarditis.
136. The clinical appearance of pemphigus vulgaris and laboratory tests used to confirm diagnosis
137. The MRI in HSV encephalitis demonstrates a temporoparietal dense lesion
138. The third (oculomotor) nerve nucleus complex lies in the midbrain.
139. Threadworm infection is common in children and institutions
140. Toxocara canis a parasite found globally, carried by dogs.
141. Treatment of erysipelas is with the penicillins, but macrolides and cephalosporins can also be used.
Chloramphenicol use is limited due to risk of aplastic anaemia.
142. Tuberous sclerosis presents with flesh coloured papules (angiofibromas) on the face and fibromas in the
central nervous system
143. Type 3 hyperlipidaemia is associated with tendon xanthoma, pancreatitis and premature cardiovascular
disease.
144. Typical features of systemic sclerosis include facial telangiectasia, small mouth, and beaking of the
nose.
145. Unintentional weight loss may be a sing of underlying malignancy.
146. Urinary free cortisol or 1 mg overnight dexamethasone suppression test has 95% sensitivity and
specificity for diagnosing Cushing's syndrome
147. Varicella pneumonia occurs in up to 20% of adults with chickenpox, appearing three to five days into
the course of the illness. In adults with pneumonitis, treatment with aciclovir is warranted.
148. Where there is a tension pneumothorax neither oxygen alone nor needle aspiration are the definitive
treatment and a chest tube must be inserted. The definitive treatment is the one that will lead to resolution of
the pathology and a needle aspiration (or thoracocentesis) will not be big enough to allow the tension
pneumothorax to resolve.
149. Xanthelasma and corneal arcus are seen in common 'polygenic' hypercholesterolaemia, familial
combined hyperlipidaemia and familial hypercholesterolaemia.
150. Yellow nail syndrome is caused by hypoplastic lymphatics and is characterised by the triad of
lymphoedema, pleural effusions and yellow discolouration of the nails.

477
M Y Elamin
MBBS, DTM&H, MCTM, MRCPI 1& 2
CRITICAL

1. A high-PEEP (open-lung) ventilator strategy will reduce atelectotrauma.


2. A profoundly hypothermic patient who fails to respond to three shocks should not receive any further
shocks or drugs until they are rewarmed.
3. A raised PaCO2 in acute asthma is an important sign that the patient is deteriorating and intubation may
be required.
4. Acute cocaine overdose is initially best treated with benzodiazepines.
5. Although less commonly placed in the 21st century, pulmonary artery catheters may be beneficial in
cases of right ventricular dysfunction to assess filling.
6. Aortic stenosis occurs frequently in the population suffering proximal femoral fractures.
7. ARDS is a severe lung syndrome characterised by acute onset of bilateral infiltrates and severe
hypoxaemia in the absence of cardiac failure.
8. Asthma is potentially an acute life threatening event which requires careful assessment, treatment and
monitoring.
9. Blood cultures should be taken prior to antibiotics, unless this would cause a significant delay to their
administration.
10. Brain stem death testing confirms absence of respiratory effort and tests a series of cranial nerve
responses.
11. Central venous catheters should be positioned correctly in the vasculature to avoid potentially fatal
complications.
12. Common intra-operative management goals in mitral regurgitation include avoiding raised SVR,
avoiding bradycardia, and avoiding factors exacerbating pulmonary hypertension.
13. CVC blood stream infection is an important cause of morbidity and mortality in critical care patients.
Considerable research has been undertaken to address this.
14. Diabetes insipidus is an important complication of neurosurgery; diagnosis requires an understanding of
urinary and serum sodium biochemistry.
15. Dual antiplatelet therapy should be continued following the insertion of a drug eluting cardiac stent
insertion for a year unless the surgery is urgent.
16. Early fluid resuscitation is the most important measure in the prevention of acute kidney injury secondary
to rhabdomyolysis. Large volume depletion occurs due to sequestration of water by injured muscle. Most
studies target urine outputs of 3 ml per kilogram per hour or >300 ml per hour.
17. Elimination of weak acids from the body can be promoted by alkalinising the urine.

478
M Y Elamin
MBBS, DTM&H, MCTM, MRCPI 1& 2
18. Fat embolism syndrome is a clinical diagnosis secondary to the presence of fat globules in the lung
parenchyma and circulation that are released, typically following long bone fractures.
19. For diagnosing myocardial infarction in anaesthetised patients, serial troponin assay is a critical
biochemical test.
20. Guillain-Barré syndrome is a potentially life threatening condition that requires careful monitoring and
potentially intubation. However, if managed correctly the prognosis is excellent.
21. Hyperchloraemic metabolic acidosis is a documented complication of neobladder formation.
22. Hypocalcaemia is an important complication of thyroid surgery, and you need to be able to recognise the
clinical presentation of this condition.
23. Hyponatraemia should be corrected slowly, except where seizures or significant neurological dysfunction
occurs.
24. Intensive care trainees should be aware of the indications for emergency renal replacement therapy.
25. Intra-aortic balloon pump counter-pulsation may be an effective bridging therapy in cardiogenic shock,
particularly in the setting of acute myocardial infarction.
26. Lambert-Eaton syndrome and myasthenia gravis are important examples of how disease pathologies can
affect the biochemistry of the neuromuscular junction.
27. MRSA requires specific extended spectrum antibiotics for effective treatment.
28. Necrotising fasciitis is a rapidly fatal condition that requires emergency treatment.
29. Needle thoracostomy (NT) is the most rapid method of achieving lifesaving access to the pleural space if
a tension pneumothorax is rapidly progressing.
30. NICE guidelines on head injury (CG176) recommend that patients with known coagulopathy or on
treatment with warfarin should have imaging by CT, with results analysed within one hour of request.
31. Non-invasive ventilation is an open circuit system.
32. Oseltamivir is the oral treatment of choice for H1N1 infection.
33. Patients with Guillain-Barré syndrome have very limited therapeutic options. A short course of
intravenous immunoglobulin is one of the few treatments that have proved effective.
34. Post-extubation stridor occurs frequently in extubated patients and may be associated with respiratory
failure due to airway obstruction requiring reintubation.
35. Retroperitoneal haematoma is a rare complication of patients on DOACs.
36. Seizures can be divided into partial or complex and with careful examination the focus of the seizure can
be found.
37. Seizures, metabolic acidosis and arrhythmias represent severe toxicity. When the QRS is widened,
sodium bicarbonate should be given until this normalises or plasma pH reaches 7.50.

479
M Y Elamin
MBBS, DTM&H, MCTM, MRCPI 1& 2
38. Severe sepsis has a high mortality that is reduced by early recognition and administration of appropriate
antibiotics.
39. Severe tricyclic overdose with adverse features can be treated with sodium bicarbonate.
40. Symptomatic spontaneous pneumothoraces of more than 2 cm size should not be treated conservatively.
41. The first line test for correct placement of a nasogastric tube is a pH check of gastric aspirate using pH
indicator paper. There is no place for the 'whoosh' test.
42. The first step in post-cardiac arrest care is to give aspirin and clopidogrel. This can usually be achieved
quickly and easily whilst other investigations and treatments are organised.
43. The management of tachyarrhythmias requires the candidate to demonstrate knowledge of unstable vs.
stable features and then broad vs. narrow complex arrhythmias to instigate treatment correctly.
44. The most appropriate point for chest drain insertion for a pneumothorax is in the "safe triangle" in the
mid-axillary line.
45. The order of removing personal protective equipment is important to reduce contamination.
46. Understanding the typical blood gas findings seen in aspirin poisoning is important when trying to
determine the substances on which obtunded patients have overdosed.
47. Unmeasured cations or anions account for the difference between measured and calculated osmolality.
48. Unopposed beta-antagonism should be avoided in malignant hypertension until the cause is identified
49. von Willebrand disease can often present for the first time following surgery. In day-to-day practice,
anaesthetists will be expected to know about common bleeding disorders and be able to manage them.
50. When considering hypertensive disorders of pregnancy, pre-eclampsia and eclampsia must be
distinguished from pregnancy induced hypertension.
51. When treating severe Guillain-Barré syndrome it is necessary to measure the forced vital capacity using
spirometry to monitor for signs of respiratory muscle deterioration.
52. Where atrial fibrillation is of an acute onset, cardioversion is the preferred method (this may be chemical
or electrical in nature). For stable patients, chemical cardioversion manoeuvres should be tried first.

480
M Y Elamin
MBBS, DTM&H, MCTM, MRCPI 1& 2

You might also like